September, 1881. AUSTRALASIAN SCHOOLMASTER. 47

September, 1881. AUSTRALASIAN SCHOOLMASTER. 47

3?/    .

x + -# + 63=180 coins.

From these the answers are readily obtained—

30 half-crowns, 20 shillings, 20 threepenny pieces. Ans.


rn — 1

“l


3367 = a


s


(Ì)8 - I


1024 x


1024 x 58975 x 4


65536


8.    Solve, if you can, the simultaneous equations—

x + lly + 53z = 400)

3*+ 73?/ + 512 = 401 > .

19a; + 89?/ + 35r =» 40S )

Explain.

From 1 and 2 we get 140?/ + 108r = 799,

From 1 and 3    ,,    I40y + 1083 = 799$-,

From 2 and 3    ,,    140// + 108z « 799g.

From this it is manifest that x, y, and 2 have different values in the three equations.

9.    A man has £5 worth of silver in half-crowns, shillings, and threepenny pieces. He has in all 70 coins. If he changed the three-penny pieces for halfpence, and half the shillings for sixpences, he would have ISO coins. How many of each had he at first?

Let x, y, and z represent the numbers of the coins, x + y + z = 70 coins,

” + y +'4'= 100 shillings,

10. A person who saved every year § of what he saved the previous year, had in six years saved £3367. How much would he save in eight years ?

ft — a x

-i -i

a = 1024.

= £3685 18s. 9d. Ans.

Educational Time ft.

HABIT.

It is surprising to find, on close scrutiny, how large a proportion of the acts we perform, seemingly at the bidding and under the control of the will, are in truth performed unconsciously. Walking, running, leaping, lifting and carrying burdens—in fact, most of the actions in which the muscular.system is engaged—are accomplished without the cognisance of details. The end only is perceived ; the will is engrossed with the result, and ignores the step by which it is reached. We set out to walk on a particular road to a certain place; the feet and legs are set in motion and we continue walking, while the whole attention is absorbed with matters of thought, no concern being bestowed on the management of the limbs unless unusual or unexpected sensations attract the notice of the mind. We carry a book or a parcel, and hold it, though its possession may be forgotten. In short, there is a power of setting a part of the organism to do certain work and leaving the task to be carried out by a subordinate faculty of superintendence, which discharges its function unconsciously.

Take as au illustration one of the commonest actions performed by most of us once or twice daily—that of lifting the water-jug from a washing-stand We will to pour out some water, and expect to find it in the jug. The muscular movements requisite for grasping the handle of the vessel and raising it are performed automatically, and the proper amount of force is put out to raise the weight of waier we are accustomed to find. It happens that the servant has neglected to fill the jug, and up it goes with a bound. Again, we are walking up stairs, habit leads up to expect another step at the lop ; the leg is raised, and comes down with a jerk, because the anticipated stair is not there to receive the foot; or perhaps we are coming down, and the foot is arrested by the level floor when we expected a further descent. These are common experiences, but they possess a high significance, and may stand for typical examples of a large group of actions which are performed unconsciously. The sort of unconsciousness which characterises these acts is made evident by the nature of the misadventures which have been instanced.

When anything is done for the first time, it requires to be worked out by the will, each step of the performance is intentional ; as when a child learns to walk, or a pianoforte player to strike the proper notes with the right fingers in the due relations of time by appropriate muscular movements of hands and wrist. When once the combination of acts is mastered, the will no longer superintends the exercise. The attention is not only withdrawn, but it must be diverted, or, in other words, the performance left to the supervision of the subordinate faculty. Let any one who doubts this try to execute in detail, under the superintendence of the will, movements which, are perfectly well done without consciousness ; for example, to descend a flight of stairs rather rapidly, particularly noticing the planting cf each foot on the step below, or to strike a few chords of music with conscious control of the several fingers employed. The unwonted attention embarrasses the performance, ar.d in certain cases even renders it impossible. The explanation of this experience is that the will is wont to relegate the control of muscular movements which are habitually performed to the guidance of a sense which, while it cannot originate, is able to repeat combinations of movements to which it has been accustomed ; and having thus, so to say, delegated a portion of its authority—as the employer of a large number of workers entrusts the management of details to some confidential agent—the will ceases to trouble itself with these lesser matters, and they pass out of the sphere of consciousness.

In certain morbid states this delegation is impossible, and the muscular system will do nothing it is not directly willed to perform. Physiologists and pathologists cite the case of a nurse who could not hold a child in her arms unless she kept it constantly in mind. The moment her attention was diverted from the business in hand her arms dropped, and she let the infant fall. This instance will suffice to illustrate the principle. The conclusion is, briefly, that much that we do is done unconsciously—the actions which fall into this class are, for the most part, matters of habit; and, in proportion as things come to be subjects of habit, they are likely to be done unconsciously.

What is habit? The apologist for humanity in its least noble phase asserts that “ habit is second nature.” It would be more explicit to say that habit is memory supplemented by an unconscious reasoning on the simple lines of a direct inference from the known to the unknown. I use the phrase “ unconscious reasoning” advisedly. There are more than sufficient grounds for believing that, a certain sort of reasoning may proceed without an appeal to the judgment, and without the cognisance of the higher intellect—in short, without consciousness. While the mind is engrossed with one subject, the lesser faculties may be actively employed with another, and even work out processes which, when they come to be reviewed, appear complex. By this reasoning the sleepwalker adapts his proceedings to the requirements of the surrounding circumstances, his brain being all the while asleep and he in a state of unconsciousness. This sort of reasoning probably differs in no essential particulars from the phenomenon! of “instinct” with which the animals below man in the scale of intelligence are gifted, and which sometimes seems in its exercise to emulate the power of reflection and judgment.


Corrcspo nò circe.


TEACHERS’ SAL A RIES.

THE EDITOR OF THE AUSTRALASIAN SCHOOLMASTER.

Sir.—That the payment of teachers’ salaries, based on numbers, which now obtains is most injurious to the interests of education in this colony, cannot be denied.

The system forces the best teachers into cities and towns, where they aro not all absolutely needed, from the country schools, whore they aro positively required.

For the urban and suburban taxpayer, apart from the highly efficient state school, can avail himself of the grammar school, the high school and his University, while his less fortunate fellow tax-payer in the country is compelled (having no choice) to use the next stato school, managed by a teacher of a lower professional grade.

This is unquestionably wrong. It is a wrong inflicted on the country which must continue to be intensified so long as payments on numbers continue.

Costly schemes of classification have boen proposed, and abandoned, simply because, by them, payments were coupled with numbers, and, it may now be safely predicted, that no scheme of classification but that which will class the teachers irrespective of numbers, will succeed.

I send you, as promised in my last letter, a synopsis of my scheme of classification ; you must noither publish it, nor discover its contonts to anyone, as I do not wish, in propria persona;, to repeat the history of Solomon’s poor wise man.

But when a scheme is called for. which will give to the country its right share of teaching-power, check the enormously increasing expenditure, stablish finality, and content the teachers, you may then publish it, but not till then.    Alpha.


HEAD TEACHER, country school, Gippsland district, a’lotmcnb 20 x 30, percentage 78'709, extra allowance, £18 os. a year, desires to EXCHANGE with ASSISTANT, Melbourne or Suburbs. For Address apply Schoolmaster Office. ______

HEAD TEACHER, country school, allotment 30 x 50, eleven miles from Warrnambool (vacancy for workmist.ress), desires to exchange with teacher on North-eastern railway, or near Avenol. Address “Self-will,” v\ arrnambool P.O.

I WARREN BALL’S “Hints to Candidates for Teachers’ and Matricu * lation Examinations,” Is.; posted, Is. Id. Mullen, Melbourne.

CANDIDATES for EXAMINATIONS prepared by correspondence or Otherwise. I. Warren Ball, South Yarra.

H EAD TEACHER, 30 x oO school, Station at door, wishes exchange _ another Head Teacher, same al'cTment. Vacancy workmistress. Address—“ Delta,” Schoolmaster office.

r I +EACH.ER of a 30 x 50 school, in the Western Di trict, would exchange for 20 to 30 in the neighbourhood of Kyneton, Maeedon or Kilmore-Address “ H. B.,” P. O., Maearthur.

HEAD TEACHER, Castlemaine District, allotment 50 to 75, wife sewing mistress and P. O., wifi exchange with first or second assistant. Address—“ Beta,” P. ()., Castlernaine.

HEAD TEACHER, country, would exchange with assistant, town or suburbs. Apply to “ Desirous,” care of P. Matthews, E-q., Arehi ert, 52 Collins-strect.

T J) EAD “ONCE A WEEK,” the Best Family Paper Published. JLa Circulation 10,000-


s,


rpATE’S PARCELS POST EXPRESS

FIXED PRICE.

NO EXTRAS. NO TROUBLE.

Delivery to door at any address in

m

s. d.

41b. s. d.

61b.

8. d-

101b. s. d.

201b. s d.

Great Britain ...

4 6

6 0

7 6

10 0

14 0

Continent of Europe, America, & Canada

7 6

9 6

12 016 0

21 0

Sydney, Hobart, Launceston

2 6

3 0

3 6

4 0

6 0

New Zealand Ports (except West Coast) Adelaide, Brisbane.

4 0

4 6

5 0

5 6

7 6

No further charge whatever. Very small increase for heavier weights. Delivery at country addresses in Australia, inland carriage only added.

Receiving office—

FREDERICK TATE, 13 Market-st., Melbourne TATE’S PARCELS POST EXPRESS'. DELIVERY at DOOR any address in the world. EITHER to or from Britain from 4s. 6d. TO or from other Australian ports from 2s. 6d. NO further CHARGE whatever. No trouble. Any SIZE, weight, or shape.

EVERY"po37i¥lir~A^SISTANCE afforded.

I N Qb I RJ Its plainly answered.

Henceforth Parcels handed to W. R. SUTTON’S Branches, every town in Great Britain, delivered in Melbourne at nearly similar rates.

Head Office:—

FREDERICK TATE, CUSTOMS AGENT, &c, 13 Market street, Melbourne.

A LEX. M‘KINLEY & CO.,

GENERAL

PRINTERS AND PUBLISHERS, 61 Queen-street, Melbourne.

MULLER’S MATRICULATION MANUALS.

POPE’S ESSAY ON MAN, Epistles 1 to 3; and TEN ESSAYS of ADDISON, from the “Spectator.” In One Volume, Crown 8vo, Limp. Is.

POPE’S ESSAY ON MAN, Epistles 1 to 3 ; and TEN ESSAY'S of ADDISON, from the “Spectator.” Prepared with Notes for the use of Candidates for the Matriculation Examinations at the Melbourne University. By E. E. Morris, M.A. In One Volume, Crown 8vo, Cloth. 3s.

VIRGIL’S AENEID, Book XII., and CUESAR’S GALLIC WAR, Book YI. Prepared with Notes for the use of Candidates for the Matriculation Examinations at the Melbourne University. By James Clezy, M.A. Second Edition, Revised. Fcap. Svo, Limp. 2s. VIRGIL’S AENEID, Book XII., and CiESAR’S GALLIC WAR, Book VI. Translated by James Clezy, M.A. Fcap. 8vo, Sewed. 2s. 6d.

FIRST BOOK OF AUSTRALIAN BOTANY-. Specially designed for the Use of Schools. By W. R. Guilfoyle, F.L.S., C.M., R.B.S., London; Director of the Melbourne Botanical Gardens. Illustrated, Crown 8vo, Sewed. 2s. Cd.

MULLEN’S

New Classified Catalogue of School, College, and Technical

EDUCATIONAL WORKS

May be had gratis on application, or posted on receipt of address.

SAMUEL MULLEN,

Wholesale & Retail Bookseller & Stationer, 29 & 31 COLLINS ST. E., MELBOURNE.

Q C. EXAMINATION.

TUITION BY CORRESPONDENCE.

MR. THOMAS BOARDMAN, First-class Honor-man of the Denominational School Board, Prepares Teachers for the Certificate Examination by Correspondence. Terms moderate.

Address—

4 5 PRINCES STREET, CARLTON

TAMES CLEZY, M.A.,

<0    MELBOURNE.

CLASSICAL & SHAKSPEARE SCHOLAR (1869)

PREPARES CANDIDATES For Matriculation (pass or honours), and for the subsequent Degree Examinations of the University.

Course of Lessons by Correspondence in Latin Grammar, Translation, and Composition.

Terms on application personally or by letter. 5 GORE STREET, FITZROY.

Price One Shilling, By Post, Is, 6d.


NOTICE.

MR. H. M'KTNLEY

SOLICITOR, CONVEYANCER, And

PROCTOR,

Has Removed to

86 COLLINS STREET WEST.


In Bookwork and General Publishing our large experience is a guarantee of all work being executed in the best stylo, while having a first-class stock of the best and latest material ensures expedition and good workmanship.

The following papers are issued from this office :— WEEKLY.

“Punch,”    “Bulletin,”

“ Willing Work,”    “ Once a Week.”

FORTNIGHTLY.

“ Australian Law Times,” “Jewish Herald,” MONTHLY.

“ Schoolmaster.’’    “ Monthly Messenger.”

BUY AND READ

THE NEW    WEEKLY PAPER.

Prico Threepence.

“ONCE A WEEK,”

A Journal of entertaining Literature by well-known Authors ; also Stories, Sketches and Interesting Topics.

16 PAGES.    PRICE THREEPENCE.

ALL BOOKSELLERS.

Alex. M‘Finley and Co., Publishers, 61 Queen Street, Melbourne.

££ EPA RT MENTAL EXAMINATION OF TEACHERS.

TEACHER S’ GUIDE

TO

AUSTRALASIAN EXAMINATIONS.

Containing the

PROGRAMMES & EXAMINATION PAPERS

of

Victoria    South Australia

New South Wales    Auckland, N.Z.

Queensland    Wellington, N.Z.

And Tasmania.

108 PAGES WITH STIFF COVER.

The above book contains the Programmes and Examination Papers of December, 1877, of all the colonies, and is reduced to the low price of

ONE SHILLING.    BY POST, Is. Gd.

ALEX. M‘KIN LEY & CO,

PRINTERS AND PUBLISHERS, 61 QUEEN STREET,

MELBOURNE.

SO HEAD MASTERS, SECRETARIES OF BOARDS OE ADVICE, AND OTHERS.

ALEX. MTCINLEY & CO.,

Having made considerable additions to their stock of Bookwork and Jobbing Type, are prepared to execute orders in every description of

GENERAL PRINTING.

All orders entrusted to them will be printed in a satisfactory manner.

Alex. MHainley & Co., Printers, 61 Quben-st.

NOW READY.

jy£ ILTON    PARSED.

Prick 2s.

By J. J. BURSTON,

(Author of “State School Arithmetic”).

Also Ready, the Fourth Edition of the

STATE SCHOOL    ARITHMETIC.

By

JOHN J. BURSTON,

North Sandhurst State School.

Printed and Published by Alex. M'Kinley & Co., 61 Queenstreet, Melbourne, under the auspices of the Victorian Teachers Union.

AUSTRALASIAN

AND LITERARY REVIEW.

Vol. III., No. 28.

OCTOBER, 1881.

Subscription

Yearly, Gs. Gd. ; Half-yearly, 8s. 6d


BLACKIE

Comprehensive

& SON’S

School Series.

Popular Edition, Id. each number. the Numbers.

9.    Sentences.—Two lines on each

Page.

10.    Plain and Ornamental Let

tering.

11.    Exercise Book.—Wide Ruling

with Margin.

11J-. Home Exercise Book,—Same as No. 11 but 8vo size. Price Id.

12.    Exercise Book. — Ruled in

Squares.

12£. Home Exercise Book.—Same Ruling as No. 12, but 8vo size Price Id.

13.    Exercise Book.—Ruled for

Book-keeping.

14.    Essay Book,—Ruled for Com

position, Ac.

15.    Exercise Book for Begin

ners. Ruled for Small Text. X. Copy-Book Protector and Blotter. Keeping tbe Books Clean. One Penny.


THE COMPREHENSIVE READERS.

Each Book is Illustrated in a highly instructive and artist ic manner,

Primer I.—32 pp.,..................paper cover, l£d.; cloth cover, 2|d.


2d. ;


3d. 4d. 4d. 6d.

Bile. Is, 6d.

2s. 2s. 6d.


Adopted by tbe London, Liverpool, Manchester and other School Boards, and by the National Board of Education in Ireland.

VERE FOSTER’S WRITING COPY-BOOKS

The efforts of Publishers to provide teachers with the means of properly training their pupils in the Art of Writing deserve the highest commendation ; and no names stand higher in this department of scholastic work than Vere Foster and the Messrs. Blackie."—Educational News.

Opinions of H.M. Inspectors in Education Blue Book Reports.

Mr. M'Callum. 11. M. Inspector, says—

( “ More progress is made by Vere Foster’s than by any other 1 method which has come under my notice.”

Mr. Brewer, \ H.M. Inspector, • says— 1

) With books like Vere Foster’s there is no excuse for the [ slovenly no-style of writing too often presented to me.”

Mr. Vtarburton, \ H.M. Inspector, ■ says— i

“ I wish that the use of the excellent copy-books such as Vere ( Foster’s was begun earlier and more persisted in.”

Mr. A’etcell, j H.M. Inspector, says1

| I know of no series by means of which children can be so I quickly taught to write with freedom and legibility.”

Rev. J. Lomax, | H.M. Inspector, saysI

\ “ The introduction of Vere Foster’s copy-books in some of my [ schools has been attended with marked success.”

Superior JEditioti, 2d. each number.

Contents of

1.    Strokes, Easy Lettbrs, Short

Words.

1£. Long Letters, Short Words, Figures.

2.    Long Letters, Short Words,

Figures.

2£. Words of Four, Five, or Six Letters.

3.    Capitals, Short Words, Fig

ures.

3|. Sentences of Short Words.

4.    Sentences. Mostly composed of

Short Words.

4£. Select Quotations from Shakespeare.

6.    6. Sentences.—Maxims, Morals,

and Precepts.

6*. Sentences, in Writing of Three Sizes.

6£. Sentences, in writing of Two Sizes.

7.    Sentences, and Christian

Names.

8.    Sentences.—One Line on each

Page.

Primer II.—48 pp.,.

Primer Complete—80 pp.,................

Reader I.--64 pp., (Abridged Edition,) .............

Reader. I.—96 pp.,....................................... cloth boards,

Reader II.—128 pp.,......................................

Reader III.—196 pp., ....................................

Reader IV.—288 pp.,

Reader V.—320 pp.,...

Reader VI.—384 pp.,..

As specimens of good reading books, they cannot bo excelled. The exercises are carefully arranged so as to suit the meanest capacity, and at the same time have a tendency to make useful impressions ou the minds of young scholars.”—Rducational Guide.

ADDITIONAL READING BOOKS.

The Newspaper Reader, Selections from tbe Leading Journals of the Nineteenth Century on events of the day. By Henry Find later Bussey, and T. Wilson Reid. Foolscap 8vo, cloth boards, 288 pp. price 2s.

The British Biographical Reader, Sketches of prominent Men by Eminent Authors. With Introductory Notes by the Editor. Illustrated by numerous authentic Portraits. Foolscap 8vo, cloth board* 288 pp., price 2s.

The Shakespeare Reader, being Extracts from the Plays of Shakespeare, specially selected as fulfilling Article 28 and Schedule IV. of the Education Code. With Introductory Paragraphs and Notes Grammatical, Historical and Explanatory. Foolscap 8vo, 160 pp. cloth, Is.

Myths and Legends of Ancient Greece and Rome. A Hand book of Greek and Roman Mythology, for Schools and private Students. By E. M. Borons. Illustrated from Antique Sculptures, Foolscap 8vo, 330 pp., cloth 3s.

Poetical Reader, for the use of Elementary Schools in England and Scotland. Foolscap 8vo, 224 pp., cloth Is. 6d.

London, Past and Present. A Reading-book for Elementary Schools. With numerous authentic Illustrations. Foolscap 8vo, 288 pp., cloth 2s.

The Scottish Historical Reader; being Extracts from Eminent Writers Descriptive of Events in Scottish History, with Introductory notes. This volume will supply a fuller account of leading events in Scottish history than can be embodied usually in School histories, and being composed chiefly of extracts from the works of approved writers will form excellent reading lessons. It is adapted more especially for the requirements of the Fourth Standard.

HOME LESSON BOOKS.

The object of this series is to place in tbe hands of the Pupil a certain amount of Standard work, which will enable him to grasp the leading facts of English History, Geography, and Grammar, and at the same time lighten the work of the Teacher. The questions in Arithmetic are mixed in order to test the pupil’s knowledge.

Further Particulars and Catalogues from All Booksellers,

Or their Representative, GEO. STILLIE, care of Messrs. COWAN & Co.,

WHOLESALE STATIONERS, MELBOURNE.


JSejwjffl geprfmeni,

INSPECTORS’ REPORT FOR THE YEAR 1880.

The following passages extracted from the Reports furnished to the Education Department by the Inspectors, will not be without interest:— Thomas Brodribb, Esq., M.A.

INFANT TEACHING.

When I first took charge of this district, its infant teaching seemed to compare rather unfavourably with that of my previous district (the Western) ; but I am happy to report a considerable advance in this respect, good organisation and individual care being now the rule. I would, however, like to see more object lessons, more pictures, more reference to such external matters as would brighten up for little children the dulness of teaching. In two of the best infant schools that I have seen (Warrnambool 1743 and Barry’s Reef 885), the driest work, such as learning letters or figures, is made fairly interesting by being put in the most varied forms : letters are shown on a tablet, are chalked on a black-board, are exhibited and changed with moveable cardboard letters, are picked out from among others, are taken to pieces, and are given some verbal or fanciful association—the result being that children learn rapidly without apparent effort or weariness.

A mere course of reading, writing, and arithmetic, even though seasoned with rhymes and supplemented with school drill, is probably an insipid dish for the appetite of early childhood, for the infant mind ever finds sustained effort or abstraction wearisome or difficult. As far as I have observed, simple lessons in natural history seem the best suited to train attention into a habit, and to quicken interest in the outside world ; but still something more is wanted—something that will begin to educate the constructive powers of children. Here probably we shall bo led in time to recognise the merit of the Kinder Garten system, and to profit by Froebel’s teachings to the extent, at least, of introducing into our infant departments some of his simpler apparatus, and by showing our trained female teachers how to use it.

IMAGINARY OBJECTIONS TO THE SPREAD OF EDUCATION.

No doubt any one who now objects to the diffusion of useful knowledge will rightly be regarded as a veritable fossil ; but I have heard objections urged against the results rather than the theory of popular education. These objections amount, in brief, to two : first, that education causes young women to be disinclined for domestic service, preferring to it factory life, or some such occupation ; and, secondly, that it makes young lads show a distaste to manual trades, and seek rather to swell the ranks of clerks. I am inclined to think that these objections can be urged with only the smallest degree of truth. With regard to the first, it may be remarked that, even without any spread of education, the diversity of occupations now opening up to women would naturally draw off many who otherwise would seek a maintenance in domestic service; and to this reason another may be added : the literature of the seventeenth and eighteenth centuries show us that a kinder and better relation formerly existed between masters and servants than now prevails, and that then domestic service was socially belter, and often prospectively more advantageous. Pepys, in his famous Diary, speaks of the difficulty of engaging a suitable waiting-maid to replace one that bad left. The one that had quitted his service was a good singer, and his difficulty lay in finding a successor gifted with a voice who, like the former girl, should be able to join her master and mistress in the musical trios which they practised. Nowadays it would be thought strange for a gentleman of Pepys’ high social position to have his ser-vants°up to the drawing-room in order to practise part music with them. Again, in the Spectator, we read of servants being set up in business by their masters, or otherwise placed in a position of independence ; and the later dramas and earlier novels show the same, kindly, friendly relations. Looking at these facts, one must admit that there has been a falling oil in the standing nnd advantages of domestic service ; while it is certain that many other fields for woman’s labour have been opened up ; and to my mind these two circumstances sufficiently account-for any alleged difficulty of getting domestic servants, without in any way blaming our educational system for the natural result of other causes. With regard to the other objection, there is perhaps some truth in it: every lad that can write, spell, and cast accounts is a possible clerk ; and there is a danger that too many will seek a precarious and dependent living in such capacity, rather than the more stable and independent vocation of a handicraftsman. The prejudice against mechanical trades must wear away with time and the diffusion of education ; nay, if, as I hope may soon be the case, technical schools be established, tastes for various handicrafts will be developed, and mechanical arts will rise both in efficiency and social estimation • while our scheme of national education will be freed from the charge of diverting our youth from mechanical trades,

RETROSPECT.

By comparison we measure what is great or little—nay, often what is good or bad. Applying this test to the state of public instruction, and comparing its present condition with that of a dozen years ago, we shall find real cause for congratulation. We were then drawing towards the close of the period when the teachers’ ranks were too often recruited from the failures in other walks of life ; but classified teachers, though not the exception, were by no means the rule; and, when fouud, were commonly possessed of the lowest grade of classification, of what we now regard as so merely provisional a certificate that any special examination for it has been abolished. Even six years ago, the proportion of unclassified teachers was large ; and I then had in my district 33 unclassified and 107 classified teachers. Then, too, few of the teachers were trained, and few were acquainted with modern improvements;

while the school furniture was often rough and unsuitable, and the buildings were occasionafiy awkward and ill-planned, with poor means of warming or ventilation.

The briefest comparison will now show a great and salutary change : the school buildings are airy, cheerful, often handsome structures, erected with the utmost regard to sanitary and educational requirements ; while generally, though not wholly, the furniture is so designed as to give rest and promote convenience. Bnt more than this : every teacher in the Victorian service holds a diploma of qualification ; most of our pupil teachers go steadily on to the certificate examination ; we have in full working a sound and comprehensive course of training, which constantly adds to the ranks of our more highly qualified instructors; and, finally, we have at hand a large number of classified applicants, by whom our own vacancies can be quickly filled, aud by whose services other colonies are glad to benefit.

Report of Charles A. Topp, Esq., M.A., LL.B.

.    BOARD OF EXAMINERS’ WORK.

As the work of the Board of Examiners seems to increase year by year, and to seriously interfere with the inspectoral duties of its members, it becomes a question whether the functions of the present board might not, with advantage, be divided among several. There are various distinct classes of examination, viz., those for the State school exhibitions, those for students in training, those for pupil teachers, &c. It might be practicable from the inspectoral staff to form separate boards to deal with each of these, and so avoid the too engrossing claims on the time of the present board. The continuous preparation and perusal of examination papers for six or seven hours a day during the three or four hottest mouths of the year impose a mental and physical strain which is likely to affect injuriously the energies of the members of the board during the ensuing eight or nine months. It would also be a great boon if the bulk of the examinations could be held at midwinter instead of in midsummer ; not only would the strain on the examiners be less, but—a more important matter—the candidates would be placed in more favourable circumstances for dealing with the several papers. It must be almost impossible for an examinee to do himself justice at a written examination, when the thermometer records a temperature of 90° in the shade, and a strong north wind is blowing.

INSTRUCTION.

I may venture to express an opinion that, now that our schools arc, as a rule, officered by qualified teachers, properly trained in method, and generally anxious to do their duty, we shall see no further improvement of importance in the instruction given in State schools until the order and extent of the subjects of free instruction are reconsidered and rearranged. It rests with the department to initiate the next advance ; and just as efficient instruction and rapid progress are much facilitated, where a judicious time-table provides that subjects of instruction are taken in suitable order and for the right length of time, so must the efficient instruction and rapid progress of a child depend greatly on his being introduced to the various branches of knowledge in proper order and at an age when his faculties are fit to deal with them. I look forward to the happiest results from a careful but thorough revision of our course of free instruction. My views as to the direction in which I considered change desirable were fully explained in a paper read before the education section of the Social Science Congress last year, and I find that many inspectors point out changes urgently needed.

Robert Craig, Esq.

There are two parts of my district which appear to be over-supplied with schools, the Yan Yean road and neighbourhood, and the neighbourhood of Broadford. Generally, a rural district is considered well provided for if the schools are not more than four or five miles apart ; but in the localities referred to schools are found not more than three miles apart while there is one that is scarcely two miles from another.

In .some of the country townships a denominational school maintains a struggling existence side by side with the State institution, and at the same time that it does not itself thrive, it mars the success of the other. In one small village, all the children, if gathered under one roof, would make a prosperous school of about fifty, the teacher would throw that spirit into his work which the sight of a well-filled school imparts, and the pupils would derive benefit from the healthy competition which exists only where a considerable number are taught together. As the matter stands, the children are split up into two small cheerless schools. They are in neighbouring buildings, and, as may be expected, have no neighbourly feelings to each other. They form two camps, always unfriendly, and sometimes at war. Apart from the other disadvantages of this division, one cannot help regretting that the two sets of children, instead of learning to be good citizens through friendly competition in the school-room and playground, are kept apart, and led to regard each other almost as aliens.

Charles Tynan, Esq.

FURNITURE AND APPARATUS.

As the maps of Australia and Australasia supplied by the Department do uot appear to me to be of a sufficiently good character, either in their general construction or in the detailed information given, I should like to see maps of the separate colonies in more frequent use in our schools. Now that almost direct railway communication has been established between the two great capitals of Australasia, and in view of the advent of the day when the different colonies are united under one Federal Government, I am certainly of opinion that a much better knowledge of Australian geography than our children can now be credited with should be insisted upon, and could easily be secured.

COUNTRY SCHOOLS.

As might be expected, the instruction in the town schools is of a generally good character, The positions in the large town schools being

generally regarded as the prizes of the profession, it follows that, as a rule, the best teachers are to be found there, and good work is consequently the result. I say, as a rule, advisedly, for, without doubt, some of the most incapable teachers 1 have met with are still to be found in schools in close proximity to the metropolis.

With regard to the state of instruction in the country schools in my district, I fear that I cannot speak in very favourable terms. In the county of Mornington there are very few schools in which I have been able to report the management as satisfactory, and a combination on the part of the teacher of the qualities honesty, zeal, and ability is so rare as to evoke from me more than passing notice. Wherever any weakness in a school seemed to me attributable only to a teacher’s inexperience or lack of ability, I endeavoured, by making such practical suggestions and recommendations as I deemed necessary, to secure better work ; but where I had reason to impugn the teacher’s honesty or zeal, I brought the matter prominently under the notice of the Department, when such action was taken as effected an immediate change for the better in the cases of the particular schools affected.

NIGHT SCHOOLS,

Further experience confirms me in the opinion expressed in my previous reports as to the utter unworthiness of night schools, The recent regulation on the subject has had the salutary effect of closing the great majority of these schools, though a few still remain in the Metropolitan South district to entail considerable expense on the State without conferring corresponding advantages on the scholars in attendance. After some years’ experience on the subject, I have no hesitation in saying that, in the best managed night school I have ever inspected the expense incurred by the State is by no means commensurate with the intellectual benefits derived by the scholars, to make no mention of the probability, nay, the almost certainty of serious moral injury resulting.

RECORDS.

I regret to say that the school records in this district are not at all satis-torily kept. Besides cases of wilful falsification of the rolls, for which the offenders were severely dealt with, there have come under my notice, during the year, several cases of gross carelessness in the way of incorrect marking, and in some cases positive neglect to mark the roll at all. Notwithstanding the adoption by the Department of such stringent measures iu dealing with the offenders, I fear that the serious offence of wilful falsification is much more frequent in this district than is generally supposed. I shall make it my special business during the ensuing year to devote my energies and attention to discovering all such delinquencies, and in all cases shall recommend the Department to use the utmost severity in dealing with the delinquents.

Alfred Claribeux Curlewis, Esq., M.A.

Our secular system, no doubt, has its weak point. I allude to the want of familiarity with the language of that standard of the English tongue, the authorised version of the Bible, a want of familiarity shown by teachers and pupils. A teacher once objected to my using, in a sentence for parsing, the phrase “ they began to anger him,” urging Morell’s dictum against the use of nouns as verbs. Under the circumstances I consented to withdraw the unfamiliar word, merely expressing my regret (for the teacher was preparing for the ministry) that his knowledge of Morell was greater than his knowledge of the Bible, in which such use of the word is not infrequent.

I have had, in several instances, to examine in the art of teaching for a license those who had failed to pass the examination for the second year’s course of training. In all these cases the candidates had been admitted on passing cither the matriculation, or the ordinary examination for admission to training, and in none of these cases were the teaching abilities .of the candidates—as might be expected from the little experience they had had of teaching—of a high order. I do not know of any case of one admitted on his first-class pupil-tcachcrship failing at this examination. It seems a pity that, if we can get a sufficient supply of candidates from the pupil-teachcrs, we should admit those who are less likely to pass into the Melbourne institute, and, if they fail, are then thrown on our hands. There is, however, a want of eagerness among pupil-teachers for the training school, perhaps due to a feeling which seems to exist among them, and to which Messrs. Brodribb and Tynan refer in their last reports—an idea that there are means by which promotion may be more quickly secured than it would be by the possession of a training certificate.

I am still of opinion that, in cases of unavoidable absence of children from result examination, an allowance should be made. The six children of a gatekeeper may be removed by the Railway Department, and the teacher of the school they attend may thus be rpade to suffer in purse and prestige from a circumstance over which he has no control.

I am glad to find that the new regulation as to night schools has reduced to its minimum that necessary evil, and prevented the rise of those small schools which used suddenly to spring into being, drag out a weakly existence of a month or so, and then disappear.

James Holland, Esq., M.A.

BUILDINGS.

I have always found the school premises tidily kept at the time of my visits, and believe that it is the rare exception to find carelessness in this respect. There may be one or two cases of neglect in the district, but not more. If so, they have not come under my notice. In a straggling district like this, however, the exact whereabouts of the inspector is pretty well known, and slight deficiencies in this or any other matter may occur during part of the year, and yet be attended to prior to his visit. In compact districts, the constant liability to inspection has the effect of producing regular and uniform care and attention with regard both to school and school buildings.

FURNITURE.

I would again urge on the Department the advisability of not supplying slates, copy-books, and reading books until the intending recipients produce a certificate from the nearest clergyman or justice of the peace that the parents are too poor to procure them for the children. I have not the slightest hesitation in saying that the best reading and writing is found in schools where all the children provide their own books.

INSTRUCTION.

As tested by the result examinations, a very marked improvement has been shown during the year. The percentages, this year, have been unusually high. I am pleased, too, to be able to report that I see year by year a steady advance in the neatness and style of the papers received at the result examinations. Many of these are neatly and nicely written out, so as to facilitate correction, and show considerable taste in the arrangement of the work.

The proposal to do away with result examinations has not been received with favour. The strongest argument in its favour is the persistent recommendation of the system by the inspectors. They have nothing to gain by it. The work of the result examination is not inviting. There is nothing more monotonous than the treadmill work of examining young childrenday after day. Nothing but the deep-rooted conviction that these, or similar, examinations are absolutely necessary to the efficient and successful carrying out of the Education Act can induce inspectors to advocate their retention. There might be some modifications made in matters of detail. Slight examination iu programme work might be required in addition to the usual questions. On the other hand, as recommended by Mr. Curlewis, in his report for 1876, “ Concessions should be made in all cases of sickness or accident, where a doctor’s certificate is produced, and iu cases where children have left the district.” Generally, more discretionary power with regard to absentees might be left in the hands of the inspectors.

AGE AT WHICH CHILDREN ARE ADMITTED TO SCHOOL.

I think the minimum age at which children arc allowed to bo enrolled much too young. It is a pitiful sight to see babies of three years of age sitting in the gallery on a hot day, with their poor little limbs cramped in one position for half-an-hour or more at a time, and inhaling the over-heated atmosphere of a crowded infant room. It is absurd to suppose that such mites arc learning anything. They would be much better out in the open air, breathing “ God’s glorious oxygen.” Parents find it convenient to make a nursery of the school, and, no doubt, settle the account in doctor’s bills, and in the injured health of their oilspring. No child under five, or, at lowest, four years of age should, in my opinion, be admitted into a State school.

Walter M. Gamble, Esq.

■    BUILDINGS,

The residents in Sandhurst district No, 2 ought to be satisfied with the school accommodation provided for them by the State. 1 can only recall three localities where school-houses are needed to meet present requirements, and in each of these I have taken the preliminary steps to bring the matter under the Department’s notice. On the other hand, in two or three localities, the school houses already erected have ceased to be necessary, i.e., the school population has decreased, and thero is no immediate prospect of any increase of children in the neighbourhood, Murray Road, near Heathcote, and Mosquito Greek, arc cases in point; I learn that these schools are to be closed. A suitable building has been erected at Bailieston, so that the last unsuitable building in my district has been superseded.

Although it is now several years since the planting of school reserves was suggested, one must confess to a feeling of disappointment to see how little has been done in the time. Teachers ought to be alive to the fact that‘‘sins of omission ’’arc noted by the district inspector; and that since promotion may depend on the absence of unfavourable reports, it is for their interest to attend to the improvement of the school-ground.

DISCIPLINE.

It may not be complimentary to some, but it is nevertheless true, that the discipline maintained in the majority of the metropolitan schools which I have inspected compares unfavourably with that observed in Sandhurst schools    ‘

On one point—that of corporal punishment—I do feel considerable satisfaction that the department has, despite the pressure brought to bear during the year, re-affirmed its original ukase of 1873 against the flogging of girls. The thanks of every woman are due for the continued immunity of their sex from corporal punishment in schools, and, as our girls are thus protected from personal indignity, I am sanguine that in the next generation the brute who assaults a woman, whether wife or not, will be viewed as a hideous phenomenon, and execrated accordingly.

TEACHERS.

A perplexing question has frequently forced itself upon my consideration. What is to be done with those teachers occupying the better positions in schools, who not only do not deserve promotion, but are also unworthy of the positions which they now hold ? My reports to the office indicate instances of teachers in receipt of large salaries who are not making an adequate return.

I know that it is a question encumbered with difficulties; but if merit is to “ rise,” demerit must be displaced, no matter whether found in a half-time school, or in one with a thousand in average attendance.

If a teacher in a small country school absented himself from duty without leave, and the matter came to the knowledge of the Department, be would and should learn that a repetition of such conduct meant degradation if not dismissal ; and there should be no ground for believing that the same penalties would not fall on those in charge of the largest schools.

Hitherto, in matters educational, the race has not always been to the swift, nor the battle to the strong ; time and chance have happened to all alike. In future, we may hope that the anomalies will be reduced to an inappreciable minimum.

Alexander Stewart, Esq.

When the schools that are recommended are built, the requirements erf this district, as far as school accommodation is concerned, will be met. Of course under this remark are not included the two or three isolated localities where there are not sufficient numbers to warrant the establishment of even a half-time school. It would appear that, at the Colac Town Common school, the accommodation is insufficient, even with the addition of the rented building—at best only a make-shift; but it has to be remembered that the Colac school, which is only about two miles distant, is not nearly full. Tkere are, in all, some eight buildings which are rented, and Which, as a rule, serve their purpose as schools very fairly ; but tbe school accommodation cannot be called complete until those and one or two others, like that at Ilawkesdale, are superseded by new buildings.

Looking back at the work done during the past year, it is pleasing to note that it appears steadily improving. There are, it is true, a few instances where I have had to report unfavourably of the work, but happily these are few, and I have reason for hoping that this year they will be fewer still. I would indulge the hope that ere long unsatisfactory teaching will be one of those things “ that were.” Heading—under which is included comprehension of the matter read—is not so successfully taught as some of the other subjects. There is too much of what is called “ hearing children read,” instead of “teaching them to read,” while it is often forgotten that to give a satisfactory reading lesson it is necessary to make good use of the black-board. Neither can it be too strongly impressed upon teachers that, unless the meaning of the word or phrase given to the scholars is a living expression to tfcem, instead of another as lifeless to them as the one of the book, it is a waste of time to give it. I find that in those schools grammar and geography are best taught, and consequently best understood in the lower classes, where systematic class-teaching, combined with ample black-board illustration, is the rule.

In my opinion it would be an improvement if the Central Training Institution had its ranks recruited from those pupil teachers who, at the close of their own course, passed creditably the same examination as is now required for students at the close of the first year’s course of training. The advantages of this plan are obvious. At the same time it could be arranged that candidates of exceptional ability, other than pupil teachers, be allowed to enjoy the benefits of our excellent training institution.

Bogs Cox, Esq.

I have frequently oalled the attention of teachers to trifling repairs that were needed to the school furniture that could have been made right with a few nails or screws, and the expenditure of ten or fifteen minutes of time. But when anything is wrong about the school building or furniture, most teachers look to the Department to make it right, In old times, when the furniture and apparatus had to be kept in order out of school fees, any little jobs of the kind I speak of were performed by the teachers.

It is very rare indeed to see such a piece of school furniture as I saw last week at Eastville (1245). The teacher could not get a notation frame from the department, so she took the frame of a slate, fixed into it a row of ten knitting neodles, upon each of which she had strung ten quandong stones, and she had at once a neat and effective notation frame, without any cost to the department, and only a little trouble to herself. The teacher in question is, in other respects, one of the smartest in the district.

Teachers who are not able to compare their own work with others get so wedded to the ways they are used to that an inspector finds it very difficult to make them see that they might change their system of working with much profit to their schools. I think that a system of exchanges for short periods should be encouraged by the department. An assistant in a lar$e town school would gain valuable experience by taking temporary charge of a country school, while the teacher of a country school would be improved through observing the systems adopted in the management of large classes. Even where a head or assistant teacher exchanged for a time with one in the same class as his own, good effects would follow, as a man who really wiahed to improve himself could learn from the faults as well as from the superior merits of another school than his own,

The exception now is to find a school where the conduct of the children is not good. The only serious breach of discipline I have to record is unpunctual attendance, find where that occurs I think the teacher must be held responsible for it. Last week I paid two unexpected visits to schools only a few miles apart. Going to the first I overtook a number of children running, and asked one of them what was the punishment for being late ? The boy answered, “ No purfchment, sir. We are never late ; the teacher doesn’t like it.” In this school every child was prosent twenty minutes before morning roll-call. In the other case, though the school opened later, and the children had more time to assemble, nine of them lost their attendance mark through being absent at floll-<*vll. If the children had been asked in that school, I am afraid they would not have answered as in the other, that “ the teacher didn’t like it.”

I would reoommend that an abstract of all the circulars to teachers which are still in force should be printed and sent to every school in the colony, to be attached to the copy of the rules already furnished to the schools, I am frequently told by teachers that they are unaware of matters upon which definite instructions have baen furnished by circular. There are, of course, rnaoy yofing teaohers who have pot seen circulars issued before their time, and there are old teachers from whose memory the directions contained in a circular, read once and put away, may have passed. If those circular instructions were embodied with the rules, there could then be no mistake in the minds of teachers.

Henry Shleton, Esq., B.A.

FEMALE TEACHERS.

The employment of female head teachers is becoming more extended. I would suggest that the license to teach be abolished, or that females be required to pass an examination of equivalent difficulty to that passed by males. The attainments required of a female licensee are not sufficient to qualify her to teach to the full extent of the programme, e.g.—in grammar, analysis of sentences, and in arithmetic fractions, decimals, &c., are not required.

I would again draw attention to the constant changes going on among the head teachers of the district, and the increased difficulties which such changes place in an inspector’s way. A very large proportion of the teachers are inexperienced persons who, having succeeded in passing the literary part of the examination, have been appointed to the charge of small schools. Though these teachers are fairly qualified iu point of attainments, they possess little or no knowledge of teaching or school management. It is, of course, part of an inspector’s duty to instruct such persons on these points, but it is wot encouraging to find that, successful after one or more examinations iu the art of teaching, the teacher has fled the district, and that another recruit has taken his place. Yet this is what is coustantly occurring ; and a more important view of the matter, one has only to examine a school to which a new teacher has been appointed to discover how the school has sufEered by the mere fact of the change.

I find that the bulk of the people eagerly seize on the opportunities afforded by the State, and cheerfully send their children long distances, four, five, and even six miles. Some, on the other hand, barely comply with the compulsory clause, and with a view to this frequently consult the teacher as to the number of days which the children have attended; while others, of the same class, but of more business-like habits, keep records themselves ; and, when the Act has been satisfied, keep the children at home. In farming districts, however, where the labour of the children is of some money value, this is, perhaps, to be expected. Samuel Summons, Esq., M.A., LL.B.

The percentages obtained by some schools were not so high as in the previous year, yet in many cases I was better pleased with the work done. Part payment by results, acting as a stimulus to the teacher, and forcing him to give a certain amount of attention to every pupil, is without doubt necessary for the effective working of our present State school system. I regret, however, to report that the result examinations unduly influence the instruction. By slight changes comparatively in the form and mode of conducting these examinations much can be done to cause the instruction given to be less mechanical and more intelligent. The defective management of the infant department is apparently a fault not confined to this district. Teachers iu charge of these classes seldom utilised the appliances at hand, and their efforts to arouse and secure the attention were rarely well directed. The want of proper preparation was a fault rather common, and especially noticed in the lessons given by young teachers. Mr. Fearon points out that “ teachers who do not prepare their lessons become more and more inefficient, instead of improving, as time goes on.”

Richard Philp, Esq., M.A,

During the past year it has been my rather disagreeable duty to hold enquiries into several alleged cases of misconduct on the part of teachers and also to bring before the notice of the Department one very grosscase which happened under my own observation. Besides these instances, I have, in my unnoticed visits to the schools, seen many irregularities which, though not amounting to actual misconduct, show a careless and indifferent way of carrying on the business of the schools. Want of punctuality in attendance at their work by the teachers, and, inconsequence, by the scholars too; dismissing school before the appointed time ; rolls not marked ; times of arrival and departure of teachers at and from their schools recorded for days yet to come ; school-rooms and out-offices not well kept; and numerous other details of school management neglected or improperly done. All have convinced me that one great want of the present system of education in this country is the want of local supervision of the schools, No doubt the Education Act, by creation of Boards of Advice, provides for this, and, in theory at least, leaves nothiug to be desired. In large towns the system may, and Í believe does, work well enough ; but in the country districts it is impracticable. Without entering at all into the question whether or not the persons elected to Boards of Advice are fit and proper persons to exercise any control over the teachers, it is sufficient to state that there are some school-board districts which are so large, and in which the members live at such distances from each other, that, for all practical purposes there is no Board of Advice, while in other parts there is no Board at all, even in name; and still, in these parts, there are many schools and many teachers. In the absence of Boards of Advice, they are left absolutely without supervision, and it is not very wonderful (for teachers are no better and no worse than other people) that they should lapse into loose and irregular habits. It has been suggested to me by a gentleman, himself a member of one of these country Boards of Advice I have alluded to and who is quite alive to the defects of the present system, that perhaps the shire councils might he induced to take some charge of the schools. These bodies are generally composed of the best and most influential members of the community, and their members are sufficiently numerous to be well distributed over the country. I think the subject is one calling for the serious attention of the department, and I place his suggestion before them. Whether or not any change is made in the present Boards of Advice, I think it would be well to draw up directions for their use when visiting schools. The “ duties,” as defined in the Act, are not particular enough for use by men who have no special knowledge of schools. They are asked to record their opinion as to the general condition and management of the schools,” a direction which they sometimes construe as giving them a right to interfere with teachers in matters wfith which they have no right to interfere, while it is so vague in its terms that it fails to suggest to them many things that they might well take notice of, and, in doing so, do real service to the schools. It is a matter of ordinary business that a teacher shall be punctual and regular in his attendance at work ; that he shall follow his time-table, keep the records of his school correctly, see that the school-room is cleaD, well aired, and well heated ; that the offices are in proper order ; and many such things which require no technical knowledge in one who goes to see that they are done, but upon the doing of which a great deal of the effective work of the school depends. So far as 1 have seen, not only is the local supervision of schools imperfect, but, when they are visited by local authorities, what is to be done and what is to be looked after is not understood.

J. Dennant, Esq.

Perhaps no better method of estimating the benefits the Education Act has conferred on the community could be adopted than to travel amongst the schools in some of those remote localities, far removed from rail or coach, where almost the only strange face the children look upon during the whole year is that of the school inspector, who comes to see that they are educated up to the same standard as their fellow scholars in town or city. Were it not for the facilities the Education Department has afforded during the past few years, these children would have grown up in the blankest ignorance. In a few places, where, from accidental circumstances, a school has only recently been opened, it is found that every scholar, from 15 years of age downwards, has to go into one common class (the first), and start by learning the alphabet.

I am glad to observe that the study of science has at once become popular with almost every teacher, both male and female, and during the current year many of the subjects specified will be taken up, the requisite books being now readily obtainable. There is no doubt but that the result will be highly beneficial to teachers and scholars, in enlarging their minds and leading to habits of observation, &c.

(To be continued.)

TEACHER AND PUPILS.

Relations op the Teacher to the Pupils.—It is not necessary or advisable that the teacher should display his authority on all occasions before his pupils. Such a course cannot but weaken it, and decrease the respeckjwhich ought to be felt by the pupils towards their instructor. He should rather endeavour to make the children feel that they are under the control and supervision of a superior and stronger mind, whose decisions must on all occasions be acquiesced in. The thoughtful teacher will readily discover various means by which he can influence his pupils in an indirect and yet very effectual manner. For instance, he should carefully encourage every indication of pleasurable feeling on the part of the studious pupils, and he should endeavour to increase and develop that feeling. Moreover, he should appeal to so many of the senses as possible in every exercise, and present the particular subject in a variety of aspects, so as to suit the capacity of every child, and to appeal to the strongest sense in every individual child. For this purpose he should employ pictures, sketches, and specimens where possible, especially in the case of the lower classes, where the eye is the most powerful instrument of education, and thus make the instruction intuitive as far as possible.

Another most important matter which ought never to be lost sight of, and which is nevertheless very frequently disregarded as a matter of no practical importance whatever, is the proper condition of the schoolroom, so as to secure the health and comfort of the pupils. The room ought to be properly ventilated, and the temperature of a proper degree. It ought to be kept in as perfect a state of cleanliness as may be consistent with the inevitable dirt and disorder of a school. In fact, the room ought to be to some degree a model of cleanliness, which the pupils would bear in mind in their own homes. It is impossible to over-estimate the immense moral importance of a well-regulated and properly-cleaned 6chool-room. The children should be shielded, as far as possible, from corruption and moral taint, and this can only be done by making a weekly visit to every part of the premises, and carefully examining the whole building. Then, again, the apparatus and school furniture should be carefully cleaned periodically, not only for the purpose of preserving them in proper order, but also with the view of practically inculcating upon the children the necessity for habits of cleanliness and order. The habits formed at school will in all probability be carried into their own homes in future years, and thus the teacher may exert a powerful-influence for good on the whole lives of those under his care.

Further Relations op the Teacher and Pupils.—The teacher must carefully strive to assimilate his instruction, in language and subject-matter, to the mental condition of his pupils. In order to do this effectually, care should be taken in the first place to ascertain the exact acquirements as well as mental capacities of his pupils, and afterwards by carefully preparing his lessons for each class. There can necessarily be but little success, and but few appreciable results of teaching, w'hec it is of a desultory and disconnected kind. The children can scarcely remember loose, unsystematical instruction, which costs the teacher little or no effort to impart, and which he has made no endeavour to bring down to the level of their own comprehension. Much of the wasted time and the instruction thrown completely away, might have been utilized if the teacher had properly understood the mental state of his pupils, and had then endeavoured to impart instruction in an agreeable and suitable manner adopted to their condition. This is, in fact, one of the great secrets of success in all teaching—to understand how

much the pupil knows, aud then to impart information concerning that which he does not koow, iu the method and manner best suited to his mental condition.

Method of Instruction.—The instruction imparted must not only be sound and accurate in matter, but also imparted in the manner most suited to the requirements of the pupils.

(a) . The language must be as simple and free from difficult and technical terms, even in the case of the higher classes. Technical term« should always, indeed, be introduced with much caution. The idea, embodied in or expressed by the particular technical word which tho teacher wishes to impress upon the class, ought first to be carefully explained ; and after it has bGen made perfectly clear and intelligible, the technical term employed to express it should then be brought forward. It is always objectionable to use unexplained technical terms, leaving the class to arrive at the idea conveyed by it ns best they ntay. The result always is, that the children’s notions respecting such words arc either very vague or quite erroneous, so that much of the oral distraction addressed to them is perfectly useless aud unintelligible.

(b) . The instruction must, as far as possible, be connected with the previous knowledcje of the pupils. The first rudiments of knowledge having been imparted, the teacher should build up the superstructure byproceeding from the known to the unknown, but always connecting them together, so that the child will be assisted to remember the facts by association as well as by the effect of memory. Apt illustrations aught also to be frequently employed to bring out unknow* or unintelligible acts more clearly.

(c) . Another point to which the teacher’s attention ought to be directed is, the exemplification of the practical utility of all the knowledge imparted. A child is far more likely to remember knowledge which hs feels can be applied to the ordinary purposes of every-day life, than if he was unable to perceive that the recollection of it would be of b*tb little if any service to him hereafter.

(d) . The instruction must also be carefully graduated, so that the knowledge may be thoroughly accurate. Much of the vagueness and inaccuracy which is so common in primary schools, is no doubt to be attributed to the want of proper graduation. If a child be suddenly removed, for example, from an elementary reading book to one that Is much more advanced, it will readily be believed that, it will never snccood in mastering the language and style, unless very great care and attention is bestowed on it—more than can be given in the ordinary class instruction. The child should therefore be led gradually and carefully from one difficulty to auother, as the only means of making the instruction thorough.


THE TOILET—DRESS, NEATNESS AND SUITABILITY.

As Dr. Doran remarks, Man is the only animal born without being provided with a necessary costume ; plante die that naan may live, and animals are skinned that the lords of the creation may be covered.” It» is therefore essential that the toilet should be a matter for thought anujl consideration for every one.

Now this chapter is not intended to be a dissertation upon fashion ; that I leave to the dressmakers’ monthly periodicals, for

‘‘ Our dress stl'l varying,

Nor to forms confined,

Shifts like the sand, tho sport Of every wind

I shall simply show what is the style and character of dress appropriate for wear on different, occasions.

Iagiee with Dr. Watts, that

“ It is in go'od manners, and not in good dress,

That the truest gentility lies

but still I think the two go very much together, and that dress has a certain effect on the character and manners. Most people hold that tho reverse of this is true, and that a person's dross is influenced by hi« character. Probably each has an influence on the other ; but be that as it may, an ill-dresfied man is never so much at his ease as a well-dressed man, and I believe that mean aud shabby clothing has an unconscious hold on the mind.

Costly thy habit as thy purse can buy.

But not expressed in fancy—rich, not gaudy;

For the apparel oft proclaims the man.”

I have elsewhere protested against “best” rooms and “ company” manners, and would here remark that the habit of being particular in our attire only when we appear before “ company” is a bad one, and an “ ill habit has the force of an ill fate.” The eyes of those who form our home circle should never be distressed by an untidy appearance. Circumstances may forbid our garments being either rich or costly, but neatness and simple elegance can always be shown in every dress and at every season. “ My wife appears decent enough in her apparel to those who visit us in an afternoon ; but in the morning she is quite another figure,” writes a oioinplainaot.

There is no easier method by which to detoct the real lady from the sham one than by noticing her style of dress. Vulgarity is readily distinguished, however costly and fashionable the habiliments rnay be, by the breach of certain rules of harmony and fitness. No one, perhaps, can dress perfectly without a genius for it, but every one can avoid vulgarity and slovenliness, and attain the average standard of gentility. Neatness we have spoken of as a requisite element, and another and all-important one is suitability—suitability as to various times and seasons—suitability as to age.

A dresa which would look perfectly well on one occasion will appear


out of place and vulgar on another. A costume in which a young lady looks bewitching, makes an older one look absurd and ridiculous.

Our neighbours on the other side of the Channel, who are always held up to us as models of taste, arc very particular in these points—neatness and suitability—and we must own that an Englishwoman rarely presents an appearance as elegant as a French woman, even though the attire of the former may be gorgeous, and that of the latter simple and plain in the extreme. The French excel, too. in the assortment and harmonising of colours. They never dream of decking themselves in all the hues of the rainbow ; one, or at the most two colours predominate in their whole dress; and whatever the colours chosen, they are selected with a view to suit the complexion of the wearer. Alas and alack ! for Englishwomen in respect to these matters. Here, you see one with drab face and drab dress ; there, one bedizened like a harlequin ; some in silks and satins, lace and jewels, when the occasion demands that they should be plainly clad in stuff's and linen; mother and grandmothers affecting a style of juvenility which would look charming on their descendants, but ill-becomcs their grey heads and wrinkled brows. The old lady I saw at a college concert, who had placed on the back of her grey-haired head a brown chignon, and considered that a sufficient ornament with the addition of cap or lappet, and the one I met at a ball, attired in white satin, with her bare seventy-year-old neck and shoulders powdered and devoid of scarf or shawl, forgot the fact that the aping of juvenility “ multiplies the wrinkles of old age, and makes its decay more conspicuous.”

But lot us proceed to review the various styles of dress proper for different occasions.

The dress worn by a mother at her infant’s christening and her own churching should be plain and neat, but handsome and substantial. There is an entry in King Edward II.’s “ Household Book” to the following effect : To the Queen’s tailor was delivered five pieces of white velvet, for the making thereof a certain robe against the churching of the Queen after the birth of her son.” A silk dress will always look well ; a muslin, however beautifully fine, will look out of place.

The costume for paying calls when on foot differs from that which should be worn for the same purpose when driving in a carriage. In the former case it will be of a much plainer character. It may be light or dark, according to the season ; but it must not be gay, aud not have anything about it to attract attention, but be like that of the lady of whom Dr. Johnson said that he was sure she was well dressed, because he could not remember anything that she had on.

Carriage dress has much more licence. Handsome silks, with elaborately trimmed and sweeping skirts, feathery bonnets, and lace parasols, which would look quite out of place when walking, are suitable when driving. A few years ago light coloured gloves were invariably worn when paying calls, except by persons in deep mourning, but latterly black kid gloves have been fashionable at all times and seasons, and with every style of dress, whether simple or elaborate. These gloves are long, and are fastened with many buttons ; bangles or bracelets are worn over the glove.

The toilet for lawn parties, bazaars, flower-shows, &c., is of a brighter, gayer fashion, and affords room for the display of much taste and elegance. Young ladies attire themselves in delicately tinted fine materials —materials which have a refinement, beauty, and softness characteristic of those whom they are designed to embellish, but quite distinctive from those worn in the ball-room. These costumes are made as effective and coquettish as possible—everything that will add to the gaiety, without passing the limits of morning attire, is permissible, and the whole is crowned by a bonnet or hat of like description. In towns the former is mostly worn, but hats are allowable in the couatry for young ladies. The elder ladies should wear silks or some handsome material, richly trimmed with lace, a foreign shawl, or lace mantle, and bonnets, not hats, whether in town or country.

Costumes for picnics, excursions, and for seaside wear should be of a useful character. Nothing looks worse at these times than a thin, flimsy fabric, which will split and tear at every turn, or a faded, shabby silk ; and nothing looks better than some strong material, either one that will wash or otherwise, but of such a description that it will look almost as well at the end of a day’s hard wear as at the beginning.

Yachting dresses arc generally made of serge or tweed, as those materials are ur.spoilable by sea air and water, and at the same time possess warmth anti durability.

Lawn-tennis dresses are usually white, cream colour, or other pale shade. They arc made of strong and thick materials. The dress has a ; short plain skirt without frill or flounce ; its ornamentation consists of I embroidery in coloured crewels. The bodice is full and put into a band, which is stitched to the skirt. Another kind of costume suitable for players is that of dark blue or black jersey, which is put over the head, and fits close to the figure without fastening of any kind. A skirt of the same colour is worn. Either costume is not complete without a large apron, with bib and shoulder straps, and one large pocket across the front of the skirt, or two of smaller size. This apron is made in thick washing material, and is embroidered.

And next of indoor dresses.

A lady’s morning dress should be simple and refined, and suited to the time of day. No old “company” gown should exhibit itself and its shabbincss in the morning light, but a dress fresh as the morn itself, as inexpensive as you please, but clean and appropriate it must be. Lace, unless of a thick description, is not worn with morning attire. Honiton and Brussels would be quite out of'‘place. Neither is much jewellery consistent ; plain gold and silver ornaments arc permissible, but never precious stones, except in rings (the clear stones are only worn with full dress).

When visiting at a friend s house the morning dress may be of a slightly superior style ; for instance, a white dress embroidered with coloured crewels may be worn where a plain cotton one would be used at home, or a silk dress instead of a woollen one ; but remember that “great external display is generally said to be employed to hide internal vulgarity,” and we know of nothing which evinces not only greater want of taste, but also ignorance of the habits of good society, than smartness in morning attire, and especially a lavish display of jewellery.

Again, a dinner dress differs from that worn at a ball, although they both may be termed “ full dress.” For the former occasions silks and satins, velvets and brocades, are the materials chosen, and are trimmed with lace. The neck and arms of the wearer are now generally covered, excepting at a specially “ full dress” dinner ; the bodice is made high, but open in front, and the sleeves reach the elbow or a little below it.

Young ladies often wear coquettish combinations of soft silk, lace and flowers, in the form of caps, on this and similar occasions, or they place natural flowers in their hair in the manner most becoming to the shape of the head or the style in which the hair is arranged. Older ladies wear larger caps, or lappets, with the addition of flowers or feathers.

In the ball-room nothing but complete full dress should be worn. For young people dresses of some light gossamer fabric are chosen—net, tulle, grenaclis, or gauze, trimmed with ribbon or flowers, made low7 and with short sleeves. The wreaths of flowers worn in the hair are generally artificial, because natural ones so soon fall to pieces from the heat of the room and the movements of the dancers.

The dress of the chaperons should be similar in character to that worn at a dinner. Jewellery is generally worn in sets ; ornaments never look so well if pieces of different sets are displayed together ; that is to say, if diamonds are in the brooch, a necklet of pearls, and earrings set with, emeralds would not look w7ell if worn on the same occasion. All the ornaments should match in character as much as possible, but variety is allowed in the matter of bracelets.

( To be continued.)

REFINEMENTS OF THE DRESSING ROOM.

A London aunt has recently given some valuable practical hints on the art of beautifying the dressing room or bedroom, and carrying into it that atmosphere of lefinement which should pervade every apartment in a lady’s house. The brush cases, toilet tidy, square of muslin to throw over clothes at night, combing jackets and dressing gowns having been fully described, I will venture to bring forward one or two little contrivances of my own which have added much to the appearance and comfort of my own particular chamber. When a room does not happen to contain a hanging wardrobe, an excellent substitute may be effected by means of a set of those portable folding American pegs, which can be bought for a very small sum, fastened to the wall by strong nails. But dresses and cloaks are not sightly objects when hung up ; and if not covered they catch the dust in a manner very detrimental to their preservation. So I have adopted the plan of making a cretonne curtain (a light ground is the best) the required width and length, with several curtain rings at the top. I then procure at any ironmonger’s two of those little brass hooks to screw into the wall, which are used for hanging up cups in China closets—the largest size of these—and a strong piece of cane about three-quarters of a yard long. I screw in the hooks just over my pegs, run the cane through the curtain rings and fasten it up, the two hooks supporting each end. Thus a portable hanging wardrobe is at once made, and when the room is swept nothing need be done save to turn the flowing curtain inside out and pin it tightly round the dresses underneath it. When there are pegs behind the door the same sort of curtain can be put up. and has all the appearance of a portiere. The cretonne should match the window curtains and harmonise as much as possible with the shade of the carpet and the whole tone of the room. Boxes and trunks, which never look nice in a bedroom in their natural state, may be converted into ottomans by cretonne covers, made to fit loosely and take off and on. A flat piece lined for the top of the box, a piping cord round, and a loose flounce gathered on is the best way to make them. And when curtains, box covers, portiere, and hanging wardrobes are all made of the same pretty light cretonne to match, the effect is exceedingly good. When “ doing up ” a bedroom it is well to buy as many yards of cretonne as you are likely to want at first, in case of not being afterwards able to match the pattern. Nothing looks worse than a “ patchy ” room, and the idea should be fully carried out or not attempted at all.

A low chair is an addition to a bedroom which is almost indispensable, and I myself prefer one of the small folding ones which do not take up much room, and are so cosy to sit in over the fire during bedroom chats, and when brushing hair. The chair cushions should be covered either with the same cretonne as the curtains and boxes, or with sateen of the prevailing tone, and finished off at each side with a bow of ribbon. A little gipsy table—just large enough to write at—should accompany the chair, and be provided with a neat, pretty blotting case, small inkstand, and pen-wiper. A small hanging bookshelf is also desirable, and some framed photographs. If a bedroom, however small, is tastefully fitted up, and contains, above and beyond the needful accessories of the toilet, some suggestion of the culture and natural bent of its occupant, it is so much easier to bear any occasional illness which may confine us to it for a long or short, period. When the favourite books are within sight or reach, the writing materials at hand, and all the little objects round us which we have collected together from time to time and interested ourselves in arranging, the hours drag on far less heavily, and we can hardly deem the enforced quiet and retirement a hardship—indeed I have sometimes felt it to be the reverse in the case of slight ailments,

Sdente <§0sstp.


Regarding the influence of the moon in our atmosphere, Mons. de Paristle says :—“Along series of observations has shown that the moon, •which passes every month from one hemisphere to the other, influences the direction of the great atmospheric currents. The changes in these currents, in consequence of the prevailing moisture or dryness, are intimately connected with the relative position for the time being of the sun and moon. The distance of the moon from the equator—that is, the inclination of the moon’s path to the plane of the equator—varies every year, passing from a maximum to a minimum limit; and the meteorological character of a series of years appears to be mainly dependent upon the change of inclination, when those extreme limits have been touched. Observations prove that the rainy years, the cold winters, and hot summers return periodically, and coincide with certain declinations of the moon. Applying the deduced rule to the present period, we find that the next wet year ought to coincide with the declination of 18 degs.; therefore, with the year 1884, as the last was 1879, with the declination of 26 degs. Consequently, the dry summers should come about the middle of the intervening period between those two years—that is, they should be 18S1 and 1882,

It may not be generally known that 60 per cent, more work can be done with pulleys faced with leather than with a nude surface. Leather belts used with the grain side to the pulley will do more work and last longer than if used with the flesh side to the pulley. This is owing to the exclusion of air, consequent upon the smoothness of both surfaces. Leather belts with the grain side to the pulley can drive 34 per cent, more than when the flesh side is inwards.

The presence of magnesia in water may be ascertained by boiling the water to a 20th part of its bulk, dropping a few grains of neutralised carbonate of ammonia into about a small glassful, and adding a small quantity of phosphate of soda. The magnesia, if present, will then fall to the bottom of the glass.

A ready method of testing whether water contains organic pollution is to cork up a bottle nearly full and into which a piece of lump sugar has been dropped. If, after standing for two or three days, there does not appear a milky cloud, the water remaining pure and clear, it is free from the deleterious phosphate which is found in sewage water.

A new antimony mine has recently been discovered in the province of Quebec, America.

Herr Rinman asserts that the magnetic intensity of iron is regulated by the amount of carbon it contains.

From investigations made by a company of five underwriters in Hew York, it appears that certain kinds of silk are so laden with combustible chemicals that they are always liable to take fire spontaneously. Recent fires during transit both by rail and sea of packages of silk have led to this belief. These chemicals, it also appears, are an adulteration, and it is stated that the art has reached such a pitch that the original weight of the natural silk can be increased fourfold without the alloy being apparent.

The Engineer says :—The small quantity of carbonic acid always present in sea water is due to the incessantly renewed supply afforded by oxidation of organic matter in the sea itself, and not from the air. If this supply were not constantly maintained this constituent would vanish from the ocean. Its higher percentage in the lower strata of the sea is doubtless due to three causes—(1), to the comparative stillness of the water, whereby the diffusion of the solution is retarded ; (2), to the absence of direct contact with the air and exposure to the wind; (3), and chiefly to the increased pressure whereby solution of the gas is greatly facilitated ; for under pressure of one atmosphere and at ordinary temperatures one cubic centimetre of water dissolves in round numbers one cubic centimetre or L529 milligrammes of carbonic acid, while under double that pressure the absorption is double, and so forth, varying directly as the pressure approximately. It can hardly be doubted that this presence of a larger proportion of carbonic acid in the lowest depths of the ocean has a distinct correlation to the character of their special inhabitants.


Matter.


If we wish to ascertain the temperature of a hot bath, or of a room, we uso an instrument called a Thermometer.

(In an upper class derive the word.)

lThis may be shown by the hand, which when cold may easily be gloved, but swells with heat, so that the glove can scarcely be drawn on.

s Water is one of them.

Illus. (1) by cistern pipes in winter; (2) Bomb shells, 13 in. incHameter and 2 in. thick, with their fuse holes plugged with iron, bursting when exposed io the severity of a Canadian winter.

3Why? Because spirits of wine cannot, by any applications of science at present known, be made to freeze.

* Why glass? In order that its indications may be the more readily noted.

“Illus. a bulb by an onion or tulip.


“Explain the “zero” of the thermometer, which is 32° below freezing, hence arrive at the graduation of the scale.

7Shew its application ns seen in (a) hot, baths ; (6) rooms; (c) cucumber frames, See., See.


* Derive the word'.

«The teacher should draw diagrams of each on the black-board, and work out imaginary questions, illustrating the various scales.


I must now say a word as to the washstand contrivances. When a washstand is not made of marble, it is best to cover it with marble oilcloth, for nothing looks more common and ugly than a painted wooden stand, and the paint wears off almost directly, from the splashing of water and soapsuds. Sponges should be kept in a wire basket fastened on to the side of the washstand, and tooth and nail brushes should always be kept in one of the neat open racks which have lately superseded covered china dishes. Taste may certainly be exercised in choosing between china, without going to expense, as very pretty patterns are now sold in the cheapest china. But. to my mind, the simpler the pattern the better—a Greek border, as it is called, of any colour which suits the room, always looks neat and nice, and can be matched easily, if any of the set gets broken. The washstand screen may be made of muslin, lined with glaze calico, like the brush covers and nightgown case, which has the best effect; or one of the cane screens sold at every linendrapers may be used. Muslin drapery for the toilet table is out of fashion ; but in cases where it is necessary to adopt it, to hide a common table, of course the glaze underneath must match the other surroundings ip colour.

It adds much to the appearance of the window to arrange the muslin blind in the French way—a strip of plain book muslin gathered up and tied in the middle with a bow of ribbon, to accord with the other appurtenances of the room—blue, mauve, rod, or whatever the principal colour ay be.—i; Brachne ” in The Queen

NOTES OF A LESSON ON THE THERMOMETER,

BY J. WALKER.

M KTHOD.

I.    Principle explained.

(1)    Take a bladder filled with air and place it before the tiro. It will be found to increase in size, in proportion to its increased temperature, until, if not removed, it will eventually burst.

(2)    Our bodies are larger when we are hot.1

From these two instances we see that :

With a few exceptions- nil bodies expand by heat, and

That the greater the heat the greater the expansion of the body.

II.    Construction,

The amount of expansion heat produces may be measured by either solids, liquids, or gases. Of these, liquids are generally chosen, for solids expand too little and gases too freely, to be of great use. The liquids employed in measuring heat are spirits of wine, or alcohol, and mercury, or quicksilver ; the former being used more especially for low temperatures.3

The thermometer consists of a glass tube,'1 terminating in a bulb.5 This bulb and part of the tube are filled with quicksilver, by a peculiar process, which expels all the air, thus leaving a vacuum ; the upper extremity of the tube is then sealed by melting the glass.

But the instrument as thus constructed would not indicate how much one body was hotter or colder than another. It must therefore be graduated.

This is done as follows :—The thermometer is dipped in melting ice or snow ; the mercury at onee sinks to a certain point called the freezing point,. It is then plunged into boiling water, when the mercury rapidly rises to another point in the tube. This is called the boiling point. These points are marked, respectively, 32° and 212° ; the intervening space being divided into 180°. A thermometer of mercury cannot be graduated below 40°, because at that point mercury itself freezes, but it may be graduated upwards to 670°.0

III.    Use.

To ascertain various degrees of heat.7

Thai the thermometer does not measure the quantity of heat may be shown by the following :—Dip it into a basin of water, and then into a wineglassful taken out of the basin, when it will stand at the same height in both, although it is manifest there will be more heat in the basin than in the glass.

Remarks.-—The date of the invention of the Thermometer and the name of the originator are unknown ; but it is generally believed to have been invented about the beginning of the seventeenth century. To Reaumur is due the credit of having proposed quicksilver instead of linseed oil, which had been used by Newton. In 1721, a Dutchman, named Fahrenheit, considerably improved it; since then it has undergone little alteration.

In conclusion the teacher may notice the three kinds of thermometers, and explain their scales, thus :

(1)    Fahrenheit’s begins at 32° and ends at 212°.

(2)    Centigrade begins at 0° and cuds at 100°.8

(3)    Reaumur begins at 0° and ends at 80°.

Here it will be noticed that 9 spaces of 20° each on the F. will correspond to 6 on the C. and 4 on the It.®

At a meeting of the Paris Academy of Sciences, M, Faye agreed with Newton that the tails of comets were nothing more than a continual emission of molecules from the head of 1 lie comet. Mons. Roche, taknig this theory as a basis, has accounted for the production of all the various shapes of comet tails as witnessed by observers.

School Department—

Inspectors’ Deport for the

Year 1880     50

Teacher and Pupils ...... 53

The Toilet—Dress, Neatness,

and Suitability ...... 53

Refinements of the Dressing Room ............ 54


NOTICES TO CORRESPONDENTS.

Advertisements and other business communications should be addressed to the Publishers. No advertisements will bo inserted without a written order, or prepayment. It is particularly requested that they may be sent early in the month.

Books, music, and school appliances for notice, and all letters containing anything connected with the literary portion of the paper should be addressed To the Editor. Every communication accompanied by the name and address of the sender (as a guarantee of good faith, though not always for publication) will bo acknowledged ; but wo cannot attend to anonymous letters.


¿lustralashw Srljflulmastrr.

PUBLISHED EVERY MONTH.


MELBOURNE, OCTOBER, 1881. The report presented to His Excellency the Governor, by the Hon. the Minister of Public Instruction, on the working of the State school system of Victoria, has just been laid before Parliament. Its appearance at a time when the composition of the proposed Royal Commission of Inquiry is about to be discussed in the House of Assembly, may be considered opportune. That teachers though out the colonies may become thoroughly conversant with the valuable statistical information with which it is replete, we have inserted it in full, together with the expressed opinions of most of the inspectors on the manner in which the functions of the teachers are being performed. Even a cursory perusal of the Minister’s report must convince the most sceptical that the State schools have taken a firm hold upon the community, and that there is no ground for alarm respecting the stability of the system.

The total number of schools under the department (including night schools) has now reached 1,810. The number of children enrolled during the year was 229,723, while the number of children in average attendance throughout the year was 119,520. Taking the state of the schools in the year 1872— the first year of the enforcement of the free, secular, and obligatory Education Act—and comparing them with the year 1880, we find the following result:—For the year 1872 — Number of day schools, 1,048 ; number of scholars enrolled, 135,962; average attendance, 68,436; number of night schools, 1 ; total number of scholars enrolled, 93 ; average attendance, 20. For the year 1880—Number of day schools, 1,624; number of scholars enrolled, 216,854 ; average attendance, 115,160; number of night schools, 186; number of scholars enrolled, 12,869 ; average attendance, 4,360.

Though the general tone of the report is one of confidence in the practical educational \york achieved, the Minister has shown


CONTENTS

Science Gossip ...     54

Notes of a Lesson on the Thermometer ......... 55

Leaders—

Educational Report ...    ...    56

Dr. Andrew Bell ...... 56

Notes of the Month ...    ...    57

Educational Report for 1880-81 59

ANSWERS TO CORRESPONDENTS

“ Pedagogue.”—We can but repeat the advice so often given in the pages of the Australasian Schoolmaster. Never pen a single line to a member of Parliament about your claims to promotion or your supposed grievances that you do not wish to fall into the hands of the Department. Members of Parliament are too busily employed to make explanations for you, and unthinkingly hand to the Minister your lettei’3, that you may explain your own case. Moral : Write discreetly, when writing about yourself, and never affirm anything that you do not wish to be called upon to put to the test of proof.

“ F. U. IIenderosn,” “A. McDonald,” “A Boy,” “ E. Row,” “ E. Newman,” “Eynon,” “ Grinder,” “Samuel Friend.”—Received.

“ Inquiry.”—Pupil Teacher.

“ No. 1420.”—Music received. Notice next issue.

NOTICE TO ADVERTISERS.

In sending advertisements for insertion in the Schoolmaster, advertisers will please remit stamps for amount at the following scale :—

16 words, Ono Insertion - Is. Od 1 32 words, One Insertion - 2s. 6d. 24    ,,    ,,    - 2s. Od I Ono Inch ,,    - 4e. Od.

no disposition to exaggerate the progi'ess made in the past, nor any wish to underrate the dangers which beset the Department in the future. Two extracts will exemplify this point. “ It will be observed,”says Mr. Grant, page iv., ‘That while the number of distinct individual children under instruction in day schools during 1880 has increased beyond that for the previous year by 3276, there is not a corresponding increase in the average attendance, which is only 556 in excess of that for

1879.    .    .    . The percentage of average attendance to the

number enrolled has fallen from 52-35 in 1879 to 52-02 in 1880, As a rule day schools are open for 230 days in the year, and as the attendance of 187,210 distinct children enrolled during

1880,    resulted in an average attendance of 115,160, the number of days attended by each scholar on the roll during the year is shown to be 141-4. The number of days attended during 1879 was 143-3 ; in 1878, 141’7 ; and in 1877, 140-7.”

Regarding the cost of education, the report says, pagevi.:— “ The cost for the instruction of each child in average attendance during the year 1880 was £3 18s. Compared with the cost for the preceding year, these figures show an increase of 2s. 9d. per scholar in day schools, and of Is. 0^d. in night schools.” And on pagexiii., the drain upon the departmental funds caused by the expenditure on repairs to school buildings is thus noted :—“The annual cost of maintenance uf school buildings cannot be estimated at less than ¿£22,000 per annum ; but as the sum of £20,000 has been placed on the estimates for 1881-82, there is a prospect of being able to deal with the most pressing requirements before much further injui’y occurs to the school buildings. A sum of £50,000 to £55,000 was estimated as necessary to meet both the annual cost of maintaining the property of the Department, and of pi'oviding for such new applications as would be caused by further settlement, together with the growth and shifting of population, and there is no reason to think that this estimate is in excess of what will be needed for year's to come.”

Some of the causes which have led to the apparently small percentage of increased results in teaching, and to the self-evident increased costliness of our educational system, are stated in the report. We have no doubt tha •sufficient reasons may be assigned, and that when the indirect benefits conferred upon the rising generation by the improved methods of instruction, and by the more healthful and cheerful class of school buildings are taken into account, the State does receive a substantial return for the money expended by the Education Department.

DR. ANDREW BELL.

It is not as a disciple of Dr. Bell’s once famous Madra8 system ”—it is not as an apostle of Dr. Bell’s method—tha^ Mr. Meiklejohn has produced (says the School Board Chronicle) this biography    of the man of one idea in

national instruction. Andrew Bell is a tolerably interesting figure in the history of national education; his life forms a landmark in the progress of the work ; and hence our author, having access to special information, has been tempted to add this biography to the annals of the last hundred and twenty years of the education movement. He has taken means to make the book interesting reading. Bell was born at St. Andrew’s, and Mr. Meiklejohn begins his task with a very fresh and happy description of the ancient North-British town. He is evidently no very great admirer of Dr. Bell; but he tells a man’s story with appreciation and impartiality, and does more than any writer before him to place him before the reader as he really was, with his merits, his great strength of will, his fixity of purpose, his narrowness, and his defect of culture. It was almost in the infancy of method in instruction that Dr. Bell thought he had made a great discovery. His monitorial system has developed into the pupil teacher system, and the pupil teacher now, in the opinion of many competent thinkers and practical educationists, stands in the way of the higher development of the elementary school. When it comes to estimating the character of Dr. Bell, and putting a value upon his system, Mr. Meiklejohn does not take a high tone. He says :—

He was not an interesting man ; he was not a great man ; he had very-little insight into human nature, though here and there arc to be found

glimpses of the truth ; he was singularly narrow-minded ; and he was in several respects a terrible bore. There is in his own mind hardly atrace of education—hardly the smallest sign of literary culture.

From the worldly point of view he was a very successful man. He saved money ; he became a dignitary of the church ; and he founded the National Society. But as an educationist he would have narrowed the schooling of the poor to the very wretchedest limits; and his system has in the main disappeared. His father was a barber in St. Andrew’s. When lie left school, he went to seek his fortune in America, and took an engagement as private tutor at «£200 a year. When he died, at an advanced age, the greatest trouble of his last hours was what he should do with the £120,000 which he had amassed. Of his authorship Mr. Meiklejohn says :—

His character is faithfully mirrored in the style of his writings. Cumbrous, clumsy, chaotic, dull, even to heaviness, full of involutions, repetitions, misplaced limitations ; it is a severe penance to be obliged to read a page.

And again :

The educational works of Dr. Bell amount to several thousand pages; but they cannot be recommended to a perusal of even the most enthusiastic student of education. There is much dust, chaff, and inorganic matter in them ; and it is only here and there that one finds something worth picking up.

But Dr. Bell was a man of mark in his time. His energy was great. The manner in which he pushed his system brought the world nearer to the realisation of the necessity for public elementary schools. And Mr. Meiklejohn has given us the best life of him—a biography, which is a good readable book.


Ilotes of fíje Ktonflj.


Br this issue we publish in full the report of the Minister of Public Instruction, on the operations of the Education Department of Victoria for the year 1880-81. Extracts from the Inspector’s reports will be found on page 50. The pressure thus caused upon our space has compelled us to hold over several important items of scholastic news.

We have received from Mr. S. Mullen, of Collins-street East, a copy of “ The Melbourne University Calendar for Academic year 1881-82” ; also “ The Source of Growffh of the English Language,” by Thos. Page. From Mr. E. W. Cole, Book Arcade, Bourke-street, a New Verbatim Translation of the Sixth Book of Caesar de Bello Gallico,” by Rev. W. Matthew and Rev. Dr. Williams, M.A.

Last evening the Hon. Sir Bryan O’Loghlen laid upon the table of the House a copy of the Royal Commission, to enquire into the working of the Education Act. The following are the names of the Commissioners : Mr. Francis Ormond, the founder of Ormond College; Mr. George Meares, late Mayor of Melbourne; Mr. J. W. Rogers, Q.C., Professor M'Coy, Dr. W. H. Cutts and Messrs. Herbert James Plenty, J.P., Edmund Keogh, J.P., Henry Nicholas Loughnan, J.P., Duncan Love and Wm. Peterson.

At a meeting called for the purpose of initiating a Bible-in-Schools Association in Invercargill, held on the 30th ult., the following resolutions were unanimously passed (1.) That this meeting express its profound regret that in the New Zealand Education Act no provision was made for daily Biblc-reading in the public schools of the colony. (2.) That this meeting record its thanks to the Hon. Dr. Menzies, and those members of Assembly who supported him in his efforts in last session of Parliament to amend the Education Act in the direction of allowing Bible-reading in the public schools, subject to a time-table and a conscience clause ; and further, that the meeting express its sorrow that the bill making this amendment was thrown out by the House of Representatives. (3.) That it is desirable to form a Bible-in-Schools Association in Invercargill.”

The recently-instituted system at Cambridge, of imparting instruction to women by correspondence, comprises the teaching of English history, languages and literature, arithmetic, algebra, music, biologies, and chemical sciences. In connection with this is a Teachers’ Education Loan, managed by a committee, who lend money for fees without interest to correspondence students, on certain conditions.

The highest salary paid to female teachers in Germany is £50,

The subjects taught in French primary public schools are only six, strictly adhered to, namely Reading, writing, arithmetic, French grammar, French history, and general geography.

The Sydney Mail says “ The great majority of the women who are asking for the benefits of University education in England have no private means, and it is urged that they be not shut out from scholarships at the Universities, as they have been shut out from scholarships at preparatory schools. In many parts of the country the Endowed School Commission has been redistributing old endowments so as to increase the benefits derived from them, and to extend those benefits to a larger number of persons. But by far the larger part of these improvements has been made to apply to boys only. Girls have still been left to find for themselves what boys are fortunate enough to kavc found for them-”


The Marquis of Salisbury has lately been urging the extension of the higher branches of education to women, and their admission into our chief English seminaries.

A deputation from the Victorian Academy of Arts was introduced to the Chief Secretary on Thursday, the 29th ult., to ask for a vote of £1000 towards the putting up of an additional room, in connexion with the present building on Eastern Hill, in which the Society now carries on its business. The deputation represented that an additional gallery was necessary, for the success of the yearly exhibition of the Society’s productions, and pointed out that the Academy was an important element in the art-training of the youth of this colony. ‘ There was a night class for men at which 15 students attended, and two day classes for men and women with 80 students. There was also in connexion with the academy, a sketching club of 80 young men. It was also represented that, if instead of the building only being rented from the government, they had been endowed with a grant, there would have been no need of the application then being made, as the society could have raised money on the mortgage. Mr. Grant, in reply, suggested that a Conditional grant might be issued on terms which would admit of a mortgage. The efforts of the members of the Academy in the cause of art had his cordial sympathy. The estimates for the present year were now completed, but lie would do his best to meet the requirements of the Academy.

The proposed Southland (N.Z.), Bible-in-Schools Association will bo formed by the following gentlemen, who have been appointed a committee for the purpose :—The Revs. W. P. Tanner, P. W. Fairclougb, J. Ferguson, C. E. Ward, J. G. Paterson, Geo. Johnston, the Hon. Dr. Menzies, Messrs. J. Bain, M.U.R., Dr. Hodgririnson, T. M. Macdonald, George Trew, S. J. Deck, I). Smyth, G. Froggart, Wm. Russell <)■ Turnbull, H. Ilawson, D. L. Matheson, A. J. Ellis, J. L. M'Donald, A. Bethune, T. Denniston and J. Healey.

The Otago High Schools Board have decided to erect a bridge over the Wyndham River, in line with the road to be formed, for which they are selling land. The terms upon which the land is to be disposed of was left to the arrangement of a committee.

The Auckland Education Board have objected to the formation of free evening classes in the public schools, as proposed by the City Schools Committee.

The returns of attendances at the Otago public schools for the quarter ending 30th September show an increase of nearly a thousand. At the commencement of the quarter the number on the roll was 18.380, at the close 19,187. The average attendance was 15,190 The numbers, however, have not yet reached what they were before the outbreak of nmasles.

The following satisfactory report has been given as the result of a late inspection of the Winton State school (N.Z). Mr. P. Goyer says the school has passed an excellent examination. The only standard that is at all weak is the fourth. The writing has very much improved since last year’s examination. The drawing is very creditable indeed. The singing is good, scale practice is good, and the theory of music fair. The school-room is clean, the furniture and apparatus well taken care of. The children are very^ well-behaved,and the discipline and class movement are good. The examination papers are for the most part worked out in a very neat and methodical manner. The percentage of passes gained by scholars was 87-40.

At a meeting of the Wairo, N.Z., School Committee held on the 5th of August, Mr. John Beck was elected chairman for the ensuing year, and Mr. R. Bellenger to fill the combined offices of secretary and treasurer.

The Auckland Board of Education have decided to discontinue Hie practice hitherto followed of discounting salaries of teachers 10%. This resolution commenced to take effect on the 1st ult.

The monthly meeting of the Invercargill School Committee was held on the 27th September, Mr. Matheson in the chair. The head master’s monthly report of the central school showed 587 pupils on the roll, the average attendance being 425. The attendance at the north school was reported by Mr. Orr to be quite up to that of the prior month. The condition of the fencing was bad. It was resolved that the Education Board be requested to repair it. It was stated by the secretary that the accounts showed a deficit of about £30.

The following dates have been fixed by the Melbourne Universily for matriculation examinations during October term, 1881:—Monday, December 6, English (pass), English (honours), Arithmetic ; Tuesday, December 0, Latin (pass), Latin (honours), Elementary Botany; Wednesday, December 7, Geometry (pass), Geometry (honours), Geography ; Thursday, December 8, French (pass), French (honours), Elementary Physiology ; Friday, December 9. History (pass), History (honours), Elementary Chemistry ; Monday, December 12, Algebra (pass'), Algebra (honours), Elementary Physics; Tuesday. December 13, Greek (pass), Greek (honours) ; German (pass), German (honours). The last day of entering for this examination is Saturday, November 5, 1881. Examinations commence at 9.30 each morning.

A MEETING of the senate of the Melbourne University was held on the 4th inst., the Warden, Dr. Madden, in the chair. Dr. W. E. Hearn was elected to fill the vacancy in the council, caused by the death of the late Mr. Justice Stephens. The question of increasing the number of subjects necessary for medical students to have passed in prior to their entering upon the first year’s course for the degree of Bachelor of Medicine, was raised by Professor Halford, who initiated a debate on the matter by moving that the subjects of English History. Geography, and Elementary Physics, should be added to the six already required to be passed in. The motion was seconded by Professor Nanson, and spoken to by Messrs. Morrison, Sutherland, Dwyer, Dr, Williams, and Professor Elkington, mostly adversely. Mr. Sutherland pointed out that having to pass six subjects already frightened medical students, and he was afraid nine would frighten them away altogether, The debate was adjourned. The adoption of a recommendation of the council which would effect certain alterations in examinations was then moved by Mr. Leeper. The chief alteration recommended was toinsertin the place of sec.5,chap.

(3 of statutes and regulations, the following:—“The person delivering lectures in any subject shall be the examiner for that subject in all ordinary and honour examinations, and in addition to the examiner the council may appoint co-examiners not exceeding two in number for each subject, and the said examiner and co-examiners shall be the Board of Examiners for that subject. If for any subject no co-examiner be appointed, the decision of the examiner shall be final.” Professor Elkington moved that the matter be referred back to the council for consideration, as the proposed alteration would have a very injurious effect upon their present system. The debate was adjourned till Friday, the 14'h inst., oh the motion of Mr. E. E. Morris.

At Ingham, Queensland, a public meeting was held in the early part of last month to consider the progress of matters in connection with school accommodation there. It was reported that a considerable sum had been raised by some energetic gentlemen in the district.

A GRAMMAR school at Maryborough, Queensland, was opened on the 12th ult., when Mr. Murdoch delivered a speech appropriate to the occasion. There were enrolled 26 girls and 24 boys after the opening ceremony.

A petition was presented to the Queensland Legislative Assembly on the 13th by Mr. Macrossan from the School of Arts there, praying some assistance, as they suffered greatly from want of funds now that the Government grant had been withdrawn. The petition was laid upon the table.

The girl’s school at Maryborough, Queensland, will not be proceeded with as rapidly as could be wished. The promoters find themselves short of funds, and, therefore, will have to be more tardy in expenditure. Tenders were opened for the erection early last month, and it turned out that the lowest was for a good deal higher sum than what the trustees had in hand. Mr. C. H. Barton has been awarded the office of second master in the boy’s school.

REGARDING the giving of lessons in agriculture in the Queensland schools the Burleigh correspondent of the Queenslander says :—‘‘As we are a purely agricultural community here, a very great benefit would be conferred on the rising generation were Sir Arthur Palmer—our present energetic and practical Minister for Education—to cause the various teachers of our State schools to teach the elder boys some small lessons in agriculture, by giving them each a small patch of ground, and inculcating into them the difference of various soils—their applicability to certain kinds of crops—the effects of manuring, draining, and manipulating such soils. This might be made a most interesting and useful adjunct lo their usual lessons, and Vie of untold value to many of them in after-life. The system is iu active work in France, and is very ably handled and explained by Scudamore in his little book on Brittany. There is plenty of land attached to our schools, and, were seeds and implements provided, the rivalry would soon make the thing a great success, and this would be greatly increased were our Agricultural Society to take the matter into consideration, and devise a method of rewarding the boys.

The following dates have been fixed for the ordinary examination of the October term, 1881, at the Melbourne University Monday, November 7—Junior Greek, senior Greek, advanced surveying, metallurgy.— Deductive logic, inductive logic, mining. Tuesday, November 8.—Lower mathematics, geology and palaeontology, English language and literature, civil engineering. Junior Latin, senior Latiu, applied mechanics. Thursday, November 10.—Upper mathematics, History of the British Empire, Part II.; advanced natural philosophy. Advanced mathematics, ancient history, mining law, jurisprudence. Friday, November 11.— History of the British Empire, Part I. ; surveying and levelling, law of contracts. Poli tical economy, practical mensuration, mechanical drawing. Monday, November 14.—Comparative anatomy and zoology, surgery, obstetric medicine, law of property. Practical chemistry (1st division), forensic medicine, descriptive and surgical anatomy. Tuesday, November 15.—Medical chemistry, theory and practice of medicine, Roman law. Materia medica, clinical surgery, constitutional law. Wednesday, November 16.—Elementary natural philosophy, clinical medicine, anatomy by dissections, international law. Chemistry, mineralogy, and botany; operative surgery, law of procedure. Thursday, November 17.—General anatomy, physiology, and pathology; law of obligations, practical chemistry (2nd division). Das. and surgical anatomy (oral), junior ; des. and surgical anatomy (oral), senior ; law of wrongs.

A State school is to be erected at Lytton, Queensland. The local building committee have raised the necessary lodgment, and placed it in the hands of the Government. The committee have sufficient money, not only to erect the school, but to fence in the five-acre paddock in which it is to be built, which they intend to have done before the school is opened.

It appears that there is much diversity of opinion in the Queensland Legislature regarding compulsory attendance at State schools. An article by a local litterateur concludes as follows:—“The strangest part of these annual discussions (in the Assembly on education) is the blindness of Hon. Members to the inefficiency resulting from the non-enforcement of compulsory instruction. The chief justification for free education is the imperative necessity of making instruction universal. Yet a very large number in the aggregate of children living within reach of State Schools fail to attend them, and are growing up in deplorable ignorance. The fault, of course, in these cases liSs primarily with the parents ; but are not the Government, being armed with statutory powers to bring these parents to a sense of (heir duty, also guilty of culpable neglect in failing to enforce the law ? The very existence of State education implies that the Government have assumed parental responsibilities in this respect, and hence the non-enforcement of the compulsory provisions of the Act

when necessary is a standing reproach to those charged with its administration. To say that compulsion is not practicable is to dispute the existence of known facts, In England, in Victoria, and in New South Wales the compulsory system is enforced with beneficial results, and there can be no question that withiD certain prescribed limits it might be bene: ficially enforced in Queensland. Nor can it be reasonably held that our State Education Act has had a fair trial while this one of its provisions is allowed to remain a dead letter.”

The opening of a Free Library in connection with the Brisbane School of Arts has been under consideration for a long time, but seems difficult of effectuation. The committee lately had under its consideration an offer made by the Council, but this involving too much contingent expenditure the offer had to be declined.

On Friday, the 14th inst., the senate of the Melbourne University held an adjourned meeting to consider the recommendations of the Council in the matter of the appointment of co-examiners or assistants to examiners in ordinary and honour examinations, and for alterations in the mode of conducting examinations. Dr. Bromby presided. Dr. Morrison moved, and Dr. Fetherston seconded, “ That the senate while approving of the two main principles involved in the proposed statutes, viz. 1. Thatthe members of each board of examiners shall be jointly and severally responsible for each paper ; 2. That all the answers of every candidate in any subject shall be submitted to the decision of at least two examiners, and no candidate shall be passed or rejected without the concurrence of at least two examiners—do now proceed to the consideration in detail of the new statutes and regulations for the purpose of making such amendments as they think fit.” Several amendments were moved in this, and negatived. Dr. Hearn suggested that the new standing orders, 73 to 80 inclusive, be adopted for the purpose of considering the council’s proposals in committee, was agreed to, and on the motion of Mr. Leeper it was resolved to receive the proposals of the council and go into committee to consider them. This was done. In committee it was agreed “ That for any subject of examination, not a subject of lectures, the council shall appoint au examiner.” That the persons delivering lectures on any subject shall be examiners on that subject,” and “ Iu addition the council shall, when practicable, appoint two other examiners for each subject, and the whole of the examiners thus appointed for each subject, shall be the board of examiners for that subject.” The senate then adjourned.

Mr. Norton, during consideration of supply, in the Queensland House of Assembly, on the 6th inst., moved that it was desirable to discontinue the annual grant for scholarships. He urged that if it were true these scholarships enabled the poor man’s son to stand on the same level as the rich man’s, it was also true that it enabled the rich man to get an expensive education for his son at the lowest possible cost to himself, the greater part of the real cost being contributed by the poor man. The reply was given by the Colonial Secretary that these scholarships had done a great deal of good, not only directly, but indirectly, by encouraging other than successful pupils to increased exertion. The motion was supported by more than half-a-dozen members, on the ground that entirely free education had a demoralizing effect upon the community, and that those who were able to pay should have an opportunity of doing so ; and also that the present system was fostering an unhealthy desire amongst youth to aspire to professions and occupations free from manual labour. The motion was lost by 3 votes—IS for, 21 against.

In consequence of the rapidly increasing necessities of the district of Maryborough, Queensland, in the way of school accommodation, it has been decided to erect a new primary schoolhouse in Albert-street, of that city, amidst the large population around the adjacent foundries. Active canvass is being made to secure the required funds.

A special meeting of the School Commissioners of Otago, N.Z.,was held at Dunedin recently, at which the Gladstone Borough Council were granted a road line through the local Collegiate Reserve. The Commissioners decided, after consideration, to offer area3 under their control for sale for coal-mining purposes.

It having been decided by the Invercargill Education Board to frame regulations for the giving effect to the compulsory clause of the Education Act, they, at their September monthly meeting, passed a resolution to ask the Defence Minister to authorise the police to assist school committees to enforce the regulation.

The monthly meeting of the Education Board, Invercargill, was held on the 7th inst., when all members were present. A circular from the Education Department was read, stating that the grant for building purposes allotted to the board for the present year was £4590. The Executive Committee reported that at a meeting held on the 12th and loth September, it was resolved. (1) That Mr. Wm. McConagh be appointed temporary teacher of the Moke Creek School. (2) That Mr. Joseph Ivilburn of Miller’s Flat school, be appointed temporary teacher of Dipton schooDiffee Mr. Robert Haswe’l, resigned, (3) That the tenders of Messrs Thorn anJ Rough for erecting Schoolhouse at West Piaius for £149 15s ; for clearing one acre of bush land, for £6, be accepted. (4) That the tender of Mr. John Michie for re-roofing the schoolhouse at Campbeltown and other repairs as specified, for £47, be accepted. (5) That the carpenter’s report on the residence at Waikiwi be approved of, and that he be instructed to carry out the work as specified, cost not to exceed his estimate. (C) that the applicants of Fernhill district for anew school, be ; referred to the Inspector to visit and report. The report was adopted. A letter from the Wairo school committee, anent the Western boundary of the school district, was referred back to the committee for information as to the additional number of householders it would include. Notice of motion was given by Mr. Feld wick as follows ; —“That all applications for the creation of new School districts should include a statement of the boundaries the petitioner’s propose should include the new school district.” A deputation consisting of Messrs J. W. Hamilton, B. B. McKenzie, Instone, and Foster urged the erection of a school at Thornbury, The

Number

of

Schools.

Total Number of Children Enrolled during the Year.

Number of Children in Average Attendance throughout the Year.

Boys.

Girls.

Total.

Boys.

Girls.

Total.

Day Schools. Total in operation Less “ Struck off and attendance transferred to other Schools.”

1,644

j »

110,374

536

107,579

563

217,953

1,099

59,243

325

56,594

352

115,837

677

Balance ...

1,624

109,838

107,016

216,854

58,918

56,242

115,160

Night Schools. Total in operation Less “Struck off and attendance transferred to other Schools.”

186

10,2S5

2,581

12,869

3,421

939

4,360

Balance ...

186

10,285

2,581

12,869

3,421

939

4,360

Gen’I return for year

1,810

120,123

109.600

229,723

62,339

57,181

119,520


petition was referred to the Inspector to visit and report. It was resolved on the motion of Mr. Lurnsden, seconded by Mr. Deniston, hat plans and specifications be prepared for new schools at Olama and Limestone Plains ; and also for new residences at Otatara Bush, Heddou Bush, Pukerau, Wyndham, and Edendale, and submitted to the Executive committee, with power to invite tenders. Accounts amounting in the aggregate to £1341 2s. 9d. were passed for payment.

The Victorian Government have decided to appoint a Iloyal Commission to investigate the Education Act and its working. The names of the Commissioners have not yet been published.

Mr. Grant, Minister of Education for Victoria, attended at the Central State School, Spring Street, Melbourne, on Friday the 7th inst., and distributed the prizes gained by pupils at the annual examination. In the course of an appropriate speech, he stated that in the making of appointments, no other influence than that of merit would be his guide. Referring to the allegations as to the system of education being a godless one, he said that he considered such allegations groundless. Not only did a religious tone pervade the books, but a reverence for the Deity was everywhere inculcated, and the fact that SO % of the pupils of State schools attended Sabbath schools, indicated that the children were not left without religious teaching. He spoke in terms of favour as to the valuable assistance given by Boards of Advice ; and congratulated 'the teachers of the Central school upon the prominent position which the school had attained. Messrs J. Turner, I. Warren Ball, and P. Whyte also addressed the meeting.

The following circular to head teachers throughout Victoria has been published by the Education Department -It having been represented to the Hon. the Minister of Public Instruction, that the circular re the resting of female pupil teachers has been disregarded in a great many schools, head teachers are now informed that it is the Minister’s desire that the instructions as to female teachers not being required to stand for a longer period than an hour and a half at a time is to be strictly complied with.

EDUCATION REPORT FOR 1880-81.

The Minister of Public Instruction, Hon. .J. M. Grant, has submitted the following report on Education, for the year 1S80-S1, to Parliament:—

I have the honour to submit to Your Excellency the Report of the Education Department for the year 18S0. together with a statement of receipts and disbursements up to the 30th June, 1881. In the last report was shown the advancement in school provision made up to the end of 1879, when there were in operation 1,533 day schools. On the 31st of December of that year, 3 of these were closed, leaving 1,530 day schools in operation on 1st January. 1S80. During the year, 114 hew schools have been opened and 20 schools closed, leaving 1,624 as the total number of day schools in operation on 31st December, 1880. Of the 114 new schools, 111 were opened in districts previously unprovided with the means of State education. The other 3 supplied the place of schools which bad been closed. Of the 20 schools struck off the roll, 5 were lacking in the attendance of pupils sufficient to warrant their retention. 10 were discontinued as full-time schools and worked half-time with others, 2 were amalgamated with the same number in their neighbourhoods, and 3 were superseded by new schools. Of the 1,624 day schools in operation at the end of 1880, 1,580 were worked full-time, 43 half-time and 1 one-third time. There were thus 1,669 localities provided with State schools. This shows an increase during the year of 87 full-time and 4 half-time schools. There is no change in the number of one-tliird time schools, and the increased number of localities provided for is 95. At the end of the year 1879 there were about ISO night schools in operation, but only 121 of these furnished returns during 1880. In the latter year, 65 new night schools were established, making the total number from which returns were received 186. Of these, 60 were devoted to the instruction of boys only, 10 to Ihe instruction of girls only, and 116 to the instruction of both boys and girls.

The following table shows the number of schools in operation, and the number of scholars enrolled and in average attendance during the year :—    _

From a return of the children present in day schools on the 1st December, 1880, it was found that, out of 117,917 children, 13,014, or 11’03 per cent., had attended one other State, school during the year ; 1,233. or 1-04 per cent., had attended two other State schools during the same period; and 214, or 018 per cent., had attended three or more State schools during the year. It thus appears that of 117,917 names entered on the rolls, 16,122, or 13 67 percent., would bo found a second time. When the necessary deduction, therefore, is made from 216,854, the total number enrolled, 1S7,210 is found to be. the number of distinct individual children attending State schools during 1880. Compared with the number attending during 1879, the increase in the number of children enrolled is thus 3,276. The percentage of children enrolled in more than one school has fallen from 16T63 in 1878 and 13’93 in 1879 to 13 67 in 1880. With regard to night schools it was found from a return similar to that furnished by the teachers of the day schools, that, out of an attendance of 2,154 children on 1st December, 33’75 per cent, had been enrolled in more, than one school during the year. It the total number enrolled, 12,869, be reduced by the above percentage, it will be found that the number of district indvidual children who attended night, schools during 1880 was S.526. This shows a falling-off of 1,128 children as compared with 1S79. Combining the returns of the day and night schools, it will be seen that the total number of distinct individual children enrolled in them during the year 1880 was 195,736, being an increase of 2.148 over the number enrolled during the year 1879 ; and this increase of enrolment may be looked upon with satisfaction.

The following table allows a comparison to be made as to the progress from year to year since 1872 in regard to the number of schools and the number of children enrolled and in average attendance : —

Year.

Day Schools.

Night Schools.

Total.

4-1

O

fsj

3 'S

A c/-

Total

Number

Enrolled

Average

Attend

ance.

Number

of

Schools.

o a £

h ” a 'A W

Û) ,

S"§ %

h § c

> L os

So!

3 t

« A

Total

Number

Enrolled

Average

Attend

ance.

1872

1,048

135,962

68,436

1

93

20

1,049

136,055

68,456

1873

1,078

207,826

98,746

29

1,580*

790

1,107

209,406

99,536

1874

1,111

21C», 144

104,375

56

5,020'

2,511

1,167

221,164

106,886

1875

1,203

212,041

98,456

117

8,492

3,0,39

1,320

220,533

101,495

1876

1,317

217,038

102,515

181

14,522

4,243

1,498

231,560

106,758

1877

1,410

217,114

110,444

216

17,405

5,571

1,626

234,519

116,015

1S78

1,456

215,355

111,278

208

15,814

5,330

1,664

231,169

116,608

1879

1,533

213,726

114,604

3 80

11,049

4,655

1,713

119,259

1880

1,624

216,854

115,160

186

12,869

4,36(

¡1,810

229,723

119,520

* The figures marked (°) are an approximation only, the returns of numbers on rolls of night schools lor these years being incomplete.

It will be observed that, while the number of distinct individual children under instruction in day schools during 1880 has increased beyond that for the previous year by 3,2,76, there is not a corresponding increase in the average attendance, which is only 556 in excess of that for 1879. This check in the rate of advance is due to the epidemic diseases which prevailed to an unusual extent during ten months of the year, necessitating the temporary closing of many schools, and considerably affecting the attendance at the remainder. The percentage of average attendance to the number enrolled has fallen from 52'35 in 1879 to 52'02 in 1880. As a rule, day schools are open for 23 ) days in the year, and as the attendance of 187,210 distinct children enrolled during 1880 resulted in an average attendance of 115.160, the number of days attended by each scholar on the roll during the year is shown to be 141.4. The number of days attended during 1871 was 943 3 ; in 1878, 141 '7; and in 1877, 1-10*7. The causes which, as stated above, reduced the average attendance will account for this decrease also during 1880.

In the following table the results as to average attendance in Victoria are compared with those in other Australasian colonies : —

Colony.

Number Enrolled during the Year.

Average

Attendance.

Percentage of Average Attendance to Number Enrolled.

New South Wales ...

149,112

70,505

47 '28

South Australia ...

40,578

19,058

48-44

New Zealand ... ...

117,418

60,625

5D63

Queensland ... ...

43,305

23,818

55-00

Victoria... ... ..

229,723

119,520

52-02

Returns have been received showing the ages of 216,361 of the children enrolled in day schools during the year, and from these it appears that there were—

Under 6 years of age ... .. ..

... 29,790

Between G and 15 years ... ...

... 175,345

Above 15 years......... ...

... 11,226

Total .........

... 216,361

Returns of a similar kind received from the night schools embraced a total of 9,799 pupils, of whom there were—

Between 12 and 15 years ... ...

... 5.105

Above 15 years............

... 4,694-

Total ... ... ...

... 9,799

Reducing these numbers to correspond with individual children in attendance, we have—

the number of distinct

Under 6 Tears.

Between G and 15 Years.

Above 15 Years

Total.

Day schools... ... Night schools ...

25,776

151,720

4,442

9,714

4,084

187,210

8,526

Total... ...

25.776

156,162

13,798

195,736


Oat of every 100 children attending State schools during 1880 there were therefore—


Day Schools.

Night Schools.

Day and N ight Schools Combined

Under 6 years of age ...

13-77

13-17

Between 6 and 15 years

81*04

52 TO

79-78

Above 15 years ...

5-19

47-90

7-05

Total .. ...

100-

100-

100-


The following table shows the number of children who complied with the legal requirement of 30 days’ attendance in each quarter as compai’ed with the previous year :—


Total

Number who attended during each Quarter.

Number who completed 30 Days’

Attendance.

Percentage who made 30 Days’

Attendance,

1880.

Percentage who made 30 Days’

Attendance,

1879.

Quarter ending March 31, 1880...

171,156

120,061

70-14

69-18

„ „ June 30,1880...

174,080

131,014

75 26

73-03

„ „ Sept. 30, 1880...

174,054

134,032

77-00

80-66

., Dec. 31.1880...

170,740

113,004

66-18

74-12


It will be noticed that, while the March and June quarters compare favourably with the corresponding quarters of 1879, those ending in September and December respectively exhibit a falling-off; but this is to be accounted for in a great measure by the large amount of sickness prevalent during the latter half of the year, to which reference has already been made.

The following statement gives the number of children who completed the legal attendance for each quarter of 1880, and of those who did not, together with an analysis of the cases of irregular attendance Total number who attended March quarter, 171,156; June, 171,080; September, 174,054; December, 170,740. Attended 30 days and upwards —March, 120,061 ; June, 131,014; September, 134,032; December, 113,004. Attended less than 30 days—March, 51.095; June, 43,066; September, '10,022 ; December, 57,736. Exempt on account of not being within the school age—March, 12 359 ; June, 11,215 ; September, 11,969; December, 13,183. Exempt on account of living beyond the prescribed distance—March, 4,910 ; June, 4,151 ; September. 2,953; December, 5,067. Exempt on account of being educated up to the standard—March, 2 455 ; June, 2,351 ; September, 2,100; December, 4,092. Exempt on account of sickness—March, 5,644; June. 4,180; September, 6,209; December, 10,300. Exempt on account of having entered late in the quarter, or left before its termination—March, 10,781 ; June, 12 289 ; September, 9,922 ; December, 8,838. Balance of defaulters—March, 14,946; June, 8,880 ; September, 6,869; December, 16,256.

From the foregoing it will be seen that the greatest number of actual defaulters in any one quarter was 16,256, while the least was 6,869. For the year 1879 the figures stand respectively at 15,022 and 8,514,

Returns furnished in accordance with the provisions of section 10 of The Edvcntwio Act Amendment Act were received from 620 private schools for the year 1880, being 46 less than for the previous year. A list of these schools, with the attendance at each, will be found in appendix K. The following is a summary of the returns :—


Under 6 Years of Age.

Between G and 15 Years.

Above 15 Years.

Age

not stated.

Total.

Boys .........

2,427

13,919

1,733

591

18,670

Girls .........

2,716

15,415

1,823

700

20,654

Total ......

5,143

29,334

3,556

1,291

39,324


To these must be added the estimated attendance at 38 schools, from which, owing to various causes, no returns were received for the year 1880, though they were known to have been in operation for at least a portion of it. This estimate, based upon the returns furnished for the year 1879, is as follows :—Boys. 313; girls, 460—making, when added to the above figures, a total of 18,983 boys and 21,114 girls, or 40,097 scholars who attended private schools.

Much difficulty has always been experienced in obtaining complete returns from private schools, many of the principals strongly objecting to furnish them, while in two instances it was only under threat of legal proceedings that they could be obtained.

An abstract of the department’s receipts and disbursements from the 1st of July, 1880, to the 30th of June, 1881, will be found in appendix A, and further information is given in appendix A (1) and A (2).

In the following table is furnished information as to the proportional distribution of the funds placed at the disposal of the Department daring the last five years >


0

m

»

«

H


O

CO

00

CM ; CM * CO

; Ç- Ç5 : co

Ci IQ rt< ^ ip W ri IQ H

CM r—( rH

CO io * T*

1C to <?

T V 71

Ö

CO

CU r— io f—<

i> co io

ocoi>

CO •

. CO o

o ^ cuo o>

r-l O O

o

CO

Ò :

CO

: o co

CM rH rH

_•

C7> -4

■—1 O r— co h

cocco

CO

00

o *

. IO

t- O CO

O 00 <M

T* - r

00

rH *

CO

• U- CM

CU 1—1 r-l


CO

id

ri

co

TOSI-T8

O CD Ç5

i>*

O Ol CO CO 00 H Ç) O O OO

cu L * * *

: i ;

:

100-00

io co

lo

CO O CO CU ’—1

O

o

cp ^

rH IO

CO IQ H IQ cp

;

Ô

CO

L

U cu

oi rH

. • •

o

CO

rH

*

CO

io

CO »-H

l> (N CO o io

Ol CU

Ol CO V-

O

O

o

cp .

ip

CD IQ

» QO rH

CD rH

Ò

oo •

CD

1*0 CU

CU r-l

o

CD

r—<

rH

o

CD

CO

r-, Ol

o cu co r- io

^ LO CD

ou

r-

O

o

A

CO .

cp

cp cp

CM CM CM ^ cp

rH CQ IQ

CO

cp

rH

o

ô •

CO

¿> CM

o

s

CD

1-1

CO

HO

rH CO

CO CD IO CO

h- hH -rH

CD

o

CO

cu .

ip

dh Ol

OxfOiOO

o r- cu

CO

5

¿5 *

»o

CD CM

CI H I-H

o

CD

H

^ rH

CO

CU o

CO H io CO CO

cu o co

o c?

CO rH O CO o

* * *

*

:

r~.

»o

Th

ID rH

• • *

o

io

CO

rH

ri-

IO ^4 ^

CO -h

Ci CO CM T- ^

O

<D

CO

CO CM

r-< CO

co o —1 co Th

Ó

o

CO

IG rH

rH rH

O

CD

CU

rH


.5 £ ^ e* o Ö a a cs

*»H


CO

m Q


O


CQ cl) OQ CO

cs a

r—I O

I B*

C <»

<o

Vi bo

>..3


S >>

I-3

o 3

Ü

<d a

.

4» O

fe 03 ^ u

: « 5? a M


-H O > - .

s -*»


* o

' ö —

as

CO ci

04 **-'

' CO ^ 2 ö «) e3


8 ä

1 cr

<D


çS


p<

o    

rt t3 no J®

S » «a a a? e


! d

1 s

-*■> s .9

;

-0 S rd o -d ' d o d o x ! «OMW ®


>

: Ti ■< cn ^"0


o .2 JS

e    cd

O Cl, O .d

id » ® ii d

d H Q. U, C

oM “E-ip;


H3 >>

O <§ 01 iQ

o    ui    »    o    o

CO    ü    W    Ü    VJL

P    s    ®    d

«    d    c4    +=    2

Ü.S    §    &    d    *

id o o o

&-Ö ^ a -43 « * g g

g ’S ~ 9

CL et

sas


.2 '5


03 00

u' *3 t-< ^ I

£1, p (D O

S-s," ~


-3 W -S


CM CO    IO CD 1>cOCîO I


The cost for the instruction of each child in average attendance during the year 1880, calculated on the expenditure under items (1) and (4) of the foregoing table, was, in day schools, £3 19s. 7fd.; in night schools, £1 14s. lOd.; and in both combined, £3 18s. Od. Compared with the cost for the preceding year, these figures show an increase of 2s. 9d per scholar in day schools, and of Is. OJd. in night schools. Various causes have contributed to this increase. One of these is the largely augmented result percentage, which, though obtained in 1879, had nevertheless a partly prospective effect, regulating to a considerable extent the amount of results paid in 1880......

The following table shows the number and classification of teachers of all ranks employed in State schools on the 13st December, 1880 :—


Ì Certificated, including teachers classified in honors,..    ...

Licensed ...    ...    ...

Pending for license to teach    ...

Total Head Teachers    ...

{Certificated, including teachers classified in honors...    ...

Licensed ...    ...    ...

Pending for license to teach    ...

Total Assistants ..    ...

( Certificated    ...    ...

] Pending for certificate    ...

< Licensed to teach ...    ...

Pending for license to teach    ...

(Unclassified    ...    ...

Total Workmistresses    ..

{First class    ...    ...

Second class    ...    ..

Third class    ...    ...

Fourth class    ...    ...

Total Pupil Teachers    ...


Head


Assistants


Work

Mistresses


Pupil

Teachers


Males,

Females.

Total

597

45

674

250

19

39

1,624

•75

266

143

36S

19

28

899

1

1

15

5

558

580

61

182

78

146

88

215

103

232

1,105

1,857

2,351

4,208


Totals ...

The classification of the teachers employed is shown in the following table. Workmistresses and pupiLteachers are not included ,


Certificated,

including

those

Classified in Honors.

Licensed.

Partially

Classified.

Total.

Number employed on 30th June, 1880 ...

969

1,397

102

2,468

Number employed on 31st December, 18S0...

983

1,435

105

2,523

Increase during the six months ... ...

14

38

3

55

Decrease during the six months ... ...


It will be observed that, as already shown in a corresponding table in the report for 1879-80, no unclassified teachers are employed under the Department.

The teachers available for employment on the 31st December, 1880, comprised the following :—

Males. Females. Total.

(1.) Trainees who had completed their course ...    18    15    33

(2.) First-class pupil teachers qualified for promotion to assistantships or the charge of small schools ...    ...    ...    57    155    212

(3.) Other qualified candidates    ...    ... 39    164    203


Total    ...    ...    ...    448

It will be seen that, notwithstanding the numerous appointments made since the publication of the last report, the number of qualified teachers available for appointment increased from 375 on the 30th June to 448 on the 31st December, 1880. This increase was made up of successful candidates at the periodical examinations, and of first-class pupil-teachers who, having served twelve months after passing their final examination, had become eligible for promotion. As these teachers, together with those who leave the Training Institution every six months, fully suffice to meet the requirements of the Department, it has been decided still to discontinue the holding of examinations for a license to teach.

The reports of the inspectors, which will be found in Appendix H, are of a generally encouraging nature. Discipline continues to be well maintained, and improvement is shown in the organization of the schools. The state of instruction, as tested by the result examinations, compares favourably with that of the previous year, the percentage of passes having risen from 77-42 in 1879 to 77-76 in 1880.

The following tables enable a comparison to be made of the results obtained during the last three years in the several subjects of examination, and also by the several classes :—


I.—Percentage of Passes in the several Subjects of Examination.

. -

1878.

1879.

1880.

Reading I. ... ...

85-3

88 6

88-1

Reading 11. (comprehension of matter read) ... ..

63-6

610

59-4

Spelling .. ... ...

75 9

78-4

792

Writing ... ... ...

89'6

90-4

92-1

Arithmetic ... ...

724

76-4

77-1

Grammar... ... ...

60-4

64 2

647

Geography ... ...

751

76 4

766


II.—Percentage of Passes for each Class in the several Subjects of Examination.

Class II.

Class III.

Class IV.

Class V.

Class VI.

Class x VI

Reading I., 1878 ...

84 3

86-1

86-6

84-3

83-4

87-3

„ 1879 ...

88-3

90-3

89-0

86 0

87-8

90-9

„ 1880 ...

88-9

88-9

87'4

86-9

87-0

90-9

Reading II., 1878 ...

62-5

65-7

69 9

,, 1879 ...

59-6

63-3

73-5

,, 1880 . .

...

57-4

63-4

72-4

Spelling, 1878 ...

80-2

78-1

64-5

82-2

69-5

75 -6

„ 1879 ...

85-6

79 9

67 '6

84-1

65‘4

69-9

,, 1880 ...

85-7

81-0

68-8

85-5

66 4

73-8

Writing, 1878 ...

96-4

81-8

89 3

89-9

91 -6

94-0

,, 1879 ...

96-5

83-8

90-2

90-7

928

94 6

,, 1880 ...

96 9

85-4

93-1

92-5

937

97 2

Arithmetic, 1878 ..

72-3

78 9

67 0

67-5

76 '5

761

,, 1879 ...

75 0

87 0

69 7

69 J

79 4

76-9

„ 1880 ...

76'8

88 0

69 6

71-6

77-4

75 0

Grammar, 1878 ...

74-7

53-5

50-4

48 6

60’ 6

,, 1879 ...

76-2

60 5

54-3

55 0

63-8

,, 1880 ...

75'8

63 1

55-4

54-1

59-9

Geography, 1878 ...

...

78 8

72-3

73-3

73 2

79-8

„ 1879 .

80-5

736

72-9

78-1

79-4

„ 1880 ...

80-2

73-3

75 6

77'2

78-4


The number of schools in which instruction was given in extra subjects was 182, being a decrease of 16 as compared with the previous year The amount of fees received for such instruction has, however risen from £3600 12s, 7d. to £4083 4s. 10d., indicating a corresponding increase in the number of pupils taught. As evidence of the success attending the teaching of these subjects, it may be mentioned that iu 1880 32 pupils from State schools passed the University matriculation examination, while 59 passed the examination prescribed for the Civil Service. The corresponding numbers for 1879 were respectively 15 and 34.

Military drill was taught in 166 schools, to classes showing an average attendance of 10,685, being an increase of 47 on the number of children receiving instruction during the previous year.

Instruction in vocal music was given in 243 schools, being twelve more than in the previous year. In 194 of these, singing was taught wholly by visiting teachers; iu 25, by qualified members of the ordinary school staff; and in 24,by visiting teachers, assisted by qualified members of the school staff. The average attendance of children taught was 34,308, showing a slight decrease when compared with last year’s return, which amounted to 35,544. The visiting teachers now number 29, and it is satisfactory to note that the number of members of the ordinary school staff who give special instruction in singing has risen to 49.

Instruction in drawing was given by 14 visiting teachers and 45 qualified members of the ordinary teaching staff, the latter number showing an increase of 13. The visiting teachers, with assistance in some cases from qualified members of the staff, gave instruction in 112 schools, and the latter in 25 schools, making a total of 137 schools in which the subject was taught. The average attendance at the classes was 18,975, being a decrease of 432 children under instruction as compared with 1879.

Iu order to induce teachers and pupil teachers to pass the examinations in music, drawing, and military drill, a premium in the form of additional remuneration is offered to those employed in teaching these subjects.

The annual Exhibition examination open to State school pupils was held in December last, at which 49 candidates competed, and 11 exhibitions were awarded.

A difficulty having arisen as to the claims of certain candidates who were admitted to the examination, my predecessor deemed it advisable to increase the number of exhibitions from eight to eleven, and the regulation was altered accordingly. As this alteration met with general approval in Parliament, it is my intention to adhere to the increased number in future.

It is noticeable that, for the first time since the introduction of State school exhibitions, a girl was one of the succe sful candidates. She has since taken up her studies at the Corporate High School, Sandhurst. The successful competitors were presented from the following schools ;—


No.    1436.    Ballarat    (Mount Pleasant)...    ...    2

,,    112. Carlton ...    ...    ...    2

„    1252.    „    ........ ;;;    1

,,    1492.    Geelong...    ...    ..    _    j

,,    1278.    Melbourne    (La Trobc-street)    ...    1

,,    1406.    ,,    (Yarra Park) ...    ...    ]

,,    1566.    Sandhurst    ...    ...    ...    ]

,,    574,    Snake Valley    ...    ...    ...    1

,,    502.    Stawell ...    ...    ...


The number of exhibitioners at the present time is 45, distributed as under :—

Attending Lectures at the University, Melbourne 20


,,    Scotch College,    Melbourne    ...    ...    9

,, Wesley ,,    ,,    ...    ...    7

,,    Grenville ,,    Ballarat    ...    ...    5

,,    Corporate High School, Sandhurst    ...    2

,,    Geelong College ...    ...    ...    1

,,    St. Patrick’s College, Melbourne    ...    1

Total    ...    ...    45


Of those attending the University, 12 passed the examinations for which they were severally due, viz. .-—Eight in the Arts course ; two in Medicine ; and two in Engineering. One forfeited his exhibition through failure to pass the required examination at the end of his fourth year.

The reports furnished by the several principals on the exhibitioners attending their schools show that the conduct has been excellent and

the progress made highly satisfactory.....

The number of qualified applicants for admission to the two years’ course of training for the term which commenced in January was 108 and in July 96. Of the former, 34 were admitted to the associated training schools, and of the latter, 29. In January, 4, whose qualifications permitted of the first year’s course being dispensed with, were admitted to the Central IiainiDg Institution, and 7 w’erc similarly admitted in July.    3

The number of teachers in training during the second half of 1880 was 145. Of these, 78 were engaged in the associated schools, and 67 in the Central Institution. Of the latter, 2 were relieved from duty in November on account of illness, and 33 left the institution at the end of December, having completed their term of training.

I he qualifications held by the successful candidates for admission to the first year’s course of training, and the positions to which they were appointed in the associated schools, are shown in the following table:—    b


Teachers formerly employed in Schools.

First-class Pupil-teachers who had competed their Course.

Matriculated

Students.

Other persons who had passed the Entrance Examination.

Ma'ei.

%

a

0?

p _

Males.

ë’emales.

Males

F. males.

Males

Females.

cS

O

H

1

5

~ rc 2 £

4

Is

2'S,

Paid

Students.

3 -PP

■O

~ rS

*3

Pc(1

23 s

2 G

a o 11

cj ~ - pp

Unpaid ; Students. I

it

IS

3 A i? *■*

P CO

tn

.'S *p-PP

I Unpaid 1 Students.

2 A ’3 3* PP

I Unpaidj 1 Pupils.

Jan.

1880.

2

2

1

5

1

9

7

6

1

34

July

1880.

5

1

4

2

6

l

...

9

1

29

5

1

6

2

2

1

11

1

9

l

16

7

1

63


The ordinary course of training has often been criticised on the grc that whilst the students arc shown how to teach a single class creditably, they yet receive little practical training in the art of managing a small school, so as to keep all its classes simultaneously and profitably occupied. It is a satisfaction to remark that all grounds for this objection have now been removed by the establishment of tsvo small model practising schools, where students may learn how, unaided by a second teacher, to control and properly regulate the instruction of a small school under circumstances similar to those which will afterwards become their actual experience.

The following statement shows in a tabular form the statistical results of the general examinations held in December 1880 for teachers, pupil


Candidates for—

Total

Number

Examined.

Percentage passed in.

Passed.

Failed.

1878.

1879.

1880.

I.—(a.) Certificate of competency ...

119

484

603

33‘8

25'9

19-7

(b.) Admission to second year’s course of training ...

27

26

53

53-3

58-9

50'9

II. — (<i.) Admission to 1st year’s course of training... ...

23

15S

181

36'6

17'1

12 7

(5.) License to teach (for drawing and singing masters only) ... ...

9

9

III.—Pupil-teachers—

(a.) 1st class ...

117

106

223

64-0

43 4

52 4

(b.) 2nd class ...

180

144

324

55'7

38-3

55'5

(c\) 3rd class ...

106

153

259

59 '0

564

40-9


Buildings.

School-houses.

Residences.

Ö

Wood, &c.

O

Brick or Stone.

Wood, &c.

Part Brick or Stone.

'

Brick or Sto

Partly ofBi or Stone.

Total.

1

Attached to Schoolhouse.

Detached from School.

Attached to School.

Detached from School.

Attached to School.

Detached from School.

öj

O

B

Number provided under the Common Schools Act

175

127

10

312

19

16

12

81

4

1

133

Number provided under the Department

252

995

8

1255

41

9

738

164

2

3

957

Total number of buildings, 30th June, 1881

427

1122

18

1567

60

25

750

245

6

4

1090


The number of qualified candidates for employment has so largely increased that vacancies cannot be found for all ; and it may become desirable to restrict the certificate examination to pupil-teachers who have completed their course, and to other persons actually employed. Similarly, with respect to admission to training, it is found that matriculated students and first-class pupil-teachers are now more than sufficient to fill up the vacancies in the training schools ; and. as these two classes of can lidates are held to have better qualifications than those of other applicants, it is intended to fill all vacancies from them, and to discontinue henceforth the special examination for admission to training.

With regard to the examination for certificates of competency, the Board of Examiners report that barely one candidate in five has passed. This is six per cent, lower than last year’s proportion, and very much less than that recorded for previous years. The discontinuance of the “ Liccuse to Teach" examination may partly account for this fact; since many candidates who would probably have contented themselves with trying for the lower standard of the license to teach now address their efforts to the certificate examination; but, apart from this, it is evident that too many with meagre knowledge and insufficient preparation attempt to gain a diploma the possession of which must show, if not extensive attainments, at least sound and accurate knowledge.

It is also a matter of regret to find that, of the candidates for entrance upon the second year’s course of training, little more than fifty percent, have qualified themselves, being ten per cent, less than the average of the previous three years; and, considering the proportion of failures to be unduly large, 1 must express a hope that the care and exertions of the associates* in training may in future lead to more satisfactory results.

At the general examination, 806 pupil teachers were examined, the number being 969 in 1879. The diminution does not imply any falling off in the total of pupil teachers, but is simply the result of so many having completed their course, and attained to the first class. The result of the pupil teacher examination cannot be deemed quite satisfactory, though showing an improvement upon the work of 1879, in which year the results were unduly low. The average passes of the three classes, taken together, were 50 per cent., being 3£ per cent, above last year’s average, and less than 3 per cent, below the average of the previous three years. The first and second classes showed marked improvement upon the work of 1879 ; but the lowest class gave evidence of a considerable falling off, leading, perhaps, to the inference that sufficient care had not been taken by teachers to nominate candidates whose education and capacity fitted them for tlie position.

In six instances the services of pupil teachers were dispensed with on account of repeated failures at examination.

Examinations in music were held during the year in Melbourne, Geelong, Ballarat. Castlemaine, Sandhurst, Echuca, Stawell, Beech-worth, Sale, and Warrnambool, at which 315 candidates attended for examination in the subjects prescribed for a license to teach, and 5 for certificate of competency. Of these, 22 succeeded in passing fully for a license, and 18 will be entitled to be recorded as licensed on passing in the art of teaching ; but none of the candidates who presented themselves for the higher classification succeeded in passing.

Examinations in drawing have been held as usual in Melbourne, Geelong, Ballarat, Castlemaine, and Sandhurst. The total number of persons examined was 212, of whom 11 passed fully for a license to teach ; and 11 candidates for a license, and 1 for a certificate of competency, will be entitled to such classification ou their satisfying the examiner in the art of teaching. In addition to the foregoing candidates, 16 partly classified teachers completed their examination — 3 for a certificate, and 13 for a license to teach.

The usual examinations in military drill (theoretical and practical) were held during the year, and 73 candidates presented themselves for examination in the theoretical portion.

In the practical test, 19 candidates who had previously passed in the written work, satisfied the examiner as to their ability to impart instruction in military drill, becoming thereby entitled to the department’s certificate. In addition to those who have obtained classification under the department, 81 teachers are recorded as fully qualified through their having passed an examination prescribed by the Victorian Volunteer authorities corresponding to that held under the department.

In gymnastics two examinations were held, at which 125 candidates presented themselves, the result being that 36 passed. This makes a total of 163 teachers qualified to instruct iu gymnastics.

In August 1879 teachers were informed, by direction of my predecessor, that some elementary teaching of science should ultimately be given in State schools, and, in furtherance of this view, it was desired that every teacher should endeavour to qualify himself to instruct his pupils in at least one branch of science, to be selected at the option of the teacher from the following list, viz.Physiology, Botany, Geology, and Mineralogy; Magnetism and Electricity; Acoustics, Light, and Heat; Mechanics and Hydrostatics ; Chemistry, and Metallurgy. Examinations in these subjects were .accordingly held iu December last, at the different centres, when no fewer than 299 teachers presented themselves for examination, of whom 97 passed in one or more subjects.

The names of the successful teachers, with the subjects in which they passed, will be found under Appendix G 5.

Building operations during 1880 were not very extensive, on account of the limited funds placed by Parliament at the disposal of the Department.

A detailed statement of the work done is given in Appendix E (1), from which it will be seen that operations have been confined mainly to the urgent requirements of country districts, buildings in populous neighbourhoods having been provided in only a few eases of pressing need, where delay would have resulted in loss or serious inconvenience.

The same remark applies to the works undertaken from the end of 1S80 to the 30th June, 1881, which are set forth in Appendix E (2).

In Appendix D will be found a list of State-school buildings belonging to the Department, now 1567 in number, and providing for 151,732 children. Compared with last year’s returns, this shows an increase of 115 school buildings, with accommodation for 8120 children. Teachers’ residences are now provided for 1090 schools, showing an increase of 103 in the year.

A summary of Appendix D showing the character of the buildings and of the accommodation providedjn them, is given below

Accommodation.

Buildings of Brick or Stone.

° O §)°^ .2 v

Buildings partly Brick or Stone and partly Wood.

Total in Brick or Stone Buildings*

Total.

g 8 S’"

Brick or Stone Portion.

Wooden

Portion.

Total in Wooden Buildingst

Number of scholars ( provided, for under ) the Common Schools Act. (

17,031

8,863

1,085

669

*18,119

f9,532

27,651

124,081

151,732

Number of scholars ( provided for under < the Department (

73 245

47*133

2,821

882

*76,066

+48,015

Total Number of cho- ( lars provided for, <. 30th June, 1881 [

90,279

55,996

3,906

1 551

*94,1S5 +57,547


included in the foregoing statements are the following buildings, which have been opened since the date of the last report :—

1026.

Balwyn Road,

Carlton, Queensberry-st.,

to accommodate

100 children.

2365.

588 „

1403.

Dandenong,

«

200 „

1253.

Emerald Hill,

J)

1,080 ,,

2367.

Epsom

))

100 „

253.

North Footscray,

300 ,,

3 50S.

Hawthorn (Extension)

J)

188 ,,

2374.

Kensington,

5 )

197 ,,

759.

Kingston,

i)

122 ,,


Districts

in which Prosecutions were authorised by—

! Total X nnibor of Districts 1 in exis-: tenee dur* ; ing the j period.

Boards of Advice—

Department.

Unassisted

A ssisted by Truant Officer.

Total.

Quarter ended March 31

15

183

198

119

317

,, June 30

14

185

199

118

317

,, Sept. 30

12

187

199

118

317

i, Dec. 31

11

188

199

118

317


The maintenance of school buildings has been attended to somewhat more fully that in 1879-80, a sum of £15,000 having been made available, which is £5,000 more than the amount voted for the previous year ; but, as repairs had run considerably into arrear, it has not even yet become possible to overtake all urgent requirements.

The annual cost of maintenance cannot be estimated at less than £22,000 per annum ; but, as the sum of £20,000 has been placed on the estimates for 1881-82, there is a prospect of being able to deal with the most pressing requirements before much further injury occurs to the school buildings.

A sum of £50,000 to £55,000 was estimated as necessary to meet both the annual cost of maintaining the property of the department, and of providing for such new applications as would be caused by further settlement, together with the growth and shifting of population ; and there is no reason to think that this estimate is in excess of what will be needed for years to come. For the past two years, provision has been made for maintenance; . but no vote has yet been taken to meet the yearly claims for the establishment of new schools. .    .    .

A great number of schools are still held in leased premises, and the erection of State buildings for them, in addition to some urgently needed buddings and other necessary works, would involve the further outlay of £270,000. The change may be made a gradual one : but it is desirable that it should be effected as soon as practicable.

It was thought advisable that the educational system of Victoria should be represented at the late International Exhibition, and for this purpose two of the ordinary country school buildings were erected in the Exhibition grounds. One of these was designed for an attendance of 20 to 30 children, and the other for fully double that number. Both were provided with the usual quarters for teachers ; and the school-rooms were fitted up with furniture and apparatus of the regulation pattern, including everything needed for the ordinary working of a school.

Advantage was taken of the space thus afforded to exhibit the work of State school pupils from various parts of the colony, and also to give, by means of photographs, plans, pictures, and a model of one of the proposed city schools, an ample illustration of the dimensions, arrangements, and style of architecture adopted by the department for its school buildings.

There can be no doubt that this exhibit was highly appreciated ; and it is a gratifying fact that this young colony obtained in the Exhibition awards an honourable place, when in competition with the rest of the world.

The number of school districts in the colony at the beginning of the year was 317. No new ones have been formed ; though, in some cases, alterations were made in the boundaries of existing districts. At the commencement of 1880 only 310 of these districts possessed Boards of Advice, but during the year two more Boards of Advice came into existence. . .    .

The vacancies occurring during the year on the Boards of Advice amounted to 363, and were occasioned as follows :—261 members retired by effluxion of time, 83 resigned their seats, 9 died, 9 forfeited their seats through non-attendance, and 1 was removed by the Governor-inCouncil.

To fill these vacancies, 256 elections were held, which resulted in the return of 276 members ; but in 56 instances no candidates were nominated, and the necessary appointments were therefore made by the Governor-in-Council, as was also the case with 12 other vacancies in districts where there were no ratepayers.

The following statement shows the arrangements made during the year for the purpose of carrying out the compulsory clause of the Education Act;—

During the year proceedings were instituted by the local boards in 3,268 cases, with the following results :—2,790 convictions were obtained, 231 cases were dismissed, and the remainder withdrawn. The department authorised prosecutions in 1,458 cases, 136 of which were not proceeded with (owiug to the lateness in the quarter when the instructions were issued), 95 were dismissed, and in 1,227 cases convictions were obtained.

During 1879 special instructions were issued to truant officers, directing them to investigate as many cases as possible of children found in school hours idling about the streets; and the beneficial result of this watchfulness became apparent when, out of 294 prosecutions instituted in consequence, 254 convictions followed. In some of these cases the children were found to be in criminal companionship, and this fact was brought under the notice of the Police Department, with the view of sending them to a Reformatory.

The total number of convictions during 1880 was 4271, being 596 less than those of the previous year ; and in 100 cases the maximum penalty of £1 was inflicted.

The number of Boards of Advice which have voluntarily enforced the compulsory clause is more by seven than it was last year ; and this at least shows that their interest is not waning ; though, as already stated, the total number of individuals prosecuted is far less than in 1879.

The increased facilities afforded to all denominations for holding public worship on Sundays in State school buildings have been largely availed of during the year. In 1879 only 39 distinct congregations assembled in State school-rooms for religious worship, whereas the use of 355 schools was granted for the purpose in 1880. In 30 schools, religious services have been conducted on week day evenings, and 167 school-rooms have been used for Sunday schools.

The opportunities afforded for imparting religious instruction to the children on week days after school hours do not, however, seem to be appreciated to the extent anticipated, inasmuch as only-in 27 schools is religious instruction given at the close of the secular lessons. But, although this is the ease, it would not be correct to assume that only a small proportion of children attending Stale schools receive religious instruction. No returns on a large scale have been collected ; but one of the inspectors has made some investigation into the matter, the result being that, of 5,057 children concerning whom enquiry was made, no fewer than 79 per cent, were found to be iu actual attendance at Sunday schools, to say nothing of any who might be receiving religious instruction at home. The number quoted (5 057) includes every child found on the rolls of 26 schools in localities of widely differing character, the schools being in urban, suburban, mining, and farming neighbourhoods.

The report of the board appointed by my predecessor to draw up a scheme for the classification of teachers lias not escaped my notice ; but the magnitude of the question and the interests involved have prevented me from giving to the subject that full consideration which its importance demands.

HEAD TEACHER, country school, a lotment 30 x 50, Western district, desires exchange, G-ippsland or Ballarat preferred. Address—“J.S.,” Schoolmaster Office.

HEAD TEACHER, 20 x 30, results 88*235, wishes to exchange with assistant, Melbourne or suburbs. Lower income no objection. Address —“M.B.,” Natimuk.

HEAD TEACHER, allotm'nt 30 x 50, rising township, ngrieullur 1 district, workmistress position vacant, desires exchange witu another in suitable locality. Address—“Doceo,” Schoolmaster Office.

WANTED to exchange, 50 x 75 school, results 84, vacancy for work-mistress and pupil teacher. Lower allotment accepted if requirements suitable. Address—“V.G.,” YVangara ta.

ASSISTANT, Melbourne, wishes exchange with H.T., country school after Christinas ; one within thirty miles of Melbourne. Ballarat or Geelong preferred. Address—“ Scholastic,” G.P.O., Melbourne.

rpHE SCHOOLMISTRESS WALTZ, by No. 1,420. .Melbourne- Messrs JL G en, Allan, and Nicholson. All music-sellers, Geelong and Ballarat.

I WARREN BALL’S “Hints to Candidates for Teachers’ and Matricur » lation Examinations,” la.; posted, Is. Id. Mullen, Melbou na,

CANDIDATES for EXAMINATIONS prepared by correspondence or otherwise. I. Warren Ball, South Yarra.

TEACHER of a 30 X 50 school, in the Wi stern District, would exchange for 20 to 30 in the neighbourhood of Kynefcon, Macedon or Kilmore. Address “H. B.,” P. O., Macarthur,


c


^lex. MLINLEY & CO.,


MULLEN’S MATRICULATION MANUALS.


S'


ONE SHILLING.


T


M


rpATE’S PARCELS POST EXPRESS

FIXED PRICE.

NO EXTRAS. NO TROUBLE.

Delivery to door at any address in

21b

s. d.

41b

s.

61b d. s.

d-

101b. s. d.

201b. s d.

Great Britain

4

6

6

0 7

6

10

0

14

0

Continent of Europe,

America, & Canada

7

6

9

612

0

16

0

21

0

Sydney, II obar t,

Launceston

2

6

3

0. 3

6

4

0

6

0

New Zealand Ports

(except West Coast)

Adelaide, Brisbane.

4

0

4

(> 5

0

5

6

7

6

No further charge whatever.

Very small

increase

for heavier weights. Delivery at country addresses in Australia, inland carriage only added.

Receiving office—

FREDERICK TATE, 13 Market-st., Melbourn e 1' A T E ’S PA K CEL S P 0 S T E X P R E S S'.

DELIVERY at DOOR any address in the world.

EITHER to or from Britain from 4s. Gd,

TO or from other Australian ports from 2s. 6d. NO further C U ARGE whatever. No trouble.

Any SIZE, weight, or shape.

EVERY possible ASSISTANCE afforded. INQUIRIES plainly answered.

Henceforth Parcels handed to VV. R. SUTTON’S Branches, every town in Great Britain, delivered in Melbourne at nearly similar rates.

Head Office

FREDERICK TATE, CUSTOMS AGENT, &c., 13 Market street, Melbourne.

N O M P E T EN C Y    EX A MINATION.

T U I T I 0 N—

IN CLASS, BY CORRESPONDENCE, OR PRIVATELY.

Other Work, by Arrangement.

JAMES L. ROBERTSON, B.A., 71 Clarendon-st., Emerald Hill.

N

GENERAL

PRINTERS AND PUBLISHERS, 61 Queen-street, Melbourne.

Tn Bookwork and General Publishing our large experience is a guarantee of all work being executed in the best style, while having a first-class stock of the best and latost material ensures expedition and good workmanship.

The following papers are issued from this office :— WEEKLY.

“Punch,”    “Bulletin,”

“ Willing Work,”    “ Once a Week.”

FORTNIGHTLY.

“Australian Law Times,”    “Jewish Herald,”

MONTHLY.

“ Schoolmaster.”    “ Monthly Messenger.”

BUY AND READ

THE NEW WEEKLY PAPER. Price Threepence.

“ONCE A WEE

10 l’AGES. PRICE THREEPENCE. ALL BOOKSELLERS.

Alex. M'Kinley and Co., Publishers, 61 Queen Street, Melbourne.

POPE’S ESSAY ON MAN, Epistles 1 to

3; and TEN ESSAYS of ADDISON, from the “Spectator.” In One Volume, Crown 8vo, Limp. Is.

POPE’S ESSAY ON MAN, Epistles 1 to 3 ; and TEN ESSAYS of ADDISON, from the “Spectator.” Prepared with Notes for the use cf Candidates for the Matriculation Examinations at the Melbourne University. By E. E. Morris, M.A. In One Volume, Crown 8vo, Cloth. 3'’.

VIRGIL’S AENEID, Book XII., and CiESAR’S GALLIC WAR, Book VI. Prepared with Notes for the use of Candidates for the Matriculation Examinations at the Melbourne University. By James Clezy, M.A. Second Edition, Revised. Fcap. 8vo, Limp. 2s.

VIRGIL'S AENEID, Book XII., and CiESAR’S GALLIC WAR, Book VI. Translated by James Clezy, M.A. Fcap. 8vo, Sewed. 2s. 6d.

FIRST^ BOOK OF AUSTRALIAN BOTANY. Specially designed for the Use of Schools. By W. R. Guilfoyle, F.L.S., C.M., R.B.S., London; Director of the Melbourne Botanical Gardens. Illustrated, Crown 8vo, Sewed. 2s. 6d.

MULLEN’S

New Classified Catalogue of School, College, and Technical

EDUCATIONAL WORKS

May be bad gratis on application, or posted on receipt of address.

SAMUEL MULLEN,

Wholesale Sc Retail Bookseller & Stationer, 29 & 31 COLLINS ST. E., MELBOURNE.

Price One Shilling, By Post, Is. 6d.

JQEPAET MENTAL EXAMINATION OF TEACHERS.

TEACHER S’ GUIDE

AUSTRALASIAN EXAMINATIONS.

Containing the

PROGRAMMES & EXAMINATION PAPERS of

Victoria    South Australia

New South Wales    Auckland, N.Z.

Queensland    Wellington, N.Z,

And Tasmania.

108 PAGES WITH STIFF COYER.

The above book contains the Programmes and Examination Papers of December, 1877, of all the colonies, and is reduced to the low price of

BY POST, Is. 6d.

ALEX. M'KINLEY & CO.,

PRINTERS AND PUBLISHERS, 61 QUEEN STREET,

MELBOURNE,

C. EXAMINATION.

TUITION BY CORRESPONDENCE.

MR. THOMAS BOARDMAN, First-class Honor-man of the Denominational School Board, Prepares Teachers for the Certificate Examination by Correspondence. Terms moderate.

Address—

45 PRINCES STREET, CARLTON

TAMES CLEZY, M.A.,

MELBOURNE.

CLASSICAL & SHAKSPEARE SCHOLAR (1869)

PREPARES CANDIDATES For Matriculation (pass or honours), and for the subsequent Degree Examinations of the University.

Course of Lessons by Correspondence in Latin Grammar, Translation, and Composition. Terms on application personally or by letter. 6 GORE STREET, FITZROY.

O HEAD MASTERS, SECRETARIES OF BOARDS OF ADVICE, AND OTHERS.

ALEX. M'KINLEY & CO.,

Having made considerable additions to their stock of Bookwork and Jobbing Type, are prepared to execute orders in every description of

GENERAL PRINTING.

All orders entrusted to them will be printed in a satisfactory manner.

Alex. M'Kinley & Co., Printers, 61 Qoeen-st.

NOW READY.

ILTON PARSED.

Price 2s.

By J. J. BURSTON,

(Author of “State School Arithmetic”).

Also Ready, the Fourth Edition of the

STATE SCHOOL ARITHMETIC. By

JOHN J. BURSTON,

North Sandhurst State School.

Printed and Published by Alex. M'Kinley & Cp., 61 Queenstreet, Melbourne, under the auspices of the Victorian Teachers Union.

AND LITERARY REVIEW.

Yol. Ill, No. 29.


NOVEMBER, 1881.


Subscription

Yearly, Cs. Od. ; Half-yearly, Ss. Od


BLACK! E

Comprehensive

& SON’S

School Series.

9.    Sentences.—Two lines on each

Page.

10.    Plain and Ornamental Let

tering.

11.    Exercise Book.—Wide Ruling

with Margin.

11£. Home Exercise Book,—Same as No. 11, but 8vo size. Price Id.

12.    Exercise Book. — Ruled in

Squares.

12|-. Home Exercise Book.—Same Ruling as No. 12, but 8vo size Price Id.

13.    Exercise Book.—Ruled for

Book-keeping.

14.    Essay Book.—Ruled for Com

position, &c.

15.    Exercise Book for Begin

ners. Ruled for Small Text. X. Copy-Book Protector and Blotter. Keeping the Books Clean. One Penny.


Each Book is Illustrated in a highly instructive and artistic manner.

Primer I.—32 pp.,..................paper cover, 1 Jcl. ; cloth cover, 2$d.


“ As specimens o£ good reading books, they cannot be excelled. The exercises are carefully arranged so as to suit the meanest capacity, and at the same time hate a tendency to make useful impressions on the minds of young scholars.”—Educational Guide.


Adopted by the London, Liverpool, Manchester and other School Boards, and by the National Board of Education in Ireland.

VERE FOSTER’S WRITING COPY-BOOKS

The efforts of Publishers to provide teachers with the means of properly training their pupils in the Art of Writing deserve the highest commendation ; and no names stand higher in this depar tment of scholastic work than Vere Foster raid the Messrs. Blackie."—Educational News.

Opinions of H.M. Inspectors in Education Blue Book Reports.

Mr. M‘Callum, | ll.il. Inspector,

says1

j More progress is made by Vere Foster’s than by any other ^ method which has come under my notice.”

Mr. Brewer, | H.M. Inspector, says— 1

1 “With hooks like Vere Foster’s there is no excuse for the l slovenly no-style of writing too often presented to me.”

Mr. FTarburton, l H.M. Inspector, says— 1

i “ I wish that the use of the excellent copy-books such as Vere 1 Foster’s was begun earlier and more persisted in.”

Mr. Newell, i II. M. Inspector, says(

I know of no series by means of which children can be so quickly taught to write with freedom and legibility.”

Rev. J. Lomax, ( H.M. Inspector, says(

The introduction of Vere Foster’s copy-books in some of my schools has been attended with marked success.”

Superior Edition, 2d. each number. Popular Edition, Id, each number. Contents of the Numbers-

1.    Strokes, Easy Letters, Short

Words.

1£. Long Letters, Short Words, Figures.

2.    Long Letters, Short Words,

Figures.

2£. Words of Four, Five, or Six Letters.

3.    Capitals, Short Words, Fig

ures.

3£. Sentences of Short Words.

4.    Sentences. Mostly composed of

Short Words.

4£. Select Quotations from Shakespeare.

6.    6. Sentences-—Maxims, Morals,

and Precepts.

5£. Sentences, in Writing of Three Sizes.

6£. Sentences, in writing of Two Sizes.

7.    Sentences, and Christian

Names.

8.    Sentences.—Ono Line on oach

Page.    .

THE COMPREHENSIVE READERS.

Primer II.—48 pp.,.................. ,,    2d.;    „    3d.

Primer Complete—80 pp.,............................. „    4<1.

Reader I.04 pp., (Abridged Edition,) ............. „    4d.

Reader. I.— 96 pp.,....................................... cloth boards, 0d.

Reader II.—128 pp.,...................................... „    8d.

Reader III.—196 pp.,.................................... „    Is.

Reader IV.—288 pp., .................................... „    Is, Gd.

Reader V.—320 pp.,....................................... ,,    2s.

Reader VI.—384 pp.,...................................... „    2s. Gd.

ADDITIONAL READING BOOKS.

The Newspaper Reader, Selections from the Leading Journals of the Nineteenth Century on events of the day. By Henry Findlater Bussey, and T. Wilson Reid. Foolscap 8vo, cloth boards, 288 pp, price 2s.

The British Biographical Reader, Sketches of prominent Men by Eminent Authors. With Introductory Notes by tho Editor. Illustrated by numerous authentic Portraits. Foolscap 8vo, cloth board-288 pp., price 2s. .

The Shakespeare Reader, being Extracts from tlvc Plays of Shakespeare, specially selected as fulfilling Article 28 and Schedule IV. of the Education Code. With Introductory Paragraphs and Notes Grammatical, Historical and Explanatory. Foolscap 8vo, 160 pp, cloth, Is.

Myths and Legends of Ancient Greece and Rome. A Hand book of Greek and Roman Mythology, for Schools and private Students. By E. M. Berens. illustrated from Antique Sculptures, Foolscap 8vo, 330 pp., cloth 3s.

Poetical Reader, for the use of Elementary Schools in England and Scotland. Foolscap 8vo, 224 pp., cloth Is. Gd.

London, Past and Present. A Reading-book for Elementary Schools. With numerous authentic Illustrations. Foolscap 8vo, 288 pp., cloth 2s.

The Scottish Historical Reader; being Extracts from Eminent Writers Descriptive of Events in Scottish History, with Introductory notes. This volume will supply a fuller account of leading events in Scottish history than can be embodied usually in School histories, and being composed chiefly of extracts from the works of approved writers will form excellent reading lessons. It is adapted more especially for tho requirements of the Fourth Standard.

HOME LESSON BOOKS.

The object of this series is to place in the hands of tho Pupil a certain amount ot Standard work, which will enable him to grasp the leading facts of English History, Geography, and Grammar, and at the same time lighten tho work of the Teacher. The questions in Arithmetic are mixed in order to test the pupil’s knowledge.

MELBOURNE.


Further Particulars and Catalogues from All Booksellers,

Or their Representative, GEO. STILLIE, care of lessrs. COWAN & Co.,

WHOLESALE STATIONERS,


Sejms! geprfmeni


INSPECTORS’ REPORT FOR THE YEAR 1880.

The following passages extracted from the Eeports furnished to the Education Department by the Inspectors will not be without interest :— W. M. M. Campbell, Esq., M.A.

With few exceptions, the teachers discharge their duty very fairly, so far as regards the keeping of the buildings under their charge clean and tidy.    _

I would again call attention to the insufficiency of the information contained in the maps of the smaller size supplied by the Department, I would also urge the desirability of having a timepiece in every school. If the Department cannot undertake to supply such, I should think Boards of Advice would have little difficulty, should they feel inclined to take the matter up, in raising subscriptions to supply the schools in their districts with clocks. Some of the not unfrequently too large amount of money spent in prizes for the scholars might be profitably devoted to this object. A notation frame in every school would be an advantage.

With regard to the various-subjects of the free course of instruction,

I observe :—

Rhymes.— Allowing children to hurry over words and jumble them up together into a meaningless jargon is the most glaring fault noticed. Of course, in such instances, the children can have no idea of the sense of the passage they repeat. With a little trouble the meaning of almost all that occurs in the rhymes in the second Royal Reader might be made comprehensible to very young children, though certainly this could scarcely be hoped for with regard to the rhymes in the Irish National series, including such pieces as Wordsworth’s “Ode to a Butterfly,” and such passages as—

“ Much converse do I find in thee,

Historian of my infancy,” &c., &c.

Reading.—There are few actual failures in this subject at examinations, but at the same time really good reading is very rare. The faults most commonly noticed are those already frequently referred to by inspectors, As regards the comprehension of what is read, I observe that n addition to the explanation and questioning not being sufficiently e xhaustive (the questions asked by teachers being frequently merely a test of memory), sufficient care is not taken to insist on the scholar giving clear and grammatically expressed answers to the questions asked. To teach a child to do this is an important part of mental training, and would prevent such ridiculous answers as the followipg, which are merely examples of what I have heard:—Question: “What is meant by spectators ?” Answer : “To see it,” instead of “Those who see.” Question: “ What is meant by clamorous ?” Answer: “ A great noise,” instead of “ Noisy.” Further, the teacher is often not sufficiently careful in his explanations to bring out the exact meaning of the words. It is very unsatisfactory to hear a teacher tell his class that “garrison” means “soldiers,” without any attempt to explain that the term is only applied to soldiers under certain circumstances ; yet such imperfect explanations are not uncommon.

Arithmetic.—I can only repeat the opinion already expressed by so many inspectors that in the teaching of this subject mechanical accuracy is too exclusively aimed at. At result examinations the failures occur chiefly in notation and numeration in Class IV., and in reduction in Class V. It is very common to see 50 per cent, of the scholars fail in these, while the average number of failures in addition, subtraction, multiplication, and division (simple and compound) is, I think, under 25 per cent. Counting on the fingers is too much encouraged, and in some schools even fifth class scholars are seen resorting to this practice. It is also too common to see children seated close together working the same sums ; under such circumstances the temptation to copy is to some children irresistible, and their time is consequently worse than lost.

Grammar is generally taught with fair success as far as simple parsing is concerned. In some cases teachers err in selecting too difficult passages, or in setting children to parse with full syntax before they are at home in the parts of speech. I still think that too much, as regards grammar, is expected of Class VI., and that the structure of words should be omitted from the programme of work for that class. In connection with this subject I cannot help remarking that a good many teachers (especially the younger ones) are not sufficiently careful to be grammatical in their language. One would scarcely expect to hear a licensed teacher use such expressions as “It ain’t, “This here book,” “ He done it wellyet I have actually heard them made use of, aud in school too.

Thomas R. Hepburn, Esq., B.A., LL.B.

In the smaller schools, I have frequently to complain of the systematic neglect of the first and second classes. The teacher bestows very little of his time on these classes, leaving them to the care of monitors solely. Were it not for the assistance that the children of these classes receive from their elder brothers and sisters, this neglect would be more noticeable. I have found, on making enquiries, that the backward children in these junior classes have invariably no brothers and sisters older than themselves.

Little or no improvement has taken place in the teaching of reading. Explanation of the reading lessons of the day is the exception, not the rule. Fair progress, however, has been made in arithmetic, grammar, geography, and dictation. The writing, in many schools, admits of much improvement, and the progress made during the year is hardly satisfactory. Although, during my visits to the various schools, laying great stress on the necessity of teaching this subject well, still there is ao subject, in examining country schools, that I am more inclined to


deal leniently with. Perhaps this may account for the poor progress made.

Samuel Ware, Esq., B.A.

As a rule, I found the children properly arranged in classes, and kept constantly at work, in accordance with time-tables, which, though not satisfactory in all points, usually suited the teachers’ method of working.

The principal faults I found in the reading were indistinct utterance and mistakes in easy words, caused by haste and carelessness.

In dictation lessons, the correction of errors is often carelessly performed, the teachers overlooking the fact that this is the most important part of the lesson.

The arithmetic lessons usually consisted of a dry working out of rules, few questions requiring any exercise of the intelligence being given. In a few schools, however, I was pleased to find that the questions were sometimes presented in the form of problems, even in the second and third classes.

In many cases I found the time-sheets filled in with the hours for opening and closing the school, instead of the exact time at which the teacher entered for school duty and that at which he left.

Another fault, which I found very frequently, was that the ages in the rolls did not agree with those in the school register. This did not appear to be the result of design, but of carelessness in transferring the ages from one set of roll-sheets to another.

Frederick John Gladman, Esq., B.Sc., B.A.

During the year I have paid my two customary visits to the associated schools, and have spent about half a day in each school on both occasions. My duty to the Central Institution will not allow me to give more time to these visits.

My report on the teaching power and promise shown by each trainee has already been sent in. The general character of the first-year trainees seems much as usual

As I propose to call attention to the weaknesses that have forced themselves on my notice, and to offer such suggestions as commend themselves to my judgment, my report may give a general impression which I do not desire it to convey. I would, therefore, say emphatically that, on the whole, the country has reason to be satisfied with its teachers and proud of its schools, and I would bear willing testimony to the generally faithful and painstaking work of the associates.

Every associated school has a character of its own, and whilst some are certainly better than others, they are as a rule suited for their special work, and most of them are doing it as well as can be reasonably expected.

The difficulties connected with the position of the associates, the necessity they are under of putting their school in the first place, and the consequent relegation of pure training work to a subordinate position, have been commented on in former reports.

If we regard the associated schools solely as branches of the Training Institution, it must be noted that, in places, positive training in teaching does not receive the attention it deserves. Allowing that the function of a teacher is to “get the pupil to learn,” and that every teacher ultimately makes up a method of his own for gaining this end, yet it is most desirable, in the trainee stage, that good methods should be presented, analysed, and taught. General directions are not enough, The thoughtful and experienced teacher knows that there is no cut-and-dried method which every one may adopt in detail, that there must be a frequent reference to first principles, and that method is to be tested by its results. But to deal too much in generalized advice, without sufficiently helping the young teacher to translate it into practice, is like telling a child to “ road as he would talk” before he has acquired the power of doing so. Such directions have their value at the proper stage, but for a learner they are “too much at large;” they want breaking down and adapting to individual needs.

I am led to make these remarks, because in one school the associate treats his young pqpple too much as finished assistants, rather than as persons who want training. In other places there is a want of fidelity to a school-pattern ; the trainees seem to be groping their own way into the light, so that their teaching is crude and irregular. I am glad to say, however, that this is not the rule. Commonly there is abundant evidence that the associate has succeeded in impressing his individuality on his trainees so far as teaching is concerned. My belief is that every training school should show this in reasonable degree, and that it will do so when telling, watching, and trying are judiciously combined ‘ Experienced men like our associates can well elaborate and define thmethods they approve of ; they can teach them, and cause their trainees to understand them by lectures, demonstrations, and criticism lessons, and they should then insist on their young teachers conforming to them.’ The greatest practical difficulty is that this work requires time, and that whilst some of it could be done out of school hours, parts can only be tried in school, and at a time when the associate probably has other urgent claims pressing upon him.

Some trainees do not get enough practice in different parts of the school. Here again I am speaking solely in the interest of the trainees. Sometimes I find one who has never taught in any class above the third,' or another who has been kept almost entirely with the infants, and occasionally, though rarely, one who has not spent a fair amount of time in the lower classes. Where trainees are weak or inexperienced the school might suffer seriously from putting advanced classes under their care. But as a means of training, and of fitting the young teacher for his future work, he ought to know something, of every class in the school. He might match a good teacher in an upper class, and be employed himself more as a monitor than as a class-teacher for a time. With good supervision, and a careful graduation of demand to his ability, increasing the size of the class and gradually adding subjects of greater difficulty as his strength increases, and never over-weighting him,

even a weak teacher would gain confidence and real power. But this plan can be followed oily where there is a sufficient staff without counting the weak trainees. Where they are counted as assistants and allotted accordingly, it seems impossible to deal with them as I have here suggested.

If our associated schools are to do their work thoroughly, they should be fully and even exceptionally well-officered. Good models should be available in all parts of the school, and the associate should be able to place the trainee where he may receive the greatest benefit. So far as my experience has gone, there has been too large a percentage of trainee-assistants, who have not been sources of strength to their schools, and who have not obtained the benefit they might otherwise have received from their training. This matter deserves careful and sympathetic consideration for another reason. Associates look on it as unfair to have trainees sent to them as assistants ; they think it is too much to ask them to manage their important schools with weaker helpers than ordinary. One or two have suffered considerably in this way, and feel it keenly, and I am bound to say my sympathies go with them.

The essential weaknesses of our training system are :—

(1.) Want of sufficient care i?i selecting candidates.—The examination for admission should be of a higher and more searching character. “ Persons who have passed the literary examination for a license to teach” (Reg. xiii., 9), but who have done no more than is implied in this, either as students or teachers, do sometimes develop into useful servants of the Department. If, too, the Department have need of extra teachers, it can fall back on candidates of this kind. But first-class pupil-teachers are available, or might easily be made so. These are the best subjects for further training, and it seems likely that there will be a sufficient number of them to supply our requirements. If others be admitted, it is only reasonable that they should pass an admission examination equivalent to that passed by a first-class pupil-teacher, except in class-teaching.

(2.) Teaching shill is not made sufficiently prominent.—Trainees get into the first year, sometimes without any experience as teachers ; we have some of this kind now. Then their admission to the second year does not depend at all on the teaching power and promise they show in the associated school (Reg. xiii. 13.) It is only after they come into the Central Institution that teaching skill becomes essential to success. This could be met by admitting outside candidates on probation only, until they showed fair teaching promise, as first-class pupil-teachers have already done. No one should be admitted to the examination for entrance to the second year until he or she have shown fair skill in class management and class teaching.

(3.) Our training syllabus is not graduated and consecutive.—In framing a course, the second year’s training should be kept in view from the first,and all the earlier stages should be so constructed as to lead up to it. This implies certain changes in the pupil-teacher course, and a complete recasting of the work of the first year.

The alternative of reducing the second year’s work so as to make it fit in with the present lower stages may be mentioned. One of the most experienced and most deservedly ¡respected of our associates thinks we should look in this direction. I think not. It would be a step backward, and I am convinced that a re-arrangement of the syllabus on the opposite principle would be practicable and far preferable.

( To be continued.)


EDUCATIONAL INSTITUTE OF SCOTLAND.

The annual meeting was held in the theatre of the High School, and, as usual, there was a large gathering. Long before the appointed hour of meeting the delegates and others began to assemble in the High School yard to renew the thread of old associations. The scene was a lively one, as usual, and there was much hearty recognition of friends and “ auld acquaintances.” The exhibition of educational books and appliances attracted a large number of visitors, and was, on this occasion, very varied in its contents. The meeting of Fellows was very fully attended, but the business of election was quickly finished. The chair at the ordinary meeting of the Institute was taken at eleven o’clock by Mr. Hutchison, rector of the Stirling High School, president for the year. The vast building was completely filled, and the interest of the large audience was well sustained throughout the day. After the chair had been taken there was a discussion on the order of business, Mr. Niel suggesting that the usual interval should take place after the election of the general committee of management instead of before. The debate threatened at one time to be rather lengthy, and a delegate very aptly asked whether it was worth while to waste an hour in discussing the subject of half an hour’s interval. The delegates decided by eighty-seven to seventy-nine that the order of business should remain unaltered. The President then proceeded to deliver his address, which was rather shorter than usual, and was moderate in tone on every question discussed. He was able to congratulate the Institute on an increase of members to the extent of about 100, which he considered satisfactory under the circumstances, He hoped, however, that those who now stood aside would soon see their way to join their brethren in strengthening the walls of the Institute. On the subject of the election of Fellows, he pointed out that the smaller number elected of late years had a sensible effect upon the funds, but would add greatly to the value of these degrees as an educational distinction. On the subject of tenure of office, in connection with Sir H. Maxwell’s Bill, the President’s remarks were moderate and sensible, in harmony with the opinion of the majority of teachers at the present time. He touched also on the subject of Endowed Schools, University Reform, entrance examinations, the benefits of an academical training for teachers, the chairs of education, and the status of the


teacher (which he believes to be in the hands of the teachers themselves). The following is the text of the address in full :—

PRESIDENT’S ADDRESS.

Ladies and Gentlemen.—The President’s retiiiug address is scarcely now- expected to be what it. once was—a lengthened resume of all the educational events of the year, or an elaborate exposition of educaticnal principles. The increased space and attention given to educational news, both general and parochial, in the public papers, consequent on the greater interest which the public has in the administration of education, as well as the full reports of School Boards, local associations, and committees of the Institute, in our professional newspapers, deprive the former proceeding of much of its wonted freshness and interest; while the discussion of theories and methods now finds for itself a fitter arena at the meetings of the Annual Congress. The hurry arising from the manifold and increasing business of this meeting makes it a not altogether suitable time for expounding new educational views or discussing old ones. The ordinary and legitimate business is ample enough to occupy the greater part of your time and energies. The President, moreover, on this occasion is not formulating the opinions of the Institute; he speaks for himself alone, and bears the sole responsibility for what he says ; and I am not vain enough to imagine that my personal views on the educational questions of the day are of sufficient importance to warrant me in inflicting them upon you at length. Recognising all these facts, it is my intention to contract my remarks within the narrowest limits which I deem consistent with the feeling of profound respect which I entertain for this great meeting of professional brethren.

In the outset, I am bound to record, in a more formal manner than has yet been done, my grateful sense of your kindness in raising me to the dignity of the chair. I confess that until that honour was proposed for me my ambition had not ventured to soar so high. But in view of the Congress which was to be held in Stirling, and in the hope that I might be able to be of some use in connection therewith, I accepted—not without some feeling of alarm at my own rashness—au honour which I had done little to deserve, and a responsibility which I was afraid might bo beyond my ability to sustain. If my term of office has not been marked by any open fiasco, it is due to the efficient work of the more permanent officials, the ready aid of the vice-presidents, and the attention to business of your committees. To all of these 1 am under the deepest obligation for their kind forbearance and support, and I now tender them my most grateful acknowledgments.

GAPS IN THE PROFESSION.

In such a large body as the Institute now happily is, it is impossible for a year to pass without making many gaps in the ranks. A number every year leave the profession for other modes of life, which they find, or fancy they will find, more lucrative or less laborious. A proportion of our lady members is alw’ays undergoing a process of translation from the schoolroom to another not less legitimate and honourable sphere of woman's work. To them we may offer our congratulations, and heartily wish them every happiness. Others, however, whom we were wont to see here have ended their work on earth. I have not the means of ascertaining the number of those who have thus fallen by the way in the course of the year, or of worthily commemorating their merits. But one name at once occurs to me, the more readily as it is that of one who was a valued member of the committee of management. 1 refer to the late Mr. Robert Muir, of Locherbie, who was suddenly cut off in the very prime of his life and usefulness. His death makes the first instance of a vacancy so caused in the General Committee ; and as no provision had been made in the rules for such a contingency, his seat has continued vacant until now.

THE CONGRESS.

The efforts of local associations to instruct themselves and interest others in regard to mattersof educational importance have been seconded and brought into focus, as it were, by the larger synodical or provincial gatherings, and especially by the annual congress, which met at Stirling in the beginning of the current year. That meeting, I am glad to say, was no less successful than any of its predecessors. The number attending and taking part in the discussions could not, of course, be expected to be so large as it was in the previous year at Glasgow, but even in this regard the gathering was eminently successful. Aud it was not less so in the point of work done. The papers were of no little interest and value. Several of them—and these not the least important—were read by gentlemen not connected with the profession. This was but one indication of that deep interest, manifested also by other important services, which was taken in the meeting by all the School Boards of the midland district. The countenance and aid of the provost and magistrates of Stirling were also freely given, and deserve our meed of thanks and praise. On the success of the more festive portion of the proceedings, and particularly, the brilliance of the conversazione, I cannot enlarge, from the fact that the entire arrangement of these was due to the local association of which I am a member, and was then chairman. But this I am quite at liberty to say, that the local members exerted themselves most willingly, to the full extent of their resources, for the comfort of their visitors, and that their efforts seemed to meet on all hands with due appreciation.

THE INSTITUTE’S DEGREES.

Of the subjects which have occupied the attention of your committees during the year, it is unnecessary to speak at length. The minutes of the various meetings, as well as the statement of the treasurer, are now in your hands. The income has been sufficient for all the needs of the Institute. Although it has suffered some diminution, that has arisen from a most honourable cause. In order to increase the value of degree o

Fellow of the Institute, as a mark of professional distinction, and to make ! it an honour of which the ablest and best men among us might reasonably | be proud, arecommendation was recently made to the Board of Examiners to scrutinise all applications for the degree with the greatest care, and to admit only those who come up to a very high standard of professional ability and experience. That recommendation has been carried out by Dr. Morrison and his board with the greatest strictness, and one result of it has been a smaller income from diploma fees than was wont to be the case. We may sympathise with the treasurer over his loss of income, but I am sure we must all give our approval to the action of the Board of Examiners. The granting of these degrees was one of the original reasons for the existence of the institute. As a sort of teacher’s hallmark and license for the practice of his calling—which I believe was the original intention of them—they have been long superseded by the Government certificate. But it is still possible, by judicious action like that which has been inaugurated, to give them such value in the eyes of the profession and the public that there will be as great ambition to append the letters F. E.I. S. to one’s name as there is to add F.ILG.S., F.R.S., and other similar honorary distinctions.

On the other hand, the expenditure has been kept within bounds by the judicious management of the treasurer. One item of expense, which is less this year than usual, is that for deputations. Some may perhaps regard that as an indication of the apathy of the institute in regard to questions of the day, but the majority will perhaps look upon it as a good sign—as an evidence, in fact, that fewer of our presumed rights have been supposed to be endangered in the course of the year. Only two deputations have been required. One of these was the usual complimentary one to the Congress of English teachers, at which the institute was ably represented by Dr. Morrison and Mr. Mackay. The other was in connection with Sir Herbert Maxwell’s Bill, which sought to provide a tenure of office for teachers ; and, on this occasion, the interests of the profession were attended to by the Secretary with his usual fidelity and zeal. The matter on which legislation was proposed by this Bill is the only one that has emerged during the year in regard to which there was, or seemed to be, some measure of antagonism between teachers and School Boards. The question is not yet settled, although various modes of settlement have been suggested ; and most of you, 1 have little doubt, have to a greater or less extent made up your minds in favour of one or other of these modes.

SCHOOL BOARDS AND TEACHERS.

To me the settlement does not seem altogether free from difficulty, and, therefore, if you permit me to say a word or two on the subject, I should wish to speak with all due caution and reserve. The evil against which we seek a remedy—the capricious and unjust dismissal of a deserving teacher—is a clear enough possibility under the Education Act, and has already manifested itself in actual occurrence. A remedy, which shall secure the teacher against arbitrary treatment, while it leaves the responsibility of School Boards unimpaired, and which, at the same time, shall be within our reach, is not perhaps so obvious, although I trust it is not impossible to find. The publicity given to the notorious Leswalt case has turned public attention more than before to the subject, so that teachers have now a fairer field for arguing the question. The Leswalt case is not the only one of the kind—from facts within my own knowledge, I should be justified in saying it is not even the worst. They are said to be few; and although they are not so few as we would like them, we gladly admit that they are the exception, and not the rule. I am sure I speak for all of you when I say that we are so far from rejoicing in any such collisions between teachers and the local authority that—in spite of the argument they supply for some sort of tenure—we had much rather they bad never been heard of. The practical question, therefore, is—are such exceptional instances of ill-treatment, and the possibility of similar occurrences at any place and at any time sufficient to justify a revision of the law which regulates the relations between School Boards and their teachers? Or, as I should like to put it, would some tenure of office other than the mere will of the majority of the School Board be for the advantage of education ? because that, I am convinced, is the only question the public will be got to look at—the only form in which they can be expected to consider the case. And I am not afraid to put it in that shape ; for my conviction is strong that, whatever is good for education in general, must in the long run be for the advantage of the teacher. It has been suggested that the connection between the teacher and the Board should be looked at in the light of a mere business contract, which may at any time be lightly closed. But the fact is, that there is always a measure of discredit attached to a teacher who has been dismissed from his school. I am not sure that I would wish to see that feeling entirely removed. If the relations between Boards and teachers should become so slender in their attachment that a dismissed teacher incurred no disgrace from the fact of dismissal—I am not now speaking of the reasons, good or bad, for it— we would soon have to face a state of matters in which the teacher would regard himself as a mere hireling, whose payment measured the whole amount of his interest in his work, than which nothing more calamitous, I think, could happen to the cause of education. If the dismissal is obviously unreasonable, it may be argued that all that requires to be done is to show that it is so, and the teacher’s prospects will suffer no injury. To some extent that is true. If a man is bold enough to put his case before the public, and fortunate enough to get them to attend to it, the chances are that substantial justice may be done to his character. But meantime hd has lost the means of livelihood ; and there is the further risk that, in defending his reputation, he may acquire the character of an aggressive and troublesome person, whom other School Boards will be naturally shy of engaging. Moreover, some of the best men shrink from the notoriety which such a proceeding implies, and choose rather to suffer injustice in silence. The loss of such men, and the mental disquietude of others, who know not what the stirring of the occasionally unclean waters of parochial politics every three years may bring forth for them, cannot be regarded as a very decided gain to education,

On the other hand, I think we must admit that, in taking the power of dismissal entirely out of the hands of School Boards, there is a danger of lessening that feeling of individual responsibility which is one of the best securities for the faithful discharge of duty, I am anxious to bear my own testimony—and I think the experience of most of you will be similar—to the general excellence of our School Board management, and to the circumspection which, in most cases, has marked their dealings with the teachers under their charge.    Would this carefulness and

caution in regard to matters affecting the personal or professional character of the teacher be increased or diminished by taking from them the responsibility of final judgment 1 Would it be likely that appeals to the higher authority would be frequently and lightly made ? just as at present, although dismissal may be rare, ill-natured complaints to the Board by unwise members—there are such—are not so rare. Would the teacher’s relation to the Board assume the nature of antagonism ? And would they and he in consequence be less interested and united in the great work which they must do so far in common ? If I believed that any of these evil results were inevitable, I say at once that I would be content to let tenure go, and to trust entirely to the good sense and good feeling of the Boards.

But there is no necessity for imagining any such state of things. The difference between the teachers and the public in this matter is not so wide as some would make it appear. Their interests, in fact, are the same. No one desires to shelter the inefficient, the idle, or the vicious. Nothing has been declared more emphatically than that. It is directly in the interest of the teaching body that these worthless members should be got rid of. And it is no less directly in the interest of education that the capable and worthy should be retained. Life tenure is dead, and we would not even drop a tear upou its grave. All that is required is something that will either prevent the possibility of capricious dismissal, or, at least, remedy its effects. I think the Special Committee of the Institute has shown wisdom in resolving to look to the Education Department for the due protection of the teacher. A right of appeal from the decision of School Boards to the Sheriffs of Counties will be difficult to obtain, and, moreover, the authority which has granted a man the license to exercise his calling seems to me to be his most natural protector against injustice in the discharge of it, The right of dismissal might well and safely enough be left with the School Boards, provided they were required to obtain the sanction of the Department. This would place the teacher in a similar position with other public and parochial officers, elected by, and subject to, the direction of public boards. Were the Department found unable or unwilling to exert a final control over the power of the Boards, something—and not a little— would be gained by obtaining even its judgment in cases of alleged arbitrary dismissal. If the reasons of the Boards were found sufficient, confidence in them would be confirmed, and other teachers would not hesitate to take service with them ; if the reverse was the case, the dismissed teacher, with the Department's certificate of innocence, could freely present himself for another engagement,

FREE SCHOOLS.

However the position of the Institute in regard to this question may stand in relation to outside opinion, in petitioning against Dr. Cameron’s Free Education Bill it seemed to be quite in harmony with the prevalent public sentiment as expressed by School Boards, Town Councils, and sundry other public bodies. Disapproving as I did of the Bill in question, especially of its partial and permissive character, I may yet confess to a feeling in favour of a national system of education, which shall be universal, compulsory, and free, and which shall include within its scope secondary as well as primary education. In this I think I see a possibility of removing several of the evils w'hich at present hinder our efforts, and of consolidating and strengthening the whole educational fabric. That may be a dream far from realisation—the opinion of the country, I admit, does not yet demand it—but it is a dream which I am not deterred from cherishing by any such imaginary terror as that of “ Americanising our institutions.” We have already taken the initial step towards it, and whether we shall proceed further in the same course we shall have by-and-bye to determine. I do not doubt that this is one of the questions of the future, and our duty, therefore, is to be gradually maturing opinion on the subject.

ENTRANCE EXAMINATIONS.

The question of university reform has again been opened lately in this place in the form of a vigorous protest by the Rector of the High School of Edinburgh against the competition between the universities and the higher classes of the secondary schools. The complaint is an old one, and the remedy is not new. The often asked-for entrance examination has, at any rate, not yet been granted. It is hopeless to imagine that the standard of exit from the schools will rise, or can long continue above the mark of that for admission into the universities. If a reasonably high entrance examination were instituted, applicants would be bound to rise to that level, and there is nothing that I am aware of, even in the present condition of the schools, to prevent their doing so. Statistics such as we have often been treated to, showing the lamentable failure of junior students to pass a very elementary examination in Latin, are not quite to the point. No one who fails to pass that examination is turned away ; and, so long as that is the case, things will remain pretty much as they are. Quite as interesting a set of statistics might be prepared if some outside authority were to ascertain how many of these failed students were able to pass a similarly simple examination at the close of their session, It would not be a surprise to find that their progress had not

been much greater than it might have been expected to be had they remained another year at school. It has been argued, however, than an entrance examination might exclude many excellent students, who, although from defect of training or natural Inaptitude they may never become classical scholars, may yet distinguish themselves in philosophy or science. If that is so, what is gained by pretending to pass them through the classical portal ? Is not the real remedy for this a greater freedom in the choice of subjects of study, and an increase in the avenues to the degree ? I am not likely to undervalue the advantages of classical study, but I think we must recognise the possibilities of cultivation which other subjects afford.

ACADEMICAL TRAINING FOR TEACHERS.

I have no superstitious faith in the virtue of what is called a university education. I do not believe that the universities have a monopoly of that rather indefinite thing called culture. Sweetness and light may be drawn from other sources, Nevertheless, I do think that they afford the most convenient and easily accessible means of obtaining those elements of literature and thought by the due assimilation of which the cultured man is produced. I should like, therefore, to see them still more effectually operative than they are in the education of the teacher. Aspirants to the office of teacher should be urged to take full advantage of those opportunities of attending university classes which are now in their power—even should it be necessary, on that account, to delay for a little their entrance on the active duties of their profession. There are, of course, temptations to haste in the desire to blossom into the full dignity of mastership, and to experience at once the pleasure of earning a salary. But a complete educational equipment, which should add an academical training to the more strictly practical and professional study of the Normal schools, would open the way for more rapid promotion afterwards. It is matter for special regret that the university chairs of education do not yet seem to have obtained the place they ought to occupy in the preparation of the teacher. So far as I understand the function and scope of a university chair of education, it does not interfere with, but supplements in a most valuable way, the technical training of the Normal schools. The professors who so ably occupy these chairs are doing excellent and much-needed work in various ways which are opeu to them. By their writings and addresses at teachers’ meetings, and elsewhere, they are contributing to place the practice of education upon the sure ground of scientific principles. In this way they are influencing to good purpose many of the present members of the profession. But I fear they have not yet had the opportunity of dealing with the more plastic material afforded by youth to the extent that was anticipated by the advocates for the establishment of these chairs, among whom not the least strenuous was the Educational Institute of Scotland.

PROSPECTS OF TEACHERS.

From the fuller education of every teacher, two most desirable results would be likely to flow—the regular concatenation of all the parts of our educational system, from the lowest to the highest, and an improvement in the quality and in the public estimation of the profession. There is still subsisting something like an invidious distinction among teachers—a distinction which the separation of schools into grades (necessary for effective instruction as I believe it to be) only tends to perpetuate. However desirous an elementary teacher may be of finding an opportunity of devoting himself to instruction in some special subject for which his tastes and talents peculiarly fit him, the opportunity seldom comes to him—even in a secondary school. While, if anyone aspires so high as to indulge the hope of teaching his favourite subject from a chair in any one of our universities—with the honourable exception, be it said, of Aberdeen—he will find the fact of being a schoolmaster a greater obstacle to his ambition than almost any amount of ignorance on the subject to be professed. All these walls of separation might be broken down by the help of a higher and more uniform standard of attainment among all ranks of teachers, so that the young teacher might gladly accept the duties of the most humble post in the profession in the hope of rising, through time and by merit, to the most dignified. He might even dream of being permitted occasionally to enter the lofty region which at present he can only admire from the distance, inhabited by those shining beings, H.M. Inspectors of Schools. And if none of all these imaginations should ever become reality, no one need be afraid of becoming that “ most frightful thing” of Goethe—“a teacher who knows only what his scholars are intended to know.” We hear less now than we were wont to do of the “status of the teacher.” The phrase has, to a great extent, fallen obsolete. And it is not to be regretted • because, in the first place, it implies that dangers which were dreaded by some have not been realised ; and, in the second place, because no artificial means will raise the dignity of the teacher’s office in the public esteem so surely and so effectively as those that are within our own power.

THE CODE PROPOSALS.

The length to which my remarks have already run warns me not to enter on other subjects of interest. Upon the whole, I think we may regard the present educational outlook as satisfactory. There is more than one bit of blue in the sky. Immense activity prevails both within our own borders and beyond them. The popular interest excited by the Education Act has not yet died away, and proposed new legislation is proceeding on lines which wc have often affirmed to be the right ones. The proposals of the New Code must be regarded as eminently satisfactory. Very probably you will find in it details which you would prefer to see modified. That will form an important part of the work of the institute during the coming year. But I believe its proposals in general, as well as the regulations regarding inspectors, will be gladly accepted by teachers as an improvement on the present condition of I

affairs. They afford at least sufficient evidence—if that were still wanted—that we have at present a Minister in charge of the national education who adds to supreme ability and profound interest in each work a thorough knowledge of the details of the question.

PARTING WORDS.

In now retiring from the chair, I may be permitted to wish you all a pleasant and a successful meeting, and a safe return thereafter to the scene of your daily labours. These labours are by no means light. “ No faithful workman,” says Carlyle, “ finds his task a pastime.” If wc are ever tempted so to regard ours, we shall most probably find it a very melancholy one. But accepted in an earnest and true spirit, we shall find our labours not unmixed with pleasures of a very high and pure kind. Of those pleasures, I wish every one of you an ample share. My satisfaction in occupying the chair of the institute has been largely tempered and chastened by an ever-present sense of imperfection in the discharge of its duties—so much so that, if I make a true confession, I am bound to say that the feeling which is most strongly present with mo just now is one of relief in the near prospect of getting rid of the responsibilities of office. Nevertheless, I would not have you suppose that I am not proud of the honour which you have conferred upon me. I shall not forget this crowning mark of your kindness and confidence, and the memory of it which I shall henceforth carry with me is not likely to make me less anxious than before to do my utmost—small although I know that must at the best be—for the interests of this Educational Institute, and especially for the advancement of that noble cause of education to which we have dedicated our energies and our lives.—Schoolmaster.

EDUCATION IN THE CHIEF COUNTRIES OF THE WORLD.

By J. J. Beuzemaker, B.A.

Hf.rr Seyffarth continues to give us annually a most interesting volume on the general progress of education in the different countries of the world. The present volume, the third of the series, treats of the year 1880. On the whole, there seems little of great moment to chronicle about those countries where education has reached its highest excellence. In fact, there appears to be going on in some of them a kind of retrograde movement ; one of those stages in the evolution of culture, which we observe in everything that obeys the law's of gradual development. This slight retrogression, following a protracted period of mighty effort, is chiefly observed in the German States, in Switzerland, in Holland, and even in Austria. It seems as if, for the present at least, elementary education in these countries had been perfected to a degree almost exceeding the exigencies of actual wants. And much beyond this it is not possible to keep it, even with high ideal efforts and aspirations. As a last resort, if not as an initial impulse, the stern necessities of the circumstances, in which each individual, as well as each separate nation, is placed, determine the degree of development to be obtained. Given a new impulse, of whatever kind, manifesting itself either as a silent force or as a “blazing principle,” and there will follow a proportionate rise in the educational standard to meet the new demands. On the other hand, the march of educational progress in England, Belgium, France, and Italy, has been rapid during the last year, for in these countries the stimulus for improvement is still very great, and not yet in excess of the wants of the hour. Yet, beyond the domain of scientific necessity, there lies that of idealistic effort, equally real and beneficent in its effect, although insufficient when relied upon as an efficient cause. In several countries we read of voluntary help in education of a material, moral, and intellectual nature. This is especially the case in Switzerland and Germany, where philanthropy has lately occupied itself with such things as cannot be very well taken notice of by the state. Health-resorts have been founded in the country for the most deserving in the schools in large towns, where the pupils are kept during the holidays.

The great scholastic event of the year 1880 has been, no doubt, the International Educational Congress, held in August at Brussels, at the invitation of the Belgian Government, on the occasion of the fiftieth anniversary of Belgian independence. Delegates from most European countries and from North and South America, high officials, savants, authors, journalists, and schoolmasters and schoolmistresses, to the number of 400, had assembled to discuss the chief educational questions of the day. One of the first questions was that of the Universal School (Einheitsschule), i.e., whether the preparation for the Universities should be carried on in one uniform school or in different special schools. This is a question of greater interest to most Continental countries than to England. It is admitted on all hands, on the Continent, that the old Gymnasia, the Lycées, and the Athénées arc behind the times, and sorely in need of reformation. The question is, what to substitute for them. Can there be organised a school on sufficiently broad principles, and yet with aims clearly enough defined to be a direct road to the Universities ? Next came the question of Modern Languages. This is becoming a difficult problem in all countries. How far are the modern languages to encroach upon the ancient, and to what degree can they be made a means of a severe mental training ? That the necessity for their acquisition is being felt more and more, is incontestable; and, we believe, with the majority of the speakers at the Congress, that this will in the end over* ride the second part of the question. The Congress came to the conclusion that, only for those who intended to make Classics or Philology their special study, should the ancient languages continue to hold their old position.

The Industrial Schools and Schools of Art were then discussed, and the limits between them and the elementary schools defined. The Kindergarten system, so thoroughly established in Germany, Switzerland,

Heads,


Matter.


and Holland, and making its way at present in England and Belgium, was strongly attacked by some of the French delegates. The French are only just beginning to make experiments with the system, and it is just possible that their slight acquaintance with it may be at the bottom of their little love for it.

On one of the days, the subject of Girls’ Schools came on theto^- On this occasion the high strain of exalted idealism, which had hitherto marked the proceedings, gave way for a moment to a slight touch of human nature, which made these enthusiastic educationists akin to ordinary men and women. One Herr Erkelenz, from Cologne, had stated in his report on Girls’Schools that the Head should always be a man. This was more than the ladies present could allow to pass unchallenged. Five of them sprang to their feet to interpolate the speaker. They all spoke well, and their arguments carried conviction and gained applause. But, unfortunately, Herr Erkelenz had the last word, and summed up thus :—A head-mistress is either married or unmarried. If unmarried, she cannot be a femme modile for girls. If married, she either rules her husband or is ruled by him. In the first place, she sets a bad example to future wives, her pupils. If she is ruled by her husband—well, then there is a man at the head of the school. This argument seems to us, however, not perfectly conclusive, and, through its form, open to the objection that can be brought against most disjunctive syllogisms, that they do not exhaust all the possible cases.

As is often the case in great gatherings of this kind, mere oratory and fine phrases frequently take the place of sound argument, especially with the Latin races. At a discussion on the Training of Teachers a celebrated man exclaimed :—“ Voulez-vova des professeurs excellent s, ensignez anx jeunes gens aohereherle beauet le vrai, a aimer la verite,car la verite e'est la liberte.” (Continued applause.) Such high falutin may have suited a certain school of speculative philosophy, now almost forgotten, or may sound appropriate in the mouth of a modern apostle of sestheti-cism ; but it is singularly out of place when important practical matters form the subject of discussion.

In the German .States there is little to report in educational matters. In Prussia complaints are heard of a lack of sympathy among the authorities with elementary education. This is strange news indeed from the country that was fond of boasting that the schoolmaster had won the battle of Sadowa.

The change of government in Belgium has had a beneficial influence upon the state of elementary education in that country. Although the strife between the secularists and the clergy is by no means ended, the latter has become much less virulent in tone and less arrogant in action, being now' entirely deprived of official support.

i France is making every effort to be no longer behind in education. The Republican government may evince now and then perhaps a little too much zeal, aud thereby injure their own cause ; but there can be no doubt that, so far, they have been successful, and have made great progress. Compulsory elementary education ; the abolition of the “ letters of obedience,” i.e., of sham degrees ; the erection of day schools for girls to replace gradually the convent schools, which have done so much mischief to the social as well as to the intellectual life of the nation ; a higher standard for the qualifying examinations for teachers; the introduction of new school books, instead of those hitherto used, which were more antiquated and narrow in spirit than it is possible to describe —these are some of the chief measures that are being vigorously adopted. That the Ultramontane party are yielding with a very bad grace, and are detecting mines laid for them in the most innocuous reforms, need scarcely be observed.

In Italy education is making rapid progress, and here the Ultramontane party is so insiguificant as yet, compared with the whole body of the no 6uch friction is felt as in France. The same may be said of Portugal, where the standard of education has alw'ays been considerably higher than in neighbouring Spain. But even there we find an unusual activity displayed to educate the ignorant masses of the people. Spain, which in the middle ages, and in the 16th and 17th centuries, was second to no country in Europe in culture, had also more Universities than any other country. At present several of these universities are superfluous, and a Bill will soon be brought forward to convert some of them into elementary schools, which are so much needed. Ligas contra la ignorancia are also being formed throughout the country to agitate for gratuitous compulsory elementary education.

Education in Russia still wears too military a garb to exercise a truly civilising influence over the masses. The army of officers is scarcely separated from the army of teachers and professors. Directors of banks, governors of provinces, curators of universities are mostly generals. Everybody in Russia wears uniform, be he the most peaceable of schoolmasters or the most warlike of soldiers. Recently one of the chief inspectors visited a school in St. Petersburg. The masters were all present at the morning parade. The inspector here noticed two men in black coats standing among the group of masters, and inquired who they were. On being told that they belonged to the staff; of masters, he said with a smile, “What, masters without uniform 1 Impossible I” Elementary education in Russia is still in a very backward state. Nor is there any likelihood that it will soon improve. The war with Turkey has left Russia so exhausted and crippled in finances, and the recent conspiracies of the Nihilists have called into existence such an enormous body of state officials, that no money can be spared for educational purposes. A law on compulsory education has had to be abandoned, for the simple reason that no means could be found to cover the expenses it would entail.

We need not wonder that there is no reliable information on the state of education in Turkey. That it is in a lamentable condition, we may be certain. Nor can there be any doubt that the ignorance among the people will greatly contribute to the final breaking up of the Mussulman power in Europe, which some believe to be inevitable in the present political situation in the East.

In South America, the state of Chili seems to be in advance of the neighbouring states in education. Although there is still much room for improvement, much has already been done in the right direction. This state compares most favourably with Brazil, where the government does very little to promote the cause of education. In the United States, the schools in the larger cities alone seem to be in a satisfactory condition. Those in the smaller places, and in the country districts, are insufficient in numbers and their teachers ill qualified.

The accounts of the schools in China are still very incomplete and often contradictory. One missionary, named Hue, states that the Chinese possess excellent primary schools, and that the teaching is both systematic and comprehensive. Another missionary, Genahr, says that instruction in the Chinese schools is very inadequate, mechanical, and confined to the merest rudiments. It will require the opening up of the country by means of railways, which are said to be constructing now, before we can accurately compare the degree of Chinese civilisation with our own.

We cannot but congratulate the German schoolmaster on being able to obtain annually a compendium so full of useful information and so ably edited. A similar edition in English would be highly desirable, and would no doubt find many readers.—Educational Times.

NOTES OF A LESSON ON THE STRUCTURE AND USES OF JBONE.

1. Description.(a.) Composed of two substances. 1. Animal matter1

... Mineral matter. Periosteum (Peri-osteon).    (b.) Bony tissue hard)

spongy (c.) Hard bony tissue—Ilavereian canals—Lacunal—Canaliculi-II. Use.—For supporting body. For protection.

Specimens Required.—Bone burned in open fire to show Mineral manner; also Bone which has been kept in dilute Hydrochloni acid for some time, to show animal matter.

Method.

I. Description.

{a.) Composition.


(J.) Bony tissub.


(c.) Hard

BONY TISSUE.


II. Use.


Bone is only found in vertebrate animals. Other animals have no bone, only a substance like cartilage (chiliney). Bone is composed of two substances. (1.) Animal matter, which is soft, flexible, and when boiled resembles gelatine.

(2.) Mineral matter, which is chiefly phosphate of lime, carbonate of lime, and salts, The first gives tenacity to the bone, the second hardness and durability. In some animals, more animal than earthly matter, wrapped round the bone is a thick tough skin, the periosteum, which is plentifully supplied with bloodvessels.

1 There are two kinds of bony tissue, (1) hard compact part next the surface ; (2) spotigy part in the centre. In some bones this spongy part, in time, becomes hollow, and is filled with marrow, which in different animals is of different densities. In birds this hollow is filled with air.

2Hard bone is traversed by numerous tubes called haversian canals. These open into the periosteum, and receive blood-vessels from it. Around these tubes are arranged layers of bone, and in these are small open spaces or lacunas, which contain nutrient matter ; from the lacunae branch off numerous delicate canals, known as canaliculi, which ramify through the bone and unite with the canals and thus convey nutriment all over the bone.

3(1) Framework for moulding and supporting the body.

(2) For attachment of muscles.

(.3) Cavities for lodgment and protection of delicate organs, such as brain, &c.


Tell this.

Ask deviation and meaning of vertebrate. Name some vertebrates, andnon-vertebrates

Tell this ;—Explain cartilage.

Tell this :—Educe meaningof flexible. Tellhowanimalaud earthly matter may be separated from each other, and show bones thus treated. Educe and illustrate meaning of tenacity and durability. Illustrate this by fishes. Tell this. Ask or tell derivation of periosteum. Educe use of blood vessels

1Tell this Use specimens to show this.

Children can tell this ;—Meaning of different densities. Tell consequence. Recapitulate. 2Tell this, and 6how on diagram. Tell why so called. Draw class’s attention to this on diagram, and tell name and educe derivation of lacuna;. Meaning of nutrient. Show these on diagram, and explain that they contain blood vessels, and then educe use. Construct diagram on B. B. and recapitulate.

sEduce uses, from examples. Final recapitulation,


NOTES OF A LESSON ON THE EYE.


B. B. Sketch.—Diagram of Eye, showing Coats and Lens, and Optic Nerve. Technical terms set down under proper headings as lesson proceeds.

Apparatus, &c.—Model of Eye. Eye of Bullock. Bottle containing Aqueous Humour. Bottle containing crystalline lens.


Heads.


Matter.


Method


Structure.


Coats.


How we see.


(a.) Is a nearly round ball, about one inch in diameter, (A) 3 coats which enclose 3 lenses or humours, attached to optic nerve behind.

(1)    Sclerotic forms external wall of the eyeball, “ white of the eye.” Is an opaque, tough, fibrous membrane, cuts like leather, two openings.

Cornea, (d.) Is the circular watchglass-shaped transparent, fibrous body inserted in opening of Sclerotic coat at front of eye which admits light.

(2)    Choroid, (e.) Delicate coat of blood-vessels and pigment cells; forms middle coat of eye, and causes black appearance of pupil. (f.) When pigment is wanting the blood-vessels showing through the aperture give it a pinkish colour.

(3)    Betina [g.) is delicate coat, which lines interior of back of eye, consists partly of expansion of optic nerve.

Iris (h.) is the circular flattened perforated curtain of membrane, which is placed behind the cornea, and regulates by contraction and expansion of its opening (pupil) the quantity of light admitted to the eye.

Humours (i.) Aqueous is the clear limpid, watery fluid which fills the space in front of crystalline lens, and bathes both sides of Iris.

(4)    Crystalline Lens (k.) is the bi-convex lens-shaped transparent jelly-like body placed behind the iris, by which the light (l.) entering the eye is focussed and made to form inverted images on the retina.

(5)    Vitreous humour (m) is the large spherical, transparent glassylooking lens or humour, jelly-like, which fills up the greater part of eyeball, about four-fifths.

recapitulate.

The rays of light («.,) fall upon the cornea, pass through, and are converged by it; then enters the Aqueous (&.) humour. Those which fall upon its more central part, pass through the pupil, (c.) The rays strike upon the lens(c£.)which, by the convexity of its surface and greater density towards the centre (e.) very much increase the convergence (/.) of the rays passing through it. They then traverse the vitreous (y.) humour, and are brought to a focus (h.) upon the retina, forming there an exact image of the object, but (i.) inverted, and the sense of sight is thus awakened in the brain by the optic nerve (k).

Gen. Recapitulation.


(a.) Exhibit model and draw circles explaining it.

(b.) CJse model and explain.

(c.) Show model and obtain description from class (Greek, Skleros, hard). Show these on model.

[d. ) Elicit this from class (Latin, cornu horn),and noticeits position.

(<?.) Use model and show pigment.

(f.) Mention case of pink-eyed rabbit (familiar).

Illustrate by white mice and rabbits.

(y.) Show position and notice that part of optic nerve.

(h.) Show model and notice its position, draw attention to own case when coming from a dark room into a lighted one ; cause of this ; instance, cat’s eye.

(i.) Show some to class, and obtain description.

(Jc.) Show lens and get appearance from class ; explain this.

{m.) Exhibit some to class, and notice the difference between Aqueous and Vitreous. (Latin, Vitrum glass.)

[a.) Explain that all the rays that strike the eye do not enter.

(b.) Obtain position from class, serveBasrecapitula-tion. (c.) Have pupil shown. (d.) Use model, and explain and(e.) obtain from class.

(f ) Meaning from class.

(g.) Position of vitreous humour, explain (it) focus by magnifying glass, (i.) Explain why we see objects not inverted.

(k.) Use of optic nerve, and that braiu is cause of sight._


AT a meeting of the Invercargill High Schools Board of Governors held last month, at which Mr. Lumsden presided, a letter was read in committee from Mr. Von Tunzleman referring to the continuation of his services ; but immediate action was not taken. The report of the committee appointed to consider what steps should be taken in the matter of the further sale of endowment was briefly discussed. It was decided^to postpone action upon it till next meeting. The new teaching staff was considered in committee, and the meeting adjourned.


TIME TABLE.


¡5

O

o

¡5

w

w


<1

o

co

S

CO

! Grammar S Reading T Reading T Grammar S Grammar S Rhymes W

3 to 3.30

Derivations S Reading T Reading T T’nscription S Tables M Rhymes W

2.30 to 3

Arithmetic S Arithmetic S Dictation T Arithmetic S Reading W Writing M

2.30 to 3

Arithmetic S Arithmetic S Dictation T Arithmetic S T’nscription S Pleading W

CQCOCOhfflg

CO CO CO EH 02 ^

o

CO

OOP o

o

O O o o

ci

o

4^>

»o

Arithmet

Arithmet

Arithmet

Reading

Arithmet

Reading

o

4-»

io

c«i

Arithmet

Arithmet

Arithmet

Reading

Arithmet

Writing

1.30 to 2.5

Reading T Reading T Arithmetic S Tables M T’nseription S Tables M

1.30 to 2.5

Reading T Reading T Arithmetic 8 Tables M Reading W Tables M

1,25 to 1,30

%

Ü

O

Pi

¡1.25 to 1.30

1

Roll Call

12.30 to 1.251

Dinner

12.30tol.25

Dinner

12 to 12.30

H. Lessons &c. Do.

Do.

Transcription H. Lessons, &c.

12 to 12.30

H. Lessons, &c. Do.

Do.

Reading H. Lessons, &c.

11.35 to 12

Writing S Writing S Reading T Spelling S Tables M Tables M

11.35 to 12

Composition S Compositions Spelling S Grammar T Grammar T Tables M

11.10 to 11.35

Geography S Geography S Geography S Geography T Spelling S Reading M

11.10 to 11.35.

Grammar S Grammar S Grammar S Dictation T Geography M Reading M 1

o

5

O

Ö

Grammar S Grammar S Grammar S Reading T Reading M Writing M

10.30 to 11.10

Geography S Geography S Geography T Geography S Reading M Writing M

10.5 to 10.30

Arithmetic T Arithmetic T Writing T Writing T Writing T Rhymes M

10.5 to 10.30

Writing T Writing T Writing T Writing T Tables M Rhymes M

T—<

o

o

Roll Call

IO

Ö

o

o

Roll Call

o

s

o

CO

o»

9.30-9.50 9.50-10 Diet. Arith. T Do. Do. T Arithmetic T Arithmetic T Arithmetic T Tables M

9.30 to 10.

Arithmetic T Arithmetic T Arithmetic T Arithmetic T Arithmetic T Tables M

£

et

o

HH . S-* HH . kTV P" HH 1—< . r M HHH

Ida ss.

b-î . K* W »

r k ^ h h . Krhmhh

c3

P

Monday

Wedn’sd’y

Friday

Days

Tuesday

Thursday

« C"4

"2 II

Q

fc ^ f*.

ge-S

« Ph

rO CO

S rP

II

2 H


£ 1

si


a


(=H


pq «


t>

pH bH

S i

d oä


H o »

w _

fu ■<


ft.


O W ns (>


a


to


« go


CD

3

g

a


o O , , >0

C3 IQ CO    Tf<


>


>


ScS


»P CO ^

cm cq oo


i o CM oo


. M

_ _ . M O    o

2 ^ £ C« *C0 ^

S £ A a S'ßO

B    S 2- o


t? ¡o o? >p ir. CO ’ ¿1 ¿1 f-


Ç) I mio I io L <jq co


to So O to ®

•S 9 ^ 9 « £

© ©•“>». u d

Ph co ft f* H


School Department— Inspectors’ Report for the

Year 1880    .........

Educational Institute of Scotland ...    ...    ......

Education in the Chief Countries

of the World.........

Notes of a Lesson on the Structure and Use of Bone ... Notes of a Lesson on the Eye... Time Table .........


66 67

69

70 .. 71 .. 71 72-73


Leaders


Mutual Trust and Investment Society.

LIMITED.

(Incorporated under the Companies Statute 1864).

OFFICES: 66 COLLINS-STREET WEST, MELBOURNE.

Capital, 41100,000 in 10,000 shares of £10 each. Subscribed, £10,000. Paid-up’

£9301.

DIRECTORS:

Mr.


Robert Inglis, accountant, 64 Collins-street West.

Mr. Charles Hetherington, Secretary, 66 Collins-street West.


o

PUBLISHED EVERY MONTH.


CONTENTS

Technical Education in a ^

Saxon Town......... 73

Notes of the Month ...    ...    74

President Garfield a Teacher ...    74

Science Gossip ...     75

University of Melbourne ...    76

Victorian Education Department ............ 78

Appointments ...    •••    78

A Visit to the Kinder Garten School at South Yarra ...    78

Canon Daniel on Memory ...    78

THE

Mr. M. IT. Davies, Chairman, 8 Collins-street West.

Mr. Francis J. Smart, architect (Messrs Henderson-and Smart)

Auctioneers and Special Valuators: Messrs. C. .T. and T. Ham, Swanston»St.

Surveyor and Valuator; Mr. Percy Oakden (Messrs. Terry and Oakden).

Auditors : Mr. Thomas Inglis and Mr. John G. Shield. SECRETARY; MR. C. HETHERINGTON. MANAGING DIRECTOR: MR. ROBERT INGLTS.

SECOND ISSUE of 1000 SHARES of £10 EACH.

The chief objects of the company are; —1. To lend money to members upon the security of real property or upon bank, gas or building society shares, or upon other approved security. 2. To negotiate loans of all descriptions upon such terms as to profit and remuneration as may be agreed upon. 3. To buy and sell freehold or leasehold estates. 4. To collect and reoeive rents, debts, dividends, interest and other moneys. 5. To buy and sell shares in any public banking corporation, insurance or gas company or building society, or any incorporated company. 6. To act as attorney or agent in the management of estates for absentees and for trustees.

In order to increase the paid-up capital of the company and extend its business it has been decided to make a second issue of 1000 shares of £10 each. These shares are now offered to the public at a premium of os. per share, and may be paid either in cash or by instalments of £1 per share per month. The company has now completed its sixth half-year, and has, during its progress, in addition to paying off all preliminary expenses, paid one half-yearly dividend at the rate of 8 per cent, per annum, and four half-yearly dividends at the rate of 10 per cent, per annum. The secretary will receive application for shares until further notice, and will furnish information regarding the company to intending shareholders, together with copies of previous reports and balance-sheets.

C. .HETHERINGTON, Secretary.

APPLICATION FOR SHARES.

To the Directors of the Mutual Trust and Investment Society (Limited).—I hereby apply for    shares in the Mutual Trust and Investment Sosicty

(Limited), and inclose , being 5s. per share application fee.

Name in full.........................................

Address................................................

Dato.............................. Occupation ..........................................

NOTICES TO CORRESPONDENTS.

Advortisomonts and other business communications should be addressed to the Publishers. No advertisements will be inserted without a written order, or prepayment. It is particularly requested that they may bo sent oarly in tho month.

Books, music, and school appliances for notice, and all letters containing anything connected with the literary portion of tho paper shonld be addressed Po the Pd ¡tor. Every communication accompanied by the name and address of tho sender (as a guarantee of good faith, though not always for publication) will bo acknowledged; but we cannot attend to anonymous letters.

Du sir a 1 a s r a ir 5 r bo a 1 m aster,

NO VEMBER 23,    1881.

The deference shown by the Education Department of Great Britain to the opinions of the Executive Committee of the National Union of Elementary Teachers is an evidence of what can be accomplished when teachers act as a body, and with unswerving persistency. It also serves to illustrate the soundness of the advice we tendered to the Hon. the Minister of Public Instruction in this colony, when he first announced to Parliament his intention of introducing a new scheme of classification of teachers. By closely associating themselves together, and taking an active part in the discussion of all questions aftecting the progress of popular education in England, the National Union of Elementary Teachers have at length forced the Government to pay some respect to their views. Before proceeding to introduce an amended code, the political head of the Education Department—Mr. Mundella,

M.P.—requested the association to offer suggestions upon the reforms required in the code. A committee was thereupon appointed, termed the “ Code Committee,” to draw up a report upon the revision of the code. The third progress report of this committee, dated Sept. 17th, 1881, is now beforeus. From this we gather that the views of the elementary teachers have been “ largely taken into account by the vice-President of the Council when preparing the proposals for the revision of the code recently placed before Parliament.” While acknowledging this, the code committee point out that, as the efficiency of the reforms now proposed will depend upon the manner in which the principles contained in the “ proposals ” are applied and embodied [in the code itself, the following general principles should underlie any reform of the education code :—

1.    That greater freedom of teaching should be given to the teachers,

both as to methods and classification.

2.    That while the individual and mechanical results of instruction

should be secured, the higher educational work, the general tone, and the methods and organization should be more considered than was the case under the Revised and New Codes.

3.    That any reduction in the amount of the Parliamentary Grant for

Elementary Education, or the transference of the duty of making such grants from the General Exchequer to the local rates or to voluntary contributors, would be fatal to the educational progress of the country.

4.    That it is advisable to make a larger part of the grant depend on

fixed conditions (say, average attendance), and less on those which are variable and uncertain (e. g.. Examination).

5.    That the difficulties of half-time school's require that in any system

of grants based on average attendance, special arrangements should be made for children presented under Art, 20.

6.    That the pressure upon scholars and teachers by the undue

importance attached to the per-centage of passes in elementary subjects, should be removed or reduced.

Upon the system of making the Government subsidy dependent upon the number of individual passes, the committee say :—*

“ If No. 14 be so modified as to allow of the maximum grant being paid on something less than 100 per cent, of the possible passes, and a reasonable allowance be made for dull scholars, the injurious pressure on scholars and teachers will be relieved. They are nevertheless convinced that if the present form of No. 14 be inserted in the code, the evil results of the old system will be intensified, and that no relief whatever will be accorded either to scholars or teachers. The rage for high percentages now existing in the minds of some managers and teachers will undoubtedly be increased under the present form of No. 14. The committee desire to reaffirm their opinion that the maximum grant for elementary subjects should be attainable on passing 75 per cent, of those possible. The liberty of classification and of teaching, so much to be desired in the interests of education, cannot be secured without the concession now asked for, or some greater liberty in respect of the representation of scholars in the same standard. They also desire to express their satisfaction that the grant for average attendance under Art. 19 (A) is retained, and they recommend for consideration the advisability of enlarging the proportion of grant payable under this article, and of reducing that payable on the various examinations. They consider that it would be an advantage to make a larger part of the grant depend upon certain sources, and that only so much money payment should depend on the • examinations and other variable conditions as would secure proper attention to the instruction.”

With regard to the proposed granting of certificates to University graduates, the committee remark :—“ After a close examination of the probable effects of this proposal, the committee are of opinion that some danger exists of introducing into the profession persons possessing little or no practical skill. While valuing the scholarship indicated fby a degree, the committee know it to be useless for elementary school purposes, if unaccompanied by skill and experience in the art of teaching. They therefore advise that greater safeguards should be provided against this danger, and as a present means to that end, the words £ two years ’ should be substituted for ‘ one year ’ as the period of probation before graduates are admitted to the Certificate Examination. Having regard to the want of practical skill, the committee desire to point out the inability of these graduates to act as efficient teachers for sixty children. The committee think that in justice to the large number of apprenticed and trained teachers who, drawn into the profession

by the Department’s regulations, are now unable to obtain appointments, the operation of this proposal should be deferred.”

Among the suggestions made by the Union and adopted by the Government may be named :—

The abolition of the payment on the individual pass of each scholar.

The abolition of endorsements on Certificates after they are raised to the 1st class.

The raising of the standard for certificates.

The raising of low certificates by re-examination.

The re-organization and the partial opening of the Inspectorate.

The regulation of the supply of teachers, by the limitation of the number of pupil teachers.

The closing of the bye-entrances to the profession.

The abolition of the Child’s School Book.

The simplification of the Needlework Schedule.

The granting of “Means of Appeal.”

Speaking of the “ Code Committee’s” report, the editor of the Schoolmaster says:—“We will not now discuss the criticisms and amendments of the Committee, but commend them to the serious attention.of our readers, and trust that every teacher will make a point of attending the association meetings at which they are discussed, and seeing that his views have their weight in forming the opinion of the association which is transmitted to the Executive. Teachers, whether members or non-members of the National Union, have now a grand opportunity, and if they do not seize it they will have themselves to blame. The chief thing teachers look for is the dethronement of the idol of percentages, and the great relief to children and teachers which this will bring. The individual payment for the individual pass to which the power of this idol has been so justly ascribed is abolished. We hail the abolition with the greatest satisfaction ; but the abolition alone will not dethrone the idol whose worship has demoralised our system, nor remove the unnecessary pressure. Payment on the average attendance, if cent, per cent, of passes is to be required for the maximum grant, will only increase the worship of the idol and intensify the pressure. We are, therefore, glad to find that the Code Committee have adhered to their former recommendation, and insisted that the maximum grant should be attainable on the passing of 75 per cent, of those presented, and that there should be little or no inducement to go beyond. This is a most vital point, not so much for teachers as for education, and upon the adoption of this (we do not tie ourselves to this exact percentage), or some similar plan, the educational value of the Code will depend. There is some reason to believe, from the Vice-President’s speeches, that he does not intend to ask for cent, per cent, of passes for the maximum grant; but we cannot too strongly press upon teachers that this is the point in the proposals which should receive their most earnest attention.”

The recent appointment of a Royal Commission to enquire into the working of the Education Act should, we think, stimulate State school teachers to support the committee of the Victorian Teachers’ Association in any action which they may deem advisable to take to relieve the profession from some of the disabilities under which it at present labours, and to secure the adoption of a scheme of classification that will place teachers on an equal footing with the highest branch of the Civil Service. All that is wanted is that the teachers should act with promptness and with cordiality.

The press at large has been seriously taking into consideration Mr. Mundella’s proposition to admit, as teachers in elementary schools, university graduates who have served one year as assistants in a school under inspection, and have received certificates of skill in teaching the “ Standards.” Mr. Mundella makes the somewhat startling declaration, in defence of this step, that the Department has applications by the dozen, and almost by the hundred, from university men who want occupation. This statement, at first sight, awakens curiosity. What sort of university men can those be who desire to teach the three It’s? But we must remember that the word “ university man” no longer means what it used to mean. Formerly a “ university man ” belonged to Oxford, Cambridge, or Dublin ; now, he may belong to London, Durham, Victoria, or the Queen’s Universities of Ireland, without counting the very

respectable universities of Aberdeen, St. Andrew’s, Edinburgh, or Glasgow^, or those of Sydney, Melbourne, Calcutta, or Montreal, which are all British universities, and all give authority to a graduate to call himself a “ university man.”" At several of those universities, where no residence is expected, there are a large number of men who have taught themselves, and by praiseworthy efforts achieved even honours in the examinations. These are, we presume, the graduates who have applied to Mr. Mundella ; or there may be among them some of the failures of the residential universities—men who would take to anything, if they could get anything; or there may be some who would take up the life of elementary schoolmaster, as foreseeing that there is good and honest work to do, and that there are very fair prizes—much better prizes than are open to clergymen of mediocre ability, no pushing power, and no interest.

We may, however, be assured that the number of the higher class of university men, who will content themselves with a life spent in teaching School Boai'd children the elements of knowledge, is not large. On the other hand, there is a certain significance in the step, though it is hesitating and put forward with timidity. May we see behind it the first advance of a scheme for the organisation of middle-class education! This has already been hinted at in more than one paper. A body of university men, trained in the art of teaching, might become most useful in the working out of some comprehensive scheme of secondary education. We cannot suppose that Mr. Mundella’s object is merely the relief of the “ dozens ” of university men wdio have applied to him for employment. We make, however, a note of comment on the article of the New Code, and beg our readers to bear it in mind. We would not willingly exclude good candidates from the work of tuition; but in this particular instance, we would ask vdrether more does not remain behind.—Educational Times.


TECHNICAL EDUCATION IN A SAXON TOWN.

Mr. Felkin, the author of this interesting account of the notable work of technical education carried on in the comparatively small city of Chemnitz, in Saxony, is a Nottingham man who has resided some years at Chemnitz, and his experience in the English and in the Saxon town enable him to draw important comparisons and conclusions as to the value of specific technical instruction. Speaking of the difference between these days and those when the apprentice system flourished, the editor says :—•

The introduction of machinery into nearly every branch of industry has greatly changed the character of the relationship that formerly existed between the master and his apprentice ; and the application of science to industrial operations has, in not a few cases, transferred from the foreman or works' manager to the practical science teacher the key to these mysteries, and has rendered necessary, for workmen of every grade, a different kind of training from that which was considered sufficient not many years ago.

We speak of the editor apart from the author because the work is produced at Gresham College, for the Council of the City and Guilds of London Institute. But the report on the educational institutions of Chemnitz is entirely the work of Mr. Felkin, and it is he alone who places Chemnitz side by side with Nottingham, and who says :—

As Chemnitz is a large manufacturing town, like Nottingham, resembling it too in the fact that hosiery is one of its staple manufactures ; and, moreover, as the town of Chemnitz has already taken away the glove trade from Nottingham, and is, in the opinion of many, slowly undermining the trade in cotton hosiery too, it cannot but be important to the people of Nottingham to know something of the educational advantages which have enabled the Saxons to do this. For, in the writer’s opinion, neither in physique nor in energy and natural ability are these Saxons equal to Englishmen. On the contrary, the human raw material in Saxony is inferior to that of the midland counties, and yet the weaker race takes the bread out of the mouth of the stronger, and competes with it in the markets of the world. What enables it to do this? The answer to this question will partly be found in the educational advantages which the people of Chemnitz, and of other German towns undoubtedly possess.

Chemnitz increased from a population of 54,000 in 1864 to a population of 89,000 in 1879. It is the largest manufacturing town of Saxony, and is surrounded by small towns engaged in manufacture. Its main industries are machine-building, cotton


and wool spinning, weaving, and hosiery and glove manufactures. Mr. Felkin gives a full detailed account of the manufacturing industries of Chemnitz, and of the great strides its manufacturers have made in competition with the manufacturers of this and other countries ; he next tells us all about the general schools of the place, and finally describes with gi’eat detail and circumstantiality the technical schools. What he has to tell about the general schools is scarcely less interesting than his account of the technical schools, which are established distinctly to meet those new conditions of industrial life under which the apprenticeship system is going practically out of existence. The technical schools are under the control of the Home Minister. The Royal State Technical Educational Institution consists of a higher technical school, a builder’s school, foremen’s school, a technical drawing school. Then there are the higher weaving school, the hosiery school, the agricultural school, the school for hand weavers, the school for tailors, and the trade schools for males and females. We cannot make clear what this system is and how it works, because it is a matter of description and detail. This is to be found in the book, and it is a book which all who are interested in one of the most important branches of the education question ought to procure and read thoughtfully, without loss of time. Chemnitz is beating Nottingham out of the race in great fields of manufacturing industry, and Mr- Felkin shows how Chemnitz does it.


PRESIDENT GARFIELD A TEACHER.

President Garfield was a teacher. He taught (says the Schoolmaster) in the family, the district school, and the college. But, in common with many successful public men in America, he chose teaching, not as a career, but a stepping-stone to something else. However good for the individual and the reputation of teachers, this must be detrimental to education. While we are glad to learn that this disposition to adopt teaching as a half-way house on the road to something better is on the decline in America, signs are not wanting that it is on the increase in our own country. The causes, however, are different. In America the man plans it as one of his aids in his upward course; in England, what we may call official pressure, the worry of Codes and Inspectors, is driving him from work which he loves. Mr. Grove bore testimony to this before the Birmingham District Union many months ago, and Mr. Gardner, at the last meeting of the Executive, expressed the pain with which he heard in all parts of the country the almost feverish desire of teachers to quit the work of education for spheres in which honest work was better recognised, and the position was more secure. It is painful to hear so constantly experienced teachers say how gladly they would “ get out of it.” We trust that something will be done in the coming Code to remedy this by making the conditions of a teacher’s life pleasanter and less subject to needless worry and annoyance. This will not be unless teachers are on the alert. Those who wear the shoe know where it pinches. Mr. Mundella Las will, and teachers should show him the way to make their life pleasant in their work, and open up a career for them that no thoughts may enter their minds of making their profession a mere house of call on the journey of life.


Ilotes of ibc    onib.


The Education Department have decided that the time for closing the schools for the Christmas holidays will be left to the head teachers.

Professor Andrew has been appointed the Hon, the Minister of Public Instruction to act as examiner in Mathematics and Physics at the Science Examinations, in the place of Mr. Pirani, deceased.

THE Brisbane Week says :—“ Queensland children appear to be more regular in their attendance at school than those of any other of the Australian colonies, as shown by a report issued by the Education Department of Victoria. According to that document, the average attendance in Queensland was 55 00; Victoria, 52'03 ; New Zealand, 51-63; South Australia, 48'44 ; and New South Wales, 47*28.”

It is gratifying to learn that the boys’ and girls’ grammar schools at Maryborough, Queensland, are increasing their number of pupils. As a


Consequence, the teaching staff is to be increased also. Four scholarships, of the annual value of £20 for three years, have been donated by the Council.

Writing on the motion for withdrawal of State scholarships, made by Mr. Norton in the Queensland Assembly, the Queenslander trenchantly remarks :—“ Surely if there is anything which should cause a man to blush in the knowledge that his son was profiting by a State scholarship, he ought to be doubly ashamed if he permitted him to bold a bursary founded by a private benefactor. In the former case he, as a taxpayer, would be a contributor to the fund by which he benefited ; in the latter, he would accept a free gift from a private benefactor. The truth is, that the rhetorical flourishes about receiving State charity were nothing but empty sentiment. ... As private effort has in all cases proved itself inadequate to secure the universal spread of knowledge essential to the well-being of a self-governing democracy, we in Queensland have applied the machinery of State government to effect the purpose. . . . There need be no more shame in winning a bursary than in walking through a State-supported museum, or strolling in a State-supported garden.

Tenders have been received as follows by the Queensland Govern-ment:—For the erection of a kitchen and covered way to the State school at Seven Mile Rocks, John Storie, £80. For teacher’s residence at Blenheim, J. Byers and Son, four months, £2S8 10s.; T. Jetfcott, £261 ; and Thomas Christie, £215 10s. For the erection of a State school at Lytton, T. Carey, £598 10s.; I. L. Sands, £749 ; J. Meise, £658; Hardy and Vowles, £587 los.; and J. Salisbury, £720. For the erection of a State school at Allora, A. Gordon, four months, £315. For effecting some repairs, including painting, at Ashgrove School, J. Storie, £60

At the annual scholarship examinations held recently in connection with the Ipswich (Queensland) Grammar School, there were a large number of examinees. The following carried off scholarships :—Peter Macgregor, the Tiffin Scholarship, two years; Theodore R. Barrymore (West Ipswich), the Trustees’ Scholarship, two years ; James Griffiths (North Ipswich), the Thom Scholarship, one year ; Louis Heiner (North Ipswich), the Special Trustees’ Scholarship, one year. None of these boys are over 17 years of age.

The foundation-stone of a new High School was lately laid at Waitaki, N.Z., by Mr. Slermiski, M.H.R. The bon. gentleman was presented with a handsome silver trowel and ivory mallet in memento of the event.

A special meeting of the Winton (N.Z.) School Committee was held, as appointed, on the 12-th ultimo, a fair number of auditors being present; Letters from Messrs. Taylor, M'Cormack and Butler were read. They complained that some of their children were not progressing as well as they (the parents! had anticipated they would. It was proposed that the letters be sent to the board to be dealt with ; the chairman suggesting, however, that the committee should take action. A member then proposed that the present teacher be asked to resign. The secretary moved an amendment to the effect that the parents named be requested to send their children more regularly to school, the irregularity of attendance being, he urged, the real cause of their slow progress. The register showed they had not attended more than 50 per cent, of the time they had been attached,

The Auckland Board of Education have increased the salary of its Inspector to £550 per year.

The Education Board of Dunedin have passed a resolution calling the attention of the Government to the injustice done to the Otago district by taking as the basis for the appointment of sites for schools local population instead of school attendance.

In a cricket match between the Central and Gladstone Public Schools at Invercargill, the latter secured an easy victory.

A meeting of the Invercargill School Committee was held on the 24th ultimo ; Mr. Matheson presiding. A statement presented showed that the average attendance for the month was as follows :—Central School, 468 ; North School, 163 ; and South School, 420. The attendance at the North School had been lessened by sickness to a greater extent during October than when the measles prevailed in the other schools. Necessary repairs at the Central and South Schools, also extensive repairs at the North School, were referred—the former to the Visiting Committee to deal with, and the latter to the Education Board. Repairs to Mr. Gurr’s residence were also referred to the Education Board, with a recommendation that they be effected.

The ordinary monthly meeting of the Invercargill High Schools Board was held in the first week of this month, Mr. Lumsdenin the chair. The report of Mr. Blanchtlower stated that three new boys had entered the school since the commencement of the new quarter, bringing the roll number up to 57. The average attendance was much higher than previously, it now being 52. The teacher recommended that an examination by an outside examiner should be held before Christmas. On this head it was resolved, after some discussion, that the Inspector-General of schools be written to with a view of ascertaining when be would be down to inspect the school. The report from the head mistress of the girls’ school stated that the number of pupils attending during the quarter was 48, the average daily attendance during the last month being 43'4 The sale of the board’s endowments was further considered at length, and the ultimate decision was to ask the permission of the Government to sell sections 37 and 41 at Wairio, on terms of credit; and also to be allowed to dispose of, in a similar manner, all the unleased sections in Gore, at the upset price named in the recommendations of the subcommittee appointed to investigate and report—section 11, block 2, however, to be supplemented.

The agricultural college at Wellington, N. Z., has now a total acreage of 661 acres, of which 626 are tilled and sown. The institution’s live stock comprises 1110 sheep, 58 pigs, and 14 horses. The buildings are being enlarged by the addition of a new wing. This is greatly welcomed, for the present building is overcrowded, and a number of applicants have had to be refused.

The monthly meeting of the Southland Education Board was held on the 4th inst., Mr. T. M. M'Donald in the chair. A special report from the Executive Committee stated that new school buildings at Limestone Plains and Otama, and residences at Wyndham, Pukerau, Edendale, Otatara, and Heddowbush, in course of erection. The ordinary report of the same committee recommended several smaller works. Mr. Stocks was appointed temporarily to the Miller’s Flat school. It was decided to elect a school committee for the new district of East Hokonui, at a public meeting to be held for the purpose. The question of sites for schools at Biversdale, Waikaka, and Otama was dealt with, and variously referred, and after passing accounts to the amount of £1926 Is. lid., the meeting adjourned till the 2nd prox.

The monthly meeting of the Otago School Teachers’ Association was held as usual in the first week of the month, when several new members were admitted. Mr. Qurr drew attention to the fact that the difference between thesalaries of certificated and uncertificated teachers was only £20 per annum in Otago,and that teachers of that district were paid much lower for gaining high average, &c., than those of any other district in New Zealand. He thought this perhaps was caused by the issuing of bonuses by the boards instead of by the Department,and moved that the latter body undertake the duty of isssue. Mr. Neil seconded the motion, and it was carried unanimously. Mr. Neil read an interesting paper on “ How to meet country school teachers’difficulties,” which was much appreciated, and the meeting terminated.

A MEETING of the senate of the Melbourne University was held on the 18th ult,, to consider the new standing orders, Dr. Madden (the warden) in the chair. The chief alterations effected by the clauses considered—84 to 96—relate to the election of wardens and councillors, and provide that elections shall take place at meetings of the senate, and that voting shall be by ballot if more than one candidate put up for a vacancy. The voting is to be done somewhat upon the proportional system, and will be conducted on the principle of excluding those with fewest votes on papers whereon voters have marked their preferences by numbers opposite names, as 1, 2, 3 &c., the excluded votes being given to those on the same paper, next in order of preference.

A well-attended meeting was held at Castlemaine, Victoria, on the 25th ult., to enable Messrs. Pearson and Patterson to address themselves to the question of the appointment of an Education Commission to enquire into the working of the Act, then under consideration in the Assembly. Mr. Patterson, in the course of his remarks, appeared greatly apprehensive lest the proposed enquiry would lead to an interference with the secular principle of the Education Act. He objected to the commission because nearly all the gentlemen composing it were, he said, known to be in favour of religious instruction in State schools. The proposed inquiry, he argued further, reflected impliedly upon the previous administrations, and also upon Parliament itself; and complained that the country Boards of Advice were not represented upon it. He could see that the stated over-costliness of the system was not the real grievance, but that the Roman Catholics, numbering one-fifth of the population, desired to educate their own children in their own way, and to participate in the money set apart by Parliament for general instruction. Mr. Pearson dwelt principally on the phase of the question as effecting and affected by the Roman Catholic claims. He would be ready to concede all the Catholics asked if it could be done without jeopardy to the free, compulsory, and secular character of the Act. This, however, he felt, could not be done, and he argued that no commission could satisfy them, who would always be attached to their own foregone conclusions.

A meeting of the council of the Melbourne University was held on the 25th ult., at which Dr. Bromby presided. An agreement was arrived at that examinee’s papers should be returned by examiners to the registrar within one month after being given them for scrutiny. Additions to the medical school buildings were arranged to be considered at a conference with the faculty. A letter was read from Mr. Thos. A’Beckett stating that he did not intend to continue as lecturer on procedure after the close of the year. On the suggestion of Professor Elkington it was decided that as the examination for the Cobden Club prize had not been sufficiently advertised, it be postponed until the February term, 1882, the right of any candidate who was then qualified to enter being barred.

At an adjourned meeting of the Senate of the Melbourne University, held on the 25th ult,, it was resolved that the council may meet at some central place in the city, should it be found advisable, in future. It was also agreed that senate meetings should be held in May, August, and November of each year, besides meetings convened by the wardens. After a number of minor matters, such as manner of voting, no quorum, notice of meetings, &c., had been disposed of, additional orders for the regulation of procedure in the dealing with statutes and regulations, and in the conduct of the business of committees. These included the usual as to resolution, meetings, committee, and manner of procedure reassembling, &c.; the principal being, “That a committee shall consider such matters only as shall have been referred to it by the Senate.”

The Social Science Association will, this year, hold its annual Session in Dublin, in the month of October. The following are the subjects set down for discussion in the Education Department:—1, “What alterations is it desirable to introduce into the system of national (primary) education in Ireland ? 2. In what way may intermediate education be best promoted, under the Act of 1878 ?    3, How may the higher educa

tion of women be most efficiently advanced in Ireland ?” The programme

IS rather a meagre one, but as these subjects are all of local interest, others will probably be added later on ; and persons desirous of reading papers would do well to communicate with the General Secretary of the Association, at its offices in Adam-street, Adelpln, before the final arrangements for the Congress are completed.

I he schoolmaster has become iu his corporate capacity, as lie so frequently is in his individual, a connecting link between the various classes of society. He who breaks down the distinctions of caste, and bridges the chasm which separates class from class does good service to hi» country. Lord Mayor M'Arthur has been particularly active in this cause. His reception of the representatives of the National Union was a graceful recognition of their important functions in the State, and doubtless paved the way for the invitation of the Trades’ Union delegates to the Mansion House. We believe with the delegates that by his courtesy the Lord Mayor has broken a barrier between the various classes of the people, and has shown to the world that commerce and labour go hand in hand to build up the prosperity of the community. We gladly acknowledge that these are not the least of the services which the Lord Mayor has rendered during his year of office, and trust that other of our influential classes, in official and private life, will follow the example 60 well set by his lordship of seeking to unite all social grades.

We have received from Mr. M. L. Hutchiuson, bookseller, of Collms-street West, a choice assortment of Christmas and New Year’s Cards ; also specimen pages of cards containing British Birds printed iu oil colours, specially suited for school prizes. It is pleasing to note the advance which has been made this year in the adoption of art culture in the production of these popular season greeting cards.

A SUCCESSFUL dramatic entertainment was given by the Mitchell (Queensland) Dramatic Club recently, in aid of the local State school. A committee has been organised in connection with this school, whose business is it to raise funds for its enlargement. Subscriptions are solicited.


Sbzuntt anb gxi fossip.


We owe the following scientific item to Mr. Cosmo Newberry, of Melbourne :—A spray of sulphate of iron injected into a cavity where the smoke of nitro-glycerine compounds exist will decompose and destroy the noxious vapour.”    ,

A second electric railway is about to be opened in Berlin. The success of the first is quite up to expectation.

Me. W. T. Blandford, Fellow of the Royal Society, contests the theory that the great plain of Northern India is a great sea basin. It has long been taught that the great plain traversed by the Indus, Ganges, and Brahmapootra was once the basin of a great sea. Mr. Blandford says this large tract of land has been always above the sea level through a long period of geologic time.

Gulf streams and their effects weie descanted upon by Mr, D. S. Houghton, of York, recently. He said the gulf stream and its countercurrent, the Labrador current, produced important effects upon the climate of adjacent parts. The northern hemisphere was warmer than the southern from lat. 0 deg. to lat. 30 deg., and it was colder than the southern above 40 deg. The lighter temperature of the southern hemisphere in the temperate latitudes was explained by the existence of those gulf streams there, while there was only one in the North Atlantic, and a partial one through Behring Straits. The general effect of the gulf stream was to make the climatal range of temperature less. It had no effect upon summer heat, or upon the maturing of plauts and trees. The January temperature in the North Atlantic at 70 deg. were raised by the gulf stream, whilst the July temperature remained unaffected. The effect of the cold currents wdiich were indirectly caused by the warm ones, in preserving the proper equilibrium, was nothing upon the January temperatures. The effect of the cold water was to raise the July temperature, and leave the Jauuary untouched, and the influence of the warm made the January temperature moie warm, and left the July untouched.

In a paper on the origin of coal, Mr. E. Wethend, a geologist of repute, says that on land where coal was found grew the vegetation of the period—Kepidodendrous, Sigillaria, Calamities, &c. As the land sank and the water encroached the land vegetation was gradually washed away, but the roots remained in many cases, and those which offered the greatest resistance to decay arc the ones preserved in a fossil state— hence the occurrence of stigmaria. As the waters advanced the ground would become swampy, and reeds, mosses, and other vegetable products spring into life, and it is to vegetation of this kind that Mr, Wethend ascribes the formation of coal. He concludes (a) that coal was not found from vegetation of the Lepidodendroid type, and that therefore the stigmaria found in the underclays are not the roots of the vegetation which gave rise to the coal. (?;) That the varieties of coal, and the changes which take place in one and the same seam are not due to metamorphism, nor are they dependent upon the contorted state of the surrounding strata, but arise from the greater or less chemical decomposition of the vegetable mass, influenced by the circumstances under which it was submerged.

The South Kensington repository is being so crammed with archmolo-gical, antiquarian, and high art objects, that it is proposed to turn out the merely technical.

The value of coal and fuel exported from the United Kingdom during 1880 was £8,372,933 ; this is, however, but a small ratio of the total value of all the coal produced during that period.

A contributor to the Engineer has calculated that the proper num-


Corneille.


6. Translate—


ber of steps for a man to take per minute whose leg is 33 5 in. long is 64-7 ; that is, such a rate would be that at which he would walk with least effort. lie has based his calculations on the supposition that the leg swings as a pendulum, and taken into account the relation between the point of suspension and centre of pedal gravity.

Sulpho-carbonate, it appears, has a double value as a dressing to vines. It kills the phylloxera—or has done so ; and, solved in water, it acts as a manure.

The rusting of machinery may be prevented by the application of the following mixture as stated :—Dissolve half an ounce of camphor in one pound of melted lard ; take off the scum, and mix in as much fine black lead as will give it an iron colour. Clean the machinery, and smear with this mixture. At the end of 24 hours rub clean with a smooth linen cloth. It will keep clean for months.

Messrs. Crosby, Lockwood & Co., London, have published a useful work, which deals largely with the coal and iron industries of the United Kingdom, Mr. Richard Meade being the writer.

A most brilliant sable surfacing may be produced on iron and steel by applying a mixture of turpernine and sulphur, boiled together, with a fine hair brush. The turpentine evaporates, and leaves on the metal a thin layer of sulphur, which unites closely with the iron by heating over a spirit or gas flame.

The large deposit of ozokerine lately found at Gisbourne, New Zealand, is said to contain 50 per cent, of paraffin, 10 per cent, of kerosene, 10 per cent, of light oils, 20 of heavy oils, and 10 of earthy matter.

The docks of Liverpool cover 403 acres ; those of London 434 acres.

Two French scientists—De La Tour and Du Breuil—have recently devised an economical and quite new method of extracting sulphur from its associated ores. They utilise the chemical principle of elevation of the boiling point by allowing salt to dissolve therein. Chloride of calcium is the salt selected, on account of its fixity, cheapness, and complete inertia when in contact with sulphur and gangue at the temperature of 120deg., at which the process of extraction is effected. Two rectangular boilers are used, coupled and inclined ; and these are heated alternately from one hearth. The boiling liquid, containing 66 percent, of chloride, is introduced into one vessel previously filled with ore, and while liquation is taking place in it—which occurs after a lapse of a couple of hours—the other boiler, in which the operation is finished, may be emptied and charged afresh with ore. The chief advantages of the method are—small cost of extraction, great purity of sulphur, fusion possible throughout the year, and almost complete extraction.

alnibcrsitn .of JfUlkrttnxc.

The following are the test papers set for candidates for the Matriculation Examination, December, 1881, conducted by Mr. I. Warren Ball and Mr. H. J. L. Batten : —

FRENCH.—[For Pass.]

1.    Translate—

Néron se voyant autant d’esclaves que de sujets, ne consulta plus que le déréglement de son esprit insensé. On vit un empereur comédien, qui jouait publiquement sur les théâtres comme un acteur ordinaire. Il croyait même exceller en cet art. Il paraissait souvent sur la scène la lyre â la main, suivi de Burrhus et de Sénèque, qui applaudissaient par complaisance, lâcheté ordinaire aux philosophes de tous les temps, dont la froide morale ne tient pas contre la volonté d'un tyran. Lorsqu’il devait chanter en public, des gardes étaient placés de distance en distance, pour puuir ceux qui n’avaient pas cté assez sensibles aux charmes de sa voix.

2.    Parse every verb in question 1.

3.    Translate—

Maître Corbeau, sur un arbre perché

Tenait eu sou bec un fromage.

Maître Renard, par rôdeur alléché,

Lui tint à peu près ce langage :

Hé ! bonjour, monsieur du Corbeau !

Que vous êtes joli 1 que vous me semblcz beau !

4.    Translate into French—

How are you ? Pretty well, thank you.

How loug has he beeu at home ?

It is line weather.

He is seventeen years old,

What o’clock is it?

Have you any white wine ?

Is this the way to the Exhibition ?

He is a painter.

What news is there to-day ?

Come and see me this day week.

Do you know Melbourne?

I kuovv my lesson.

Bring me a wine-glass.

Take the milk-jug.

5.    Write—(a) the 3rd person sing, of the future absolute, the preterite

definite, and the imperfect subjunctive of s’en aller, finir, battre, rire, faire, savoir, vivre, connaître;

(b)    6 differently formed feminines.

(c)    6 differently formed plurals.

6.    Translate into French—

But the judgments of God are always filled with wisdom and mercy.

As soon as Victor came to himself, as if his nature had been entirely changed, he accused himself of having beeu the cause of all his misfortune : he confessed that he had deserved all that had happened to him, and from that day became the gentlest of beings. All those who approached him pitied him. One would wish to restore his sight, but one cannot prevent oneself from saying that he has gained much by losing the use of his eyes.

7, (a) When does the subject follow the verb ?

(b) When does the adjective follow the noun ?

For Honors.

1.    Question 2 in the pass-paper,

2.    Question    4 in the    „

3.    Question    5 in the    ,,

4.    Question    6 in the    ,,

5.    Translate into good English—

Eraste—“ Quoi, seule et sans Tircis ! vraiment c’est un prodige ;

Et ce nouvel amant déjà trop vous néglige Laissant ainsi couler la belle occasion De vous conter l’excès de son affection.”

Mélite— Vous savez que son âme en est fort dépourvue.

Eraste—“ Toutefois ce dit-on depuis qu'il vous a vue, Il en porte dans Pâme un si doux souvenir Qu’il n’a plus de plaisir qu’ à vous entretenir.”

Pymante—“ Pardonnez-moi la faute que j’ai faite Un berger d’ici près a quitté ses brebis Pour s’en aller au camp presque en pareils habits ;

Et, d’abord voui prenant pour ce mien camarade,

Mes sens d’aise aveuglés ont fait cette escapade.”

Corneille.

7.    Albert—“En quel gouffre de soins et de perplexité

Nous jette une action faite sans équité.

D'un enfant supposé par mon trop d’avarice Mon cœur depuis longtemps souffre bien le supplice:

Et quand je vois les maux où je me suis plongé Je voudrois à ce bien n’avoir jamais songé.

Tantôt je crains de voir, par la fourbe éventée.

Ma famillee en opprobre et misère jetée ;

Tantôt pour ce fils-là qu’il me faut conserver Je crains cent accidents qui peuvent arriver.”

Moliere.

8.    Give the dérivation of ten French words of Latin origiu in this paper.

LATIN.—[For Pass.]

(Do not abbreviate any latin ivord. )

1.    Translate—

Qualis ubi ad terras abrupto sidéré nimbus It mare per medium ; misersis, heu, praescia longe Horrescunt corda agricolis ; dabit illc ruinas Arboribus, stragemque salis ; met omnia late ;

Ante volant, sonitumque ferunt ad litora venti ;

Talis in adversos ductor Rhoeteius hostis Àgmen agit ; densi cuneis se quisque coactis Agglomérant. Ferit ense gravem Thymbraeus Osirim Archetium Mnestheus, Epnlonem obstruncat Achates Ufentemque Gyas ; cadit ipse Tolumnius augur,

Primus iu adversos telum qui torserat hostis.

2.    Parse the words in italics in question 1, giving the principal parts of verbs and the nominative and genitive singular of nouns.

3.    Translate—

His sunt arbores pro cubilibus : ad eas se applicant, atque ita paulum modo reclinatæ quietem capiunt. Quarum ex vestigiis cum est animadversum a venatoribus, quo se reeipere consuerint, omnes co loco aut a radicibus subruunt, aut accidunt arbores, tantum ut summa species earum stantium relinquatur. Hue cum se consuetudine reclinaverunt, infirmas arbores pondéré affligunt atque una ipsæ concidunt.

4.    In question 3 decline his, cubilibus, quarum, loco, species, pondéré ipsæ.

5.    Translate—

(a) L A te decurrit ad me haustus liquor.

2.    Hoc sustinete, majus ne veniat malum.

3.    Non is sum qui hoc faciam,

4.    Id hoc facilius eis persuasit.

5.    Quæ in eo repreheudat ostendit.

6.    Ad multain noctem pugnatum est.

7.    Quid sui consilii sit, ostendit.

(b. ) They asked the gods for a king.

You and I will go to the town.

It is reported that Cæsar will attack the town.

His sister lived at Rome.

He advises him to make forced marches.

I do not care a straw.

I fear that he will not do this.

6.    (a) Give the principal parts of pango, gigno, cerno, solvo, pendo,

hæreo.

(b) Write subjunctive pluperfect, 2nd, plural, of Nolo.

Passive, imperative, future, 3rd. plural of fero.

Infinitive, future of fio.

Subjunctive, perfect, 2nd, sing,, of fruor.

7, Explain with examples :—

Cognate Accusative,

Genitive of Valuation,

Double Dative,

Ablative of Measure,

Factitive Accusative,

Historical Infinitive.

HONORS.

1.    As in Pass-paper.

2.    „    „

0.    !>    ))

7.    „    „

8.    Translate freely :—

Delicta majorum immeritus lues,

Romane, donee templa refeceris,

Aedesque labentes deorunqet Foeda nigro simulacra fumo.

Dis te minorem quod geris, imperas :

Hinc omne principium, hue refer exitum.

Di multa neglecti deberunt Hesperiae mala luctuosm.

TranslateInter tree jam prmmissi Albam erant equites, qui, multitudinem trad :cerent Boman.

9. Legiones deinde ductac ad diruendam urbem. Quae ubi intravere portas, non quidem, fuit, tumultus ille, nec pavor, qualis captarum esse urbium solet; quum, eft'ractis portis, stratisve ariete muris, aut arce vi capta clamor hostilis et cursus per urbem armatorum omnia ferro flammaque miscet ; sed silentium triste ac tacita moes-titia ita defixit omnium animos, ut, prae mctu obliti quid reliquerent, deiiciente consilio, rogitantesque alii alios, nunc, in liminibus starent, nunc errabundi domos suas, ultimum illud visuri, per-vagarentur.

10.    Translate into idiomatic Latin :—

No sooner was it known that these towns, the latter of which is not two days’ march from Paris, were in the hands of the enemy, than that great capital, defenceless, and susceptible of any violent alarm in proportion to its greatness, was filled with consternation. The inhabitants, as if the Emperor had been already at their gates, fled in the wildest confusion and despair, many sending their wives and children down the Seine to Bouen, others to Orleans and the towns upon the Loire.

Note,—Bouen : —Botomagus. Orleans : — Genabum. Loire :— Ligeris.

11,    Translate:—

Quae est ista in commemoranda pecunia tua tam insolens osten-tatio? Soluene tu dives? Quid si ne dives quidem? Quid, si pauper etiam ? Quem enim intelligimus divitem ; aut hoc verbum in quo homine ponimus ?

ENGLISH.—[For Pass.]

1.    Make a complete analysis of the following passage:—“My chief companion, when Sir Roger is diverting himself in the woods or in the fields, is a very venerable man who is ever with Sir Roger, and has lived at his house in the nature of a chaplain above thirty years.”

2.    Thus nature gives vs (let it check our pride)

The virtue nearest to our vice ally’d :

Reason the by as turns to good from ill And Nero reigns a Titus, if he will.

The fiery soul abhorred in Catiline.

In Decius charms, in Curtius is divine.”

(a)    Parse the italicised words in this passage.

(b)    Explain the allusions in the last four lines.

(e) Refer to the context all subordinate clauses or phrases.

3.    Derive, giving root, prefix and suffix, the following words .—Invitation, discouraged, accompanied, jovial, behaviour, philosophic.

4.    Correct or justify :—

(a)    Neither of the books are in the room.

(b)    Who did you speak to about John and I.

(c)    This is the man’s hat whom I knew was expected to-day,

(d)    Clubs is trumps.

5.    Write the seventeen lines commencing—Pride still is aiming at the blest abodes.

(!. “ I distinguished myself by a most profound silence.” Write a brief redsume of the essay in which these words occur.

The surfaces of all bodies arc perpetually flying off from their respectiveffiodies one after another.” Refer this passage to the context,

7. Write a short essay on “ Temperance.”

For Honors.

1.    As in the pass-paper.

2.    As in the pass-paper.

3.    Write a brief description of Macbeth.”

4.    Describe the construction of the metre and quote six lines having peculiarities in the scansion.

5.    Comment upon and refer to the context :—

Fortune on his damned quarry smiling—

Except they meant to bathe in reeking wounds Or memorise another Golgotha,

I cannot tell :—

—Upon my head they placed a fruitless crown.

C, Quote Macbeth’s remarks upon hearing of the Queen’s death.

7. (a) Give the substance of Bacon’s essay upon Travel.

(b) Mention any obsolete usages therein.

{c) Explain and refer to the context Beware how in making the portraiture thou breakest the pattern.

EUCLID.

(Five propositions must be written out correctly.)

Book 1.

1.    The supplement (or exterior angle) of a triangle is greater than either of the other angles.

2.    The three angles of any triangle are equal to two right angles.

3.    If the square on one side of a triangle is equal to the sum of the squares on the other two sides, the angle contained by these two sides ia a right angle.

Book II:

4.    The square on the sum of two straight lines is equal to the squares on the straight lines and twice the rectangle contained by them.

5.    To divide a given straight line into two parts so that the rectangle contained by the straight line and one part shall equal the square on the other part.

G. To draw a straight line the square on which shall equal a given rectilineal figure.

Book III.

7.    One circumference of a circle cannot cut another in more than two points.

8. The angle at the centre of the circle is double the angle at the circumference upon the same base, that is, upon the same part of tho circumference.

9.    To bisect a given circumference, that is, to di vide it into two equal parts.


GEOGRAPHY.

1.    (a) State in which hemisphere is the greater rainfall, and why.

(b)    State in which hemisphere the trade winds are more constant,

and why.

(c)    State in which hemisphere the range of temperature is greater’,

and why.

2.    (a) Contrast the fauna of the Old World with that of the New.

(b) Describe the causes of two constant currents, one flowing approximately parallel with a meridian, and the other parallel with the equator.

3.    (a) Draw a rough outline of England.

(b) Show on the above map by dotted lines the counties on tho coast, give their names, their chief towns, and the rivers on which they are situated.

4.    Give the chief foreign possessions of Great Britain in each division of the world.

5.    Give any general information you can relating to Italy.

6.    Describe in order N. to S. the Zones of fauna and flora, with characteristic animals and plants respectively.


ARITHMETIC.

1.    Write down in words the quotient and remainder found by dividing nine thousand seven hundred and forty-three billions four hundred millions and thirty-seven by three hundred and three thousand millions and thirty-six,

2.    Reduce to its lowest terms :—

3.    Simplify.

(i+*)x(S + g)x(ft + i)xtf I ?)

(; + ?)x(:W)x(Yf£)x(* + f).

4.    Arrange in order of magnitude :—•§$•!, ■§£•£,

5.    Simplify 10.1ÒÌ x .40307 4-1.7.37 x 2.343.

G. Extract the cube root of 15G25000.

7.    A person borrows £50 to be repaid in twelve monthly instalments of £5, beginning one month from the date of the kum. What is the rate per cent of interest paid ?

8.    A man has 4000 trees to plant ten acres of ground with in parallel rows 12 feet apart. Plow many will there be over? (Take the ground as a square, and allow for the trees going along the edges.)

9.    Find the price of 4£ percent, stock, when the income from £1050 invested in it is £52 10s.

10.    Six sheep are worth 34 geese ; 12 sheep are worth one cow ; three cows are worth one horse ; 10 geese cost £4 17s Gd. Find the value of a pair of horses.


SCHEME OF ANALYSIS.

Sentence.

Kind of Sentence or Clause.

Distin

guishing

Letters.

Connective8 _ (if any) introducing Sentences.

Subject with Attributes.


Predicate.

Prime

Verb.

Subjective Complement with Adjuncts.

Object of Prime Verb, with Complement and Attributes.

Adverbial Adjuncts of Prod

Adverbial ¡’Other Adver' Objectives. ¡bialElxtonsion


Bktanun (Bimtniian guarirne«!


APPOINTMENTS.

Joseph Irvine, H.T., Bealiba, 749: JohnM'Leod, H.T., Caralulup, 974 • John Bardin, H.T-, Granito Flat, 243-4 ; Elizabeth Ponsford, H.T., Taripta East, 2184 ; Margaret Murray, 2nd Asst., Shepparton, 1469 ; John Quinlan, H.T., Moora, 1991; Wm. S. Best, H.T., Kororoit and Diggers’ Rest, 1933; Paul Shugg, H.T., Pearsontown, 1456 ; James Smith, H.T., Narioka, 2214 ; Peter J. Macnarnara, H.T., North Pannoobamawn, 1853; William Waldon, H.T., Uppor Moondara, —; Lydia Bennett, H.T., Muckleford, 402; Margaret J. Webber, H.T., Echuca North, 1780 ; Flora A. Marston, H.T., Major’s Line, 1798 ; Sarah Jane Flea, H.T., Wappentake Creek, 1841; Robert Dickson, H.T., Yarroweah, 2425; Samuel Bolitho, H.T., Wakiti Creek, 2298; George R. Hancock, H.T., Purnim, 1016; George R. Mott, H.T., Cape Bridgewater, 741; Fanny Duke, H.T., Bootapool, 1545 ; Margaret Walsho, H.T., Willowmavin, 2179; John W. Taylor, II.T., Shel-bourno, 950; John C. Corbett, H.T., Timmering East, 2177; Elizabeth Roberts, H.T., Hawkinston, 2431; Goorgo Wiigley, H.T., North Fitzroy, 1490; Arthur Sanders, II.T., Carlsruhe, 115; George Ick, Toolamba West, 1888; Caroline J. Rennie, H.T., Muckleford, 402; Edward F. Williamson, II.T., Jootho East, 2423; Joseph G. Haughton, |H.T., Newry, 2074; John Whitbourn, JLL.T , Rowsley, 2183; Martha L. Rutter, H.T., Granito Flat, 2434; Emma Anderson, II.T., Goldie, 1173 ; James Thomson, H.T., Drouin North, 2435; Susannah Hollands, H.T., Muckleford, 402; Henry J. H. Irwin, H.T., Parupa, 854; Robert Craig, H.T., Bruthen, 1141; Richard Nunwoek, 2nd Asst., King-street, 1689; Joseph W. H. Lugg, H.T., Lake Burrumbeet, 368; Finlay Matheson, H.T., Katunga, 2269; Henry Stielow, II T., Lake Meering, 2351; Joshua Ingamells, H.T., Hawthorn, 293 ; John J. Edgoose, H.T., Rosebrook, 526 ; Wm. J. Trembath, II.T., Kensington, 1146 ; Fanny Duke, H-T-, North Walmer, 2264; Emily Short, H.T., Strathbogie West, 2267 ; Lilian Lucas, H.T., Barrachee, 23-45 ; John M. Bardin, H.T-, Morwell North, 2439 ; Margaret M. McEvoy, H.T., Oxley Flats, 2347 ; Emma Fairhall, H.T., Barnawm South, 2254; Wm. H., King, H.T., Batesford, 1845; John II. Refshange, H.T., Loyola, 1953; Agnes Muir, II.T-, Black Range, — ; James F. Glennon, H.T., Gheringhap, 261,


CANON DANIEL ON MEMORY.

The following is a summary of the chief points treated of in a paper recently read by Canon Daniel, the principal of Battersea Training College:—

The rev. gentleman said that he would follow Latham’s classification of memories into the portative—mere mental carpet bags ; the analytical or mental pigeon-holes; and the assimilative or mental stomachs. Memories were classified according as they dealt with subjects. The portative memory was shown by its recollecting power, remembering merely the form, while it had not assimilated the subject. There was nothiug more in this kind of memory than the retention of the form. Teachers who trusted to this kind of memory nearly always made their pupils averse to the subject in hand. So in after-time very little remained but a few odds and ends of this and that—a kind of mental scrap-book, Modern teachers who used this kind of memory urged that they did so only as a foundation. But an unsound foundation affected the whole superstructure. Inspectors sometimes made the mistake of examining children only as to the form of knowledge they had gained. The test should show whether the pupils had thought about the matter for themselves. The analytical memory was one that arranged the knowledge for convenience of reproduction. It was pigeon-holed on a plan, and the possessor held the key. Teachers of this plan often did the analysis for their children, and so left no room for the exercise of their pupils’ powers. This plan could not be used without reason. Knowledge must be methodised as it was acquired. We must know what pigeon-hole each bundle was in. Bad memories were often the result of a muddled acquirement of knowledge. The assimilative memory worked knowledge into the mind until it was a part of it. It digested it into a permanent and living force. Teachers should turn the knowledge they imparted round and round in new lights and combinations, so as often to repeat their lessons under the guise of novelty. Looking at pictures, plans or models, and drawing or making sketches of what was seen, all helped to prolong the attention on a subject without weariness. In using each of these kinds of memory we must remember that the portative memory, like a carpet bag, might be full of holes; that the analytical memory enabled us to recover what was lost, but that the assimilative memory did not allow knowledge to be lost, because it was a part of the possessor. He then passed on to the work of the memory, and said that impressions needed time for lodgment in the mind, and so should be prolonged till they had taken root there. This was not effected by bare repetition, which did but produce satiety. A divided attention weakened the impression of each of its ideas, and so teachers must remove the causes of distraction by offering to their pupils greater and more powerful attractions. Again, attention was automatic or volitional, a very important distinction ; for volitional attention in young children was very weak. So they especially needed powerful attractions appealing to their senses to win their attention from the number of counter-attractions which solicited their will. With increasing years children’s volitional attention increased, and so intellectual attractions were required which appealed more to their sympathy. The habit of attention was only formed by systematic use of it. Very young children could not be much influenced by high motives of duty, truth, or economic advantage. They lived chiefly in the present, and so must be approached through°it. Properly conducted examinations were considered powerful quickeners of


attention. Every task set should be carefully examined and appraised. Examinations -were only mischievous if they produced distraction. They strengthened the roots of knowledge when properly conducted. “A child who knew his work entered the examination like a war-horse who scented the battle from afar.” Speaking of examinations,generally, he said that teachers disliked them on account of there being so much work to do in so short a time. The Times newspaper had said, “ What kills is not work, but worry.” He would also add, “ Work per se and too much of it.” The reader said that an attitude of attention favoured the exercise of attention, and so good class-drill was useful. Proper safety-valves must be provided or the children would be very inattentive, though seemingly very attentive. Emotional excitement was a pow'erful aid to attention, for if pleasurable we willingly prolonged it, and were glad to recur to it ; while if painful it compelled us to attend to it. He lastly touched on the classes of association by which matter was reproduced, 1. Associations of contiguity which were low in order and possessed by the beasts. 2. Associations of Resemblance and Difference, which were more valuable as the mind must compare, discriminate, reason. 3. Associations of Cause and Effect, which were valuable, especially in the teaching of science, if we guarded against the fallacy of Post hoc ergo propter hoc. The Canon was listened to for nearly an hour with great atttenion, and very loudly applauded at the conclusion of his paper.

A VISIT TO THE KINDER GARTEN SCHOOL AT SOUTH YARRA.

The merits of the Kinder Garten system received lately in this paper ample consideration from the pen of Miss Wilhemnia Rule, a lady skilled in the matter, and it may well be asked wby Frobel’s principles, which have found such favour with advanced educationalists, have not yet been applied to the State course of training, so as to give every teacher who may be charged with the management of infants an insight into the principles of a system that can do so much to strengthen and develop the mind of early childhood. Happily, our contributor lias for some time past been giving to the public an able and a practicable exposition of the merits and advantages of Frobel’6 teaching. In Walsh-street, South Yarra, Miss Vaughan, a lady who has studied the system at home, is now carrying on in full operation a Kinder Garten school ; and from the happy faces and interested looks of her little protégés, there can be no doubt of the success of her work in leading children to the domain of knowledge by the path of pleasurable exertion.

The special studies consist mainly of a rapid way of teaching reading by a phonetic plan, and of teaching writing or drawing by tracing simple patterns on copy-books ruled in small squares, thereby leading to a ready command of pen or pencil ; also lessons on form and color are given, besides practice in the folding or plaiting of colored papers, building and designing with toy bricks, and even the simpler kinds of modelling in clay. In fact, such tasks are undertaken as will please the taste of children, while being skilfully designed to train their thinking powers, and to develop a taste for construction ; and in harmony with this purpose it should be noted that play and physical exercise find a fitting place in the actual school course.

A taste for reading will, with average opportunities, make a man a scholar, while a taste for school is at least likely to lead to rapid progress in the path of knowledge ; and from the pleasant associations mingled in this system with a young child’s school course, the happiest results may be expected. Learning with little effort, taste for steady application and mental flexibility may thus be easily promoted, while a strong foundation will be laid, and a preparation effected with the plastic disposition of early childhood that will facilitate study and lead to successful effort at a more advanced period.

We have reason to think that the parents have to thank Miss Vaughan much for doing a good work, the fruits of which will be neither few nor unimportant ; and, while hoping that many children may be brought within the sphere of that lady’s amiable and beneficial influence, we earnestly hope that not only other Kinder Garten schools may be opened ; but also that the benefits of this excellent system may not be confined to children of the well-to-do classes, but that others of humble station may likewise share in its advantages and privileges.

PUBLICATIONS RECEIYED.

We have received a copy of the Schoolmaster Waltz, published under the nom de plume of State school 1420. This is a graceful composition, and of more than average merit. It has been well printed by Messes. C. Troedel tc Co., and can be obtained from all the leading music sellers in Melbourne, Ballarat and Geelong.

Mr. M. L. Hutchinson’s “ Australian Almanac and Educational Register” for the year 1882 is to hand. This useful publication has now reached its twenty-third year, and certainly exhibits no signs of decrepitude. The list of teachers has been revised up to a later date than that of the Education Report, which will make it of special value to State school teachers.

Mb. M. L. Hutchinson, bookseller, Collins-street West, has forwarded us specimen copies of an additional shipment just landed of Messrs. Blackie and Son’s Comprehensive School Series, consisting of : The Elementary Historical Reader, pt. I. and II. ; The Geographical Reader, pt. IV.; The Prologue to the Canterbury Tales ; The Lady of the Lake; Geography, Scotland and Ireland; and the Home‘Lesson Book I.

I.


WARREN BALL’S “Hints to Candidates for Teachers’ and Matricur lation Examinations,” Is.; posted, Is. Id. Mullen, Melbourne.


GOVERNMENT


ADVERTISEMENTS.


c


ANDIDATES for EXAMINATIONS prepared by correspondence or otherwise. I. Warren Ball, South Yarra.


H


EAD TEACHER, allotment 30 x 50, results 82, desires exchange bet ween Belfast and Colac. “ Novo,” Post Office, Warrnambool.


M


ALE ASSISTANT, Melbourne, wishes to exchange (after Christmas) with Assistant, Geelong. Address, “ Lingua,” G.P.O., Melbourne.


HEAD TEACHER, allotment 30 x 50, rising township, agricultural district, worktnistress position vacant, desires exchango with another in suitable locality. Address—“Doceo,” Schoolmaster Office.

HEAD TEACHER, country school, allotment 30 x 50, Western district, desires exchange. Gippsland or Ballarat preferred. Address—“J.S.,’ Schoolmaster Office.


HEAD TEACHER, Western District, allotment 20 x 30, four rooms, four acres land attached, wishes exchange with Head Teacher or Assistant Geelong or Melbourne. Address, A. Z. Macarthur.


HEAD TEACHER, 8 miles from Euroa, allotment 20 x 30, 76-81 per cent. Coach passes every day mile from the school, would exchango with an assistant in a large town, (sea-sido preferred). Address, “ Diana,” Euroa.


Education Department, Melbourne, 12th Novembor, 1881. SCIENCE.

AN EXAMINATION of TEACHERS in the different sciences enumerated below will be held at school No. 391, Spring-street. Melbourne, on Friday and Saturday, 16th aud 17th December, 1881, commencing at 9 30 a.m. : —

Physiology

Botany

Geology aud Mineralogy Magnetism and Electricity Acoustics, Light and Heat Mechanics and Hydrostatics Chemistry Metallurgy.

Only teachers who are recorded as holding a certificate of competency under the department, or who shall have passed in all subjects except Time Table and Collective Lesson, with notes thereof, and Class Drill for such classification will be permitted to attend this examination.

Notice of intention to attend, specifying the subject intended to be taken up, addressed to the Secretary, Education Department, Melbourne, must be received on or beforo the 30th November, 1881.

T. BOLAM,

Acting Secretary.


AN ASSISTANT ;n town, salary £114*, wishes to exchange with Head Teacher in country in a better position, Apply, “Anser,” General Post Office.

EAD TEACHER, near Melbourne, allotment 30 x 50, results 90, wants exchange. Vacancy for Workmistress. Address, Borung, G.P.O.,


H


Melbourne.


HEAD TEACHER, 30 X o0 school, Shcpparton district, 5 miles from Railway station, wishes to exchange with Head Teacher in South Gippsland. Address, Exchange, Shepparton.


HEAD TEACHER, 23 miles from Melbourne, daily coach, wants exchango with assistant in Town or Suburb. “Desirous,” care P. Matthews, Esq., 52 Collins-street.


HHEAD TEACHER Country School, Salary £132, residence free, desires to exchange with any assistant in Melbourne or Suburb. Address G. Wylie, Esq., 67 Drummond street North, Ballarat.


HEAD TEACHER, 20 to 30, good chance for 30 to 50, results 88-235) wishes exchange assistant Melbourne or Suburbs. Lower income accepted. “ M.B.,” Natimuk.

A CARD.—A COUNTRY TEACHER, through failing health, wishes to exchange for a cooler climate. Assistantship in a town preferred.

No. of children on roll, 60. Department allotment 30 x 50, building new, opened this year, 30 x 20 feet, four-roomed dwelling attached.

Shooting, fishing, boating, bathing, hunting, riding, cricket, and permanent water. Every inducement. Salary about £165. Postal address, <l Rubicon ” Cherry Tree, Big Hill, Sandhurst.


Punch Almanac, 1882


Will

be

Published


6tii DECEMBER, 1881. 6th DECEMBER, 1881.


Will

be

Published


Punch Almanac, 1882.


ALEX. M'KINLEY & CO., PUBLISHERS 61 QUEEN ST.


GOVERNMENT


ADVERTISEMENTS.


Education Department,

Melbourne, 12th November, 1881. GYMNASTICS.

AN EXAMINATION of TEACHERS desirous of becoming qualified to IMPART INSTRUCTION in GYMNASTICS will be held at the Central school, No. 391, Spring-street, Melbourne, on Thursday and Friday, the 8tli and 9th December, at 9.30 a.m. The practical portion will be taken on the 8th, and the theoretical on the 9th.

Notices of int ention to attend this examination must be lodged at this office not later than Wednesday, the 30th November, 1881.

T. BOLAM,

Acting Secretary.


Education Dopartmont, Melbourne, 12th November, 1881.

A N EXAMINATION of TEACHERS and CANDIDATES for EMPLOY-J_A_ MENT as teachers in State Schools, will be held at school No. 391 Spring-street, Melbourne, on Thursday and Friday, loth and 16th December’ 1881, commencing at 9.30 a.m. This examination will bo for certificate of competency only.

Candidates will be required to give notice of their intention to attend addressed to the Secretary, Education Department, Melbourne. All such notices must be received on or before the 30th November, 1881.

All candidates, other than teachers, assistants, first-class pupil teachers and workmistresses actually employed in State schools, desirous of attending the examination, will be required to pay a foe of 10s. beforo being examined.

Satisfactoiy evidence that each candidate, if a male, is upwards of eightcon and, if a female, upwards of seventeen years of age, with certificates of sound health and moral character, must accompany each notico to attend examination.

T. BOLAM.

Acting Secretary.


Education Department. Melbourne, 12th Novooibcr, 1881. DRAWING.

AN EXAMINATION of PERSONS desirous of qualifying thorn selves to 1EAC.FI DRAWING in State schools will be held at the Central school, No. 391, Spring-street, Melbourne, on Wednesday and Thursday the 7th and 8th Deocmber, at 9.30 a.ra.

Candidates not employed in State schools will be required to pay a fee of 10s previous to being examined.    '

Notices of intention to attend this examination must bo lodged at this offic# not later than Wednesday, the 30th November, 1881.

T. BOLAM,

Acting Secretary.

Education Department, Melbourne, 12th November, 1881. MUSIC.

A N EXAMINATION of PERSONS desirous of qualifying themselves to A— TEACH MUSIC in State schools will bo held at the Central school, No. 391, Spring-street, Melbourne, on Monday and Tuesday, the 5th and 6tll December, at 9.30 a.m.

Candidates not employed in State schools will be required to pay a fee of 10s previous to being examined.

Notice of intention to attend this examination must bo lodged at this office not later than Saturduy, the 26th November, 1881.

T. BOLAM,

Acting Secretary.

Education Department, Melbourne, 14th November, 1881 MILITARY DRILL.

AN EXAMINATION of TEACHERS desirous of qualifying themselves to impart instruction in Military Drill will bo hold at School No. 391 Spring-street, Molbourne, on Monday, the 12th Decomber, 1881, commencing at 9.30 a.m.    h

The examination will bo a written one, and will comprise—

1. Squad Drill, at intervals.

2. M.arching.

3. Squad Drill in single rank 4. Squad Drill, in two ranks.

5. Company Drill.

Candidates passing in the written examination, as abovo. will be roquired to undergo a practical test in addition thereto before becoming fully qualified as instructors in drill.

Notices of intention to attend must he received at this office not later than Monday, the 5th December, 1881.

T, BOLAM,

Acting Secretary.


rp ATE’S PARCELS POST EXPRESS

FIXED PRICE.

NO EXTRAS. NO TROUBLE.


Delivery to door at any address in

21b

s. d,

41b. j 61b. s. d. s. d-

101b. s. d.

201b. s. d.

Great Britain

4 6

6 0 7 6

10 0

14 0

Continent of Europe, America, & Canada

7 6

9 612 0

10 0

21 0

Sydney, Hobart, Launceston

2 6

3 0 3 6

4 0

G 0

New Zealand Ports (except West Coast) Adelaide, Brisbane.

4 0

4 G 5 0

5 6

7 6

No further charge whatever. Very small increase for heavier weights. Delivery at country addresses in Australia, inland carriage only added.


Receiving office—

FREDERICK TATE, 13 Market-st., Melbourne TALK’S I'AKCKLK POST EXPRESS. DELIVERY at DOOR any address in the world. EITHER to or from Britain from 4s. 6d. TO or from other Australian ports from 2s. 6d. NO further C H A RO E whatever. No trouble. Any SIZE, weight, or shape.

EVERY possible ASSISTANCE afforded. INQUIRIES plainly answered.

Henceforth Parcels handed to~W, R. SUTTON’S Branches, every town in Great Britain, delivered in Melbourne at nearly similar rates.

Head Office :—

FREDERICK TATE, CUSTOMS AGENT, &c., 13 Market street, Melbourne.

QOMPETENCY    EX A MINATION.

T U I T I 0 N—

IN CLASS, BY CORRESPONDENCE, OR PRIVATELY.

Other Work, by Arrangement.

JAMES L. ROBERTSON, B.A., 71 CLABEXDO?f-8T., EmEBALD HlLL.


j^LEX. MCKINLEY •& CO.,

GENERAL

PRINTERS AND PUBLISHERS, 61 Queen-street, Melbourne.


In Bookwork and General Publishing our large experience is a guarantee of all work being executed in the best stylo, while having a first-class stock of the best and latest material ensuros expedition and good workmanship.


The following papers aro issued from this office WEEKLY.

. “Punch,”    “Bulletin,”

“ Willing Work,”    « Once a Week.”

FORTNIGHTLY.

“ Australian Law Times,” “Jewish Herald,”

MONTHLY.

“ Schoolmaster.”    “ Monthly Messenger.”

BUY AND READ

THE NEW    WEEKLY PAPER.

Price Threepence.

“ONCE A WEEK,”

16 PAGES.    PRICE THREEPENCE.

ALL BOOKSELLERS.

Alex. M‘Kinley and Co., Publishers, 61 Queen Street, Melbourne,


MULLER’S MATRICULATION MANUALS.

POPE’S ESSAY ON MAN, Epistles 1 to 3; and TEN ESSAYS of ADDISON, from the “Spectator.” In One Volume, Crown 8^0, Limp. Is.

POPE’S ESSAY ON MAN, Epistles 1 to 3; and TEN ESSAYS of ADDISON, from the “Spectator.” Prepared with Notes for the use of Candidates for the Matriculation Examinations at the Melbourne University. By E. E. Morris, M.A. In One Volume, Crown 8vo, Cloth. 3s.

VIRGIL’S AENEID, Book XII., and CiESAR,’S GALLIC WAR, Book VI. Prepared with Notes for the use of Candidates for the Matriculation Examinations at the Melbourne University. By James Clezy, M.A. Second Edition, Revised. Fcap. 8vo, Limp. 2s.

VIRGIL'S AENEID, Book XII., and CTESAR’S GALLIC WAR, Book VI. Translated by James Clezy, M.A. Ecap. 8vo, Sewed. 2s. 6d.

FIRST BOOK OF AUSTRALIAN BOTANY. Specially designed for the Use of Schools. By W. R. Guilfoyle, E.L.S., C.M., R.B.S., London ; Director of the Melbourne Botanical Gardens. Illustrated, Crown 8vo, Sewed. 2s. 6d.


O c. EXAMINATION.

TUITION BY CORRESPONDENCE.


MR. THOMAS BOARDMAN, First-class Honor-man of the Denominational School Board, Prepares Teachers for the Certificate Examination by Correspondence. Terms moderate.

Addbess—

4 5 PRINCES STREET, CARLTON

TAMES CLEZY, M. A. ,

<0    MELBOURNE.

CLASSICAL & SHAKSPEARE SCHOLAR (1869)

PREPARES CANDIDATES For Matriculation (pass or honours), and for the subsequent Degree Examinations of the University.

Course of Lessons by Correspondence in Latin Grammar, Translation, and Composition.

Terms on application personally or by letter. 5 GORE STREET, FITZROY.


S-


MULLE


S


New Classified Catalogue of School, College, and Technical


EDUCATIONAL WORKS

May be had gratis on application, or posted on receipt of address.


SAMUEL MULLEN,

Wholesale & Retail Bookseller & Stationer, 29 & 31 COLLINS ST. E., MELBOURNE.


Price One Shilling,

By Post—In Victoria, Is. Gd. ; Out of Victoria, 2s


D


EPARTMENTAL


EXAMINATION


TEACHERS.


OF


TEACH ER S’ GUIDE

TO

AUSTRALASIAN EXAMINATIONS.

Containing the


NOTICE.

MR. H. M'KTNLEY

gOLICITOR, CONVEYANCER,

And

PROCTOR,

Has Removed to

86 COLLINS STREET WEST.


PROGRAMMES & EXAMINATION PAPERS

of

Victoria    South Australia

New South Wales    Auckland, N.Z.

Queensland    Wellington, N.Z.

And Tasmania.

108 PAGES WITH STIFF COVER.

The above book contains the Programmes and Examination Papers of December, 1877, of all the colonies, and is reduced to the low price of

ONE SHILLING.

By Post—In Victoria, Is. 6d.; Out of Victoria, 2s

ALEX. MTiINLEY & CO., PRINTERS AND PUBLISHERS, 61 QUEEN STREET,

MELBOURNE,


rpO HEAD MASTERS, SECRETARIES X OF BOARDS OF ADVICE, AND OTHERS.

ALEX. MTvINLEY & CO.,

Having made considerable additions to their stock of Bookwork and Jobbing Type, are prepared to execute orders in every description of

GENERAL PRINTING.


All orders entrusted to them will be printed in a satisfactory manner.


Alex. M'Kinley & Co., Pbintees, 61 Qhben-st.


NOW READY.

ILTON    PARSED.

PfilCE 2S.

By J. J. BURSTON,

(Author of “State School Arithmetic”).

Also Ready, the Fourth Edition of the

STATE SCHOOL    ARITHMETIC'

By

JOHN J. BURSTON,

North Sandhurst State School.

Printed and Published by Alex. M‘Kinley & Co., 61 Queenstreet, Melbourne, under the auspices of the Victorian Teachers Union.


AND LITERARY REVIEW.

Yol. Ill, No. 30.


DECEMBER, 1881.


Subscription

Yearly, 6s. 6d. ; Half-yearly, 3s, 6<]


BLACKIE

Comprehensive

& SON’S

School Series,

Popular Edition, Id. each number. the Numbers.

9.    Sentences.—Two lines on each

Page.

10.    Plain and Ornamental Let

tering.

11.    Exercise Book.—Wide Puling

with Margin.

11*. Home Exercise Book.—Same as No. 11, but 8vo size. Price Id.

12.    Exercise Book. — Puled in

Squares.

12£. Home Exercise Book.—Same Puling as No. 12, but 8vo size Price Id.

13.    Exercise Book.—Puled for

Book-keeping.

14.    Essay Book,—Puled for Com

position, &c.

15.    Exercise Book for Begin

ners. Ruled for Small Text. X. Copy-Book Protector and Blotter. Keeping the Books Clean. One Penny.


THE COMPREHENSIVE READERS.


“ As specimens of good reading books, they cannot be excelled. The exercises are carefully arranged so as to suit the meanest capacity, and at the same time hir»e a tendency to make useful impressions on the minds of young scholars."—Educational Guide.


Adopted by the London, Liverpool, Manchester and other School Boards, and by the National Board of Education in Ireland.

VERE FOSTER’S WRITING COPY-BOOKS

The efforts of Publishers to provide teachers with the means of properly framing their pupils in the Art of Writing deserve the highest commendation; and no names stand higher in this depai tment of scholastic work than Vere Foster and the Messrs. Blackie."—Educational News.

Opinions of H.M. Inspectors in Education. Blue Book Reports.

Mr. M'Callum. 1 , U.M. Inspector, says— (

‘ More progress is made by Vere Foster’s than by any other method which has come under my notice.”

Mr. Brewer, ( , II.M. Inspector, says— (

‘With books like Vere Foster’s there is no excuse for the slovenly no-style of writing too often presented to me.”

Mr. W arburton, f ( U.M. Inspector, says— (

I wish that the use of the excellent copy-books such as Vere Foster’s was begun earlier and more persisted in.”

Mr. Newell, l ( II.M. Inspector, says(

‘ I know of no series by means of which children can be so quickly taught to write with freedom and legibility.”

Rev. J. Lomax, f t U.M. Inspector, says— (

‘ The introduction of Vere Foster’s copy-books in some of my schools has been attended with marked success,”

Superior Edition, 2d. each number.

Contents of

1.    Strokes, East Letters, Short

Words.

l-£. Long Letters, Short Words, Figures.

2.    Long Letters, Short Words,

Figures.

2fc. Words or Four, Five, or Six Letters.

8. Capitals, Short Words, Figures.

8$> Sentences op Short Words.

4.    Sentences. Mostly composed of

Short Words.

4$. Select Quotations from Shakespeare.

5,    6. Sentences—Maxims, Morals,

and Precepts.

5$-. Sentences, in Writing of Three Sizes.

6£. Sentences, in writing of Two Sizes.

7.    Sentences, and Christian

Names.

8.    Sentences.—One Line on each

Page.

Each Book is Illustrated in a highly instructive and artistic manner.

Primer I.—32 pp.,..................paper cover, l.Jd. ; cloth cover, 21 d.

Primer II.—48 pp.,.................. ,,    2d.;    „    3d.

Primer Complete—80 pp.,............................. „    43.

Header I.—64 pp., (Abridged Edition,) ............. ,,    -id.

Reader. I.—96 pp.,....................................... cloth boards, Od.

Reader IL—128 pp.,...................................... „    gd.

Reader III.—196 pp...................................... „    Ls.

Reader IV.—288 pp., ....................................    la,    Gd.

Reader V—320 pp.,....................................... ,,    ’    2s.

Reader VI.—384 pp.,...................................... 2s. Gd.

ADDITIONAL READING BOOKS.

The Newspaper Reader, Selections from the Leading Journals of the Nineteenth Century on events of the day. By Henry Findlator Bussey, and T. Wilson Reid. Foolscap 8vo, cloth boards, 288 pp. price 2s.

The British Biographical Reader, Sketches of prominent Men by Eminent Authors. With Introductory Notes by the Editor. Illustrated by numerous authentic Portraits. “ Foolscap 8vo, cloth board-288 pp., price 2s.

The Shakespeare Reader, being Extracts from the Plays of Shakespeare, specially selected as fulfilling Article 28 and Schedule IV. of the Education Code. With Introductory Paragraphs and Notes Grammatical, Historical and Explanatory. Foolscap 8vo, 160 pp. cloth, ls.

Myths and Legends of Ancient Greece and Rome. A Hand book of Greek and Roman Mythology, for Schools and private Students. By E. M. Borens. Illustrated from Antique Sculptures, Eoolscap 8vo, 330 pp., cloth 3s.

Poetical Reader, for the use of Elementary Schools in England and Scotland. Foolscap 8vo, 224 pp., cloth ls. 6d.

London, Past and Present. a Reading-book for Elementary Schools. With numerous authentic Illustrations. Foolscap 8vo, 288 pp., cloth 2s.

The Scottish Historical Reader; being Extracts from Eminent Writers Descriptive of Events in Scottish History, with Introductory notes. This volume will supply a fuller account of leading events in Scottish history than can bo embodied usually in School histories, and being composed chiefly of extracts from the works of approved writers will form excellent reading lessons. It is adapted more especially for the requirements of the Fourth Standard.

HOME LESSON BOOKS.

The object of this scries is to place in the hands of the Pupil a certain amount of Standard work, which will enable him to grasp the leading facts of English History, Geography, and Grammar, and at the same time lighten tho work of the Teacher. The questions in Arithmetic are mixed in order to test the pupil’s knowledge.

M E L B O U R N E,


Further Particulars and Catalogues from All Booksellers,

Or their Representative, GEO. STILLIE, care of Messrs. COWAY & Co,,

WHOLESALE STATIONERS,


tffoal g epuri ment.

LONDON SCHOOL BOARD.

Mr. Edward North Buxton, at the meeting of the London School Board, on 6th October, made the following remarks from the chair :—

A few weeks before we separated for the recess, I invited from you an expression of your wishes as to whether I should maintain the custom, introduced by my predecessor, of making an annual statement of our work. A considerable majority were in favour of my continuing the practice of recording our progress, and of noting the most important matters that have come to the front during the year. I need hardly say that this entirely accords with my own inclination, and I have much pleasure in presenting for your recollection, and for the information of those who send us here, a brief account of our operations during the past twelve months. In doing so I shall, as far as possible, coniine myself to facts carefully ascertained, and, where I have to draw my inferences fiom them, I wish it to be understood that the opinions indicated are my own opinions, and that no one is committed to them but myself. I feel sure that the most prominent thought which occupies our minds on this day of our re-assembling for another year’s work is of the heavy personal losses which we have sustained since the corresponding day last year. Both Sir Charles Reed and Mr. Rogers were unceasingly occupied with the details of our committees almost up to the day of their deaths, and if their unselfish devotion to School Board work did not, as some believe, shorten their lives, at least it may be truly said of them that they died in harness. It is a satisfaction to us to hope that we shall before long possess a valuable presentment of their well-remembered features in two busts, which will be placed in this room, but a more lasting memorial of these good men exists in the impress which each of them has made upon the character of our work ; the one by his wise leadership and foresight, the other by his enthusiastic and unremitting toil, and both of them by the “ infinite capacity for taking pains ” which is said to amount to genius. The work which is transacted in these offices is so varied and multifarious, that from the first we have found “ division of labour ” necessary, and in reviewing our operations it is convenient to considei them under the headings of the departments which are controlled respectively by our seven standing committees. Ihese are Statistics, Works, School Management, Compulsory Bye-Laws, Industrial Schools, Educational Endowments, and Finance.

STATISTICAL COMMITTEE.    _

The Statistical Committee, whose business it is to ascertain the number of children of school age in a given area, the number of school places already provided, and—after making the necessary deductions for illness and other causes—to recommend to us the new schools that may be necessary to provide for the balance, stands first in the natural sequence.

CENSUS of 1SS1.

The most important event of the year in this department has been the national census, taken in the spring of the year, to which we have looked forward as an assured basis on which to found our estimates. In 1871 the Registrar-General supplied the Board with particulars, from the census of that year, of the name, age, and address of every child between three and thirteen, and, by a subsequent house to house visitation, we supplemented this information with the school, if any, that the child was attending, and the reasons for non-attendance. We were thus armed with a thoroughly accurate record of the numbers for whom school accommodation must be provided ; and for some time we have hoped to be furnished in the present year with the same important aid to our work and invaluable check upon the unofficial census which is taken annually by our visitors. This boon has, however, been refused to us. When itis remembered that the visitors have frequently to encounter suspicion and reticence from the parents in completing their schedules, and that an error of even one per cent, may mean an excess or deficiency of school supply representing a capital charge of over £100,000 on the ratepayers, it is to be hoped that this decision is not irrevocable, and that the assistance which it is in the power of one public body to give to another will not be withheld. Even if there are reasons of sufficient weight to prevent our having access to the records of the children’s names and ages, these do not apply to furnishing us with the totals of such children included in each of the blocks into which it has been found necessary to divide our district for statistical purposes. For the present, however, we have to content ourselves with such general information as is contained in the report of the Registrar-General and the inferences which we are able to draw from it for ourselves. The gross result for London of the census of 1881 is that it has increased by 565,454 persons during the past decade, and now numbers 3,832,441. That is to say, it exceeds in population the total of the nineteen largest provincial towns. The Registrar-General writes ‘ The population of London has almost exactly doubled itself in the course of forty-one years, whereas the population of the rest of England and Wales has taken fifty-seven years to multiply in an equal degree. The metropolis has thus being gaining its proportions as compared with the country at large ; and, whereas at the beginning of the century out of ten inhabitants of England and Wales one lived in London, the proportion has now risen to one out of seven.” The meaning to us of the above figures is, that even if we had already met the necessities of the case, there are, taking tjie proportion of one-sixth allowed by the Education Department, about 10,000 children of school age requiring elementary school places added every year, and that the ratio of growth increases every decade. In 1871 the Board found that in London the proportion of children between three and thirteen years was 20-86 per cent, of the total population, as compared with 23"58 per cent, in England and Wales. Applying this reduced proportion to the new census/ we arrive at a total of 799,447, But in 1871 we found, by careful inquiry in every case, that a further deduction of, on the whole, one-seventh must be made for those over the ninpenny class. Making this deduction from the present figures, we find that the total number of chidl-ren between three and thirteen now to be provided for is 685,240. If we now compare this total with the number tabulated by the visitors in their last annual scheduling we find that they entered in their books the names of 658,272 as actually requiring school accommodation, showing a deficiency from the calculated number of 3-8 per cent. The Board will remember that in estimating the accommodation needed in any district we have been in the habit of adding ten per cent, to the number shown by the visitors’ schedules, believing, that owing to the shifting character of much of the population and other causes, that proportion escaped notice. But the figures given above seem to show that the visitors’ approach much more nearly than was supposed to the real numbers—a result which, considering the difficulties which they have to encounter, is very creditable to them. It must, however, be remembered that these figures leave altogether out of account nearly seventy thousand children between thirteen and fourteen, a large number of whom now fall under the operation of the bye-laws. We have added during the year twenty-five new schools, accommodating 25,393 children, which brings up the provision in Board schools to 236,024, and in all efficient schools to 502,095. Thus Mr. Mundella did not overstate the case when he said, a few weeks ago, in presenting his annual budget, “ In London great deficiency of school accommodation still exists.” The Board have already decided to build nearly 100,000 additional school places to meet the deficiency, many of which places are now rapidly approaching completion.’ From the above statement it will be seen that the functions of the Statistical Committee cannot cease at present; and though we may look forward to some future time when, the ground being covered with houses up to the limits of the metropolitan area, the population of London cannot continue to increase indefinitely, and when it may be for those who come after us to hold their hands, it cannot be so for us. The movement of the population from the centre to the outskirts of London is more marked in the present census than in any former one. The central area, comprising the resignation districts of St. George’s, Hanover-square, Westminster, Marylebone, St. Giles Strand, Holborn, London City, Shoreditch, Whitechapel, and St! George’s-in-the-East, has decreased 7'8 per cent, in the past ten years! It may be thought that this rapid diminution has rendered some of the school places provided by us superfluous. We found this area, however, fairly well supplied, and we have built comparatively few new perma! nent Board schools within it. Of these there are 45, accommodating 36,723 ; and they have on the rolls 39,351. No doubt some of the volum tary schools in this central district have suffered, but this diminution is clearly the result of a migration over which we have no control, and should be borne in mind in considering the attendance at voluntary schools over the whole metropolis.

In considering the larger district outside of this limited area, we find a very different state of things. The wider zone has increased in the past decade at the rate of 27-6 per cent. I transcribe from the Registrar-General’s report a table showing the rates per cent, of increase of each of the registration districts comprised in the outer belt, from which it will be seen that the further we move from the centre the moro rapid is the increase.

Districts inside of Central Area,

Inc

1861—71.

rease per ce 1871—81.

rnt.

1861—81,

Kensington ... ...

4S-8

24-4

85-1

Fulham ... ... ...

64-9

73-8

1866

Chelsea ... ... ...

12-1

23-9

38'9

Hampstead ......

69-0

40-8

137-8

Pancras.........

11-4

6-7

18-8

Islington.........

37 6

32-2

819

Hackney...... ...

50 0

49-2

123-8

Bethnal Green......

14-3

5-7

20-8

Stepney ... ......

2-0

1-4

3-4

Mile End Old Town ...

27-5

13*3

44-5

Poplar ... ... ...

46-9

34-5

97-6

St. Saviour, Southwark...

0-7

11-5

12-2

St. Olave, Southwark ...

20-1

10-0

32-1

Lambeth ... ... ...

28-6

21-7

56-5

Wandsworth ... ...

77-6

68-2

198-8

Camberwell ......

55-7

67-6

161-0

Greenwich ... ...

170

30-5

52-7

Lewisham ......

61-2

42-2

129 -3

Woolwich ... ...

2-8

10-1

7-0

Looking at the extraordinarily rapid growth in some of these parishes which is as certain to continue as the sun is to rise in the morning the question arises whether we are always justified in waiting till' the children are on the ground before providing for them, and whether we should not look a little more forward than we have been in the habit of doing in the purchase of sites, and building of schools. Not only will it be an economical measure to anticipate by a year or two the arrival of the population, because the sites may be so much more cheaply purchased, but when we remember that an interval of two yerrs elapses from the first recommendation of a school by the Statistical Committee to its opening, and that the numbers are in the meanwhile in many districts' increasing annually with rapid strides, it is clear that large numbers of children will be left for a time without schooling, unless we have regard, not to the present population, but to that which we may predict with certainty mill be the population two years hence. Probably it may be well to tabulate the annual rate of increase in each registration dis-

trict, and have them before us in considering the accommodation needed I commend this matter to the Statistical Committee.

WORKS COMMITTEE.

The Works Committee may be described as the executive of the Statistical Committee. Their function is to select the sites and build the schools. The choice of positions for our schools has to be made with extreme care, as we here encounter many opposing interests. Neighbouring voluntary schools are sometimes nervous about the effect on their attendance, though in a far less degree than used to be the case. Property holders are often alarmed at the effect of a Board school on the value of their houses. Other land owners are not less anxious to sell their land to the School Board, and all sorts of means are adopted to press upon the consideration of the “ Sub-Committee of Sites” the eligibility of a particular plot. That this duty is performed with care and judgment is shown by the fact that out of the large number of sites recommended to the Education Department, who have the ultimate veto, only a very small percentage have been finally ejected. The cheerful appearance, convenient arrangement, and solid construction of our schools are the result of the accumulated experience often years, centring in our architect’s department where every suggested improvement likely to be of value is adopted, as far as possible.

SCHOOL MANAGEMENT.

The appointment of our teachers, the arrangement of the curriculum and all the varied and complicated details connected with the organisation of our schools, are under the charge of our School Management Committee, “ What is to be taught ? aud how is it to be taught ?” are questions which occupy a very large part of our time. With regard to the former question, it is needless to dwell on the fact that it is not in our hands, but in the discretion of the Education Department, to lay down the main lines for our guidance. By far the greater time of our teachers is devoted to imparting the three primary subjects to our children. It is satisfactory to find that we still maintain our supremacy over the rest of the country, now established for some years, in the proportionate number of children who pass in the “three Its.” The following table shows this :—

PERCENTAGE OP PASSES.


In all schools in In London England and    Board.

Wales.    Schools.

Reading    ...    ...    SS-25    ...    ...    89-3

Writing    ...    ...    S0'44    ...    ...    87'3

Arithmetic    ...    ...    79 '9    ...    ...    83'3


But while our chief efforts arc devoted to laying the foundations of education, and the great majority of our children are still in the lowest standards, it is encouraging to note some progress in the standards to which our children attain. Thus in 1878 less than one in five of our children attained to the fourth and higher standards. This proportion has almost risen to one in three.

RELIGIOUS EDUCATION.

There is only one subject generally taught in our schools which is not included in the Code of the Education Department. This is religious knowledge. The best test of our progress in this matter is afforded in the annual examination for Mr. Peek’s prizes. ^ The numbers entering for the preliminary examination were 141,274 this year, as against 127,501 last year, and 112,979 in 1879. Mr. Ricks, who kindly superintended his laborious work, says : “ The results are in no respect inferior to those of former years, but show, in some respects, a clear improvement;” and that, “ almost without exception, the teachers give to the Bible instruction all that care and attention which the importance of the subject demands.” On the other hand, it was remarked with regret that there was a considerable falling off in the number of pupils teachers who presented themselves for the annual examination. Doubtless the pressure on pupil teachers is heavy, and there is a temptation to drop out subjects that do not “ pay.” Moreover, many of them had already entered for this examination more than once in previous years, and probably thought it needless to go over the same ground again. Still, it is very important that our teachers of the future should be adequately trained and tested in religious knowledge, and I presume that the School Managing Committee will look into this matter.

COST OF SCHOOL MAINTENANCE.

A great deal of attention has been called by the recent report of the Education Department to the high cost of school maintenance in our London Board Schools as compared with the boards of other large towns. The report of the special committee on the comparative cost of school maintenance, presented to the board in December, 1880, throws much light upon the question. It is worthy of note that the nett expenditure for school maintenance fall upon the rates has been singularly uniform since 1874. It was £1 12s 8d per child in that year, £1 12s 9d last year, and it has hardly been varied in the interval. It is, however, the gross cost that we must chiefly consider. I here give a table showing the gross annual cost per child of school maintenance for the past six years :—

Average gross annual cost per child on the average attendance in London Board schools : —


Year ended

Gross Cost.

March 25th.

£ s. d.

1874 ...

... 249

1875 ...

... 278

1876 .

... 2 11 10

1877 ...

... 2 12 3

1878 ...

... 2 15 0

1879 ...

... 2 17 G

1880 ...

... 2 19 3

1881 ...

...

... 2 17 I


It is for us to justify this expenditure to ourselves and our constituents, or, if we are unable to do so, to reduce our staff, to diminish the scale of salaries, and cut down the expenditure or apparatus and other items. I believe that when the facts are examined we shall not be recommended to take this course by those who send us here. We shall not cease to keep a vigilant watch in our committees, as we have overdone, on every item, and to economise wherever it is possible, but to depart from the general policy that we have adopted can only be done at the expense of efficiency.

EFFECT OF NEW SCHOOLS ON EXPENDITURE,

It will be remembered that we differ from other boards in the large number of new schools that we have still to open every year. Most other large towns completed, or nearly completed, their school provision some years ago, whereas in London we have still to add something like

20,000 fresh school places each year, and this has a marked effect on the expenditure per child. It must be some time before wo shall have provided for the past, and in the meantime there is an annual increment of more than 9,000 children requiring places. The effect of this is twofold. In the first place a new school is some time filling up, and therefore the cost is spread over a smaller number of children. In tire next we have every year to stock a number of neur schools with apparatus and books —a heavy item which is required to be added to current expenditure. Moreover for the first year we receive no grant. Although this does not affect the gross expenditure it reduces the average grant per child. Of course the proportionate number of new schools added yearly is less now than it was, and the effect of this is shown in the fact that we reached the turning point in the expenditure last year, and it will be scon by reference to the table given above, that the gross expenditure per child is reduced this year by 2s 2d.

MISCELLANEOUS ITEMS.

If we now examine the expenditure in detail, we find that we do not, considering the special conditions in London, compare unfavourably with other towns in the “ miscellaneous” items, and I do not doubt that this amount will be further decreased as the proportion of new schools to old diminishes. These miscellaneous items therefore do not call for further remark, except that many of them, such as fuel, rent and rates, are generally higher in London than elsewhere.

TEACHERS’ SALARIES AND STAFF.

There remains then the very heavy item of teachers’ salaries, in which will be found the chief difference of cost between London and other towns, and which, therefore, demands full consideration. With regard to the staff, our rule is to reckon the head teacher or a pupil teacher for thirty children, and an assistant adult teacher for sixty ; and we also allow a larger proportion of adult teachers to pupil teachers than is usual elsewhere. But it will be seen that the Education Department, in their New Code, which comes into force iu the beginning of next year, acknowledge the wisdom of this arrangement by adopting our scale, at least so far as to allow only sixteen children to an adult teacher. We also, undoubtedly, pay our teachers salaries which are somewhat higher than the average. The average salary of an adult male teacher under the London School Board is £144 ; of an adult female, £108 ; whereas the salaries in other schools of the country are respectively £121 2s 7d and £72 128 8d. It will, however, be remembered that the teachers in London Board schools have no houses allowed them, although this is very frequently the case elsewhere, where it is, of course, considered in the salary. Moreover, the cost of living is higher in London than elsewhere, The truth of this latter statement and its bearing on the scale of our salaries is shown by a comparison of the average gross cost of London voluntary schools with voluntary schools elsewhere. The former exceeds the latter by Gs 2,’d per child. Still, it must be acknowledged that, even after allowing for all these circumstances, we do pay somewhat more than the average market value of average teachers ; and we do this deliberately, because it is not a dead level of average ability, but the best of their class, among the teachers that we require for our schools. The deficiency of schools in London has been so great, and the proportion of neglected children consequently so large, that we believe ourselves justified in providing them with teachers who will make the most of the brief years during which the child remains under their hands.

GRANTS.

It is, however, sometimes urged that the results as indicated by the grant are not commensurate with our expenditure. To this I reply— first, that our schools are too new to have reached their maximum grantearning power, a large proportion having been established for less than five years ; secondly, that we are able to show a marked superiority in the three fundamental subjects to which our attention should bo chiefly directed ; thirdly, that the good work which is done by a first-rate teacher is not to be measured merely by the grant. The fact that the grant is so nearly equal throughout the schools of the country, whether good, bad, or indifferent, shows that it is an imperfect guide, and the President and Vice-President of the Council seem to acknowledge this in their new proposals. The latter says “ Under the ‘ special merit’ clause the inspector is to have regard to the organisation and discipline, to the employment of intelligent methods of instruction, and to the general quality of the work in each school, especially iu the standard examination, and should have power to recommend an additional grant on the average attendance, varying in amount as the school was in these respects fair, good, or excellent.” We cannot doubt that the systematic manner in which drawing, drill, and physical exercises, the kindergarten methods, &c., are taught in other schools will, under this system, tell upon our grants to the advantage of the ratepayers.

(To be continued).

The Silesian Botanical Society exhibits a machine which enables a person even to hear plants grow,

NOTES OF A LESSON ON


SUNSTROKE.


By G, H. Morton.

For an Upper Class, ordinary apparatus. Time—One hour.


Heads.


Matter.


Method


Heads.


Matter,


Method.


I. Term Sunstroke MISLEADING.


II. Temperature of the BLOOD DURING HEALTH.


III. Various WAYS OF OVERHEATING THE BLOOD.


IV. Cause of Sunstroke.


V. Precautions.


The term “ Sunstroke” is hardly suitable, as it is apt to convey the impression that a person is suddenly struck down by the direct rays of the sun. (1). That the sun is the primary cause of most cases is certainly true, but they are all due to an overheating of the blood, by whatever means brought about. Heat-stroke would, therefore, be a more applicable term.

The ordinary temperature of the blood when a person is in a state of health, both in summer 01-winter, during exertion or while at rest, varies very little on either side of 98° F. (2.)

The principal cause which regulates the temperature of the blood is as follows : When the blood gets overheated it causes a tlow of perspiration (3) from the sweat glands in proportion to the excess of heat. This perspiration evaporating on the surface of the skin cools the blood near, which passes on through the system, and warmer blood takes its place to be cooled in turn. (4.)


The blood can be heated or overheated by

(a)    Exertion, such as running.

(b)    External heat, such as a fire or the sun

(c)    By preventing its superfluous heat from escaping. (5.)

(dJ Eating fatty or other heatforming foods. (6.)

Recapitulate.


Anything which causes the blood to be heated to excess, the flow of perspiration being at the same time impeded, may bring on an attack of sun, or more properly, heatstroke. (7.)

The following precautions are deemed necessary during summer : (8.)

[a and b) Unnecessary exposure to heat to be avoided. (9.)

(c)    To wear light and loose clothing.

(d)    To abstain from fatty, or other heat-forming foods. (10.)

Recapitulate the whole.


VI. S Y M p- The symptoms are not always t oMs.    the same. In some cases the heart’s

action is stopped altogether, and with it the breathing. In others the face is flushed, with heavy breathing, and sometimes complete insensibility. But in all cases if a


(1.) Introduce to class by telling them you are about to give a lesson on Sunstroke. Ask them what sunstroke is, and you will probably get such an answer as, “ being struck down by the heat of the sun.”

Recapitulate.

(2.) Explain briefly the thermometer and its uses.

(3) Educe this by asking the class what they notice on their skin (1) after running hard, (2) if they continue running or exerting themselves.

(4.) Ask meaning of evaporation, and explain how evaporation produces coolness by reference to a canvass water-bag. The moisture on the exterior of the bag is turned into vapour - vapour = water + heat. The heat, besides, from other sources, is drawn from the water, thus cooling it. Recapitulate.

(5.) Educe by asking class how they would warm themselves on a cold day if fa) sent outside (exertion) ; (b) inside (by a fire) ; (c) if no fire (by putting on more clothes).

(6.) Illustrate the heat-forming properties of foods by reference to the staple foods of inhabitants of the Northern regions, and compare them with those of tropical countries,

(7.) Educe this by questioning on the foregoing heads.

Recapitulate.

(8.) These may be educed by referring to causes which heat the blood under heading III. Refer to cases of sunstroke occurring recently in England and France during reviews. (10.) Notice that we often here of pigs being sun-struck, probably owing to the grossness of their food,


Lecturing,


VII. Treatment.


VIII. Statistics.


IN. Probable causes of

F Ii E E D O M

from Sunstroke in this Colony.


thermometer be applied to a convenient part of the body,such as the armpits, it will show an increase up to 10 or more degrees on the ordinary temperature of the blood.*

If a medical man be not handy the patient can be safely treated in the meantime by the application of cold to the surface (11) by means of wet towels, &c., applied to different parts of the body, &c. (12.)

During the years 1869-78, 61 fatal cases of suustroke occurred iu this colony. This number is small, however, compared to some parts uf the world. In the city of New York, for instance, no less than 1S9 fatal cases occurred in the first four days of July, 1872, and no further back than the 26th of last August 60 deaths were recorded for that one day in the same city.

Our hottest days arc generally accompanied by hot. dry winds, which promote a profuse perspiration, and, owing to their extreme dryness, facilitate evaporation, which, as before shown, has a cooling effect on the blood. (13.)*


(11.) What effect will this have ?

(12.) Elicit from class.

Recapitulate.


Lecturing.


(13.) Notice that we often get very hot if exerting ourselves during misty or rainy weather— excess of moisture retards evaporation.

Recapitulate the whole.


A synopsis of the whole should appear on the blackboard at the end of the lesson, to be placed there as the teacher is dealing with each head.

This lesson, if more suitable, may be conveniently divided into two, the second one commencing at head VI., care being taken to recapitulate the former lesson before proceeding with the remaining heads.

*See Australian Health Society's Tract on Sunstroke.


[ITelbounu Hitibcrstiir.


The following are the Pass Papers set by the University at the Matriculation Examination, October. 1881 :—

GEOMETRY.

1.    Draw a straight line at right angles to a given straight line from a given point in the same.

2.    Prove that the three interior angles of every triangle are together equal to two right angles.

3.    Prove that in any right-angled triangle the square which is described upon the side subtending the right angle is equal to the squares described upon the sides which contain the right angle.

4.    If a straight line be divided into any two parts the square on the whole line is equal to the squares on the two parts together with twice the rectangle contained by the parts.

5.    Divide a given straight line into two parts, so that the rectangle contained by the whole and one of the parts shall be equal to the square on the other part.

6.    In every triangle the square on the side subtending either of the acute angles is less than the squares on the sides containing that angle by twice the rectangle contained by either of these sides, and the straight line intercepted between the acute angle and the perpendicular let fall upon it from the opposite angle,

7.    Find the centre of a given circle.

8.    The opposite angles of any quadrilateral figure inscribed in a circle are together equal to two right angles.

9.    If a straight line touch a circle, and from the point of contact a straight line be drawn meeting the circle ; the angles which this line makes with the line touching the circle shall be equal to the angles which are in the alternate segments of the circle.

ENGLISH.

1.    Analyse—“ I have heard him often lament that, in a profession where merit is placed in so conspicuous a view, impudence should get the better of modesty.”

2.    “ Grant that the pow’rful still the weak contronl,

Be man the wit and tyrant of the whole :

Nature that tyrant checks ; he only knows,

And helps another creature’s wants and woes.

Say will the falcon, stooping from above,

Smit with her varying plumage, spare the dove ?”

Parse fully each word which 'is printed in italics in the above passage.

3.    Give the derivations—including roots, prefixes, and suffixes—of— victim, temptation, meadow, parallel, observant, diffuse, escape, politics, critic, belief, ordinary, prevail.

4.    Give six instances of words which can be used either as adverbs or as prepositions, and write sentences illustrating their use in either way.


5.    What is the 1st person singular past perfect conjunctive passive of gnaw? 2nd person singular future perfect indicative active of creep? 3rd person plural past indicative active of chide ? "What is the plural number of—vortex, criterion, virtuoso, miasma, index, penny ? What is the feminine of—heritor, director, marquis, testator, margrave, czar ?

6.    Write out the 22 lines of the “ Essay on Man ” commencing “ Heaven from all creatures” down to “ a life to come.”

7.    Who is addressed as St. John at the commencement of the 1st Epistle of the Essay on Man ? Give some account of him. Who was “ Curtius ” ? Explain “the starving chemist in his golden views supremely blest.” Give a short account of the principal members of the Spectator’s Club.

S. What is the difference between simile and'metaphor ? Give instances from the text-books.

9. What is the use of prepositions in the English language ?

10. Write a short essay on—Fire.

FRENCH.

1.    Translate—“ Vous me dites des merveilles du tombeau de M. de Montmorency, et de la, beauté de Mesdemoiselles de Valençay. Vous écrivez extrêmement bien, personne n’écrit mieux : ne quittez jamais le naturel, votre tour s’y est formé, et cela compose un style parfait. J’ai fait vos compliments à M. de la Rochefoucauld, et à Madame de la Fayette, et à Langlade : tout cela vous aime, vous estime, et vous sert en toute occasion.”—Mme de Scvigné.

18 Février, 1671.

2.    Parse the following words iu the above passage :—dites, Mesdemoiselles, écrivez, personne, mieux, sert.

3.    What is the rule for the formation of the feminine of adjectives ending in fand x 1

4.    Whât lettex-s terminate the substantives which retain the same form in the plural that they have in the singular.

5.    Which two cardinal numbers in French take an s in the plural ? Write out phrases exemplifying this.

6.    Translate—The lady to whom I am writing. The men who are singing. The books which you read.

7.    Write out the present subjunctive of être and avoir.

8.    Into how many conjugations are French verbs divided? Give an example of each.

9.    Translate—I should not speak, I should not have spoken. Should I speak ? Should I not have spoken ?

10.    What do'you understand by a pronominal verb? Give an example

11.    Distinguish between a direct and an indirect regimen.

12.    What is the general rule as to the agreement of the past participle of a verb active accompanied by the auxiliary avoir, when the direct regimen is placed before the participle ? Give an example,

13.    Translate into French—I want a pencil. There are several in my desk. Describe all you saw in that beautiful city. Are you my friend’s sisters ? Yes, we are. I am studying French ; I know its rules well ; but its pronunciation seems difficult to me. If I do not see you to-night, it is because I shall not be able to go out. Wait till the tragedy is over and we will go out together. The books which I have seen. The books which I saw bought.

14.    Translate into French—“ A knowledge of living languages” (says Rollin) “serves as an introduction to all the sciences. By its means we arrive, almost without difficulty, at the perception of an infinite number of beautiful things, which have cost their inventors long and tedious labours. By its means all ages and all countries are open to us. It renders us to a certain extent contemporaries of all times and citizens of all nations, and enables us to converse even at the present day with all the wisest men that antiquity has produced, who seem to have lived and to have laboured for us.

German.

1.    Translate into English— Sie kchertc ins Dorf zurück, es kam ihr leer vor, und in der Dämmerung, als sie die Kinder des Rodel bauern ein wiegte, konnte sie nicht ein einziges Lied über die Lippen bringen, wahrend sie sonst immer sang wie eine Lerche. Sie musste immer denken, wo jetzt ihr Brudersei, was man mit ihm rede, wie man ihn empfange, und doch konnte sie sich das nicht vorstellen. Sie wäre gern hingeeilt und hatte gern allen Menschen gesagt, wie gut er sei, und dass sie auch gut gegen ihn sein mögen ; aber sic tröstete sich wieder, dass Niemand ganz und überall fur den Andern sorgen könne. Und sie hoffte, es würde ihm gut thun, dass er sich selber forthelfe.

2.    Give the verbs in the above passage which are in the subjunctive mood, and explain why they are in the subjunctive.

3.    Decline throughout: Ein weisser Duft; das kleine Dorf; die grosse Fluth.

4.    What classes of nouns are generally masculine ?

5.    Give the 1st person singular Impf. Indie, and Past Participle of the following verbs :—Schlagen, ziehen, nehmen, gebieten, dringen, zerstören, vollziehen, and give their meanings.

6.    What classes of verbs do not take the prefix ge in the Past Participle ? Give examples.

7.    Give the Present Indicative and the Present Subjunctive, in full, of the following Verbs :—Dürfen, nachfolgen, antworten, beurtheilen, schicken.

8.    How do conjunctions affect the position of the verb in a sentence ?

9.    How is the German Present Participle used; and how is the English Present Participle chiefly rendered into German ?

10.    Give the Demonstrative Pronouns. Under what circumstances is the neuter used invariably without regard to the gender and the number of the noun ?

11.    Translate into German—I ought to have done it. When the lesson is finished, you may go. He closed the door and went quickly

up stairs. A year ago. Early in the morning. Had you only told me of it, the letter would have been sent. They say that the unfortunate man has lost his money. The dog has sprung into the water ; the man is standing under the tree. He is older than I. Seeing that the planets resemble the earth, arc they therefore inhabited ?

12. Translate into German—A blind peasant, who could go about without a guide, lived with his daughter in a small village. One evening the daughter suddenly became ill, and the blind man had himself to seek for help. He put a light into the lantern, took it in his hand and went out. An acquaintance, who met him in the street:, remarked, “ I always thought you a prudent man, now I see that you have become childish ; for the light can be of no use to you.” “ I do not carry the light for myself,” replied the old man, “but for you and such as you that you may see me, and step out of the way.”

LATIN.

C.<ESAR, Book VI. ; Vergil, Aenoid, Book XII.

(N.Jd.—In parsing any word taken from the extract, on the paper before you, add the rule from the Grammar, when you can, for its concord or government. State the 'principal parts of the verb, after the example in the Grammar, then its voice, mood, and tense. In parsing a substantive or an adjective, give the nominative and genitive singular, the number, gender and case.

1.    Translate literally—Interim confecta frumentatione milites nostri clamorem exaudiunt: praecurrunt cquites ; quanto res sit in periculo cognoscunt, IIic vero nulla munitio est, quae perterritos recipiat: modo conscripti atque usus militaris imperiti ad tribunum militum centurion-esque ora convcrtunt: quid ab his praecipiatur exspectant. Nemo est tam fortis, quin rci novitate perturbetur. Barbari signa procul conspi-cati oppugnatoiue desistunt: redissc primo legiones crcdunt, quas longius discessisse ex captivis cognoveraut: postca despecta paucitate ex omnibus partibus impetum faciunt.

2.    Select the verbs, in the above extract, that are in the subjunctive mood, and give the reason, in each instance, for its use.

3.    Parse ;—Confecta, periculo, perterritos, usus, ora, redissc.

4.    Translate into English—(a) Tunc recordatus est praeteriti temporis quo puer adhuc, cum posset jucunda fratrum aut aequalim consuctudine frui, cum illis dissidia ac saepc jurgia inierat, quorum nunc magnopere eum poenituit. Eheu, secum reputavit, eqnidem tunc temporis parum intellexi, quanti amicus sit mstimandus. (¿0 Nil adeo Fortune gravis miscrabilc fecit, ut minuant nulla gaudia parte malum.

5.    Put into Latin—(a) The cause of all good men is the same, my son.

(b)    The very mention of that great sorrow has weakened my voice with weeping. (c) I wish (utinam) thou hadst forgiven him. (d) Corn was in no place of so much value as my friend, the merchant, reckoned.

(c)    Let me know where you are, what you are doing, what you have done, and what you are going to do. (/) One does one thing, another another ; but it is the mark of a Christian to do good to all.

6.    Translate literally—

Ut primum discussae umbrae et lux reddifa mcnli,

Ardentis oculorum orbis ad maenia torsit Turbidus, eque rotis magnam respexit ad urbem.

Ecce autem, flammis inter tabulata volutus Ad coelum undabat vertex turrimque tenebat,

Turrirn compactis trabibus quam eduxerat ipse Subdideratque rotas pontisque instraverat altos,

Jam jam fata, soror, superant; absiste marari;

Quo deus et quo dura vocat Fortuna, sequamur.

Stat conferre manum Aencac, stat, quidquid acerbi est,

Morte pati ; neque me indecorcm, germane, videbis Amplius. Hunc, oro, sine me furere ante furorem.

7.    Decline ipse, ulerque, unusquisque, paterfamilias, domus.

8.    Give the dative and ablative plural of dea, filia, genu, iter, bos-quercus, senex,

9.    Compare celer, acer, difficilis, pius, maledicus, benevolus, nequam,

10.    Write down the perfect and supine of coco, lacesso, gcro, haurio misceo. applico, revello.

11.    Explain what is meant by the sequence of tenses.

12.    Give an example of (a) the genitive of quality ; (b) the double dative ; (c) the double accusative ; (d) the use of the gerund and the gerundive.

GEOGRAPHY.

I.

1.    Draw, to the full size of your paper, a map showing, in order, the countries on the Mediterranean sea-board of Africa, and the position of their chief cities ; and give a brief summary of any facts of interest you may know concerning either the countries or the cities.

2.    Give an account descriptive of the different routes from Melbourne to London. State which yci •'•ould prefer, and for what reasons.

3.    What animal and vegei.w productions useful to man come from India. Sweden, Spain, and the South of North America respectively?

4.    Write down what you consider to bo the most important facts in the political geography of Chili.

II.

1.    Mention, in order, the towns, capes, river-mouths, and inlets of the sea which would be passed during a voyage from Melbourne to Adelaide.

2.    Mention six cities of the British Empire larger than Melbourne. Give their locality, their population, and any facts of importance concerning them.

3.    Give a short geographical description of the Lowlands of Scotland.

4.    Describe, with reference to the adjacent towns and provinces, the course of the three rivers which you consider the most important in the British Empire.

P + 1 P-1


lies between

P + 1

ax2 + by- + cz2 = 0 ax +by +cz =0 yx +zx + xy =0


and


0.


86


xl USTE AL ASÌ AN SCHOOLMASTER.


December, 1881.


1.    Explain the means by which the shape and size of the earth have been ascertained.

2.    Explain the theory of the tides. Mention some places at which the ordinary rise and fall is at a maximum, and some other place at which it is at a minimum, and give the reasons in each case.

3.    What is the ordinary annual rainfall in Victoria? Mention some country in which the yearly rainfall is less than in Victoria, and some other country in which it is greater, giving the reasons in each case.

4.    Describe as well as you can the influence of forests on the condition of a country;

ELEMENTARY PHYSICS.

1.    What is meant by the mass of a body, and why cannot we accept weight as a fundamental method of estimating mass ?

2.    State the first law of motion, and explain how you reconcile this law with ordinary experience of the tendency of bodies in motion to come, sooner or later, to rest.

3.    Rapidly revolving grindstones have been known to burst into pieces. How do you account for this result ?

4.    What is meant by the Centre of Gravity of a body ? How would you proceed in order to find the position of the centre of gravity of a plate of metal of irregular oatliue ?

5.    The length of the pendulum of a clock is half a metre. Calculate how many oscillations it will make in an hour.

G. Describe the Mercureal Barometer, and explain how it is used for determining the height of mountains.

7.    Describe fully the process of making and graduating an ordinary ’Mercureal Thermometer.

8.    Explain carefully the various actions that take place when a kettle of cold water is placed on a fire and left there until it boils.

0.    Describe an experiment in which water is frozen by means of its own evaporation.

10. What were the principal improvements in steam machinery introduced by James Wi*tt?

ELEMENTARY PHYSIOLOGY.

1.    Describe the circulation of the blood through the human heart, statiug the results produced by the contraction of its several chambers, and explaining the action of its valves.

2.    Draw a rough sketch of a horizontal section of the eyeball, and name in order the different tunics and humours.

3.    What are the structures entering into the formation of a perfect joint ? Enumerate the chief varieties of such joints, and give examples.

4.    Describe the secretions taking part in digestion, aud their actions upon the principal classes of foods.

5.    What are the differences between inspired and expired air ?

The following are the Honour Papers set by the University at the Matriculation Examination, October, 1881 :—

English.

1.    In the following passage parse fully each word which is printed in italics, and explain the construction of the sentence in each case :—

“ Let your highness

Command upon me ; to the which my duties Are with a most indissoluble tie.”

“ How you shall bid God ’ild us for your pains.”

“ For the poor wren,

The most diminutive of birds, will fight,

Her young ones in her nest, against the owl.”

“ Making the green one red." t: Out, out, brief candle !”

<! They have tied me to a stake ; I cannot fly,

But, bear-like, I must fight the course.”

“ Was it not yesterday we spoke together ?”

“ If he had been forgotten,

It had been as a gap in our great feast,

And all-thing unbecoming.”

“ If such a one be fit to govern, speak :

I am as I have spoken.

Fit to govern !

No, not to live. 0 nation miserable.”

Ay, sir; there are a crew of wretched souls That stay his cure ; their malady convinces The great assay of art.”

2.    State what changes have taken place in the usage of the relative pronouns since the Elizabethan period. Explain the use of each of them in modern English, illustrating your explanation by examples.

3.    Analyse, and also paraphrase, the following sentence :—“The referring of all to a man’s self, is more tolerable in a sovereign prince, because themselves are not only themselves, but their good and evil is at the peril of the public fortune.”

4.    Explain the meaning which each of the following words bears in its context, and give its derivation :—Seeling, cancel, gouts, adage, harp’d, baited, burses, humours, dudgeon, warrant, saucy, chaudroD, mansionry, sequester, aculeate, bias.

5.    State in what sense an adverb can be said to qualify a noun. State also what class of adverbs qualify adjectives and adverbs.

G. Give briefly the argument of Bacon’s Essay on Anger.

7. Modernise the following passages, and give reasons for the alterations which you make :—

“ There be that delight in giddiness.”

“ If you listen to David’s harp you shall hear,”

“ The first man took hold of it and found means it was told the Queen.”

“Sylla did a little resent thereat.”

“ Nature will lay buried a great time.”

“ Are ye fantastical ?”

“ The worm that’s fled hath nature that in time will venom breed.”

“ I have words that would be howl'd out in the desert air.”

8.    Give some account of Banquo, Fleance, Lady Macbeth, illustrating your description with passages from “ Macbeth.”

9.    What reasons are there for believing that some parts of Macbeth” were not written by Shakespeare ?

10.    What meanings does the preposition “ of” bear ? Give examples.

11.    From what languages has English been principally recruited ? At what periods and throagh what channels were Latin words mostly introduced ? Give examples.

12.    Write an essay on—Extremes meet.

ALGEBRA.

1,    Prove that

a(b-c) (h + c — a)-+ b(c - a) (c + a - b)2 + c[a -b) (a + b - c)2 = 0.

2,    Prove that when m and n are positive integers

am. a" = am + ",

and assuming this result to be true for fractional and negative exponent interpret the meaning of am when m is fractional or negative.

3,    If the equations

x2 + ax + bc = 0 x2 + bx + ca = 0

have a common root, show that their other roots satisfy the equation. x2 + cx + ab = 0.

4. Prove that whatever real valuer may have, ax- + hx + c and never differ in sign except when the roots of the equation ax- + hx + c = 0 are real and different and x is taken so as to lie between them.

If y; be greater than y, then for all rea’ values of x the expression

x2 -2qx + p2 v , ^    P~1 x- + 2 qx+jF

6. If prove that

a

b - c c - a + a - b

~• If —-= 77 =—r — &c., prove that each of these ratios is equal to

/pan + qbn + rc'n + .... \ _L \y;a''1 + qb'n + rc'n + ,...) n

Solve the equations

x2    y2 _ z2    x3 y3 z3

a    b c    a3 + b2 ^ c- '

8.    Define a geometrical progression, and show how to find the sum of any number of terms of such a progression. The sum to infinity of a certain geometrical progression is s, and the sum to infinity of the series formed by taking its first, third, fifth, &c., terms is s'. Find the first term and the common ratio of the series.

9.    Prove that in the scale of which the radix is r, a number is divisible by r - 1 when the sum of its digits is so divisible, Show that, in the ordinary scale of notation, the difference between the square of any number consisting of two digits and the square of the number formed by interchanging these digits is divisible by 99.

10.    Find a formula for the number of combinations of n things taken r together. If the number of combinations of 2n things taken n-1 together be to the number of combinations of 2n — 2 things taken together as 132 to 35, find n.

_ 11- Enunciate and prove the Binomial Theorem for a positive integral exponent. If n be a positive integer prove that

l+.z n{n-1) l+2z ~nl + nx+ L2 (T+ nx)_ n(u~l) (n-2)    1 + 3x

1.2.3    (1 +nx)3 + kc-~{>

12. Solve the equations

(i) ^ x2 + 3x +1 + ^ Sx2 + 5x - 7 = 2 \rx2 + x-4 (ii) ax2 + by2 — a2% + b2y = a3 + b3.

9. If sin 2A— \r 2 find sin A.


LATIN.

Livy, Book I. ; CLesar, Book YI.; Horace, Odes, Book III.; Vergil, Aeneid Book XII,

1. Translate into Latin prose—Some days before the execution of Socrates, Crito came to him in prison before daybreak, found him in a sweet sleep, and seated himself quietly by his bed, in order not to disturb him. When Socrates awoke, he said, “Why are you so early today, friend Crito?” Crito mentioned that he had received intelligence, that the next day the sentence of death was to be put into execution. “ If it is the will of God,” answered Socrates, with his usual calmness, “ So be it.”


ENGLISH.

1.    In the following passages parse fully each word which is printed in italics:—“ It was, so please your highness.” “Certainly great persons had, need to borrow other men’s opinions to think themselves happy.” “ Who wear our health but sickly in his life, Which in his death were perfect.” “Whatman ¿are, I dare.” “Returning were as tedious as go o’er.” “ Say what the use, were finer optics given t’ inspect a mite.” “ And sundry blessings hang about his throne that speak him full of grace. ’ “It is worthy the observing.”

2.    Make a complete analysis of the following passage :—“ My worthy friend has put me under the particular care of his butler, who is a very prudent man, and as well as the rest of his fellow-servants, wonderfully desirous of pleasing me, because they have often heard their master talk of me as of his particular friend.” Also, write out in full, each of the attributive and adverbial phrases which occur in the passage, assigning to each of them its place in the grammatical construction of the sentence.

3.    (a) What property of prepositions distinguishes them from conjunctions ?    (b) To what parts of speech, other than nouns or pronouns, are

prepositions sometimes prefixed ? Explain the construction in such cases, (c) When a phrase is formed by a preposition prefixed to a noun or its equivalent, how can it be determined whether it is an attributive or an adverbial phrase ?

4.    Paraphrase the following passages, and point out in what respects they differ from modern usage :—“ A servant or a favourite, if he be inward, and no other apparent cause of esteem, is commonly thought but a by-way to close corruption.”

“ Noble Banquo,

That hast no less deserved, nor must be known No less to have done so.”

“ And something from the palace ; always thought That I require a clearness.”

“ My noble partner

You greet with present grace and great prediction Of noble having and of royal hope,

That he seems rapt withal.”

“ Diet in such places where there is good company.”

“ Wail his fall

“ Who I myself struck down.”

5.    Write out the 24 lines from the “ Essay on Man,” commencing : — “ But errs not nature,” down to “ is to submit.”

6.    Reproduce briefly in your own words the last scene in which the witches are introduced in “ Macbeth.”

7.    Explain the meaning in the following passages of the words which

are printed in italics :—“ Who can ingress the forest ?”    “ The crack of

doom.” “And the receipt of reason (shall be) a limbec only.” “That mixture of falsehood is like allay.” “ When hearing should not latch them.” “My mind she has mated.” “It mates aud masters the fear of death.” “ Art not without ambition but without the illness should attend it.” “ Which is a great adamant of acquaintance.”

8.    Give the derivation of the following words :—Farnham, Llandaff, Scarborough, Winchester, Fenwick, Kilbride, Strathmore, scourge, ally’d, squire, savage, baronet, gentle, jovial, sleave, file.

9.    State briefly what distinction Bacon makes between goodness and goodness of nature, and what he says regarding the errors of goodness and the signs of goodness.

10.    (a) What is there unusual in Addison’s use of the words“ utter”

and “ venture ” ?    (b) What is the meaning of the word “ gust,” and in

what context does Pope use it in the “ Essay on Man ” ? (c) “ Much may be said on both sides.” On what occasion did Sir Roger de Coverley give this decision ? (d) What does Pope imply by the epithet “nice ” as applied to the bee ?

11.    What classes of words are exclusively of Anglo-Saxon origin, and what classes are generally so ?

12.    Write an essay on—Opportunity.

GEOMETRY AND TRIGONOMETRY.

1.    Two equal circles touch at A and a circle of twice the radius is described having internal contact with one of them at B and cutting the other in P and Q. Prove that the straight line AB will pass through either P or Q.

2.    If two circles touch each other externally the circle described on the straight line joining their centres as diameter will touch their common external tangents.

3.    Inscribe a circle in a given triangle. From the angles A, B, C of a triangle AD, BE, CF are drawn perpendicular to the opposite sides meeting them in D, E, F respectively. Show that the point in which these perpendiculars meet is the centre of the circle inscribed in the triangle BEF.

4.    Describe an isosceles triangle having each of the angles at the base double of the third angle. In the figure of this problem what portion of the circumference of the smaller circle is intercepted by the larger?

5.    Prove that the perpendicular drawn from the right angle of a rightangled triangle to the base is a mean proportional between Ihe segments of the base; and also that each of the sides is a mean proportional between the base and the segment of it adjacent to that side. From any point P on the circumference of a semicircle, PQ is drawn perpendicular to the diameter AB. Through M, the middle point of PQ, are drawn straight lines at right angles to AM and BM, meeting AB in the points a and b respectively. Find the ratio of ab to AB.

6.    Divide a given straight line in extreme and mean ratio. Show that the straight lines joining the alternate angular points of a regular pentagon cut each other in extreme and mean ratio.

7. Describe the two units used in expressing angles numerically, and investigate an equation connecting the numbers which express the same angle in terms of those units. In measuring the length of an arc of a circle whose radius is 100 feet an observer is liable to make an error of £ inch. Show that this is equivalent to making an error of about 34^" in determining the angle which the arc subtends at the centre of the circle.

Show that the exprsssion obtained

admits of four values ; find them and explain them.

10.    Prove that sin (.r + ?/-!-z) sin y = sin (.r + y) sin (y +2) - sin x sin 2.

11.    A person walks from one end A of a straight wall a certain distance a towards the west, and observes that the other end B then bears

E.S.E. He afterwards walks from the end B a distance ( ^2 + 1) a towards the south, and finds that the end A bears N.W. Show that the wall makes with the east an angle whose contangent is 2.

12.    Find an expression for the radius of a circle which touches one side of a triangle and the other two sides produced. If rQ be the radius of a circle inscribed in a triangle and rx,r^,rz the radii of circles which touch each side respectively and the other two produced, show that the area of the triangle = \r r6 rx rz.

2. Translate into English—(a) Sex item alias centurias—tribus ab Romulo institutis—sub iisdem quibus iuauguratae erant, nominibus fecit; ad equos emendos dena millia aeris ex publico data, et quibus equos alerent, viduae adtributae, quae bina miliia aeris in annos singulos pen-derent. (¿)—taking the Latin—Spectavere furcis duodens ab terra spectacula alta sustinentibus pedes.

3.    Translate into English—Darius praefecit Megabazum, virum Per-sam, quern magnopere honoraverat, hoc in eum verbo coram Persia dicto. Mala punica comedere cupiverat Darius ; qui postqum primum aperuit malum, quaesivit ex eo frater Artabanus, 1 Quidnam esset cujus tantum sibi numerum esse cuperet, quantus numerus granorum in punico malo ?’ Cui Darius respondit, 1 Yelle se tot Megabazos habere ; hoc enim malle quam Graeciam suae potestati subjectam.’

4.    Put in Latin—(a) Poems are often admired without being understood. (b) It makes no difference to me at all, though perhaps it does to another, (b) You are envied and you are maligned, but there is nothing to prevent your living at Rome or at Athens, at Carthage or in London, (d) I have no doubt his relievers have been sufficiently answered. (e) My brother and I fear you will forget us in our absence ; but having promised to return to Athens by the 5th February, we hope you will have it in your power to be as good as your word, (f) If I may say so without offence, you paid too much for that horse of yours ; it is not worth a straw.

5.    Explain aud connect with the context—Paulo supra hanc momoriam —primum pilum duxerat—Adclarassis—eum frequentiae taedere—dedier *'**■* siris—ab decumana porta—conditum lustrnm—silentium triste actacita moestitia.

6.    Translate the following extracts; note the different readings, if any, and scan the first stanza, marking the quantities :—

(a)    Vos Gaesarem altum, militia simul Fessas cohortes addidit oppidis,

Finiré qnaerentem labores Pierio recreatis antro,

(b)    Unico gaudens mulier marilo Prodeat justis operata sacris,

Et soror clari ducis et decorae

Supplice vitta

Virginum matres juvenumque nuper Sospitum.

(c)    Sume, Maecenas, cyathos amici Sospitis centum, et vigiles lucernas Prefer in lucem ; procul omnis esto

Clamor et ira.

{d) Ut in ejusmodi difficultatibus, quantum diligentia provideri poterat providebatur, ut 1 ♦Tns in nocendo aliquid prater-mitteretur, etsi omnium au,.,n ad ulciscendum ardebant, quam cum arique militum detrimento noceretur.

7.    Translate into English—

Nec minus Aeneas, quamquam tardante sagitta Interdum genua impediunt cursumque recusant,

Insequitur, trepidique pedem pede fervidus urguet:

Inclusum veluti si quando ilumine nactus Cervum aut puniceae saeptum formidine pennae Venator cursu canis et latratibus instat;

Ule autem. insidiis ctripa territus alta,

Mille fugit refugitque vias : at vividus Umber Haeret bians, jam jamque tenet, similisque tenenti Increquit mails, morsuque elusus inani est.

8.    Point out any peculiarities in the following constructions Raptim quibus quisque poterat elatis; nequiquam fallís dea; quisque

suum populatus iter; regina tui fidissima ; abstiueto, dixit, irarum calidaeque rixae ; luctatur eripere ; uxor invicti Jovis esse nescis.

II.

1.    What is the genitive plural of bos, terrester, celer, nummus, sestertious, modius, faber, drachma ?

2.    The future infinitive passive is applicable to all genders. Why ?

8. Adduce an example of oratio obliqua, and convert the same into

oratio recta.

4.    What do you understand by the infinitive as subject, the infinitive as object, and the infinitive in exclamation ?

5.    Parse—Nactus ; passis (erinibus) ; pereuntis; Rhodopen; 'Mnesthea; farre ; coiisse.

G. Express in Classical Latin Prose the poetical forms—Invadunt martem ; me verius unum pro vobis foedus luere ; irasci in cornua temptat; Orcum moror.


HISTORY.

I.

1.    A Statute was passed forbidding men to possess more than 2,000 sheep. When was this Statute passed, and under what circumstances ; and what were its effects?

2.    State the history of the Star Chamber and its jurisdiction.

8. Describe the polity of the Saxons—In matters of legislation, in matters of justice, in matters of rank, in matters of war, in matters of land.

4.    Describe the circumstances which led to the enactment of the English poor laws, and explain the nature and effects of those laws.

5.    Describe and contrast the circumstances of the union of England with Scotland, and the circumstances of the union of England with Ireland.

6.    Describe briefly, giving dates, the circumstances of the acquisition or founding (as the case may be) by the English of South Australia, Canada, Cape of Good Hope, Calcutta, Bombay, Madras.

.    .    . n.

1.    Explain fully the rivalry of the Senate and the Equites, and the policy of Cains Gracchus with regard to the latter.

2.    Explain and illustrate the doctrines of Euemerus (Evemerus) and their effects on Roman society.

d. Describe, giving dates, the legislation of Rome with regard to large estates, and state the results.

1. Draw a map of Sicily as known to the Romans, naming the principal places ; and sketch the history of the Siculi up to the first revolt of the slaves.

r    III.

1.    What was the motive for the Attic law as to Ostracism ? Describe, giving dates, some of the principal instances in which this law was applied.

2.    What was the origin, and what was the nature, of the Athenian clcruchies (klerouchiai) 1 State the principal instances, describing the geographical position,

3.    Sketch the western coast of Asia Minor, naming the principal places, and stating the principal events associated with them.

4.    State and explain the position taken up by Socrates with regard to —1. Physics. 2. Ethics. 3. The Sophists.


CONTENTS


School Department—

London School Board ... 82 Notes of a lesson on Sunstroke 84 Melbourne University ... ■    ...    84

Physical Exercise for Girls ...    88

Leader ...... ...... 88

The Work of Industrial Schools 89 School Board Members’ Term of Office ...... ...    90


Notes of the Month ...... 90

Science and Art Gossip ... 91 Victorian Education Department ...    •••    ••• 92

New Zealand Education Department ...... ••• 93

Matriculation Examination-

Algebra ......... 94

Arithmetic ...... 94


THE

Mutual Trust and Investment Society.

LIMITED.


(Incorporated under the Companies Statute 1864).

OFFICES: 66 COLLINS-STREET WEST, MELBOURNE.

Capital, £100,000 in 10,000 shares of £10 each. Subscribed, £10,000. Paid-up

£9301.

DIRECTORS:


Mr. M. Lt. Davies, Chairman, 8 Collins-street West.

Mr. Francis J. Smart, architect (Messrs Henderson and Smart)

Auctioneers and Special Valuators : Messrs. C. J. and T. Ham, Swanston-St. Surveyor and Valuator: Mr. Percy Oakden (Messrs. Terry and Oakdcn). Auditors : Mr. Thomas Inglis and Mr. John G. Shield.


Mr. Robert Inglis, accountant, G4 Collins-street West.

Mr. Charles Hetherington, Secretary, 66 Collins-street West.


SECRETARY : MR. C. HETHERINGTON. MANAGING DIRECTOR : MR. ROBERT INGLIS.

SECOND ISSUE of 1000 SHARES of £10 EACH.


The chief objects of the company are:—1. To lend money to members upon the security of real property or upon bank, gas or building society shares, or upon other approved security. 2. To negotiate loans of all descriptions upon such terms as to profit and remuneration as may be agreed upon. 3. To buy and sell freehold or leasehold estates. 4. To collect and receive rents, debts, dividends, interest and other moneys. 5. To buy and sell shares in any public banking corporation, insurance or gas company or building society, or any incorporate company. 6. To act as attorney or agent in the management of estates for absentees and for trustees.

In order to increase the paid-up capital of the company and extend its business it has been decided to make a seeond issue of 1000 shares of £10 each. These shares are now offered to the public at a premium of 5s. per share, and may be paid cither in cash or by instalments of £1 per share per month. The company has now completed its sixth half-year, and has, during its progress in addition to paying off all preliminary expenses, paid one half-yearly dividend at the rate of 8 per cent, per annum, and four half-yearly dividends at the rate of 10 per cent, per annum. The secretary will receive application for shares unti, further notice, and will furnish information regarding the company to intending shareholders, together with copies of previous reports and balance-sheets.

C. HETHERINGTON, Secretary. APPLICATION FOR SHARES.

To the Directors of the Mutual Trust and Investment Society (Limited).—I


hereby apply for    shares in the Mutual Trust and Investment Sosiety

(Limited), and inclose , being 5s. per share application fee.

Name in full.........................................

Address................................................

Date.............................. Occupation..........................................


PHYSICAL EXERCISE FOR GIRLS.

At a recent meeting of the London School Board, Mrs. Westlake said the resolution did not commit the Board *to any particular kind of physical exercise for girls ; it simply affirmed 'that the girls should be in the same position as boys, and that there should be some one to look after their physical training and development. For some years the drill-inspector had charge of the girls ; but, even if it were desirable that girls should be put through military drill, he had not time now to attend to them. Physical training was more necessary in the case of girls than in that of boys, as the home work of girls tended to check their physical development. It would be for the Committee to enquire what system should be introduced into their schools, and to report in the future, but they ought to affirm the principle that girls should have physical training as well as boys. They had had a very competent lady to give instructions in this training for two years to their teachers, and as to the value ol the instruction there could be but one opinion. The children were immensely improved in their growth, in their deportment, and in their work. But up to the present time the attendance of the teachers to receive instruction had been voluntary, aud the consequence was that their Code of physical instruction had been almost a dead letter. It was all very well to say that the teachers should receive instruction in the colleges ; but if they waited till the colleges imnroved their system they might wait for twenty years. Miss Davenport Hill aud herself had organised an exhibition of the system of physical training in the summer The members of the Board were all invited, and those who attended would be able to give their opinion. Medical men also attended and spoke in the highest terms of the usefulness of the instruction, and she would like to know where Mr Heller got his medical opinion. The effect of the traiuing upon the walk and bearing of the children was most apparent ; and the exercises were so popular that when they were given the attendance was always sure to be good. She urged upon the Board the importance of moral and physical as well as intellectual training for the children. They could not have sound minds without sound bodies Tiiere was a danger of doing the children physical injury in the schools’ Hiul the l>oard ought to provide against this by giving them good physical training. This, the Swedish system, had become universal in many countries, and was making its way gradually over the whole of the Continent. England was behind other countries in this matter.

ANSWERS TO CORRESPONDENTS.

“No. 2056.”—We do not charge for class of advertisement sent.

NOTICES TO CORRESPONDENTS.

Advertisements and other business communications should be addressed to the Publishers. No advertisements will bo inserted without a written order, or prepayment. It is particularly requosted that they may be sent early in the month.

Books, music, and school appliances for notice, and all letters containing anything connected with the literary portion of the paper should be addressed f.o the Editor. Every communication accompanied by the name and address of the sender (as a guarantee of good faith, though not always for publication) will be acknowledged; but we cannot attend to anonymous letters.

BIRTH.

Moncttu. — On the 27th November, at Townsend-st., Albury, N S.W., the wife of Alex. Moncur, State-school No. 2956, Mokoan, Benalla, of a daughter.

Australasian: Srljuulmastrr.

PUBLISHED EVERY MONTH.

MELBOURNE, DECEMBER, 1881. The Royal Commission recently appointed to inquire into and’ report upon the working of the State school system of education in Victoria has now fairly entered upon its duties. At a meeting held on the 21st inst., the chairman—Mr. J. Warrington Rogers, Q.C.—read a very ably-written memorandum showing the scope of the Commission, the subjects of inquiry, and the order in which, in his opinion, the business should be taken. Resolutions adopting the memorandum, fixing the date at which the examination of witnesses shall commence, and affirming the desirability of their meetings being open to the Press, were passed, and the Commission adjourned till the 11th day of January next.

After referring to the terms of the Royal Commission, Mr. Rogers remarks :—

“ As the commission directs all financial economic considerations to be regarded in reference to the maintenance of the efficiency of the system of public instruction, it follows that the inquiry must be entered upon in the twofold aspect of financial economy and efficiency. These two considerations affect the entire population of the country. The second subject of the inquiry, the ‘alleged grievances of a portion of the population,’ relates more especially to that portion of the population only which alleges the existence of grievances. The question of the existence of these grievances, and, should they be found to exist, the best means for their redress can be better inquired into after the question of the general expense and efficiency of the system has been exhaustively disposed of, especially as in the course of the larger inquiry much statistical information will be collected, which must have an important bearing in determining the direction of the more limited, although very important, inquiry which will remain, I would therefore suggest that the commission should, in the first place, direct its inquiry into the general administration and organisation of the existing system, and that the question of the grievances of a portion of the population should form a separate and subsequent part of the inquiry,”'

It is then pointed out that the inquiry respecting the administration of the Act should embrace : (1) Tire children ;

(2) the school-buildings; (3) the instruction and discipline of the schools; (4) the teachers ; (5) inspection and control; (6) the departmental expenditure. With regard to this branch of the Commissioners’ work, Mr. Rogers says :—

“The inquiry into the subject, ‘ The Children,’ at once divides itself into—1. The number of children in the colony of statutory school age ;

2.    The number and ages of children receiving education at state schools ;

3.    The number at state schools under the age of six ; 4. The number at state schools above the age of 15 ; 5. The average age at which the pupils attain the required standard ; 6. The proportion of boys to girls;

7. The number of pupils learning extra subjects ; 8. The number and ' effect of night-schools ; 9. The number and character of ragged schools ; 10. The largest number of pupils in any one school ; 11. The smallest number of pupils in one school ; 12, The number and effect of mixed schools ; 13. The number of separate schools ; 14. The number of pupils who have attended a greater number of days than the minimum number of days required by the act ; 15., The greatest number of attendances by individual pupils ; 16. The lowest number of attendances by individual pupils who have not been treated as defaulters ; 17. The number of children at private schools, or receiving private tuition. (The 10th section of the Education Act Amendment Act requires returns to be made to the department, but provides no penalty for false returns.) It will be from inquiries in this direction that the commission will be enabled to ascertain to what extent children of thriftless or very poor parents are excluded to make room for pupils of a class of

eople in better circumstances. ,    .    . The Third

ubject—‘ The Course of Instruction and Discipline.’ This subject may be divided into—1. The general discipline of state schools, including the question of the control by head masters over pupils after school hours, and especially in reference to the influence of the general system in training the youth of both sexes in good habits of morality, industry, and quiet demeanour. 2. The statutory instruction required by the act. 3. Instruction in extra subjects permitted by the act. 4. The time within which a pupil of ordinary capacity, well taught, should be able to complete his education up to the statutory school standard.”

With a view to economising time and expense, it is recommended that the evidence of teachers as to their ability and willingness (or otherwise) to give general religious instruction, if the law should permit of it being imparted, should be taken at this point of the investigation.

That the Commission may make an exhaustive report upon the status and work of the teachers, together with that of the school inspectors, Mr. Rogers divides this branch of inquiry into :—

1. The Teachers.—(a) The education of teachers and their necessary proficiency and moral character ; (b) their appointment ; (c) their classification; (d) their promotion; (e) their supervision ; (f) their remuneration by salary or otherwise ; {g) the proper proportion of the number of teachers to a given number of scholars ; (h) the rights and duties of teachers as to extra subjects. 2. Inspection and Control.—(a) The relation of the central department to the local authorities ; (6) the inspectors, their status, powers,duties, and remuneration ; (c ) boards of advice—qualification, election, duties, and powers of members. In reference to this branch of the inquiry,fit may be desirable to.consider how far the centralisation of the present system might be reduced by improving the system of election of the boards of advice, and giving to them an increased power and greater responsibility.

Speaking upon the vital question of “ Expenditure,” the Chairman pointed out that the cost of the Department for the year 1880-81 amounted to more than half a million. The remuneration of teachers being over ¿£400,000, while the maintenance allowances reached no less a sum than ¿£44,722,

i.e., for maintenance allowances to teachers for school expenses, ¿£29,684, and for maintenance of buildings, ¿£15,038. Upon this subject Mr. Rogers remarks :—

“It will be necessarjf to take evidence as to these large figures. It may be that the salaries and emoluments of teachers arc not in excess of what is necessary to secure efficiency, and that the number is not in excess of the requirements of the service; but it will, I think, be necessary for the commission to enquire closely into this question, as the subject of the appointment of teachers ‘greatly in excess of the number authorized by the regulations ’ seems to have attracted the attention of the audit commissioners in the report of the 7th December, 1880. It is to be noticed that the cost of the system has been gradually increasing. Possibly the expense of the system might be much reduced if pupils were not received into the schools before the statutory age of six years, and their education to the school standard was completed by the time they reached the age of 12 or 13 years, instead of extending the time over the period from 6 to 15 yeai-s.”

THE WORK OF INDUSTRIAL SCHOOLS.

The charges which have recently been put into circulation concerning the management of St. Paul’s Industrial School are calculated to make a very unpleasant impression on the public mind. The time has not yet arrived to pronounce a final judgment as to the validity of these charges. The whole of the evidence in support of them has not yet been received. It may be assumed, too, that to this, as to most cases, there is another side, and that other side, of course, must also be heard. There is some ground for the expectation that, when the facts of the case are fully brought out, its aspect may be to some extent modified. Not to speak of a tendency to exaggeration which the ungallant might characterise as feminine, it ought to be kept in mind that Mrs. Surr and several other members of the School Board for London are possessed by an inveterate feeling of almost fanatical dislike to the very idea of Industrial Schools, and that there is, what may be called, a standing feud between them and Mr. Scrutton, the chairman of the Industrial Schools Committee, in connection with this subject. At the same time, it must be admitted that Mr. Scrutton and his committee have been somewhat unfortunate in respect to some departments of their operations. Of the integrity or good-heartedness of Mr. Scrutton or any other member of the committee, no rational person can entertain the least doubt. But this is not the first time that the anti-industrial School party have been able to adduce complaints against their administration which must be confessed to have been not altogether unfounded. The committee—especially the chairman—did not come off with flying colours in respect to either the case of the Shaftesbury or of the Upton House School.

The worst effect which is to be feared from such complaints is that the public might be brought altogether to lose faith in Industrial and Reformatory schools. From this point of view, it is fortunate that Sir William Harcourt should have found occasion to speak at the opening of an Industrial School at Cokermouth, and more fortunate that he was able to give such gratifying information as to the success ot these institutions. The Home Secretary was able to adduce the most irrefragable ovidence that crime has diminished in a very remarkable degree since about the period that Industrial and Reformatory schools began to be. established, and especially since the large increase of elementary schools consequent'upon the passing of the Education Act of 1870. This improvement may not all be the effect of education in general or of the training given in Industrial and Reformatory schools. Other beneficial influences of similar origin and character have happily been actively at work during the same period.

These facts are, however, not only specially gratifying to all who have taken an enlightened and philanthropic interest in the cause of popular education. They are, it may be for different reasons, equally calculated to secure for the work of education, and for such institutions as Industrial and Reformatory Schools, the sympathy of the large class whose special concern is that they get value for their money.

The suggestions and hints thrown out by the Homo Secretary as to desirable changes in the law and its administration in relation to juvenile offenders we cannot now afford space to discuss. This is the less to be regretted that this subject has already received fi’om ourselves and the press generally considerable attention. With the tendency and aim of the proposed changes every humane and intelligent person must i sympathise. The great desideratum is to dimish the number of offenders, and to accomplish this result no other effective means can be employed than education. On this subject Sir W. Harcourt offers some observations which are equally applicable to all educational institutions, and which, it is to be hoped, will receive the serious consideration of managers of all kinds of schools. The right hon. gentleman, speaking of the school about to be opened, said, “I would say of this school what I deeply feel—’that its future success will depend upon the personal care and supervision which is bestowed upon it by those who have taken so much pains to found and originate it.” In confirmation of this conviction Sir William referred to “ a very painful example recently of a school which had totally failed where the whole system has gone to pieces ” because “there were no persons who have devoted themselves to the superintendence and care of the sojfool.” This is one very important practical lesson. It seems to us that this is a matter to which more attention ought to be given at School Board elections than it has hitherto received. There is a class of candidates who, when they happen to be elected, make a point of attending all public meetings of the Board, and of doing everything which tends to make themselves notorious, but who exercise no “ superintendence and care ” with respect to the real work which they are elected to perform. All such candidates should be decisively rejected, irrespective of party, personal, and every other consideration. Another lesson relates to the importance of securing rightly qualified persons to manage and to do the work of teaching. Speaking again of the new school at Cokermouth, Sir William says, “ I am quite certain, from what I have heard that one of the main things has been attended to. You have spared no pains or expense in getting the fittest master and manager • and, of all the economies, the worst is where you have an institution where everything depends on the individual who has control of it, you should take a second-rate instead of a first-rate man.” These are wise words and worthy of the man, the office which he holds, and the occasion on which they were uttered. —Schoolmaster.

SCHOOL BOARD MEMBERS’ TERM OF OFFICE.

The length of time during which School Board members should hold office has been discussed in the columns of The Times. It has been suggested that five years instead of three would be an advantage. It would, in our opinion, be a better plan if the elections were to continue triennial, with the provision that one-third or one-half of the members should retire by rotation. This would always secure a number of experienced members on the Board, and obviate many of the difficulties, if not the dangers, of the present system. The discussion has given one of the London members an opportunity of classifying an 1 describing his fellows at the Board. In the opinion of the Rev. H. D. Pearson there are two classes of members—those who with great pains try to gain a mastery of details, visit schools, and attend committees diligently; and those who are for the most part unwilling to do this, so appear only on the Board day, and make orations more or less prepared. It would seem, however, that these oratorical members complain that they are “left out in the cold but Mr. Pearson has a remedy for their grievances. He suggests that they should give a little attention to the business of the Board, and altogether depart from the ways of obstruction. It is not very pleasant to be told that the work of the Board is almost at a deadlock, in spite of the chairman’s exertions, because questions which might be settled in ten minutes occupy hours, while the most important educational matters are delayed. Among these, Mr. Pearson names the question of“ inclusive salaries for teachers.” If the public return members to the Board, great in promise, but in fulfilment almost next to nothing, simply because they are local bigwigs, or ambitious wire-pullers, altogether indifferent to the work which they are supposed to control, the blame must rest upon the public shoulders. Earnest men must sympathise with the feeling of dependency in which Mr. Pearson writes as follows :—“ It is very trying to have to listen to long, dreary, dis

cursive, acrimonious harangues from the least well informed among us, while those who, however they may differ, take a real interest in elementary education, whose whole heart is in the matter, who devote their time and whatever mental powers they possess to the work, who scorn to make the Board a medium for electioneering cannot but sometimes wish themselves to be anywhere else but at the Board.—Schoolmaster.

|lotcs of iln lltontb.

A deputation from the residents of Parkeville waited on the Minister of Education, to request that a strip of land through Trinity College Grounds, should be made into a pathway between the Sydney Road and Madeline Street, so as to form an easy communication between their residences and Carlton. The trustees of Trinity College appointed a deputation to object to this, and urge that their grounds should not be tampered with. The Minister has promised that a path shall be made through the University Grounds. As there is some talk of building new houses for the professors, and they need them, it would be an ornament to the grounds if an avenue were formed at this place, trees planted all along, and dwellings, with each aA"extensive piece of ground, erected at the side.

The first meeting of the Education Commission was held on Monday 12th inst. Preliminary business, only, was transacted.

Over £1,000 has already been subscribed towards a fundfor the foundation of a Girls’ Grammar School, in Brisbane, Queensland.

For the December Matriculation Examination, 674 candidates entered their names at the following places Melbourne, 429 ; Ballarat, 96; Geelong, 66; Sandhurst, 26; Stawell, 12 ; Creswick 12 ; Hamilton, 11 ; Warrnambool, 11 ; Daylesford, 7 ; Hobart, 2 ; Launceston, 2 ; Brisbane, 3.

^ The success of the ladies in the recent Honour Examinations, at the University of London, in arts, sciences, and medicine, was something wonderful. Two ladies have obtained their B.Sc. degree, with high honours, and several others have taken good honours in other courses.

A new series of the Melbourne Review will commence in January, 1882. The prospectus which announces this, says :—“ Under the impression that the Review might still be made to present as varied and attractive an appearance as was at first contemplated, the proprietors have made a change in the Editorial Management. The Review will in future contain shorter articles on a greater diversity of subjects. It will still be thoroughly eclectic, in the sense of being confined to no special policy in any subject; articles will, as heretofore, be admitted to its pages, no matter what side of the question they may take. It is intended, however, to exercise a strict supervision ; so that the liberty of free discussion may not lead to personalities, or to needlessly offensive references. The next number, to be published on the 1st of January, will contain articles by the following authors :—ft. Murray Smith, Esq., M.P. ; Rev. James Lambie, M. A. ; Mrs. Webster ; H. G. Turner, Esq.; W. E. Johnstone, Esq., LL.B.; Hon. Robert Stout ; with several others.”

Dr. Siemens, in a lecture at the Birmingham Midland Institute, which now counts 2688 students, objected to the national systems of education, on the grounds that they afford no encouragement to originality or innovation.

A meeting of the Public Schools Floral Society of Adelaide was held on Saturday, November 19tli. Mr. J. A.Hartley, B.A., B.Sc., presided, and there was a satisfactory attendance of members. From the Secretary’s and Treasurer’s reports it appeared that the Society was in a flourishing condition. A larger number of teachers had become members than iu any previous year, and the receipts from the late show were greater than usual. The Secretary was instructed to thank those ladies and gentlemen who had given their services as Judges ; and also Messrs. Boult and Fisher for their organ recitals. It was resolved that a first certificate of merit should be awarded to Mr. Masson, of the Central Model School, Lor his excellently drawn diagrams illustrating physical geography history, &c., which were exhibited at the Show,

Arrangements are being made for the establishment of a new Australian College of Agriculture. It is proposed that 11 scholarships shall be given by it for State schools.

A resolution, reported to Parliament from Committee of supply, agreeing to an estimate of expenditure on State school buildings, to £80,000, has been adopted.

The new system of examining at matriculation was introduced for the first time at the examination held this month; A few mistakes, that caused some confusion, were made at some of the examinations.

The examinations for teachers were held on the 15th and 16th inst. Some of the candidates object to the custom of breaking up the Arithmetic by the Reading examination. This system greatly hinders the nervous candidate, who needs to be helped by perfect questions rather than disheartened in the midst of his work.

The Australian Health Society’s examination on “ Health in the House” was held in November, and the results were made known a few days ago. The three first prizes were awarded as follows ¡—Frank Crowther, Hawthorn Grammar School, and George Halford, Hawthorn Grammar School, equal with prizes to the value of £4 each, and Jane Schutt, of the Presbyterian Ladies’ College, with a prize value £3. Other prizes were awarded—in all £20 worth.

The government have decided to grant the allotment near the University to the Roman Catholics, to be used only for college purposes.

Mr. Ellery, the government astronomer, has been requested by the Minister of Education to prepare an Elementary Text Book of Astronomy for use in state schools. This he has promised to do with as little delay as possible.

There is some chance of all children under six years of age, who certainly should not be pent up all day, being excluded from the State schools unless a small fee be paid for their tuition.

A CONCERT in connection with state school No. 1896, was held in the Prahran Town Hall for the benefit of the prize fuud, Mr. J. Stewart, correspondent of the Prahran Board of Advice, conducted the arrangements, and Mr. J. C. Rennie, the government singing master, the music.

The Congregational College, Melbourne, held its annual meeting, in the beginning of this month, in their hall in Russell-street. The Rev, E. Day presided. The Treasurer’s report showed the receipts to be £422 and the expenses £512,

The Sydney Health Society have been urging upon the Minister of Education for that colony the necessity for the introduction of instruction upon sanitary subjects into the public schools. It is likely that they will gain their wish, as the Minister has given a favourable reply.

A meeting of the Musical Association of Victoria was held at Glen’s rooms on Saturday, the 10th instant. Mr. C. Thompson was made an associate, and the Messrs. Fysh and M'Lachlan members.

From Brisbane we learn that the jury who tried Michael John Minnis, a school teacher, for the murder of William I’illinger, near Broadwater, on Sept. 30th, were unable to agree, and the prisoner will have to be tried again.

Mr. Ormond, with his usual fine taste, has determined to devote the large sum of £5000 to one of the best of objects, the establishment of a college where working men may hear interesting and instructive technical lectures.

A BAZAAR was held on the 15th instant for the benefit of the Children’s Hospital. A sum between £300 and £400 was realized.

The Christmas number of the Sketcher contains a fine coloured supplement with a picture representing “ A few early Squatters.”

A deputation from the Prahran schools waited on the Secretary for the Marquis of Normanby, to request his Excellency’s presence at the distribution of prizes for that district. The Secretary replied that it was too late to make arrangements this year.

J he Presbyterian Ladies’ College speech day was held at the Athenseum Hall, on Wednesday, the 14th instant; when the Hon. J. MacBain presided. The Head-master’s report was in every way satisfactory. There had been 10 more pupils during this year than last, and the health had been unusually good. Two girls had passed matriculation with credit, and one had obtained an exhibition.

The Carlton State schools have collected the handsome sum of £300 for the purchase of prizes to be distributed at the annual speech day, which will take place on the 23rd December, in the Melbourne Town Hall, The Mayor will preside.

The “Aragabond” has commenced the writing of a new play for Dampier. The scene is laid in America and Australia.

The South Australian Register in a leader on the school standards, says :—“The probability is that in any country where the standard aimed at is unduly high the pupils in a few of the schools are injured through cram, while the majority suffer from want of thoroughness.”

Sir James M'Culloch, on the Toorak College speech day, said that the State Education system did not reach the people for whom it was intended, as might be seen by the number of boys who sell newspapers in the street. He estimated the cost of building and repairing at £600,000 per year.

The Rev. C. Du Port, the Government Inspector of Schools for Berks, Wilts, and Surrey, speaking of the lack of culture amongst pupil teachers, tells that a lad who had reached the close of his apprenticeship as a teacher, being asked the meaning of some familiar lines from Hamlet’s soliloquy, “ To be or not to be,” answered—“ This passage means that when we are dead no dreams can disturb us then. By shuffling off this mortal coil means, by trying to get out of dying, which is impossible.”

The Fortnightly Review for October, in speaking of the advantage which school readers, published by private firms, have over those determined upon by Government, says :—“ Let any one place side by side the books from which children are now taught in the schools and the books which their grandfathers, or even their fathers used, and it will be impossible for him to come to any other conclusion than that immense improvements have been made, both in the matter and in the appearance of the books. These improvements are due to private enterprise, and nothing else.”

_ The average number of school hours in England is 36 a week ; in Germany 31 ; and in France 40. Or reckoning holidays, which in England are twice as long as on the continent, the ratios are as 6. 8 and 11.

An “ Inquirer” in the Glasgow Weekly News asks—“ Would any of your mathematical readers have the kindness to tell me how to get the equation—


AP2 _

AAJ'2 1 +


^&rom AP'2 = A Mi + PMi, AMI    ’


As given in 1 Todhunter's Trigonometry for Beginners,’ chapter II, article 23, page 11 ?” In the next issue “ Answerer” says—“ I think “ Inquirer” might with great advantage buy an Algebra for Beginners before commencing the study of Trigonometry. In case he cannot do so, however, I will give him a problem to study* As7 = 3 + 4:|=1 + |. When he thoroughly understands this, let him try the question he gives.


Science    anb    %ti    (gossip.

A young student named R. Sutton, forwarded to the Royal Society of Victoria a paper, which was read at the last meeting, on a new method he had discovered of storing electricity. He was requested to send a cell so that it might be tested.

The Victorian Electric Company have commenced to build a large place in Russell-street, and have arranged for the importation of several powerful engines to work their electric machines.

It is proposed to light the Flinders-street station by electricity. This will cost three times as much as at present, but the light will be greatly improved.

Nature, of the 29th September, pays a graceful tribute to the late Professor Pirani, who was a contributor to its columns.

The committee of the Victorian Academy of Arts have offered two silver medals as prizes to the “ life schools.” One for the best drawing in black and white of a male figure, and the other for the best painting in oil colours of a human head. The competitors for the painting medal commenced on Saturday last, and will be allowed eight Saturdays to complete the pictures.

M. Buvelot has completed another picture—A Scene at Bacchus Marsh—representing the green fields of an early summer, with a creek trickling through, and cattle straying in the distance.

Mr. Drouhet has invented an apparatus which, he believes, will facilitate the using of electrical lights in houses. This is effected by a combined system of batteries. Boxes containing stored electricity will be left at the houses and will be changed at regular intervals for fresh ones.

Mr. Fletcher, of the Art Gallery, in Collins-street, has received a large and fine collection of water colours by the last mail.

A WRITER in the Saturday Review says that, on the whole, were every house lighted by electricity, and every factory to receive its power in that form, the danger of accident would be much lass than that which now exists from gas and boilers, for no bad workmanship or carelessness could cause accident.

On the lake in the Public Gardens at Boston, may be seen a pleasure-boat which is worked like a bicycle. The paddle is located above the middle of the hull, and the wheel is worked after the fashion of a velocipede by a man who sits above it. It is encased in a metallic sheathing, which in turn is covered by a beautiful and elegant swan modelled in copper. The boat does not attain a very great speed, but glides beautifully and smoothly over the waters, and is easily propelled.

Wherever we turn in scientific journals we see the accounts of new inventions connected with electricity. The two latest are the “ Electric Lamplighter” and the “ Electric Balloon.” The latter is propelled by force obtained from a Dynamo-Electric Machine. If any method of propelling balloons is to succeed we believe that this will be the one, as the electric power is the most easily carried of any that has yet been discovered.

The Field Naturalists’ Club of Victoria has lately received some substantial additions to its library.

The Royal Society of Tasmania has received a very large addition to its library, consisting of about 130 bound volumes, together with a large number of official books, reports, pamphlets, &c., from Mr. E. S. Hall.

At the last meeting of the Royal Society of South Australia, Professor Tate stated that he had found, near Golden Grove, Quinetia Urvilla a plant not previously discovered in South Australia, though common in Western Australia, aud which was probably indigenous here, not imported from the latter colony as Baron Von Mueller surmises.

CARL Bock, late commissioner for the Dutch Government, has published a book of his travels in Borneo, called, “ The Head Hunters of Borneo.” Among other natural history facts noticed, the remarkable tenacity of life of the Loris Tardiqradus is told in the following words : — One day I wounded one, and knowing its tenacity of life, I strangled the little animal, then cut it open and pierced its heart. An hour elapsed before I wanted to skin it, and when I took down the body I found it still alive, its lovely eyes wide open. When, hoping to finally despatch it, I pierced its brain with a needle it began to shriek, and still some minutes elapsed before it was actually gone.”

One of the most interesting sights at the Paris Electric Exhibition was the Siemens’ Electric Tramway constructed by M M. Boistel and Sappey, the engineers for Messrs. Siemens. It is “ a first practical solution of electric traction in the case of a tramway.” It is quite plain that this electric power will soon be extended and developed so as to be applied to railways.

The Bolometer, a new instrument, invented by Professor S, P. Tanglcy, is capable of indicating a change of temperature as minute as 1-100,000th of a single centigrade degree.

Arrangements for an Electrical Exhibition at the Crystal Palace are progressing. A great feature will be the display of the Edison light at the Concert Hall. It is believed that Mr. Edison has solved the problem of how to produce a light to supersede gas in houses.

Nature, of Nov. 3rd, says :—“ Since 1869 the Otago (New Zealand) Acclimatisation Society has, we learn, liberated 157,041 young trout, and has sent 136,110 trout over to various parts of Otago. Since 1874 it has liberated 34,900 salmon fry, and in 1879 and 1880 it liberated 790 perch and sixty tench. Young American white-fish, let loose in the lakes in the Rotorua district about two years ago, have been recently met with by the natives, but as soon as it was discovered what the fish were they were returned to the water. Toe native.? are delighted at the discovery.”

Widmvm (BìmcvAiBn gtji'arímcnf.


The following are the papers set by the Department for a Certificate of Competency at the Examination held December, 1881 :

ARITHMETIC.—Set I.

(Time allowed two hours.)

1.    Explain the terms Notation, Multiplicand, Cypher, Volume, Ratio’ Concrete N umber.

2.    What number is that from which if you deduct eighteen millions and seventeen, and to the remainder add five times the difference between three hundred millions seven hundred and thirteen and seven hundred thousand eight hundred and ninety-five, the sum will be two thousand and eighty-six millions forty-nine thousand three hundred and two ? Numerate your answer.

3.    Find by practice the value of 97 cwt. 2 qrs. 131bs. 5]ozs. at £4 12s. 8]d. per quarter.

4.    If 80 horses and thirty-six cows eat up the grass of a 200-acre field in 110 days, how long would it take 90 cows to eat pip the grass in another field the area of which is ten-elevenths of the former ?—[N.B.—6 horses eat as much as 18 cows.]

5.    State and explain the rules for determining mentally—(a) What 3d. a day will amount to in a year, allowing G days to the working week. (¿) What G articles cost at 15s, a gross, (c) The product of 25 and 156.

G. At what rate per cent, per annum, simple interest, will £142 16s, produce for interest £23 lGs, in 5] years.

7, Simplify.

_L 2_

. 2t ~ 39 + »*______•

0yt ' T’l t °* ^2 ~ § (r~~


8.    What decimal of a cwt. is the difference between '0281 of a qr, and •025 of a lb. ?

(Answer to four places ?)

9.    Reduce to a simple decimal correct to six places—


(8-5 x -00085 x -085)- 0005 + .0085


•05068    69-574

' -905 + -86


- -056


[W.R,—Work by decimals.]

10.    How many gallons of water fall on a flat roof 92ft. 5in. long and 13 ft. 4 in. broad in a rainfall of -4 of an inch ? What height would the water in a tank whose area is 100 sq. ft. be raised by receiving the rainfall on this roof ? (277] c. in. = 1 gallon.) Answer to three places of decimals.

11.    If by selling an article at 10s. 6d. I lose 16 per cent, of the cost price, what would be my loss or gain per cent, if I sold it at 153. ?

12.    If a map drawn to a scale of 1] in. to the mile has an area of 4sq. ft. 138sq. in., what area of country is represented on the map? What is the distance between two places which are 5-]- inches apart on this map ?


BOOK-KEEPING.

13. (a) When I buy goods for part ready money, part credit, and part bills, under what accounts and on which side of them mu3t entries be made ?

(5) What are Bills of Exchange? Rule aud name the necessary columns for Bills Payable.


(o) Post the following transactions :—

1881.

&

s.

d,

Jan. 3rd. Goods on hand......

. 1,500

0

0

,, 5th. Sold Roberts and Co.—

£

s.

d.

GO lbs. currants at 9d.....

... 3

7

6

100 lbs. sugar at 5d. ... .

,. 2

1

8

---

— 5

9

2

,, 6th Bought of McPherson & Co.

80 cwt, sugar at 28s. ...

... 112

0

0

75 chests tea at £5 10s. ...

... 412

10

0

— 524

10

0

,, 7th. Cash sales this day ...

... •

,,

190

0

0

,, ,, Paid McPherson & Co.

,,

524

10

0

„ 8th. Lost a £100 bank note .

100

0

0

,, „ Received Legacy ... .

..

100

0

0


Set II.

(Time allowed three hours.)

1.    Explain the terms : Numeration, Subtrahend, Digit,SArea, Proportion, Prime Number.

2.    The sum of twenty-nine billions nine hundred and ninety-five thousand three hundred and sixty-four millions nine hundred thousand two hundred, and of five thousand and ninety-seven millions four hundred and nine is divided by a certain number, and the quotient is five hundred thousand aud seven. Find and numerate the divisor and the remainder.

3.    Find by practice the value or 2G acres 2 roods 12 perches 15] yds. of land, at £18 10s. 11]4 per acre.

4.    If the royalty on 65 tons of quartz, yielding 7 dwt. to the ton, is


6.    What principal will amount to £606 13s. 4d., in 3] years, at per cent, simple interest ?

7.    Simplify —


_ 1 4-1._    1

¥2 V 2 — TFF


? of 8


£8 10s. 7]d., when gold is worth £3 15s. an ounce, what will be the price of gold if the royalty on 100 tons of quartz, giving 5 dwt. to the ton, is £9 13s. 9d., the royalty in each case being the same percentage of the gross yield.

5. State and explain the rules for determining mentally—(a) What 15 shillings per week will amount to in a year. (b) The interest on £41, for 36 days, at 5 per cent. ? (c) The quotient when 9250 is divided by 250.

of 53

H

8.    Reduce to the decimal of 4 quarters the difference between ■£ of -] of -875 of a peck, and -A- of 2-85 bushels. Answer to three places.

9.    (a) Find the quotient when the sum of '0085, '08651, and ’4 is divided by the product obtained by multiplying "87 by the difference between "00413 and ’04136. (Correct answer to three places.) (b) Divide -4612608 by 2562-56.

10.    Find the expense of painting, at 6s. per square yard, the walls and ceiling of a room whose length and breadth are each 15 feet 3 inches, and whose height is 12 feet.

11.    If a man sells a horse for £19 16s., and thus gains 10 per cent, of the cost price, what would be his loss per cent, if he sold it !for £17 13s. ?

12.    A cistern has two supply pipes which can fill it in 4 and in 5 hours respectively, and two discharge pipes which can empty it in 3 and in 7 hours respectively. Supposing the cistern to be full at 5 p.m, on Monday, and all the pipes to be thrown open together at that hour, when would the cistern be empty ?

BOOK-KEEPING.

13, (a) What is the use of the Bill-book? Rule and name the necessary columns for Bills Receivable.

(h) Write out an “ Acceptance” in correct form.

(c) Post the following transactions :—

1881.

£

s.

d.

Jan. 3rd.

Cash on hand...........

. 1,500

0

0

Goods ............

. 1,700

0

0

„ 4th.

Bought of Robertson & Co.— £ s.

d.

130 yds. silk at 6s. 6d 42 5

0

170 yds. velveteen at 4s. 6d. 38 5

0

- 80

10 0

,, 5th,

Sold Hogarth and Co.— £ s.

d.

75 hats at 13s. 6d. 50 12

6

185 prs. boots at 18s. 180 0

0

212

12

6

And received on account ...

106

6

3

„ 6th.

Paid Robertson and Co., for silk ...

42

5

0

if

Paid clerk’s salary .........

15

0

0

a if

Paid annual subscription to Hospital

5

0

5

GRAMMAR.—Set I.

(Time allowed two hours-and-a-half.)

1.    Parse with full syntax the words in italics in the following :—.

Valour, religion, friendship, prudence, died At once with him, and all that's good beside ;

And we, death's refuse, nature’s dregs, confined To loathsome life, alas! are left behind :

Where we (so once we used) shall now no more,

To fetch day, press about his chamber door ;

From which he issued with that awful state,

It seemed Mars brohe through Janus’ double gate ;

Yet always tempered with an air so mild,

No April suns that e'er so gently smiled :

No more shall hear that powerful language charm,

Whose force oft spared the labour of his arm :

No more shall follow where he spent the days In war, in counsel, or in prayer and praise ;

Whose meanest acts he would himself advance As ungirt David to the ark did dance.

All, all is gone of ours or his delight In horses fierce, wild deer, or armour bright :

Francisca fair can nothing now but weep Nor with soft notes shall sing his cares asleep,

—Marvel, on the Death of the Lord Protector,

2.    Analyse according to Morell’s second scheme the following passage :—

But yet, because all pleasures wax unpleasant If without pause we still possess them present,

And none can right discern the sweets of peace That have not felt war’s irksome bitterness,

And swans seem whiter if swart crows be by (For contraries each other best descry),

The All’s architect alternately decreed

That night the day, the day should night succeed,

3.    Give fully the derivation of the following words, the meaning of

the component parts, and the language from which each is derived :_

Slaughteryard, inquiry, Southampton, polyglot, deprivation, octopus. '

4.    Give the general rule for the comparison of adjectives of two syllables. There are three classes of exceptions to this rule ; what are they? Give examples.

0.    What is a noun sentence ? Give examples of the various places which the noun sentence may take in complex sentences.

6.    Write each of the following sentences in a correct form, and state the reason for each correction :—(a) This account is very different to what I told you. (?;) They make such acquirements, that suit them for useful avocations, (c) Neither John nor Robert swim well. (d) Their etymology as well as their use show them to be adjectives.

7.    Give the name and number of the feet in each of the following lines:—

Lightly they'll talk of the spirit that’s gone And o’er his cold ashes upbraid him.

He shook the fragment of his blade,

And shouted, “ Victory !”

Set II.

(Time allowed two hours-and-a-half.)

1.    Parse with full syntax the words in italics in the following :—

“ Oh,” cried the goddess, “ for some 'pedant reign !

Some gentle James to bless the land again ;

To stick the doctor’s chair into the throne,

Give iaw to words, or war with words alone,

For sure, if Dullness sees a grateful day,

’Tis in the shade of arbitrary sway.

Oh 1 if my sons may learn one earthly thing,

Teach but that one, sufficient for a king ;

That which my priests, and mine alone, maintain,

Which, as it lives or dies, we fall or reign :

May you, my Cam and Isis, preach it long,

The right divine of kings to govern wrong.

Prompt at the call, around the goddess roll Broad hats, and hoods, and caps, a sable shoal;

Thick and more thick the black brocade extends,

A hundred head of Aristotle’s friends.

Nor overt thou, Isis, ovantvncj to the day (Though Christ-Church long kept prudishly away)

Each staunch polemic, stubborn as a rock,

Each fierce logician, still expelling Locke,

Came whip and spur, and dashed through thin and thick,

On German Crouzaz and Dutch Burgersdyck.

Pope, The Dunciad.

2.    Analyse according to Morell’s second scheme the following :—

My mother I When I learnt that thou wast dead,

Say, wast thou conscious of the tears I shed ?

Hovered thy spirit o’er thy sorrowing son,

Wretch even then, life’s journey just begun?

Perhaps thou gav’st me, though unfclt, a kiss,

Perhaps a tear, if souls can melt in bliss.

Cowper, Lioies on his Mother's Picture.

3.    Give fully the derivation of the following words, the meaning of the component parts, and the language from which each is derived ;—Wood-stock, bibliophile, bracelet, commingle, collateral, diagnosis.

4.    How are Primary Derivative Verbs formed? Give examples.

5.    Morell gives a list of words which may be used as “ Connectives of the Adjective Sentence.” Name them, and give examples.

6.    Write each of the following sentences in a correct form, and give the reason for each correction :—(a) When we seen these sort of things going on we run off as quickly as we could. (b) A lot of us, asknowed the lay of the country, started for Mount Feathertop. (c) I would not dress like you do without you paid me for it.

7.    Scan the following verses, giving the name and number of feet in each line :—

Sweet are the uses of adversity,

Which, like the toad, ugly and venomous,

Hides yet a precious jewel in his head.

Know ye the land where the Cypress and Myrtle Are emblems of deeds that are done in their clime.

GEOGRAPHY.—Set I.

(Time allowed two hours.)

1.    Explain the lunar method of finding the longitude.

2.    Specify the causes on which rainfall depends, and give some instances of excessive rainfall.

3.    The mean pressure of the atmosphere is estimated as greater in the region of the “ Horse Latitudes,” and less in the regions of the Equatorial and Polar calms. To what causes is this owing ?

4.    In what parts of the world arc the following chiefly carried on :— The cod, herring, whale, pilchard, and anchovy fisheries ?

5.    Draw a map of Africa, south of Walvisch and Sofala Bays, to the Cape of Good Plope, and mark on it the following :—

States, S)'c.—Cape Colony, Natal, Kaffirland, Orange Free State, South African (or Transvaal) Republic,{Great Namaqua Land, Country of the Bechuanas, Zululand, and the Kalahari Desert. Ilivers.—Limpopo, Orange, Vaal, Tugela. Capes.—Delgado, Good Hope, Corrientes, Agulhas. Bays.—Table, Delagoa, Algoa, Santa Cruz. Mountaiois.—Draken-berg, Nieuwveld, Zvvarte Bergen. Towns.—Cape Town, D’Urban, Graham’s Town, Pietermaritzburg.

6.    Where and what are the following : Mention any circumstances for which they are severally remarkable :—Warrego, Mocha, Stoke-upon-Trent, St. Helena, Utah, Jena, Feathertop, Socotra, Coruna, Titicaca, Batoum, Versailles,

Set II.

( Time allowed two hours.)

1. Describe the consequences which arise from the inclination of the earth’s axis to the plane of its orbit.

2.    Show by diagram aud description the boundaries of the various belts or zones of the calms, and of the prevailing winds.

3.    When it is noon at Philadelphia, what is the time at Belgrade, their longitude being respectively 75° 9' W. and 20° 30' E. ? Explain the principle of ascertaining longitude by the difference of local time,

4.    Where are the manufactures of lace, porcelain, linen, and opium chiefly carried on ?

5.    Draw a map of the Baltic Sea, showing the river mouths and the towns on the coast.

G, What aud where are the following? Mention any circumstance for which they are severally remarkable :—Puy dc Dome, Jumna, Mara-caybo, Abrupt, Mobile, Torres-Vcdras, Barcoo, Hartz, Tycrs, llagusa, Hong Kong, Albany.

SCHOOL MANAGEMENT. Set II.

(Time allowed two hours.)

1.    (a) State the various subjects included under the “ organisation” of a school. (¿>) Explain the objects to be aimed at, and the errors to be avoided, in drawing up a time-table ; (i) for a single class under one teacher; (ii) for a school under one teacher ; (iii) for a school where several assistants and pupil-teachers are employed.

2.    State—(a) How you would ascertain in what class to place a new pupil. (b) How aud when you would determine to transfer a pupil from one class to another.

3.    Describe briefly the Home Lessons you would give to a fifth class or each day of one week, mentioning exactly the nature aud quantity of work in each subject.

4.    Give six problems in arithmetic suitable for a fourth class ; such problems being framed to show how to apply rules already learned to practical questions. State how you would explain to the class how to reduce one of the problems to an example in known rules.

5.    State what comments and explanations you would give on the following passage occurring in a reading lesson of the fifth class :—

Triumphal arch that fill’st the sky When storms prepare to part 1 I ask not proud philosophy To teach me what thou art.

Still seem, as to my childhood’s sight,

A midway station given For happy spirits to alight Betwixt the earth and heaven,

Can all that optics teach unfold Thy form to please me so,

As when I dreamt of gems and gold Hid in thy radiant bow?

6.    What is meant by moral education ? To what extent does it form part of State School education ?

7.    When and where should the following entries bo made in the school records :—“Class at Result examination” ; Number of cancelled attendances” ; “ Teacher’s arrival” ; “ Number present at drill” ; “ Transfer of pupil” ; “ Number of meetings in the quarter.”

DICTATION;

( Time allowed one hour and a half.)

“ Corn is the food most convenient and most suitable for man in a social state. It is only by the careful cultivation of it that a country becomes capable of permanently supporting a dense population. All other kinds of food are precarious, and cannot be stored up for any length of time : roots and fruit arc soon exhausted ; the produce of the chase is uncertain, and if hard pressed ceases to yield a supply. In some countries the pith of the sago palm, the fruit of the bread-fruit tree, the root of the esculent fern, and similar food, supplied spontaneously by nature, serve to maintain a thinly scattered and easily satisfied population ; but man in these rude circumstances is invariably found depraved in body and in mind, and hopelessly incapable of bettering his condition But the cultivation of corn, while it furnishes him with a supply of food for the greater part of the year, imposes upon him certain labours and restraints which have a most beneficial influence upon his character and habits;—R.R., Book vi., pp. 123 4.

COMPOSITION.

(Time allowed one hour.)

Write an essay upon or reproduce the substance of the lesson on— Coal, Cleanliness, The Burning of Moscow.

PENMANSHIP.

( Time allooved quarter of an hour.)

Large ITand.—Electrolysis. Small Hand.—Copenhagen, the capital of Denmark.


|tciu £ citi ¿mb (Bìmcaiioa gcjrartmcnt.


The following are the questions set at the Result Examination of the Public Schools in the District of Westland, December :—

Sixth Standard.

1.    Writedown all the sums of money between £149 19s lOJd, and (including these two) add them together.

2.    A person buys a horse for 55 guineas, keeps it 37 days at a costof 7s 10-j- a day, and sells it for £52, lie might have hired a horse for 15 8d a day clear. By how much is one plan more profitable thant he other ?

3.    E’ind the G.C.M. of G72, 1440, and 3172 ; and what is the small es number they will all divide without a remainder?

4. 10 - (I* + 3* + 2-A-+A)- ? : If of *4- 3h of * - ?    .

o. Reduce 7s 4d to the fraction of 2s 9d, and what fraction of 91b. 9oz. 12dwt. is 61b. 7oz, 16dwt. ?


6.    Find the sum of 7*907, 8*365, 9*0724, *00615, and 11*25 ; the difference of 1*9675 and *00212 ; the product of *0092 X *167 ; and the quotient of 854*2296 -+ *00192.

7.    A gentleman, after paying an income tax of 9d in the £, has £225 6s 6d remaining. What is his income ?

8.    As 156 : 78 : : ? : 550, Find the third term.

9.    8263 @£99 19s Ilfd.

10. 8cwt. 3qr, 111b, 8oz. @ £4 16s Sd per cwt.


Seventh Standard,


6. Solve the equation

x — a x + a lax


a -1 a + b a'ì - b'2 2ax


1.

2.

3.

4.

5.

6.


Find the value of:


.o


31


■2 *-U


lx

U


Find the value’,of f of -£b of -gj* of 17s. Gkl.

What fraction is2| acres of T7C of a square mile? Reduce *00176 and 12*4583 to vulgar fractions. Reduce 13cwt. 2qr. 131b. 9oz. to decimal of 1 ton. Find the value of 17 square yards 8 feet 37 inches


3s 4-Jd. per


yard.

7.    If the costof carrying^ ton of goods 185 miles be 12s 6d, how many cwt. could be taken 240 miles for the same money ?

8.    If £16 4s be paid for lodgings from July 10th to November 22nd what should be paid from September 3rd to October 2Sth ?

9.    If a 41b. loaf cost 7d when wheat was 42s per qr., what should be the weight of a twopenny loaf when wheat is 48s per qr. ?

10.    Find the interest on £2000, from January 5th to February 9th, at 6 per cent.

11.    What principal would produce £68 Is 3d interest in 3 years at 4 per cent ?

12.    A can do a piece of work in 26 days which B can do in 32 days ; they work together 4 days, then A leaves off. In what time will B finish it ?


MATRICULATION EXAMINATION.

October Term, 1881.

The following are the Algebra and Arithmetic Papers set by the Melbourne University, with solutions :—

ALGEBRA.—Pass.

Solutions by J. Sutherland.

1. Divide x3 -\-y3 +z3 + 3y2z + 3yz2 by x-xy —xz + y- +

2 yz + z2.

x2 — an/ + y2 -xz -f 2yx + z2)

x3+y3+z3 + 3y2z + 3yz2 ( (x + y) x 3 -\-y 3 —x'2z+ '2xyz -j- xz2 — xyz-\- 2y2z + y2z x2z + z3 + y2z--j- 2yz2 +xyz - xzx2z + z3 + y2z + 2yz2 +xyz-xz2

Answer, x + y + z.

2.    Prove that (a + b -I- c) (be + ca + ab) - abc = (!> + c) (c + a) (a + b)

(a-^-b + c) (bc + m + ab) -abc-=(a + b) (bc + ca) + (a + b) {ab) +c(bc + ea)

+ abc — abc.

= (a + b) (bc+ca + ab + c2)

— (a + b) (b + c) (a + c) Q.E.D.

3.    Find the Greatest Common Measure of x3 — 7x— 6, x3 — 13x+12

x3-7x-G)x3 - 13a +12(1 a:3lx— 6


-6.T+18

x — 3 ) x3 — lx — G ( x2 + 3x a- 2

x3 - 3x2


3x2 — Tx — C 3x2 —9îc


2a:-6 2a:— 6


a2 - ¿>2

. *. a2 — x(b - a)

. . x = -— . Ans. b-a

7. Solve the simultaneous equations

x + 2y + 3z = 5 ( 1 )


ax — as + bx-ab — ax — ai + bx + a a2 — b*2 2a2 + 'lab = lax


2a* + 3y + z = l (2)

3x + y + 2z = 6 (3)

2z + 4y + Gz = 10 y + 5z = 9 3x + Gy + 9z = 1 o 5y + 72 = 9 5 y + 25a = 45 182 = 36 .*. 2 = 2 y= -1 x = 1

8.    A’s annual income is half of B’s and B spends £60 a year more than A. At the end of two years A has saved £200 and B £600. What are their yearly incomes ?

Let x be J’j income, then 2a; is B’s. y + 60 is B’s.

2x - 2y = 200

4a;-2y-120 = 600

9.    Solve the equation

2a; + 1    2a; - 3


Multiply (1) by 2 Subtract (2) Multiply (1) by 3 Subtract (3) Multiply (4) by 5 Subtract (5)

From (4)

„ (1)


(4)

(5)

Answer.


Let y be i’s expense, and Ans.


2a: = 520 a; = 260


2 + 2


(x + 5) (x+ 1 j "^2a - 1 (x + 5) (x - 1)'

• *•    ~ 2x + x - l + 2x2 + 10a; - 32 - 15 - 22 - x -

3x- + 32- 18 = 0 ^ _ 3 + /9l 216    -3 + 15

6 : = 6 '


22-2 = 0


— 3 or 2. Ans.


10. A rectangular court is a feet long and b feet broad, and a path of uniform width runs round it. If the area of the path beone m,h part of the remainder of the court, find the width of the path.

The area of the path is-T^- feet

7)1 + 1

Let x be the width.

2ax + 2a(b-2x)=-ab vi +1


-422+ 2 (a + b)x-


ab

m +1


= 0.


-2(a + bU ç


16ai

- + 4 (a + b) -m r 1    '    '


-8

2(a + b)+    /4«2 m + 4b^vi + 3abm + 4aa — 2ab + b%

“ ^    vi T1


= ^ j a-\-b+ •a2m +b2m + 2abm +a2 - 2ab + b‘< ^    m +1


Ans,


ARITHMETIC.

Solutions by J. Sutherland

1, Write down in words the quotient and remainder obtained by dividing six hundred and seventy-one thousand five hundred and forty-three millions two hundred and sixteen thousand four hundred and thirty-eight by the remainder obtained by subtracting forty-five millions six hundred and seventy-nine thousand from five hundred and twenty-three millions six thousand and forty-two.

523006042 - 45679000 = 477327042.

477327042) 671543216438 (1406 47732704*2


Answer x- 3.


Reduce to a single term-


1


7* +12


+ -


1


1942161744

1909308168


a*2 - 02 + 6 1

+ ■


a*2 - 62 + 8


1


Jx + 12 X2 - 5a* + 6 -6a* + 8 (x -4) (x -3) 2    x- - 3a: + 2 + a 2 - 5a* + 6 - 2a*2 + 82


(x - 4) (oj- 2)    O-l) 0-2) (x -3) (2

5. Solve the equation


-4)


0-4) o-l) - = 0. Ans,


3285357638

2S63962252


421395386


x + 10    2 + 1

o' + *


x + 11 x + 2

* + -


x +10 x+ 11 C0 ¡ 8 ~ x + 9


æ + 8 ' x + 3    a?+ 9 1 a: + 4

x + 2 x+l x2 + 10.2 + 9a: + 90 - o;2 x + 4 x + 3''    22 +92 + 82 + 72


8a; - Ila:-88


x2 + 3a* + 2a* + 6 - aff - 4a; - x - 4 __2_ 2

-t- 4a; + 3a; + 12    ' '22 + 4a; + 3x + I2=a;2 + 9a: + 8a; + 72

V £Si + 17a:*i-72«=«:2 + 7a; +12 .*, 102= ~G0 ,*,2=—6, Ans.


Answer—One thousand four hundred and six.    Remainder—Four

hundred and twenty-one millions, three hundred and ninety-five thousand, three hundred and eighty-six.

2. Give in a decimal form the result obtained by simplifying—

This expression = x    x + x -R- = f + T23- = T°3 = = 3.

Answer—3*


3. State the rule for the calculation of compound interest. What is the difference between the simple and compound interest upon £45 for 2 years at 44 per cent ?

To find the compound interest:—Find the interest for the first year ; add it to the original principal ; call the result the second principal ; find the interest on this for the second year ; add it to the second principal; call the result the third principal; find the interest on this for the third year, and so on.

Simple interest on £45 for 2 years is

45x9x2 81

200 ~20 ~£41s*

The compound interest is 1*045 x 1 045 x 45 - 45 pounds.


49T41125

45

4-141125

20

2-8225

12


8. Find the cost of making a tennis court 103 feet in length and 39 feet m breadth at a cost of 3 -4A per square yard.

“    103

39

927

309

.    .    4017

3s. -4 is a sixth of a pound, and h is one twenty-fourth of a shilling.


6)4017


1 -015 1-045

5225 4180 10450

10*92025

45

5460125 4368100

49-141125

.*. The compound interest is £4 2s. 9’87 The difference is Is. 9-87d. Answer.

4. Find the value of l||+l| + |y|?-4^ by decimal and by vulgar fractions.


9-8700


The expression equals 18 + J + f + T?,--g +

,0 250 + 400 + 24-= 1S+ 1000 Also 13-25 +1*4 + 8-024 - 4-125-= 22-674 - 4-125 = 18 549.


125_    549

~lb1000-


A ns.


669 10 S 7


9)677 17 44


15 6 44


Answer. £75 6s 4M.


24 ) 4017 ( 167 24

161 144

177

168


9.    A boat’s crew rowed four miles in twenty minutes against a stream flowing three miles an hour. How long would they take to row the same distance with the stream ?

They can row 4 + 1 + 1 miles in 20 minutes =» 6 miles in 20 minutes .-. it will take them + x 20 minutes = sv° = 134 minutes. Ans.

10.    Three persons whose estates are worth respectively £1000, £755, and £645 per annum, buy 100 railway shares among them, each buying a number proportional to his estate. How many shares does each buy ?


1000 755 645

2400

Answer.


(1)    buys ÜH x 100 = ^ =41

(2)    '

(3)


2 +0 0 a 1 :> i ■! 0 if


x 100 = x 100


31 lì-

m


411, 311£ and 26-7,


24(3046§(126

24


64

4S

166

144


= 1H.


Find the square root,


H


5. A bankrupt who owed his creditors £3046 13s. 4d. possessed property, the total value of which enabled him to pay a dividend of 6s. 114d. in the £. What is the loss sustained by the whole of the creditors ?

They lose £1—6s. Ill in the pound =* 13s. O.j, in the £.

.-. out of £304G| they will lose 3046§x 13 + 3046^ + 24 shillings. 3046§

13

39606 8 126 11J

20) 39733 74 1986 13 74 22|

24

£1986 IBs. 7^d. Answer.

6.    How much per cent, is £34 of £55 ?

rate = 100 times the interest divided by the principal.

rate = 10°^«®=£61*

55    11

Answer. £GlTflT per centum.

7.    The area of a circle is found approximately by multiplying the square of the radius by 31416. What is the radius of a circle whose area is 3277"o99864 square feet?

Divide 3277*599864 by 31416 and we obtain the square of the radius 3-1416 ) 3277-599864 ( 1043 29 3141 6

135999 •

125664

103358

94248 91106

62832

282744

282744

3

1043 29 (32-3

9

62

143

124

643

1929

1929

Answer, 32-3.

Curious relics of bygone times arc picked up at times and in out-of-the-way places, but we, The Schoolmaster, did not think to find one in such an unexpected quarter as the advertisement columns of our contemporary, which more especially represents the interests of the managers of the Church schools. In a recent issue the following advertisement appeared Mistress ; Gardener; Choir.—Wanted, at Michaelmas certificated Mistress, for Mixed School. Must be thorough Church-woman, good disciplinarian, and successful Teacher. Sunday-school. Good house (mainly furnished), and garden. Salary £40, and half grant, which this year is £22 14s , but might be much increased. Hus. hand as Gardener, 4'c , and sing in choir Wages about 12,y. a-wed-. Address, Vicar.” Shade of Sir James Kay-Shuttleworth. what have we here? Has the world rolled back and brought the cobbler’s lap-stone and the housewife’s vegetables again into the schoolroom ? The worthy Somersetshire vicar does not say which lie wants most—the schoolmistress or the gardener—or why he thinks it likely he shall find them so commercially joined in wedded harmony. He has a mind, however, not to be misunderstood. The mistress of the mixed school might be so ambitious as to think that the vacancy for her husband was one of those delightful situations bringing in hundreds which gentlemen’s head gardeners deservedly obtain, so the vicar is careful to specify the wages —about 12s. a week, not those of a workman’s slab. The benevolent vicar may be desirous of indicating a desirable possible opening to some aspiring youth working in a neighbouring village, on marriage thoughts intent, but withal would desire to get quantity, if not quality, for his money—a schoolmistress, a gardener, and a bass or tenor for the village choir, all for the magnificent sum, say—for the language is rather ambiguous—of £93 4s. a year. In this incongruous association of offices the wife is the bread-winner, and the weight of the household is to fall on her, and this is the position which a clergyman of the Church of England thinks suitable and proper for the certificated mistress of the village school.

BEAT) TEACH EE Country School, Salary £132, residence free, desires • to exchange with any assistant in Melbourne or Suburb. Address G. Wylie, Esq., 67 Drummond street North, Ballarat.

EAD TEACHER, rising township, permanent allotment 30 x 50 Workmistress Position vacant, desires exchange after holidays! Would accept lower allotment, locality and other requirements being suitable.' Address, “Inducement,” Schoolmaster Office.

HEAD IEACHER, country, 30 x 50, results 88"235, wishes exchange Assistant, Melbourne or Suburbs. Address—“M.B.. care of Mr Eoth, Tivoli Place, South Yarra.”

HEAD TEACHER, Country School, good 30 x 50, inspector’s percentage 75, wishes to exchange with another, same allotment. Addres— “Scribo,” Schoolmaster Office. 2 1

T


ATE’S PARCELS POST EXPRESS

FIXED PEICE.

NO EXTEAS. NO TEOÜBLE.


Delivery to door at any address in

21b

8. (1.

41 b, s. d.

61b. 8. d*

101b. s. d.

201b. s. d.

Great Britain ...

4 6

6 0

7 6

10 0

14 0

Continent of Europe, America, & Canada

7 6

!

9 612 0

16 0

21 0

Sydney, Hobart, Launceston

2 6

3 0

3 6

4 0

G 0

New Zealand Ports (except West Coast) Adelaide, Brisbane.

4 0

4 6

5 0

5 6

7 6

No further charge whatever. Very small increase for heavier weights. Delivery at country addresses in Australia, inland carriago only added.


Eeceiving office—

FREDERICK TATE, 13 Market-st., Melbourne T A T E’S FAROE L S POST EXPRESS, DELIVERY at DOOR any address in the world. EITHER to- or from Britain from 4s. 6d. TO or from other Australian ports from 2s. 6d. NO further CHARG-E whatever. No trouble. Any SIZE, weight, or shape.

K ZK    possible ASSISTANCE afforded.

INQUIRIKS plainly answered.

Henceforth Parcels handed to YV. E. SUTTON’S Branches, every town in Great Britain, delivered in Melbourne at nearly similar rates.

Head Office :—

FREDERICK TATE, CUSTOMS AGENT, &c., 13 Market-street, Melbourne.


0OMPETENCY    EXAM INATION.

TUITION—

IN CLASS, BY CORRESPONDENCE, OR PRIVATELY.

Other Work, by Arrangement.

JAMES L. ROBERTSON, B.A., 71 C.LARENDON-BT., EMERALD HlLL.


^LEX. MCKINLEY & CO.,

GENERAL

PRINTERS AND PUBLISHERS, 61 Queen-street, Melbourne.


In Bookwork and General Publishing our large experience is a guarantee of all work being executed in the best stylo, while having a first-class stock of the best and latest material ensures expedition and good workmanship.

The following papers are issued from this office ;—


WEEKLY.

“Punch,”    “Bulletin,”

“ Faithful Words,”    “ Once a Week,”


FORTNIGHTLY.

“Australian Law Times,” “Jewish Herald,”

MONTHLY.

“ Schoolmaster.”    “ Monthly Messenger.”


BUY

AND READ

THE NEW

WEEKLY PAPER.

Price Threepence.

uON CE

A WEEK/5

1G PAGES.

PRICE THREEPENCE.

ALL

BOOKSELLERS.


Alex. M'Kinley and Co., Publishers, Cl Queen Street, Melbourne.


I WARREN BALL’S “Hints to Candidates • for Teachers’ and Matriculation Examinations,” Is.; posted, Is. Id. Mullen, Melbourne.


Q C. EXAMINATION.

TUITION BY CORRESPONDENCE.


MR. THOMAS BOARDMAN, First-class Honor-man of the Denominational School Board, Prepares Teachers for the Certificate Examination by Correspondence. Terms moderate.

Address—

45 PRINCES STREET, CARLTON


TAMES CLEZY, M.A.,

O    MELBOURNE.

CLASSICAL & SHAKSPEARE SCHOLAR (1869)

PREPARES CANDIDATES For Matriculation (pass or honours), and for the subsequent Degree Examinations of the University.

Course of Lessons by Correspondence in Latin Grammar, Translation, and Composition,

Terras on application personally or by letter. 5 GORE STREET, FITZROY.


Education act amendment act, i876.

PRIVATE SCHOOLS.

Proprietors and Principals arc reminded that by Section 10 of the Education Act Amendment Act, 1876, they are required to furnish to the Education Department, each year in the month of January, a return showing the name and surname, sex, ago last birthday,residence andjnumber of school-days attendance of each child who attended their respective schools during the preceding year, j Forms for the purpose may be obtained from this office upon personal or written application.

N.B.—Before transmitting the returns to the Department, principals and others should be careful to see that the forms are properly fdled up and signed.    T. Bo lam, Act ing Secretary.

Education Office, Melbourne, 1st December, 1881.


CANDIDATES for EXAMINATIONS prepared by correspondence or otherwise. I. Warren Ball, South Yarra.


HEAD TEACHER, about 20 miles from Ballarat, wishes to exchange with an Assistant in large Town. Address—“ A.B., care of Vale, Sturt-street, Ballarat.”


HEAD TEACHER, Protestant in Catholic com, munity, desires exchange into Protestant district. Average between 40 and 50 ; results, 72; workmistress vacant; 50 miles from Melbourne, 7 from raiiway_station, Post-office. Very salubrious; wood, water, abundant.    Address—“Toby, General

P.O., Melbourne.”


s.


MULLE


S


New Classified Catalogue of School, College, and Technical

EDUCATIONAL WORKS

May be had gratis on application, or posted on receipt of address.

SAMUEL MULLEN,

Wholesale Sc Retail Bookseller & Stationer, 29 & 31 COLLINS ST. E., MELBOURNE.


Price One Shilling,

By Post—In Victoria, Is. Cd. ; Out of Victoria, 2s


D


EPAKTMENTAL EXAMINATION OF TEACHERS.

TEACHERS5 GUIDE


TO

AUSTRALASIAN EXAMINATIONS.

Containing the

PROGRAMMES & EXAMINATION PAPERS

of

Victoria    South Australia

New South "Wales    Auckland, N.Z.

Queensland    Wellington, N.Z.

And Tasmania.

108 PAGES WITH STIFF COVER.


Tho above book contains the Programmes and Examination Papers of December, 1877, of all the colonies, and is reduced to the low price of ONE SHILLING.

By Post—In Victoria, Is, 6d.; Out of Victoria, 2s

ALEX. M‘KIN LEY & CO., PRINTERS AND PUBLISHERS, 61 QUEEN STREET,

MELBOURNE,


T


0 HEAD MASTERS, SECRETARIES OF BOARDS OF ADVICE, AND OTHERS.


ALEX. M'KINLEY & CO.,

Having made considerable additions to their stock of Bookwork and Jobbing Type, are prepared to execute orders in every description of

GENERAL PRINTING.


All orders entrusted to them will be printed in a satisfactory manner.


Alex. M'Kinley & Co., Printers, Gl Quben-st.


NOW READY.

ILTON    PARSED.

Price 2s.

By J. J. BURSTON,

(Author of “State School Arithmetic”).

Also Ready, the Fourth Edition of the

STATE SCHOOL    ARITHMETIC*

By

JOHN J. BURSTON,

North Sandhurst State School.

Printed and Published by Alex. M'Kinley & Co., Cl Queenstreet, Melbourne, under the auspices of tbo Victorian Teachers Union.


REVIEW.


AND LITERARY

Yol. III., No. 31.


JANUARY, 1882.

Subscription

Yearly, 6s. 6d. ; Half-yearly, 6s, Cd

BLACKIE

Comprehensive

& SON’S

School Series.

the Numbers.

9.    Sentences.—Two lines on each

Page.

10.    Plain and Ornamental Let

tering.

11.    Exercise Book.—Wide Ruling

with Margin.

11£. Home Exercise Book,—Same as No. 11, but 8vo size. Price Id.

12.    Exercise Book. — Ruled in

Squares.

12£. Home Exercise Book.—Same Ruling as No. 12, but 8vo size Price Id.

13.    Exercise Book.—Ruled for

Book-keeping.

14.    Essay Book.—Ruled for Com

position, &c.

15.    Exercise Book eor Begin

ners. Ruled for Small Text. X. Copy-Book Protector and Blotter. Keeping the Books Clean. One Penny.


Adopted by the London, Liverpool, Manchester and other School Boards, and by the National Board of Education in Ireland.

VERE FOSTER’S WRITING COPY-BOOKS

The efforts of Publishers to provide teachers with the means of properly training their pupils in the Art of Writing deserve the highest commendation ; and no names stand higher in this department of scholastic work than Vere Foster and the Messrs. Iilackie."—Educational News,

Opinions of H.M. Inspectors in Education Blue Book Reports.

Mr. M‘Cdllum, ~] H.M. Inspector, < says— 1

| “ More progress is made by Vere Foster’s than by any other 1 method which has come under my notice.”

Mr. Brewer, ( H.M. Inspector, says(

“With hooks like Vere Foster’s there is no excuse for the slovenly no-style of writing too often presented to me.”

Mr. Warburton, ( H.M. Inspector, says— (

“ I wish that the use of the excellent copy-books such as Vere Foster’s ivas begun earlier and more persisted in.”

Mr. Newell, ( H.M. Inspector, says— (

I know of no series by means of which children can be so quickly taught to write with freedom and legibility.”

Rev. J. Lomax, ( H.M. Inspector, says(

The introduction of Vere Foster’s copy-books in some of my schools has been attended with marked success.”

Superior Edition, 2d. each number. Popular Edition, 1J. each number.

Contents of

1.    Strokes, Easy Letters, Short

Words.

1&. Long Letters, Short Words, Figures.

2.    Long Letters, Short Words,

Figures.

2£. Words or Four, Five, or Six Lettrrs.

3.    Capitals, Short Words, Fig

ures.

3£. Sentences of Short Words.

4.    Sentences. Mostly composed of

Short Words.

4£. Select Quotations from Shakespeare.

6.    6. Sentences.—Maxims, Morals,

and Precepts.

6*. Sentences, in Writing of Three Sizes.

6^. Sentences, in writing of Two Sizes.

7.    Sentences, and Christian

Names.

8.    Sentences.—One Line on oach

Page.

THE COMPREHENSIVE READERS.

Each Book is Illustrated in a highly instructive and artistic manner.

Primer I.—32 pp.,..................paper cover, l£d.; cloth cover, 2$d.

Primer II.—48 pp.,.................. ,,    2d.;    „    3d.

Primer Complete—80 pp.,............................. „    4d.

Reader I.--64 pp., (Abridged Edition,) ............. „    4d.

Reader. I.—96 pp.,....................................... cloth boards, Od.

Reader II.—128 pp.,...................................... „    8d.

Reader III.—196 pp.,.................................... „    Is.

Reader IV.—288 pp., .................................... „    Is,    6d.

Reader V.—320 pp.,....................................... „    2s.

Reader VI.—384 pp.,...................................... „    2s.    Od.

“As specimens of good reading books, they cannot be excelled, The exercises are carefully arranged so as to suit the meanest capacity, and at the same time have a tendency to make useful impressions on the minds of young scholars.”—Educational Guide.

ADDITIONAL READING BOOKS.

The NEWSPArER Reader, Selections from the Leading Journals of the Nineteenth Century on events of the day. By Henry Eindlater Bussey, and T. Wilson Reid. Foolscap 8vo, cloth boards, 288 pp. price 2s.

The British Biographical Reader, Sketches of prominent Men by Eminent Authors. With Introductory Notes by the Editor. Illustrated by numerous authentic Portraits. Foolscap 8vo, cloth board-288 pp., price 2s»

The Shakespeare Reader, being Extracts from the Plays of Shakespeare, specially selected as fulfilling Article 28 and Schedule IV. of the Education Code. With Introductory Paragraphs and Notes Grammatical, Historical and Explanatory. Foolscap 8vo, 160 pp. cloth, Is.

Myths and Legends of Ancient Greece and Rome. A Hand book of Greek and Roman Mythology, for Schools and private Students. By E. M. Borons. Illustrated from Autiquo Sculptures, Foolscap 8vo, 330 pp., cloth 3s.

Poetical Reader, for the use of Elementary Schools in England and Scotland. Foolscap 8vo, 224 pp., cloth Is. 6d.

London, Past and Present. A Reading-book for Elementary Schools. With numerous authentic Illustrations. Foolscap 8vo, 288 pp., cloth 2s.

The Scottish Historical Reader ; being Extracts from Eminent Writers Descriptive of Events in Scottish History, with Introductory notes. This volume will supply a fuller account of leading events in Scottish history than can be embodied usually in School histories, and being composed chiefly of extracts from the works of approved writers will form excellent reading lessons. It is adapted more especially for the requirements of the Fourth Standard.

HOME LESSON BOOKS.

The object of this series is to place in the hands of the Pupil a certain amount of Standard work, which will enable him to grasp the leading facts of English History, Geography, and Grammar, and at the same time lighten the work of the Teacher. The questions in Arithmetic arc mixed in order to test the pupil’s knowledge.

Further Particulars and Catalogues from All Booksellers,

Or their Representative, GEO. STILLIE, care of Messrs. COWAN & Co.,

WHOLESALE STATIONERS, MELBOURNE,

MATRICULATION EXAMINATION,

October Term, 1881.

ALGEBRA.—Honours.

Solutions by John B. O'Hara,

1.    Prove that

a(b-c) (b + c-a)3 + b(c-a) (c + a-by + c[a-b() {a + b-cf — 0.

The expression is = {ab-ac) [a- + b" + c2 + 2bc-2ab-2ac) +two similar expressions.

= (a- + b2 + c2) {ab - ac + bc-ab + ac - be)

+ 2bc(ab - ac) - '¿(a -b - - a2c 2) + 2ab[ac - be)

- 2(a.2c2 _ J2C2) + 2ac[bc - ab) - 2(b2c% - a^bz)

= 2 | ab2c — abo2 + a2 be - ab*c + abc2 — a2 be |

- 2 | a2?;2 - fl26>2 4- a-c2 - b-c~ + b~c2 - «2^2 1 = 0.

2.    Prove that when and n are positive integers

am. an = am + n,

and assuming this result to be true for fractional and negative exponent, interpret the meaning of am when m is fractional or negative.

See Todhunter’s Algebra. Theory of Indices.

3.    If the equations

x2 + ax -f be = 0 x- + bx + ac = 0

have a common root, show that their other roots satisfy the equation. x2 + cx + ab = 0

#2 + ax + bc — 0......(1)

x2 + bx + ac — 0......(2)

(1) - (2) gives us x(a-b)-c{a-b)=*0. Now a-b is not zero, for then the two equations would be identical.

Hence we obtain x — c and this satisfies both equations.

Now product of roots of first equation — be, and since c is a root .*. b is the other root.

Similarly a may be shown to be the other root of (2).

Also sum of roots of first equation = - a. But cis one root- (a + c) must be the other.

. \ b = - [a + o); for b was shown to be the other root.

co + b + c = 0.

This relation could also be found from (2)

Now we have to show that a and b satisfy the equation x2 + cx 4- ah = 0. Substitute a for x in the expression x^+cx + ab and we obtain a(a + b + c) which is = zero from above, since a, when substituted for x makes x- +cx + ab = 0, a is a root of the equation x3 + cx + ab = 0.

Similarly b may be shown to be a root, . \ a and b satisfy the equation x - + cx + ab — 0.


4. Prove that whatever real value x may have, ax2 + bx + c and a never differ in sign except when the roots of the equation ax2 + bx + c = 0 are real and different and x is taken so as to lie between them.

For first part of this see Todhunter’s Algebra. Theory of Quadratic equations.


If p> be q, then for all rea1 values of x the expression


x2 - 2qx + y2 X- + 2 qX+p2


lies between


p+q

x2 - 2qx+ v-

Put —g-,- ,-—0 == ?/.

a?3 + 2qx + p2 J


q- and «


p-q


Then x will be found to be


?(1 + ?/) + A/2( 1 + y)2 -p2[\-vy2

~~ i -y

The expression under the radical sign = (7 + qy - p + py) (7 + qy + p- py)

= ■{ y (p + q) + q - vj j y(q -p) + q + p |


=U2-p2)


p-q{

p + q(


y


_p + q {

p-q I


and as 72 -p2 is negative, one of the other factors must be negative to make x real: y must lie between    0


p-q

p + q


and


p + q p-q


5. If


prove that



... 11

Iporq (3) we have, by dividing by oryz,—■+ —

x y


!/z

~y + 2


0


Substitute this value of x in (2) and we get

_ ciyz

y+ *

.'. -ayz+by2 + cs2 + byz + cyz=0 . yz{b + c - a) + by2 + cz2 •= 0 ; but from (1) Inf- + cz2 = - ax'2

. az2 ‘    b + c —a

And since the equations are symmetrical

btJ2


by multiplication xhfz3


+ by+ cz-0


CZ2

^ a + b -- < abcx2y2z2


{a + b - c) (a-b + c)(b + c~a)

(a + b-c)(a-b + c)(b + c-a')=> -abc

.a2b + «2c + ab2 + ac2 + b2c + bc2 — a3 - 1)3 - c3 — 3abc = 0 (Todhunter, pp. 19) a{c - «)(« - b) + b[b - c)(a -b) + c[b - c)(c - a) = 0 _ abc ' ' b — c +.c - « + a-


7. If


-, = p ==—-, =&c., prove that each of these ratios is equal to 0 c

/pa71 +qb“ + rc" + .... \ 1 + qb'n + rc'n + ....) n


Solve the equations

x2    y3 _ z3    «s

a    b c    a2 ^ b1 ' c1

For first part of this question see Todhunter’s Algebra, page 226. a


similarly


V'


+ ■


a;2= y2    3:

/ «3_. \7 "¿3

.y3--*L

J /a3 V7/;3

Z3


I «3


• •.Substituting these values in the equation — =*1+ V +


we have


xf

a


a2 In


.r = 0 or

a?8

x3

«2

+ m

/ «3

M / ¿g

C2 / ~c2

1

f 1

1 1

' a

* I û2 + /¿3b+ Srfc

i 1 1

1 1

= x ■

{ a'2 + co fab

a J ac j

1 =X-

fbcf J ac + /afa

a / bo a / be


z


C2


f abf / ac L / bo

Since the equations are symmetrical, y and 2 can easily be found. Thus by changing x to y, also a to b we find y, &c.

8. Define a geometrical progression, and show how to find the sum of any number of terms of such a progression. The sum to infinity of a certain geometrical progression is s, and the sum to infinity of the series formed by taking its first, third, fifth, &c., terms is s'. Find the first term and the common ratio of the series.

Let a, ar, art &c., be the series where a is first term, and r common ratio.

a    s' 1    s- s'


Then


r


1 - }’2


r _ __ then by substitution = s'. a can be found


9. Prove that in the scale of which the radix is r, a number i9 divisible by r- 1 when the sum of its digits is so divisible, bliow that, in the ordinary scale of notation, the difference between the square of any number consisting of two digits and the square of the number formed by interchanging these digits is divisible by 99.

(1)    For first part of this question see Todhunter, page 252.

(2)    Let xy be the number, then in the ordinary scale of notation this number = 102; + y.

When the digits interchange the number = 10y + x

Now (10x + y)2 - (10i/ + «)3

= (11z + 1L/)(9z-92/)

= 99(a;2 -y2) and is . \ divisible by 99.


10. (i) Find a formula for the number of combinations of n things taken r together, (ii) If the number of combinations of 2n things taken n- 1 together be to the number of combinations of 2n — 2 things taken together as 132 to 35, find n.

(i)JFor first part of this question see Todhunter, page 280.


'ii) The number of combinations of 2>i things taken ?i-l together

I 2n


n—1 | n + 1


The number of combinations of (2ii—2) things taken n together is

| 2n—2


\n\n—2


|_n_j 11—2

I 2n—2


2»


_ _ 132 _ 2n (2»,—1) _ 132

n—1 | ?i +1    35 * '(» + !)(»—1)    35


140»2—70» = 132??,2—132 . ’.S»2—70» + 132 = 0 -6)(8»—22) = 0 .-.)) = 6.

;i Enunciate and prove the Binomial Theorem for a positive eg’ral exponent, _ ¡If n be a positive integer prove that 1 + ® »(»-1), 1+2®

1~nl + nx +    12 (l+ «*)"»

n[n-l)(n-2)    1 + 3®

"    1.27T"“- (i+»®)3 + &c-~0.

This expression =

n[n — 1)    1 n(n-1)

"7 2 (1 -»®)2 + 1.2

3*


1 -


1 -


1 + nx 1 + nx ( 7i{>i - 1) (n -2)

l |T (1 + »®)

M(»-l)    1


1 n{n- 1)(»- 2)

8~    172.3

7§0l - !)(» - 2)


1 + nx |_2    (1 + nxy    J_3

x n(n-l){n—2)


nx »{» 1)


= 1-


1+ nx '    1    (1 + nx)

1    ) n nx


2-


2x } (1 + nxf f


(1+»©)8 f +&C<


1


(1 + 7)X)


3    &c,


n 3

/ 7 ....... + See,

(1 +nx)


(iii.) xy    +    — = a......(1)

0*    +    ¿—6......(2)

+    —• = <?......(3)

From (1) xyz — az—x from (2) xyz = bx—y from (3) xy. = cy-

. •. az—x — bx—y...... (4)

az—x = cy—z ...... (5)

From (4) y = bx + x—a: substitute in (5) and we get

itz—x = box + cx—acz—z £ (1 + a + ac) = x(l + c + be)

1 + c + be


l+a + ae Similarly y may be shown <=


1 + 6 + n b


1 + il + cic

Substituting these values of y and z in (2) we get (1 + c + be) (1 + 6 + a6)    l+6 + «6

x'2    (1 +a + ac)2 + "i + a + ac ~

.,x'2 (1 + e + be) (1 + b + ab) = 6 (1 +a, + ac)'2—(1 + b + ab) (1 +a + ac) = (1 +a + ac) [b + ab + abc,—1—6—ah)

= (1 + ® + iH’) [abc—1)

(1 + a + ac) (abc 1 *'• 92 = (l+c + 6o)(l + 6 +76)

. .9 _    (1 + c + be) (abc—1)

* * *    (1 + a + acj (1 + 6 + czb

(1 + b + ab) (abc—1)

W* ~ (1 + a + ac) (1 + c + be)


1 + nx


1. 2

!_(„_!)    1    +(»-Dl«-2)

1 + nx I    1 + nx


1


Science imb %xt cOosriip.


(1+»®)^

—&c.


- 1 i


nx '

1n nx

(i

1 '

\ H

1 + nx j

1 1 + IV,

® \

1 7iXj

1 i

rv

i

nx

!

1 + nx 1

1 + 71X

1 + nx

s


f    1 j n — 1 ) nx _nx_1

l 1 + ‘nx)    ( 1 + nx 1 + nx)

.    = zero.

12. Solve the equations


(i) v7- + 3x +1 + ^3x- + 5® - 7 = 2 \rx2 + x - 4

(ii) ax- ■\-by‘l

= a2x + 6

"~y = a

x(

' 1 '

(iii) y

(z+4

)=6

r i >

X + -

k V >

>-


(i*) ?2 + 5®—7 + ^ ®2 + 3® +• 1 = 2/®2 + ®—4 Now we have identically (3®2 + 5®—7) — (®2 + 3® +1) =* 2 (®2 + x—4) That is ( \f+ 5©—7“ /®2 + 3® + l) (V73®2 + 5®—7 + ^£2 + 3®+ l) = 2( /x2 + ®—l)2

Now divide the original equation into this and we get J 3®2 + 5®—7— v' ®2+3®+ 1 = v72 + a;_ 4.

Add this to the original equation, and we get

2 'J 2X1 + 5©—7 = 3 7r2 + ®—4 . •. squaring 12®2 + 20®—28 =s 9©2 + 9®—36 3©2 + ll® = -8

11    /11\2    8    121 121—96 25

©-+ 3® + ( 6 ) -    3 + 36 ~    36 “36

II 5    ,    8

‘‘x = —~6 + 6 =-lor—3

ii.) a©2+ by 2 = a3 + 68 ...... (1)

a2© + 6 2 y a3 + 63...... (2)

«3 + 53—<72®


From (2) y =


62


\ substituting this value of y in (1) we get

(a3 + 63)2—2af!x(a‘l + 6') +


a®2 +


.'. fl®2(a3 + 63)—2»2®(a3 + 63) + (fl.3 + 63)2 = 63(a,3 + 63)

. •. dividing by «3 + 63 ax’l—2a3u + a 3 + 63 = 63

a®2—2a2® + a3 = 0    ,\ ®2—2a® + <z2 = ()

(x—<v)2 = 0 ® = a

5y substituting this value of ® we find y = b.


The plaster model of the statue of the Queen has been placed, for a time, on a pedestal at the intersection of North Terrace and King William Street, Adelaide.

At a meeting of the Technological Commission of Melbourne, held on January 5th, it was decided that the next exhibition of the works of the pupils would be held in May next, and that each district should be examined separately.

Professor Draper, professor of Chemistry at the Cambridge University, died on January 5th.

Mr. It. J. L. Ellery has been appointed a trustee of the National Gallery, Victoria.

At the second half-yearly meeting of the Victoria Sketching Club, the following office-bearers were appointed :—Committee—Misses Rae, Stone, Ballard; Messrs. Campbell, Mather, Watson, and Wilson; Miss Earles, treasurer; Mr. A. C. Trapp, secretary.

At a special meeting of the School of Arts, Rockhampton, Queensland, held on January 5th, for the purpose of electing a secretary, Mr. George Potts was chosen in preference to several candidates.

A Roumanian mechanic recently submitted to the Chamber of Bucharest a plan for a submarine vessel, and a committee appointed to examine it are satisfied with it. The Government has been authorised to meet the expense of construction. It is a vessel designed to run under water for twelve hours, without renewal of air. It is to be a steam vessel, will travel faster than a sailing ship, and will be lighted by electricity. It is to be sunk to a depth of thirty metres by opening certain valves, and may, if necessary, continue indefinitely under water, the air in the vessel may be renewed by an apparatus sent up to the surface.

A valuable specimen of quartz, studded with gold, was stolen from the University Museum on the night of the 10th instant.

A PORTRAIT of Christopher Columbus has lately been found in the colonial office at Madrid, the work evidently of a contemporary artist. The inscription it bears shows clearly that it is a genuine portrait. It runs, “ Columbus Lygur, Novi Orbu repertor.” The painting, which is still in good condition, represents the original as about 40 years of age, dark, and with pointed features.

The Birmingham Arts Gallery has lately received donations which will in all amount to £90,000.

In 1831 it was found that ink. of a remarkably good quality could be made by treating an infusion of galls with vanadote of ammonia. This was at first found to be too expensive, but it has Bince become so cheap that it can now be used for ordinary inks.

The Brackenbury Natural Science Scholarship at Baliol College, Oxford, has been awarded to Mr. T. F. M'Arthur, of Manchester Grammar School.

From Nature we learn that In consequence of an appeal from Mr. Leigh Smith’s relations and friends the council of the Geographical Society resolved to ask the First Lord of the Admiralty to receive a deputation from their body, accompanied by experienced Arctic navigators, who will urge upon him the necessity for taking immediate steps for the relief of the Eira.

Mr. E. C. IIore has presented the Geographical Society with a map of Lake Tangauyika, which he constructed from careful surveys and observations made during his explorations.

Puofessor Haeckel is making a scientific exploration of Ceylon, which will occupy him for the next three months.

At a lecture delivered at the Midland Institute, Birmingham, Prof. Robert S. Ball stated that “The tides are increasing the length of the day.” The discoverer of this was Mr. G. H. Darwin, Fellow of Trinity College, Cambridge,


Mutual.Trust and Investment Society.

LIMITED.

(Incorporaieel under the Companies Statute 1864).

OFFICES: 66 COLLINS-STREET WEST, MELBOURNE.

Capital, £100,000 in 10,000 shares of £10 each. Subscribed, £10,000. Paid-up

£9301.

DIRECTORS :


Mr. Robert Inglis, accountant, G4 Collins-street West.

Mr. Charles Hetberington, Secretary, GO Collins-street West.


The first line of telegraph in China, from Shanghai to Chinkiang, has been opened.

The flashing system of telegraphy has been made so successful that the French in Tunis converse by it daily in spite of all the efforts of the insurgent Arabs to prevent it.

Mr. F. M, Balfour, F. R.8., has been elected President of the Cambridge Philosophical Society.

Mr. Edward B. Tylor, in a paper on the Sacred Myths of Polynesia, says, Prof. Bastian, on a late journey made to enrich the Ethnological Museum of Berlin, stayed a short time in New Zealand and the Sandwich Islands, and there gathered some interesting information as to native traditions, some not yet published, and some which have been neglected (if ever met with) by European students. The documents now printed in a small volume all strengthen the opinion which has for years been gaining ground among anthropologists as to the civilisation of the Polynesians. It is true that they were found in Captain Cook’s time living in a barbaric slate, and their scanty clothing and want of metals led superficial observers even to class them as savages. But their beliefs and customs show plain traces of descent from ancestors who in some way shared the higher culture of Asiatic nations.”

The Journal and Proceedings of the Royal Society of New South Wales show that within the last 25 years, although the land has been greatly denuded of trees, no diminution in the amount of rainfall has taken place as might have been expected.

THE MATRICULATION EXAMINATION.

The new scheme for the matriculation examination at the University has at last been subject (o the test of experience. The history of the scheme has been given in our columns, and the result of the test to which it has been submitted this month, has been looked forward to with keen interest by all who are interested in education, but by none with greater anxiety than the originators and earnest advocates of the plan, of whom our Head-master is one. No final, nor even an approximate verdict can be offered until the results of the recent examinations appear. Not that results are always or at once a fair test of the merits of a plan, but because we shall be then able to ascertain two things which arc of great importance—(1) Whether the proverbial indignant parent of the plucked candidate is more content with the verdict of a board of examiners than with that of a single examiner, and (2) whether papers set and revised by a board are better tests of a candidate’s knowledge than these set by a single examiner. We have little doubt that experience will answer the first query in the affirmative; the knowledge that the plucked candidates have been rejected by at least two examiners, after an independent revision of their answers, will completely stop incessant complaints from parents and from teachers, which have for years past been a fruitful source of annoyance to the University authorities. With regard to the second point, we fear that the confidence which the promoters of the scheme had in its being adapted to remove all chances of errors being made in setting examination papers was misplaced, for the daily newspapers have recently teemed with letters from candidates, as well as from their guardians and tutors, complaining of the papers set. for honours in English and in French, and for pass in chemistry. We arc bound to say that the complaints are just. It passes belief that such an important paper as that for the honour examination in England should haveignored half the work prescribed, and that of the remainder a considerable portion was set from the proper author, indeed, but the questions were upon a part not. included in the syllabus of the examination.

ft is conceivable that an examiner in preparing the paper might make such a blunder, for we have had bitter experience in the past of similar lapses, but it is intolerable that a board of examiners should carelessly pass such slip-shod work. So, too, when the examiner in chemistry—an able chemist, but not versed in conducting University examinations—set one, and we might say, two questions on the chemistry of metals, a portion of the subject distinctly excluded from the limits of the examination, and the questions were passed by the board of examiners for the group of subjects containing chemistry, can we suppose for a moment the paper was ever really revised by the board ? The honour examination in French was the laughing-stock of the schoolmasters; there was hardly one question that might not have been included iu the pass-paper. The examiner’s accurate and extensive knowledge of the literature, history, and philosophy of the French language is too well known to allow of our supposing that he is wholly answerable for the paper ; but he must share the responsibility with the chairman of the board of modern languages for having directly discouraged earnest teachers, who for the past year have been devoting their energies to an extensive study of the best authors in prose and in verse of a certain prescribed period of Freftch literature. The painful conclusion forced upon us is that we have the name and not the reality of examining boards ,* we have all the evils of the independent examiner, plus the shirking responsibility, which is thrown upon a sham board ; and if the University does not see to it that the resolution of council and senate is carried into effect, all the evils under which for a long time we have groaned will be intensified. The council showed the importance it attached to the boards by making the salary paid to each examiner depend chiefly upon the performance of his duties as a member of the board, and by keeping the chairman free, when possible, from setting papers; it rests with the council to insist upon these duties being satisfactorily performed. It has recently rightly shown its determination to uphold the dicipline and moral tone of the University fearlessly ; it is hardly less important for it to see that the examinations are conducted according to the statutes and the regulations.— Wesley College Chronicle,

CONTENTS

Matriculation Examination—Algebra    ...    ...    ...    ...    ...    98

Science and Art Gossip...... ...    ...............99

The Matriculation Examination ...    ...    ...    ...    ...    ...    100

Leaders—Victorian Education Act...... 100

The Revised Testament ..................101

Elementary Education Act......... 101

Public Elementary Education in Ireland .........102

Notes of the Month ...    ............... ......102

Parliament and the Universities ...    ...    ...    ...    ...    ...    103

Victorian Education Department—Examinations Papers, Dec., 1SS1 ... 107

THE

Mr. M. II. Davies, Chairman, 8 Collins street West.

Mr. Francis J. Smart, architect (Messrs Henderson and Smart)

Auctioneers and Special Valuators: Messrs. C. J. and T. Ham, Swanston-St.

Surveyor and Valuator: Mr. Percy Oakden (Messrs. Terry and Oakden).

Auditors : Mr. Thomas Inglis and Mr. John G. Shield. SECRETARY : MR. C. HETHERINGTON. MANAGING DIRECTOR : MR. ROBERT INGLIS.

_ SECOND ISSUE of 1000 SHARES of £10 EACH.

The chief objects of the company are:—1. To lend money to members upon the security of real property or upon bank, gas or building society shares, or upon other approved security. 2. To negotiate loans of all descriptions upon such terms as to profit and remuneration as may be agreed upon. 3. To buy and sell freehold or leasehold estates. 4. To collect and receive rents, debts, dividends, interest and other moneys. 5. To buy and sell shares in any public banking corporation, insurance or gas company or building society, or any incorporated company. 6. To act as attorney or agent in the management of estates for absentees and for trustees.

Iti order to increase the paid-up capital of the company and extend its business it has been decided to make a second issue of 1000 shares of £10 each. These shares are now offered to the public at a premium of os. per share, and may be paid either in cash or by instalments of £1 per share per month. The company lias now completed its sixth half-year, and has, during its progress in addition to paying off all preliminary expenses, paid one half-yearly dividend at the rate of 8 per cent, per annum, and four half-yearly dividends atthe rato of 10 per cent, per annum. The secretary will receive application for shares unti, further notice, and will furnish information regarding the company to intending shareholders, together with copies of previous reports and balance-sheets,

C. HET11 ERIN GTON, Secretary.

APPLICATION FOR SHARES.    '

To the Directors of the Mutual Trust and Investment Society (Limited).—I hereby apply for    shares in the Mutual Trust and Investment Sosiety

(Limited), and inclose , being 5s. per share application fee.

Name in full........................................

Address................................................

Date.............................. Occupation ..........................................

ANSWERS TO CORRESPONDENTS.

J. B. O’HARA.”—Thanks. Solutions of Geometry and Trigonometry inserted next month.

Received.—“J, J. M,” “ Hugh Hughes,” “ R. E. Gilsenan,” “ T. F. Corbett, LL.B ”

Elisiralashni j§dja0lmasfn\

PUBLISHED EVERY MONTH.

MELBOURNE, JANUARY, 1882. State school teachers in Victoria will learn with pleasure that the Secretary of Education has returned by the mail steamer after enjoying his well-earned holiday in Europe. The change of Government that took place shortly after he left the colony has led to several important projected alterations in the working of the State school system. These will at once engage Mr. Brown’s attention, and, doubtless, the observation and experience of the elementary school systems of Europe, gained while absent, will materially aid him in forming a correct judgment upon them. The Hon. Jas. Macpherson Grant having assumed office as Minister of Public Instruction during his absence, Mr. Brown Avas formally introduced to the Minister by the Acting-Secretary, Mr. Bolam, on the occasion of his visiting the department on the 13th inst. Mr. Grant warmly congratulated the Secretary on his return and upon the improved state of his health. Among the questions now awaiting a decision, are the exclusion from the schools of all children who have not attained the age at which the Act renders education compulsory, and the closing of night schools, excepting in very special cases. With regard to the first named, it is manifest


that the influx of infants into State schools has become an evil, and must be restricted. Apart altogether from the question as to whether or not it is injurious to the physical development of young children, to force the growth of their mental powers and keep them for hours together in close, heated rooms, we hold that it is unwise to encumber the edu-tion Act with the cost of providing schooling and nurses for children whose education was not in question when the battle of free, secular, and obligatory education was fought out in Victoria. AVhat the friends of popular education desired was to inaugurate a system of public instruction, under which the whole of the children in the colony of school age should be brought into school, and be prepared to enter upon the active business of life, altogether free from sectarian training. School age is clearly defined in the Act, and ought never to have been departed from. Owing to the infringement of the Act in this respect, serious embarrassment has arisen in consequence of the enormous additional expenditure it has entailed upon the department. To provide ample building accommodation of the character of the present State schools, and togivea sound education to the youth of the wholecol-ony, is certainly all that the State ought to be called upon to do, and is a work of sufficient magnitude to tax the energies of the Education Department. The care of infants may well be left to their parents, or to private seminaries.

The subject of night schools is beset with special difficulties. Many children have been allowed to pass the legal school age without education, while the necessities of the parents of others are such as to render it imperative that they should contribute something to the support of the family circle. Exceptional circumstances, therefore, do arise in which it is incumbent on the Department to open free night schools. 1 hese, however, ought to be conducted under the best of masters, men who are strict disciplinarians, as well as good teachers.

In an article criticising the latest edition of the Revised ISew Testament, the School Board, Chronicle remarks :— Glorious old JohnDryden used to say that everything suffers by translation except a bishop, and we fear that the revised translation of the New Testament now before us is no exception to the rule of that great poet and critic, whose own translations of classical works were complete demonstrations of his rule. The revisers were entrusted with the work of revising the Holy Scriptures translated in King James’s reign, on the understanding that they were “ to increase its fidelity without destroying its charm,” and “ to render a work that had reached a high standard of excellence still more excellent.” We wish that the excellence of execution of the task assigned to the revisers had in any way equalled the excellence of their aim, confessing, as we do, our deep disappointment at the results, which are for the most part anything but satisfactory. The best work done by the revisers is the excellent text of the original Greek which they have published along with the translation; and the translation itself is not without many merits, though by no means are these merits of such a character or of such weight as to counterbalance the gross errors of good taste and mistranslation of which the revisers have been guilty.

As samples of admirable and useful correction we note St. Luke viii. with the happy an 1 accurate rendering, “Then cometli the Devil, and taketh away the word from their hearts, that they may not believe and be saved,” where the Authorised Version gives us “ out of their hearts,” erroneously; for the Greek not only does not warrant it, but the sense of the context shows that the good seed was never in their hearts, and therefore could never be taken out of their hearts. Again, << Deliver us from the Evil One,” in the Lord’s Prayer, instead of the loose and general term “ evil” is much truer to the Greek and in happy concord with the earliest versions and the comments of the Fathers. It is curious to note here that the word Devil is really etymologically the evil—thevil—and so Devil, as God is the Good Being. Again, in St. Luke vi. 35, the revisers give us, “ Love your enemies, and do them good, and lend, never despairing,” with a marginal note:—“ Some ancient

authorities read despairing of no man.” Here the Jacobean translators perverted the Greek by translating “hoping for nothin«: again”—a sense which the Greek never bears. Here we regret, however, that the revisers did not still further improve their rendering by following the reading of the Sinaitic codex, confirmed as it is by the three Syriac vesions, and accordingly rendering it “ driving no man to despair.” Speaking generally, however, the greatest improvement of the revisers will be found in the careful rendering of the Greek article and the Greek tenses—a portion of Greek scholarship but little understood in the days of King James’s translators.

The education of the humbler classes of the community lies somewhat outside the sphere of the majority of our readers ; yet we cannot refrain from making a few comments on that grand experiment of Mr. Forrester’s ten years ago—the Elementary Education Act—and the results which have flowed from it. If we are not ourselves engaged in educating the class for whom that Act was passed, we are at least ratepayers, and we bear our share of the burden of maintaining the Elementary Schools. That is the lowest ground for taking interest in this great subject. We are also however, Englishmen, and therefore deeply interested in whatever concerns the education of the people, our fellow subjects amongst whom our lot is cast.

At the recent Church Congress held in Newcastle, several papers were read on the First Decade of the Education Act, and were followed by the usual discussions. Both papers and discussions were full of interest and suggestive facts. Thus, as appeared from Mr. Heller’s paper—a valuable contribution to the statistics of the subject—the number of children on the school registers, in the year 1870, was 1,949,070, while in 1880 it was 3,895,824. That is to say, it had actually doubled ! One trembles to think what would have become of half the children in England if this Act had not been passed. Again, the school provision has more than doubled. Ten years ago there was room for 1,878,584 scholars in our Elementary Schools; there is now accommodation for 4,240,753. The average attendance was, in 1870, 1,152,389, and, in 1880, 2,750,91G. One of the most noteworthy facts in connection with this vast extension of elementary education is, that there has been a very considerable increase of Voluntary Schools, notwithstanding the multiplication of School Boards; and, in spite of all that was once said in favour of secular education, it has become apparent that religious training is desired both by the Boards and by the majority of the parents all over the country. Nothing is more instructive, on this point, than the experience of the Birmingham Board. They began there by having a purely secular form of education. The conviction was, however, soon forced upon them, that morality, at least, ought to be taught; they thereupon introduced a “ Text-Book of Morals,” and a year afterwards the Bible itself found its way into the schools. An interesting question for consideration is the all-important point, how far the Education Act has benefited the morality of the country. There are, we find, more boys in reformatories than there were ten years ago; but then more care is taken to send boys to these schools, and an increase in the number may and ought to show a decrease in crime. It seems to all but violently prejudiced people, as if the teaching of morals in schools must be better than no teaching at all; and we believe, with Professor Huxley, that the maintenance of the religious feeling is the most desirable of all things, and that it cannot be maintained without the Bible. Although it may be too soon to pronounce a very confident opinion on the subject, we can hardly see how it is possible for the morality of the country not to have improved. That lads living under bad home influences, and among immoral surroundings, will suddenly become honest, patient, virtuous members of society, we do not at all expect; but we must remember how good a thing it is for a boy or a girl to learn, even for two or three years only, the ways of wisdom.

The connection of actual knowledge with religion and morals is a subject on which people’s minds are curiously vague. There are many who seriously believe, that a boy who learns to read write, and cipher, is far advanced, not only on the road to wisdom, but virtue. By reading, to be sure die may help himself

We are not disposed to agree with Sir Patrick Keenan that, so far as principle is concerned, the problem is solved, and that all that is now needed is to extend and complete the system, complementing it with compulsory attendance. We are rather prone to think that it is not the business of the State to hand over the duty of the education of the poor, and the money with which to carry it on, to the priesthood of the various theological creeds. Education in the hands of the Churches appears to us to be a survival from mediaeval times. It is nearly all that remains of the old grip of the sacerdotal power upon the secular Government. If the Churches feel it to be their duty to educate the race, let them find the money and carry on the work as far as the people are willing to accept the services at their hands; but if the State is to find the money and educate the children, it should be done on State and not on ecclesiastical principles, and for State and not for ecclesiastical ends. These are views of national education which must prevail more and more in a free community, and we imagine the time is gone by when the British Government can make new and costly terms, on the great question of national education, with the principle of concurrent endowment or State subsidy to education by the Churches.—School Board Chronicle.


liâtes of tlje UTottfjj.


because he may read good books ; on the other hand, he may do himself a very great deal of harm by reading bad books. Writing and arithmetic have absolutely no connection with morals; nor have geography, mechanics, or facts of any kind, except those historical facts which show the advantage of being on the side of the angels. Yet the learning of these things may have its moral aspect, by awakening a spirit of curiosity and research. All we can do, in this respect, is to lead the children into the right path, and to encourage them to keep in it. Our ancestors were wise in one thing, for, whether they taught the children to read or not, at all events they taught them their catechism ; and the reformers of the Birmingham school have been taught a salutary lesson, and been compelled to go back on their footsteps, because they did not, at the outset, ask themselves how best to inculcate the right conduct of life.—Educational Times.

PUBLIC ELEMENTARY EDUCATION IN IRELAND.

The newspapers generally, in their reports of the proceedings of the Social Science Congress at Dublin, do not reproduce at any great length the interesting opening address in the Education Section, delivered by the Chairman of the Section, Sir Patrick J. Keenan,the Resident Commissioner of National Education. A full reprint, however, of Sir Patrick Keenan’s speech lies before us, and we are tempted to call attention to it as an important contribution to the consideration of the great-estcoming education questioned the age. We cannot help agreeing all round that not much longer may be postponed the solution of the great and difficult problem of national elementary education in Ii-eland, and the first element towards the solving of the question is the diffusion in this country of correct and clear information as to the existing state of things in the Sister Isle. Sir Patrick Keenan has left it to other hands to give us full information as to the kind and the measure of existing ignorance and neglect of instruction among the poor in Ireland, but he has placed within the reach of all inquirers a lucid statement of the system of national education now in operation, enabling us to consider whether Parliament ought to content itself with the extension and completion of that system, or whether, in providing elementary education for Ireland as it has been provided for England and Wales,new principles must be introduced.

Sir Patrick Keenan is an almost ardent advocate of the extension of education on the lines of the existing system,which may be described as the denominational, or “ concurrent endowment ” system. In presenting an historical summary of education in Ireland, with illustrative glances at experiments in national education in other countries, his main object appears to be to show that all schemes of national education fail which are not of a denominational character, or which do not recognise the ministers and teachers of religion as the principal co-operators with the Government—if not the chief agents of the Government in the education of the people more or less at the cost of the public.

That is really the point at issue. We invite all who are interested in this complex and almost painful problem to study carefully Sir John Keenan’s pamphlet, and to see how far they can agree with him and how far, in differing from him, they can find reasons for taking the opposite view and for expecting success where he would only look for failure.

Sir Patrick draws a melancholy and disastrous picture of the failure of the Kildare Place Society, which was established in 1811, on lines similar to those of our British and Foreign Schools Society, one of whose articles was i( Bible reading without note or comment.” Bible reading was, of course, in Ireland, a heresy. This was not the principle of leaving religious teaching outside the field of national education. It was not negative. It was an actual carrying of war into the Roman Catholic camp. On that rock the Society split. With the appearance of neutrality in the matter of religious instruction it was in fact, in its relation to the predominant x-eligion in Ireland, a Protestant scheme of national education. At the instance of the late Lord Derby the Kildare system was superseded by the denominational and concurrent endowment system now in operation.

The Rev. A. F. Harding, a teacher in Christ College, Tasmania, committed suicide, on account of disappointment in love, whilst he was on a visit to Melbourne.

Mr. Quick, M.L.A., of Victoria, has taken his ad sundem degree of Bachelor of Arts at the Sydney University.

The well known novelist, Mr. William Ainsworth, died on January 2nd, at the advanced age of 77 years.

Copies of the Planisphere and the explanatory references compiled by Mr. Ellery for use in State Schools, will be sent to each school, and instruction in astronomy will soon be included in the daily work of the upper classes. Two thousand have been printed at the Government printing offices.

The chairman of the Education Commission visited and inspected the Education Department on Wednesday last.

A teacher named Leonard Woodruff, who, in June last, collected some money and failed to hand it over to the owner, was arrested on January 5th.

The Government Gazette notifies the appointment of Mr. J. Thorn, formerly assistant master at the state school at Kilmore, to succeed J. C. Hayes, dismissed.

The first meeting of the class for instruction in Telegraphy was held in the lecture-hall of the Technological Museum, on Monday last.

The Parliamentary Library of Tasmania has received a case of books from Professor Spenser F. Baird of the Smithsonian Institution, Washington. It contains the publications of the several Executive departments of the Government of the United States. The Parliament Library has already received upwards of a thousand volumes from this scientific society.

At a meeting of the Richmond Board of Advice, held on Monday, Jan. 9th, a letter was received from the Education Department stating that application had been made to the Crown Lands Department for the reservation for school purposes of land previously withdrawn from sale, viz., allotments 39, 40, 2 and 3.

The Warragul Guardian says: A new state school at King Parrot Creek is much needed, and we are informed that the Board of Advice have more than once drawn the attention of the Education Department to the fact. There are, we understand, about 40 children who would attend school, but out of that number only about one-fourth have been attending lately, though the weather has been fine ; and we are informed that the reason of the poor attendance is the unsuitability of the building now occupied. The teacher has no accommodation at King Parrot Creek, and consequently has to live at Drouin, three and a half miles distant,

A State school is to be built at Coy’s Diggings, near Heathcote, at a cost of £263 17s. 9d.

The Minister of Education has refused to alter the hours during which the poll is taken for members of the Board of Advice, on the ground that no complaints have been made by the voters themselves, and that not much interest is taken in these elections.

Telegraphic despatches announce the death, at the age of 66, of Henry Dana, who wrote “ Two years before the mast.”

A competitive examination will be held at the Melbourne School of Music, in February next, for two musical scholarships.

The Education Commission met at the Treasury Buildings on Wednesday last, and made arrangements to examine the witnesses on 18th of February next.

The senate of the University of Melbourne met on Tuesday, December 21st, for the purpose of electing a member of the council in the place of Professor Andrew, who had resigued in his turn. The only two candidates for the position were the retiring member and Professor McCoy. Professor Andrew was returned by a majority of 13. The numbers polled were 7 5 and 62.

The question as to whether very small children are to be excluded from attendiug State schools has not yet been decided. The inspectors are to meet and confer with the Minister of Education on the matter.

A full time school has been opened at Lubeck, in Victoria at the request of a deputation from some of the more important inhabitants.

Troubles of the eye and ear in children are often traceable to defective teeth. Dr. Sexton regards irritation of the jaws as among the chief causes of the increasing near-sightedness among school children.    .    .    .

The International Geographical Congress at Venice, m September last, adopted a resolution in favour of the construction of a general geological map of the world, which will be made at Berlin under the direction of some of the most eminent geologists from all the European countries.

“ ASSISTANT,” writing to the Argus, Melbourne, says there are many rules with regard to the appointment of teachers that ought to be altered by the Education Commission. For instance, he complains that the youth who has had merely a year or two of training at the training institution, is often placed over the head of an equally clever man who has had several years of practical experience in schools.

Mr. G. Wilson Brown, secretary to the Education Department, arrived in Melbourne by the Royal Mail Steamer Khedive, from Venice, on Thursday evening last. On the next day he visited the department, and he will commence attending to his duties next week.

Mons. A. C. Aucher, B.A., will deliver a lecture on Friday evening, February 3rd, on “ The French language and how it should be taught.”

The London Quarterly for October contains an interesting article on George Eliot,

The University of Sydney, which recently opened its classes and degrees to women, has received a donation of £5,000 for the endowment of scholarships for poor students, The donor connects the gift with the opening of the University to women.—Educational Times.

The statistics of England show that out of 17,000 schools, Church of England possesses 11,000, Wesleyans only 569, and Catholics 758. There are 3433 Board Schools, and 1438 British, the Board Schools, however, are by far the largest. The Church schools employ 7512 teachex*s, and the Board schools 8920.

The report of the Council of Education in England says :—The extent to which the training colleges have contributed to the supply of efficient teachers in England and Wales is shown by the fact, that out of 13,521 masters employed in schools in 1879-80, 8,129 or 60'12 percent, had been trained for two years, 1,130 or 8’36 per cent, for one year, and 287 or 2T2 per cent, for less than one year, while 3,975 or 29'4 per cent were untrained. In like manner, of 17,901 school mistresses, 8017 or 44-79 per cent, bad been trained for two years, 1075 or 6'01 per cent, for one year, and 255 or F42 per cent, for less than one year, 8554 or 47'78 per cent, were untrained. Many of those untrained had been under the best teachers, had passed their pupil teacher course, and were serving as assistants of large schools at the time of passing examination for the certificate.

There has lately been erected, near Cockermouth, in the county of Cumberland, a large new Industrial School, which the Home Secretary for England considers the latest, and he believed most perfect development of thescheme of an Industrial School. A thoroughly efficient master has been secured, and the whole is to be under the management of a committee of eighteen Magistrates, under the guidance of a very able chairman.

On the 17th of December last, the Bishop of Adelaide, Dr. Short, resigned his position as Chancellor of the University of Adelaide. On the 30th the Church of England Synod’s Standing Committee presented an address to His Lordship, on his retirement from the office of Bishop of the See of Adelaide.

The Philharmonic Society of Adelaide gave their first concert in the Town Hall, on December the 23rd. At this concert Handel’s Messiah was produced.

At a meeting of the Port Adelaide School Board of Advice held on January 5th., 30 parents were present to answer accusations of neglecting to send their children to school, but. most of them escaped with a caution, as the Board deemed it necessary to take proceedings in the police court in one case only ;

At a meeting of the Hindmarsh (South Australia) Institute, on Friday evening, January the 6th, the Secretary intimated that a Dramatic Club was being formed amongst members of the Institute and others with apparent success. Entertainment Committee to ma e arrangements for a series of concerts (three) on behalf of Piano and Library Funds.

SOME handsome buildings have been added to the public school accommodation in South Australia during the year 1881. At Glenelg, a large stone building capableof accommodating 250 pupils has been erected at a cost of £2936 10s., and at the following places also schools have been established at the coits named, and capable of holding over 50 children :—Goodwood, £2,398 15s. 3d.; Nailsworth, £2,520; Oaklands, £1,014 5s.; Port Augusta West, £1,315; Port Pirie, £600; Quorn, £1,100; Snowtown, £1,028 ; Stockwell, £971 ; Payneham, £1,820.

In Sydney, the total receipts for public school fees for the year just completed, were £46,347. In Adelaide the total revenue under the head of Education amounted to £19,900, which is an increase on last year’s receipts of somewhat between £1,500, and £1,600.

The election of Lord Rector of the University of Aberdeen, Scotland, was held in the beginning of November, and resulted, according to expectation, in the election of Dr. Baine, late Professor of Logic in the University.

Professor Huxley has resigned the position of Secretary of the London Royal Society, after occupying it for ten years. Dr, Michael Foster, professor of physiology in the the University of Cambridge, will likely succeed him.

The late Mr. McDouald, J.R., M.P., has bequeathed to the University of Glasgow the whole of his books on mining, and £1,000 for the foundation of bursaries to be held by young men who have worked underground as miners.

Messrs. Reid and Barnes, architects, of Melbourne, have called for tenders for the erection of three professors’ houses to be erected in the University grounds.

Mr. H. Dempsey has been appointed head-master of Asburton (New Zealand) Borough School.

The returns of the monthly attendance of the Invercargill High School showed boys 52 and girls 44'31.

At the invitation of Alderman Bale, (Queensland) chairman of the committee of the school, 220 of the children of Bowen Bridge school assembled on his grounds, and were entertained with a long programme of sports as a breaking-up fete.

A correspondent of the Launceston Examiner says :—The Roman Catholic clergy resolved to establish in Launceston a school for their youth, with the primary object of giving instruction in the tenets of the catholic religion. The first step was to withdraw from the Board of Education St. Joseph’s schoolroom, which had been used for many years as a public school. The Roman Catholic congregation was warned recently from the alter, that all children of the church must be withdrawn from the protestant schools, and entered in the new school under a penalty of excommunication.

The receipts for Christ College, Tasmania, for the year 1881, amounted to £1471 13s. 2d., and the expenditure £1086 17s. 6d., leaving a balance of £384 15s. 8d.

The Nelson Colonist (N.Z.) says that a horrible disease is breaking out among the children attending the Riwaka school.

News has been received by the mail that William Sutherland, B. A., has obtained his B.Sc. at the London University, and at the Honor ex aminations gained first-class honors and the Physical scholarships—£50 a year for two years. Mr. Sutherland gained a State school exhibition from the Model school in 1873, coming second on the list. He then studied at Wesley College, from which he matriculated with ci'edit and took the mathematical entrance exhibition. In 1878 he completed his

B.A. course, and in the following year, having been appointed Gilchrist scholar, proceeded to London University, where in his first year he won the Cloth-weavers’ scholarship. Some original work in electricity which he did during his course was favourably received by the Physical Society of London.

PARLIAMENT AND THE UNIVERSITIES.

Four Old Collegians are members of Parliament; Legislative Council : The Hon. Thomas F. Cumming. Legislative Assembly ; The Hon. Robert Ramsay, Mr. William Shiels, LL.B., and Mr. James Gibb.

At the Universities upwards of 350 old pupils have passed Matriculation, and about ninety have obtained Degrees.

The alterations in the Matriculation Examination, which were referred to in my last report, came into operation for the first time this year. For the ordinary Pass, the principal change has been the addition of four science subjects to the ten already open for selection. The most important change, however, has been the institution of a higher or Honor examination, in four groups of subjects—Classics, Mathematics, Modern Languages, and English. The ultimate effect on education of the new system will doubtless be beneficial ; but in the examinations held this year there has been a marked want of uniformity in the difficulty of the papers set in different subjects. This has been the case notably in the Honor papers, which in some subjects, such as English and French, were but slightly more difficult than those set for the Pass. In the Mathematical subjects, on the other hand, the Honor papers were so much higher than the Pass that the difference in the standard represented for an average boy at least two years’ school work. It is to be hoped that in future the Examiners will avail themselves of the power given them by the University to secure that there be something appi-oaching uniformity in the standard of the papers set in the various subjects, both for Pass and Honor,

As the Matriculation Examination under the new system includes every subject of school instruction, it will now be practicable to carry out to a fuller extent the division of the college work into two or three different courses of study, These, whilst coinciding to some degree, will be so arranged as to have a distinctive classical, scientific, or practical character. Boys will thus have an opportunity of studying the subjects suited to their mental capabilities and best calculated to prepare for their future career in life. Each pupil in the upper school will be required to select one or other of these courses, from which he will not be allowed to deviate. In this way every boy will study a sufficient number of subjects to qualify him for passing matriculation, and I would impress on parents that they could have no better test than that examination of the use which their sons have made of their time and opportunities at school.

I would further remind parents that the mere passing of the Matriculation Examination is not a sufficient preparation for entering the University, and that only those who are able to pass in honors can really benefit by the lectures, or expect to pass at the end of the first year. I trust, therefore, that every boy who intends to prosecute his studies after leaving school, will attend for a year or two in the sixth class, in which all the honor work is taught, and in which I shall in future allow no boy to remain who does not show by his application and industry that he has entered the class with the object of studying.—Scotch College Report,

ISutoxwn (Btracatxott gLprimenl


Our readers will be glad to learn that Mr. G. W. Brown, Secretary of the Education Department, has arrived in Melbourne by the mail steamer, after nearly twelve months absence from the colony. Mr. Brown paid a short visit to the Education Office on the 13th, and was introduced to the Minister who has taken office since Mr. Brown’s departure. Mr. Brown’s leave extends to the end of the month, but we understand that it is his intention to resume the duties of Secretary at once. We believe that Mr. Brown’s health has been improved by the holiday and voyage.

The Royal Commission on Education propose to sit to take evidence on the 18th February. The first witness who will be examined will be the Secretary of the Department, Mr. Brown.


The following are the papers set by the Department for Exhibitions at the examination held December, 1881 :—

GRAMMAR AND COMPOSITION.

( Time allowed two and a half hours.)

1.    Parse fully the words printed in italics in the following passage :— King Henry.—Your wondrous rare description, noble earl,

Of beauteous Margaret hath astonished me ;

Her virtues, graced with external gifts,

Do breed love's settled passion in my heart :

And lihe as rigour in tempestuous gusts Provokes the mightiest hulk against the tide,

So am I driven, by breath of her renown,

Either to suffer shipwreck, or arrive Where I may have fruition of her love.

Suffolk.— Tush \ my good lord ! this superficial tale Is hut a preface of her worthy praise :

The chief perfections of that lovely dame (Had I sufficient skill to utter them)

Would make a volume of enticing lines Able to ravage any dull conceit,

And, which is more, she is not so divine,

So full replete with choice of all delights,

But with as humble lowliness of mind,

She is content to he at your command ;

Command, I mean, of virtuous chaste intents,

To love and honour Henry as her lord.

2.    Analyse—Shakespeare indeed has said that a rose by any other name would smell as sweet ; but there are some things which are not roseB, and which are thought to smell a great deal sweeter by any other name than by their own.

3.    State the rules for the apposition of nouns, and for the agreement of the relative with its antecedent ; and give examples of verbs used absolutely.

•1. (a) Write each of the following sentences in a correct form, stating the rule violated in each case :—Who do you think I saw yesterday ? Neither John nor his sister Mary were present. After he done his home lesson, he spoke quite kind to his little brother. Between you and I, he acted like his brother did, (h) Show how each of the following words may be employed as different parts of speech, naming in each case the part of speech :—that, as, what, for, fast.

FRENCH.

( Time allowed two hours.)

1.    Translate into English—En vain implora-t-il, dans ses derniers instants, quelqu’un qui daignât lui donner la mort ; personne ne voulut lui rendre ce dangereux service, “ Quoi !” s’écriait-il dans son désespoir, “est-il possible que je n’aie ni ami pour défendre ma vie, ni personne pour me Fêter ?” Il serait difficile d’exprimer la joie des Romains lorsqu’ils apprirent sa mort. On arbora publiquement le signal de la liberté, et le peuple se couvrit la tête d’un chapeau semblable à celui que prenaient les esclaves après leur affranchisement. Le sénat n’y fut pas moins sensible ; Néron avait dessein de l’abolir, après avoir fait mourir tous les sénateurs, Lorsqu’il apprit les premières nouvelles da la rébellion, il fórmale projet de faire massacrer tous les gouverneurs des provinces, et tous les généraux d’armée, comme ennemis de la République ; de faire périr tous les exilés ; d’égorger tous les Gaulois à son armée ; d’empoisonner le sénat entier dans un repas ; de brûler Rome une seconde fois, et de lâcher en même temps dans les rues toutes les bêtes réservées pour les spectacles afin d’empêcher le peuple d’éteindre le feu.

2.    Parse fully with French syntax the preceding words in italics.

3.    Translate into English—Les habitudes de Napoléon étaient extrêmement simples, et il était d’une frugalité remarquable. Il déjeûnait à neuf heures et demie precises, sur un petit guéridon en bois d’acajou, couvert d’une serviette. Ce déjûner ne durait pas huit minutes ; il ne se prolongeait que lorsque Napoléon aspirant à redescendre à la vie privée, fermait son cabinet et se mettait à son aise. Alors, rien n’égalait sa douce gaîté ; ses expressions étaient rapides et pittoresques. Très souvent le préfet du palais lui proposait de recevoir dans ces instants quelques personnes favorisées, C’etaient, en général, des savants du premier ordre : Napoléon entretenait chacun du sujet favori de ses études ; il interrogeait avec autant d’abandon que d’affabilité. Bonaparte dînait seul avec l’impératrice ; le dimanche, il réunissait sa famille. Un seul service composait le repas ; Napoléon choisissait de préférence les mets les plus simples et ne buvait que du Chambertin fortement trempé. Le dîner durait de quinze à vingt minutes et etaitê toujours terminé par une tasse de café,


4.    Parse fully with French syntax the preceding words in italics.

5.    Give the following grammatical forms (a) Active Subjunctive

Imperfect 3rd plural of “buvoir.” (h) Indicative Preterite 1st plural of “ naitre.” (c) Indicative Present 2nd plural of “croitre.” (d) Indicative Present 3rd plural of “devoir.”    (e) Subjunctive Imperfect

3rd plural feminine of “ se flatter”.

6.    (a) Distinguish between the verbal adjective and the present parti

ciple a3 regards signification and inflection, (b) When are at, to, in rendered by a.; when by e«- ? (c) When are passive verbs followed by de and when by par ?    (d) (1) What exceptions are there to the rule that

cardinals do not take the mark of the plural ?    (2) What peculiarity is

thereabout the expression vingt-et-un? (e) In what respect does the preposition en differ from all the other prepositions in regard to government of a following verb ? Give examples.

7.    Translate into French :—(a) I will eat your black bread, and we

shall both gain by the exchange.    (h) We ought to reflect well before

undertaking anything, (c) This man must be enormously wealthy to offer 20 louis to whosoever shall find his dog. (d) He confided his design to only seven officers ; one of them Irish, the other Scotch, (c) The two armies came in sight of each other, at 2 o’clock in the afternoon, at a place called Culloden. (f) They lost all hope when, after passing two days in this frightful place, no one came to their assistance. (g) Your Father knoweth of what you have need before you ask it of him. (h) Two consuls between whom he was sitting, seeing him burst out laughing asked of him the reason for it.

LATIN.

( Time allowed two hours).

1.    Translate into English—Natio estomnis Gallorum admodum dedita religionibus ; atque ob earn causam, qui sunt affecti gravioribus jnorbis quique in proeliis periculisque versantur, aut pro victimis homines immolant, aut se immolaturos vovent, administrisque ad ea sacrificia Druidi-bus utuntur ; quod, pro vita hominisuisa vita reddatur, non posse aliter deorum immortalium numen placari arbitrantur ; publieeque ejusdem generis habent instituta sacrificia. Alii immani magnitudine simulacra habent, quorum contexta viminibus membra vivis hominibus complent, quibus succensis, circumventi flamma exanimantur homines. Supplicia eorum qui in furto aut [in] latrocinio aut aliqua noxa sint comprehensi gratiora diis immortalibus esse arbitrantur ; sed quum ejus generis copia deficit, ctiam ad innocentium supplicia descendunt. Deum maxime Mer-curium colunt. Hujussunt plurima simulacra, hunc omnium inventorem artium ferunt, hunc viarum atque itinerum ducem, hunc ad quaestus pecuniae mercaturasque habere vim maximam arbitrantur.

2.    Parse with full syntax the following words from the above passage— Morbis, reddatur, placari, immani, contexta, viminibus, succensis, noxa.

3.    Translate into English—At ii qui in jugo constiterant, nullo etiam nunc usu rei militaris percepto, nequein eoquod probaverant consilio per manere, ut se loco superiore defenderent, neque earn quamprofuisse aliis vim celeritatem que videraut imitari potuerunt: sed se in castra recipere conati iniquum in locum demiserunt. Centuriones, quorum nonnulli ex inferioribus ordinibus reliquarum legionum virtutis causa in superiores erant ordines hujus legionis transducti, ne ante partem rei militaris lau-dem amitterent fortissime pugnantes conciderunt. Militum pars, horum virtute summotis hostibus, praeter, spem incolnmis in castra pervenit; pars a barbaris circumventa periit. Germani, desperata expugnatione castrorum, quod nostros jam constitisse in munitionibus videbaut, cum ea praeda quam in silvis deposuerant trans Rhenum sese receperunt.

4.    Parse with full syntax the following words from the above passage :—Constiterant, quam, profuisse, vim, causa, considerunt, summotis, incolumis.

o. Translate and explain the words in italics :—

Quos ex Cisalpina Gallia consulis Sacramento rogavisset Taxo, cujus magna in Gallia copia est se exanimavit Hospites violare fas non putant.

6.    Decline—Virus, os, neuter, arcus, quisque ; and write out the following :—

Infinitive Future of Loquor.

Subjunctive Past Imperfect 2nd singular of Malo.

Subjunctive Present Imperfect 3rd singular of Fio.

Passive Indicative Past imperfect 3rd plural of Sisto, Imperative Future of Memini.

7.    (a) Give the Lai in for eighteen hundred and eighty-first.

(b)    Name the prepositions that govern two cases, and mention the

peculiarity of the preposition tenus.

(c)    Explain, with examples, what is meant by Inceptive Verbs.

8.    Translate into Latin—

(a)    The Ubii, who had previously given hostages, sent ambassadors

to him for the purpose of clearing themselves.

(b)    Csesar compelled the Nervii to surrender, and to give him hos

tages.

(c)    The Druids are accustomed to absent themselves from war, and

they do not pay tribute.

(.d) It is a well-known fact that many kinds of wild beasts are born in this wood.

(e) Who doubts that it is the duty of all men to do good to their parents and children.

(/) Who will dare to prevent us from worshipping God as we please ?

(,g) No one will deny that the Romans destroyed, in one year, two most prosperous cities.

([h) I am disgusted at the folly of men who drink too freely,

)*


4a2 b + 2a~^2a — b/ ' 6a2 + 5ab + b2


•05)


= -80:4-290.


•008 5 + ”04 2 1


•08


1


(ii)


EUCLID.

{Time allowed two hours.)

1.    {a) Give the origin of the names Euclid and Geometry, (b) (1) Describe generally the contents of the first book of Euclid, and (2) describe particularly, but briefly, the parts into which it may be divided according to the matter dealt with, (c) Distinguish fully a theorem from a problem, show of w'hat parts each consists, and point out an example of each in the following questions, indicating and naming its essential parts.

2.    {a) From a given point draw a straight line equal to a given straight line. (b) Write out the definition, axioms, and postulates referred to in showing that what is required in the foregoing question (a) has been done.

3.    Show that if the equal sides EE, DFol the triangle DEF be produced to G and II respectively, the angle GEF shall be equal to the angle EFH.

4.    Show that if at a point in a straight line two other straight lines upon the opposite sides of it make the adjacent angles together equal to two right angles, these two straight lines shall be in one and the same straight line. (a) What name is given to the method of proof employed in Euclid to establish this proposition ? (b) What is its distinctive feature ?

5.    In every triangle the angle opposite to the greater side shall be greater than the angle opposite to the less, Prove this.

6.    In the triangles PQR, STV, the two sides PQ, PR are equal to the two sides ST, SF, each to each, namely, PQ to ST, and PR to SF, but the angle QPR is greater than the angle TSV: prove that the base QR is greater than the base TV.

7.    Show that all the exterior angles of any rectilineal figure, made by producing its sides successively in the same direction, are together equal to four right angles.

8.    Describe a parallelogram equal to a given square and having an angle equal to a given obtuse angle.

9.    If in the triangle EFG the square described on the side EG is equal to the squares described on the sides EF, FG, prove that the angle of is a right angle.

ALGEBRA.

(Time allowed two hours.)

1.    What is Algebra ? Mention some of its uses.

2.    Explain these terms, giving examples of them:—An exponent, a simple expression, a compound expression, the terms of an expression, the square root, a formula.

3.    Express algebraically, the sum of «.and of twice the remainder after taking from three times y four times thc^excess of z over unity is multiplied by itself, and the product is divided by a number which, when multiplied by itself twice, wrould produce the quotient obtained by dividing x by five times the sum of y and z.

4.    (<t) Explain the term : Greatest Common Measure. (6) Why is it not very appropriate in Algebra ? What would be a better expression for it ? (c) To what operations is the finding of the G. C. M. subservient ’

(d) Find the G.C.M. of 3«* - 10«3 + 9x2 -2x and 2«2 —7«3 + 2x* + 8x.

5.    Find the simple expression that must be added to a5 + 2asx2(a-x) + 3ax{ai - «3) to make it an exact multiple of ¿2 + x2.

6.    Simplify

/9a?-\-ccb b    ci \    Sab

7. Solve the equations

... 2-4?, 6(-215aj

w

3cc —    1 x 12a:3—16m2+4a;

8.    A waggon-load of wheat and oats, weighing 2^ tons, is worth £17 5s 4d at 2s a bushel of oats (40 lbs.), and 4s 9d a bushel of wheat (60 lbs.). How many bushels of each are there?

9.    A person bought a horse for a certain sum, and after keeping it a month sold it for £6 4s less, by which he lost 1 of the prime cost and -A, of the keep ; but, had he kept it a month longer and sold it for the same sum, he would have lost a of the prime cost and ^ of the keep : find the prime cost and the keep per month.

10.    A number consists of three digits whose sum is 12; if it be in

creased by 9, the digits in the units’ and tens’ places will be interchanged ; but if it be increased by 90, those in the places of tens and hundreds will be interchanged ; find the number.    '    /

ARITHMETIC.    /

(Time allowed two hours.)

1.    Write down in words in a separate line the value of each figure in the number—70006050040'320105.

2.    (a) Write in words the meaning of each of these expressions :—

i-§xf + T\; |-§x(f + T3T); (*-*)(* + *);£=!•

(b) Reduce each of them to its simplest form.

3.    (a) Express the following statement^ decimals and signs The excess of 6 tenths over 75 ten-thousandths is multiplied by 8 hundredths and to the product is added twice the remainder after subtracting from 4 hundredths the quotient obtained by dividing 8 millionths by 1 thousandth. (b) Reduce to a single decimal the arithmetical expression of the foregoing statement, (c) Find the value of this decimal in ounces, pennyweights, and grains, when the unit is a cental (100 lbs. avoir.).

4.    A thousand mils being equal to £1, between what number of mils and the same number of farthings would the difference in value be exactly a penny ?

5.    The wheel of a railway carriage being 1 foot 10 -J-x inches in diameter, find how many turns it made in a journey of 100 miles, 58 chains 58’5S links assuming that at each of 18 stoppages the brake caused it to slide along 2J- yards instead of turning. Take the circumference of the wheel as 3f times the diameter.

6.    Two places are under the same meridian ; one is in lat. 37 deg. S., the other in lat. 17 deg. 33 min. 36 sec. S. Unavoidable deviations would make the route of a telegraph line between these places 50T05 miles longer thau the direct route. How many posts would be required for the line, allowing one for every ten rods, and taking a degree equal to 69 J-th British miles.

7.    Two cannon-balls have been equally heated throughout ; one is six inches, the other 4 inches, in diameter. The quantity of heat that each of them will impart to any object will be proportional directly to the square of its diameter and inversely to the square of its distance from the object. If at the distance of six feet from the smaller ball a Centigrade thermometer indicates 24 degrees, at what distance from the larger one would a Reaumur thermometer indicate 24*3 degrees, a degree of the latter being greater than a degree of the former, in the proportion of 5 to 4 ?

S. A rectangular field is 18-25 chains long and 13-5 chains broad. It yields per acre 4 tons 7 cwt. 12 lbs. of potatoes, of which f is worth £4 Is. 8d. a ton, and the remainder £3 10s a ton. (a) Find the area of the field in acres, roods, and perches, and find by Practice (h) the total yield,

(c) the value of the yield of one acre.

9.    If the excess of the mercantile discount over the true discount on a bill of exchange cashed 6 months before maturity is 4s. 4^7d., find the amount of the bill, the value of money being 6 per cent, per annum.

10.    A grocer buys 1 ton 1 cwt. of moist sugar for £19 12s. ; at what price per lb. must he retail it so as to make a profit of 23| per cent, on his outlay, supposing that while it is on his hands it loses by evaporation 1£ per cent of its weight ?

DICTATION.

(Time allowed half-an-hour.)

Columns of flame would rise and sink along the surface of this sea, and huge volumes of black smoke suddenly shoot into the air, as if volcanoes were working below. The black form of the Kremlin alone towered above the chaos—now wrapped in flame and smoke—again emerging into view—standing amid thisscene of desolation and terror,like Virtue ip the midst of a burning world, enveloped but unscathed by the devouring element. Napoleon stood and gazed on the scene in silent awe. Though nearly three miles distant, the windows and walls of his apartment were so hot that he could scarcely bear his hand against them. Said he, years afterwards—“ It was the spectacle of a sea and billows of fire, a sky and clouds of flame; mountains of red rolling flames, like immense waves of the sea, alternately bursting forth and elevating themselves to the skies of flame above. Oh ! it was the most grand, the most sublime, the most terrific sight the world ever beheld 1”—R.R., Book VI., p. 118.

GEOGRAPHY.

( Time allowed half-an-hour.)

1.    State what you know of the scenes and results of the explorations of Stuart, Mungo Park, Sir S. Baker, and Sir John Franklin.

2.    From what parts of the world do we severally obtain the following commodities -.—Ermine fur, spices, raw silk ? and say from what countries the following timber is mainly procured Teak, fir, logwood, sandalwood.

3.    Give approximately the following particulars respecting Victoria ; {a) Limits of latitude and longitude, (b) Area in square miles, (c) Population; and say what rural occupations are mainly carried on in Victoria.

4.    In what way and at what period did the following severally become British possessions India, Tasmania, Cyprus, Channel Islands, Fiji, Gibraltar ?

5.    What and where are the following ? Mention some circumstance of interest with which each is connected :—Potchefstroom, Zurich, Piia, Staub-bacb, Greenwich, Dargo, Cotopaxi, Alderney.

6.    Specify the locality of the following battle fields or sieges Naseby, Balaclava, Plevna, Waterloo, Richmond, Metz, Londonderry, Mooltan.

7.    Name the principal coal producing countries of the world, and say in what other countries large coalbeds, though unworked, are known to exist.

The following are the Science Papers set by the Department at the Examination, held December, 1881

BOTANY.

(Time allowed three hours.)

1.    Name the Natural Orders to which the following plants belong : — Murray Pine, She-oak, Beet, Melon, Duke of Bedford’s Tree (Senecio), Native Currant (Richea), Erodium, Kangaroo Grass, Pultenaea, Grass Lily, Bottlebrush, Diuris, Native Honeysuckle, Borouia, Petunja.

2.    Give the characteristics of one of the following Natural Orders :— Composite, Umbellifcrse.

3.    Describe the difference between an erodium and a geranium.

4.    Give a short description of the principal kinds of roots.

5.    Describe the characteristics of the Pome, Bacca, Follicle, Pepo, Capsule, Achene, Drupe.

6.    Describe the constituents of a living cell. Name some of the most important cell contents, their chemical composition and their uses in the plant’s life. Describe the growth of cells.

7.    Describe the reproductive and nutritive organs of the Pea and of the Acacia,

8.    Give a description of the internal structure of the stems of Monocotyledons and of Dicotyledons.

9.    To which division of the vegetable kingdom do Gymnosperms belong ? Name several orders of the same.

METALLURGY.

( Time allowed three hours.)

1.    What are the chief characteristics of fire clays?

2.    How is the value of a fuel determined ?

3.    Describe a reverberatory furnace,

4.    Compare the qualities of iron in its three conditions : wrought iron, cast iron, steel.

5.    On what docs the proportion of fluxes in iron smelting depend ?

6.    What are the common ores of iron, lead, zinc, copper, mercury,

tin ?    •

7.    What are the chief chemical and physical characteristics of gold ?

8.    Describe the Welsh method of copper smelting.

9.    What is plumbago, and why is it used in the manufacture of crucibles ?

10.    How is silver separated from lead by Pattinson’s process ?

PHYSIOLOGY.

( Time allowed three hours.

1.    Describe the structure and composition of muscle, striated and unstriated, and give an account of the condition known as rigor mortis.

2.    Give a description of the larynx, so far as it serves as the organ of articulate speech,

3.    Name and describe the divisions of the intestinal canal, and mention the sources of the various secretions poured iuto it.

4.    Give a full account of the structure, composition, and functions of the red blood corpuscles.

5.    Describe the mode of origin of the motor and sensory spinal nerves, and the observations and experiments by which their respective functions have been determined.

(i. Give a description of the ear from the membrane of the tympanum inwards, and state what is supposed to be the use of the several parts,

7.    What are the advantages, physiological and economical, of a mixed

diet ?

8.    What are the characteristics of a healthy dwelling?

9.    Mention the chief causes of the prevalence of ophthalmia or “ Blight,” and give some account of its prevention and treatment.

GEOLOGY AND MINERALOGY.

( Time allowed three hours.)

1.    Name and briefly describe the principal formations of the Tertiary

period.

2.    Name some of the characteristic fossils of the Upper Devonian

rocks.

3.    What geological formations occur in Victoria ? State what you know of their distribution.

4.    Why should the position of marine strata above sea level be referred to the rising up of the land, not to the going down of the sea.

5.    Describe the physical and the chemical characters of—copper, iron pyrites, arsenical pyrites, olivene, hornblende, cassiterite.

(!, How would you distinguish cassiterite from wolfram, calcite from selenite, galena from stibnite ?

7. Explain what is meant by the axis of a crystal.

S, How is the rhombohedron derived from the double six-sided pyramid.

9.    What name would you give to rocks composed of quartz, felspar, and hornblende ; hornblende and felspar ; augite, olivene and felspar ?

10.    Name and describe the minerals and crystal forms submitted.

MAGNETISM AND ELECTRICITY.

( Time allowed three hours.)

1.    What are the'magnetic elements at any place on the earth’s surface ? Describe a method of determining any one of them ; or, draw a rough outline map of Australasia, and trace on it the Isoclinic Lines of 50°, 55°, and .00°.

2.    A vertical bar of soft iron is brought near to the south end of a compass-needle in Melbourne ; describe and explain the effect on the needle according as it is on a level with the upper or the lower end of the bar.

3.    (a) An excited rod of sealing-wax is brought near to the knob of a gold-leaf electroscope; (b) the knob is touched by the finger; (c) the finger is removed ; (d) the sealing-wax is removed ; (e) the sealing-wax is again brought near to the knob ; (f) the sealing-wax is removed, and an excited glass rod is brought near to the knob ; [g) the glass rod is removed, and the knob is touched with the finger. State clearly what happens at each step, and explain the cause by means of rough diagrams.

4.    Describe fully the construction and action of the Electrophorus. What is the source of the energy which produces electrical separation? or. Describe fully the construction and action of the Torsion Electrometer. What is the law of the intensity of Electrical Attraction and Repulsion obtained by its uso ?

5.    What is the theory of the action of Lightning Rods? What is meant by the “ Back Stroke ” ? How is it caused ?

(i. A plate of amalgamated zinc is dipped into a vessel of water acidulated with sulphuric acid. Is the electric state of the zinc and the water changed ? A plate of platinum is dipped into the same water, and the two plates are connected by a copper wire ; what happens? The wire passes over and parallel to a compass-needle; how is the needle affected ?

7. Describe the construction of the Astatic Galvanometer. Why is the plate carrying the divided scale generally made of copper ?

8.    \Vhy is a Rheotome necessary to the action of an induction coil? Describe the construction and action of the Rheotome used for small coils, or, Why is a Relay necessary on long telegraphic lines ? Describe the construction and use of any form of Relay.

9.    Describe an experiment showing the production of a current in a wire by the inductive action of the earth ; or, describe Groves’ Gas Battery, and explain how it makes manifest the effect of Galvanic Polarisation in a cell,

10.    Express by an equation the connexion between the Intensity or Strength of a current, the electromotive force, and the internal and external resistance of a galvanic circuit containing a galvanic battery. Show that if the electrodes need to complete the circuit are short thick copper wires or bands, there is little gain of intensity by using several similar cells instead of one.

CHEMISTRY.

(Time allowed three hours.)

1.    Describe the apparatus used in the electrolytic decomposition of water. At which pole of the battery does the Oxygen appear ?

2.    What gas would you obtain by passing a current of Cl gas through an excess of solution of ammonia, and what are its properties ?

3.    Describe the preparation of pure HC1, of H2S04 and of IINOa; and what are the impurities generally present in commercial specimens of these reagents, and how would you detect them ?

4.    How would you distinguish Arseniuretted, Antimoniuretted, Phos-phuretted, and Sulphuretted Hydrogen from one another ?

5.    What are the principal constituents of—I. Milk. II. Urine.

6.    Mention the varieties of medicinal “ mineral waters,” and describe the means used in severally identifying them.

7.    One gramme of Hydrogen measures 11-19 litres at 0° Cent, and 760 millimetres Barometer, One gallon English equals, say, 4-54 litres. From the above data, calculate the amount of Oxygen gas, in gallons, at 10° Cent, and 740 millimetres Barometer obtainable from 100 grammes of Chlorate of Potassium.

8.    What physical and chemical facts are taken advantage of in the ventilation of dwelling-houses ?

9.    What are the main physical and chemical data required in the examination of soils ? Mention also the chief inorganic constituents of plants.

10.    Give your reasons for considering common air a mixture, and not a chemical compound.

ACOUSTICS, LIGHT, AND HEAT.

( Time allowed, three hours.)

1.    Newton, by calculation, deduced the velocity of sound in air at 0° C. as 916 feet per second. What was the source of his error, and how was it corrected by Laplace ?

2.    How would you ascertain if two tuning forks which, sounded separate^, appear to be in unison are really so ?

_ 3. What is resonance ? Explain the part it plays in the production of different vowel sounds of the same pitch by the human voice.

4. A pipe and a violin string are tuned to the same note in one room, and are then taken to another of a different temperature. Will they then be in unison ? Why ?

_ 5. What is interference ? Briefly describe experiments which illustrate interference of waves in the case of sound, light, and heat respectively.

6.    State the law of reflexion of light. Trace the course of a pencil of light by which an eye will see the image of a luminous point after one reflection from each of two parallel plane mirrors.

7.    Explain briefly the law of refraction by means of the wave theory of light.

8.    Explain clearly why a crack in a pane of glass is sometimes visible as a bright band, and why the crack sometimes throws a dark shadow on the ground.

9.    A rod is placed in a pencil of light proceeding from a luminous point. The shadow is thrown upon a screen. Explain briefly the production of the bands which occur on the edges of the dark shadow; or, Explain the construction of the Common Astronomical Telescope, and show how its magnifying power is determined theoretically,

10.    Explain by analogy the difference between heat and temperature.

_ 11 • What is meant by Latent Heat ? When heat becomes latent there is an apparent loss of energy ; is the loss real ? Why ?

12.    Describe any method of finding the specific heat of a substance, What are the defects of the method ?

13.    Describe an experiment for determining the mechanical equivalent of heat. Explain the phrase, t

14.    A plate of glass is interposed between a common fire and a thermometer, and afterwards betvyeen the sun and the thermometer; the difference of the readings of the thermometor is a. A plate of rock salt of the same thickness as the glass is then used for the glass, and the difference of readings is b. Which is greater, a or b ? Why ? or, From two similar rifles five rounds of blank and of ball cartridge respectively are fired, the weight of powder in each cartridge being the same. Which rifle will be the hotter after the firing ? Why ?

MECHANICS AND HYDROSTATICS.

(Time allowed three hours.)

1,    What is meant by the terms “ action” and “ reaction” in the third law of motion ? A bullet, weighing an ounce, is fired from a rifle weighing 10 lbs. ; the bullet leaves the rifle with a velocity of 1000 feet per second ; what is the velocity of recoil in the rifle?

2.    The acceleration of gravity is 32, when a foot is the unit of space, and a second that of time. Explain what this means ? What number will denote the same acceleration when a mile and an hour are the units of space and time respectively ?

Define the terms Energy, Potential Energy. Kinetic Energy. A stone. 11 b. in weight, is projected vertically upwards with a velocity oi 100 feet persecon 1; what will 1 e its potential and its kinetic energies at the end of two seconds?

4.    What can be inferred as to the direction and magnitude cf forces

which keep a body at rest, («) when there are two, (J>j when there are three forces? A triangle, A B (\ is made of straight rods, hinged at A, B, and G. An elastic band, P Q. is passed round the rods A B. A tending to draw them together ; P Q is not parallel to B ('. Show by a diagram the direction of the pressure at each hinge.    _

5.    A body, lib. in weight, describes a circle of 5 feet radius with

uniform velocity of 20 feet per second, on a horiz ntal smooth table ; a string fastened to the body pass«    in the table, at the

centre of the circle. What weight below the table would the string just support ?    .

(i. How is the total pressure on an immersed surface determined ? A conical wine-glass is filled with water ; a similar glass is filled, covered by a plate and inverted. Compare the total pressures on the glasses, assuming that the centre of gravity of a conical surface is in its axis at a distance of two-thirds of the axis from the apex of the cone.

7.    What is meant by the specific gravity of a body ? A glass of water is placed in a scale-pan, and counterpoised by weights in the other pan ; a piece of lead, weighing one lb. (sp. gr. 11). is dipped into the water by a string without touching the glass. Will the balance be disturbed? Why ? If so, what weight would restore, it ?

8.    State Boyle’s law, and by it explain the action of the air pump.

9.    Why does a balloon inflated with coal gas ascend ? Why does a rocket ascend ?

10.    Describe the construction and use of any form of hydrometer.

11.    Bubbles rising in a tumbler of aerated water increase in size. Why ? Why docs the froth in a tumbler of porter rise in the glass?

The following are the papers set by the Department for the Examina tion of ¡Students in Trainiug, at the examination held December, 1881:—

I-IIBTOKY.

{Timeallowed three hours.)

1.    Give an account of the life and influence of Godwine, Earl of

W essex.

2.    Give, with dates, a brief account of each of these events :—Strong-bow’s invasion of Ireland, The Treaty of Bretagnv, The first Poor Law, The Self-renouncing Ordinance, The Act of Settlement.

3.    Give particulars, including dates, of the occurrences for which the undermentioned places are remarkable t—Smitlifield, Bosworth, Dover, Quatre Bras.

4.    Write a brief account of the following persons :—Simon de Montfort; Richard Neville, Earl of Warwick; John Pym; George Stevenson.

5.    Write out a list of the Plantagcnet Kings ; giving the dates of their accession, and a short description of the character of each.

6.    Sketch the administration in Ireland of Wentworth, Earl of Strafford. To what end did his policy mainly point. ?

7.    What was the Solemn League and Covenant, and what was it designed to accomplish ?

8.    When and under what circumstances was the Bank of England established ?

9.    Narrate the circumstances which led to the American war of Independence.

10.    Describe how the establishment of two important Australian industries is mainly owing to the services rendered to them by Mr. John Macarthur.

11.    What was the work of the Hcntys in the settlement of Victoria? State what you know about Wm. Dutton.

12.    Give dates and particulars of the following events:—The founding of the colony of New South Wales, The publication of the first Victorian newspaper, McKinlay’s Expedition, The establishment of Responsible Government for Victoria,

13.    Narrate the circumstances oi the early discovery of gold in Australia.

CHEMISTRY.

( Time allowed three hours.)

1.    Explain the following terms :—Solution, sublimation, distillation, filtration, crith.

2.    Upon what principles are the terminations -ous’ -ic, -ate, -itc affixed to chemical compounds?

3.    Describe briefly the preparation of coal gas.

4.    Name the compounds of nitrogen and oxygen, giving the leading characteristics of each.

5.    How is iodine obtained ? What use is made of it, and what is the method of testing for it ?

G. Explain “ diffusion of gases,” and give the law of its velocity. How is this property of gases a benefit to mankind ?

7.    In a quantity of sulphuric acid, the sulphur weighs ISGlbs ; what weight of oxygen and hydrogen is present ?

8.    Compare phosphorus with its allotropic forms. Describe the preparation of safety matches.

9.    What takes place in the following Equations ?—Na2 8 + CaCO.3 = Nr2 COs + Ca S; 3 Ca (POs)2 + 10 C= P4 + (la (P04)2 +10 CO

10.    Describe the mode of occurrence, preparation, and properties of carbonic dioxide. Explain the use of it in the production of very low temperatures.

11.    What substances are used as disinfectants? Explain their action.

•    BOTANY.

(Time allowed three hours.)

1.    In the following description explain the words printed in italics :— ‘•The leaves of Labiatm are never a He mate, mostly opposite, sometimes whorleU. The stamens are less in-number than the lobes of the corolla. The an!hers are usually disi net y t wo celled. Each of the ovaries contains only owe ovule ; the radio e of the embryo is occupying a basal position.”

2.    Explain the statement that the various parts oi a flower are modifications of the leaf.

3.    D ‘scribe briefly the differences in structure between monocoty-ledonous and dicotyledonous plants in regard to root, stem, leaf, and flower.

4.    Enumerate the sub-classes of monocotyledons». Name two natural

orders of each sub-class.    .

5.    Describe botanically the stem, leaf, flowers, and fruit of the plant placed before you.

6.    Explain the formation of the fruit of the gooseberry, the strawberry, the raspberry, and the mulberry.

7.    Describe the characters of one of the following natural orders ;—(a) Orchidacem, (b) Epacridere.

8.    Refer to its order and sub-class each of the following plants :— Orange, loquat, cocoanut, banana, grape vine, pineapple.

9.    Name a wild (lower belonging to each of the following orders:— Com posit as, liliaccm, legumino-ic, epacridese, rutaceae, rammculaceas.

10.    From what plants and from what parts of them are the following economic products obtained flax, iodine, vanilla, quinine, arrowroot, sago ?

PHYSICS.

( Time allowed three hours)

1.    Explain the terms :—Unit of force, tenacity, conservation of energy, sublimation, prism, magnetic meridian, electric density.

2.    A body is projected vertically upwards with a velocity of 250 feet a second. Find its velocity after 3 seconds, and state in which direction it is then moving.

3.    What is a pendulum ? Explain its use in clock-work. To what property is its value due ?

4.    Explain the construction of a spirit level.

5.    A uniform cube of iron, whose side is 4| inches long, floats in a basin of mercury. Find the depth of the immersed portion of the cube ; specific gravity of iron being 7'2, and of mercury 13‘G.

6.    If a metal wire GO feet long, and with a cross section of -r-t--th of a square inch, be drawn out J-th of an inch by a weight of 201 bs.. what weight will be required to increase by -).j inch the length of a wire of the same metal 40 feet, long, and with a cross section of )-c> of an inch ?

7.    Show how changes in temperature atl'eet measurements of volumes, weight, density, and time.

8.    What generally occurs when a ray of light strikes a medium different to that it has been passing through ? What exceptions are there to the general rule?

9.    Explain what takes place when a small pencil oi light falls on a double convex lens in the following cases :—(a) Pencil of parallel rays, {}>) pencil of diverging rays at a considerable distance from the lens, {cj similar pencil close to the lens.

10.    Describe Coulomb’s Torsion Balance.

11.    Describe a galvanometer.

12.    Explain the cause of the flash of lightning, of its destructive effect, and of the thunder by which it is accompanied.

PHYSIOLOGY.

( Time allowed three hours.)

1.    Describe the bones of the spinal column and pelvis.

2.    Give an account of the structure and functions of the skin.

3.    Describe the shape and appearance, and give the approximate size of the red blood corpuscles in the following animals Musk Deer, Ostrich, Shark. ¡Snake, Toad. How may human blood be distinguished?

4.    Draw a diagram of the intestines, and describe them. What is meant by intestinal digestion ?

5.    Of what does the gastric juice chemically consist ? Describe an experiment made for producing artificial digestion.

6.    Describe the organ of taste. What do we generally mean by the sense of taste ?

7.    In what way does the nervous system regulate the flow of blood, so as to cause blushing or pallor ?

8.    What happens when a limb is said “to go to sleep?” What is the difference between afferent arid efferent impulses? Compare the speed of nerve impulses with that of other forces.

9.    Give examples of the diurnal waste of the human body, and state some simple proofs of this waste. What sensations impel us to repair it?

10.    Show by a diagram the system of ventilation you would recommend for a dwelling room, and give reasons for your recommendation.

ALGEBRA.

{Time, allowed, three hours.)

1.    Explain, and give examples of, a binomial, the power of a number, the cube root, a surd, a prime number, a measure of a number, simultaneous equations, an adfecte'd quadratic.

2,    (a) Write in words the meaning of each of the following expressions -—•

%c-3ti y. lx + 5y ; 3 n(x- y) . (lx + %)2 ;

______ l;X

(2x- \*>y . Ix + byA- 92x- 3y-f- -gw

(b) Find the value of each expression when    and ?/«=*2.

3


x

2"


(0


4x2-2 7-16x + 4x2'


..... 2æ+3

H —


8


3. By the formula for finding the cube of a binomial find the cube of. a—2b—3c.

4.    O) Extract the square root of

4x~4x(y—7/4) + y*—2 y3 +y*.

(b) What must be the length of one edge of a cube containing 244‘140625 cubic inches?

5.    (a) Investigate and state a rule for finding the Least Common

Multiple of two algebraical quantities.    ■

(b)    To what operation is the finding of the L.C.M. subsidiary ?

(c)    Find the L.C.M. of x2 - i, x3 + 1, and cc4 — 1.

6.    Simplify

f a + b a2+b2} _ f a3 -b3    a-b)

\ a—b^aM — b2 j ' | a3+b3    a + b \ ‘

7.    Solve the equations :

5x — l 7x — 1

_2_

lx — 3    ‘¿x — 5

3

... 3 - 2x 2x — 5

U)r025-2YT7=1

7y-3x

Q    - O + A

8.    A boy bought a certain number of oranges at 3 for 2d. and as many at 4 for 5d. ; he sold them all at 2 for 3d. and gained 5s. od. How many oranges did he buy ?

9.    The actual value of a cake of amalgam (gold and mercury) is £805 16s. 8d. at £4 an ounce for the gold and Id. an ounce for the mercury; but if it contained the gold and the mercury in equal quantities, its value would be £1,104 11s. 8d. How much of each does it actually contain ?

10.    A person bought some cases of wine for £30 ; if he had obtained 5 dozen bottles more for the same sura, the price per dozen would have been 6s. less. How many dozen did he buy? Interpret the negative answer.

EUCLID.

( Time attoived three hours.)

1.    Explain the terms :—Construction, demonstration, indirect proof, conversely, or the converse of a proposition, alternate angles, parallelograms about a diameter, a rectilineal figure inscribed in a circle, the side subtending an angle,

2.    Show that if two angles of a triangle be equal to each other, the sides opposite to the equal angles are equal to each other.

3.    Two triangles have two angles of the one equal to two angles of the

other, each to each, and one side equal to one side, similarly situated in regard to the equal angles in each ; state and prove their other points of equality.    *

4.    If a parallelogram and a triangle be upon the same base and between the same parallels; the parallelogram shall be double of the triangle ? Prove this,

5.    Let the straight line PR be divided into any two parts in the point Q ; show that the rectangle PR, RQ is equal to the rectangle PQ, QR together with the square on QR.

6.    The straight line BE is divided into two equal parts in the point F, and into two unequal parts .in the point G ; prove that the squares on EG, GE together are double of the squares on BP, FG.

7.    Show how to find the centre of a given circle.

8.    Prove that, in a circle, the angle in a semicircle is a right angle; the angle in a segment greater than a semicircle is less than a right angle ; and the angle in a segment less than a semicircle is greater than a right angle.

LATIN.

( Time allowed three hours.)

1.    Translate into English—Ut enim adolescentibus bona indole prm-ditis sapientes senes delectantur, lcviorque fiteorum senectus. quiajuven-tute coluntur et diliguntur : sic adolescentes senum prseceptis gaudent, quibus, ad virtutum studia ducuntur. Nec minus intelligo me vobis, quam mihi vos esse jucundos. Sed videtis, ut, senectus non modo languid a atque iners non sit, verum etiam sit operosa ct semper agens ali-quid et moliens; tale scilicet, quale cujusque studimn in superiore vita fuit. Quid ? qui etiam addiscunt aliquid ? ut Solonem versibus glorian-tem videmus, qui se quotidie aliquid addiscentem dicit senem fieri : ut ego feci, qui Grmcas literas senex didici; quas quidem sic avide arripui, quasi diuturnam sitim explore cupiens, ut ea ipsa mihi nota essent, quibus me nunc exemplis nil videtis. Quod quum fecisse Socratem in fidibus audirem, vellem equidem etiam illud : discebant enim fidibus antiqni: sed in literis certe elaboravi.

2.    Translate into English, Moriendum enim certe est, et id incertum, an co ipso die. Mortem igitur omnibus horis impendentum timens, qui poterit animo consistere ? De qua non ita longa disputatione opus esse videtur, quum recorder, non L Brutum, qui in liberanda patria est inter-scctus ; non duo Decios, qui ad voluntariam mortem cursum equorum in-citaverunt; non M. Atilium qui ad supplicium est profectus, ut fidem hosti datum, conervaret; non duo Scipiones, qui iter Poenis vel corpori*

bus suis obstruere voluerunt ; non avum tuum L. Paullum, qui morte prit collega in Cannensi ignominia temeritatem ; non M. Marcellum, cujus interitum ne crudellissimùs quidem hostis honore sepulturæ carere passus est ; sed legiones nostras, quod scripsi in Originibus, in eum sæpe locum profectas alacri animo et erecto, unde se nuuquam redituras arbitraren-tur. Quod igitur adolescentes, et ii quidem non solum indocti, sed etiam rustici, contemnunt, id docti senes extimescent ?

3.    Parse fully the italicized words in the above passages, and give the full derivation of the following words iners, voluntariam, insipientem satietas, nuper.

4.    Give the genitive singular and the nominative and genitive plural of : iter, quercus, caro, merces, funis, cinis, and celer. Decline talis fully.

5.    (a) Write down the following forms Passive Subjunctive Past Imperfect 3rd singular of Fero. Participle Future of Odi, Subjunctive Perfect 3rd plural of Gaudeo. Imperative future 3rd plural of Nolo. Subjunctive Past Imperfect 2nd plural of Loquor. Passive Infinitive

Perfect of Mordeo.

(b) Give the principal parts of the following verbs, marking in each case the quantity of the penultimate:—Intelligo, frango, reperio, ejicio, premo, cresco.

6.    Explain the idioms used in the following sentences :—Corioli oppidum captura est. Turpitudo pejus est quam dolor. Quid tua id refert ?

7.    (a) Distinguish with examples between the uses of the genitive and ablative of price, quality, place, where, (b) Give examples, showing the two simple ways of expressing comparision in Latin.

8.    Translate into Latin .-—(a) Then the old man is said to have read to the judges that play which he had just written, and to have enquired whether that appeared to be the verse of a dotard (desipiens). (b) Socrates says that the younger Cyrus, King of the Persians, a monarch excelling in talent and renown, was civil and courteous to Lysander, and showed him a field sown, well fenced, and carefully planted, (c) On Salinator (dative) boasting and saying as follows : It was by my exertions, Fabius, that you retook Tarentum,” Fabius, laughing, replies,

“ Certainly ; for had you not lost it I should never have retaken it ”    

FRENCH.

(Time allowed three hours.)

1.    Translate into English—De là il passa en Angleterre, où il se perfectionna dans la science de la construction des vaisseaux : il repassa en Hollande, et vit tout ce qui pouvait tourner à l’avantage de son pays. Enfin, après deux ans'de voyages et de travaux auxquels nul autre homme que lui n'eût voulu se soumettre, il reparut en Russie, amenant avec lui les arts de l’Europe. Des artisans, de tout espèce IV suivirent en foule. On vit pour la première fois de grands vaisseaux russes sur la mer Noire, dans la Baltique, et dans l’Océan ; des bâtiments d’une architecture régulière et noble furent élevés au miliea des hutte moscovites. Il établit des collèges, des académies, de3 imprimeries, des bibliothèques les villes furent policées ; les habillements, les coutumes changèrent peu; à peu, quoique avec difficulté : les Moscovites connurent par degrés ce que c’est que la société. Les superstitions même furent abolies : la dignité de patriarche fut etiente ; le czar se déclara le chef de la religion ; et cette dernière entreprise, qui aurait coûté le trône et la vie à un prince moins absolu, réussit presque sans contradiction, et lui assura le succès de toutes les autres nouveautés.

2.    Translate into English—D’abord il sembla vouloir réconcilier le roi avec la république ; il envoya des lettres circulaires, dictées en apparence par l’esprit de concorde et pa la charité, piégés usés et connus, mais où les hommes sont toujours pris ; il écrivet au roi de Suède une lettre touchante le conjurant, au nom de celui que tous les chrétiens adorent également, de donner la paix à la Pologne et à son roi, Charles XII répondit aux intentions du cardinal plus qu’ à ses paroles : cependant il restait dans le grand duché de Lithuanie avec son armée victorieuse, déclarant qu’il ne voulait point troubler la diète ; qu’il faisait la guerre à Auguste et aux Saxons, non aux Polonais : et que, loin d’attaquer la république, il venait la tirer d’oppression. Ces lettres et ces réponses étaient pour le public. Des émissaires qui allaient et venaient continuellement de la part du cardinal au comte Piper, et des assemblées secrètes chez ce prélat, étaient les resorts qui faisaient mouvoir la diète : elle proposa d’envoyer une ambassade à Charles XII, et demanda unanimement au roi qu'il n’ appelât plus les Moscovites sur les frontières, et qu’il renvoyât ses troupes saxonnes.

3.    In the preceding passages parse with French syntax the words printed in italics.

4.    Write out the following grammatical forms :—Imperative 3rd plural of Yalsir. Subjunctive Imperfect 3rd plural of Survivre. Indicative Preterite 1st plural of Vaincre, Indicative Present 1st singular of Mourir. Subjunctive Imperfect of Yavoir.

5.    (a. Distinguish with examples between the reflected and the reciprocal verbs. (b) How may expressions implying necessity, obligation, or want be rendered in French ? Give examples. (c) Distinguish between pas and point. When is pas used in preference to point ?

6.    («-) State with examples the different ways in which an infinitive depending on a verb is rendered in French, (b) When are personal pronouns, used as subjects, placed after the verb? Give examples. (c) Give Dc Fivas’classification of adverbs, with an example of each.

7.    Translate into French—(a) Somebody told me that they owed their misfortunes to their faults, (b) Many of your friends speak ill of one another, (c) You surprise me very much by telling me that you cannot translate this passage, (if) In prosperity it is agreeable to have a friend; but it is a necessity in misfortune. (e) Provided you know the ruling passion of anyone, you are sure to please him. (/) Ballarat gold is worth seventy-eight shillings an ounce, (g) There are many occasions when it would be better to be silent than to speak, (h) I accuse him of hindering me from reading your two speeches.

ENGLISH LITERATURE.    '

(Time alloived three hours.)    >    _

1.    Narrate the story of Caedmon, and describe the subject of his poem.

2.    What were the special services to English literature rendered by Wyclif and Chaucer ?

3.    Mention and account for the peculiar Celtic and national elements

which distinguish early Scottish poetry.    _

4.    What is blank verse ? Name the writer who first used it, stating

when he lived, and giving the title of his work. Mention subsequent writers who adopted it, and say for what kind of composition it is peculiarly suited.    _

5.    When and by whom was the “Faerie Queene” written? Give a brief outline of its scope and purpose.

6.    Give a sketch of the origin of the English Drama.    ,

7.    At what period did each of the undermentioned writers flourish? give a brief general description of their works:—Sir John Maudeville, Thomas Hobbes, William Cowper.

S. For what class of English words are Greek roots specially used ? Give examples, with the meaning of the roots.

9.    Show how the Law of Contraction has operated in the following words :—Mansion, vulgar, camp.

10.    In the following extracts point out the faulty metaphors, explaining the grounds of their faultiness :—

(a)    “ Was the hope drunk

"Wherein you dressed yourself?”

(b)    “ Justice’s eye, it stands to sense,

Can never stomach such transgressions,

Nor can the hand of Providence "Wink at your impious expressions.”

11.    Specify, with examples, the principal causes of obscurity in style.

12.    Explain the nature of the difference between scientific and non-scicntific composition.

13.    Name the poem in which each of the following passages occur, its author, and the period during which he flourished .—

(a)    “ I wol you tell a tale that I •

Learned at Padow of a worthy clerk.” .

Who was the worthy clerh ? In what sense is the word clerh used ?

(b)    All hail, pure lamp, bright", sacred, and excelling ; Sorrow and care, darkness and dread repelling ;

Thou world’s great taper, wicked men’s just terror, Mother of truth, true beauty’s only mirror,

God’s eldest daughter ; 0 ! how thou art full

Of grace and goodness ! 0, how beautiful !

(c)    Not for to hide it in a hedge,

Not for a train attendant ;

But for the glorious privilege Of being independent.

(cl) Dew drops are the gems of morning,

But the tears of mournful eve !

(c)    “ Sorrow,” said Mahmoud, “is a reverend thing ;

I recognise its right, as king with king.”

(f)    “ Then her countenance all over

Pale again as death did prove ;

But he clasped her like a lover,

And he cheered her soul with love.”

The following is the paper set by the Department for Examination in School Management at the Examinations held December, 1881.

SCHOOL MANAGEMENT.-Set I.

(Time allowed two hours.)

1.    A school has weekly 24 hours’ instruction. Give the time you would allot to each of the programme subjects in each class for boys and for girls, and the number and length of the lessons.—[N.B.—Singing and Drawing, Drill and Gymnastics are programme subjects.]

2.    (a) What are th5 marks and advantages of good classification ?

(b) Explain in detail the steps you would take to classify a new school.

3.    Describe briefly the Home Lessons you would give to a 4th class for each day of one week, mentioning exactly the nature and quantity of work in each subject.

4.    Draw up a set of questions in geography for the 3rd class, framed so as to test whether the subject has been taught intelligently and made nteresting by the class teacher.

5.    State what comments and explanations y7ou would give on the following passage occurring in a reading lesson of the 5th class :—

Tell me not in mournful numbers,

Life is but an empty dream !

For the soul is dead that slumbers,

And things are not what they seem,

Life is real ! Life is earnest !

And the grave is not its goal ;

“ Dust thou art, to dust returnest,”

Was not spoken of the soul.

Not enjoyment and not Sorrow,

Is our destined end or way ;

But to act that each to-morrow Find us further than to-day.

6.    What are the means for securing order, attention, and interest in a class? To what causes are difficulties in obtaining these requisites due ?

7.    State exactly when and by whom the entries in the Teachers’ Timesheet are to be made. What entries are to be made in the Roll-sheets each day ? State exactly when each entry should be made.

The following are the papers set by the department for general examination of pupil teachers, held December, 1881 :—

COMPOSITION.—First Class.

“ Above the clouds 1”—not only no rain, no mist, no dpw, but a scorching syn, and an air, both by day and by night, dry to almost an alarming degree. The further we advance, and the higher we ascend, the drier becomes the air ; while at the same time the strength of the northeast trade-wind is continually decreasing, and at the height of about six or seven thousand feet has pompletely died away.

Not that it has ceased elsewhere as well, for the driving clouds below show that it is still in its accustomed violence there. The distant movements of those rollers of white cloud betray that it must yet be raging down there in all its strength, tearing the mist piecemeal, and bowing down the heads of suffering palm-trees, and lashing the sea into foam-crested waves. Heaven grant that no cry of shipwrecked mariners be borne on the breeze ; and, more still, that no evil thoughts be engendering in the cities of men.

It was when our party on the mountain were in the fullest enjoyment of their daily and nightly views of the heavens, that their friends in the towns of Tencriffe near the sea-coast wrote to them most sympathisingly :    “ Oh, what dreadful weather you must have been suffering !

Down here we have had for three weeks the most frightful continuance of storms—constant clouds, rain, and howling winds; and if that was the case with us, what must it not have been with you at the greater height. ”

Yet at the greater height, at that very time, the air was tranquil and serene, the sky clear, and bad weather entirely confined to that lower depth in the atmosphere, beneath “the grosser clouds.”—p. 15, Jill., Book V.

Second Class.

Onward she comes, in gallant combat with the elements, her tall masts trembling, and her timbers starting on the strain ; onward she comes— now high upon the curling billows—now low down in the hollows of the sea, ns if hiding for the moment from its fury ; and every storm-voice in the air and water cries more loudly yet, “ A ship 1”

Still she comes striving on ; and at her boldness and the spreading cry, the angrj7 waves rise up above each other’s hoary heads to look ; and roundabout the vessel, far as the mariners on her deck can pierce into the gloom, they press upon her, forcing each other down, and starting up, and rushing forward from afar, in dreadful curiosity. High over her they break, and round her surge and roar ; and, giving place to others, moaningly depart, and dash themselves to fragments in their baffled anger ; still she comes onward bravely.

And though the eager multitude crowd thick and fast upon her all the night, and dawn of day discovers the untiring train yet bearing down upon the ship in an eternity of troubled water, onward she comes, with dim lights burning in her hull, and people there, asleep ; as if no deadly element were peering in at every seam and chink, and no drowned seaman’s grave, with but a plank to cover it,were yawning in the unfathom able depths below.—p. 263, R.Jl., Book IV.

GEOGRAPHY. —First Class.

Set I.

( Time allowed one hour,)

1.    Comment on and explain these lines :—

“ The very law which moulds a tear,

And bids it trickle from its source—

That law preserves the earth a sphere,

And guides the planets in their course.”

2.    In latitudes above 30° N. and 20° S., the range of temperature from summer to winter, and from day to night, in places on the same parallel of latitude, is least on the west coast, and greatest on the ease coast of a continent. State the cause of this law, and give some examples in illustration of it.

3.    Give an account of the Trade Winds, stating fully how they arc caused.

4.    Describe the mountain system of the Austrian Empire.

5.    What and where are the following:—Birmingham, Tobago, Burrumbeet, Tyrol ?

6.    A bushman rides on horseback from Beechworth to Castlemaine. Name the counties and rivers over which he passes.

Bet II.

( Time allowed one hour.)

1.    State the several proofs of the earth’s sphericity.

2.    In the Northern Hemisphere, in latitudes above 30°, the mean temperature of places on the same parallel of latitude is higher on the west coast of a continent than on the east coast. State the cause of this law, and give some examples in illustration of it.

3.    Describe the following winds, stating where they occur, and what is their effect:—Typhoon, Sirocco, Harmattan, Simoon, and Etesian Winds. Define the following words ¡—Cyclone, breeze, gale, and hurricane.

4.    Describe fully the great lake system of North America.

5.    What and where are the following :—"Wittenberg, Sumbawa, Hind-marsh, Manitoba ?

6.    A bushman rides on horseback from Portland to Camperdown. Name the counties and rivers over which he passes.

GRAMMAR.—First Class.

Set I.

[Time allowed one hour.)

1.    Parse with full syntax the words printed in italics in the following passage :—

Poor wives of fishers 1 Ah, ’tis sad to say,

Our sons, our husbands, all that we love best,

Our hearts, our souls, are on those waves away—•

Those ravening wolves that knew nor ruth nor rest,

Think how they sport with those beloved forms,

And how the clarion-blowing wind unties Above their heads the tresses of the storms;

Perchance even now the child, the husband dies 1

2.    Analyse according to Morell’s second scheme from “ Ah, ’tis sad” to “ belovdd forms.”

3.    What is a noun sentence? What are the places it may take, in the complex sentence of which it forms a part ? Give an example of each.

4.    Give the meanings and derivations of the following words, the language from which each is derived, and the meaning or force of each part ¡ — Antitype, collision, reinvest, ferry.

5.    Name the metre in which the following lines arc respectively written, and mark the accented words and syllables in each :—

(a) Rome shall perish ! Write that word In the blood that she has spilt.

(h) There was a sound of revelry by night.

0.    Give the rule for the employment of the comma in separating sentences, with an example. What are the exceptions to this rule ?

Set II.

(Time allowed one hour.)

1.    Parse with full syntax the words printed in italics in the following passage :—•

For we can never tell where they may be Who, to make head against the tide and gale,

Between them and the starless, soundless sea,

Have but one bit of plank, with one poor sail.

Terrible fear ! We seek the pebbly shore,

Cry to tiic rising billows. “ Bring them home."

Alas I What answer gives their troubled roar To the dark thought that haunts us as we roam.

2.    Analyse according to Moreli’s second scheme from *• We seek ” to “ wo roam.”

3.    Define a compound sentence, and give, with examples, the two most common logical relations in which co-ordinate sentences stand to each other.

4.    Give the meanings and derivations of the following words, the language from which each is derived, and the meaning or force of each part:—Synonymous, awry, implacable, expectorate.


A RITH M ET IC.—First Class. Set I.


( Time allowed one hour and a half.)

1.    A field 13 chains 14 yards long and 12 chains 16 yards broad was ploughed by C horses in 2 days of 8 hours each ; find the length of a field 14 chains 7 yards broad which can be ploughed by 8 horses in 3 days of 7 hours each.

2.    (a) Simplify


(4-3*

(b)    Express in the lowest terms a central (100 lbs.) as a fraction of a ton.

3.    (a) A degree of the earth’s circumference being equal to 60 geographical miles or 6Q-& British miles, express a geographical mile as a decimal of a British mile exactly.

(/;) Reduce 018 to a vulgar fraction in its lowest terras.

(c)    Work out the result of

(•3162276- -052)—(-001105 4- -85).

4.    What will £725 amount to in 3 years at 4| per cent, per annum oo ‘>ipound interest. ?

5.    State and explain the rules for determining mentally—(a) The interest on £85 for IS days-at 5 per cent, per annum, (b) The value of a ton at 2kl. a lb.

6.    (a) Explain the terms : Bills Receivable and Bills Payable.

(b) Post the following transactions :—

1881.

Sept. 26th. Bought for cash 25 saddles at £3 5s. each.

Bought from W. Souter 10 sets of buggy harness at £7 10s. a set, and gave him my bill at 2 months for £75.


Sept. 27th. Sold to James G reen saddle and harness for £15.

Cash sales this day, £20.

Nov. 29th. Paid my acceptance to W. Souter, £75.

Set H.

( Time allowed one hour-and-a-half.)

1.    With a single-furrow plough 11 acres 2 roods is ploughed in 12 days of 8 hours each ; in how many days of 10 hours each will 57 acres 2 roods be ploughed with a double-furrow plough, supposing that the latter ploughs 8 acres in the same lime that the former takes to plough 5 acres ?

2.    (a) Simplify    *


(b) Express in the tion of a mile.


0 ~UX A + eU 2M (i+T’V °* M U’* A

lowest terms 7 chains and 11 yards (lineal)


as the frao-


3. Simplify


+


3.    (a) What number of dwts. and grains is equal to ’5625 oz. ? ( Reduce -0439 to a vulgar fraction in its lowest terms, (c) Multiply tl quotient of '84035-r ’67 by the quotient of 136-5-f ‘065.

4.    At what rate per cent., simple interest, will £2650 amount ■ £2959 3s. 4d. in 3 years and 4 months ?

5.    State and explain the rules for determining mentally—(a) The i; terest on £65 for 7 months at 6 per cent, per annum. (b) What 9-Jp a week will amount to in a year.

6.    (a) Explain the term double entry in book-keeping. On wb; principle does it proceed? (b) Post the following transactions :—

1881.

Nov. 4th. Bought of James Welch 25 tons of coal at 28s a toi

Paid him £20 on account.

Sold John Brown 10 tons of firewood at 10s. 6d. ton, and 3 tons coal at 35s. a ton.

Received from him £5 on account.

Nov. 5th. Paid subscription to hospital £5 5s.

,, wages to employes ... £6 10s.

Nov. 7th. Sold for cash 4 tons coal at 32s. a ton.

Dec. 1st. Paid James Welch £15.

SUBJECTS OF FIFTH ROYAL READER.—First Class.

Set I.

(Time allowed half-an-hour.)

1.    Describe the Frigate Bird. How does he commonly procure hi food ?

2.    Give an account of the Battle of the Nile.

3.    State what you know about the following :—Millet, sago, flax.

• Set II.

(Time allowed hadf-an-hour.)

1.    Describe the Condor. On what does it feed, and how is it hunted?

2.    Give an account of the expedition of the Pilgrim Fathers.

3.    State what you know about the following :—Tapioca, hemp, rice.

GRAMMAR.—Second Class.

Set I.

(Time allowed one hour.)

1.    Parse fully every word in the following sentence:—“lie wa aware that there was a chasm beyond, by which he believed the cscapt of the animals to be cut off.”

2.    Name all the distinguishing adjectives, and point out their respeC' tive uses.

3.    (a)That cannot be used for who in all circumstances.” Explain this, (b) Describe and exemplify the use of what and of as as relatives.

4.    Give the auxiliaries of Tense, the part of the principal verb with which each of them is joined, aud the name of the tense so formed.

5.    State the meaning of each prefix and affix in the following words and the language from which it is taken :—Antipodes, bicyclist, dribblet, forgiveness, parallelism, postcript.

6.    From what words, and by what processes have these nouns been derived ¡—Belief, length, shaft, shock, stitch, theft?

Set II.

1.    Parse fully every word in the following sentence ¡—“The oyster is yet alive, and to force the shell open with violence might injure the pearl that lies bidden within.”

2.    “Their ctymohgy as well as use show a, an, and the to be really adjectives.” Prove this statement.

3.    (a) What words are combined with the simple pronouns to make

compound ones? (b) With what form or forms of the simple pronoun is each of them combined ?    (e) What effect has each of them on the

meaning of the simple pronoun ?

4.    Give the auxiliaries of Mood, the part of the principal verb with which each is joined, and the name of the mood so formed.

5 State the mcauing of each prefix aud affix in the following words and the language from which it is taken ¡—Descendant, disbeliever, dotard, eulogize, supervision, winsome.

6. From what words, and by what processes have these verbs been derived -.—Advise, breathe, graze, set, smear, twirl ?

AIIITIIMFTIC.—Second Cl ash.

Set I.

(Time allowed one hour.)

1.    A rectangular block of stone is 1 yard 2 ft 6 in, long, 1 ft, 6 in. bn ad, and 91u.thick : find its weight if a cubic foot weighs 1 cwt 3 qrs. 22 lbs.

2.    The value of a chain weighing 2oz. lldwts. lflgrs., made of 16 carats fine, is £15 10s. ; find the value of one weighing loz. 18dwt., mads of gold 20 carats fine, assuming (lie workmanship to be equal in thetwc cases.

ttV of (1

4.    (a) Express -1875 as a vulgar fraction in its lowest terms.

(b) Divide ill - .sum of -95, -0002, and ‘001 by the difference betweer •8 and 1 '04.

5.    State and explain the rules for determining mentally :—(a) Tb( price of ‘20 dozen of brooms, at Is. Id. each, (b) The interest on £140 for six months, at 5 per cent, per annum.

Set If.

(Time allowed one hour.)

1. A room is 16ft. long, l ift. wide, and 12ft.6in. high; the door, windov and fire-place occupy 75 square ft ; find the cost of paperiDg the wall with paper 1ft. 9in, wide, at 4s. Gd, per dozen lineal yards.

■1SJ.


4. (a) Find the decimal exactly equivalent to -§£.

(b) Divide the difference between '2408 and *25 by the product of


•0023 and '016.

5. State and explain the rules for determining mentally—(a.) The value of two gross of pencils, at SJ-d. each pencil. (b) The price of 3cwt. of flour, at £9 6s. 8d. a ton.

GEOGRAPHY.—Second Class.

Set I.


( Time allowed half-an-hour.)

1.    Describe fully the hunting of the bison and the dangers attached to it.

2.    Write a brief account of the invasion of Britain by the Romans.

3.    Give a short account of the life of Sir Richard Arkwright.

4.    For each of the following words give another of the same pronunciation but different spelling, and coustruct short sentences showing the applications of the words forming each pair :—Faint, slow, leak, done.


2.    If 45cwt. Bqrs, 61bs. of oats contains 128 bushels 1 peck, how many bushels are there in 3 tons of wheat, the weight of a bushel of oats being to that of a bushel of wheat as 2 is to 3 ?

3.    Simplify

t___    2^ + 1-^ of wy

H — 3-[Time allowed forty minutes.)

1.    If the circle of illumination coincided with the equator, in what position would the earth be ?

2.    What is the length of a degree upon the earth’s surface? Explain how it is ascertained.

3.    Where and what are the following Morea, Simplon, Toulouse, Iviza ?

4.    Name in order the principal rivers falling into the Mediterranean and Adriatic from Gibraltar to Venice, stating through what country each flows.

Set II.

(Time allowed forty minutes.)

1.    Explain what would happen if the axis of the earth were not inclined to the plane of its orbit.

2.    Show how the difference of time between two places indicates their difference of longitude.

3.    What and where are the following :—Crimea, Schreckhorn, Toulon, Cerigo ?

4.    Name in order the principal European rivers from the Archipelago to the sea of Azov, both inclusive, and say through what countries they severally flow,

SUBJECTS OF FOURTH ROYAL READER.—Second Class.

Set I.

[(Time allowed half an hour.)

1.    Write what you know about the means adopted in searching for pearls.

2.    Give an account of the battle of Bannockburn.

3.    Mention the principal events in the reign of Edward IV.

4.    For each of the following words give another of the same pronunciation but different spelling, and construct short sentences showing the applications of the words forming each pair :—fur, manner, moan, climb.

Set II.

DICTATION.—Third Class,

As the herd approaches us, it swings round its front, at right angles, and makes off westward. We dash forward and divide it into two parties. We also separate, some of our hunters following one part of the herd ; the others, the remainder.—p. 127, R.H., Book IV.

GEOGRAPHY.—Third Class.

Set I.

(Time allowed twenty minutes.)

1.    Where are the following ;—Cape Schanck, Exmouth Gulf, Groote Islands, Freycinet Peninsula ?

2.    (a) Where are the following mountain ranges, and what is the direction of each ¡—Blue Mountains, Darling Range, Tararua Mountains, Wanderer Range? (b) State accurately the position of the following towns :—Kapunda, Timaru, Port Albert, Rockhampton.

3.    In the border traffic state what merchandise mostly passes—(a) From Victoria into New South Wales. (b) From New South Wales into Victoria.

Set II.

(Time allowed twenty minutes.)

1.    Where are the following :—Cape Grim, Lacepede Bay, Port Denison, Banks Peninsula?

2.    (a) Where are the following rivers, and into what do they flow :— Roper, Genoa, Clutha, Arthur? (I/) State accurately the position of the following towns :—Toowoomba, Albany, Grafton, Casterton,

3.    Specify the chief exports of New Zealand.

GRAMMAR.—Third Class.

Set I.

(Time allowed half-an-hour.)

1.    Parse simply every word in the following sentence:—‘‘She thus drank at her ease of the water, which, but for her clever thought, she would never have been able to reach.”

2.    Divide the following words into syllables, stating the rule applicable to each case:—presentiment, current, aerial, ablest, subordinate, writer.

3.    Write out Morell’s table of Nouns, but in place of his examples set down the following ones in their proper subdivisions :—A Cicero, civility, coat, famine, iron, king, mob, to read, Richard, Spaniard, ton, war.

4. Form sentences containing— {a) The possessive plural of Norman ;

(b) The objective plural of which; (c) The superlative of often; (d) The 3rd per. plural of the pluperf. pot. pass, of forbid.

Set II.

(Time allowed half-an-hour.)

1.    Parse simply every word in the following sentence.— “It, afterwards did all that a dog could do to prove how thankful it was for the pity and kindness shown to it.”

2.    When a syllable, such as ing, ed, or er, has to be added to a word ending with a consonant, in what cases must this consonant be doubled, and in what not ? Give examples.

3.    Write out Morell’s table of Adjectives, but in place of his examples set down the following ones in their proper subdivisions :—any, bluish, comfortable, daily, dark, either, few, great, ten, tenth, slow, yonder.

4.    Form sentences containing—(a) The possessive singular of the masculine of marchioness ; (b) The possessive plural of who ; (e) The positive of the adverb more ; (d) The 3rd per. plural of the past snbj. (or condl.) pass, of wring.

ARITHMETIC.—Third Class.

Set j.

{Time allowed half-an-hour.

1.    Find by Practice the cost of making 203 miles 65 chains 55 links of railway, at £3,050 15s. Od a mile.

2.    A person buys 4 tons 13cwt. 3qrs of sugar for £153 2s. 6d. ; at what price per ton must he sell it so as to make a profit of £21 17s. 6d, on the whole quantity ?

3.    Show how you would calculate mentally—(«) The price of 36 articles at 7£-d, each, {b) The price of a dwt. at £4 Is. Sd, an ounce.

ARITHMETIC.—Third Class.

Set II.

{Time allowed half-an-hour.)

1.    Find by practice the value of 130 tons 12 ewt, 2 qrs. 16^ lbs. of tea, at £228 13s 4d a ton.

2.    A milk retailer buys milk at the rate of 10s 7vjd for 10 gallons 2 quarts 1 pint. His weekly receipts from the sale of 88 gallons 1 quart and 1 pint are £7 7s 3|d. Find his weekly profit.

3.    Show how you would calculate mentally—{a) The price of 12 lbs. of silver, at 4s 9d an ounce, (b) The price of 56 lbs., at £5 15s a ton.

SUBJECTS OF THIRD ROYAL READER.—Third Class.

Set I.

(Time allowed half-an-hour.)

1.    What is sponge ? Where is it found, and how obtained ?

2.    Describe fully the habits of the ostrich. How is he hunted ?

3.    What were the two plants brought into England by Sir Walter Raleigh ? State all you know about their introduction.

4.    Give the meaning of each of the following words: —Horizon, petition, gait, incident.

Set II.

(Time allowed half-an-hour.)

1.    Of what are ink, pens, and pencils severally made ?

2.    Describe fully the habits of the beaver. How is it caught ?

3.    Narrate the story" about Henry V. and the Lord Chief Justice of England.

4.    Give the meaning of each of the following words:—Matron, wail, stubborn, senate,

HEAD TEACHER, country, 30 x 50, results 88‘235, wishes exchange Assistant, Melbourne or Suburbs. Address—care of Mr. Roth, Tivoli Place, South Yarra. ____

HEAD TEACHER, 20 miles from Ballarat, allotment 20 to 30, per centage 93, post-office attached, beautiful country, excellent residence, is willing to exchange with assistant in Melbourne, Geelong, or leading suburb. Address, “ Civis,” Schoolmaster Office.

HEAD TEACHER, 23 miles from Melbourne, daily coach, wants exchange with assi tant in Town or Suburb. “ Desirous,” care P. Matthews, Esq., 52 Collins-street.    ___

HEAD TEACHER, near Melbourne, would exchange with Up-country Teacher, with allotment 50 x 75. Address, Exchange,” care of Mr Bruce, Stationer, Elizabcth-street, Melbourne.

HEAD TEACHER, near seaport town, allotment 30 x 50, residence, attached, wishes to exchange with another near town or railway Address, “ X. Y. Z.,” Schoolmaster Office.__

HEAD TEACHER, allotment 30 x 50, Workrmstress’ position vacant wishes exchange with another near Geelong or Melbourne. Address,  Spiro,” G.P.O., Melbourne.

HEAD TEACHER, country, desires exchange, 30 x 50; results 81, house, three rooms, near large railway town. Address Bona Fide,” Echuca, ECOND ASSISTANT, North-Eastern District, main line, allotment 250 x 275, wishes to exchange with any assistant, Melbourne or Suburbs. Address, “ Opportunity,” Schoolmaster Office._________

^ONCE A WEEK.”

PRICE THREEPENCE.

Continued Stories, Tales, Sketches, Illustrations, Puzzles, &c.

FOR EVERY HOUSEHOLD.

s.


NOTICE.

MR. H. M'KTNLEY

gOLICITOR, CONVEYANCER, And

PROCTOR,

Has Removed to

86 COLLINS STREET WEST.


rp ATE’S PARCELS POST EXPRESS

FIXED PRICE.

NO EXTRAS. NO TROUBLE.

Delivery to door at any

21b

41b

61b

101b.: 201b.

addrets in

8.

d.

s.

d.

8.

d-

S.

d. s.

d.

Great Britain ... Continent of Europe,

4

6

6

0

7

6

10

014

0

America, & Canada

7

G

9

612

0

16

0 21

0

Sydney, Hobart, Launceston

1

2

6

3

0

3

6

4

0 6

0

New Zealand Ports

(except West Coast) Adelaide, Brisbane.

4

0

4

6

5

0

5

6i 7

6

No further charge whatever. Very small increase for heavier weights. Delivery at country addresses in Australia, inland carriage only added.

Receiving offiee-

TATE’S T

A R C a LS P O 8 T B X P K E 8 8.

DELIVERY

at DOOR any address in the world.

EITHER to

or from Britain from 4s. 6d.

TO or from

other Australian ports from 2s. 6d.

NO further

CHARGE whatever. No trouble.

Any SIZE, weight, or shape.

EVERY possible ASSISTANCE afforded.

INQUIRIES

plainly answered.

Branches, every town in Great Britain, delivered in Melbourne at nearly similar rates.

Head Office :—

FREDERICK TATE, CUSTOMS AGENT, &c., 13 Market street, Melbourne.

Q O M P E TENC Y    E X A MI NATION.

TUITIO SPIN CLASS, BY CORRESPONDENCE, OR PRIVATELY.

Other Work, by Arrangement.

JAMES L. ROBERTSON, B.A.; 71 C CAKES DON-ST., EMERALD HlLL.

A LEX. MUCIN LEY & CO.,

GENERAL

PRINTERS AND PUBLISHERS,

Cl Queen-sfreot, Melbourne.

In Bookwork and General Publishing our large experience is a guarantee of all work being executed in the best stylo, while having a first-class stock of the best and latest material ensures expedition and good workmanship.

The following papers are issued from this office :— WEEKLY.

“Punch,”    “Bulletin,”

“ Faithful Words,”    “Once a Week.”

FORTNIGHTLY.

“Australian Law Times,”    “Jewish Herald,”

MONTHLY.

Schoolmaster.”

“ Monthly Messenger.”

BUY

AND READ

T H E N E W

WEEKLY PAPER.

Price Threepence.

“ONCE

A WEE K

16 PAGES.

PRICE THREEPENCE.

ALL

BOOKSELLERS.

Alex. M'Kinley and Co., Publishers, 61 Queen Street, Melbourne.

EAD TEACHER, Country School, allotment 30 x 50, four-roomed residence, extra worth £15 annually, results 80, would exchange with Head Teacher of same allotment, if school be within 6 miles of railway-station ; or with 50 x 75 allotment. Warm climate preferred. Address, “Nihil,” P.O., Echuca.

HEAD TEACHER, Country Scl.ool, 20 x 30, percentage 70, wishes exchange. Address, “ C.B.A.,” care of A. M’Kinley & Co., Queen-street, Melbourne.

rpHE SCHOOLMISTRESS WALTZ, by No. -L.    1,420. Melbourne—Messrs. Glen, Allen, and

Nicholson. All music-sellers, Geelong and Ballarat.

I WARREN BALL’S “Hints to Candidates * for Teachers’ and Matriculation Examinations,” Is.; posted, Is. Id. Mullen, Melbourne.

CANDIDATES for EXAMINATIONS prepared by correspondence or otherwise. I. Warren Ball, South Yarra.

TPXAMINATIONS.— UNIVERSITY

and DEPARTMENTAL. .

TUITION in CLASS by Correspondence or otherwise. Teems, &c.,

JAMES L. ROBERTSON, B.A. (Melbourne).

71 Clarendon Street, Embrald Hill,

MULLEN’S

New Classified Catalogue of School, College, and Technical

EDUCATIONAL WORKS

May be had gratis on application, or posted on receipt of address.

SAMUEL MULLEN,

Wholesale & Retail Bookseller & Stationer, 29 & 31 COLLINS ST. E., MELBOURNE.

Price One Shilling,

By Post—In Victoria, Is. (id.; Out of Victoria, 2s

“QEPARTMENTAL EXAMINATION OF TEACHERS.

TEACHER S’ GUIDE

TO

AUSTRALASIAN EXAMINATIONS.

Containing the

PROGRAMMES & EXAMINATION PAPERS

of

Victoria    South Australia

New South Wales    Auckland, N.Z.

Queensland    Wellington, N.Z.

And Tasmania.

108 PAGES WITH STIFF COVER.

The ahovo book contains the Programmes and Examination Papers of December, 1877, of all the colonies, and is reduced to the low price of

ONE SHILLING.

By Post—In Victoria, Is. 6d.; Out of Victoria, 2s

ALEX. M‘KIN LEY & CO.,

PRINTERS AND PUBLISHERS,

61 QUEEN STREET,

MELBOURNE.

0 C. EXAMINATION.

TUITION BY CORRESPONDENCE.

MR. THOMAS BOARDMAN, First-class Honor-man of the Denominational School Board, Prepares Teachers for the Certificate Examination by Correspondence. Terms moderate.

Address—

45 PRINCES STREET, CARLTON

TAMES CLEZY, M. A. ,

O    MELBOURNE.

CLASSICAL & SIIAKSPEARE SCHOLAR (1869)

PREPARES CANDIDATES For Matriculation (pass or honours), and for the subsequent Degree Examinations of the University.

Course of Lessons by Correspondence in Latin Grammar, Translation, and Composition.

Terms on application personally or by letter. 5 GORE STREET, FITZROY.

TO HEAD MASTERS, SECRETARIES OF BOARDS OF ADVICE, AND OTHERS.

ALEX. M‘KINLEY & CO.,

Having made considerable additions to their stock of Bookwork and Jobbing Type, are prepared to execute orders in every description of

GENERAL PRINTING.

All orders entrusted to them will bo printed in a satisfactory manner.

Alex. M'Kinley & Co., Printers, 61 Quben-st.

NOW READY.

ILTON    PARSED.

Pricb 2s.

By J. J- BURSTON,

(Author of “State School Arithmetic”).

Also Ready, the Fourth Edition of the

STATE SCHOOL    ARITHMETIC.

By

JOHN J. BURSTON,

North Sandhurst State School.

Printed and Published by Alex. M'Kinley & Co., 61 Queenstreet, Melbourne, under the auspices of the Victorian Teachers Union.

AUS YH AL, ASX AN

®©©fflO0OO

AND LITERARY REVIEW.

Yol. III., No. 32.


FEBRUARY, 1882.

(    Subscription

\ Yearly, Gs. Gel.; Half-yearly, 3s. Cd

BLACKIE

Comprehensive

& SON’S

School Series.

the Numbers-

9.    Sentences.—-Two lines on each

Page.

10.    Plain and Ornamental Let

tering.

11.    Exercise Book.—Wide Ruling

with Margin.

11£. Home Exercise Book—Same as No. 11, but 8vo size. Price Id.

12.    Exercise Book. — Ruled in

Squares.

12jL Home Exercise Book.—Same Ruling as No. 12, hut 8vo size Price Id.

13.    Exercise Book.—Ruled for

Book-keeping.

14.    Essay Book—Ruled for Com

position, &c.

15.    Exercise Book for Begin

ners. Ruled for Small Text. X. Copy-Book Protector and Blotter. Keeping the Books Clean. One Penny.


Adopted by the London, Liverpool, Manchester and other School Boards, and by the National Board of Education in Ireland.

VERE FOSTER’S WRITING COPY-BOOKS

The efforts of Publishers to provide teachers with the means of j roper!y training their pupils in the Art of Writing deserve the highest commendation ; and no names stand higher in this depai t/nent of scholastic /cork than Vere Foster and the Messrs. Blackie.”—Educational News.

Opinions of H.M. Inspectors in Education Blue Book Reports.

Mr. M'Callum i (I H.M. Inspector',says— (

More progress is made by Vere Foster’s than by any other ' method which has come under my notice.”

Mr. Breicer, ( (( H.M. Inspector,says— (

With books like Vere Foster’s there is no excuse for the slovenly no-style of writing too often presented to me.”

Mr. H arburton, ( <( H.M. Inspect:/-,} says— (

I wish that the use of the excellent copy-books such as Vere Foster’s wras begun earlier and more persisted in.”

Mr. Newell, i « II.M. Inspector,says— (

I know of no series by means of which children can be so quickly taught to write with freedom and legibility.”

Rev. J. Lomax, ( t< II.M. Inspector,says— (.

The introduction of Vere Foster’s copy-books in some of my schools has been attended with marked success.”

Superior Edition

, 2d. each number. Popular Edition, Id. each number.

Contents of

1.    Strokes, Easy Letters, Short

Words.

1£. Long Letters, Short Words, Figures.

2.    Long Letters, Short Words,

Figures.

2£. Words of Four, Five, or Six Letters.

3.    Capitals, Short Words, Fig

ures.

3£. Sentences of Short Words.

4.    Sentences. Mostly composed of

Short Words.

4£. Select Quotations from Shakespeare.

5.    6. Sentences.—Maxims, Morals,

and Precepts.

o£. Sentences, in Writing of Three Sizes.

6£. Sentences, in writing of Two Sizes.

7.    Sentences, and Christian

Names.

8.    Sentences.—One Line on each

Page.

THE COMPREHENSIVE READERS.

Each Book is Illustrated in a highly instructive and artistic manner.

Primer I.—32 pp.,..................paper cover, l£d.; cloth cover, 2£d.

Primer II.—48 pp.,.................. ,,    2d.;    „    3d.

Primer Complete—80 pp.,............................. „    4d.

Header I.--64 pp., (Abridged Edition,) ............. „    4d.

Reader. I.—9G pp.,....................................... cloth hoards, 6d.

Reader II.—128 pp.,...................................... „    8d.

Reader HI.—196 pp., .................................... „    Is.

Reader IV.—288 pp., .................................... „    Is,    0d.

Reader V.—320 pp.,....................................... „    2s.

Reader VI.—384 pp.,...................................... „    2s.    Gd.

“ As specimens of good reading books, they cannot be excelled. The exercises are carefully arranged so ¡is to suit the meanest capacity, and at the same time have a tendency to make useful impressions on the minds of young scholars.”—Educational Guide.

ADDITIONAL READING BOOKS.

The Newspaper Reader, Selections from the Leading Journals of the Nineteenth Century on events of the day. By Henry Findlatcr Bussey, and T. Wilson Reid. Foolscap 8vo, cloth boards, 288 pp. price 2s.

The British Biographical Reader, Sketches of prominent Men by Eminent Authors. With Introductory Notes by the Editor. Illustrated by numerous authentic Portraits. Foolscap 8vo, cloth boards 288 pp., price2s.

The Shakespeare Reader, being Extracts from the Plays of Shakespeare, specially selected as fulfilling Article 28 and Schedule IV. of the Education Code. With Introductory Paragraphs and Notes Grammatical, Historical and Explanatory. Foolscap 8vo, 100 pp. cloth, Is.

Myths and Legends op Ancient Greece and Rome. A Hand hook of Greek and Roman Mythology, for Schools and private Students. By E. M. Berens. illustrated from Antique Sculptures, Foolscap 8vo, 330 pp., cloth 3s.

Poetical Reader, for the use of Elementary Schools in England and Scotland. Foolscap 8vo, 224 pp., cloth Is. Gd.

London, Past and Present. A Reading-book for Elementary Schools. With numerous authentic Illustrations. Foolscap 8vo, 288 pp., cloth 2s.

The Scottish Historical Reader; being Extracts from Eminent Writers Descriptive of Events in Scottish History, with Introductory notes. This volume will supply a fuller account of leading events in Scottish history than can be embodied usually in School histories, and being composed chiefly of extracts from the works of approved writers will form excellent reading lessons. It is adapted more especially for the requirements of the Fourth Standard.

HOME LESSON BOOKS.

The object of this series is to place in the hands of the Pupil a certain amount ot' Standard work, which will enable him to grasp the leading facts of English History, Geography, and Grammar, and at the same time lighten the work of the Teacher. The questions in Arithmetic are mixed in order to lest the pupil’s knowledge.

Further Particulars and Catalogues from All Booksellers,

Or their Representative, GEO. STILLIE, care of Messrs. COWAE & Co.




*Actual attendance on day when return is made.

This is so far satisfactory, as it shows that while the first effect of compulsion, by bringing in children of irregular habits, was to drag down the average figure, the percentage of attendance gradually improved as these began to taste the sweets of learning, and became more orderly and regular. It has, however, been almost stationary since 1877, but we can only regard 80' per cent, as a disease which must not be allowed to become chronic.


Rfo ol Ucparfmcnf.

LONDON SCHOOL BOAKD.

{Concluded from Page 83).

SUPPLY OP TEACHERS.

The stamp of masters and mistresses which we require is not too common One obvious way of diminishing the cost and improving the efficiency, of our schools is by increasing the supply of trained teachers. The board has no training colleges of its own ; and not only is the number of these institutions limited, but, in many of the denominational colleges, inducements are held out to the students to enter only schools of that denomination. In one case at least an undertaking to this effect is required as a condition precedent to their entering the college. The percentage of accommodation in undenominational colleges is only 13’7 of the whole, whereas the proportion of children in Board schools, as compared with those in voluntary schools, is much larger than this. The School Boards of the country do not, therefore, enjoy free trade in this matter, and their area of selection is somewhat restricted. Nearly 40 per cent, of the certified teachers of the country do not have the advantage of college training at all. Cannot somethirg be done to bring the supply more nearly up to the demand by inducing Parliament to increase the number of training colleges in proportion to the vast extension of public elementary schools? I am aware that the Education Department in their recentre-port express an opinion that the training colleges are sufficient for the needs of the country ; but, on the other hand, it appears that out of

4,000 candidates for admission, 3,000 passed the qualifying examination, and only 1,500 could be admitted. Why should the remaining 1,500 be hindered in following the career they desire in the way that will be most advantageous to us and to them ? To provide room for them is a measure that would be in every way a gain to the country,

LOCAL MANAGERS,

It cannot be pretended that we have in all cases secured the best type of head teachers for our schools. The nomination in the first instance rests with the local managers. Not unfrequently they have some deserving young teacher who has been working under their eyes in one of the schools under their charge, and whom they very naturally desire to promote. I fear that in some instances this causes them to overlook the superior qualifications and experience of candidates from the outside. One of our inspectors writes this year :—“ Many very good outside teachers, I am told, decline to enter a competition which they believe is only nominal.” I hope that this is not often the case, but still the fact remains that the candidates of most distinction, and possessing all the necessary qualifications, are sometimes passed over for some teacher of inferior ability and experience. I do not, in making these remarks, desire to reflect at all on our local managers, whose co-operation we value highly, and who supply the element of local control and individual interest which it is impossible for most of us to give to the 300 schools under our charge, but rather to show that we cannot dispense entirely with our veto on appointments—a veto which is, however, seldom used.

SCHOOLS BUILT UPON THE ADULT TEACHER SYSTEM.

We are watching with interest the comparative results of schools built upon the pupil teacher system and those upon the adult assistant teacher plan. The great majority of our schools are arranged for the former but a few have been designed for the newer system, which, many think, will be the schools of the future. It would, I think, be misleading to tabulate results at this early stage.

SCHOLARSHIPS.

The remarkable success of the winner of one of our School Board scholarships has excited renewed attention 'to this very interesting subject. W. E. Barker, then a pupil of eleven years of age in the Portmau Chapel, Church of England, school, won in 1873 the Mortimer Memorial scholarship—the first presented to the Board, After a distinguished career at the City of London school, he obtained an Entrance scholarship at Trinity College, Cambridge, and he has this year taken the highest place in the first part of the classical tripos. As he has not yet completed his academical career or, I hope, his roll of honours, I do not wish to comment upon his success further than to point to the great value, from a public point of view, of these scholarships. Formerly, children born in a humble station who showed marked genius, had only the chance liberality of powerful patrons to rely upon, but the scholorships which have been given to us by generous donors are, with one or two exceptions, open to all the children in the public elementary schools of London, and though they are at. present few in number, any boy or girl of real ability has an opportunity by their means of passing to one of the publicfschools, and, in case of boys, proceeding thence to the university, and afterwards of performing high service to the State in the profession for which he may be best qualified. Let us hope that wealthy and generous corporations and individuals will come forward in larger numbers to extend our power of usefulness in this direction. There are also many endowments and charities in the city and elsewhere, of which it is not too much to say that they are at present worse than wasted, and which ought to be, and before long no doubt will be,¡applied to this or some allied purpose. We have now forty-nine of these scholarships, twenty-nine for boys and twenty for girls. Most of them are tenable for four years. Thus there are on an average about twelve competed for in each year, Out of the whole number of winners thirty have hailed from Board and nineteen from voluntary schools. Mr. Barker’s success does not stand alone. Of the next two scholars in seniority, one js about to go into residence at St. John’s College, Cambridge,‘.and another at St. John’s College, Oxford —each of them having won a foundation scholarship. Three others have passed the Matriculation Examination of the London University, two of them in the Honours Division, and the other high in the First Class. The remainder are for the most part still either at the city of London School, or some other school of high standing, the masters of which seud us. from time to time, separate reports on each scholar. Some difficulty has been experienced from the fact that the scholars are expected to proceed to one of the higher schools of London, where, in the case of boys, Latin and Greek, and in the case of girls, French, is an important part of the curriculum—subjects in which children from elementary schools cannot be expected to be proficient. Arrangements have, therefore, been this year made for special instruction in Latin and French to the scholars, in the few weeks that intervene, before they enter the high schools, so that they may start fairly on their new career.

THRIFT.    _

The recent circular of the Education Department on the subject of thrift is addressed to members of School Boards among others, and calls our attention to the great encouragement of saving and the valuable habits promoted among young children by the establishment of penny banks in connection with elementary schools. Several years ago the Board passed a resolution in favour of the establishment of such banks, and 21 of them were started through the agency of the National Penny Bank. It is worthy of consideration whether the number might not be considerably increased.

COMPULSORY BYE-LAWS.

The attendance both at Board and voluntary schools is supervised, and the compulsory law enforced where necessary, by the Bye-laws Committee, The average attendance at Board schools is now 203,334, and at voluntary schools 178,518, During the year it has increased at Board schools by 10,222, while in voluntary schools it has diminished by 2,384. The net increase in the whole of the efficient schools is 7 838. This is not to be accounted for by any greater stringency on the part of our compulsory staff. Indeed, owing to circumstances to which I shall presently refer, we have been considerably hampered during the past few months in the duty of compelling the attendance of the children of unwilling and careless parents. It is probably due to the increased willingness of parents to avail themselves of the advantages afforded them, and to the fact that new schools have been opened where they were much needed. Our staff of visitors and their superintendents, is selected with extreme care, and the tact and discretion with which, with rare exceptions, they perform their difficult duties ought to be acknowledged by us.

PERCENTAGE OF ATTENDANCE.

I believe that except in those districts where the deficiency of school places is still large, there are not now a great number of children whose names are not on the roll of some school, but we cannot shut our eyes to the fact, that out of every five who are thus on the roll, only four attend school on any given day. This is, perhaps, the greatest blot to be found in our work, and the fact that we do not stand alone in this experience will not make us relax our efforts in an earnest endeavour to find a remedy. The following figures show how we stand in the percentage of average attendance to number on the roll compared with jether large towns:—London, SO' 4 ; Liverpool, S238 ; Manchester, 71'5 ; Birmingham, 73- ; Leeds, 70’ ; Sheffield, 73 5 ; and Bristol, 65'. I also append a table which indicates the progress we have made in Board schools in this matter since we commenced work :—

Percentage of average attendance on average number on the roll in Board schools.

1874    ...... 71'5

1875    ...... 74'3

1876    ...... 78'3

1877    ...... 80-2

SUMMARY JURISDICTION ACT.

The Summary Jurisdiction Act, which some one has irreverently called the “ Dilatory Jurisdiction Act,” passed in 1879, has now been in operation for a year and a half. The objections expressed by us to this Act, as far as it affected the working of our compulsory bye-laws, have been amply confirmed by subsequent experience, and I believe I am correct in saying that this Board is practically of one mind in condemning it._ The superintendents of visitors are also unanimous and most emphatic in their condemnation of the new system and in their desire of a return to the former one, under which a short term of imprisonment ensued if the fine was unpaid. For this procedure the alternative of a distress warrant for non-payment is now substituted—really afar harsher punishment, and one that falls with more severity on the wife and children than on the responsible head of the family ; a punishment, moreover, which, owing to the delays inseparable from the system, is lacking in that element of promptness which is so necessary to impress the improvident parents with whom we frequently have to deal. The effect of the delay and uncertainty attending this cumbrous machinery is that in a very large proportion of cases where fines are imposed the defendants leave the court without paying them, trusting that their case may be lost sight of, or that in the weeks which must elapse they may shift their quarters and elude the vigilance of our officers ; or perhaps preferringtbe heavier punishment in the future to the small but immediate sacrifice. They are not of those who would “ rather bear those ills they have than fly to others that they know not of.” A short time ago I had occasion to adjudicate upon a batch of School Board cases in Essex. Out of twenty

on that occasion deserves full recognition by ns, and the behaviour of the boys, the whole of whom were removed from the ship, furnished a proof of the excellent discipline maintained. We have good evidence that the great majority of the boys from the ship aud from the Brentwood school—ninety-two per cent, according to the inspector’s report of last year—turn out well, aud become orderly and useful citizens, instead of leading lives of crime, which would have been their inevitable fate if they had not been removed for a time from the surroundings. The letters received from many of them after leaving show that they retain a grateful recollection of their school life, and something very like affection for the officers.

JUVENILE CRIME.    _

The result of a wide application of the Industrial Schools Acts^in London is shown in the steady reduction of juvenile crimes since 1870 ; the number of commitments in that year having been, for bovs, 8,619, aud for girls, 1,379; while for last year the numbers were 4,786 and 798 respectively. The fact is that what was formerly done h cally and partially by voluntary means, is now done completely and over the whole area of London, and it is difficult for a child of school age to associate with thieves or bad characters, even for a few weeks, without his case coming under the notice of one or other of our visitors, or of the industrial schools officers.

THE TRUANT SCHOOL.

The Truant School at Upton House is, in fact, a penal school for children over whom their parents have lost control. The term of detention is short—on an average three months—the discipline sharper and the life less agreeable than in our other industrial schools; drill, for instance, being substituted for a portion of the ordinary play hours. The object of sending boys to this school being to cure them of persistent truanting, and to prevent them from sinking into worse courses, it is satisfactory to find that this end is attained. A record is kept of the average attendance of the boys after undergoing their term of detention, and from this it appears that, up to the Midsummer recess, they make an average of 92’ per cent., i.e., higher than is usual among the best class of children. Nor is it to be supposed that the deterrent effect is confined to those who have endured the punishment. The conditions of life in Upton House are well known, and arc the subject of wholesome dread. The school accommodates sixty boys, and the number of applications for admission always exceeds the vacancies. It is therefore proposed to build an additional school of this character for 100 boys, and land has been secured at Willosdon for this purpose.* PARENTS’ CONTRIBUTIONS.

I regret to have to refer again, in this department of our work, to the injurious effect of the Summary Jurisdiction Act, 1879. Last year the Government Inspector of reformatory and industrial schools made the following prophecy :—“ It is not impossible, in fact, I think it very probable that the clauses of the Act which deal with the recovery of penalties, &c., may seriously interfere with the collection of parental contributions by rendering the recovery more difficult.” This year, speaking from experience, he says :—“ The abject poverty of the class from which the children are generally drawn, and the difficulties under the Summary Jurisdiction Act of enforcing the collection, contribute to keep down the amount by which the Treasury benefits,” There is no one connected with the administration of the Industrials School Act who has not felt the danger of weakening in parents the sense of responsibility for their children, and of putting a premium on neglect and disobedience of the law by relieving the careless of the support of their children. Previously to 1879, Parliament endeavoured to guard against this danger by providing that the parent should contribute a weekly payment towards the maintenance of his child. True that even at that time the law was far from effectual, from the difficulty of enforcing the payments, but, at least, a serious attempt was made to collect them, and a considerable sum was collected. But the new law makes the process far more difficult by constituting these contributions ordered to be paid by the parents a “ civil debt”—i.e., it throws on the Treasury the burden of proof that the parent has the means, and wilfully neglects lo pay. Of course, it is very difficult to prove that the class whose children find their way into industrial schools are able to pay. They are often improvident persons in receipt of good wages, whose lives alternate between extravagance and starvation. As long as the law provided a term of imprisonment in default they had a strong inducement to keep up the payment. The law was good for them as well as for the community ; but now, in the absence of any power of enforcing the contribution, this wholesome check is likely to be weakened. The change was, no doubt, intended to be a merciful one, but it was perfectly gratuitous, as there is the best authority for the fact that the procedure prior to 1879 “seldom, if ever, resulted in undue pressure on the parent.” EDUCATIONAL ENDOWMENTS.

The Educational Endowments Committee examines all schemes proposed by the Charity Commissioners for the future regulation of educational charities in London, and continues to watch the measures affecting this question which are introduced into Parliament. The Bill for the regulation of the City Parochial charities, introduced by Mr, James Bryce, would have secured a large amount of support from both sides of the

$ The need for this is illustrator! by the following case

William R. was ordered in November, 1879, to be sent to a truant school. Unfortunately, in this case, as in others, the boy was, in consequence of a want of vacancies in the truant school, about the streets for many months, misleading also a younger brother. When a vacancy at Upton House occurred , the boy had procured a situation which he has since lost, in consequence of carelessness or misconduct. In the meanwhile a summons had to be taken outforthe younger brother James, aged ni"C ; but, owing to the example and temptation of the brother William, it had had no permanent effect in improving his attendance. Could the elder boy have been sent, as directed, to the truant school, “there is no doubt,” the superintendent says, “that both boys would have been at school during spring and summer.”


defendants fined, eighteen left the court without payment, and this ■ experience is I believe universal. As an instance of the disproportion of j the ultimate punishmeut, I may mention that in one recent case a fine of Is. and 2s. costs remaining unpaid, a distress warrant was at length ¡ resorted to, and resulted in costs to the amount of £1 11s. 6d. falling on j the defendant. It is true that there is a clause in the Act under which, when a magistrate is satisfied that the defendant has “ no goods,” or that it would be more injurious to his family to have them distrained upon than suffer imprisonment, he may inflict the latter penalty ; but as a rule, in London, at least, the magistrates are unwilling to make use of this provision. In those courts where this is .not so, we return, in such cases, to the system that prevailed before the Act was passed, but only after a long and troublesome process. The result is that there arc at the present moment 2,701 fines unpaid in London, that in recovering the remainder there is great loss of time owing to the double hearing before the magistrates, and the additional notices that have to be served, and that the respect which ought to be paid to the law is seriously weakened. Compare this with the state of things which existed under the Small Penalties Act, which ten years’ experience enables us to say worked smoothly and without friction. Under that system 41,581 fines were imposed, and in only a fractional percentage of cases did the defendants suffer the term of imprisonment. I appeal to members of the Board who are brought into contact with the parents to confirm me, when I say, that at that time, we carried public opinion with us in the administration of the law, even among the lower classes. It is, of course, easy to raise an outcry against “ the cruelties of the School Board” in imprisoning parents, and attempts at this sort of agitation have frequently been made, but they have met with no response from the classes chiefly affected ; as a matter of fact, under the old form of procedure, the alternative of imprisonment was nominal, because the fine was, with very rare exceptions, paid without question. I am not ashamed to say that we have been afraid to enforce this Act except to a limited extent, and we have applied for distress warrants, during the last ten months, in only 673 cases. I need hardly add that our fears have not been for the personal unpopularity which we should incur, but for the effect of an adverse public opinion aroused against the Acts which we have to administer. There appears to be now no alternative but either to apply for distress warrants wholesale, or to allow our compulsory powers to become a dead letter. We have endeavoured to make the best of the new law as it stands, but, at the same time, we have not hesitated to lay before the Home Office our objections to its provisions, and our reasons for thinking that we ought not to be exposed to the odium of enforcing it; and we have asked, hitherto without success, for an amending clause exempting offences under the Education Act from its operation. We have been told by some that we ought to rely more upon persuasion and less upon compulsion, but I need not remind members of the Board that we have always used, and shall continue to use, persuasion, and that we only resort to further measures when all other means are exhausted. During the year 11,786 summonses have been issued, but 81,209 notices have been issued to attend the B meetings, at which parents are personally warned, and this is without reckoning the still larger numbers of preliminary notices A, and the daily and hourly visits, warnings and expostulations of our visitors. These constitute the machinery of our persuasion, but as long as the law of compulsion exists we must be armed with powers to enforce it, when it is persistently disobeyed. To prove that these powers are used with consideration, I may cite the words of Mr. Mundella, who lately said in the House of Commons : “ He believed that the London Board exercised their compulsory powers with great discretion, and made few mistakes.”

BELGRAVE JUDGMENT,

The recent decision of five judges of the High Court, known as the Belgrave judgment, has an important bearing on the question of arrears of fees. It seems to establish as law that where a child is sent to school without the fee he may be legally refused admission, and the parent prosecuted, I do not, however, think it likely that the Board will take this course.

FREE SCHOOLS.

On the 7th of July the Board passed a resolution instructing the school management committee to consider the advisability of establishing a limited number of free schools in certain districts of the Metropolis.

As this is still in the hands of the Committee, I simply note the fact as a new departure that may have an important bearing on the future policy of the Board.

INDUSTRIAL SCHOOLS COMMITTEE.

We have now under the management of our Industrial School Committee three industrial schools established by the Board, and 840 placesre-served for the use of the Board in schools under voluntary management. There are at the present moment, 3,078 children in industrial schools who have been sent there at our instance. The school at Brentwood accommodates 100, and there are 100 boys in it. The Government inspector, describing the result of an unexpected visit to the school, reports it “ in fair order.” The ship “ Shaftesbury,” which till lately was reckoned to accommodate 350 has now been certified for 500—the full complement which she is capable of receiving—and it is expected that this number will be made up in a few months. As many members of the Board have lately visited the ship on the occasion of the prize-giving, when Lord Shaftesbury, after whom she was named, gave away the prizes, I need not enlarge on the admirable order of the ship or the discipline and appearance of the boys. The inspector, who has since paid his annual visit, says :—“ The ship has gone on steadily increasing in efficiency (throughout the year.” In January an exceptionally heavy storm broke the ship from her moorings, and for a time she was in a very critical condition. The energy and presence of mind of the officers

1877-8.

1878-9.

1879-80,

1880-81.

1881-2.

5\S

5.15

5 T9

6-2S

6T5


House, but it was not difficult for powerful persons interested in the maintenance of the present wasteful and mischievous state of things to get this Bill, which was a private one, talked out. A settlement in this direction cannot, however, be long delayed. A scheme for the re-organisation of Christ’s Hospital, which is now under consideration of the Commissioners, is worthy of note from the value of the endowment, the antiquity of its foundation, and the important principles involved, Here again reformers have to encounter the resistance of vested interests, but it is to be hoped that the scheme will ultimately provide for the reception of a considerable number, at least of the most promising children, from the elementary schools into the secondary schools.

FINANCE.

In the earlier years of the existence of the Board the expenditure for school Board purposes naturally increased rapidly as the deficiency of accommodation was provided for, and the work was organised. During the last five years the rate in the pound has been much more consuant, as the following table shows :—■

There are many reasons for thinking that the present scale will not be greatly exceeded in the future. The rateable value of London will doubtless continue to increase, and improvements in the grant may also be anticipated. Lord Spencer said, in the House of Lords, in explaining his new proposals: “The best schools will earn a little more than hitherto, and the worst a little less;’' and our schools are certainly in the former category. When they have been longer established, and more completely organised, we may look forward to improving our position still further. We may also hope by rigid examination of details in committee—the best, if not the only, way in which economists can really serve the Board and the ratepayers by effecting reductions—may result in our finding some items of expenditure which may be limited or dispensed with. On the whole, I see no reason to expect nmch addition to the burdens which the ratepayers have to bear at present, unless, indeed, the voluntary schools find themselves unable to carry on their work, a result which is certainly not desired by us, and of which there is little present indication. The gross annual expenditure last year (ended 25th March) was £1,235,360 9s, 3d.—a very large sum in itself; but the rate in the pound does not compare unfavourably with that in the other large cities in the country. Thus, while Manchester and Liverpool are exceptionally low, the rate in Birmingham, Bradford, Leeds, and Sheffield exceeds that of London. I have now touched upon all the points to which I think it necessary to call your attention on the present occasion. You will observe that I have not attempted a complete review of all the departments of our work, but have preferred to confine myself to those matters which have been most prominently before us during the past twelve months.

Before resuming his seat the Chairman took the opportunity of thanking the Board for the cordial and generous support which the members had given him as their chairman, and which he trusted they would continue to vouchsafe to him. (Applause.)—School Board Chronicle.

MR. SHARPE’S REPORT ON TRAINING COLLEGES.

Mr. Sharpe, in his recent Report (Blue-book for 1880-81), touches upon the present over-supply of teachers ; but ho thinks the evil is only temporary. Outgoing students, we are told, are now willing to take such low salaries to commence with that they can hardly be underbid by untrained teachers, while the probable increase in the ratio of assistant teachers to pupil teachers will increase the’demand for trained men. There is not much comfort to be derived from the first of these considerations, for untrained teachers can, of course, lower their terms, and where is the competition to end ? But the increased employment of assistant teachers would relieve the overstocked market in two ways : by increasing the demand, and by cutting off the supply. There is always a danger, however, of rushing into extremes. If we are to have trained masters we must have candidates for admission into our colleges, and it has hitherto been found that we can only depend with certainty upon pupil teachers. It may be said that we might reduce the number very considerably and still leave sufficient to fill our colleges ; but this is hardly satisfactory. There ought to be a number large enough to necessitate a selection. If colleges compete with each other for every available candidate, the qualifications of the candidates need not be very high. Even now the papers worked at the scholarship examination are far from being satisfactory. Mr. Sharpe gives extracts from the reports of the various examiners, and the general complaint is that thr memory only is cultivated. In history and geography, questions concerning facts are far better answered than those requiring intelligence. In Euclid the book work was generally fairly done, but the deductions were seldom attempted. Mr. Sharpe tells us that those candidates who succeeded in gaining admission into training institutions presented upon the whole, “a good average of physical and intellectual power.” But ho regrets that many of the unsuccessful candidates were not struck off the rolls in the middle of their apprenticeship-. “ before they were stereotyped for life as inferior members of the teaching profession.”

It is satisfactory to learn that, judging by last Christmas examination, the students show an increasing degree of intelligence in their answers ; but still there are complaints of want of thought and common sense. Mr, Temple is very severe, not only upon students, but upon

teachers generally. Speaking of a question in the School Management iaper he says :    “ It seems to me to be a very good question, but it

requires some small amount of thinking independent of routine, and that is beyond the capacity of ninety-nine training college students or certificated teachers in a hundred. Mr. Claughton, reporting upon the histoiy papers, declares that “more marks were lost by the want of a little thought and common-sense than by ignorance of the facts of history.” Mr. Morgan Owen gives a list of absurd and amusing blunders which he met with in the papers sent to him. A student observed that “ the annual cleaning should do for the schoolroom what the great fire did for London.”

Mr. Sharpe’s remarks upon the teaching of history and geography in training colleges seem to us to be sound and sensible. He thinks the mere outline of facts in both these subjects should be committed to memory during pupil-teachersbip, and that a philosophical treatment of the facts of history and geography, involving not more than one lecture a week in each subject, should he attempted at college. Moreover, this should only last during the first year of training. In the second year’s course, the students should, with few exceptions, be able to lay aside one or both of these studies, and give increased attention to languages and science But, though we agree with Mr. Sharpe's opinions so far we cannot endorse his remarks on the best means of teaching geography in schools, “Why should not the child’s geographical studies begin, we are asked, “where the man’s geographical readings end, with tales of travels and adventure, and of the campaigns of our many wars?” With just as much reason it might be asked, Why not allow the young pianist to begin with popular* tunes instead of troubling him with scales? In order to understand tra\ el, or to form any clear notion of a campaign one must have some knowledge of geography. Mr. Sharpe seems to forget that there is no royal road to learning geography any more than there is to learning anything else. The truth is that he does not seem to value the subject. It was probably never made interesting to him as a child, and lie thinks, therefore, that, per sc, it never can be interesting. — The Schoolmaster.    "


SYLLABUS OP PARSING AND ANALYSIS.

We have just received from Mr. George Robertson, publisher, a copy of the “Syllabus of Parsing and Analysis prescribed by the University of Melbourne for the Pass Examination at Matriculation, with explanation, directions, and examples.” By II. Venables, B.A., Exeter Coll., Oxford, and University of Melbourne.

Considering the important nature of the change made in the style of the examination in English at the University, Mr. Venables has done well for students by issuing this Syllabus. In the present issue we can only give the “ Scheme of Tenses ” the “ Scheme of Parsing,” and the “ Scheme of Analysis,” which are as follows :—

SCHEME OF TENSES.

_fart OF THE VERB “ TO WRITE,” ACTIVE VOICE.


Present

Indicative.

Conjunctive.

I write I am writing I do write

(If) I write I be writing I do write

Perfect

I have written I have been writing

I have written I have been writing

Past

I wrote I was writing I did write

I wrote I were writing I did write

Past Perfect

I had written I had been writing

I had written I had been writing

Future

I shall write I shall he writing

I should write I should be writing

Future Perfect

I shall have written I shall have been writing

I should have written I should have been writing

Present

Infinitive.

Participles.

To write To be writing

Writing

Perfect

To have written To have been writing

Having written Having been writing


A complete scheme of the verb can be readily constructed from the above, but “ shall ” and “ should” of the first person become “will” and “ would” in the second and third persons.

In parsing and analysis every tense phrase is to be dealt with as though it were a single word.

No verbs may be treated as auxiliaries except the auxiliaries of tense mentioned in the scheme : whenever auxiliaries are irsed as notional verbs, they must be treated as verbs of incomplete predication, or as ordinary transitive verbs.


"Write the sentences to be analysed.


Words-


Classification.


Inflexion, &c.


Syntax.


2.


(Priu.


Simple ) ) Cookl. Priu.

asforn8”“-.


( Subst.....

■! Adj.—Attrof c e (Adv.—Mod. ** ( Subi. of « * ) Obj. of * *

N.B —A sentence is not to be divided into co-ordinates solely because it contains a compound subject, predicate or object.


(Co-ord. Sabord. ;


os. g* m


Noun


C Proper ) Common 1 Material ( Abstract


{Feminine \ C Singular ( Nominative | I Neuter f [ Plural ' j Possessive J (.Common J    (Objective )


(1. Nominative of Address

2.    Absolute

3.    Subject of (Che verb) * *

4.    Governed by (the verb) * *

5.    Object governed by (the preposition) | # # suppressed

G. In apposition with * *

7. Complementary to (the copulative verb) * *

(8. Used as an adjective (give syntax accordingly)


! Principal Sentences by A.

Co-ordinate Ciause3 by ABC, &c. Subordinate .. I Sentences ) ,    .    -

Co-ord. Sabord. { Clauses { b> a b c’


( Qualitative 'l Adjective < Quantitative > (_ Distinctive )


(Positive (degree) 1 J Comparative (degree) 1 ”1 Superlative (degree) ( (Nil (If not compared) J


(1. Qualifying ")

2. Limiting    >• * *

I 3. Distinguishing 3

■{ 4. Complementary to (the copulative verb) * *

5. Used as a substantive (give syntax accordingly)

I 6. Used as a substantive pronoun (give syntax accordingly) (7. Used as an adverb (give syntax accordingly)


Write the conjunction or conjunctive adverb used, or say “contained in the relative Si * * ”


Pronoun


fPersonal Personal andDemonstrative !

I Interrogative f I Relative (Indefinite J


(1st per. \ \ 2nd per. / 3rd per. >

r s

ipto- 'lobjvej


If used as a substantive give syntax Cits antecedent * * accordingly, and in the case of < its antecedent * * relatives add, referring to    (    suppressed

If used as an adjective (give syntax accordingly)


If a subordinate sentence or clause occur underline it, and distinguish it by the letter under which it is to be analysed.


Tense phrases are to be taken as single words (see the scheme of Verb).


See No. 5.


Verb


("Transitive (. Intransitive


f Place Time

Cause and effect

-g ( Simple    Manner

> of-j Degree, in-


1st per 2nd per 3rd per. Nil


r Sing. 1 Pill. ( Nil.


(Pres.

Perf.

Past


1


(Ind.

Imp.


Past ! j    ! ( Act. )

P.Perf.MConjve.j>{PassJ

(Part. J


Future


(F. Perf.J


r i. 23

< f-

I 6.

(7-


Agreeing with its subject * *

Attributive infinitive to * *

Adverbial infinitive to * *

Complementary infinitive to * *

Infinitive absolute

Used as a substantive (give syntax accordingly) Used as an adjective (give syntax accordingly)


mf

2.“ 3 S-? 2 < X O


Here enter adverbial expre sions consisting of sub-i „ stantives governed in the objective case by prepositions! ~ -expressed or suppressed.    j e’«

9.    gA


-«) Con- , < Injunctive y


chiding measure and certainty

Affirmation and Negation


(Positive (degree) ~) J Comparative (degree) I | Superlative (degree) (Nil (if not compared)J


Modifying *


February, 1882.    AUSTRALASIAN SCHOOLMASTER


See No. 5.


>2


•Ö.O b


Ja“


See No. 5.

10.


a £.


K O


Preposition


* * : g


Nil


Governing * * * and indicating the relation ((Snbst.) * between it and the    ■< (Att.) * *

(. (Action) * #


a    C Simple    1

§ Co-ordinative < Adversative g    (.Illative

B    C Time

Cause

Purpose    |

(.Condition    J


S Subordinative ofk j


Nil


Connecting * with *


jo


* * 1 Interjection


Nil


Nil


FULL NOTES OF A LESSON ON THE “ BIRD OF PARADISE”

By Miss S. Nickels, P.T., No. 1459.

Apparatus Required,—Black-board, chalk, duster, lead, satin, map of world to show habitat of bird, Piece of metal to show metallic lustre. Picture of bird, and if possible some plumes of feathers to illustrate lesson.

Introduction.—There was once brought into England from the East Indies the skins and plumage of very singular birds ; these birds seemed to have no feet, and the natives of those places whence they were obtained called them in their language, “ God’s Birds”—hence the name, “ Bird of Paradise.”

Description.—A beautiful bird, size of a pigeon. Body coffee-brown, gold and white plumes under each wing. Head, yellow. Legs, lead colour. Bill, bluish grey (in some kinds black or yellow).

Kinds.—Five. Common, Red, King,'Superb, Six-shafted or Golden bird,

HABITS.—Resemble jay or jackdaw, restless and prying. Hopson ground ; fond of bathing, and of displaying its beautiful feathers.

Where Found.—New Guinea and Arru Islands.

Uses.—Ornaments for head-dresses.

How Obtained.—Shot by natives who hide in trees.


Head.


Matter.


Method.


Heads.


Matter.


Method.


of a bluish-green tint; while from the back of the neck rises a shield, in form like that on the breast, but larger and longer, and of a rich black, tinged with purple and bronze. The feet are yellow, and the bill black. The plumage of the Six-shafted or Golden Bird is black in the shade, but glows (15) i in the sun with bronze and purple. On the throat and breast are broad feathers of a rich golden hue, exhibiting in a bright light blue tints. Back of the head is yellow and green. In front is a white satinlike spot, and from the sides spring six slender feathers like wires, with small oval webs at the extremities (1G). On each side of the bi east rise masses of soft feathers which greatly increase its apparent bulk (17).


Description


Kinds,


A very beautiful bird about the size of a pigeon. The body is of a rich coffee-brown colour. From each side under the wings spring magnificent plumes (1) of long gold and white feathers. These are very much longer than the body, and make the bird appear larger than it really is. At each side the brilliant (2) shining gold colour extends to more than half the length, and gradually fades (3) all round the edge into pure white ; a 1 strip of rich red brown runs through the gold colour for about a quarter of its length. The head is yellow, throat green, legs and feet lead colour, (4) and the bill greyish-blue.


Although the name “ Bird of Paradise” is given to these birds they are not all exactly similar in appearance (5). Some are very magnificent and rare (6). Five kinds :—Common Bird, Red, King Bird, Superb Bird, and the Six-shafted or Golden Bird. The Common Bird of Paradise same as described above. The Red Bird of Paradise is so called because ic has beautiful red wings. The King Bird is the most beautiful of the tribe, hence its name, The feathers of the head are short and velvety (7), and shaded into > rich orange beneath. From the breast downwards the body is like the softest white gloss silk, while across the breast a baud of deep metallic green (8) separates it from the red throat. Above each eye is also a spot of green. The bill is yellow, and the legs are blue. On each side of the breast under the wings are tufts of grey feathers about two inches in length, terminating (9) in a broad band of emerald green (10). These are raised into fans when wings are expanded. In the middle of the tail are two feathers like slender wires, about five inches long, diverging (II) into a double curve of a metallic green colour (a). The plumage of the Superb Bird of Paradise generally appears to be of a black colour, but when the sun shines on it that of the neck shows a rich bronze tinge, while the head is covered with scales of a brilliant green and blue. Over its breast is a shield (12) of somewhat stiff feathers with a rich satiuy gloss (13)


(1) Plumes—show what this means. (2) Brilliant— explain, (3) Explain what is here meant by reference to light dying away, or colour of some flower, bright at one part, and lessening till it seems lost at edge, See. (4) Exhibit a piece of lead, drawing attention to the dead dull colour —a kind of grey.

Briefly recapitulate details of bird, making class repeat salient points. Question upon what has been mentioned.

(o) Dissimilarity may be illustrated by reference to the parrot - tribe. All parrots are not alike in colour, size and beauty. (6) Very grand to look at, but not often to be found, because few only. Some stones are precious because rare. (7) Soft like velvet. (8) Some things have an appearance of metal, that is, shine like metal. We call it metallic lustre or metallic brightness.

(9)    Explain meaning of word ; allude to railway terminus.

(10)    Explain meaning of word. (11) Show meaning, by lines on blackboard. (a) What do you mean by a metallic green colour? (12) Kind of cover — refer to shields of Roman soldiers, and show print of same to lower class. (13) Show meaning by exhibiting satin. (14) Show bronze colour. (N.B.—In giving lesson to junior class only mention one or two characteristics of


Their

Habits.


Where

Found.

Uses.


How

Obtained.


Their habits strongly resemble (18) the jay or jackdaw from their restless, prying disposition. They move on the ground by hopping, and are very fond of bathing, and seem to take a pleasure in keeping their plumage tidy (19). They are also fond of displaying their long plumes, generally in forenoon after a bath. The body then assumes an erect position, the feet clinging very tightly to the perch, otherwise the bird would fall backwards (5) ; the wings are raised fully extended (c), while the whole body shakes, and the lovely ornamental feathers are expanded, their ends hanging over in a most graceful manner. While thus showing themselves to the greatest advantage, they suddenly commence jumping and turning about on the perch in a very excited manner

(20) , at the same time uttering loud cries, much louder than their ordinary notes, which consist of a series of loud but pleasing notes like “ wank, wank, wok.”

New Guinea and Arru Islands

(21) .

Plumes and feathers are used as headdresses by native chiefs as a . mark of rank (22). They are also exported from islands to civilized countries, where they are used as ornaments for ladies’ bonnets, hats, dresses, &c. In cousequence of being so rare they are very valuable and bring a large price.

The natives go in parties to search for the haunts (23) of the Bird of Paradise. When they find them they climb into a large tree frequented by the birds, build a kind of bower over head, and wait till early day. As soon as the birds assemble the natives shoot them with, blunt-headed arrows (24), and after obtaining as many as possible, take out the inside, cut off the feet and legs, and preserve the skins by wrapping in cotton, with some sweet-smelling gums to keep away moths and other insects. Therefore the Europeans first had a very erroneous (25) idea of this bird. As it had no stomach and legs they thought it never alighted on the ground, and that it fed on the air, or the dew from heaven.


each kind, thus : Red bird is remarkable for its red wings. King bird most beautiful of the tribe, breast fans and wiry tail feathers. Superb bird, for the shields on breast and head. Six-shafted bird, for the six slender feathers like wires on each side of head, and tufts of soft feathers on each side of breast.) (15) Seems to be on fire —like fire. (16) Ends furthest from the body. Show on black board. (17) Make the bird appear larger than it really is. Illustrate by wool on sheep or curly haired dog. Quest, on foregoing.

(18) Restless, prying, like the magpie, always looking into something as if curious. (19) Draw moral lesson on cleanly habits. (20) Refer to habits of other familiar birds. (b) Why would body fall backwards, if bird did not bold tightly to perch by claws ? (o) Spread out as wide as possible. Illustrate by arms of body.


(21)    Show on map. Examine on habits and places.

(22)    All uncivilised nations mark people of higher rank by outward show, either by jewellery or dress.


(23) Ask meaning of haunts. (24) Ask why bluntheaded arrows. Because sharp arrows would enter the body, and consequently the blood would spoil feathers


(25) Erroneous— mistaken ideas.

Question.

Now give a short summary of whole lesson. Let class repeat blackboard sketch. Glean board and then examine.


ANSWERS.


shall

follow

shall ( follow ) whose


would


himself

advance

as

ours

can

nothing

but

weep


asleep


GRAMMAR EXERCISE.

BY ROBERT CRAIG, M.A., BRIGHTON.

ANSWERS to questions on parsing in the paper in Grammar set, December, 1881, for a Certificate of Competency :—

Set I.

{Time al'owcd, two hours and a half.)

1, Parse with full syntax the words in italics in the following :—• Valour, religion, friendship, prudence, died At once with him, and all that’s good beside;

And we, death’s refuse, nature’s dregs, confined To loathsome life, alas ! are left behind ;

Where we (so once we used) shall now no more,

To fetch day, press about his chamber door ;

From which he issued with that awful state,

It seemed Mars broke through Janus’ double gate ;

Yet always tempered with an air so mild,

No April suns that e'er so gently smiled ;

No more shall hear that powerful language charm,

Whose force oft spared the labour of his arm ;

No more shall follow where he spent the days In war, in counsel, or in prayer and praise;

Whose meanest acts he would himself advance As ungirt David to the ark did dance.

All, all is gone of ours or his delight In horses fierce, wild deer, or armour bright ;

Francisca fair can nothing now but weep,

Nor with soft notes shall sing his cares asleep.

—Marvel, on the Death of the Lord Protector,

2.    Analyse according to Morell’ssecond scheme the following passage :

But, yet, because all pleasures wax unpleasant If without pause we still possess them present,

And none can right discern the sweets of peace That have not felt war’s irksome bitterness,

And swans seem whiter if swart crows be by;

(For contraries each other best descry),

The All’s architect alternately decreed

That night the day, the day should night succeed.

3.    Give fully the derivation of the following words, the meaning of the component parts, and the language from which each is derived blaughteryard, inquiry, Southampton, polyglot, deprivation, octopus.

4.    Give the general rule for the comparison of adjectives of two syllables. There are three classes of exceptions to this rule ; what are they ? Give examples.

5.    What is a noun sentence ? Give examples of the various places which the noun sentence may take in complex sentences.

6.    Write each of the following sentences in a correct form, and state the reason for each correction :—[a) This account is very different to what I told you. (J) They make such acquirements, that suit them for useful avocations, (c) Neither John nor Robert swim well, {d) Their etymology as well as their use show them to be adjectives.

7.    Give the name and number of the feet in each of following lines :—

Lightly they’ll talk of the spirit that’s gone,

And o’er his cold ashes upbraid him.

He shook the fragment of his blade,

And shouted “Victory 1”

1. Parsing.

all    Noun com., sing., neut., 3rd per. nom, or subject, to “ died,”

understood.

that    Rel. pronoun, referring to its antecedent “ all,” neut., 3rd

per. sing., subject to “is.”

is    Verb irreg., intrans., neut, indie, pres,, 3rd per sing, agree

ing with its subj. that.”

beside Adj. of quality, attributive to “all,” or (but less satisfactorily) a prep., governing “ these things” understood, death’s Noun abstract, sing., neut., 3rd per., possessive case, governed by, and attributive to, “refuse.” dregs Noun common, plural (used in plural only), 1st per., nom. case in apposition with “ we.”

confined Verb reg., trans., passive participle, attributive to “we.” alas    Interjection, expressing sorrow.

where Conjunction, joining the sentences, “ we are left behind,” and “ we no more shall press.”

to fetch Verb reg., trans., act., inf. of purpose, modifying “shall press,” that is, it is used as an adverb to “ shall press.” press    Verb reg., trans., middle voice, infin. mood, dependent on

“ shall.” In reality it is a noun infinitive, forming the object of “ shall,” which once meant “ to owe.”

Mars    Noun proper, sing., 3rd pers., nom., in apposition with “it,”

and completing “seemed.”

broke Verb irreg., intrans., active, perf. participle (for the more usual form broken), attributive to “ Mars.” yet    Conjunction, adversative, joining the sentences, “It seemed

Mars,” and “ he (was) always tempered.” no    Adj. of quantity, limiting “ April suns. ’

April    Adj. of quality, attributive to the noun “suns.”

e’er (ever) Adv. of time, modifying “ smiled,”

charm Verb reg., trans., with obj. “ us” understood, or intrans., if no reference to any object is intended, infin. mood, attributive to “ language,” and forming indirect object or secondary completion to " shall hear.”

Verb, auxiliary to “follow. ”

Verb, reg., trans., governing “ him” understood, infin. mood after “shall.” See “shall press” above.

Verb reg., trans., act., indie., future, 1st per.,plur,, agreeing with its subject “ we,” understood.

Pronoun, relative, referring to its antecedents “ prayer and praise,” neu. gen., plural number, poss. case, governed by, and attributive to, “ acts.”

Verb irreg., trans., indie, mood, past tense, 3rd per. sing., agreeing with its subject “ he.” Wouldis not here a sign of possibility or uncertainty, but indicates that “he” was accustomed to advance or promote the meanest acts of prayer and praise ; it being supposed that a man does often or habitually what he wills, desires, or likes to do. Accordingly, “ would” is not parsed here as an auxiliary indicating the potential mood, but as a principal verb in the indicative mood. Scholars will recall a similar use of the Greek thclein and philein.

Pronoun, compound, personal, 3rd sing., masc,, nom., in apposition with “ he.”

Verb reg., trans., act., infin. mood, used as noun, object, to “ would.”

Conjunction, copulative, continuative, joining the sentences, “he would advance” and “ David did dance.”

Pronoun,personal, standingmot for our only, but for “our delight,” sing., 3rd per., objective case after “ of.”

Verb, irreg., trans., indicative, present, 3rd per. sing., agreeing with its sub. “Francisca.”

Noun abstract, 3rd per. sing., obj. after “can” (meaning originally know, therefore, able to do.)

Conjunction, continuative, equivalent to “ unless” or “ except,” joining the sentences, “ Francisca can nothing” and “ she can weep.”

Verb irreg., intrans., infin. mood, used as a noun in obj. case after “ can” understood. Possibly weep is the infin.' as a noun in objective case after but, used as a prep. ; but this view appears the less satisfactory of the two when we consider the sentences, “She desires nothing but to weep,” “ She likes nothing but to weep.” In these sentences the “to” before the infinitive cannot be omitted, which seems to show that the infinitive is really governed by the verb “desires” or “likes” understood, which require to before the succeeding infinitive, whereas when “can” is used in the sentence preceding but to is not required before the infinitive succeeding “but.” “She can nothing now but weep, i.e., except she can weep.”

Adj. of quality, attributive to “ cares,” and forming indirect object (factitive) to “sing.”

2. Analysis according to Morell’s second scheme :—

Sentences.

Kind of Sentence.

Subject.

Predicate

Object.

Extension of Predicate.

(a) Because all pleasures wax unpleasant.

Adverbial of cause to (70

(Because) all pleasures

wax

unpleasant (nom. completion)

(6) If without pause we still possess them present.

Adverbial of condition to (a)

(If) we

possess

them

(direct)

present

(indirect)

without pause (manner)

(c) And (because) none can right discern the sweets of peace

Adverbial of cause to (h)

(and because) none

can

discern

the

sweets of peace

right (manner)

(d) that have not felt war’s irksome bitterness.

Adjectival to “ none ” in (c)

that

have felt

war’s irksome bitterness

not (negation)

(e) And (because) swans seem whiter

Adverbial of cause to (A)

(and because) swans

seem

whiter

(nom.

comple

tion)

(f) If swart crows be by

Adverbial of condition to (e)

(if) swart crows

be by

{g) For contraries each other best descry

Adverbial of reason to (e) and (/) taken together

(For)

contraries

each

descry

other

best (manner)

(h) The All’s architect alternately decreed.

Principal

The All’s architect

decreed

sentences (0 and

O')

alternately (order of time)

{i) That night (should succeed) the day.

Noun sent, obj, to (70

(that)

night

should

succeed

the day

(J) (That) the day should night succeed.

Do.

the day

should

succeed

t ho night

* See Abbot’s “ How to Parse," p. 110

3,    Derivations. See Morell’s Grammar and Chambers’s Etymological Dictionary.

4,    & 5. See Morell’s Grammar.

6.    (a) “ This account is very different from what I told you.” The verb differ is followed by from, not to ; and, consequently, the derived adjective, different, should likewise be followed by from rather than by to. It must be allowed, however, that different to is making its way into respectable usage.

(b)    “They gain such acquirements as fit them for useful employments.” It is good usage to say “ they make acquisitions,” but not to say, “ they make acquirements.” Such requires«« for its correlative. Suit does not express the intended meaning so well as Jit. Avocation, according to its derivation, means, not a regular vocation or employment, but something that draws a man off from his vocation . Therefore in the sentence submitted above “employment” or “occupation” would be a better word.

(c)    “ Neither John nor Robert swims well.” Grammarians tell us that in such a sentence the verb must be in the singular, the act of swimming being denied of John and Robert separately : “ Neither swims well,” that is, “Neither John swims well, nor Robert swims well.” But, notwithstanding what grammarians lay down, there is a tendency to regard the action in question as denied of both subjects taken together, and to put the verb in the plural. Instances of this may be found iu writers of such repute as Macaulay and Matthew Arnold. The former wrote in one of his letters : “ Neither Mr. Wdberforce nor my uncle Babington come up to him” (not comes) ; and the latter in his critique on Byron says, “Neither Wordsworth nor Leopardi are of the same order as the great poets.”3 Students of French do not need to bs informed that in that; language the correct usage in such cases is to put the verb in tho, plural.

(d)    “ Their etymology, as well as their use, shows (not show) them to be adjectives,” that is, “ their etymology shows it, as well as their use shows it.”

7.    The first two lines are scanned thus :—

Lightly they’ll 1 talk of the I spirit that’s [ gone And o’6r    | his cold fish | -es upbraid | him.

The first line consists of three dactyls and one syllabic of a defective foot. If this foot at the end were completed by two unaccented syllables, the line would be technically described as Dactylic Tetrameter ; as it stands, it is Dactylic Tetrameter Catalectic—Catalectic meaning defective.

The second of the above lines consists of an iambus, followed by two anapaests and one unaccented syllable of an incomplete foot. The line may be described as Anapaestic Tetrameter Catalectic.

The last two lines are scanned thus :—

He shook j the frag 1 -ment of I his blade And shoVit | -ed Vic [ -tor-y j

The first is iambic tetrameter, the second iambic trimeter.

THE SCHOOL AGE QUESTION.

This following is the report of the inspectors of State schools who recently considered the question of the proper minimum and maximum ages of children enrolled in such schools :—

Education Office, February 9, 1882.

Sir,—We have the honour to forward a report on the proceedings of the Conference, of Inspectors, held, by order of the Minister, “to place on record their opinion on the subject of admitting to State schools children under or above the compulsory school ages.”

On the first day of meeting, discussion arose as to whether the Conference should take into consideration the financial aspects of the question, or regard it only from an educational point of view. In support of the latter course, it was urged that inspectors, as such, did not possess the requisite data to enable them to judge accurately of the financial effects of any change in the regulations as to admission of children, and that the expense of working the Education Act was one of general policy, on which the members of the Conference, as professional advisers, were hardly in a position to pronounce an opinion. It was urged, on the other hand, that the secretary’s memorandum seemed to indicate the Minister’s desire that all the bearings of the question should be considered.

It was resolved, by a majority of three, to consider from the more restricted point of view the subject of excluding children under or over the compulsory school ages.

Certain resolutions wore then adopted iu respect to the age of admission to state schools, the question being considered purely in its educational aspects. All these resolutions were, however, rescinded at the next meeting of the conference, it having been ascertained from the secretary of the department that the Minister wished the conference to consider all the bearings of the question before expressing an opinion on the subject.

The first resolution subsequently adopted was the following :—

“ It is the opinion of this conference that the educational benefits derived by children under five years of age do not warrant expenditure at the same rate as that incurred for older children.”

This was carried unanimously.

Iu the discussion of this resolution, it was pointed out that expense might be lessened in large schools without refusing admittance to young children, and without impairing the ellicicncy of their instruction, by allowing fewer adult teachers to infants than to the older children, and by using older pupils as paid monitresses. At present, the same number of assistants and pupil teachers is allotted to 100 infants as to 100 older children, and the most highly paid assistant frequently has charge of the infant department.

The conference next considered the question whether any alteration in the age of enrolment of children under six was desirable. On this subject great diversity of opinion was expressed, some inspectors urging that no change was necessary or advisable : others considering that to confine children under five for four or more hours in a school was injurious to their health ; while others again thought that the age of admission might with advantage be raised to four years or even to four years and a half.

The following resolution was finally adopted by 10 votes to 7 :—

“ Iu the opinion of this conference, it is desirable that no child under the age of four years and a half should be enrolled iu state schools.”

A similar resolution, in which four was fixed as the minimum age, had been carried by 12 to 5 ; while a motion that the minimum age should be five years was lost, seven voting for it and nine against it.

The argument in favour of continuing the present practice of enrolling children at three years of age were briefly (1) That this system has prevailed here as well as in England for many years without objection ;

(2) that in private schools the same proportion of young children attend as in state schools ; (3) that the parents are the best judges as to whether their very young children are better off at school or at home ; (4) that the exclusion of young children would lead to irregularity in the attendance of the elder ones, who would sometimes^ be kept at home to mind the little ones; (5) that youug children are better iu large airy schoolrooms than in ill-ventilated cottages and in ill-drained back yards and lanes ; (6) that owing to the free system of education, few, if any, good and inexpensive private elementary schools exist to which young children could be sent if excluded from state schools ; (7) that children can hardly begin too young to learn habits of obedience and order, and that a little schooling forms a pleasant and desirable break in the day’s play ; (8) that though the infant instruction now given in many state schools may be ill adapted for very youug children, this defect could be remedied by alteration in the course of instruction, and by special training for infant teachers.

In favour of raising tlio age of admission to school, the following arguments were adduced : —

(1) That requiring very young children to sit or stand in one position, or to maintain attention for even half an hour is unreasonable, and even cruel ; (2) that such children are better off in the open air, even playing in gutters, than if kept for two or more hours breathing vitiated air in a room not infrequently hot and crowded ; (3) that the bulk of the lessons of the infant classes are monited for very 'young children, and are insufferably irksome to them ; (4) that the necessity for instructing them interferes with the work of the higher classes, and frequently monopolises the attention of the senior female assistant, who is thereby prevented from exercising supervision over the elder girls and giving tone to the upper part of the school ; (5) that the argument as to the exclusion of children between three and four and a-half, or five, causing irregularity in the attendance of the elder sisters, would apply with still greater force iu the case of children under three years of age, who, notwithstanding, have never been enrolled, and that the State undertakes to educate children, not to provide nurseries for babies, or to assist mothers in the management of their households.

The Conference, having come to a decision as to the age under which children should not be enrolled, proceeded to pass a resolution that children over that age (four and a-half) and under six, should not be excluded from school. This was carried by eleven to six, the minority being in favour of the minimum age being fixed at five. Subsequently, a resolution was unanimously adopted that it is not desirable to exclude from State schools children between the ages of five and six.

The Conference then, with only one dissentient voice, adopted a resolution affirming the undesirability of altering the existing practice in regard to the admission of children over 15.

An unanimous vote was also given that it was undesirable to exclude any children already enrolled in State schools.

The Conference proceeded to consider what effects to teachers might follow the approval by the Minister of the resolutions which had been adopted. It seemed evident that if the present regulations as to the payment of teachers were continued, while the average attendance at the schools was reduced by excluding children under four and a-half, the incomes of many teachers would suffer. From returns prepared for the Conference, it appeared that the exclusion of children under four and a-half years would diminish the average attendance by about 3 per cent On the assumption that this decrease would be uniform, the Conference endeavoured to ascertain in how many cases the allotments of schools would be reduced, The number of such allotments was found to be 310, the majority being schools of the two lowest grades. Under the existing regulations, therefore, it was estimated that over 300 head teachers would probably suffer a loss of income ranging from ¿glO to £30, the greater number losing nearly the latter amount, a sum which, in many cases, would amount to 20 per cent, of their whole income ; and where a teacher’s wife was also workmistress, the loss in the joint income of the two might be as much as 34 per cent.

The conference were unanimously of opinion that it would be unjust to cause individuals to suffer to so serious an exteut owing to a change in public policy, and various methods were mentioned by which so great an evil might be averted, such as lowering the limits of allotments, retaining the salaries of reduced teachers as at present until equivalent positions could be offered them. Finally it was decided to simply pass a resolution pointing out the injurious effects on teachers’ incomes of any raising of the miuimum age of admission to school, leaving it to the Minister and the administrative branch of the department to devise measures to prevent such losses.

The conference therefore adopted the following resolution nem con :—

“ While the foregoing recommendations Nos. 2 and 3 are, in an educational point of view, considered desirable by a majority of the inspectors, it must be pointed out that they cannot be effected under the present regulations without a serious reduction of the incomes of many teachers— teachers with the smallest salaries being the most affected,

The conference would deprecate the lowering of teachers’ incomes, which, under the present regulations, would follow the adoption of the recommendations made, as such reductions have a tendency to check a cheerful and efficient performance of duty, and to diminish confidence in the administration of the act.”

The conference then discussed the question of the age of admission to school, as it might affect the state. It was not, however, thought necessary to pass any special resolution on the subject from this point of view, such aspects having been considered when the resolutions already mentioned were passed.

We have the honour to be, sir, your obedient servants,

J. Brodribb,

Charles C. Topp,

To the Secretary Education Department. S. James Swindley.

THE EARL OF CHESTERFIELD’S SCHEME OF EDUCATION.

By F. Howard, A.K.C.

Few men, perhaps, have been more misunderstood and more misrepresented than the Earl of Chesterfield. Hardly anyone has a good word to say for him, and his name is seldom or ever now mentioned but for reprehension or reproach. The unfavourable view which has generally been taken of him, aud which he by no means wholly deserved, is mainly to be traced to the savage epigram of the morose moralist, Dr. Johnson, which has had so great an effect in creating a prejudice against a man who was, at an}' rate, no worse than most of the powdered beaux and fine gentlemen of the period in which he lived, and who would certainly not have suffered by coutrast with such intimate and unprincipled associates of the great lexicographer himself, as Topham Beauclerk and the poet Savage. His letters, which have been so indiscriminately condemned, it is only fair to remember, were strictly private communications written under the confidence of secrecy ; and their unauthorized publication, revealing, as it did, all the little artifices suggested by parental solicitude, was a malicious and treacherous act—a circumstance in itself, one would think, sufficient to disarm any hostile or overstrained criticism. Considering the conditions under which these letters were written, and the general profligacy of the times, the wonder is that this correspondence contains so much which still merits our attention. First impressions, it is said, are generally the truest; and the world never found much harm in these compositions, until they were denounced by an authority that can scarcely be considered impartial.

It is not. however, my intention, in the present paper, to enter at length into the disputed question of the Earl of Chesterfield’s character, or to attempt to give anything like a general account of his life, or of the various capacities in which he distinguished himself, whether as statesman, diplomatist, orator, writer, wit, or man of fashion ; but rather to confine my remarks to those points which more immediately interest us as teachers, by drawing attention to the celebrated scheme which he devised for the education of his son, to whom he was deeply attached. Strongly impressed with the narrowness and illiberality of an English education, such as he himself had received, and such as he was familiar with in the schools and colleges of his day, he determined to adopt a plan of his own which he hoped would be more likely to secure the special objects he had in view. Accordingly, at the outset, he takes upon himself the task of instruction, writing letters to his son, then under seven years of age, living with his mother apart from the paternal roof—letters plain, simple and sensible, containing the first elements of knowledge suitable to the capacities of a child of his age. After a few years spent in this manner under the personal care of his father and a private tutor, the lad is despatched to Westminster school, where he passes through the usual course of instruction. During this time the correspondence is duly maintained, the letters, mostly addressed from Dublin Castle, where the Earl was then residing in the capacity of Lord Lieutenant of Ireland, being filled with excellent advice and earnest entreaties to bis son to make the most of the valuable time he was then spending at school. Finding that the task of keeping up this regular correspondence somewhat interfered with the performance of his official duties as Secretary of State, on his return to London his lordship (who was never in very robust health) resigned all public employment, that he might devote himself entirely to the task which he had set before himself as the one object of his life.

On leaving Westminster School, young Stanhope was sent abroad, first to Germany to make himself master of the language, study under a professor the Laws of Nations, and the Jus 'publicum Imperii, and by mixing in the society of persons of high rank acquire the finished manners of a well-bred gentleman. After a residence of a year or two in Germany, he passed on to Italy, pursuing much the same course, remaining a short time at Venice, Rome, and Naples, but prolonging his stay in Turin for a twelvemonth. According to the directions of his father, he spent his time in giving his mornings to attendance at academical lectures and to reading with his tutor, his afternoons to the exercises of riding, dancing, and fencing, and his evenings to the polished society to which his father could easily procure his admittance.

Having now become proficient in German and Italian, he next removed to Paris,where he remained for a couple of years, attending the courses of study at the University, but giving the greater part of his time to fashionable society, in order that he might acquire “ those manners, that address, that tournnre of the beau monde, ” which was thought so indis-pensible to qualify him for his future destination. His education now beiug considered complete, young Stanhope was at length released from attendance at lectures and the restraint of tutors, and fairly started in life on his own account, to undertake the more important duties that afterwards awaited him.

Such, in the main, was Chesterfield’s scheme. From the outset he seems to have formed a clear idea of what he wanted, and then to have set to work in the most straightforward and direct manner, regardless of the extent to which he might be departing from established practice. Intending his son for a public career, as a senator and diplomatist, he sent him to complete his education, not at the English Universities, of which he had no high opinion, and where his son would infallibly have been brought up a theologian or a mathematician, or have confined his attention mostly to Latin aud Greek ; buttothe Continent, in order that be might become a good liuguist, obtain some acquaintance wtth the political state of Europe, and acquire an intimate knowledge of mankind. “ The end which I propose by your education,” he writes, “ and which (if you please) I shall certainly attain, is to unite in you all the knowledge of a scholar with the manners of a courtier, and to join, what is seldom joined in any of my countrymen, Books aud the World.” The reverse of Rousseau, who thought all men should be educationally cast in the same mould, Chesterfield aimed at a special preparation for a particular profession. We recognise here an idea which may almost be considered the germ of what we now call “Technical Education,” or at least that principle which is every day receiving a fuller acknowledgment on all sides, that education to a considerable extent should have a direct reference to the practical duties of life.

It is greatly to be regretted that a scheme elaborated with so much care and thought should not have proved more successful than it did. In spite of all the pains bestowed upon him, Mr. Stanhope must, on the whole, be pronounced a failure, aud that not from any inherent defect of his own ; he was no genius, and geniuses are not to be manufactured even under the most favourable conditions. The truth is, the Earl made the mistake, so common to men of real abilities, of thinking that what they can do themselves, anybody else can do, if they like. “ I am very sure,” he says, “ that any man of common understanding may, by proper culture, care, attention, aud labour, make himself whatever he pleases, except a good poet,”—a very favourite notion with many people, but one to which we should no more think of according our assent, than that by dint of culture an oak tree could be transformed from a cabbage. Undoubtedly, nothing is done without perseverance ; but perseverance is frequently an endowment or gift, and comes natural to persons possessed of strong and vigorous minds. What we call idleness, carelessness, inattention, and the like, proceeds not unfrequeutly from some mental or constitutional defect which we may not be able exactly to define or analyse, but which certainly cannot always be entirely overcome or controlled. Education can bring out what is in the individual, but it cannot go beyond that, or create a faculty in which he is deficient.

I pass on, now, to notice in detail some of Chesterfield’s opinions on education. I need not dwell much upon the question of Foreign Languages, since young Stanhope was exceptionally favoured in this respect, his mother being French, and three parts of his daily prattle when a child being in that language. Other foreign languages he learnt by the best of all methods—residence abroad. It may be mentioned, however, as showing the backward state of education in those days, that when Mr. Stanhope entered the House of Commons, he was said to bo almost the only man in England who could understand and speak German.

Nor did Chesterfield carry his fondness for foreign tongues to the entire neglect of his own. He was very emphatic on the subject of English, and that, too, at a time when its systematic study was a thing unheard of in the great seats of learning in this country. He was careful to impress upon his son the necessity for a close study of the best English authors, such as Swift, Addison, Dryden. Pope, Atterbnry, and Boling-broke. For the earlier writers he had no particular fondness. Shakespeare he thought a rude, irregular sort of genius, greatly in want of culture and finish ; but for the polished writers of his own time—Pope and Addison, and the rest—he had a great admiration ; and aristocrat as he was, felt himself highly honoured with their acquaintance and company. His love of ornament, which he sometimes carried to an excess, shows itself conspicuously in his conception of literary style. His rule, whether for speaking or writing may be thus summarised—Take care of the style, the matter will take care of itself. “The same matter,” he says, “occurs equally to everybody of common sense, upon the same question ; the dressing it well is what excites the attention and admiration of the audience.” “Were I either to speak or write to the public, I should prefer moderate matter, adorned with all the beauties and elegance of: style, to the strongest matter in the world, ill-worded and ill-delivered.” “Your chief business,” he says, in another place, “is to shine, not to weigh. Weight without lustre is lead.” For the cultivation of an elegant style, he recommended translations of choice passages from various languages into English, with repeated corrections until the ear was satisfied as well as the understanding.

Geography he thought an important subject. By geography he understood an acquaintance with maps and localities—the various countries, especially of Europe, their provinces, towns, trade, finance, commerce, their naval and military armaments, police, government, and general political condition. Many persons I know do not attach a very high educational value to this kind of knowledge. It does not, they say,

greatly cultivate the mind ; and it is a sort of information that may be easily got at by an occasional glance at an atlas or a clip into a gazetteer. But Chesterfield defends it on the ground of its extreme utility. It is a portable kind of knowledge, he says, which a man can carry about him, and which saves him the trouble of consulting maps and books. And, certainly, in the present day, when all parts of the world are by various contrivances brought into immediate communication with each other, an acquaintance with local geography is one of the most useful things which a young man going out into the world can acquire.— The Educational 'Times.    (To be continued.)

HINTS TO TEACHERS FROM WISE HEADS.

No 4.

BY MARIA AND L. EDGEWORTH.

To avoid, ill education, all unneessary severity, and all dangerous indulgence, we must form just ideas of the nature, and use of rewards and punishments. Let us begin with considering the nature of punishment, since it is best to get the most disagreeable part of our business done the first,

Several benevolent and enlightened authors* have endeavoured to explain the use of penal laws, and to correct the ideas which formerly prevailed concerning public justice. Punishment is no longer considered, except by the ignorant and sanguinary, as vengance from the injured, or expiation from the guilty. We now distinctly understand, that the greatest possible happiness of the whole society must be the ultimate object of all just legislation ; that the partial evil of punishment is consequently to be tolerated by the wise and humane legislator, only so far as it is proved to be necessary for the general good. When a crime has been committed, it cannot be undone by all the art, or all the power of man ; by vengeance the most sanguinary, or remorse the most painful. The past is irrevocable ; all that remains is to provide for the future. It would be absurd, after an offence has already been committed, to increase the sum of misery in the world by inflicting pain upon the offender, unless that pain were afterwards to be productive of happiness to society, either by preventing the criminal from repeating his offence, or by deterring others from similar enormities. With this double view of restraining individuals, by the lecodection of past sufferings, from future crimes, and of teaching others by public examples to expect, and to fear, certain evils as the necessary consequences of certain actions hurtful to society, all wise laws are framed, and all just punishments are inflicted. It is only by the conviction that certain punishments are essential to the general security and happiness that a person of humanity can, or ought, to fortify his mind against the natural feelings of compassion. These feelings are the most pain-fid and the most difficult to resist, when, as it sometimes^unavoidably hajipens, public justice requires the total sacrifice of the happiness, liberty, or perhaps the life, of a fellow-creature, whose ignorance precluded’him from virtue, and whose neglected or depraved education prepared him, by inevitable degrees, for vice and all its miseries. How exquisitely painful must be the feelings of a humane judge in pronouncing sentence upon such a devoted being. But the law permits of no refined metaphysical disquisitions; it would be vain to plead the necessitarian’s doctrine of an unavoidable connection between the past and the future in all human actions ; the same necessity compels the punishment that compelled the crime ; nor could, nor ought, the most eloquent advocate in a court of justice to obtain a criminal’s acquittal by enterin« into a minute history of the errors of his education.

It is the business of education to prevent crimes, and to prevent all those habitual propensities which necessarily lead to their commission. The legislator can consider only the large interests of society ; the preceptor’s view is fixed upon the individual interests of his pupil. Fortunately both must ultimately agree. To secure for his pupil the greatest possible quantity of happiness, taking in the whole of life, must be the wish of the preceptor ; this includes every thing. We immediately perceive the connection between that happiness and obedience to all the laws on which the prosperity of society depends. We yet farther perceive that the probability of our pupil’s yielding not only an implicit, but an habitual, rational, voluntary, happy obedience to such laws must arise from the connection which he believes, and feels that there exists, between his social duties and his social happiness. How to induce this important belief is the question.    ,

It is obvious that we cannot explain to the comprehension of a child of three or four years old all the truths of morality ; nor can we demonstrate to him the justice of punishments by showing him that we give present pain to ensure future advantage. But though we cannot demonstrate to the child that we are just, we may satisfy ourselves upon this subject and we may conduct ourselves during his non-age of understanding with the scrupulous integrity of a guardian. Before we can govern by0reason, wo can, by associating pain or pleasure with certain actions, «fve habits, and these habits will be either beneficial or hurtful to the pupil ; we must, if they be hurtful habits, conquer them by fresh punishments, and thus we make the helpless child suffer for our negligence and mistakes. Formerly in Scotland there existed a law which obliged every farrier who. through ignorance or drunkenness, pricked a horse’s foot in shoeing him to deposit the price of the horse till he was sound, to furnish the owner with another, and in case the horse could not be cured the farrier was doomed to idemnify the injured owner. At the same rate of punishment, what idcmnification should be demanded from

a careless or ignorant preceptor ? _________

‘Ecccaiii), Voltaire, Blackstoue, &c.

When a young child puts his finger too near the fire he burns himself ; the pain immediately follows the action, and they become associated together in the child’s memory ; if he repeat the experiment often, aud constantly with the same result, the association will be so strongly formed that the child will ever afterwards expect these two things to happen together ; whenever he puts his finger into the fire he will expect to feel pain ; he will yet farther, as these things regularly follow one another learn to think one the cause, and the other the effect. He may not have words to express these ideas ; nor can we explain how the belief that events which have happened together will again happen together is by experience induced in the mind. This is a fact which no metaphysicians pretend to dispute, but it^has not yet, that we know of, been accounted for by any. It would be rash to assert that it will not in future be explained ; but, at present, we are totally in the dark upon the subject. It is sufficient for our purpose to observe that this association of facts, or of ideas, affects thejactions of all rational beings, and of many animals w ho are called irrational. Would you teach a horse or a dog to obey you? Do you not associate pleasure or pain with the things you wish that they should practice or avoid ? The impatient and ignorant give infinitely more pain than is necessary to the animals they educate. If the pain, which we would associate with any action do not immediately follow it the child does not understand us ; if several events happen nearly at the same time, it is impossible that a child can at nist distinguish which arc causes and which are effects. Suppose that a mother would teach her little son that he must not put his dirty shoes upon her clean sofa; if she frowns upon him, or speaks to him in an angry tone at the thatjinstant he sets his foot’and shoe upon the sofa, he desists, without knowing that the dirt of his shoes was the motive of his mother s prohibition ; but he has only learned that putting a foot upon the sofa and his mother s frown follow each other; his mother’s frown, from former associations, gives him, perhaps, some pain, or the expectation some pain and consequently, he avoids repeating the action which immediately preceded^the frown. If, a short time afterwards, the little boy, forgetting the frown, accidentally getsupon the sofa without his\shoes, no evil, follows ; but it is not probable: that he can, by [this single experiment discover that]hisJshoes have made all the difference in the two cases. Children are frequently so much puzzled by their confused experience of impunity and punishment that they are quite at a loss how to conduct themselves. Whenever our punishments are not made intelligible, they are cruel; they give pain without producing any future advantage. To make punishment intelligible to children, it must be not only immediately, but repeatedly and uniformly, associated with the actions which we wish them to avoid.

W hen children begin to reason, punishment affects them in a different manner from what it did whilst they were governed like irrational animals, merely by the direct associations of pleasure and pain. They distinguish, in many instances, between coincidence and causation ; they discover that the will of others is frequently the immediate cause of the pain they suffer; they learn by experience that the will is not an unchangeable cause, that it is influenced by circumstances, by passions, by persuasion, by caprice. It must be, however, by slow degrees that they acquire any ideas of justice. They cannot know our views relative to their future happiness; their first ideas of the justice of the punishments we inflict cannot, therefore, be accurate. They regulate these first judgments by the simple idea that our punishments ought to be exactly the same always in the same circumstances ; when they understand words they learn to expect that our words and actions should precisely agree, [that .we 'should keep our promises, and fulfil our threats. They next learn that as they are punished till for voluntary faults, they cannot justly be punished it has been distinctly explained to them what i3 wrong or forbidden, and what is right or permitted, I he words right or wrong, and permitted ox forbidden are synonymous at first in the apprehensions of children ; and obedience and disobedience are their only ideas of virtue and vice.    “    .


fomspoitbtitcA


MATRICULATION EXAM.—OCT. TERM, 1881 .-ARITHMETIC

PARER.

TO THE EDITOR OP “THE SCHOOLMASTER."

Sir.—Allow me to draw your notice to the following relative to solu* tions to arithmetic which appeared in your issue of December:—

Question 2

HR — tv) + | H-x|x(£xTsD j should be + j f x |x(£ + $) j

No. 8. A tennis-court, &c., at3‘4£s. per square yard, the die. point has been inserted in error ; the working given shows 3’4 = |- of a £. It should be 3s. 4£d. per yard. As the question appears the solution is puzzling to young people.

Till I received the University questions I could not get Mr. S.’s answers to 2 and 8.—Yours, &c.,    George D. Miles.


In the course of the month of November 22 new vessels, with a total tonnage of 31,170, were launched from the Clyde ship building yards, persons were picked rather than elected, and that no chance was given Physiology to Dr, McLauin.


CONTENTS

School Department—

Answers to Correspondents ...

123

London School Board ...

114

Leaders—

Mr-Sharpe’sReport on Train-

Salaries of Teachers ...

123

ing Colleges.........

116

Royal Commission on Edu-

123

Syllabus of Parsing and

cation ... ... ...

Analysis .........

116

Salaries of School Teachers

Full Notes of a Lesson on

under the London Board

124

“ Bird of Paradise” ...

118

Notes of the Month ... ...

124

Grammar Exercise ...

119

Science and Art Gossip ...

126

The School Age Question ...

120

Victorian Education Department—

The Earl of Chesterfield’s

Appointments, &c.......

127

Scheme of Education ...

121

Drill Papers.........

127

Hints to Teachers from Wise Heads .........

122

Drawing Papers ... ...

127

ANSWERS TO CORRESPONDENTS.

«< TYRO.”—“The only weapons he has to fight with are, etc. Has governs which understood, having weapons for its antecedent. To fight with may be either parsed as a prepositional verb depending on weapons, and forming an adjectival infinitive ; or it may betaken as “ with (which) to fight, the preposition governing which understood, and also referring to weapons. The only difficulty of the passage is found in its eliptical construction, and vanishes when the elipses are supplied.

“ J. Thomas. Jun. ”—The papers asked for will be found on“page 127.

“ J. Clezy.”—'Will be inserted next month.

“A Country Teacher. ’—Will be attended to.

“ An Inquirer.”—(1.) Four hours is simply the minimum time specified in the Education Act. (2.) If school opens at 9 a.m., there should be an interval of 10 minutes for the younger children during the morning. (3.) The headteacher has power to compel the attendance of assistant teachers from 9 a.m. to 5 p.m., if the work of the school demands their attendance. But, of course, an appeal may be made to the Department, should an assistant deem the request of the head-master unreasonable. The Department does not think hours too long for a school day,

“It. Sabestin.”—Received, 6?. 6d. to 31/12/81.

Received.—Thos. Brown, Thos. Godden.

A LITERARY TREAT.

A Booh lleplete with Humor,

With Illustrations and Descriptive Letterpress,

Will be published shortly.

ALEX. M'KINLEY & CO., 61 QUEEN STREET.

NOTICES TO CORRESPONDENTS.

Advertisements and other business communications should be addressed to the Publishers. No advertisements will bo inserted without a written order, or prepayment. It is particularly requested that they may be sent early in the month.

Books, music, and school appliances for notice, and all letters containing anything connected with the literary portion of the paper should be addressed To the Editor. Every communication accompanied by the name and address of the sender (as a guarantee of good faith, though not always for publication) will be acknowledged ; but we cannot attend to anonymous letters.

^wstralashin Stljualmasitr.

PUBLISHED EVERY MONTH.

MELBOURNE, FEBRUARY, 1882. Educationists at home are beginning to see that fixity of salary, no less than fixity of position, is essential to the full development of national elementary education. Scholastic work makes a more constant drain upon the nervous system than ordinary employments. In these days of competitive and test examinations the teacher who commands success is the man whose every thought is occupied in the detail work of the school, and in devising means for calling into activity the dormant powers of his pupils. To allow of his mind being thus pre-occupied with school duties, a teacher must be kept free from pecuniary embarrassment, and care with regard to the position he holds. Every instance in which the brain becomes occupied with questions affecting emolument or status, is a loss to the school system. The mental power absorbed in fighting against the existing defects and the threatened changes of the State school system is so much force abstracted from the practical work of elementary instruction. It is the nonrecognition of this fact by the Department of Education in the past that has so greatly retarded the work of elementary education in Victoria. State school teachers have been all along kept in a state of anxiety and uncertainty, either with respect to their incomes or their status. Instead of bavino-a position assigned them which would leave them free to devote the whole of their energies to the practical work of education, they have been kept in a constant state of uneasiness as to their ultimate classification, while the system of school examination and payment by results have taxed all their ingenuity to meet the idiosyncrasy of inspectors and the vagaries of departmental regulations. Every change of Ministry has been a source of uneasiness to the State school teacher, because of the danger of some alteration or other being attempted. We are pleased to notice that the evil effects of this state of things were admitted by the Inspectors at the recent conference meeting. From the report of their proceedings, it will be seen that the Inspectors were opposed to any change being made in the admission of infants which would necessitate a reduction of teachers’ salaries, on the ground that “ such reduction would have a tendency to check that cheerful and efficient performance of duty ” so essential to the successful working of the State school system. This may be taken as an indication of the change which is gradually developing itself in the minds of educationists as to the system of payment adopted for elementary school teachers. A system of payment which, like theexist-ing system of payment by results, in its operation is so lacking in its fixity, as to “check the cheerful and efficient performance of duty,” is totally unsuited to a scheme of national education,and should be amended. We have long contended that state school teachers ought to be paid a fixed salary, the same as any other branch of the civil service. Prejudice, and not reason, has hitherto prevented the introduction of this much needed reform in Victoria. The article which wo reprint from The Schoolmaster will show that the School Management Committee of the London School Board have recommended that Board to adopt the principle of payment by fixed salary, to commence at a minimum amount, and be increased until the maximum salary obtainable has been reached. From calculations made it appears that the change proposed would at first entail an additional cost to the Board, and its adoption has consequently been delayed. It also appears that “ the salaries paid to headteachers would be about the same as before, while the assistant certificated teachers would receive a little less, and ex-pupil teachers a little more.” This, however, is a matter of detail which could be otherwise adjusted.

This month is big with educational matters. The Royal Commission has begun its sittings, and has received from Mr. Gilbert Brown most exhaustive and comprehensive testimony as to the general framework of the Education Department. The enquiry into matters Educational has begun in good earnest. Then we have lately had the once common, but now obsolete, institution of a conference of Inspectors, to discuss projected alterations and possibilities, and it is of this conference we specially desire to speak.

There can be little doubt that the Inspectors were gathered together in order that the Minister might have the advantage of their judgment in advising him whether the educational interests of the country would suffer by excludin'- from free schooling all children over fifteen and under six years of a"e.

From the report of the Conference, it would seenAhat the inspectors aio averse to any such sweeping exclusion as that proposed. Recognising, however, the fact that the State institutions are not designed to be partial nurseries, they aie in favour of discouraging the attendance of very young infants. They, therefore, simply recommend that the ago for enrolment be raised from three to four and a half years, and they strongly oppose any other difficulty being thrown in the way of school attendance. But even the excision of children aged three to four and a half years may cause a lowering of some allotments, and, mindful of the justice of the case, the Conference has recommended that in such cases, the incomes of the teachers affected should not suffer. Thus the action of the inspectors, while considerate of the welfare of public instruction, has shown them to be regardful of the teachers’ personal interests. With a few (happily only a few) teachers, inspectors have been looked upon as persons towards whom they must cherish a natural antagonism ; but this fact should show that the inspection staff are alike the friends of educational progress, and friends to the best interests of the teachers.

SALARIES OF SCHOOL TEACHERS UNDER THE LONDON BOARD.

The question of the re-adjustment of the scale of teachers salaries has occupied the    attention    of the    School Board for

London for some    considerable time.    More    than once it has

been found necessary to re-arrange the terms upon which the teachers serving under the Board have been engaged without a satisfactory solution of the question having been arrived at. At present there are many anomalies existing with respect to the remuneration received by different teachers. The best men do not in all cases receive the highest salaries, the instances of practical injustice    in the    rates of    payment    received by the

various teachers    in the    Board’s    employ    being sufficiently

numerous to render a further revision of the scale desirable. For some considerable time the School Management Committee of the Board has considered the whole subject, and, as a result, has presented to the Board a report which, if adopted, would have the effect of entirely altering the principles upon which salaries have hitherto been arranged. In place of a fixed salary and a share of the Government Grant, the Committee proposed a system of payment by fixed salaries, commencing at a certain minimum amount, to be gradually increased until the maximum obtainable had been reached. The particular salary to be paid to any head teacher was made to depend upon the number of children the school was built to accommodate. Roughly speaking, this would about equal, on the average, one pound per head. Elaborate calculations have been made with respect to the cost to the Board of the teaching staff it employs under the now scheme, compared with the amount at present paid. On the whole, it appears that the salaries paid to head teachers would bo about the same as before, while the assistant certificated teachers would receive a little less, and ex-pupil teachers a trifle more.

We think that most teachers will admit that the substitution of fixed salaries, if unaccompanied by a reduction in the amount paid, for the present system of payment—partly by fixed salary, and partly by a share of the Government Grant—would be beneficial to both schools and teachers. So far, therefore, the scheme in question is likely to meet with their approval. It is not so certain that the amount of salary paid should be made to depend almost wholly on the size of the schools. It is true that teachers of schools, worked under exceptionally difficult circumstances, are to receive, under the new scheme, more than the size of their schools would warrant. It must also be admitted that the teacher of a large school should receive more than the teacher of a small school, regard being had not only to the amount of work thrown upon the teacher—which, in some cases, might be quite as great in one as in the other—but chiefly to the greater burden of responsibility which the management of a large school entails. We are not sure, however, that the master of a school providing accommodation for three hundred and forty boys deserves forty pounds per annum more than the master of a school built for three hundred. As a matter of fact, we arc inclined to believe that the Committee has gone too far in the direction of classifying the schools. Some time since we advocated the adoption of a scale of fixed salaries, advancing gradually from a minimum amount to a maximum. But -we did not propose the classification of the schools under the Board into more than three or four categories. There might be three grades of schools, one including those built for 150 and under in each department; another, those accommodating from 150 to 300 ; and a third, those over 300. Together with such an arrangement, a system of pi'omotion might be established, by which, as a rule, the teachers of the larger schools might be selected from the most successful of those who bad been working

in smaller schools. We should think some such arrangement to be preferable to the plan of the Board, by which the salary to be awarded is in each case settled by the particular circumstances of the case. Theoretically, the latter plan may seem the more suitable, but unless there were very frequent changes of schools (which is objectionable), there would probably be as many cases of hardship as there are at present. The simpler the plan the better will it work ; and while we approve of the general principles upon which the scheme of the Committee has been constructed, we fear that, like its predecessors, it is too elaborate.

The scheme presented to the Board on Wednesday occasioned considerable discussion, with the result of its being referred again to the Committee for reconsideration. It appears that its adoption would occasion a temporary increase of the rates. It has been calculated that an additional outlay of £6,082 monthly would be required. In time the extra amount receivable by the Board from the Government grants would suffice to meet this increased expenditure. A sum of £51,697 in all, would, however, be required to meet the advance made to the teachers, an amount about equivalent to a rate of one-halfpenny in the pound. This consideration, no doubt, weighed with members, and combined with others to make them reluctant to adopt the scheme in its present state. Seeing how long the matter has already been under discussion, and the near approach of the triennial election of the Board, it is quite possible that the question may remain for the next Board to settle.—The Schoolmaster.

liâtes jof % JJlontij.


A new edition of Keith Johnson’s physical and descriptive Geography has recently been published, which, by omitting the historical sketch and the elaborate and expensive maps, has been placed within the reach of pupils as well as of teachers. There are few geographies that contain so much accurate and interesting matter as this one does.

The National Schoolmaster, in an article on St. Paul’s Industrial School, says : — “ Iu 1870, when the Education Act was passed and the first school board elected, it was assumed by their advocates almost as a matter of course that for purity of purpose and proper management the new institution was to take a position by itself out of all comparison with boards of guardians, town corporations, and all other municipal bodies. Some ten years of chequered experience have, however, conclusively demonstrated that school boards are not exempt from the ordinary failings and imperfections that beset bodies of men acting in similar capacities. It then goes on to give the evidence of some pupils of the institution who seem to have been most shamefully treated in many ways;

Mr. R. M. P. Michelsen, a member of one of the school boards, writing to the Argus, complains of the delay which must be caused in executing small repairs to schools while the present system of carrying them out exists. Great expense and waste of time is caused by the necessity of sending some person of authority up from Melbourne to inspect the part which requires repair, even should the repairs be of the slightest nature. He therefore suggests that the boards of advice should have power, subject to the Minister of Education’s approval, to authorise minor repairs to school buildings which require an expenditure of not exceeding £5. This, he says, would not only save the Department a considerable sum annually, but it would also give boards of advice some active, useful, and responsible work to do.

The tender of Mr. R. Young has been accepted for the erection of a brick building for a new State school in Yere-street, Collingwood. The cost of this building will be about £4093.

The State school exhibitions, £35 a year for six years, were gazetted on Friday, 20th ult., by the Minister of Education. There were 64 candidates, and the total possible number of marks was 850. This year out of the 11 successful competitors two were girls and six boys. School No. 260 at Geelong was the most successful, carrying off three exhibitions. From that school Joseph Hewison came first on the list with 663 marks, Sydney J. Grace, sixth-with 549, and Emily C. M’Fadzean ninth with 529. John S. Robertson, from school No. 112, Carlton, was second with 584 marks ; Thomas Gray, No. 1976, Sandhurst, third with 580 ; Arthur Astley, No. 156, Richmond central, fourth with 577; Wm, Ceruty, No. 1409, Williamstown, fifth with 565 ; Arthur Cameron, No. 1106, Yarra Park, Richmond, seventh with 544 ; Angus M. Sween, No. 1117, Briagolong, eighth with 558; Henry J. Winning, No. 1046, Hurdle Creek, tenth with 629, and Elizabeth Whyte, No. 391, central, eleventh with 518.

Mons. A. C. Aucher, B.A., Paris, delivered a lecture in the Assembly Hall, Coliins-street, on February 4th, on “The French language and how to teach it.” He recommended that the chief thing taught should be conversation, and that candidates at matriculation should be examined in conversation. He said that this would do away with the system of cram at present in vogue, and would tend to give pupils a thorough and practical knowledge of the language. The lecture was made exceedingly


interesting, and contained some hints that might well be taken up by tne authorities of the University and schoolmasters generally,

It is stated that the Church of England in the United Kingdom in ten years has more than doubled the accommodation of its elementary schools, and now provides places for as many as one million one hundred and sixty-four thousand children, while the school boards provide places for only one million and eighty-two thousand. At the present time the church of England takes twice as many children as the boards do.

The Argus publishes a list of the ages between which children arc compelled to attend school in the various colonies. It appears that Victoria is the highest between 6 and 15. In New South Wales the ages are 6 and 14 ; in Tasmania, 7 and 14 ; in South Australia, 7 and 13, and in New Zealand, 7 and 13.

The Mendelssohn Quintette Club, which has been giving such interesting entertainments in the neighbouring colonies, gve its first performance in Victoria, in the Opera House, on the 14th instant.

The conference of school inspectors, convened for the purpose of considering the State school ages question, after a considerable amount of discussion, which occupied them during three sittings, came to an understanding, on Monday, the 6th instant, when it was resolved to recommend the Minister of Education to allow all children at present at the schools to remain there, but in future to exclude all children under four and a half years of age. It was also resolved that the Minister be informed that the inspector did not consider it advisable to make any limit as to the maximum age at which pupils should be allowed to attend. Mr. T. Brodribb, presided as chairman at each of the sittings.

The foundation stone of St. John’s church and school, on Bircbgrove Road, Balmain North (Sydney), was laid by the Dean of Sydney, on Saturday, the 4th of February.

Statistics given in the Year Book just published by Mr. H. H. Hayter, government statist, contain some interesting facts connected with Education in Victoria. It appears that out of the 10,572 people who were married in 1880, 221 males were unable to sign their names to the register, and 21G females. This shows a great improvement upon 1S70, when the proportion of those unable to sign was nine per cent., instead of four per cent, as shewn in 1880. Four out of every hundred is a very satisfactory figure as compared with those of the different parts of Great Britain. In Scotland twelve out of every hundred are unable to write ; in England 17 ; in France 24 ; and in Ireland 31. It is expected, however, that the four will soon disappear, and not only will all Victoria be able to write their names, but will write fluently.

The long delayed results of the Matriculation Examination were published on Friday, January 27th. Twenty-five boys from various State schools in the colony figured on the list of those passed, and first of first class in Greek and Latin Honours stands the name of Mr. John May, assistant teacher in the State School, Cambridge Street, Collingwood. By a University regulation, passed in connection with the new scheme, and which takes effect now for the first time, the exhibitioner in any group of honour subjects must be under the age of 21, and Mr. May, being slightly over the regulation age, has to forego the title of exhibitioner, together with the money-prize of twenty-five pounds which it carries. For a young State school teacher, whose daily hours are taken up by his duties, and who has only his evenings for study, to have beaten the picked students of the grammar schools, is indeed an honour to be proud of.

On Wednesday, February 1st., a deputation waited upon the Minister of Education from the Lonsdale and Bourke wards board of advice, to urge that he should not cause St. John’s State school in Lonsdale Street to be closed. They pointed out that it would inconvenience a large number of children in the neighbourhood, some of whom lived fully half-a-mile from the nearest school. Mr. Grant said the rent of the building was £150, and it was a matter for consideration whether it was worth while keeping it open, but he would give the matter further consideration.

On February 1st., a deputation, representing the school board of advice at Darebin, waited upon the Minister of Education, and requested him to close the school, known as Separation school, at Yan Yean, as it was not needed. The minister promised to grant the request. The deputation also requested that a singing master should visit their district, but the Minister informed them that it was the rule only to send singing masters to schools which had an average attendance of more than fifty pupils but that the matter would be inquired into. On the same date a deputation of residents of Frankston, complained of the harsh treatment exercised by the head-master of the State school of that district, and Mr. Grant replied that an inspector would be sent down to inquire into the matter.

A liedertafel has been successfully formed at Sydney, of which His Excellency the Governor has promised to be patron.

At a meeting of the Senate of the Sydney University, held on the first of February, it was resolved to offer the chair of Professor of Natural History to Mr. W. J. Stephens, and that of Anatomy and Physiology to Dr. M'Lauin.

It is notified that Mr. Francis Redding, an officer of the Education Department, who has been for many years private secretary to many successive Ministers of Education, has been appointed assistant secretary to the Education Commission.

A meeting of the Sydney Branch of the Australian Biological Society was held on Monday, 6th February, at which Dr. M'Coy, of the Melbourne was named as President of the Society.

The Church of England Synod of Adelaide has received a telegram from England, stating that it is not intended to appoint any one in the place of Bishop Short, and the Synod is certain of its right of election.

The quarterly meeting of the Victorian Council of Boards of Advice was

held on Thursday evening Mr. J. M. Templeton m the chair. There were present:—A. F. W. Saunders (St. Kilda); A. Serpell (Oakleiglv); W. Bain (Learmonth) ; G. P. Barber (Warrnambool) ; W. Baker (Castlemaine); C. Craike (Corio) ; J. Whitehead (Ballarat); A. B. Clemes (Stawell) ; and J. Ingram (Beechworth). In the absence of Mr. I. Warren Ball, secretary of the council, Mr. W. Rain was appointed secretary pro tern. It was resolved that the Minister of Public Instruction be requested to pass a sum of £50 per annum for the purpose of remunerating some person outside the Department for doing necessary work in connection with this council and future conferences. Mr, Barber urged the introduction of simple Bible-reading into the State schools, and moved that the following questions be submitted to the various Boards throughout the colony :—1. Is your Board iu favour of simple Bible-reading in the schools, on the understanding that there be a conscience clause exempting from attendance children whoso parents or guardians object ?    2. If so, should such Bible-reading take place

before, during, or after the ordinary school hours ? On the motion of Mr. Baker, seconded by Mr. Whitehead, it was resolved “ That in the opinion of this Council four years of age should be the minimum fixed for the admission of children to the State schools.” Mr. Whitehead called attention to the pecuniary loss sustained by teachers in consequence of the small attendance of children on days of public festivity, and a motion that such small attendances should not bo allowed to prejudice teachers’ salaries was carried unanimously. The president (Mr. J. M. Templeton), vice-presidents (Messrs. J. Ingrain, and G. P. Barber), secretary (Mr. I. Warren Ball), and Messrs. Saunders and Baker were appointed a sub-committee to bring the various resolutious passed by the Conference of Boai ds of Advice before the Royal Commission.

The death is announced of Mr. Grenville Murray, a contributor to the periodical literature of London and America. Among his more successful works are Men of the Th ird Republic, Round about France, and The Member for Paris. His latest work was Side Lights of English Society, published last year.

The election of a free day pupil for the Wesley College was held on Thursday. There was only one nomination, viz., that of Thomas Philip Gough, who was accordingly elected.

A telegram has been received at Christchurch, from Melbourne, stating that Mr. A. Fletcher, who occupied the post of superintendent of the British art gallery at the Melbourne Exhibition, will hold a similar position at the Christchurch Exhibition.

Mr. J. Lake, of the Church of England Grammar School, gave some very interesting public readings at the Prahrau Town Hall, on Thursday last. The musical interludes and illustrations were very capably supplied by Madame Boema, Mr. F. Lineker, Mr. Bergin, and M r. II. Curtis (violiu), Mr. J. Summers presiding at the piano. Madame Boema gave Donizetti’s song, “ Non m’ami piu,” with excellent effect. Mr. Bergin sangGatty’s “ True till Death,” and Laud’s “ When Night is Darkest,” and Mi’. Lineker also contributed two songs, with violin obligato by Mr. H. Curtis. Barnett’s trio, “ The Magic-wove Scarf,” was rendered with most pleasing effect by the three vocalists named. The entertainment may be regarded altogether as a successful inauguration of the new movement in which so much interest has been evinced.

At a meeting of the College of Preceptors of England, held in England on the 19th November last, Mr. J, Wilson read a paper on Education, during which he produced a report of Professor Mahaffy, of the Dublin University, saying “ It appeared to me, all over Ireland and England, that the majority of boys, without being overworked, were overwhelmed by the multiplicity of subjects, and instead of increasing their knowledge had truly confused it. Whenever I asked the masters to point me out a brilliant boy, they replied that the race had died out. Is it possible that this arises from any inherent failing of the stock, and not rather from some great blundering in the system of our education. The great majority of thoughtful doctors with whom I conferred agreed that this was due to the constant addition of new subjects, to the cry after English grammar and literature, and French, and German, and natural science, to the subdivision of the wretched boys’ time into two rO ours a week for this and two hours for that, alternate days for this and alternate days for that. In fact to the injurious system of so teaching everything that they can receive intelligently nothing.”

Mr. Samuel Morley, M. P,, has offered to give the handsome sum of £500 towards the foundation of a library for children in connection with the Nottingham Free Library, which at present is open only to persons above fifteen years of age. Mr. Morley thinks that, in very many children, a lively interest iu reading is attained before that age, and therefore some means should be given them for reading. He does not, however, intend that children of all ages shall be allowed into the new library, but he has not as yet stated at what age he would wish to exclude them.

A deputation from the Council of the Boards of Advice waited on the Minister of Education on Monday, to request a grant of £50 a year for the purpose of obtaining clerical assistance, as the Council intended launching out into new and important work. That the request would be granted was promised by the Minister,

“Teacher,” writing to the dry?«, recommends that the expensesof the Education Department should be reduced by allowing the Public Works Department to take in hand the building of schools, thus doing away with the necessity for all the extra iuspectors of works employed by the Education Department. This, he says, would be far more reasonable than reducing the teachers’ salaries.

In the report of the Inspectors of State schools, on the school age question, the following arguments in favour of continuing the present practice of enrolling children at three years of age were briefly (1)

Science anti %xt (gossip.


Several towns in America are now lighted by symmetric towers, built of steel, 250 feet high, with six electric lights each of 6000candle power. The light from these towers illuminates the whole town like moonlight

at a very reasonable cost.

In the principal Australian colonics there were at the end of December, 1880, the following numbers of miles of telegraph lines in the course of construction :—New South Wales, 501; Victoria, 132; South Australia, 515 ; Queensland, 567 ; total, 1715. The number of miles of railway open at the end of 18S0 was as follows :—New South Wales, 849 ; Victoria, 1199; South Australia, 667 ; Queensland, 633 ; Tasmania, 172 ; Western Australia, 90, and New Zealand, 1258. Total 4867.

During the nine months ending on September 30th, 1881, there were 69 failures of railway carriage tires in the United Kingdom.

The report of the Accidents in Mines Commission in England commences by stating that: One of the most important facts to which they would draw attention is the great improvement which, so far as safety is concerned, has taken place during the last 30 years. Whilst the total number of deaths remains almost the same, the number of persons employed. has nearly doubled.

William E. A. Anton communicates the following interesting item to Nature :—“About the age of 17, a young man, a congenial deaf-mute, was twice attacked by fever ; some weeks after recovery he began to hear, and, in process of time, to understand speech. This naturally disposed him to imitate what he heard, and attempt to speak, and he is now understood tolerably well, but what is singular is that he retains the Highland accent just as Highlanders do who are advanced to his age before they begin to learn the English tongue.

The gold, silver, and bronze medals awarded to British artists and exhibitors by the Royal Colonial Commission for the Melbourne Exhibition have been received, and are in course of distribution at the office of the Agent-General for Victoria, 8 Victoria Chambers, Westminster.— Illustrated London News.

An officer of a Fire Brigade in Revel has invented a fire extinguishing powder, which, if thrown on any fire, raises such a smoke as to smother the flames. It was recently tried for the purpose of extinguishing a fire that broke out in a house. All the doors and windows were shut, and a couple of boxes of the powder cast amongst the flames. When the doors were opened dense volumes of smoke came forth, but the flames were gone.

Cast, of Cologne, has invented an apparatus, by the means of vrhich a copy of any object may be traced from its reflection.

The Glasgow Weekly Herald says :—We hear of a cheap and effective sanitary invention which has been christened the deodorising punkah or chemical lung. It is simply a rough towel, stretched and kept saturated with carbolic acid or caustic soda in solution. This waved punkah fashion in a sick room or crowded workroom purifies the air in a very short time.

The Seimen’s electric railway has been a great success.

An Oriental Art Exhibition was opened in Glasgow during the week ending December 24th,, the principle object of which is to illustrate the


That this system has prevailed here as well as in England for many years without objection ; (2) that in private schools the same proportion j of young children attend as in state schools ; (3) that the parents are the best judges as to whether their very young children are better oil at school or fat home ;    (4) that the ex

clusion of young children would lead to irregularity in the attendance of the elder ones, who would sometimes be kept at home to mind the little ones ; (5) that young children are better in large airy schoolrooms than in ill-ventilated cottages and in ill-drained back yards and lanes ; (6) that owing to the free system of education, few, if any, good and inexpensive private elementary schools exist to which young children could be sent if excluded from state schools ; (7) that children can hardly begin too young to learn habits of obedience and order, and that a little schooling forms a pleasant and desirable break in I he day’s play ;

(S) that though the infant instruction now given in many state schools may be ill adapted for very young children, this defect could be remedied by alteration in the course of instruction, and by special training for infant teachers; the report further stated, of arguments against it, (1) That requiring very young children to sit or stand in one position or to maintain attention for even half an hour is unreasonable and even cruel ; (2) that such children are better off in the open air, even playing in gutters, than if kept for two or more hours breathing vitiated air in a room not infrequently hot and crowded ; (3) that the bulk of the lessons of the infant classes are unsuited for very young children, and are insufferably irksome to them ; (4) that the necessity for instructing them interferes with the work of the higher classes, and frequently monopolises the attention of the senior female assistant, who is thereby prevented from exercising supervision over the elder girls and giving tone to the upper part of the school ; (5) that the argument as to the exclusion of children between three and four and a half or five, causing irregularity in the attendance of the elder sisters would apply with still greater force in the case of children under three years of age, who, notwithstanding, have never been enrolled, and that the state undertakes to educate children, not to provide nurseries for babies, or to assist mothers in the management of their households.

The results of the Teachers’ Examination will, most probably, be published during next week.

In Scotland the School Board have commenced prosecuting large numbers of parents for neglecting to send their children to school.

decorative art of Japan and Persia. Amongst some paintings lent by the Marquis of Bute are two curious works by Arab artists, and the birth of Christ by Jean Theudore. The nativity is also represented by an elaborate alto-relievo in pearl, and in the same material there is a large carving of the resurrection, and a model of the Holy Sepulchre. At the opening of the Exhibition, Sir Phillip Cunliffe Owen stated that he took a great interest in the movement for opening a large museum for these exhibits.

At a meeting of the Institution of Engineers and Shipbuilders, held in Glasgow in December. Principal Jamieson, of the college of science aud arts, stated that 1400 or 1500 lamps for electric lighting had been fitted up in different large steamers, all having been brought into use for this purpose within six months, and he supposed, judging from the rapidity with which they were supplanting the old oil lamps, nothing else would soon be used for the lighting of ships. He then described how 117 lamps had been fitted up onboard the Servia, at a cost of about £1000, or eight guineas per lamp. As yet the lamps have only been used in two mines, as the miners are very cautious, and are waiting the result of these two trials.

Science for All gives the following little experiment, which will give an idea of how the sea water is kept so clear, “ Take two glass jars, or two plain tumblers will do, and fill them with water ; dissolve in one of them a few grains of common salt, and add to both some common white china clay, stirring them well up, so that the clay is well diffused through the water, rendering it opaque. Care must be taken not to stir the one without the salt with the rod used for the one with the salt. After stirring put both aside, aud notice how long the clay takes to settle in each jar, and the water becomes clear and transparent. A most remarkable difference will be noticed between the two. The water containing the salt in solution will be quite clear in one or two hours, according to the quantity of salt dissolved, while the pure water will not be clear for twenty-four hours, or even thirty-six hours if the water we use is very pure and soft.” We see from this that the attraction of the salt for the water prevents the latter from holding the particles of clay in suspension.

Mr. Morris, of London, gives the following good advice to those who are commencing the study of art :—“ Hold fast to distinct form in art. Do not think too much of style. Set yourself to get out of you what you think beautiful, and express it as cautiously as you please, but, I repeat, quite distinctly and without vagueness. Always think your design out before commencing it on paper. Do not begin by slobbering and messing about in the hope that something may come of it.”

Very much time and very much ingenuity has recently,year after year, been expended in the attempt to invent machines which will tend to save brain labour in the working out of long and intricate calculations, and the fascination for the production of these machines is something enormous. Far more study has been expended in these attempts than in any other invention, and yet most of those discovered have turned out to be rather failures. The invention of logarithms, and afterwards the preparation of logarithmic tables has given almost incalculable assistance to mathematicians. A calculator has recently been discovered which seems to possess at least some advantages in long and intricate problems, but it is of little use for the shorter ones, as the trouble of preparing it would not make up for the saving. It is too intricate for explanation.

The chief difficulty found in the use of China ink is its tendency to dissolve when washed over with water colour, but a corrective has recently been discovered which consists in using a weak solution of bichromate of potash for the water ordinary employed to mix the ink. This will probable be of use to those who are studying art. _

A new explosive has been discovered by an English engineer residing in Austria, which is intended to supplant gunpowder. The initial velocity is said to be much greater than that of gun-powder. It docs not foul or heat the barrel of the weapon, and the smoke of the discharge is scarcely visible.    ,    .

It is reported that one of the domestic fire engines, called Dicks’ chemical fire engine, has lately been the subject of some experiments. A large pool of tar and wax was extinguished by it in 16 seconds after ignition.

A new compound for artificial building stone has been produced by Messrs. Grimzweig and Hartman. They mix pulverised cork, clay, water, glass solution, and hair together; force the mass into a mould, and dry it by heat. The result is a stone of extreme lightness, not absorbent of moisture, and not subject to decomposition.

The first public exhibition of the torpedo boat destroyer was held on November 14th. A dummy projectile was used that is, one of wood,without any explosive. It passed through the target at 5 feet below the surface of the water, and altogether travelled 600 feet iu about 6 seconds. The French officers were especially interested in the experiment, and though they at first pronounced it an impossibility to operate a gun constructed on such principles and with submerged muzzle successfully, as many engineers have done before them, they were obliged to acknowledge that the theory had proved correct. Astonishment was depicted in every line of their countenances when they saw the projectile rise to the surface beyond the target after having traversed the distance from the muzzle of the gun and through the netting without making even the faintest ripple on the surface.

It was found, says the Glasgow Weekly Herald, by Dr. Bon, of Bres fan, that when young tadpoles were subjected to special kinds of food (in one case vegetable food being given in another mixed vegetable and animal) a large preponderance of females were developed, 95 per cent, of 1440 tadpoles.    ...    .

A curious application of electricity is described in la Lumiere Electrique, It consists in a device to prevent military conscripts practising fraud as to their stature by bending their knees. When the youth stands erect against the measuring post, the hind parts of the knees press on electric contacts, causing two bells to ring; the ringing ceases when there is the least bending. The sliding bar which furnishes the measure has also a contact, which is pressed by the head, whereby a third electric bell is affected. For a correct measurement, three bells should ring simultaneously. This system, the invention of M. Cazala, is now employed in the Spanish army,—Glasgow Weekly Herald.

The ordinary use of chloroform is fatal one time in 2873, that of ether one time in 53,204. It is now said that most of the danger with either of them may be avoided by first putting the patient slightly under the influence of alcohol.

Dictorinn: (Sfotcattmx Ueprtmenf.

APPOINTMENTS.

Eliezer Sparkman, H.T., Warngar, 2174; Thomas Starr, H.T., Youar-ang North, 2301 ; Margaret M. McEvoy, H.T., Oxley Plat, 2347 ; E. G. Fairhall, H.T., Bamawn South, 2154 ; Kate S. Cox, H.T., Bradford, 2153 ; Mary McGuire, H.T., Cape Otway, 2352; Wm. V. Cock, H.T. Drik Drik, 971; Julia P. Thomson, H.T., Yan Yean, 2097 ; Edward C. Morris, H.T., Quambatork, 2443 ; T. Atkinson, H.T., Gardiner, 1604; James R. Stobo, H.T., Devil’s River, Ford’s Creek (half time) 1323 ; Susan Powell, H.T., Tooradin, 1503; Frank Yale, H.T., Melbourne Road, 1740; A. B. Just, H.T., Boosey South, 2209; James Kelly, H.T., Emu Bridge, 2446; Donald McIntyre, H.T., Silver Creek, 2438 ; Charles Reddin, H.T., Allandale, 2420 ; William H. Lane, H.T., Muckleford, 1927 ; A. M. Butler, H.T., Upper Joyce's Creek, 1257 ; Willehmina Mathison, H.T., Strathmer-ton, 2440; Agnes Broadbent, H.T., Darragun, 2430; J. Muir Jeffrey, H.T., Whitfield and Dogamoro (half-time) 2441; JohnH. Potts, H.T., Shepherd’s Creek, 2447 ; King W. Pobjoy, H.T., Moorabool East, 2448 ; Annie Shiress, H.T., Wanalta Creek, 2287 ; William C. Bass, H.T., Rokewood, 531; Annie Wain, H.T., Miner’s Right, 1703; Agnes Findlay, 2nd Asst., Ballan, 1435; Jane M. Downs, 2nd Asst., Rrrshworth, 1057; Isabella Browne, 1st Asst-, Ballarat, 1071; James Thom, II.T., Kilmore, 1568; William T. Jamieson, H.T., Tongala, 1851; C. II. Whitley, H.T., Gun-bower Creek, 1994: Mary A. McMahon, 1st Asst., Guildford, 264; T. H. Robertson, H.T., Hillsborough, 889 ; Helen M. O’Neile, H.T., Lake Goldsmith, 929 ; Sarah O’Reilly, H.T., Moyton, 1335 ; John C. Jennings, 2nd Asst., Golden Point, 1493; Wm. C. Allnutt, H.T., Mt. Pleasant Dam, 1891; Mary Glancy, H.T., Tahara Bridge, 2451; Richard B. Chew, H.T., Car-dinia, 2139: James Clancy, H.T., Tamleugh North, 2453; Charlotte E. Murdoch, 1st Asst., Ironbark, 323; Michael J. Coffey, 2nd Asst., Ironbark, 323; Wm. H-Beck, H.T., Waggarandal North, 2249; Herman Lampe, H.T., Queenstown, 128; William S- Binder, H.T., Victoria Valley, 1209; Paul Shugg, H.T., Greta South, 2452; George Carusen, H.T., Warren-bayne, 1448; James McCann, H.T., Warrion, 1308 ; William Cruickshank, H.T., Lovely Banks, 1497; Annie Rastrick, 1st Asst., Taradale, 614; Christopher R. Smithwick, H.T., Merino, 2115.

The following are the] papers set by the Department for Examination in Drill, at the Examination held December 1881 :—

Morning Paper.

(Time allowed three hours.)

Squad Drill, with Intervals

1.    Write in detail the position of a soldier when standing at attention,

2.    Write, according to “General Rule,” the “Instruction of the Recruit.”

3.    Write an account of “ Recruit and Squad Drill,” omitting reference to sections.

4.    Describe standing at ease by numbers.

5.    How many motions are there in three-quarters “ Left About— Turn”? Describe the motions and movements of the feet.

6.    Give in detail the “First practice” of the “ Extension motions.

7.    Define—(a) An “Alignment,” (5) “Base points,” (c) “Company,” (d) “Front,” (e) “File.”

8.    Describe the “ Right-hand salute, by numbers.”

9.    Accentuate (by signs, - long, (J short), the commands, “Right—

Form,” “Left-about—Form,” “ Half left——Turn,”    “Squad—Halt,”

“ Right-about—Turn.”

10.    Describe in detail the method of “ Dressing a squad with intervals.”

11.    A squad standing atfease is ordered to “ Stand easy” ; what word will bring it to its former position

Marching.

12.    A squad receives the command Right—Turn ; describe the movement, also that of Front—Turn.

13.    How many paces are taken in a minute in “ Quick” and in “Double” time?

14.    A squad stepping out is required to resume the usual length of pace ; what word of command should be used ?

15.    Describe the Step—Back, and the length of pace used.

16.    Give words of command and describe how to change feet.

17.    Give words of command, and explain Right —Close by numbers.

18.    A squad is ordered to close a certain number of paces to the right; describe the movement and the rule laid down for doing this.

Squad Drill in Single Bank.

19.    Describe the “ Formation of a squad in single rank.”

20.    Give words of command in detail, and describe “ Dressing when halted.”

21.    Describe “ Dressing together.”

22.    A squad requires dressing ; how is it done, and what words of command are used ?

23.    A squad marching in quick time is ordered to alter to the double, and vice versa; explain how it is to be done, giving all words of command.

24.    A squad at the “Halt” receives the command “Right—Wheel” explain the movement.

25.    Describe how to change front by “File Formation.”

26.    Explain how a squad will change direction on the march.

Afternoon Paper.

( Time allowed three hours.)

Squad Drill in Two Banks.

27.    Describe the formation of a squad in two ranks.

28.    A squad at the halt requires dressing ; give the word of command and full instructions how it is done.

28. Explain the “ Diagonal march.”

30.    A squad moving to the right in fours is ordered to form to the right-about; describe the movement, giving all words of command.

31.    Give instructions to a squad to form fours deep, and close it on the centre ; also to re-form two deep from either flank.

32.    Dismiss your squad, giving full instructions to do so.

Company Drill.

33.    Describe the “ Formation and telling-off ” of a company.

34.    Explain who “ The officers,” &c., of a company are, also their dut ies.

35.    Describe how a company in column (on the march) will wheel into line, giving full instructions to guides and markers.

36.    Give full instructions to a company in line to break into column to the left, including the duties and positions of guides and markers.

37.    Describe in detail how a company on the march will “ Countermarch,” giving all words of command.

38.    Instruct—(a) A company how to ‘ Change ranks,” (5) Give instructions for it to resume its original front.

39.    w) A company advancing is required to reduce its front by break ing off files ; describe the movement, (h) Give instructions for the file broken off to be brought to the front again.

40.    Tell off a company (without rifles) of 26£ files.

4L How many paces do you allow for the company you have told off ? Show how you calculate it.

42. Post the officers and supernumeraries of a company in columu of sections.

The following is the paper set by the Department for License to teach Drawing at the Examination, held December, 1881 :—

PRACTICAL GEOMETRY.

CUme allowed half-an-hour.)

1.    Construct an equilateral triangle having a given height.

2.    Construct a rhombus having a given base and a given angle.

3.    In a given circle inscribe a regular pentagon.

HEAD TEACHER, country, 30 x 50, results 88‘235, wishes exchange Assistant, Melbourne or Suburbs. Address—“ALB., care of Mr. Roth, Tivoli Place, South Yarra.”_________________________________________,

T pEACHER, Benalla district,'.50 to 75, (strong) would exchange 30 to 50 o 1 assistant town. Vacancy pupil teacher. Good residence. Addiess

Teacher, 157 Queensberry-street, Hotham.___________

•TTTANTED to exchange 30 x 50 school, Western district, for a 20 x 30 \ V school, within 100 miles of Alclhourne, K., Schoolmaster office._

HEAD TEACHER, country, near large railway town, would exchange Allotment 30 to 50, results over 80. Residence. Healthy climate. Address, N.Y.Z., Echuca.    _________

HEAD TEACHER, country school, 20 x 30, residence, Post Office, near proposed Railway Station, would exchange, owing to health, with Assistant or Head Teacher, lower allotment. Address, “ Exchange,” care Burn, bookseller, Geelong. ________________________________________________

HEAD TEACHER, school 30 X 50, desires exchange with assistant any where. Address, J.E.H., Ball in.___    _______

HEAD TEACHER, near Melbourne, would exchango with ’assistant in Melbourne or Suburbs. “Desirous,” care P. Matthews, Esq., 52 Collinstreet.    ______

HEAD TEACHER, good 30 x 50 school, workmistress vacant; healthy seaside town, boating, fishing, bathing; salary, with extras and work-mistress, £210; wishes exchange with 30 x 50 school near Ballarat or Melbourne. Address, “ Pedagogue,” Malmsburv._______

HEAD TEACHER, 20 X 30 school, near Inglewood, desires exchange, As sistant or Head Teacher, Ballarat or Castlemaine districts preferred Income of £137, lower accepted. Address, “ Scholastic,” Kinypaniel, P.O.

HEAD TEACHER, country school, allotment 30 x 50, result® 80 desires exchange with another. Address. C.D., Schoolmaster office._

HEAD TEACHER, allotment 30 x 50, workmistress’ position vacant, desires exchange. Lower allotment accepted if other requirements suitable. Geelong or Melbourne district preferred. Address, “ Dominie,” G P.O., Melbourne.

HEAD TEACHER, Gippsland, near Lakes, a’lotment 20 x 30, good chance for 30 x 50, percentage 8P691, three roomed residence, postoffice, desires exchange. Address, “Alpha,” Bairnsdnlc.__________

FIRST ASSISTANT, in Western District, wishes to exchange with any assistant in a large town, allotment 150 to 175. Address, “Adorn,” Schoolmaster Office.

rPATE’S PARCELS POST EXPRESS


L


FIXED PRICE.

NO EXTRAS

N

0

T

RO

U

B

LE.

Delivery to door at any

21

b

Db.

61b

101b.

201b.

addra s in

s.

d.

S.

d.

8.

di

S.

d.

s

d.

Great Britain ...

4

G

G

0

7

G

10

0

14

0

Continent of Europe,

!

America, & Canada

7

0

9

G

12

0

10

0

21

0

S y d n c v, JI o b a r t,

Launceston

2

6:

3

0

3

G

4

0

6

0

New Zealand Ports

j

(except West Coast)

Adelaide, Brisbane.

4

0;

4

6

5

0

o

G

7

6


No further charge whatever. Very small increase for heavier weights. Delivery at country addresses in Australia, inland carriage only added.

Receiving office—

FREDERICK TATE, 13 Market-st., Melbourne TATE’.s PARCELS POST EXPRESS. DELIVERY at DOOR any address in the world. EITHER to or from Britain from Is. Gd. To or from other Australian ports from 2s. 6d. NO further CHARGE whatever. No trouble. Any SIZE, weight, or shape.

EVERY possible ASSISTANCE afforded.

I MQi ERIES plainly answered.

Henceforth Parcels handed to W. RTSUTTON’S Branches, every town in Great Britain, delivered in Melbourne at nearly similar rates.

Head Office :—

FREDERICK TATE, CUSTOMS AGENT, &c., 13 Market street, Melbourne.


TOMPETENCY

j    _


EXAMINATION. T IJ I T I O N—

IN CLASS, BY CORRESPONDENCE, OR PRIVATELY.

Other Work, by Arrangement.

J A M E S L. R O B E R T S O N, B . A ., 71 Clakendon-st., Emerald Hill.


A LEX. MCKINLEY & CO.,

GENERAL

r R I N T E R R A N D P U B L I S H E R S , 01 Queen-street, Melbourne.


In Bookwork and General Publishing our largo experience is a guarantee of ajjpwork being executed in the host stylo, while having a first-class stock of the best and latest material ensures expedition and good workmanship.

The following papers are issued from this office WEEKLY.

" Punch,”    “Bulletin,”

“ Faithful Words,”    “ Once a Week.”

FORTNIGHTLY.

“Australian Law Times,”    “Jewish Herald,”

monthly.

Schoolmaster.”    “ Monthly Messenger.”


HEAD TEACHER, 30 to 50, near Numurkah, wishes to exchange with teacher same allotment, in a cohl climate. The above is in rising township, which will shortly be railway terminus. Only reason for exchange, delicate wife. Address,  Pax,” Post Office, Melbourne.


T WARREN BALL’S “Hints to Candidates L» for Teachers’ and Matriculation Examinations,” Is.; posted, Is. Id. Mullen, Melbourne.

CANDIDATES for EXAMINATIONS prepared by correspondence or otherwise. I. Warren Ball, South Yarra.


I


M PERI AL REVIEW,


Quarterly, 23. Four Numbers, Subscription, 7s, Gd., including postage anywhere,

Publishers :

ALEX. M'KINLEY & CO.,

61 Queen Street, Melbourne.

YpXAMINATIONS.—UNIVERSITY

and DEPARTMENTAL.

TUITION in CLASS by Correspondence or otherwise. Terms, &c.,

JAMES L. ROBERTSON, B.A. (Melbourne). 71 Clarendon Street, Emerald Hill.

MULLEN’S


s-


New Classified Catalogue of School, College, and Technical

EDUCATIONAL WORKS

May be had gratis on application, or posted on receipt of address.

SAMUEL MULLEN,

Wholesale & Retail Bookseller & Stationer, 29 & 31 COLLINS ST. E., MELBOURNE.

Price One Shilling,

By Tost—In Victoria, Is. Gd. ; Out of Victoria, 2s


D


EPARTMENTAL EXAMINATION OF TEACHERS.

TEACHERS’ GUIDE

TO

AUSTRALASIAN EXAMINATIONS.

Containing the

PROGRAMMES & EXAMINATION PAPERS of

Victoria    South Australia

New South Wales    Auckland, N.Z.

Queensland    Wellington, N.Z.

And Tasmania.

108 PAGES WITH STIFF COVER.


Q C. EXAMINATION.

TUITION BY CORRESPONDENCE.


MR. 1HOMAS BOARDMAN, First-class Honor-man ot the Denominational School Board, Prepares Teachers tor the Certificate Examination by Correspondence. Terms moderate.

Address—

4 5 PRINCES STREET, CARLTON


TAMES CLEZY, M. A. ,

MELBOURNE.

CLASSICAL & SHAKSPEARE SCHOLAR (1869)

prepares candidates

lor Matriculation (pass or honours), and for the subsequent Degree Examinations of the University.

Course of Lessons by Com spondence in Latin Grammar, Translation, and Composition.

Terms on application personally or by letter. 5 GORE STREET, FITZROY.


rpO HEAD MASTERS, SECRETARIES -L OF BOARDS OF ADVICE, AND OTHERS.

ALEX. M'KINLEY & CO.,

Having mado considerable additions to their stock of Bookwork and Jobbing Type, are prepared to execute orders in every description of

GENERAL PRINTING.


All orders entrusted to them will be printed in a satisfactory manner.


Alex. M’Kinley & Co., Printers, Gl Queen-st.


BUY AND READ

THE NEW WEEKLY PAPER. Price Threepence.

uONCE A week;

16 PAGES. PRICE THREEPENCE. ALL BOOKSELLERS.


Alex. M'Kinley and Co., Publishers, Gl Queen Street, Melbourne.


The above hook contains the Programmes and Examination Papers of December, 1877, of all the colonies, and is reduced to the low price of

ONE SHILLING.

By Post—In Victoria, Is. 6d.; Out of Victoria, 2s


ALEX. M'KINLEY & CO., PRINTERS AND PUBLISHERS, 61 QUEEN STREET,

MELBOURNE,


M


NOW READY.

ILTON PARSED.

Price 2s.


By J. J. BURSTON,

(Author of “ State School Arithmetic”).

Also Ready, the Fourth Edition of the

STATE SCHOOL ARITHMETIC

By

JOHN J. BURSTON,

North Sandhurst State School.


Printed and Published by Alex. M'Kinley & Co., 61 Queenstreet, Melbourne, under the auspices of the Victorian Teachers Union.


AUSTRALASIAN

AND LITERARY REVIEW.

Yol. HI., No. 33.


MARCH, 1882.


Subscription

Yearly, Gs. fid. ; Half-yearly, ¡Is. Gd.


M. L. HUTCHINSON, Wholesale and Retail Bookseller and Stationer,

15 COLLINS STREET WEST, MELBOURNE,

SUPPLIES ALL

SCHOOL REQUISITES AT LOWEST PRICES.

NOVELTIES BY EVERY MAIL STEAMER.

BlaACKIE AND SON'S COMPREHENSIVE SCHOOL SERIES.

W. & R. CHAMBERS’ EDUCATIONAL COURSE.

CHAMBERS' ENCYCLOPAEDIA, NEW EDITION, 1881, 10 VOLE, CLOTH & HALF-CALF.

AGENT FOR HENRY THACKER & CO.’S “INK OF INKS” AND “PEN OF PENS.”

Ward, Lock & Co.’s lew Books for Every Home and School,

Including

THE WORTHIES OF THE WORLD: A Series of Historical and Critical Sketches of the Lives, Actions, and Characters of Great and Eminent Men of all Countries and Times, with numerous Portraits; one vol., cloth 7s. 6d,

THE UNIVERSAL INSTRUCTOR; on, Self-Cultuee foe All. Fully and accurately illustrated; to bo completed in three vols.; vol. I. now ready, 7s. 6d.

EVERY MAN HIS OWN MECHANIC: A complete guide for amateurs in every description of Constructive and Decorative Work, with 800 illustrations; one vol., cloth, 7s. 6d.

HAYDEN’S DOMESTIC MEDICINE, Edited by Dr. Lmnkester, assisted by Distinguished Physicians and Surgeons; numerous illustrations ; one vol., cloth, 7s. 6d.

BEETON’S ILLUSTRATED ENCYCLOPAEDIA, embracing Biography, Geography, History, Science, Art, and Literature, with about 2000 illustrations and 4000 pages; in four fine vols., half-bound, 42s. The cheapest Encyclopaedia ever published.

M. L HUTCHINSON,


GLASGOW BOOK WAKE HOUSE,


15 COLLINS STREET WEST, MELBOURNE.


Scij00Ì department.

THE GROWTH OF INTERMEDIATE EDUCATION IN GERMANY.

By H. Baumann, M.A., F.C.P.

The present system of Intermediate Education, so deeply rooted in tlie soil of the German Empire, can boast of a history quite as old as the empire itself. We may trace the beginnings of Teutonic school-organization, as you are well aware, to the days of Charlemagne, the patron of Alcuin. " It was by the Emperor’s orders that this learned and pious Englishman, the first educational minister on Germanic soil, reorganized the Imperial Court School, and scattered the seeds of Benedictine learning throughout the Christianized north.

Had the Emperor’s dream of a Frankish “ Culturstaat ” been realized, the revival of European learning, which ushered in an era of free thought and liberal education, might have been forestalled by fully five or six hundred years.

We cannot here debate the question why his successors—who, indeed , except the Othos, deserved that name?—showed themselves so utterly unworthy of the noble inheritance of the wise and generous Charles, History, the sphynx, loves dark riddles. Why medireval mankind, sharp-witted, enterprising human kind, would not reap and utilize the goodly things of knowledge, which wasted in the cloister-libraries, seems one of the gravest questions to be solved by the modern historian.

Yet the darkest of these dark centuries seemed, in other respects, full of fiery enthusiasm, and gave, in their own way, proof of immense productive energy. True, the golden harvest of Egypt and of Athens, the great lessons taught by Rome and Byzantium, all lay hidden and buried, packed away in musty parchment volumes, food for hungry cloister rats, rather than for the brains of an incurious race of humanity. But in the midst of an intellectual waste, there rose, like figures of a poet’s dream, the heroes and myths of fair Hellas into life again.

Orlando and Siegfried slew giants and dragons without number ; but, in matters of learning, they were like babes in the wood, Their age, the age of chivalry, was not deficient in the arts of poetry, sculpture, or architecture. It possessed heart-stirring epics and tender lyrical thoughts. What it most urgently needed, most deplorably neglected, was a little rudimentary instruction. The knights and the ladies, heroic and romantic though they were, wanted lessons on the art of writing and the rule of three. Wolfram von Eschenbach, the poet, would have collapsed in an easy dictation exercise. The finest lords and ladies at the king’s table lacked enlightenment, not on the rule of courtesy and taste, but on the mysteries of the ABC and the rules of spelling.

Poor noble-hearted Ritter Eisenfaust may often have felt the defects of his education ; but how was he to cure them? The age of lion-hearts had no books, no sciences, no accessible literature ; it had not kept up its Latin, and quiet forgotten its Greek. Its philosophy was hollow and wordy, and as to its theology and jurisprudence, the less we say about them the better. It had, above all, lack of schoolmasters and schools. Of schools, you say ? What had become of the learned Scoti, the devoted Anglo-Saxons, the great Benedictines, the Canonical brethren, the Mendicant Preachers,—what of all their schools? Had Alcuin and Rhaban, John Scotus and Ekkchard, Notker and Iso, all lived and taught in vain ?

Well, their schools still w'ere, but they were not what they had been in the glorious days of St. Gallen and Fulda. In the general turmoil of Imperial affairs, these old monkish establishments had partly grown useless, partly crumbled away in sheer despair; and, as to the new establishments, they hardly yet deserved the name of schools. All that we can discover, in the way of educational seminaries, to about the close of the 13th centnay, coniines itself to two categories, viz., the theological classes of the monastic houses and the little day-schools in the towns.

In the former, the young monks learnt to repeat prayers and to sing hymns in the dull Latin jargon of the times; or, if very far advanced, might mount up to the flowery wisdom of Isidor. or descend into the mysteries of allegory. The brightest of their wits were employed in raising sand and mud from the “ Ars G rammatica” of Donatus, or the “ Doctrinale ” of Alexander de Vi'la Dei. The friar-students—and this is our chief reproach against them—thought meanly of the vernacular tongue of their fathers, and studied the works of nature from a stupid rendering of Aristotle, or from anecdotes of Pliny and Aclian.

The other class of mental nurseries, intended to benefit the sons of town-artisans, was equally modest in its scope. The three R’s of our days, or the famous Trivium of an earlier age, were out of the question. Even the Dominicans and Franciscans never attempted a sound rudimentary instruction of their scholars. What they gave them in the way of intellectual training, was a compound of the Paternoster, the distichs of Cato, and Cisiojanus, i.e., a versified sort of pocket-calendar. But their imaginative faculties they allowed to run wild in the wood of saintly legends, and ghastly supernatural speculation. The Yolksschule or elementary school of modern days had yet to he created.

The schoolmasters of that period, if wc may honour them with such a title, bore an erratic character. They were the knight-errants of pedagogical history, the buffoons of a romantic age. To judge from their humorous Latin songs, and from the accounts of Thomas Platter, Luther, and others, the Bacchantes, even of the 15th century, seem to have possessed a universal fitness for every occupation between a highwayman and the Duke’s jester. They took to teaching only in their days of distress.

The first streak of light, in this gloom of barbarity, appeared on the southern horizon. From Italy, once before the home of Hellenic culture, came the glad tidings of a new reign of art and literature. In her free cities, the writers of Imperial Rome, rescued from the cruel ravages of time, were once more allowed to live and breathe. Rising from the mould and dust of ages of neglect, they marched forth to serve as models of a new and ambitious race of men. Under the roof of noble Petrarca, Cicero was restored to life, while the royal Homer resumed his reign under the auspices of his brother poet Boccaccio.

Greek scholarship was first smuggled into Western Europe by the Byzantine friars of South Italy. It held its official entry when the scholars of Byzantium, driven from their great city, were forced to become teachers in the Italian towns. A liberal study of the ancients was now boldly resumed. John of Ravenna and Emanuel Chrysoloras shone as teachers and guides of a new race of schoolmasters. Their efforts made the abstruse jargon of monachism give way before the polished style of Quintilian, Cicero, and Isocrates. The Italian school of Humanists thence arose, and broke the chains of Scholasticism. It was at this juncture that the scholars of Italy went to ransack the cloister-libraries of Switzerland and Germany. They took away easily what the Teuton friars, in their ignorance and superstition, saw no occasion to refuse. They could not fail, however, to excite, in many a youthful scholar across the mountains, an intense desire to visit the land of the great Classics, and to un-learn, in Ferrara or Florence, what had been instilled into him at Paris or Cologne. The schoolmen of the German universities shuddered at the new-fangled language and ideas imported from the south. The Latin of Cicero and Horace was, to their ears, a language of heretics, trampling under foot the authority of the Church Fathers. Greek and Hebrew, to them, were an abomination, an invention of the Evil Spirit, goading on rebellious minds that they might search the Scriptures aud correct St. Jerome. But their power, the power of the Thomists and the Aristotelians, was on the decline. There was a feeling abroad that the dark syllogisms of scholastic theology could not save mankind from the utter misery which prevailed in all classes, Erfurt, the great university of the 15th century, became a stronghold of the opposition, and Dalberg, the Medici of “young Germany.”

Thus, we are gradually landed in the period of German Humanism. But the teaching of the great Italian and Byzantine scholars had to approach the land of Scholasticism by side-ways and back-doors. It was through the medium of the Dutch Jeromians that the new blood of Platonism and Hellenic culture was gradually filtered into the educational systems of the Teutonic North. John Wessel, of Friesland, the friend of Sixtus IY. and one of the most brilliant scholars of the 15th century, may be named in their foremost ranks. It was he who, at considerable personal risk, exposed the schoolmen in the University of Heidelberg. Of greater importance, though of a humbler cast of mind, was, before his time, Thomas aliempis, the famous mystic of Zwoll. To him we are indebted, not only for those popular religious tracts which have immortalized his name, but also for the instruction of two enlightened educational reformers, Rudolf Lange and Moritz von Spiegelberg, the forerunners of Luther and Melanchthon. We may notice, as a favourable sign of the approaching change, that already they had introduced profane authors into the schools established in Westphalia.

As scholars they were far surpassed, however, by Agricola, another Frieslauder. “ Agricola,” to use the flowery language of his fond admirers and contemporaries, “guided, the first accomplished scholar among the Teutons, the Pierian maid, across the horrid Alps.” From hisshortstay at Heidelberg, about the year 1482, we may date the literary renaissance, and with it the new educational birth of Germany. Her great social reformers did not appear before the next century. But we may notice, as two important fellow-workers of the past, Dringenberg, the founder of the well-known school at Sclilettstadt, in Alsatia, and Alexander Hegius. The latter must be measured by his pupils. From his establishment at Zwoll proceeded some of the ripest scholars, both Dutch and German, of the 16th century, among them Desiderius Erasmus.

We have now mentioned a name as high and lofty as a Chimborazo among the mounts of human fame. In a history of human culture, Erasmus would occupy two interesting volumes ; in a short sketch of practical school-reform like ours, we must dismiss him in two lines. The Humanists, it must not be forgotten, never intended practical reform, neither in the Church nor in the field of education. Their powerful attacks upon the friars and their corrupt practices made Europe rejoice. Their “ Epistolae obscurorum virorum,” together with the satires of Hutten and the works of Erasmus himself, while proposing to make men laugh over monkish absurdities, opened up a new era of free thought and liberal education. But they stopped on the threshold of active enterprise. The great deeds of Reuchlin, Erasmus, and their illustrious party, including in its ranks the finest wits of the age, were merely of a negative character, at least in the field of education. _

The first impulse to practical school-reform was given by Luther and Melanchthon. It was a part of their Protestant creed to do away with the fabrications of a doting monkery, and to neutralize the blunting effects of their Aristotelian drudgery by the establishment of town schools after the Humanistic type. Erasmus had clearly shown the way in which a liberal study of the ancients ought to be pursued in the school-room. It remained for schoolmasters to carry out his wishes in

detail.    ,

Luther at all times most emphatic when angry, was never more so than when ho alluded to the friars as teachers. “ Twenty, nay thirty „ears” he says, “they study without perfecting themselves, either in the Latin or the German tongue, not to mention the vicious mode of life by which our noble youth have been debased.” In his beautiful address to the magistrates of the German cities, he expresses his deep sorrow that “ everywhere in the German land the schools are left to decay, the high schools declining in strength, the cloisters going to ruin, and nobody, save those intended for the Church or the Monastery, receive any instruction at all

Luther repeatedly urged the necessity of a liberal study of the Latin, Greek, and Hebrew languages. “For we cannot preserve the Scriptures,” he says, “ without Hebrew and Greek, They are like the sheaths wherein resteth the sword of the spirit, they are the vessel which holdeth their draught. If we lose the languages, we shall not only lose the. scriptures, but also the correct use of the Latin and German tongues.”

It is important to notice that he includes in his scheme of education History and Mathematics : but we cannot suppress a sigh that his preaching on such an important subject should have been so little attended to by his followers. “ If I had children of my own,” he says,

“ I should have them instructed, not only in the languages and in history, but also in Singing, Music, and Mathematics. How grieved I feel that I have read so little of the poets and the historians ! Instead of this, I have been obliged, by wearisome drudgery, to fill my head to such an extent with the philosophers and sophists, that I now find it a difficult task to rid myself of them.”

Thus far Dr. Luther. We meet a kindred strain of exhortation in the scholarly addresses of Melanchthon. He seems most in earnest when he commends the study of the Greeks. To him they were the true fountains of human wisdom. But both he and Luther condemn a portion of Aristotle’s works, as being unfit for the bringing up of Christian youths.

The German cities and princes responded to the appeal of the Reformers with an astonishing readiness. From the soil which had felt the mild rays of Italian Humanism and received the first seeds of modern free thought, sprang, almost spontaneously, Latin Schools, Town Academies, and Gymnasia, promising a great harvest of human progress and happiness. If they failed to bring about such a desirable result, it was from no lack of enthusiasm. Public and private munificence poured forth their showers upon them, so that they quickly multiplied and became a large generation. Bugenhagen at Hamburg, Sturm at Strassburg, the princes of Württemberg, Saxony, Hesse, and Brandenburg, each in their particular domain, moulded and hammered into shape what the Humanists and Reformers had sketched out in visionary outlines.

The ancients were henceforth studied in a more rational and profitable manner than had been the case for upwards of a thousand years. The feeble productions of the Byzantines were replaced by the Greek grammar of Melanchthon, the handbook of Alexander de Villa Dei by works more in harmony with the aspirations of the age. New and better editions of the classics were made, and printed books gradually introduced into the schools. Mathematics, Geography, Modern History, Science still remained in abeyance. Even at the University, the Professor of Mathematics kept to the four species ; but Rhetoric, Dialectics, Prosody, Mythology, Astronomy, and Grammar commenced to be treated in connection with the great classical authors. Aristotle and the Church Fathers, at last, were obliged to dine at one table with Horace, Lucian, and a motley crew of poets and orators. The Greek Testament replaced the Vulgate, and St. Augustine had to make room for Isocrates and Demosthenes. In the midst of beneficial changes like these, however, we grieve to see that the mother-tongue still remained sadly neglected, and that the chief object of education seemed to remain the acquirement of a polished Latin diction. In observing the slow progress made in this direction, one feels tempted to enquire, whether the famous battles of the Humanists, the stirring drama of the Reformation, had been enacted merely in order to force a new vocabulary into the hands of dozing pedagogues.

Since Agricola’s time, the example, first set by Politian and other Italian Humanists, had found enthusiastic followers in our northern lands. The great wits of the earth, England not excluded, now wasted their best time and strength on mimicking the writers of the long-forgotten past, and on forging sentiments which were not their own. History has passed her judgment on their doings. Eobanus Hessus, the poet, had he not borrowed his language from Rome, might now be one of the proudest names of German literature.

The majority of schools caught the prevailing disease. A shallow Latin colloquialism saturated the whole educational system. To become a perfect Ciceronian was now the crowning glory of a college career. Everything was done to stimulate the taste for rhetorical shows and classic sham-fights. Reuchlin himself had written Latin plays for the use and benefit of German students. Melanchthon allowed Terence to be performed by his boarders. The market-places of quiet college towns rang from fierce theological wranglings of Huttenian vigour, and it soon became the fashion of the day, as in the times of Ekkehard of St. Gallon, to make the scholars speak, write, feel, and think in a Latin jargon of the latest and toughest make. As the German fathers were ashamed of their vernacular speech, their sons in college were punished for using it, if only on the sly ! Arminius had fought his Varus-battle in the marshy wood of Teutoburg in vain.

The most pleasing feature in this educational movement of the Reformation was the generous impulse which it gave in matters of school organization. The growing number of young town-lads, crowding to the new founts of wisdom, necessitated more elaborate schemes for a graduated and rational class-instruction than could have been found in the monastic school. Hence, for the first time in Germany’s school history! we meet a uniform plan of classification, a fixed standard of knowledge’ corresponding with the ages of the pupils or the sections of their school.’ The modern town school thus commences its early career as one of thé greatest factors of German life and Teutonic culture.

The Reformers themselves had supplied a scheme for remodelling the dilapidated school-fabric of the past. Melanchthon, after having visited the schools of Saxony, issued in 1528, by the Elector’s order, the so-called

Book of Visitation ( Visitations biichlein), introducing a radical change into the school systems of central Germany. The plan of instruction proposed commends itself by its utter simplicity and clearness. No lofty-ideal, no revolutionary theory of pedagogical pyschology is kept in view. The intellectual feasts, prepared by the “ Praeceptor Germaniac,” are of the plainest kind. l’or he, Melanchthon, was himself one of the most popular and practical of teachers, and all the characteristics which strike us in his scholastic scheme can also be traced in his numerous school books.    ( To be continued.)

THE EARL OF CHESTERFIELD’S SCHEME OF EDUCATION.

By F. Howard, A.K.C.

('Continued from page 122.)

Although Chesterfield, as already remarked, had no exaggerated idea of the value of Latin and Greek literature, he still thought a competent knowledge of it a most desirable thing to be attained, as well for its own sake as for a prudent condescension to the general prejudice in its favour. Yet, even here, his uniform good sense is apparent. For the small pedantries of classical learning he cared nothing. He valued it, rather, for the models it afforded of literary style, for the perfection of its poetry, and the elegance of its oratory. As in English he was only partial to the best writers of the most polished period, that of Queen Anne’s time, so in Latin a “ gentleman scholar,” he thought, need read no other authors than those of the Augustan age. The Latin of Plautus and Terence was, in his opinion, “ bad, illiberal Latin,” which one would no more think of imitating than an Englishman would now write in the style of Chaucer or Spenser. For the writers of the Golden Age, however, he had an unbounded admiration, and his remarks upon their several styles and characteristics show a true and critical appreciation. Cicero he considered a consummate artist in language, and lie often quoted that saying of his, that it was a noble thing in man to excel others in that which all men excel the lower animals—speech. For Horace he had a high opinion. “ The more you read him, the more you will like him,” he says. The Art of Poetry he thinks his masterpiece, and makes the reflection that has probably occurred to almost everybody who has read that work— that the rules there laid down arc as applicable to almost every part of life, as they are to the making of poetry. “To avoid extremes, to observe propriety, to consult one’s own strength, and to be consistent from beginning to end, are precepts as useful for the man as for the poet.” Tacitus, he thought, had a peculiar terseness of style which often renders him obscure. Moreover, he wrote at a time when the Latin language had greatly degenerated from the purity of the Augustan age. “ But he knew and describes mankind perfectly well; and that is the great and useful knowledge.” As might be expected of one of his peculiar turn of mind, Chesterfield preferred the brilliancy and elegance of Ovid to the correctness and coldness of Virgil. The Metamorphoses was a favourito work of his, as it has always been with men of taste ; and he often gives quotations from it as examples of just arid accurate descriptions.

But the subject to which he attached the highest importance of all was History—especially Modern History—in the days, too, when it was the fashion to treat history contemptuously, as of no more value than “an old almanack.” We make much now, and rightly so, of Dr. Arnold, who was the first to introduce Modern History into the curriculum of public schools; but in awarding credit in this matter, we should remember that Chesterfield, a century before Arnold’s time, not only advocated the claims of Modern History to a prominent place in a liberal education, but made it the corner-stone of his own system. He thought History the most useful thing men could study, because of the practical lessons of wisdom which it teaches. He talked much of the Philosophy of History. Whether, indeed, there is such a Philosophy, is a dcbateable question ; for, in treating of human affairs, we can never be quite sure either of our facts or our inferences. As it has been well said, a true science of History would require all-knowledge to record it, and all-wisdom to interpret it. But still history aims at giving us some insight into the ways and doings of men, and so far it may be considered the most profitable of all studies. His method of dealing with history was good. There arc, ho says, two eyes of History, Chronology and Geography. The one tells us when, the other where, an event occurred—two important factors in our knowledge of historical events. Without Chronology, History would be a confused heap of facts, in which all sequence and development of cause and effect would be entirely lost sight of. With reference to Geography, he recommends his son, in reading History, always to have at hand a map for ready reference. You may know, or think youknow, your Geography “ in the lump,” and you will probably soon forget it again ; but it is the looking for a particular place in the map, upon some particular account, that serves to fix it indelibly in your memory. He recommends an attention to the great epochs of History.

“A judicious reader of History will save himself a great deal of time and trouble by attending with care only to those interesting periods of history which furnish remarkable events, and make eras ; going slightly over the common run of events. Sorne people read history, as others read the Pilgrim’s Progress; giving equal attention to, and indiscriminately loading their memories with, every part alike. But I would have you read it in a different manner; take the shortest general History you can find of every country, and mark down in that History the most important periods, such as conquest, changes of kings, and alterations of the form of government; and then have recourse to more extensive histories, or particular treatises, relative to these great points.”

I have now gone through the principal topics upon which Lord Chesterfield thought it necessary to dwell when writing to his son upon educational questions. There are other points I might allude to, did time and space permit:, particularly that t<? which he attached considerable importance—a knowledge of the world, and those prudential rules of conduct which all men who desire to play a becoming part in life find it more or less necessary to adopt. No part, perhaps, of his Lordship’s letters have been more vehemently attacked than this. It has been objected, that he took too unfavourable a view of mankind ; that many of his precepts were calculated to inspire distrust, and an artful, designing conduct. But there are some people who never will take things in a right light. A little tact and ,9avoirfaire are found useful in every station of life ; but, surely, to none is it more necessary than to those in the position in which young Mr, Stanhope was placed, who are entrusted with the conduct of important affairs, and whose lot is cast in the courts of princes and sovereigns, in the midst of so many rival claims and opposing interests.

To those in the habit of regarding Chesterfield as a man of loose principles, it may perhaps be a surprise to know that he inculcated in his son a high regard for the purity of moral character. He distinguished between what he called “vices of the heart,” and those faults of youth which flow more from the warmth of the blood than from any vicious propensity. To these he could extend some indulgence, but for the others he had nothing but the sternest reproof and condemnation. “ I will recommend to you a most scrupulous tenderness for your moral character, and the utmost care not to say or do the least thing that may ever so slightly taint it.”

In summing up the general impression produced by his letters, it must be confessed, I think, that, on the whole they present the writer in a by no means unfavourable light. They show that, beneath the formality of artificial manners, the domestic affections had a strong hold over his heart. Fashion had not altogether stifled nature, Lenaturel Vemporte svr la politique. So far from recognising any counterpart of that caricature under which Charles Dickens has represented him, as a cold, heartless parent, one is rather inclined to believe that there were in those days few fathers half so solicitous, and so tender, as he was. Nor can one help contrasting these letters with those much belauded epistles of Madame de Sdvignd, who lived a cat-and-dog life with her daughter ; but, no sooner were they apart, than, knowing her letters would bo shown about, and afterwards appear in print, she wrote to her chore jille in the most endearing and affectionate terms. Chesterfield,'on the other hand, never had the least idea that his letters would ever be read by a single person but his son ; and he was careful to make provision that they should never fall, by inadvertence, into the hands of improper persons— a precaution that was ultimately defeated, unfortunately for himself, by a member of his own family. On the premature death of his son, when it became known for the first time that the latter had been secretly married, the Earl, now grown old and infirm, purchased all the letters from the widow. But she had surreptitiously taken copies of them ; and, no sooner was the old man dead, than she basely sold them to a publisher.

Surely, this is a fact that deserves to be kept in view in forming a judgment upon Chesterfield’s character. His letters, unlike manyothers which have become famous, were never written with an eye to the effect they might produce upon the public. On the contrary, they express his most intimate thoughts, without a suspicion that any unfriendly eye would afterwards criticise them. No doubt some unfavourable features arc revealed ; but the picture, if not a flattering one, is at any rate a genuine likeness, and that is more than can be said of many writings of this kind. No man is a hero to his valet, and Chesterfield has suffered from a too close and minute inspection of his private thoughts and life. Did we know as much about his chief detractor, Dr. Johnson, for instance, in those early years of his, of which he could never be induced to speak, when he was living an out-of-elbow sort of life as an unknown scribe in London, perhaps the world might modify their impressions of him. But it has been Chesterfield’s misfortune to be subjected to a narrow scrutiny, for which few under similar circumstances would be prepared. If this has made clear some defects, we must not conclude that he was an exceptionally bad man, or one any worse than the majority of his class. My purpose in these remarks is, however, in no way to defend what may be considered reprehensible, but to give an impartial view of a theory of Education which at one time excited much attention, and has been exposed to much obloquy, but which, I believe, few who condemn it have taken the trouble to examine for themselves, —Educational Times.

HINTS TO TEACHERS FROM WISE HEADS.

No. 4.    (Continued from page 122.)

BY MARIA AND L. EDGEWORTH.

Whatever we command to be done, or rather whatever we associate with pleasure, they imagine to be right ; whatever we prohibit, provided we have, uniformly associated it with pain, they believe to be wrong. This implicit submission to our authority, and these confined ideas of right and wrong, are convenient, or apparently convenient, to indolent or tyrannical governors; and they sometimes endeavour to prolong the reign of ignorance, with the hope of establishing in the mind an opinion of their own infallibility. But this is a dangerous as well as an unjust system. By comparison with the conduct and opinions of others, children learn to judge of their parents and preceptors ; by reading and by conversation they acquire more enlarged notions Of right and wrong, and their obedience, unless it then arise from the conviction of their understandings, depends but on a very precarious foundation. The mere association of pleasure and pain, in the form of reward and punishment, with any given action will not govern them ; they will now examine whether there is any moral or physical necessary connection between the action and punishment; nor will they believe the punishment they suffer to be the consequence of the action they have committed, but rather a consequence of their being obliged to submit to the will of those who are stronger or more powerful than they are themselves. Unjust punishments do not effect their intended purpose, because the pain is not associated with the action which we would prohibit; but, on the contrary, it is associated with the idea of our tyranny ; it consequently excites the sentiment of hatred towards us, instead of aversion to the forbidden action. When once, by reasoning, children acquire even a vague idea that those who educate them are unjust, it is vain either to punish or reward them ; if they submit, or if they rebel, their education is equally spoiled ; in the one case they become cowardly, in the other headstrong. To avoid these evils there is but one method ; we must early secure reason for our friend, else she will become our unconquerable enemy. As soon as children are able, in any instance, to understand the meaning and nature of punishment, it should in that instance be explained to them. Just punishment is pain inflicted with the reasonable hope of preventing greater pain in future. In a family where there are several children educated together, or in public schools, punishments may be inflicted with justice for the sake of example, but still the reformation and future good of the sufferer is always a principal object; and of this he should be made sensible. If our practice upon all occasions correspond with our theory, and if children really perceive that we do not punish them to gratify our own spleen or passion, we shall not become, even when we give them pain, objects of their hatred. The pain will not be associated with us, but, as it ought to be, with the fault which was the real cause of it. As much as possible we should let children feel the natural consequences of their own conduct. The natural consequence of speaking the truth is the being believed ; the natural consequence of falsehood is the loss of truth and confidence ; the natural consequence of all the useful virtues is esteem, of all the amiable virtues love, of each of the prudential virtues some peculiar advantage to their possessor. But plumpudding is not the appropriate reward of truth, nor is the loss of it the natural or necessary consequence of falsehood. Prudence is not to be rewarded with the affection due to humanity ; nor is humanity to be recompensed with the esteem claimed by prudence. Let each good and bad quality have its proper share of praise and blame, and let the consequences of each follow as constantly as possible. That young people may form a steady judgment of the danger of any vice, they must uniformly perceive that certain painful consequences result from its practice. It is in vain that we inflict punishments, unless all the precepts and all the examples which they see confirm them in the same belief.

In the unfortunate sou of Peter the Great wc have a striking instance of the effects of a disagreement between precept and example, which in a less elevated situation might have escaped our notice. It seems as if the different parts and stages of his education had been purposely contrived to counteract each other. Till he was eleven years old, he was committed to the care of women, and of ignorant bigoted priests, who were continually inveighing against his father for the abolition of certain barbarous customs. Then came baron lluysen for his governor, a sensible man, who had just begun to make something of his pupil, when Prince Menzikof insisted upon having the sole management of the unfortunate Alexey. Prince Menzikof abandoned him to the company of the lowest wretches, who encouraged him in continual ebriety, and in a taste for every thing mean and profligate. At length came Euphrosyne, his Fin-landish mistress, who upon his trial for rebellion deposed to every angry expression which, in his most unguarded moments, the wretched son had uttered against the tyrannical father. Amidst such scenes of contradictory experience, can we be surprised that Alexey Petrovitch became feeble, ignorant, and profligate ; that he rebelled against the father whom he had early been taught to fear and hate ; that he listened to the pernicious counsels of the companions who had, by pretended sympathy and flattery, obtained that place in his confidence which no parental kindness had ever secured ? Those historians who are zealous for the glory of Peter the Great have eagerly refuted, as a most atrocious calumny, the report of his having had any part in the mysterious death of his son. But how will they apologise for the Czar’s neglect of that son’s education, from which all the misfortunes of his life arose?

But all this is past for ever; the only advantage we can gain from recalling these circumstances is a confirmation of this important principle in education ; that when precept and example counteract one another, there is no hope of success. Nor can the utmost severity effect any useful purpose, whilst the daily experience of the pupil contradicts his preceptor’s lessons. In fact, severity is seldom necessary in a well conducted education. The smallest possible degree of pain, which can in any case produce the required effect, is indisputably the just measure of the punishment which ought to be inflicted in any given case. This simple axiom will lead us to a number of truths which immediately depend upon or result from it, We must attend to every circumstance which can diminish the quantity of pain, without lessening the efficacy of punishment. Now it has been found from experience, that there are several circumstances which operate uniformly to this purpose. We formerly observed, that the effect of punishment upon the mind of children, before they reason, depends much upon its immediately succeeding the fault, and also upon its being certainly repeated whenever the same fault is committed. After children acquire the power of reasoning, from a variety of new motives, these laws with respect to punishment derive additional force. A trifling degree of pain will answer the purpose, if it be made inevitable ; whilst the fear of an enormous proportion of uncertain punishment will not be found sufficient to govern the mind. The contemplation of a distant punishment, however severe, does not affect the imagination with much terror, because there is still a secret hope of escape. Hence it is found from experience, that the most sanguinary penal laws have always been ineffectual to rcstraiu from crimes. Even

If detection be inevitable, and consequent punishment equally inevitable, if punishment be not inflicted as soon as the criminal is convicted, it has been found that it has not, either as a preventive or a public example, its proper power upon the human mind. Not only should the punishment be immediate after conviction, but detection should follow the offence as speedily as possible. Without entering at large into the intricate argument concerning identity and consciousness, we may observe, that the consciousness of having committed the offence for which he suffers ought at the time of suffering to be strong in the offender’s mind. Though proofs of his identity may have been legally established in a court of Justice, and though as far as it relates to public justice, it matters not whether the offence for which he is punished has been committed yesterday or a year ago ; yet, as to the effect which the punishment produces on the culprit’s own mind, there must be a material difference.    (To he co7itinued.)

EARL SPENCER’S SPEECH AT OPENING OF A NEW SCHOOL AT BRADFORD.

Earl Spencer, who was very warmly received, said : There is no town and no part of the country so forward in the work of education as Bradford and the county of York: Bradford will always have its name connected with that Act to which the country owes so much with regard to schools. It is the Act of 1870, which my right hon. friend Mr. Forster carried through the House of Commons, which will provide in every part of the country, in towns as well as in counties, the means of education to all the people. Before that Act was passed there was a great deficiency, not only in education, but in the places in which education could be given. Now, I am happy to think, there is hardly a corner in the whole of this our great land where parents cannot send their children to school, and where a sound and practical and useful education may not be given to them. Last year we were able to carry on this great work by making more perfect the machinery which the law has devised for bringing children to school. I remember the time very well when any mention of what is called compulsory attendance in schools would raise a storm of opposition in various classes of the community. But now we have got practically established throughout the breadth of this kingdom compulsory education, and I believe I could appeal to you in support of that principle. Now, Bradford is especially distinguished, as we have heard this mprning, by the manner in which the School Board has carried on its work. It has doubled the school accommodation and the school attendance in this town, since it came into operation. And this good work, as we have heard, not only extends to the schools under the School Board, but also to other elementary schools, of which there are a large number in this town. These latter schools have benefited, as well as the others, by the introduction of the wise and excellent system of education established under the Act of 1870. It is a great pleasure to me to see these admirable buildings which are about to be opened for instruction in this neighbourhood. We have heard of the expense that School Boards go to with regard to their schools. Of course, any unnecessary expenditure is very wrong, and it is the duty of the Education Department—who are responsible to Parliament for the loans that are made to the local authorities—to see that no unnecessary expenditure or extravagance is incurred. But I think it is very clear that one of the most important things in connection with our schools is to have them not only salubrious for the children, but to have them cheerful, and places to which the children will like to come. There is nothing, I believe, that does more to promote general discipline and improvement amongst our children than that they should be accustomed to cheerful and bright rooms, and be continually in a healthy, instead of an unhealthy atmosphere. Besides the instruction and education given, the general effects to which I have referred are among the benefits derived from schools in this country, Therefore I think it is a wise thing that your schools should be made as cheerful and healthy and commodious as possible, not only that the masters may be able to give their instruction in the best and most convenient form, but that the pupils who come, sometimes from crowded streets where they do not get as much fresh air as they should, may be benefited by the change to large and salubrious schools. I think, tkcreforc, that any expenditure which is made to provide for these requirements is not thrown away, and it is wise for School Boards to lay out money in order to secure these great objects ; at all events it is satisfactory to find that in a large place like Bradford, where popular opinion is rightly so sure to be made known, the ratepayers have supported the School Board in what they have done to secure good schools in their town. We often hear, and we have heard it in Parliament more than once, that the education given in the primary schools of the country is not what it should be, and that we are trying to do too much in the way of education—that the children in the schools get crammed and do not get healthy education, and educational discipline—that we are aiming at giving education which ought to be given only in higher schools, and that we are leaving altogether the lines which Parliament laid down when it sanctioned very large grants towards primary schools in this country. I believe you will bear me out when I entirely deny all these statements. I believe that the education given in the schools is sound, useful, and practical, and that we are not departing from the lines which Parliament has laid down for our guidance. I am sure you will all agree with me that we deprecate from the bottom of our hearts anything like superficial education or cramming. What we want to get is a sound and useful foundation of education among all our children, and that during the short time they are able to remain at school they should be taught as much as

can be taught them, but only so much as can be taught them in a sound | and practical manner. In our schemes for education no doubt there are a great variety of subjects, some of which are of a scientific and higher kind. It is to these particularly that some people object, but I believe it will be generally found that in schools where these subjects are taught the foundation of education is made more secure, and is better conducted than in schools where these systems are not followed. The fact is that constantly one of these subjects is one upon which the ! teacher possesses particular aptitude, and besides that, children generally themselves take an interest in subjects of this kind, when you would not be able to develop an interest in their minds on other subjects, I think, then, I am not wrong in saying that you will constantly find in schools, where subjects like botany or physiology or kindred subjects are taught, that the elements of education to which I attach the greatest possible importance, and without which no education is perfect, are taught more thoroughly than in other schools. In Bradford you have been almost the first to try this ; you have got some schools which I was most anxious to see. You have a higher class of schools among your elementary schools. These schools are more or less in the kingdom experimental schools. Perhaps you will take exception to the use of that term ; but I say they are to some extent experimental, because there are not many places where they have been established. I went just now to one of them, and I saw the work which is being conducted there with the greatest possible interest. There the better children from the other elementary schools, by means of scholarships or exhibitions, get better education under perhaps a higher class of teacher. You have higher subjects taught, but I cannot think that these subjects are those which would encroach, or, to use a common expression, poach upon the ground occupied by the grammar schools. I should think all these means ought to help the higher education and give a stimulus to it. There is no doubt that one of the things we most want in this country is to have the opportunity of giving to children of ability and studious habits the means of going up the ladder of education from one branch of it to the other. There is hardly anybody in this kingdom now who will deny the right of the poorest workman in the country to have his child educated, so that if he shows ability and aptitude that child can mount to the top of the tree or education in this country. In order to do that you must have scholarships and exhibitions from one school to the other—exhibitions from the elementary to the grammar school, aud scholarships from the grammar school to the University. And I am happy to say that by the munificence of many of your citizens in various parts of the kingdom—and I hear in Bradford itself—these means have been provided. These schools to which I have referred occupy the intermediate stage. But I believe it can be argued and shown that they do not give except in a very few instances —those perhaps accidentally—education to a class who can afford to send their children to the grammar school. They, to a great extent, give education to a class who require it as much as any class in the kingdom, and besides that, they gave education to the cleverer and most successful boys from the various elementary schools in the town. I think by this method you are doing a very great work, and I hope to see some more of the schools to-day. It will be very instructive to me in considering any plans that may be made to carry out this object.

I do not want to detain you at any great length. I merely wish to refer to some of the changes that have been made recently by Mr, Mundella, who ably assists me in the Education Department, on a subject to which he has given the greatest possible attention, I mean some of the changes we are about to make in the Code. It is of great importance that these changes which have not been made or actually drawn out, should be well and thoroughly considered. I shall not now go iuto them in detail, but I wish to refer to one or two points in regard to them. We have found that the system of payment by result has led to one drawback, which we deplored very much—it had a tendency to keep the reward to all schools on the same level. The master who produced a number of children taught in the most admirable manner received only the same payment as the master who only just “scraped through” his children in the standard of education. We thought that had a bad and prejudicial effect, and wc were most anxious to relieve it by giving grants in different gradations, more to those which arc best, aud perhaps less to those that are the worst, while the average would receive probably about the same as they receive now. That, I think, is a change which will be of great importance and of great value in the working of the Code. It may be difficult to carry out these changes. We are not going in future to require the examination of every child. It is a somewhat complicated thing to explain, and I shall not attempt to explain to-day the methods by which wc hope to arrive at payments by result—from the average payments for children instead of by individual payments. But that, I think, will be a change which will be well received in the country and will work well. Wc have made some other changes with regard to teachers and the requirement of more assistant teachers, and with regard to the limitation of children who are under one teacher. All these different changes, wc hope, will be of utility, and will promote popular education through the country.

I shall not any longer detain you. I have ventured to make these few remarks, and have tried to prove to you the deep interest which I feel, not only personally, but as representing the Department in the education of the country, particularly the interest I feel in seeing the schools established in Bradford, and I have particular pleasure in coming here to this room to see so many who most probably have children in the school who evidently take a deep interest iu the proceedings of the day.

It is my duty to declare this school open, and in doing that I most heartily wish it every success, and that it will tend to the good of the population and to the prosperity of Bradford in general, (Loud applause.)— The School Board Chronicle.

NOTES OF A LESSON ON PLANTS.


By Miss S. Nickels, P.T., No. 1459, Amherst.

For combined VI, and V.

Introduction,—When we put a seed into the ground we expect, by-and-bye, to see something sprout up, and grow daily larger. What is the name of this thing which we watch so earnestly ? A plant! We are going to-day to trace a plant in its growth, and examine its various parts. Apparatus.— Black-board, duster and chalk.

Objects Required.—Carrot, turnip, sorrel root, grass with fibrous root, radish, bulb or gladiolus root, a pear or a vine-leaf.


Heads.


Matter.


Method,


(1.) Elicit this by showing a plant with its roots cut off. Stand it on the ground and let them see result.

(2.) Explain with reference to our breathing,

(3) Give other illustrations, or obtain names from class ; show plants named, if possible to obtain same.

Recapitulate aud question briefly.


(4.) Elicit this.


(5.) Clive proofs of being clothing, and organ of perspiration.

(6.) Prove this by referring to plants growing in dark, bleaching of celery, of cabbages, and lettuces.

(7.) Ask for name of some others that turn to sun.

(8.) Compare sap to blood in an animal’s body.


(9.) Elicit most of this if possible.

(10.) Ask for names of others that are carefully stored.

(11.) Illustrate by children amusing themselves by blowing thistle seed about, and sticking burrs in each other’s clothes—thus fulfilling one of the great ends of nature, “ the diffusion of plants.”


(«) Fibrous : Like threads, as in grasses and shrubs,

(b) Creeping ; Large horizontal fibres with tufts of small fibres springing therefrom as in Sorrel; the tufts arc the real roots. These kinds of roots are very tenacious of life.


(2.) Stem


(9.) Leaves


(4.) Sap


I'r>) Seed


Heads,


Matter.


Method.


Life of Plants (13)


Uses


Plants imbibe (13) carbonic acid gas which is a compound of oxygen (a gas) and carbon (solid matter) (14.) Plants have the power of separating these two. They keep the carbon to form their structure and restore the oxygen to the atmosphere. This carbonic acid gas is poison to animals, being the air which they breathe out; but the oxygen which plants give out is necessary to animal life, so that plants purify the air again. This shows the mutual dependence of the animal and the vegetable kingdoms.

Plants are used in a great variety of ways.

Food for man and beast, for industrial purposes as building, &c. medicinal purposes (15).


(12.) Mention circumstance of birds conveying seed of cloves to islands where tree had been destroyed in East Indies by order of those in authority, to secure a monopoly of trade, thus their intentions werefrus-trated by birds.

Question.

(13.) Compare plant with our bodies. What keeps up our frame? Food! Same with plant ; must have something to build its frame, only not exactly same form as that our bodies get.

(14.) Ask how plants imbibe gas— by pores. Spoken of in first part of lesson.

(15.) Give examples of these uses and obtain from class information illustrative of the waysin which plants are useful to man.

Question,

Give a brief recapitulation of the lesson, and question.


Parts—    The root supports plant by fix-

(1.) Root ing it in the ground (1) ; also a channel for nourishment of plant. At end of roots there are minute swellings, called spongioles, full of pores to absorb moisture from ground. There are pores in every part of plant above ground for exhalation (2) but no pores in roots (except in spongioles) because not being exposed to atmosphere, nothing to exhale.

There are five kinds of roots each differing in appearance from the others.

(c)    Tap: Tapering to a point like radish and carrots,

(d)    Bulbous : Round like a lily or onion ; this is not really the root, but a swelling of the stem of plant; it is, however, generally termed the root.

(e)    Tuberous : Knotty swellings as potato (3).

The stem is the part through which sap circulates and from which leaves and flowers spring ; stem is not always apparent as in some vegetables. Functions of the root and stem are different ; the first draws up nourishment from soil, the 2nd distributes it over the plant. (4.)

Leaves arc a clothing agaiust cold and heat: these are the organs of perspiration (5). They are of a green colour owing to light, which has a very powerful effect upon plants ((>). Light acts beneficially on upper side of leaf, but injuriously on under side ; hence former always turn to it, also some flowers (7) as daisy, &c.

Sap is the moisture, found between the bark and stem in young wood ; at some periods of year will flow more readily than at others, particularly when plant is wounded. Sap is at rest till leaves appear and distribute it profusely like perspiration. May find out where sap is by watering plant with coloured water. Sap will then appear of same colour. (8.)

After plant has come to maturity it flowers (9) and bears seed in order to propagate its kind.

Seed is the beginning or life of plant, therefore it is always taken great care of, as in nuts (10) in down, cones, See.

Various ways of dispersing seed, as thistle-down carried by wind (11) carried by birds (12), wafted by waves of ocean.

WRITING.

Pupil-Teachers may do much harm to a school by carelessness and inattention on their part during the writing lesson. Copy-books are taken home, and are among the chief means a parent has of finding out whether due attention is paid to a child. Bear this in mind, and see that nothing is wanting on your part to send out copy-books that shall speak well for the school. You do not live in an age of sharpening quills and setting copies, so that all your time may be devoted to the inspection of the copies, and pointing out defects. Never allow any child to write more than two lines without your seeing them, In looking round, you will always find that many make the same error in the formation of a letter. In this case, take the blackboard and make the letters properly on it, calling the attention of the whole class to the common errors you find.

Always take the trouble to commend a clean, carefully written copy. It will also encourage the child who has written it, and spur the others on to imitate it, if you call the attention of the head teacher to it as deserving special notice. See that the children sit in an easy posture while writing, and that they hold their pens easily. Although rules have been laid down “ from time immemorial” for “ holding the pen,” it is almost impossible to secure uniformity in this matter. All hands are not the same shape. A child with a short, thick, stiff hand cannot hold a pen with as much ease, aud as gracefully, as one with a long, slender hand. Therefore, due allowance must be made in such cases. See that every child is supplied with a good pen, as well as clean ink.

Those of you who teach the lower classes (especially in infant schools) sometimes find it a wearisome task to teach the little ones how to make letters after they have learned strokes and tarns. I find that generally the difficulties rest on ymir shoulders.§A copy is set, a, b, c, d, and so on, and the children are expected to imitate it without any further explanation. Had the letters been “classified,” and a certain numbertaken each day, the results would have been much more satisfactory. For example, before allowing the children to begin writing on their slates, take the blackboard and have a talk about what you are going to do. Now, children, to-day we are going to learn to make those letters which have ‘ round o' in them. You can make o first—then see me put a long stroke turned at the bottom, and it will make d. ” In like manner take g, g, a. Then, when these are mastered, take those letters which just touch the lines, top and bottom, Next, those with strokes above the line, and those with strokes below the line, and so on. But you must interest the children by making them yourself on the blackboard first, and talking about them as you do so. When the letters are thus learned, take a few words during the week ; but let them have but two letters in them at first, and extend very gradually to more. Thus the writing lesson may be made very interesting, even in the lowest classes ; and though you may think the process rather a slow one, you will find out th at it is a sure one.

n la the references the first number after the name of the book denotes the chapter, the second the verse. I Cor. ix. 10., means the first epistle to the Corinthians, the ninth chapter, and the tenth verse.—A. C.


THE LANGUAGE OF THE REVISED VERSION OF THE NEW TESTAMENT.

By Robert Craig, M.A., Brighton.

It is proposed to compare the revised version of the Hew Testament and the authorised version in such a way as may interest those who like to study somewhat minutely the grammar, the history, and the resources of the English language. It will be convenient to compare them first as to the forms of words and phrases, next as to the choice of words, and lastly as to the order of words.

As the terms Authorised Version and Revised Version will have to be used frequently, they will for the sake of brevity be denoted respectively by the letters A.V. and R.V.

It is well known that at the time when the A.V. was made (a.d. 1611) the spelling of the language was very unsettled, the same words being spelt differently, not only by different writers, but also by the same writer in different places. That these differences do not exist, except to a very small extent, in the A.V., as we now possess it, would appear to be owing to alterations introduced from time to time by those entrusted with editing it since 1611, particularly by Dr. Blayney, who in 1769 issued under authority a corrected edition of the A.V. The comparatively modern and uniform spelling of so old a book is thus accounted for, but a few inconsistencies still remain, which can be attributed only to oversight. In Luke ix. 62.* we find plough ; in 1 Cor, ix. 10, we find ploiv ; plat in one place (They had platted a crown of thorns, Matt, xxvii. 29), plait in another (the adorning of women, let it not be that outward adorning of plaiting the hair, 1 Pet, iii. 3). The latter inconsistency is not retained in the R. V., the former is. In the A,V. words now ending in c, like public, heretic, lunatic, are found with a final h, aspublich. This was the mode of spelling such words till the end of last century. In the R. V. the present spelling of public and publicly is adopted ; for lunatic, ‘epileptic’ is substituted, and for lieretick, “ a man that is heretical see Matt. i. 19 ; iv. 21 ; Acts xviii, 28 ; Tit. iii. 10. On the other hand, the final h appears to be purposely retained by the revisers in havoch, for they have used this word with this spelling in their translation of Gal. i. 13., “I made havock of it,” where the A. V. has, “ I wasted it.” The reason for such inconsistency is not obvious.

In the A.V. the word shamefacedness occurs. “ I will that women adorn themselves in modest apparel with sha-mefacedness and sobriety ” (1 Tim. ii. 9). Philologists tell us that the word should be spelt shame-fastness, because it has nothing to do with face, but comes from shame, and the Saxon faest, meaning perfectly, very. This view is adopted in the R.V., where the word is spelt shamefastness in the verse above cited. This noun presupposes the adjective shamefast. In the edition of Shakespeare’s Richard III., by Mr. W. Aldis Wright, one of the learned editors of the Cambridge Shakespeare, this form of the adjective is given : “ Conscience is a blushing shamefast spirit,” Act 1, Scene 4, line 135, The ordinary editions give the form “shamefaced.” Mr. Wright says: “ The spelling ‘ shamefast,’ which gives the correct form of the word, is supported by the authority of the majority of the quartos. But the corruption which has prevailed to this day was already countenanced by the reading of the folios 1 shamefac’d.’ [The quartos were printed in Shakespeare’s lifetime, the first folio in 1623. He died 1616.] In the same way, in the third part of Henry VI., iv., 8, 52, ‘shamfac’d ’ is the spelling of the folios, while in the original play, the true tragedie of Richard, Duke of York, it is ‘ shamefast.’ The corruption is at least as early as Sidney’s Arcadia (quoted in Richardson’s Dictionary).” A quotation in Latham’s edition of Johnson’s Dictionary, from Spenser’s Faerie Queen, exhibits a curious inconsistency in the spelling of this adjective and its derived noun :—

“. ... She is the fountain of your modesty ;

You shamefaced are, but shamefastness itself is she.”

The same dictionary quotes from Dryden: “None but fools, out of shamefacedness, hide their ulcers.” A more modern authority, Sir Walter Scott, uses the form shamefaced in the Lady of the Lake, Canto II., Stanza 24 :—

“ The Hush of shamefaced joy to hide,

The hounds, the hawk, her cares divide.”

This spelling, corrupt though it may be, has obtained so respectable a standing for no other reason, doubtless, than because it presents a meaning more easily seeu by all than is that of the correct form, just as sweetheart has taken the place of the philologically correct sweet-hart, or sweet-art, formed after the analogy of braggart. As people are apt to believe that sweetheart must have to do with heart, so they naturally connect shame with face, and write “shamefaced,” the token of shame being usually manifest in the face, and “ shamefaced,” rather than[“ shamefast,” readily suggesting itself as the accurate opposite of “brazen-faced.” In substituting shamefastness for shamefacedness the revisers seem chargeable with pedantry. The ordinary reader will not understand the word in the former spelling so well as he would in the latter.

Ought, meaning anything, occurs frequently in the A.V. ; for example, “ If any man say ought unto you,” Matth. xxi. 3 ; “ Forgive if ye have ought against any,” Mark xi., 25. According to its derivation from the article a and whit, a little thing, its proper spelling would appear to be aught, and this is the form adopted in the R.V. But by a strange inconsistency the form nought is retained for the negative of aught. For instance: “All, as many as obeyed him, were dispersed and came to nought," Acts v. 36 ; “ Why dost thou set at nought thy brother? ’ Rom, xiv., 10.

Spunge (Matth. xxvii. 48) is altered to sponge, but bason is left unaltered. (John xiii., 5.) The R.V, always has e after they in judgment, the A.V. omits the c. In both, the form entreat means to use or to handle; the form intreat, to request. Luke xviii., 32 ; “He shall be mocked and shamefully entreated.” Luke xv, 28; “His father came out and intreated him” (to come in). The A.V. has enquire, the R.V. has inquire (John iv. 52 ; 2 Cor. viii. 23); but the R.V. has, inconsistently, enquired in Luke xviii. 36. The A.V. has the noun recompencc, the R.V. recompense. (Luke xiv. 12, and elsewhere.)

Some obsolete forms in the A.V. arc displaced in the R.V.    by kindred forms still in use,    and    consequently

more intelligible. In Romans xi. 24, the A.V. has, “ Much more shall these, which be the natural branches, be grajfed into their own olive tree.” In the R. V. graffed here gives place to grafted. In Acts xxvii. 40, where the A.V. has hoised : “They hoised up the mainsail to the wind and made toward shore,” the R. V. has : “ .Hoisting up the foresail to the wind, they made for the beach.” In Matt. xiii. 21, “He dureth for a while,” is altered to; “He endureth for a while,”

On the other hand, some archaic forms have been retained ; though substitutes more easily understood were available, Hale meaning haul has not been displaced. “ Saul haling men aud women committed them to prison,” Acts viii. 3. ; “ Give diligence to be quit of him, lest he hale thee to the judge,” Luke xii. 58. Haul or drag would have been more intelligible. In passing, attention may be called to the parallel, though vulgar, expression : “ lie was grilled for it, or he was pulled before the court.” In some places in the A. V., or is used in the sense of before. It is really the same word as ere, and as the first syllable in ear-ly. It would seem to have been better to spell it ere in Acts xxiii. 15 : “ We, or ever he come near, arc ready to slay him,” but it remains or in the R. V.

The word and followed by if in several places of the A. V. presents a difficulty which the inattentive reader perhaps will never observe, but by which many attentive, and not uninstructed, readers may be puzzled. “ But, and if that evil servant shall say in his heart,’ etc., Matt. xxiv. 48 ; “Doth this offend you ? What and if ye shall see the Son of Man ?” John vi. 61, 62. This combination of and if occurs also in at least three other passages, Luke xii. 45 ; 1 Cor. vii. 11 ; 1 Pet. iii.

14. We are assisted in understanding it, when we are told that and was formerly used in the sense of if. For example : “ Fortune is to be honoured and respected, and it bee but for her daughter’s confidence and reputation,” [Bacon, “ Essay of Fortune”). When it had this sense, it was often spelt an:An two men ride of a horse (that is one horse), one must ride behind.” (Shakespeare, Much Ado, iii. 5, 44.) An coupled with ¿/"occurs frequently in Shakespeare, for example :

.    .    . You may think my love was crafty love,

And call it cunning ; do, an if you will.

King John, Act iv. Scene 1. ; (also in 6th. Royal Reader, p. 344). In this and other passages which could be quoted, the two conditional conjunctions together seem to have no more force than either of them singly.

Dr. Abbott (Shakespearian Grammar, p. 75) cites some passages in which an if, or and if, as he prefers to write it, lias, in his opinion, the meaning of if indeed, but he expresses this opinion with some diffidence ; and in one of the passages (Richard III. iii., 1, 148) where he thinks it has this meaning, another great Shakespearian scholar, Mr. W. Aldis Wright, thinks the an is redundant, quoting the similar meaningless reduplications, “ or ere,” “ for because,” &c., to be found frequently in the works of him who was, perhaps, as truly the most careless of writers as he was the grandest of dramatists. Of the five places enumerated above in which and if appears in the A.V. the revisers have struck out the and iu three, namely, Matth. xxiv. 48 ; Luke xii. 45 ; John vi. 62 ; and they have retained it in the rest. They have moreover purposely inserted it in their translation of 2 Cor. iv. 3, where the A.V. has if only; they therefore have intended it to convey a definite meaning beyond that conveyed by if alone. To discover this meaning we have to look into the original, and we then find that the Greek which the revisers represent by this archaic and if means if indeed, if even. It may safely be asserted that not one reader in ten thousand will take this meaning out of the phrase and if in the R.V. The and will be regarded as the ordinary copulative conjunction having no meaning or having some unusual meaning which cannot easily be discovered. Would it not therefore have been better, if the archaism were to be retained, to retain it in the form an if, which would have been intelligible to most readers through its frequent occurrence in Shakespeare and in some well-known modern imitations of old poetry, for instance, Southey’s “ Well of St. Keyne ?” No doubt a better course still would have been to strike it out altogether and insert in its place if crun, or some equivalent phrase. This was the course adopted by the late learned Dean Alford, in the revised version of the New Testament which he published in the year 1870.

It is well known that the indefinite article an was originally the numeral ane or one, and that in course of time, through the inclination to ease the pronunciation, the n got rubbed off before a word commencing with a consonant. “ Ane kinges dohter ” came to. be written “ A king’s daughter.” But an continued to be written before words beginning with the aspirate long after it was shortened to a before other consonants. For example, wc have in the A.V., “ This is an hard saying,” John vi., 60 ; “ He is an hireling,” John x., 13 ; “ Make not rny Father’s house an house of merchandise,” John ii. 16, 17; “an hundredfold,” Matt. xiii. 23 ; and so in numerous other places. The practice of writing an before the aspirate was generally observed till

CONTENTS

The Growth of Intermediate

The Language of the Revised

Education in England ...

130

Version of New Testament

135

The Earl of Chesterfield’s

Leaders—

Scheme of Education ...

131

Department of Education ...

136

Hints to Teachers from Wise

Congress Consultations ...

137

Heads............

132

Notes of the Month ......

138

Earl Spencer’s Speech at Open-

Science and Art Gossip ...

139

ing of a New School at Brad-

St. Andrew’s University ...

140

ford ...... ......

133

Victorian Education Department—

Notes of a Lesson on Plants...

134

Appointments ......

141

Writing ... ... ... ...

134

Reviews, &c..........

141


ANSWERS TO CORRESPONDENTS.

Received.—“J. Castle,” “A. Hemmings,” “ Miss L. Newton,” “J. Hansford,” “ J. Sutcliffe,” “ T. Slattery, ” ‘ '• Robert Cole,” “ W. Porteous,” “ C. Shugg,” “ Miss M. Jackson,” “ G. Tilley.”

“S. S. Glass.”—June 30, 1881.

“C. Is.”—December 31, 1 SSI.

“J. S.”—June 30, 1882.


TO BE PUBLISHED FIRST WEEK IN APRIL.


PUNCH'S SOCIALITIES.


ALEX. M'KINLEY & CO., 61 QUEEN STREET.


NOTICES TO CORRESPONDENTS.

Advertisements and other business communications should be addressed to the Publishers. No advertisements will be inserted without a written order, or prepayment. It is particularly requested that they may be sent early in the month.

Books, music, and school appliances for notice, and all letters containing anything connected with the literary portion of the paper should be addressed To the Editor. Every communication accompanied by the name and address of the sender (as a guarantee of good faith, though not always for publication) will be acknowledged ; but we cannot attend to anonymous lottors.


INSTRUCTIONS TO SUBSCRIBERS.

Lady subscribers, when remitting their subscriptions, will please state whether their papers are to ho addressed Mrs. or Miss.

Subscribers will please send P. O. order or stamps, when remittance is under ¿£1.


Australasst an Sfljoclmasttr.

PUBLISHED EVERY MONTH.


lhe beginning of the present century. In Sir Walter Scott’s writings it is not wholly discarded. In Stanza xxii. of Canto ii. in the “Lady of the Lake,” he has both “a human tear” and “an hero’s eye,” In stanza 21 of Canto vi.. he has “ an herald’s voice” ; in stanza 3 of Canto i, he writes :—

“A hundred dogs bay’d deep and strong,

Clattered a hundred steeds along.”

In stanza 3, of Canto v., he writes:—

An hundred men might hold the post,

With hardihood, against a host.”

Even Macaulay, in one of his letters, wrote :—“ An hundredfold,” (“ Life by Trevelyan,” small edition, Yol. ii., p. 36-1) ; while in one of his essays he wrote, “ Our wealth has increased a hundredfold,” (“Essay on Southey’s Colloquies”). It is now established as correct to use an, not a, before all words commencing with the aspirate, unless the first syllable is unaccented, in which case we may use either a or an. To be in accordance with the best usage we must say “ a history,” “ a habit,” “a hundredfold,” but we may say either “an historical work,” or “a historical work,” “ an habitual act,” or “a habitual act.” Macaulay certainly, in these cases, prefers a in his essays ; for example, he has “a historian,” “ a historical novel,” “ a heroic poem,” (Essays in one vol., p.p. 306, 61, 282); but Mr. Matthew Arnold writes, “an historical sense,” and in a passage quoted by him from the “ Dublin Eeview ” there occurs the phrase, “anhabitual consciousness,” (‘'Essays in Criticism,” pp. 68 and 225).

The an before the accented aspirate in the A.V. has been changed for a in the R.V. ; “ a hard saying,” “ a hireling,” and so on. But I have not in the latter met with a case of either a or an before the unaccented aspirate.

The A.V. uses«» before cu, sounded you, as “an eunuch of great authority” (Acts viii. 27); it sometimes has “such a one,” and sometimes “such an one” (2 Cor. ii. 7 ; Philem. i. 9); in all these places the R.V, has a. The usage of good writers in this respect is not quite settled. Macaulay writes, “an usurpation,” “an useless display.” (Essays pp. 107,110). In a very recent work in the English Men of Letters series, “ Locke, by Mr. Fowler,” the author of Fowler’s Logic, we find the phrase“«» Unitarian” (p. 69). But the prevailing practice in such cases is to use a.

The same cause that led to an being retained before vowels after being shortened to a before consonants, continued the use of mine and thine before vowelsafter they had been shortened to my and thy before consonants. In the A.V., for example, we have in Acts x. 4, “ Thy prayers and thine alms are come up for a memorial before God.” ¡So “my brethren” but “ mine enemies,” (Matt. xii. 48; Luke xix. 27). This usage is adhered to in the R. V. also, and is still partially observed by the poets though obsolete in prose.

Before the aspirate sometimes mine and thine, and sometimes my and thy are used in the A. V. For example, “ the thought of thine heart,” in Acts viii. 22 ; and “thy heart is not right,” in verse 21 of the same chapter; in John xx. 27, “ my hands,” but in Rom. xvi. 23, “ Gaius, “mine host;” in Acts iv. 28, “ thy hand,” but in v. 30 of the same chapter “thine hand.” Most of such discrepancies the revisers appear to have expunged by putting my and thy in place of mine and thine before the aspirate, as “my host,” “thy heart,” “thy hand,” etc.; but, owing apparently to negligence, they have not attained complete uniformity on the point; for we still find in John ii. 17, “The zeal of thine house,” while in Acts xi. LI, we read, “Thou and all thy house,”

The form an hungred, occurring several times in the A. V., is retained in the R. V. borne few may need to be cautioned against supposing the an here to be the article an. This view might be induced by supposing that “ He was an hungred ” is an ellipsis for “ He was an hungred person, ” (M att. iv. 2); but this supposition is seen to be untenable when we read, “His disciples were an hungred,” Matt. xii. 1. Dr. Abbott considers this an to be a form of on, used with an intensive force before participal adjectives, (“ Shakespearian Grammar,” p. 34). To avoid its confusion with the article, the revisers would have done well to put a hyphen after it, thus : “ au-hungred.” A better course still would have been to replace this archaic compound by the simple adjective hungry.

[This subject will be continued, from time to time, according as the writer has leisure.]

Tub new theory propounded by Mr, C. H. Darwin, of the influence and importance of the tides with regard to geology, is creating a stir amongst scientists at home. Part of this theory is thus expressed by Mr. Darwin :—“ The more rapid alteration of day and night (in bygone ages) would probably lead to more violent storms and the increased rotation of the earth would augment the violence of the trade winds, which, in their turn, would affect oceanic currents. Thus there would result an acceleration of geological action.” In an old paper, published by Immatmal Ivaint, in 1754, on the subject whether the length of the day has altered, says, “ If the earth were a solid mass without any liquid the attraction of the suu and moon would not alter the rate of rotation round the axis. ... If, however, the mass of a planet includes a considerable amount of liquid, the united attractions of the sun and moon, by moving this liquid, impress upon the earth a part of the vibrations thus produced. The earth is in this condition.” He proves that the moon has the greatest effect, with a tide in opposition to the rotation of the earth, and states, “ we have here a cause, on which we can count with certainty, incessantly reducing the rotation.”

MELBO URNE, MARCH, 1882. State school teachers have all along averred that whatever distrust of the Education Department existed within their ranks arose principally from two sources. First, the undue weight allowed to political patronage ; and secondly, the keeping of a secret register, or “Black book,” containing the private opinions of Inspectors upon the character and qualifications of each teacher. It was freely conceded that the ability and integrity of the Secretary and the responsible officers of the Department were unimpeachable. But it was contended that so long as their decisions were being continually overruled by the Minister of Public Instruction, and their judgment was biassed by the secrets of the “ Black book,” perfect confidence could not be imposed in the impartiality of appointments and promotions made, or in the punishments inflicted, by the Department. That the teachers had good grounds for the position they thus took up must be admitted by all who have read the press imports of the proceedings of the Commission now engaged in examining into the working of the Education Act. The only gentlemen yet examined by the Commission have been the Secretary of the Department, G. W. Brown, Esq.,

M.A., and the Inspector-General, J. Bolam, Esq., M.A. Yet from the testimony borne by these gentlemen alone a mass of evidence has been educed which clearly establishes the baneful effects of political patronage over the Department. Whenever a gross injustice has been perpetrated against the teachers, whether it be in the matter of promotions or in the mode of dealing with offences, it was shown to have arisen from the undue interference of the political head of the Department for the

time being. Prominent amongst the instances of Ministerial mal-administration stand the cases of moral delinquency. Now, while we are glad to find that not only the Commissioners, but the whole press of the colony are unanimous in condemning the leniency with which the Department have dealt with these cases in the past, and in demanding that, in future, no amount of political patronage shall avail teachers who tamper with the virtue of the youth placed under their charge, or whose self-command is insufficient to the maintenance of sobriety,    we must point out that the mere fact of

the chairman of the Commission grouping all the cases of proved and imputed immorality together has left an exaggerated impression on the public mind. The instances cited by the chairman extend over a very large number of years, and most of them are those of teachers who were transferred to the Education Department from the Denominational and National School Boards when the present Act came into force. It is universally admitted that no profession and no considerable body of men can be named that is without its “ black sheep.” When, then, the number of teachers serving under the Education Department is duly considered, we venture to say that the instances of moral delinquency are as few, if not fewer, than those to be met with in other professional walks of life. The Commission have, however, done the State school teachers a service by bringing prominently before the public the dangers to which our school system is exposed in this respect. The interest of every honourable and qualified teacher is involved in the expulsion of the immoral and the incompetent from the Department. Sympathy for the family of a delinquent must in no instance be allowed to override the claims of the children and the honour of the profession. With regard to the “ Black book,” the admission made by the Secretary of the department—that the disparaging remarks of an Inspector entered therein against the teacher referred to by the Chairman of the Commission, was made against the wrong teacher, and that the error was not discovered till the case had been brought before the Commission—ought of itself to be sufficient to condemn the practic e of keeping such a secret book. Who can say how many teachers have been refused merited promotion through a similarly damning note having been entered in error against them in this same record 1 The system which prompted the opening of this book was essentially a vicious one, and we trust that one of the results of the holding of the present Commission of enquiry, will be to make an entire change in the system under which it is kept. No doubt it is necessary that Inspectors should report to the Department the opinions they have from time to time formed as to the character and qualifications of the teachers, and that a record of such opinions should be kept for reference ; but every teacher ought either to be furnished with a copy of each entry against him, or to be allowed access to the official record upon himself, if he at any time desires. In a free and democratic community like this, the espionage involved in the keeping of a secret record book against the teachers such as the one referred to is an anachronism. In the working of the Education Department there is no need of a Star Chamber. Every teacher should know what his official status is in the Department. So far as the enquiry of the Commission has yet proceeded, it has only tended to strengthen the opinion we have before expressed, that the Legislature must step in and free the Department from the demoralizing influence of political patronage, or our education system will be destroyed.

CONGRESS CONSULTATIONS.

If unity is strength, then teachers ought to be strong. Their strength, as measured by their readiness to unite, has been on the increase within the last few years. Every annual gathering lias witnessed a great strengthening of the bonds which unite the elementary teachers of the United Kingdom, The N. U. E. T. in England has progressed with steady pace ; the Educational Institute has gradually widened its borders ; and the Irish organisation has been coming much nearer to the time when all its ways shall be those of pleasantness, and all its paths be peace. The necessity which is unruled by laws must

account, in great measure, for this increase in number and determination. The public attention has to be attracted, and only in the aggregate can the teachers hope to make their voices heard with due effect. By means of School Boards the country exercises a much greater influence for good or evil upon the teacher’s fate than in the days when voluntary agencies were the chief rulers of the schools. To help the members of these Boards in the proper discharge of their duties, and to resist the infliction of any unjust regulations, it becomes an increasing necessity that teachers should be united. They have the common rights of citizenship. It is only discharging a simple duty to themselves to speak freely of the tilings which concern their immediate interest, and about which their experience enables them to speak with the voice of authority. We are glad to observe, at the beginning of another year, that this combination, and this confident appeal to the public through its several channels of communication, is receiving its due reward in the way of attention. The Congress of teachers in Edinburgh has been well reported by the general press, and the same thing is, to a certain extent, to be recorded of the gathering in Dublin. The two meetings were not exactly of a similar kind. The Dublin meeting concerned itself with many business details, while the Edinburgh Congress was chiefly occupied in discussing a variety of purely professional subjects, or those which appealed directly to the public interest. The meetings of the one were to a certain degree private, while in the other case the gathering was open to all who chose to enter.

Much more than meets the eye is accomplished at these annual gatherings by the deputations who carry good wishes and the interchanges of opinion from one part of the kingdom to the other. The public record of what they succeed in doing is always imperfect, and the better part of their work is unseen and unperceived by tlie superficial observer. In private conclaves, and in the social intercourse which accompanies the discharge of public business, they are able to arrive more rapidly at the exact state of feeling among the brethren of other districts. They hear and can be hoard regarding much which it is altogether impolitic to discuss in public meetings. The conversazione, which now forms an important part of some of these annual gatherings, has a professional and educational value which transcends by far any social influence it may exercise. We feel assured that the better part of the work which was accomplished during the recent holiday season by Messrs. Grove and Sykes in the North, and by Messrs. Heller and Greenwood in the sister isle, has been that which was done altogether apart from the public and reported speechifying. It is clear that the present system, which finds favour with the educational authorities, regarding the distribution of the annual grants, is condemned by the overwhelming majority of the teachers of Great Britain and Ireland. They know their business, and these friend ly interviews enable them to compare their opinions and to discuss them for their mutual advantage. The advice they offer is not to be lightly passed over, and the lesson of these Congress gatherings is encouraging to those who are striving to extend the work of professional unity. The influence is not to be measured by the mere attendance at the meetings, but by the number who are represented, and the extent to which the public has its interest excited. Sooner or later the country adopts the course which is best fitted for its welfare, and the advice of men who have so much to do with the formation of the character of successive generations cannot be without its increasing value. At this particular crisis of the educational world, when the golden opportunity is at hand to amend a system which abounds in error, the people and their representatives have received the opinions and the advice of teachers. If their remonstrances against the system which is now demoralising both the teachers and the taught be otherwise than successful, it will be a reproach to the Department and a grievous injury to the cause of Education.

It was only natural to expect that some part of the Scotch Congress should be devoted to the subject of Higher Education and its kindred subject of Endowments. There has never been the sharp distinction between the elementary and the secondary course of education in Scotland which has, unhappily, prevailed so long in England. The means were at hand in almost the humblest school to give the poorest scholar the opportunity of

|to,its    of .% glonilj.


improving himself in higher subjects if he had the ability and the desire. By means of bursaries the clever lads were able also to find their way to a higher class of school—to the universities themselves. The endowments which were left for this purpose have been carefully managed, and it was only a just thing on the part of Dr. Campbell to claim, in the course of the discussion, that “ there was very little of what might be called great abuse of educational bequests such as bad been dealt with in England.” What is needed now is a careful revision of the original bequests to bring them into harmony with the spirit of the times in which we live. To respect the original intentions of the pious founder is a duty which should be the first consideration in the re-adjustment of any educational bequest, but there need be no hesitation in altering the letter of the law where it is altogether inconsistent with the wishes orrequirementsof the world as it is. Tbe Education Act has now made the work of popular education a national duty, and it is no longer necessary, therefore, to spend the money which was left in various districts for that particular purpose. If the money was left for the benefit of a special class, however, it ought in simple justice to be still applied for their advantage, To pay fees for the struggling poor who are unwilling to rank themselves with paupers, and to provide opportunities for an extended course of instruction to those who give promise of intellectual superiority, are ways in which the wishes of the dead can still be realised. To provide scholarships open to the competition of the deserving poor is a much better way in which to spend the educational bequests which were designed for the needy, than to hand over the income for the benefit of a secondary institution. If technical schools and institutions of a higher order are required to meet the wants of modern times, the expense of their maintenance should be defrayed by those who need them. If they are for the advantage of the country at large or a particular section of the community, it is from either of these that the necessary funds should be derived. To help those who are struggling upwards is the legitimate object of an educational endowment, and we are glad, therefore, to perceive from the reports of the discussion at the Edinburgh Congress that it is in this direction that the movement for an alteration of the law is likely to meet with the popular and parliamentary support.

Mr. E. E. Morris, who has for some time past held the position of head-master of the Church of England Grammar School, Melbourne, has resigned that place. He will hold the position pending the appointment of a successor.

The Education Commission met on Thursday, and the examination of Mr. J. Bolam was postponed in order that Mr, Brown might offer a few remarks with regard to the teachers’ record book. He also gave the following facts :—The proportion of children under six years of age was slightly larger in private than in State schools. There was 12 per cent, more of the children attending the State schools in 1SS0 than in 1872. Boards of advice were not looked upon as necessary evils.

Ax a meeting of the senate of the University of Melbourne, held on Wednesday last, an election was held of a member of the council in place of Mr. Rogers, Q.C., resigned. The candidates were Professor M'Coy, nominated by the very Rev. Dean Macartney and the Hon. H. J. Wrixon ; and Professor Strong, M.A., nominated by the Bishop of Melbourne and Dr. Alexander Morrison. The result was—For Professor M'Coy, 95; for Professor Strong, 54. Professor Andrew thanked the senate for having at last meeting elected him a member of the council.

IN commenting on the proposals and the suggestions made for their amendment by the N. U. E. T. and the Educational Institute, the Scotsman has something to say iu regard to the request to limit the percentage to 75 or 80 per cent, so as to secure the maximum grant. It argues as follows :—“If weight were given to that consideration, one of the chief ends of payment on results would be defeated. The teacher would lose nothing by neglecting the slow and backward scholars, and he would have no inducement to attend to.them. That would not be a healthy or a desirable arrangement. It would tend to bring back the old and very objectional state of things that prevailed before the New Code was introduced, when school masters devoted all their energies to the clover scholars, whom it was easy and interesting to teach, and neglected the laggards.” To this we reply that the contention of the overwhelming majority of teachers and educationists (not sham, but real,) is against the system of payment by results, and it is on this ground that we have contended in their name for its absolute abolition, or such a modification as will enable school work to be done iu a better way than now. It is not from selfish nor from sordid motives that teachers are thus advisiug the Department. They know their business

better than leader-writers in the Scotsman can, aud they are offering their advice on educational grounds with which they are familiar. If the Scotsman knew the subject thoroughly with which it professes to deal, there would have been a prompt and decisive editorial excision of the nonsense contained in the concluding part of our quotation.

The University of London has taken an important step in resolving to confer a new degree, to be called the “Teachers’ Diploma.” It is to be under the Seal of the University, and signed by the Chancellor. It is to be delivered at the Public Presentation for Degrees to each candidate who has passed a successful examination. No one will be examined, however, who has not already graduated. The examination will test the practical ability, as well as the information of the candidates, and will combine a close scrutiny into the knowledge regarding the theory of the art of teachings. These diplomas will be certificates of merit of the highest order, and their possession will no doubt be greatly advantageous to those who gain them. Another peak has been provided in the mountain system of the educational world, and we trust that many of those who are already connected with the London University will prepare themselves for the ascent.

The programme of technological examination in connection with the city and Guilds of London Institute for the year 1881-82 has just been issued. It contains a number of new syllabuses of subject, such as electric lighting and transmission of power; coal-tar distilling and spirit rectification; bread making ; the manufacture of linen; liuen bleaching, dyeing, and printing ; weaving and pattern designing. A new feature in this programme is the grouping together of allied industries, and so arranging the syllabuses of instruction that subjects which are common to two or more branches of any industry are put under one heading. This arrangement further commends itself as offering suggestions for a curriculum of study to be pursued in such technical colleges as those of Bradford, Leeds, Huddersfield, &c., which are now being erected for the development of trade in those districts. There appears to be a considerable increase in the number of registered teachers of the Institute, showing a satisfactory development in this part of its work. The conditions of the registration of teachers become much more stringent after March, 1882. Hitherto, owing to the lack of practical teachers, combining a knowledge of the principles of science and of the practice of trades, the teachers registered under the Science and Art Department have been permitted to hold courses in technology. After March next, it appears that those only who have obtained honours at the Institute’s examination in the subject to be taught will be registered a3 teachers.

The following works have been decided upon and tenders have already been called for them :—1. Removal of wooden building from Kororoit to school No. 1933, Digger’s Rest. 2. Wooden building for school No. 1934, Tarkedia. 3. Hardwood building for school at Jannasba South.

4. Stone building, for school No 1090, at Sorrento ; 5. Wooden building for school No. 213, Numurkah.

Stephen Lade is announced in the Government Gazette, for March 8th., as having been appointed returning officer for the school-district of Byland, Glenbumie, and Willowmain, No. 271, in place of B. M'Manus who has resigned.

Messrs. W. and R. Chambers have added to their list of school books a collection of about two hundred sentences in prose and poetry for exercises in analysis. The sentences are of various degrees of difficulty, some being of the simplest kind, and others selected to suit the most advanced students. This book must necessarily be of the greatest advantage both to teachers and scholars, and will save a degree of unprofitable work.

A new series of copy books has recently been published by Messrs. Longmans, Green & Co. of London. This series is intended to prevent children (as far as is possible) from copying from their own bad writing, which gets worse and worse at each line, and also to free them from the inconvenience of the sliding copy slips. The first head line is printed dark, but in each alternate line after that, there is a pencilled outline which the pupil is expected to fill in, and which will supply the copy for the next line.

Moffat’s Explanatory Reader recently published, contains a series of interesting and instructive lessons. It is illustrated.

The new “ Historical Reader,” published by Collins, “gives an outline of English History from the accession of Henry VII. up to the present time.”

The statements made by J, L. Bashford, M.A., in his book on the Elementary Education iu Saxony, must be peculiarly interesting to Victorians. He states that the religion prevailing in a parish is the religion taught in the principal school of that parish, but provision is made for the minority which cannot establish a school of its own. In this case, the children of the minority in religion are given a secular education at the parish school, and the religious education is left to their sect. They may also be taught an undenominational education until they are twelve years of age, when they are compelled either to be confirmed or take their religious training on themselves. “ All patronage in the hands of ecclesiastical bodies or private persons is taken away from them and handed over to the State.” The schools are good. The staff of masters and mistresses is not only capable of performing well the work placed in their hands, but is heart and soul devoted to it.

The Bishop of Carlisle, one of a deputation from the National Society who waited upon the Ministers of the Imperial Parliament, said : “ It was monstrous that children of eleven or twelve years of age should be taught about the conservation of energy, before they knew the first principles of mechanics.”

Mr. Bright, at the laying of the foundation stone of a new Board school, in the course of his remarks, said : “ We have had universities, it is true, and two universities with enormous wealth, a wealth which, I believe, has beeu very lavishly and wastefully employed. We have had these universities, but they have given advantages, such as they are,

Sciente ìxrùs %xi


Professor Morton has called attention to the danger of fire arising from the use of electricity. “ When naked wires are used as conductors, and when both are, as is sometimes the case, merely nailed or stapled to wall or floor side by side, there is a great chance that some stray scrap of wire, a falling nail or pin, may short-circuit the line and become red-hot in an iustant. Loose wires are again a source of danger as they may be momentarily short-circuited, and arcs set up of a dangerous nature at the point of contact. Moreover, the lamps themselves arc not


mainly to the rich . .    . The grants to School Boards in England have

been ¿10,739,000, of which, London alone has had £3,885,000. Now, I think that is, after all, a little bit of light thrown on the gloomy picture which I have j ust been painting. These grants are not made for the purpose of educating the rich, or what are called well-off people. Now, every working-man, every father and mother of a family, to whom the grant will be a great concession, and a great blessing in regard to the education of their children, should bear this in mind, that this grant—of course they pay their share, because we pay taxes—is given with the express object, not of educating the rich, but of educating those who live by the labour of their hauds and upon their weekly wages.” He went on to tell how education had almost extinguished drunkenness in Saxony,

The following motion, which was proposed at the London School Board, was denounced as illegal by the solicitor of the board, and was therefore withdrawn: “That the last use granted on good report to the headteachers of the Render Street, Hatcham, school be cancelled, on the ground that no entries were made in the punishment book, while punishment was administered, and that the fact was not known to the Board when the use was granted.”

In his last official report on reformatories and industrial schools, the Rev. Sydney Turner, the inspector, says “ The third distinguishing feature of the English system, which I regard as one of the keystones of its success, has been that, while assisted and superintended by the State, the schools are essentially conducted and controlled by voluntary management, and have throughout retained an independent aud partially charitable character.”    _

A writer in the Contemporary Review, says :—“ The youth receiving a higher education must, indeed, know the Latin language, but mainly as a key to open the treasures of thought and wisdom that lie within the Latin writers, and Roman history and institutions. But if Latin requires to be known for this reason, much more does Greek, Greek literature is the freshest, the most origiual, the most stimulating of all literatures. And the Greeks tried endless experiments in political organizations, and threw their genius into every form in which the beautiful can be embodied. But for similar reasons the languages of modern nations, aud especially one’s own language, must not be neglected.”

The report of the Chancellor of the University of New Zealand says : — “ Since last session two vacancies have occurred in the senate. One of these was, I regret to say, caused by the death of the Rev. W. Johnstone. I am sure that the senate will agree with me in deploring the loss of a colleague whose constant attention at, and unremitting attention to, the business of the session evinced his very great desire to promote the welfare of the University. The other vacancy was caused by the resignation of the Rev. Dr. Stuart. Two new fellows have been appointed by the Governor, viz., Professor M'Gregor and the Rev. J. W. Salmond.”

At a meeting of the New Zealand University it was poiuted out that the University would have nothing to do with the examinations for the Gilchrist Scholarships, which are offered for students of the University. There is a strict rule forbidding the examinations to be interfered with by any but the Governor of the Colony.

The New Zealand University senate have passed a resolution to include in their regulations for admission “ ad eundem ” the Royal Irish University, the Victoria University, and the University of Adelaide. The number of undergraduates in this University is now 233.

The Board of Education, New Zealand, have accepted the following tenders for works in connection with the schools:—1. Improvements to teacher’s dwelling at Paterangi. 2; Erection of teacher’s dwelling at Onehunga, £519.    3. Grafton Road school, £1119.

At a meeting of the Board of Education, New Zealand, Mr, Moat said that some teachers did not send up all their scholars for examination. This was getting common in towns, and would soon spread to the country districts, and was likely to be a serious trouble. He thought the matter should be brought under the notice of the board.

A protest has been sent in against the election for members of school board at Manga war’s (N.Z. ) on the ground that two persons who voted do not reside in the district, and a third did not live in the house by virtue of which he voted.

The Board of Education of New Zealand have decided that the purchase of “ toys” for the use of children is illegal. The committee for Te Mata asked for “ something to amuse the children in the dinner hour.” It was resolved that they be requested to specify what they wanted.

The death is announced of Sir Charles Wyville Thomson, L.L.D., who was at the head of the scientific department of the Challenger deep-sea exploring expedition between 1872 and 1876. He was 61 years of age.

The last of Mr. J. Lakes’ dramatic recitals was given in the Athenmum on Saturday evening.

The following are the results of the recent examinations held by the Inspectors of the Education Department:—For certificate of competency, 179 passed and 471 failed ; for pupil teachers, class I., 115 passed, 988 failed ; class II., 151 passed and 97 failed, and class III., 150 passed and 123 failed. For admission to the second year’s training, 27 passed and 27 failed.

free from danger if so constructed that fragments of red-hot carbon can fall from them, as was the case not many months ago with one of Siemens’ lamps in the reading-room of the British Museum.”

Nature, in an article on “Little Electrometers,” says:— ‘ It is said that three of James’ accumulators weighing 501bs. each may, when fully charged, drive a sewing machine by a Trouve motor for a whole week, workiug five or six hours ever}' day. Motors similar to this have been fixed by Mr. Trouve in his little electric canoe, and are suggested by M. Tissandre for balloon steering. The favourite motor, however, at the present time, appears to be that of Guscom, an American electrician. This elegant little machine is only 4.) inches long, and weighs a little over two pounds only. But it is remarkably powerful and steady in its action. It can, when fed with a current of requisite strength, rotate at a speed of 500 revolutions a minute, and in that time will do from 22-26 foot pounds of work. The construction is extremely simple. There is a Siemens’ armature on a horizontal axis, within, and entirely surrounded by, the field electro magnet, which not only serves to produce a powerful magnetic field, but also acts as a rigid framework for the rotating parts, which is thus protected from injury. The contact of the wires of the circuit with the commutator is made by two springs with little metallic friction rollers at the end. The iron work is made of malleable cast iron, aud so combines the advantage of high magnetic power and of cheap production. ”

A COURSE of scientific lectures was lately delivered by Mr. Laitt Carpenter, under direction of the trustees of the Gilchrist Education Trust, in five Lancashire towns. The interest taken in these matters now amongst the whole population of the land is shown by the fact that the total audiences, chiefly composed of artizans, was more than 4000 per week, the same people going night after night. Some of the most interesting lectures were on the storage of energy and the electrical transmission of power.

It is proposed to open an electrical exhibition in St. Petersburg.

A volume of considerable interest has recently been published by Fuedenscher and Co., of Hamburg:—“Dr. Ludwig Leichardt's Buhfe an Seine Angehörigem,” edited by Dr. G. Neumayer and Otto Leichardt, a nephew of the unfortunate Australian explorer. These letters are of special interest at present, when rumours come from Australia that the journal and other traces of Leichardt have at last been found. The letters extend from 1834 (Göttingen) to April 3, 1848 (Macpherson’s Station, Fitzroy Downs). These letters give one a high opinion of Leichardt’s qualifications for the work of exploration. He had an excellent education, not only at home, but during lengthened residences in London and Paris. He had a strong love for natural science, was a shrewd and accurate observer, aud a writer of considerable graphic power. His account of life in London and Paris is decidedly interesting, and his letters from Australia during his exploring work lead one to feel that the death of the writer was a real loss to science. These letters were quite worth publishing. Nature gives a long paper by Dr. Neumayer on Leichardt as a naturalist and explorer, in which the writer justly gives a high estimation of his qualifications and character.

Mr. E. C. Hore read a paper to the Geographical Society of London on Lake Tanganyika, furnishing two maps which for the first time gives the form with any accuracy as a whole.

The reconstruction of the Tay Bridge is contemplated by Mr. W. H. Barlow, and he intends to take special precaution against destruction by high wind pressures. He has provided three lines of defence, firstly, a guard baulk of considerable height outside each rail ; secondly, a ballasted floor of sufficient strength to hold up a de-railed locomotive at auy point; and thirdly, a strong iron parapet.

A scientific commission has been appointed to determine the measures which ought to be taken immediately for the protection of the public in theatres. All of them which will not comply immediately with the provisions of the law will be closed. It was proposed at the Municipal Council that the electric incandescent light should be deemed obligatory for all municipal theatres.

The London Missionary Society’s chronicle publishes a map of Southeastern New Guinea, with the discoveries of Messrs. Chalmers, Macfarlan and Berwick marked.

Dr. James Gerkie, in the British Trade Journal, says :—“ The Gulf stream and Panama Canal will have as much effect on the Gulf stream and the climate of Northern Europe as the emptying of. a teapotful of boiling water into the Arctic Ocean would have in raising the annual temperature of Greenland.”

Phylloxera has appeared in Sicily, and is making wide ravages. The vines in the district of Messina and Callamssctta have already been almost destroyed.

Sir William Armstrong, in an address to the Institution of Civil Engineers, on our national defences,” says : — “ Where are there to be found amongst trading or passenger steamers, vessels passing a speed of sixteen knots, and engines and boilers below the water level, and having an underwater deck to save them from sinking when penetrated at or below the water line? From his own experience he knew how difficult it was to adapt mercantile vessels to the purpose of war, and how unsatisfactory they were when the best had been made of them.” Further he stated, “ Our navy was at present armed with guns which could not be expected to contend successfully with the best modern guns that could be brought against them. Our service guns had simply been overtaken in that rapid progress of artillery, which had been going on for the last eight or ten years. In the meantime, no expense should be spared in judicious experiments, seeing that the expense of experiments was trifling in comparison with that of mistakes.”

The Geographical Society have now on exhibition a relief map of the Equatorial region of Africa, constructed within 12 months for Colonel Grant by James B, Gordan.

Telephonic communication has been established betvveen two towns in Russia Turkestan.

The Chinese authorities of Shanghai have determined “ that it is the duty of all physicians to use their knowledge for the benefit of the people ; when people are sick, they must be ready to attend upon them whenever they are sent for, without regarding the hour of night or day, or the state of the weather.”

The question as to whether birds migrate by night has been solved by Mr. E, D. Scott, who, whilst observing, with a large telescope, the moon, his attention wa9 attracted to some small birds, amongst which he recognised warblers, finches, wood-peckers, and blackbirds. They were mostly flying from north-east to south-west, and they soared at the vast height of 10,000 feet, from which, probably, they had a view of various landmarks, which helped them to take their course.

Professor J. Lawrence Smith, of Louisville, ('Ey,) says -.— “Although I have probably examined more microscopic plates offrag-ments of meteorites than any other person, still I have never discovered anything like organic remains in any of them, Besides, the well known chemical composition of these bodies is averse to the existence of such remains as spoken of by Professor Halin.”

Electricity is now used in mining operations in the Loire valley for transmission of force. At the works of the Compagnie de la Ferronicre, two connected Gramme machines are employed to work a drum which stands at the head of an inclined plane 115 metres long, and having a gradient of 0.40 ctm. per metre. The generator is 1200 metres from the drum, and above ground. The load raised is about 800 kilogrammes, and the ascent is accomplished in a minute and a half. The work goes on with great regularity. The starting and stopping can be done as gently as desired. By means of a galvanoscope in the circuit, one can tell at any moment what is being done at the incline, whether the work is stopped, whether the waggons are coming up filled or not, etc.

Dr. ¡VPGowan has discovered that colour blindness is almost, if not altogether, unknown to Asiatics, although it is very prevalent amongst Europeans.

Professor W. Grylls Adams, of King’s College, gives the following theory with regard to the Aurora “ The earth’s atmosphere is drawn towards the sun, and the friction produced generates a supply of positive electricity in the air, and of uegative electricity in this globe. When the air is charged up to the discharging point, a gradual transfer of positive electricity from the poles towards the equator takes place, and auroral phenomena result, varying iu accordance with the genuity of the region, and with the temperature and moisture of the air.

The intensity of the sun’s light at the surface of the sun is calculated to be 3'4 times that of the brightest electric arc yet obtained.

Russian papers state that on December 22, 1881, at 11.20 p.m., a meteor, spreading an intense violet light, and the fall of which was accompanied by a strong explosion, was seen at Byejetsk, in the Government of Tver, and at the village Nasilovo, 27 miles distant from that town.


ST. ANDREW’S UNIVERSITY.

INSTALLATION OF SIR THEODORE MARTIN AS LORD RECTOR.

The installation of Sir Theodore Martin, Iv.C.B., to the office of Lord Rector of the University of St. Andrew’s, took place on November 22nd, in the Old Parliament Hall, in presence of a large assemblage of spectators. Previous to the ceremony, the students, who were assigned the front benches in the place of meeting, amused themselves by singing a number of topical songs. At two o’clock, Sir Theodore Martin, accompanied by Principal Tulloch, Principal Shairp, and the Professors of the University, Sir Robert Anstruther, Lord-Lieutenant of the county ; Mr. J. W. Melville, Convener of the county; Mr. Stephen Williamson, M.P. for the St. Andrew’s Burghs ; Mr. Stuart Grace, Sec,, entered the hall, and was received by the students with enthusiastic cheers. On quietness being restored, Principal Tulloch, who is ViceChancellor of the University, presided, and opened the proceedings with a Latin prayer. After reading the certificate returning Sir Theodore Martin at the head of the poll, he expressed the congratulations of the students and the Senatus on Sir Theodore Martin’s accession to office, and wished him all happiness in the discharge of his duties. (Cheers.)

Sir Theodore Martin, who was received with loud cheers, then proceeded with his address. He said :—

“ Gentlemen, as you make your way through life, you will probably find that very many of the things that give you most pleasure—that sometimes even bring you much permanent happiness—are things to which you have not only not looked forward, but which have come upon you with all the suddenness of surprise. To reach the goal of some cherished ambitiou does not always bring the satisfaction we have hoped for. Success may come too late, or it may be bought at too dear an expenditure of energy or of health. But if the aim has been worthy and worthily pursued, the years of strenuous and well-calculated effort which have been directed to attain it will surely be cheered by signs from unexpected quarters-that your work has not been in vain ; that you have made friends where you looked for none ; that in some way unknown to, nay, undreamed of by yourself, you have created in others that sympathetic interest which is at once the most precious reward for labours past, and the strongest incitement to future endeavour. In my own person, gentlemen, I have often had occasion to acknowledge the truth of what I have just said, but never more than in the circumstance to which 1 owe the honour of now addressing you from this place. When a student like yourselves, I had my ambitious dreams. What they were it would be hard for me now to recall, when the current of my life has run iu channels far different from what I then anticipated, and better things have befallen me than I could even have imagined. But if such


an aspiration could then have entered my head as possible, nothing could have kindled a warmer thrill within my heart than the thought that I should some day be elected as their Rector—spontaneously and cordially as I have been—by such a body of my young countrymen as yourselves. As it would have been then, so it is now. I heard of the intention to nominate me for that honourable distinction with a pleased surprise, for it told me—what I had never dreamed—that the younger spirits of my dear native land thought that I had done something in my day and generation not unworthy of their race. When I learned that I had been chosen, in competition with a rival whose pre-eminence in the studies he has made his own none can recognise more frankly than myself, my pleasure was mingled with pride to find that I had so many unknown friends among the young and ardent, who were being trained, as I had myself been trained, and who were equipping themselves to fight the battle of life upon the same lines upon which I had myself fought it. But, gentlemen, my pride was mingled with a strong feeling of my own unworthiness to occupy a place which has been filled by men of gifts so varied and distinguished. Believe me, I say this in no spirit of false humility. To learn, not to teach, has always been the attitude of my life ; and to more than one of these men I have long been accustomed to look up with gratitude for instruction and for guidance. I have had the happiness of knowing some of them as friends, of taking sweet counsel with them in their homes as well as in their books. Nor will you think it out of place if I say here that, among the many gratifying circumstances associated with my election as your Rector, not the least is the remembrance that the very first to offer me congratulation was one whose name will always, I am sure, be held in high honour in this University, but whose voice, alas! it will no more hear—the late Dean of Westminster. Yes, we shall hear his voice no more, and how great is that loss to those who knew and loved him it would be difficult to estimate ; but the large, generous soul that gave a charm to the fine lines of that expressive mouth, and spoke in the eager, penetrating, kindly glance of those sympathetic eyes, lives on in his books. It lives, too, in the influence which his spirit and character wrought upon those among whom he moved and worked, and, through them, will carry on the great purpose of his life, to break down the futile distinction which separate men into schools, and sects, and parties, and to bind them together in the ties of Christian brotherhood, as befits us short-lived “ travellers between life and death”—the children of a common Father, the wayfarers towards a common goal. Coming after such men as Arthur Stanley, as Froude, as John Stuart Mill—men whose lives have been devoted to the study of history, of human progress, of the great questions that bear upon the welfare of men here and hereafter—how can I hope to engage your attention, or to say anything that you will regard as a word in season. A crowded life of* hard professional work has left me little leisure for

The search of deep philosophy,

Wit, eloquence, and poetry.”

I have been well content if only I might “enjoy the things which others understand,” and the longer I live the more do I incline to the silence of the humble student, growing, as I do every day, more and more conscious of the perplexities which, in these days of change and restless inquiry, surround nearly every question that concerns the social and political well-being of our race. I cannot, therefore, hope to say anything very new to you ; or, rather, I should say, to put any old truths— for all great truths are old—before you in a new and striking way. But I am sure you will bear with what I may have to say for the sake of the sincerity and goodwill with which it is offered.

NECESSITY OF BODILY AND MENTAL VIGOUR.

Gentlemen, it seems to me that never at any period of our country’s history was it more necessary that young men, on entering into the active life for which you are now preparing, should bring with them not only a clear conception of what they mean to aim at there, but also minds and bodies well prepared for the career, whatever it may be, into which they may be thrown. Every profession, every vocation is crowded to success, and as the population goes on augmenting, the struggle for subsistence grows daily harder and harder, the competition for employment becomes keener and keener ; the standard of attainment in knowledge and practical skill becomes higher and higher, the strain upon the physical strength more severe. This being so, it becomes more than ever essential that a young man shall bring into the field the “ mens sana in corpore sano”—a sound constitution of both mind and body, a brain that has been taught to observe and to think, a moral nature disciplined to labour and to self-denial, with nerve to face difficulty, and not to be daunted by disappointment, or even by failure, and with health to support fatigue, and to profit by success. To achieve these results in the degree and manner suitable to the various spheres of life is, I presume, the purpose of all education that deserves the name. But it is in an especial degree that of the class to which you belong.

WHAT THEIR OLD EDUCATIONAL SYSTEM DID FOR SCOTSMEN.

Our Scottish forefathers had very clear ideas on this subject. They took care that every child learned, both at home and in the parish school, that he could not do his duty to God or man unless he lived an honest, sober, truthful, simple life, making the best that circumstances would permit of the faculties and opportunities that God had given him—earned the bread that he ate, and thinking itthe worst of shames not to secure his own independence, and that of those who had to lean upon him, by the labour of his brain and hands. That simple noble creed were ill exchanged for the superficial accomplishments which nowadays pass in many quarters for education. It made our country what it was, the home and stronghold of civil and religious freedom ; it triumphed over the disadvantages of a rigorous climate and barren soil; it sent out our young men to all parts of the world, to be the pioneers of industry and improvement, to earn honourable rewards in fame and fortune, and to make the blood of a ¡Scotsman widely recog-

iVutorhur    geprtment.


nised as in some measure a guarantee for integrity, for courage, intelligence, and perseverance. In that old system which had for its main object to mould and weld the character of the people into uprightness and self-reliance, book-learning was not overlooked. But as that was not to be acquired without special gifts, or the leisure of which those who have to do the rough everyday work of the world have of necessity little, no attempt was made to make it general. At the same time, however, every fair encouragement was given to those whose genius irresistibly impelled them to a life of study and research, or to seek a career in what are called the learned professions, There, too, the system had its triumphs ; for it not only bred good preachers, lawyers, and doctors, but also turned out scholars, philosophers, and men of science, who, pursuing knowledge generally under difficulties, which put their enthusiasm to the severest test, and who, having to make sure their footing at every step of their progress, knew thoroughly what they professed to know, and by books of solid value, or by important discoveries in science, proved their title to a place among the teachers and benefactors of mankind.

DEFECTS OF THAT SYSTEM AS REGARDED THE SCHOOL CURRICULUM.

The great principles by which our ancestors were guided in these matters are eternally true. To turn out good men and good citizens was their aim ; and according to their lights and their means they spared no efforts to achieve it. As regarded primary education, there was little room for improvement. Whether, as time went on, and great changes took place in the circumstances and the wants of the country,the methods pursued in the higher education were always the best, is not clear. Speaking from the experience of my own youth, the range of studies was too limited, the methods of instruction were faulty; and too much, far too much, time was spent over these studies, such as they were. We were too early taken away from learning the structure and the resources of our own language, and from its stores of historical, biographical, and other knowledge, which the opening mind of youth could follow with interest and assimilate with ease and advantage, and were sent to puzzle in a confused way over Latin and Greek, to which many of the best years of our youth were devoted, almost to the exclusion of every other study. Never can I forget the hopeless weariness of those long hours, spent by myself in the Edinburgh High School, during six years, in learning badly what might easily, under proper training, have been learnt thoroughly in one-half the time. The system was radically faulty ; for the pace at which our knowledge advanced was regulated by the idlers and the dunces, who, to say the least, formed a tolerably liberal proportion of the much too large classes of which we were compelled to form a part. What was the result ? As it cost a boy of fair intelligence really no effort to acquire all that he was expected to learn, and as no amount of attainment enabled him to abridge the regular curriculum, the studies to which he was fettered too often became to him an object of ennui and disgust. Accordingly, it was only the boys of strong character, who set up other subjects of study for themselves, and so kept their minds fresh and active, who escaped unhurt from the evils of the system. But the injury to them was not slight ; for at that age I hold that every hour lost is a serious mischief, and the mischief is more serious when the loss is linked by painful associations with studies that should have brought only delight and profit. How many of us felt what Byron so vigorously expressed in his farewell to Horace, whom “he hated so!” The weary iteration of lines badly construed and miserably translated under the handling of a prosaic system, which did not even aim at giving vitality to the foetry of our text-books, or creating a human interest in either the men who wrote it or the people of whose soul it was the finest expression, took from the Venusian bard well nigh all his brilliancy and charm, and blurred the sweetness and stately grace of his great compeer, Virgil. And this for young men who already knew and loved Milton, Dryden, Gray, Goldsmith, Burns, and Scott, who wrnre beginning to appreciate Wordsworth, and to drink instruction to mind and heart from that perennial well-head of beauty and wisdom, and humour and humanity, which is to be found in Shakespeare—young men who;- if rightly taught, might have enjoyed their Horace, their Virgil, or their Catullus, with a relish as keen as they felt for their English favourites—nay, with possibly even a keener relish, by reason of the pleasant extra effort which it costs to master them, and which fixes attention upon the subtler shades of suggestion or of beautiful diction, which young readers, ever impetuous and eager, are apt to overlook in their native writers.

UNIVERSITY EXPERIENCES.

Passing from the High School to the University, without any love for the classics, which I had ever felt almost crushed out of me, I must ever remember wilhgratitudethenewlife and interest infused iuto them for usby the then Professor of Humanity there—Professor Pillans, What had seemed harsh, crabbed, colourless, grew full of fascination and charm, and “ a perpetual feast necturcd sweets, where no crude surfeit reigned.” He taught us to read and to assimilate the thoughts, of which the words had heretofore often seemed but sapless husks. He connected the literature of Rome with its history ; he made us understand something of the men to whom it was addressed, and of the state of society in which it was produced. Thus he made it a living thing for us. He taught us to think of men—who for us had hitherto been little better than names —as human beings, much like the men who had made and were making our own history, and encouraged us to try to gather from their stories incentives to work as they had done—

Certare ingenio, contendere nobilitate,"

and lessons to guide us in the formation of opinion as to what is the foundation of a nation's greatness, and by what the greatness of a nation, once achieved, is alone to be maintained. Under such guidance, gentlemen, you may conceive with what eager delight we took to our Cicero,

our Livy, and our Tacitus ; how7 it became a positive pleasure to combat the difficulties that stood in our way as we strove to gain full insight into their meaning, and how deep a bold the great maxims of practical wisdom, which we learned from them, established upon our minds and in our memories. With what different eyes, too, did we look upon our Lucretius, our Ovid, our Virgil, our Horace ? We read them with a new light upon the page, and learned to love them with a love which has outlived the lapse of years, and been proof against the blandishments of newer claimants for our regard.

GOOD RESULTS TO RE OBTAINED BY AWAKENING INTEREST.

Why do I recall these experiences of my own ?    1 do so, because they

seem to me to contain a lesson of wide application. I do not know whether the system under which my compeers and myself suffered has been changed. I hope it has been reformed, and that not indifferently only, but altogether ; for as it was then, so it must always be, a heavy drag upon the nascent intelligence of a large section of the community. If boys or young men are to be taught, you must make sure of their feeling an interest in what they7 have to learn. Do that, and the idlest will not turn aw7ay from his studies, nor the dullest lose heart over them. Let them see the use of what they are learning ; make their understandings work upon it; waken their powers of observation and deduction ; rouse within them the feelings and the thoughts of which words are but the j symbols ; feed them, in short, with facts which they can appreciate, and not with phantasmal phrases, and I care not what you set before them, be it grammar, or history, or geography, or classics, or mathematics, or natural science, or the great leading principles of health, morality, or economy, and you will have little cause to complain of the numbers of dull boys or unsatisfactory men. Follow the opposite course—cram them, according to their various powers of absorption, with facts they neither understand nor care for, with dates and names which they have not been taught to connect with subjects of human interest; load their memories with problems which they have been schooled into working out mechanically, and with cut-and-dry opinions, which can be produced to meet the exigencies of a pass examination, and you may turn out a fair number of clever fellows, to make a figure in class lists, but in those lists, I fear, very few of the men will be found who make their mark in life by bringing into it the well-digested knowledge, the ready helpful intelligence, and the strength of character, which are the things most wanted in every sphere, and which, in the main, are tolerably certain of recognition and worldly success.    (To be continued.)

APPOINTMENTS.

Eleanor West, H.T., Norong, 2454; Emily Heath, H.T., Goodman’s Creek, 24-50; James E. Walker, H.T., Avenel, 8; A. W. White, H.T., Anderson’s Creek, 12; Margaret Ritchie, H.T., Lower Crawford, 2400; Mary Hewitt, 2nd Assist., Rush worth, 1057 ; Annie J. Holding, H.T., Woolshed, 1875 ; James Cullin, H.T., Exhibition St., 2030 ; Win. J. Jack, H.T., Cornelia, 1499; Mary D’Anson, 2nd Asst., Chcwton East, 1928; John H. Bottrell, H.T., Bullarts, 12S8 ; Alicia O’Neill, H.T., North Dam, Rupanyup, 1834; Thomas Lang, H.T., Cobaw, Langley and Baynton (half time), 2132 ; Arthur Kemp, H.T., Mt. Pleasant Creek, 1248 ; Patrick W. Considine, H.T., Reid’s Creek, 1629 ; Henry P. Seymour, II.T., Conover Bridge, 1736; William S. Doria, H.T., Quconsberry Street, 2365; Edw.

C. Bell, 2nd Asst., Queensberry Street, 2365 ; James Craig, H.T., Stuart Mill, 1024 ; John Nance, H.T., Tarranginnie, 2355 ; Robert C. B. McDonald, H.T., Upotipotpon, 244-5 ; F. W. H. Schier, H.T., Chetwynd Junction, 1777; James T. Harrison, II.T., Bootapool, 1545 ; 10. Spowart, H.T., Fine Viow, 1745 ; Thomas W. Bothroyd, H.T., Toolern, 916 ; Caleb Collyor, H.T., Smythesdale, 978; Jane M. Downes, 1st. Asst., lluntly, 306.

Standard Grammar,—Books I., If., III., adapted to Standards 2, 3 and

4. By J. M. Meiklejohn, M.A., Professor of Education in the University of St, Andrew’s, Chambers’ Course. Melbourne : M. L. Hutchinson, Collins-strcet west.

In the preparation of this graduated set of books, the technical terms of grammar have not been introduced until the mind of the pupil has been familiarised with the leading facts of English grammar. The phraseology employed and the illustrations given are well calculated to arrest the attention of children. The following may be taken as an example from Book I., page 20 ;—“ Now, it is the very first thing to be learned in grammar, what words go with each other. We must get a habit of noticing this very carefully and exactly. All the grammar that comes after is built upon these two facts :—(1) Some words go with Nouns. (2) Some words go with Verbs. Let us take a few more 1 examples : («4 The tall, strong blacksmith works very hard. Here the words the, tall, and strong go with the word blacksmith ; hard goes with works; and very goes with hard, (b) The pretty little bird sang very sweetly. Here the, pretty, and little go with bird; sweetly goes with sang-, and very goes with sweetly. (c) The small clear stream rnns gaiiy down the valley. Here the, small, and clear go with streamgaily and down the valley go with runs. (d) The timid hare ran quickly away. Here the and timid go with hare ; while quickly and away go with ran.

Also from Book 1IL, as follows :—Preparation for Parsing.—The difficulty in parsing English words consists in the fact that they have not a different form for each different state or relation. Thus the objec-

142


A USTE AL AS IAN


SCHOOLMASTEK.


Marcii, 1882.


tive case of a noun is just the same in form as the nominative ; the third person plural of any tense of a verb is the same as the first or second person plural ; and so on. This is not the case in Latin. In that language, almost always, a new relation possesses a new form. Hence Latin is very easy to parse. To get over this difficulty in English, it is necessary for the pupil to keep up, from the very beginning and all throvgh his education, the habit of noticing what words go with each other. Hence, before we take each word by itself and give a full account of it (a process which is called jjarsvng), we must form the habit of noticing the connection of one word with another, Thus we may examine the words in the sentence we are going to parse in the following manner—as given in

I. Model op Preliminary Parsing.

‘ He heard that the young queen was not quite so well.’

He goes with heard.

Heard is said of he.

That joins the two sentences Tlefieard and the young queen was not quite so well.

The goes with queen. Young goes wdth queen. Queen goes with was. Was is said of queen. Not goes with équité. Quite goes with so. So goes with well. Well goes with queen.

II. Model op Full Parsing,

‘ He heard that the young queen was not quite so well.’

He A pronoun of the third person,—singular,—masculine,—nominative case. It is the nominative to the verb heard ; because heard is said or asserted of he.

Heard A verb, weak, transitive,—active voice,—indicative mood,— past indefinite tense,—singular number,—third person—

‘ agreeing’ with its nominative he, according to the rule : ‘A verb agrees with its nominative in number and person.’ That A conjunction or sentence-joining word,—joining the two sentences, lie heard and the queen was not quite so well.

The An adjective or noun-marking word,—marks the noun queen. Young An adjective or noun-marking word,—marks the noun queen, and is compared by -er and -est, as young, younger, youngest. Queen A noun, common,—singular number,—feminine gender,—nominative case. It is the nominative to the verb was.

Was A verb, intransitive,—indicative mood,—past tense,—singular number,—third person, ‘agreeing’ with its nominative queen : according to the rule, etc.

Not    An adverb, modifying (quite.

Quite    An adverb, modifying so.

So    An adverb modifying well.

Well    An adjective or noun-marking word,—marks the noun queen.

Well is never used as an adjective, except in the ‘predicate. (That is, it is never used attributively.)

Animal Physiology. Book II., Second Year’s Course. Blackie’s Comprehensive Series. Melbourne : M. L. Hutchinson, Collins-strect West.

Last year we had the pleasure of bringing under the notice of school teachers Book I. of this very valuable scries of text-books on Animal Physiology. An enumeration of the contents will serve to show how well adapted the second book is to aid teachers who have chosen this branch of science study. The subject treated is Human Physiology : — Part I. —Mechanism of the skeleton, levers, joints, movements of the limbs, and of the body as a whole. Part II.—Digestion, circulation and respiration. Each part is replete with diagrams and illustrations, which cannot fail to make the text understood by the dullest student. The knowledge imparted in this little book is just the lcnowdedgc required by the masses, and its dissemination through the medium of our State schools would help to make the future men and women of the colony morally and physically robust.

Elementary Historical Reader. Part I., Blackie’s Comprehensive School Series. Melbourne : M. L. Hutchinson, Collins-street West.

The lessons on English History herein contained embrace the more prominent events from the earliest period to the reign of Stephen. Designed for use as a Reading Book for Standard II., the dates have been omitted from the text, and the language employed is simple, and such as a child can understand.

Fourth Reader. Blackie’s Graded Readers, edited by Maurice Paterson, B.A., Rector of Moray House Training College, Edinburgh. Melbourne : M. L. Hutchinson, Collins-street West.

Mr. Paterson has evidently set before himself a high model in the preparation of this series of Reading Books. Noble sentiment and heroic deeds are to be found within their pages. The contents-of the volume, as might be expected, make a larger demand upon the intelligence of the pupil than the earlier numbers of the series. It is a book, however, which cannot fail to fascinate the youthful mind.

Geographical Reader. Part II., England and Wales. By W, G. Baker, Lecturer at Cheltenham Training College. Blackie’s Comprehensive School Series. Melbourne : M. L. Hutchinson.

The teaching of Geography through reading lessons having been expressly encouraged by the “New Code 1880,” a series of books, compiled with a view to meet the requirements of the Code, has been issued by Messrs. Blackie aud Son. The book before us belongs to Standard III., and contains condensed information respecting the topography, physical characteristics, political Government, and commerce of England and Wales. The following extract from the chapter on “Rivers,” may be taken as evidence of the terse English in which the lessons are conveyed :—“ Rivers are useful for draining the land where the supply of rain and springs is large, and for providing water where it is scarce. They are necessary for good tillage; land without rivers is either desert or mar«fa. They are almost as necessary in manufactures as in agriculture. They often supply the power ■which drives the machinery of the mill ; and most trades, in particular those where bleaching and dyeing take part, require abundance of water. But their greatest use is as water-ways, along which the produce of the field and the mill may be carried easily and quickly from one part of the country to another, and between inland places and the sea ; they are of the highest value for inland and foreign trade. To serve all these good purposes, the rivers of a country must be plentiful, slow-flowing, deep, not subject to Hoods, and of fair length. A broad tidal estuary adds much to the value of a river for the purposes of navigation. The rivers of England fulfil every one of these conditions. They are very numerous ; no part of the country is without its stream or river, They rise in low hills, well and constantly supplied with rain and springs, and, for the size of the land, have a long course to the sea. Thus they are deep, full, slow-flowing, and seldom flooded. They empty into seas subject to high tides, the estuaries of the larger rivers are broad, and their beds have a very slight slope ; hence the tidal wave is felt far inlaud, carrying ships and barges into the midst of the country. There are two principal slopes or watersheds down which the rivers flow, the one running north and south, and the other east and west. The northern watershed consists of the Pennines, and the hills to the south as far as Salisbury Plain. The southern watershed extends through the Weald, across Salisbury Plain, and along the southwestern ranges. The two watersheds meet at Salisbury Plain, forming together an inverted T (X) or broad Y (A)- The Welsh Mountains form the watershed of Wales, down which the streams flow in all directions, but chiefly east and south. Thus the land has three main slopes—east, south, and west, of which the longest is the east, and the shortest the south. All the longest rivers empty into the North Sea except the Severn, and that flows eastward for the greater part of its course.

Cassell’s French-English and English-French Dictionary. Revised, corrected, and considerably enlarged from the 7th and latest edition of the Dictionary of the French Academy, by Professor E. Rou-baud, B.A.(Paris), 116th thousand. Melbourne: S. Mullen,Collins-street East,

By all studentsof the French language, but more especially by those who include French among the subjects chosen for Matriculation aud Civil Service examinations, the additions and amendations made iu this edition will be found to be invaluable. The large number of new words (some ‘¿,200) introduced by the French Academy iuto the last published edition of their French Dictionary, led the Messrs. Cassell, Galpin&Co., to determine upon making an entire revision of their deservedly popular dictionary. Special care has been taken to make the revision thorough. Thus,in dealing with obsolete words, regard has been had to the fact that students read ancient as well asmodern French. Instead, therefore, of expunging obsolete words, they have been marked in both divisions, so that when translating, the student may see at a glance what expressions arc not in modern use. In the French-English division] alone upwards of 3000 words have been added, as also a complete list of all the nouns of double gender, with their meanings. Signs have been introduced into the same division to indicate the plural of foreign and compound nouns, thus affording considerable help to the students in a most intricate subject. These additions have increased the size of the book by about 100 pages. The type and binding of the present edition is all that could be desired.

The Marlborough Arithmetical Test Cards. For Standards II. to VI. By T. B. Ellery, M.C.P. Standard II.. 36 Cards and 2 copies of Answers. Is. per Standard. Melbourne: M. L. Hutchinson.

These test cards should find a wide circulation in the schools of Australia. “In Standard II., a thorough training in the plain rules is provided for, and a few very easy problems are given.” The following are the questions on Card 21 of Standard II:—

1.    Divide one hundred and fifteen thousand five hundred and seventy-seven, by Eleven.

2.    Add together : Forty-eight thousand ; seven hundred thousand and nine ; eighteen ; six thousand and sixty ; seven hundred and eight thousand and three.

3.    Multiply Nineteen thousand eight hundred and fifty-six, by Seventy-nine.

4.    Take Fifteen thousand and fifteen, from Twenty thousand one hundred.

5.    168,600-f 8.

6.    George had 16 [apples. He gave Tom 3, and Harry 6 ; and how many had he left ?

In Standard III., there is every variety of exercise, and considerable pains have been taken with the reduction which many inspectors require, and with the problems which are expected from children in this Standard. In Standard IV., there is a complete course of reduction and money work, the exercises being, in all cases, based upon those set by Inspectors indifferent parts of the country. In Standard V., great attention has been paid to proportion and bills of parcels. On every card, in addition to the practice exercises, there are, at least, one proportion sum, and one bill. In Standard VI., every card covers the whole ground of vulgar and decimal fractions and proportion. Here and there a simple interest sum is introduced, and altogether there is great variety iu (he exercises.”

Domestic Economy Test Cards, in three stages. Stage III, 30 Cards, 6 Questions on each card. Chambers’ Series. Melbourne : M. L. Hutchinson,

These Test Cards cover the whole subject of Domestic Economy as taught in the Board Schools, from Chambers' series, under the new code, and their use will prove a great aid to teachers. The following questions from cards 1 and IS, of Stage III, will illustrate their use:—Card 1.— 1. Show how household work is a healthful occupation. 2. What may be used as a remedy for gum-boil? 3. Say how au invalid may be washed without risk of catching cold. 4. How would you cook beet-

root? 5. Name any sweet herbs which ought to be preserved for winter use. G. State fully how to prepare tea for six persons. Card IS. —1. What is the result of want of exercise? 2. Of what use are the pores of the skin ? 3. How may one have perfect, ventilation in a sickroom ?    4. How can bones be made useful as food? 5. Describe a good

ham. G. Give a rule for knowing pure, freshly ground hour.

Geographical Header. Books I. II. and III. How People Live on the Earth, and a Journey round the World, by J. M. D. Meiklejohn, M.A., Professor of Education in the University of St Andrew’s. Melbourne : M. L. Hutchinson, Collins-street, West.

The high position gained by Chambers’ Educational course is not. likely to suffer by the addition of the Geographical Readers, prepared by so excellent an educationist as Mr. Meiklejohn. The aim of the writer has been to lead the pupil, step by step, to an acquaintance with the geography of the world.    Commencing with a colloquially written

description of the life of man in different parts of the world, his house, dress, food, and ways of travelling, he takes the pupil on an imaginary journey round the world, after which he lets him into a knowledge of the principal facts of general, descriptive, and physical geography, and then brings him back to the British Isles, respecting which very full and valuable information is given.

The whole series is charmingly written, printed on good paper, carefully illustrated, and strongly bound. The Education department would do well, we think, to adopt Books I. and II., at least, for use in State Schools, They are well adapted for the junior classes.

Messrs. Ward and Loch's Universal Instructor, or Self Culture for All' Vol. I. Fully and accurately illustrated. Melbourne : M. L* Hutchinson, Collins-street West.

Of all the aids to self-culture recently issued from the British press, “ The Universal Instructor” stands unsurpassed, both for the character of the articles it contains, and the skill with which it is illustrated. The aim of the publishers has been to produce a work that would prove “ nothing short of a University at home. The latest thought of our time, and all modern inventions and discoveries, have had justice done to them.” Whilst appealing mainly to self-educating students, the “Universal Instructor” has not overlooked the wants of those who are fortunate enough to have tutors, and has therefore given a place to subjects of special interest to all who can take advantage of existing educational machinery. The volume before us contains well-apportioned lessons on Arithmetic, Astronomy, Botany, Chemistry, English Grammar, French Grammar, French Pronunciation, Physical Geography, Geology, German, Ancient History, Modern History, Latin, The Microscope, Music, Penmanship, and a number of excellent articles, under the head of ‘'The Friendly Counsellor.” To students residing in the country districts, this work can be confidently recommended. The price, 7s. 6d., places it within the reach of all. “It cannot be too often repeated that a good education is a fortune, and the best of all fortunes. If the ignorant would succeed in the world, they must betake themselves to study, for, in these days of competition and examination, to neglect learning is to court failure. Hence naturally arises the enquiry, What can we do to improve ourselves so as to compete on better terms with those around, not ouly in business, but in social life?” This enquiry the ‘Universal Instructor’ undertakes to answer.”

English ‘Historical Reading Booh. Chapters from English History. Parti. From the earliest times to the death of Richard III. Blackie’s Comprehensive School Series. Melbourne : M. L. Hutchinson, Collins-street West.

So long as State school teachers are forbidden to teach history, we suppose it will be useless to expect a large sale for this branch of Messrs. Blackie’s really incomparable series of school books. We wish it were otherwise, as the following extract from page 114, will show how superior the English composition of “The Historical Reading Book”is, to that of the reading book now in use “ Simon de Montfort.—The first House of Commons.—King Henry’s faithlessness alienated the best of his subjects, and chief among these was Simon de Montfort, the Earl of Leicester. He was the youngest son of the Count de Montfort in France, who had gained an unhappy celebrity in the barbarous crusades against the Albigenses. In right of his mother he succeeded to the earldom of Leicester, and he married Eleanor, a sister of King Henry. As first he was high in the royal favour, but the love turned to hatred when the earl steadily opposed himself to Henry’s foolish and tyrannical mis-government,

“ In June, 1158, the parliament which the royalists called the “ Mad Parliament,” met at Oxford. They determined, and the King was obliged to consent, that a committee or council of twenty-four members should be appointed to exercise the powers of government. Twelve members were appointed by the king and twelve by the parliament, and De Montfort was chosen as chief.

“ The parliament then enacted that four knights should be chosen by the freeholders in each county, to lay before parliament all breaches of law and justice that might occur; that a new sheriff should be annually chosen by the freeholders in each county ; and that three sessions of parliament should be held regularly every year.

“ The benefits derived from the acts of this parliament were prospective rather than immediate, for the first consequences were seven or eight years of anarchy and confusion, the fruits of insincerity and discontent on the part of the court and of ambition and intrigue on the part of the great barons. A few improvements, chiefly regarding the administration of justice, were enacted, but their slender amount did not satisfy the nation, and most of the barons were more anxious for the prolongation of their own powers and profits than for anything else. Factions also arose within the council, and King Plenry and his son Prince Edward did their best to embitter the disagreements.

“At last in 12G4 civil war broke out in the land. The king had gathered a great force in the southern counties, and the Earl of Leicester marched from London to meet him. He found the king at Lewes, in Sussex,—a bad position, in a hollow—which Henry, relying on his superiority in numbers, did not quit on the earl’s approach. Leicester encamped on the downs about two miles from Lewes. On the next morning he descended into the hollow and the two armies joined battle. Prince Edward, who was destined to acquire the rudiments of war in the slaughter of bis own subjects, began the battle by falling desperately upon a body of Londoners who had gladly followed Leicester to the field. They could not stand again the trained cavalry of the prince, and he chased them and slew them by heaps. Meanwhile Leicester made a concentrated attack upon the king, beat him most completely, and took him prisoner before the prince returned from his headlong pursuit. Before Edward could recover himself he also was charged by a body of horse and was made prisoner. King and prince were kept in confinement, and De Montfort carried on the government in the king’s name. The queen had retired to the Continent before the battle of Lewes, and having busied herself in collecting a host of foreign mercenaries, she now lay in Flanders ready to cross over and renew the civil war. The steps taken by Leicester show at once his entire confidence in the good-will of the nation and his personal bravery and activity. He summoned the whole force of the country, from castles and towns, cities and boroughs, to meet in arms on Barham Downs, and having encamped them there, he threw himself among the mariners of England, and taking the command of a fleet, cruised between the English and Flemish coasts to meet the invaders at sea. But the queen's fleet never ventured out of port, and the enterprise fell to the ground.

“ The next year, 1265, Leicester summoned a parliament which is memorable as the first in which we have certain evidence of the appearance of representatives from the cities and boroughs.

But the downfall of Leicester was near. Prince E .l ward escaped from confinement and found many ready to join with him. The Earl of Leicester, keeping good hold of the king, was at Hereford, while his eldest son Simon was in Sussex. Prince Edward was between them, and meant to prevent their junction. The young De Montfort suffered himself to be surprised by night near Kenilworth, where Edward took his horses and treasures and most of his knights, and forced him almost naked to take refuge in the castle there. The earl, still hoping to meet his son’s forces, advanced to Evesham. On the morning of the 4th of August, as he looked towards the hills in the direction of Kenilworth, he 8aw his own standards advancing. His joy was but momentary, for he discovered, when too late to retreat, that they were his son’s banners in the hands of his enemies, and nearly at the same time he saw the heads of columns showing themselves on either flank and in his rear. The earl was surrounded—every road was blocked up—he cried, 'The Lord have mercy on our souls, for I see our bodies are Prince Edward’s.” He then spent a short time in prayer and took the sacrament, as was bis wont before going into battle. Having failed in an attempt to force the road to Kenilworth, he formed in a solid circle on the summit of a hill and several times repulsed the charges of his foes, who gradually closed round him, attacking at all points. His horse was killed under him, but the earl rose unhurt from his fall and fought bravely on foot. His gallant son Henry was killed before his eyes, the bravest and best of his friends fell in heaps around him, and at last the great earl himself died with his sword in his hand. After the battle the corpse of Leicester was brutally mangled and treated with every kind of indignity ; but by the people his memory was affectionately cherished, and long after he was spoken of among them under the title of ‘ Sir Simon the Righteous,’ ”

ASSISTANT in Town, salary £114, desires exchange with Head Teacher in better position- Anser, Post-office.”

A SECOND CLASS PUPIL TEACHER desires re-appointment. Country preferred. Recommendations good. Address, “B. B., Post-office, Murtoa.”

HEAD TEACHER, allotment 30 x 50, would exchange with another average over 80. Work mis tress’s position vacant, residence ; mail three days a week. Address, “ G. P. K., Edington.”

HEAD TEACHER, allotment 30 x 50, vacancy for workmistress, desires exchange. Smaller school taken if other demands suitable. Melbourno district preferred. Address, “ Ductile, G. P. O., Melbourne.”    -

HEAD TEACHER, allotment 30 x 50, workmistress’s position vacant, desires exchange. Lower allotment accepted if other requirements suitable. Geelong or Melbourne district preferred. Address, “Dominic, G. P. O., Melbourne.”

HEAD TEACHER, 20 X 30 and Post-office, Ararat district, desires . exchange Head Teacher or Assistant. Address, Teacher, Post-office, Ararat.”

HEAD TEACHER, St. Arnaud district, near Avon River, allotment (safe) 30 to 50, results 82, Wife workmistress, desires exchange. School must have a residence, as the chief reason for seeking exchange is having to reside at too great a distance from the school. Would suit a single man admirably. Address, “Civil, Post-office, Stawell.”

HliA.D TEACHER, within twenty miles of Melbourne, would exchange, allotment 20 x 30. Good residence, sea-side preferred. Address, “ Negotium, G. P. O., Melbourne.”    _

HEAD TEACHER, country, 30 x 50, results 88’235, wishes exchange Assistant, Melbourne or Suburbs. Address—“M.B., care of Mr. Roth, Tivoli Place, South Yarra.”


144


AU STEAL ASIAN SCHOOLMASTER.


Marcii, 1882.


N.I3.— Students after Matriculating, and beforo proceeding to the University, should read the First Years’ work of their course ; otherwise the Lectures arc of little benefit to them, and their chance of passing at the end of their first year is very small.


MR. H.

M'KTNLEY

s

OLICITOR,

CONVEYANCER,

And

PROCTOR,

86

COLLINS

STREET WEST.


npO HEAD MASTERS, SECRETARIES T OF BOARDS OF ADVICE, AND OTHERS.

ALEX. M‘KINLEY & CO.,

Having made considerable additions to their stock of Bookwork and Jobbing Type, are prepared to execute orders in every description of

GENERAL PRINTING.

All orders entrusted to them will bo printed in a satisfactory manner.


Alex. M'Kinley & Co., Printers, Gl Quken-st.


rpATE’S PARCELS POST EXPRESS

FIXED PRICE.

NO EXTEAS. NO TROUBLE.

Delivery to door at any address in

21b s, (L

41b.

8. Cl.

61b. s. d*

101b. s. d.

201b. s. d.

Great Britain ...

4 6

6 0

7 G

10 0

14 0

Continent of Europe, America, & Canada

7 6

9 612 0

16 0

21 0

Sydney, Hobart, Launceston ...

2 6

3 0

3 6

4 0

6 0

New Zealand Ports (except West Coast) Adelaide, Brisbano.

4 0

4 6

5 0

5 6

7 6

No further charge whatever. Very small increase for heavier weights. Delivery at country addresses in Australia, inland carriage only added.

Deceiving office—

FREDERICK TATE, 13 Market-st., Melbourne T A T B ’ S P ARC E L S POST EXPRESS. DELIVERY at DOOR any address in the world. EITHER to or from Britain from 4s. 6d. TO or from other Australian ports from 2s. fid. NO furl her CHARGE whatever. No trouble. Any SIZE, weight, or shape.

EVERY possible ASSISTANCE a i lorded) ' ___ 1 A 0 1 I !>’ 1 ES plainly answered.

Henceforth Parcels handed to W. R. SUTTON’S Branches, every town in Great Britain, delivered in Melbourne at nearly similar rates.

Head Office :—

FREDERICK TATE, CUSTOMS AGENT, &c., 13 Market-street, Melbourne.

pUNCH’S SOCIALITIES.

PUNCH’S SOCIALITIES.

PUBLISHED V- 4th APRIL.

PUNCH’S SOCIALITIES.

PUNCH’S SOCIALITIES.

^LEX. M'KINLEY & CO.,

GENERAL

PRINTERS AND PUBLISHERS,

61 Queen-street, Melbourne.

In Bookwork and General Publishing our large experience is a guarantee of all work being executed in the host stylo, while having a first-class stock of the best and latest material ensures expedition and good workmanship.

Tho following papers are issued from this office WEEKLY.

Punch,”    “Bulletin,”

“ Faithful Words,”    “ Onco a Week.”

FORTNIGHTLY.

“Australian Law Times,”    “Jewish Herald,”

MONTHLY.

Schoolmaster.”    “ Monthly Messenger.”

BUY AND READ

THE NEW WEEKLY PAPER. Prico Threepence.

“ONCE A WEEK,”

/CERTIFICATE OF COMPETENCY. Ladies V J prepared for Examination. Evening Class-Miss Trickett, 1 Lansdowne Street, East Melbourne-

I WARREN BALL’S “Hints to Candidates • for Teachers’ and Matriculation Examinations,” Is.; posted, Is. Id. Mullen, Melbourne.

CANDIDATES for EXAMINATIONS prepared by correspondence or otherwise. I. Warren Ball, South Yarra.

JMPERIAL REVIEW,

Quarterly, 2s. Four Numbers, Subscription, 7s. fid,, including postage anywhere.

Publishers :

ALEX. M'KIN LEY & CO.,

61 Queen Street, Melbourne.

"□EXAMINATIONS.—UNIVERSITY

and DEPARTMENTAL.

TUITION in CLASS by Correspondence or otherwise. Terms, &c.,

JAMES L. ROBERTSON, B.A. (Melbourne). 71 Clarendon Street, Emerald Hill,

Qb MULLEN’S

New Classified Catalogue of School, College, and Technical

EDUCATIONAL WORKS

May be had gratis on application, or posted on receipt of address.

SAMUEL MULLEN,

Wholesale & Retail Bookseller & Stationer, 29 & 31 COLLINS ST. E., MELBOURNE.

Price One Shilling,

By Post—In Victoria, Is. fid.; Out of Victoria, 2s

EPAKT MENTAL EXAMINATION OF TEACHERS.

TEACHERS’ GUIDE

TO

AUSTRALASIAN EXAMINATIONS.

Containing the

PROGRAMMES & EXAMINATION PAPERS

of

Victoria    South Australia

New South Wales    Auckland, N.Z.

Queensland    Wellington, N.Z.

And Tasmania.

108 PAGES WITH STIFF COYER.

Tho above book contains the Programmes and Examination Papers of December, 1877, of all the colonies, and is reduced to the low price of

ONE SHILLING.

By Post—In Victoria, Is. fid.; Out of Victoria, 2s

O c. EXAMINATION.

TUITION BY CORRESPONDENCE.

MR. THOMAS BOARDMAN, First-class Honor-man of the Denominational School Board, Prepares Teachers for the Certificate Examination by Correspondence. Terms moderate.

Address—

45 PRINCES STREET, CARLTON

TAMES CLEZY, M. A.,

5 GORE STREET,

Prepares Matriculation, Pass or Honours and other University Examinations. Since February, 1874, One Hundred and Fifteen Pupils have passed various Examinations, ranging from Civil Service to the final Examination for B.A. Degree.

# In October Term, 1881, Four passed Matriculation, including Civil Service. One was first of the first-class in Greek and Latin Honours. One (a lady) obtained a second class in Greek and Lalin. Two passed for first year LL.B.

NOW READY.

ILTON PARSED.

Price 2s.

By J. J. BURSTON,

(Author of “ State School Arithmetic”).

Also Ready, the Fourth Edition of the

STATE SCHOOL ARITHMETIC.

1C PAGES.


PRICE THREEPENCE.


ALL BOOKSELLERS.


ALEX. M'KINLEY & CO.,


PRINTERS AND PUBLISHERS,


By

JOHN J. BURSTON,


North Sandhurst State School.


61 QUEEN STREET,

MELBOURNE,


Albx. M'Kinley and Co., Publishers, 61 Queen Street, Melbourne.

Printed and Published by Alex. M'Kinley & Co., 61 Queen-street, Melbourne, under the auspices of the Victorian Teachers Union.

AUSTRALASIAN

AND LITERARY REVIEW.

Vol. Ill, No. 34.


APRIL, 1882.


(    SUBSCRIPTION

{ Yearly, 6s. 6d. ; Half-yearly, 8s. Gd.


U. L, HUTCHINSON. Wholesale and Retail Bookseller and Stationer,

15 COLLINS STREET WEST, MELBOURNE,

SUPPLIES ALL

SCHOOL REQUISITES AT LOWEST PRICES.

NOVELTIES BY EVERY MAIL STEAMER,

BLACKXE AND SON’S COMPREHENSIVE SCHOOL SERIES.

W. & R. CHAMBERS’ EDUCATIONAL COURSE.

These Works obtained the only Gold Medal for British Publications at the International Exhibition.

CHAMBERS’ ENCYCLOPAEDIA, NEW EDITION, 1881, 10 VOLE, CLOTII & HALF-CALF.

Agent for Henry Thacker & Co’s. “INK OF INKS,” and “PEN OF PENS.”

Ward, Lock & Co.’s lew Books for Every Home and School,

Including

^ ORTHIES OF THE WORLD: A Series of Historical and Critical Sketches of the Lives, Actions, and Characters of Great and Eminent Men of all Countries and Times, with numerous Portraits; one vol., cloth 7s. 6d,

THE UNIVERSAL INSTRUCTOR; ob, Sklb-Cultuee bob. All. Fully and accurately illustrated; to bo completed in three vols.; vol. I. now ready, 7s. 6d.

El ERY MAN HIS OWN MECHANIC: A complete guide for amateurs in every description of Constructive and Decorative Work, with 800 illustrations; one vol., elotli, 7s. 6d.

HAYDEN’S DOMESTIC MEDICINE, Edited by Dr. Lankester, assisted by Distinguished Physicians and Surgeons; numerous illustrations; on-vol., cloth, 7s. 0d.

BEETON’S ILLUSTRATED ENCYCLOPAEDIA, embracing Biographys Geography, History, Science, Art, and Literature, with about 2000 illute trations and 4000 pages ; in four fine vols., hulf-bound, 4-2s. The eheapes* Encyclopaedia ever published.

M. L........HUTCHINSON,

GLASGOW BOOK WAREHOUSE,


15 COLLINS STREET WEST, MELBOURNE.


Stjmcri gepartmeni


THE GROWTH OF INTERMEDIATE EDUCATION IN GERMANY.

By H. Baumann, M.A., F.C.P.

( Continued from page 131.)

u The new Latin schools were to have three divisions—* Haufen’ he calls them. In the first or lowest Haufen, the younger children, after having learnt to read and to write, were initiated into Cato’s distichs and Donatos’ Grammar, and their memories fitted out with a proportionate stock of Latin vocables. In the second or middle form, musical instruction preceded and lightened the severer tasks of Grammar, Syntax, and Prosody. Alsop’s Fables wore construed ; the Colloquies of Erasmus studied ; select quotations from the ancients given home to serve as fundamental lessons of morality ; Terence and Plautus read to promote the ready use of the Latin Tongue ; easy portions of the Gospel were construed ; the Psalms and the Catechism learnt by rote. In the third or upper class we hear the first mention of Virgil. Also Ovid’s Metam., Cicero’s Officia and Epistolae ad Familiares appeared on the scene. Construction of difficult passages, together with lessons on metrical matter and figures of speech, and the usual course of Rhetoric and Dialectics, completed this simple scheme.”

Greek, Mathematics, History, Geography, Natural Philosophy, and Modern Languages still excluded from the domain of the school-room I Physical education not even mentioned ! The prejudices of monkhood, as this “ School-plan ” of Melanchthon teaches us, were still too deeply engrafted. The Colloquies of Erasmus had to supplant the whole range of our modern subjects. How much remained yet to be done !

Among other great school reformers and schoolmasters of the period, we must notice Trotzendorf, Neander, and Hieronymus Wolf, all of them connected with Melanchthon and the University of Wittenberg. The first-named, who is said to have converted the school of Silesian Goldberg into a colony of Latium, is best known to us as the inventor of a monitorial system, by which he was enabled to teach large numbers with few masters and at very trivial fees, and also for his immoderate predilection for the Latin language. Neander, the schoolmaster of Ilfeld, near the Harz mountains, shone more by his individual teaching efforts, than by elaborate routines like Sturm’s and Trotzendorf’s. His only pride were his scholars. Through them his name became a title of distinction at the universities. Such proficiency did they show in the three languages—Latin, Greek, and Hebrew—that it soon was esteemed an honour to be called a pupil of Neander. We admire his pedagogical foresight in another direction. He attempted to teach History, Geography, and Mathematical subjects, and did not despise the mother tongue. As third, we mentioned Hieronymus Wolf, the famous interpreter of Demosthenes. Wolf’s fame as an educationist rests on his activity as Rector of the Augsburg gymnasium, which he raised from five to nine classes, and which became, under his vigilant supervision, a far-famed nursery of classical, especially Greek, scholarship. His school routioe shows that he had a sounder understanding of younger boys than most of his contemporaries.

The most prominent leader in the matter of school organization the greatest school administrator of the sixteenth century, was Johann Sturm. His range of knowledge, his far-sighteduess and tolerance, made him one of the noblest types of the Protestant schoolmaster. We are not amazed if his admiring friends ranked him with the intellectual giants of antiquity. As a school organiser, however, he cannot be called an original inventor. The far-famed Academy of Strasburg was an improved copy of the large schools of the Dutch Jeromiaus. A comparison of his system and theirs would again confirm what we have heard regarding the connection existing between the Netherlands and her greater neighbour.

The nine forms and nine annual courses of hisGymnasium, together with the class promotions and examinations ; the careful moral training of his pupils, ensured by an iron discipline, and the linguistic predilections of his “ Stundenplan,” found numerous imitators in his own century, and have also become the foundation of the German State School of modern times. Specialists, it is true, have stepped into the place of his class-masters, at least in all our upper divisions. The mother language, together with History and the Ilealia, have taken the upper hand of the rhetorical and dialectical exercises of his days. Latin verse-making and stage performances are now things of the past, while Natural History and Sciences have come to the front. In Mathematics, likewise, we have taken a step in advance ; it may boldly be asserted that his most accomplished pupil could not have competed, in the art of ciphering, with a boy of the lowest form of the present Town College. The difference is still more marked when we consider the ages of the scholars. In his Octava, with little mites of six or seven, Virgil's Eclogues, the Epistles of Cicero, and the Latin Syntax—but no nursery rhymes and no toys 1 Poor little old men, how clever they must have been ! As another sign of the change which has come over men’s minds, we may also notice the fact that the modern German school has discarded prizes and rewards, which Sturm by no means despised.

We have already mentioned the name of Bugenhagen. John Bugen-hagen, surnamed Doctor Pomerauus, may be called the educational reformer of Germany’s north. In his schemes for the reorganisation of the Church in the northern countries, which originated in the earlier portions of the 16th century, he paid especial attention to educational matters. Brunswick, Hamburg, Liibeck, Pomerania, and Slesvic-Hol-stein, all are indebted to him for the reconstruction and modernisation, as it were, of their educational systems. His “ Lectionsplan” and method of school-government offer no new and distinctive feature, the “ Statutes” of Brunswick, Liibeck, etc., being merely adaptations of Melanchthou’s “ Schulplan.” In but a few cases he advised, instead of three divisions, five. This number, I may add, was afterwards also adopted in Württemberg, which soon followed in the footsteps of Saxony.

Thus far the schools of the Reformation. If this momentous movement produced no other result, it certainly roused the nation from its metaphysical dreams and strange fantastical hallucinations. The masses had suddenly conceived that, in order to approach perfection, a good deal may be done, even on this sorrowful earth, by means of intelligent study and good teaching. We welcome, as the most significant proof of this new intelligence, the establishment of elementary or German schools, which went hand in hand with that of the “ Lateinschulen.”

We now take a rapid stride across an age of barrenness and laxity. What the 15th and the 16th century, the Renaissance and the Reformation, promised in the way of educational advancement, has been partially realised only in our own. In the two centuries intervening, great religious dissensions and political dangers diverted men’s minds from what was most needful, and made this period entirely unproductive of great and wholesome changes. The aucients were still eagerly scanned and imitated, but more with an eye to colloquial polish, and a desire to shine out in the next public disputation, than with a critical appreciation of what has made them the marvel of all times. It required the genius of poets like Herder, Klopstock, Lessing, Göthe, and Schiller, to make their countrymen feel the beauty of the classical muse in the garb of their own rude but native speech. In other departments the progress made was slow and capricious. A universal exhaustion seemed to have followed upon an era of spasmodic fertility. Kepler, Galilei, Newton had revealed the mystery of our planetary system, but the most enlightened of schoolmasters still indulged in the ancient notion that the Earth was the centre of the universe. Theology predominated everywhere, most of all in the schools. The divorce of School and Church, so completely effected under the ministry of Falk, was not yet thought of. Our own and dearest science, the science of Education, still lay in its cradle ; it had yet to be nursed through an infancy of ills and obstacles. For who could speak of a modern science of Pedagogics, before men like Montaigne, Locke, Ratich, Comenius, Rousseau, and Pestalozzi first enlightened mankind on the true nature of that upon which the modern educator is to bestow his whole soul ? I mean upon the child—the modern child itself.

We may briefly notice how, during and after the troublous times of the Thirty Years’ War, the Gymnasia and Town Colleges, established at the behest of the Protestant leaders, sadly came to grief. Many were closed others had passed into the hands of the Jesuits, who, about the middle of the 16th century, commenced their education crusade against the disciples of Luther, Calvin, and Zwingli. It is no part of our duty to comment upon the ethical peculiarities of the Jesuits. In their intellectual training, based, as it was, upon the slavish imitation of the ancients and a deliberate oppression of the mother-tongue, they certainly stood upon the same plane with Sturm and Trotzendorf, We must look for the current that could widen up this narrow channel of mediaeval school methods and school-subjects in quite another direction.

About the time of the Long Parliament, Comenius, as you are aware, had endeavoured to utilize the realistic teaching of Bacon. Like him, he hoped, by an inductive or experimental process, to open up the rich mines of wisdom and power to every moderately gifted person. You are sufficiently acquainted with his Janua, his Vestibulum, his Orbis Pictus, and kindred works, to know how delightfully the good bishop of the Moravian Brethren put before his young readers all the common objects of daily life. Now this plan of teaching by objects was further developed, and practically explored, in several famous institutions of North Germany.

The first step was taken by Semler, a clergyman of Halle. In his capacity as School Inspector, he instituted, in the opening decade of the last century, mathematical classes for mechanics at Halle. The utilitarian principle strongly predominated ; the camera obscura stood by the side of a human skeleton ; the watchmaker’s art was put on an equal footing with naval construction. The. Humaniora, however, remained jealously excluded. No philosophical dreams of ideal culture floated before the originator’s mind. His school was, in name and in reality, a “ Mathematical and Mechanical Realschule.”

The second and more important move was made by Hecker, of Berlin. As a pupil and assistant teacher of Francke in Halle, in whose well-known establishment the “ useful” subjects, in the sense of Comenius, had become an almost co-ordinate factor with the older brauches of instruction, his taste for scientific studies first formed itself into an educational theory—not to say, hobby. It was he who, in 1747, gave to Germany the first technical institution of any note, the “ economical-mathematical” Realschule, as he styled it, of Berlin.

Here the objects which Erasmus and Comenius had only described and depicted in their works, were given into the pupil's hands, and all the elementary manipulations of the various trades, to which they referred, scientifically explained. Great, indeed, was the number of useful branches which this first great Real College undertook to cultivate. There were, besides most of the ordinary subjects of the Latin schools, various branches of Natural History, with Arithmetic, Geometry, Leather Manufacture, Architecture, Mining, Bookkeeping, Agriculture, Drawing,

Mechanics, and-Silkworms. But out of chaos, by slow gradations,

grew order and harmony. The giant task, to which no single school or species of schools could have lent itself with any prospect of success, might, if properly portioned out, afford occupation to a dozen collegiate species, and exhaust a millennium of schoolmasters.

From Wiese’s life of Spilleke, we learn how, in the third and fourth docades of the present century, amidst the clamours and protests of narrow-minded philologists, at length the great rival of the Gymnasium was officially installed. The Realsehule, as a class hitherto almost unknown, claimed and obtained a seat among the privileged State Schools of Prussia, and soon of all Germany. It found powerful friends both in the ministries and in the populous cities, so that one privilege after another fell to its share. But its foes were ever on the watch. When, in our own days, it asked for the full University qualifications of the Gymnasium, the battle grew more furious than ever. It rages to the present day. It keeps the best educational pens and lungs of the Fatherland in restless activity. Its war-cries linger in the ancient halls of the Universities. Its bursts of scholastic fury send a periodical thrill through Germany’s Parliaments.

Thus, then an educational dualism has been sanctioned and acknowledged by the highest authorities of the country. But the matter "has not been allowed to rest there. Technical town-classes, originally established for the benefit of humble artisans and engineers, gradually developed into a third intermediate branch, the Gewerbeschule. In the most modern and fully developed form—you must notice, I speak only of State Colleges of the highest or first order —these three schools or species of schools have each nine classes, in which an average pupil is expected to stay from his tenth to his nineteenth year. He is prepared for this course in the excellent elementary schools of the country. After its completion, the student passes a leaving examination, the so-called Abiturientenexamen, held in the school itself, which then dismisses him to one of the Universities or technical institutions. Many, however, of the scholars leave as soon as they have procured the certificate for one year’s service in the army, i.e., before they have reached the Upper Secunda or Third Class of their school. All the lower forms are, as a rule, extremely overcrowded, owing to this universal rush for the Army Qualification which, during the last dozen years or so, has extended its powerful influence also to the South German States.

With regard to their courses of instruction, the three main types may be briefly distinguished as follows : The Realsehule teaches no Greek, but fully develops two modern languages, together with Latin and the mathematical branches. The science lessons in these institutions, which as yet cannot qualify their pupils as medical students, deserve especial commendation. The Gewerbeschule, now also called Realsehule without Latin, excludes both the classical languages, but devotes additional time to drawing, science, and modern languages. The Gymnasium, on the other hand, devotes one-half of its time to the Realia and one modern language, and has been largely influenced, even in its classical studies, by the methods of the modern schools. There is, indeed, a great similarity, I might say family-likeness, between these different institutions, as far as their general educational character is concerned. Though their subjects of instruction may not be identical, the treatment of those which they have in common differs but by very delicate shades.

Thus a great step has been taken in advance. Many voices, however, can be heard which denounce this educational division of labour, and clamour for a Gymnasium with less of the classics and more of the modern branches, whereby they hope once more to rule and monopolize the educational market. Meanwhile technical education, like technical science, is striking out new routes every year, and is becoming a great social and political power. A Technical Commission, forming a branch of the educational ministry, is appointed for its special supervision. Its purest representatives are the Polytechnica and the Trade Schools. The former, of late called Technical Universities, take up the student where the intermediate institutions, especially the Gewerbeschule, leave him, and complete his preparation for one of the higher technical branches. The latter serve for the purposes of special trades. We find large establishments of this kind, especially in the kingdom of Saxony. I may mention, as best known, those at Dresden, Leipsic, and Chemnitz, Brunswick boasts of a famous college, for future engineers and architects (Bautechniker), at Holzminden, and so forth. But even these schools have been obliged to make important concessions to the State, especially in respect of linguistic instruction. As the army-qualification appoints two foreign languages—for what reason I am at a loss to know !—the Trade Schools as well as the Technical Colleges have been compelled to retrench their Science and Art programmes, in order to admit a moderate quantity of French and English.

In conclusion, I may be allowed to say a few words on the subject of State supervision, as far as it concerns the schools of Germany. Since the time of the Reformation, the princes of Germany have thought it their duty to control and guide the scholastic machinery of their principalities. The Duke Christoph of Württemberg, who, in 1559, issued his famous “ Schulordnung,” embodying the reformatory schemes of Melancbthon, Sturm, and Bugenhagen, may be called the first great Schoolmaster of the Swabians. What a detailed account this princely pedagogue gives of the duties of a teacher. The Landgrave Moritz of Hesse, another German prince of that period, went even a step further. He not only, in the teeth of the whole learned world, planned a subversion of the dull mediaeval routine by favouring the mother tongue, but, taking the pen out of the schoolmaster’s hands, wrote some very practical school-books. Every reader of educational history has also heard, how Leopold of Dessau supported Basedow’s “ Philanthropic,” and how a King of Prussia interested himself in the Realsehule of Hecker.

You will, no doubt, be curious to know how the great Frederick looked on this matter, and be little surprised to hear that he claimed to be the absolute monarch over the teaching as well as over the fighting portion of his subjects. But although he gave personal instructions on such trifling subjects as a Latin or a German grammar, the chief responsibility for the re-organization and centralization of Prussia’s schools falls upon Freiherr von Zedlitz. He, the educational minister of the great Fritz, was one of the foremost disciples of Basedow and Kant, and at the same time an ardent admirer of absolute 18th century monarchy. “The state,” this was his leading maxim, “is the central heart and the chief repository of our noblest actions and endeavours. If the child is to receive an education, an education in the highest sense of the word, it is merely in order to fit it for the service of the king.” Zedlitz learnt Greek and physical geography when a minister, and was insatiable in his desire for knowledge. He loved especially the classics; whence we may conclude that, if a follower, he was not a blind follower of Basedow. Greek, even in his time, began to rank among the compulsory subjects of the Gymnasium.

Since Frederick the Great, and especially from the time of the Napoleonic wars, the schools have more or less shaped themselves into political instruments of the German governments. Whole generations of schools have frequently vanished before a single breath or stroke of the pen of an impatient minister. The most glaring example of centralization can be observed in Bavaria, where, ever since the beginning of the present century, one arbitrary measure has followed upon another.

In Modern Prussia, the despotism of reactionary ministers like Raumer and Miihler, trampled educational individuality under foot, while enlightened statesmen, like Humboldt, Altenstein, Falk, in order to spare and promote originality—the mother of all arts, teaching not excepted— endeavoured to strengthen the hands of the headmasters, the provincial inspectors, and the teachers themselves. I am not able to follow up this interesting question of State control in detail. But I will not omit to give you the rough outlines of the present system of administration.

At the head is the Imperial School Commission (Reichs-Schulkommis• sion), the central board of the Empire, consisting in all of six members, Professors and Councillors, both of North and South Germany, who meet under the presidency of the Imperial Chancellor. Twice every year, and this is their chief function, they draw up a report concerning the recognition and classification of schools, with regard to the Volunteer Certificate or Army Qualification. Next come the Ministers of the German States with their legions of officials. The Ministry, in Prussia, is chiefly occupied in superintending, in a very general way, all examinations and also the relative standards of the Colleges. It arranges the “normal” or typical plans or instruction for each species of schools ; it fixes the salaries and pensions of the teachers, together with the fees of the scholars ; but leaves the special supervision of individual establishments to its provincial departments. In Bavaria, where the State is the main supporter of all intermediate schools, the interference of the Government penetrates every detail of school-life. Thus the teachers in Bavaria may not, as they do in Prussia, propose the books to the authorities, but must choose them from lists which the authorities propose to them.

In nearly all States we meet with an examination Body, permanent or otherwise, which tests the qualifications of the teachers, and watches over the leaving examinations of the Colleges. Such bodies exist at Königsberg, Berlin, Greifswald, Breslau, Halle, Kiel, Göttingen, Münster, Marburg, Bonn, Strassburg, Leipzig, Giessen, Rostock, Jena. All of these are University towns, and the examiners University professors. In the South German States, either a branch of the Head Department, or an annually appointed Commission, undertakes the same functions.

Last of all, we have to notice the Royal Commissioners and Inspectors, who superintend and personally visit the schools. In Prussia every province has its Educational Department, with one or two—generally one for the Roman Catholic, one for the Protestant schools—Councillors. These important functionaries issue regulations to all the Colleges of their particular province, propose remedies for all existing defects and abuses, examine the books to be introduced, superintend the leaving examinations, and periodically inspect the schools. They appoint, promote, and dismiss the masters, and, in Schools under State patronage, propose the headmasters to the Ministry. These School Councillors arc, men of practical experience in teaching, generally fonner headmasters In other German States, Ministerial Commissioners fill the same or a similar office.

I have very little more to add. The educational administration of the German States has produced results which private efforts could never have attained. If it has appropriated the Town Schools, which were founded to satisfy local rather than governmental wauts ; if it has forced all schools, whether public or private, into the same mould, and has crushed private enterprise like a hostile rival ; if it has made of every schoolmaster a bowing official, and has encroached upon the natural right of the parent ; it has also, by its vigilance, raised the efficiency and dignity of the teachers, increased the number of Colleges and Collegians, and checked the over-hasty despatch of unripe lads to the labour market.

It is proposed to hold a Grand International Juvenile Industrial Exhibition in the Garden Palace, Sydney, at the close of the present year. Initiatory steps have been taken to carry out the proposition. Many friends of the youth think this an opportune time to hold such an Exhibition, having the following objects in view :—1. To stimulate the energies and display the abilities of the rising generation of the colonies, 2. To encourage a spirit of emulation by competition. 3. To develop the inventive, constructive, and artistic faculties of all, and to gather new ideas from the works of others. 4. To create a taste for the fine arts and ornamental works, which will incite them to occupy their leisure hours in making home attractive with the results of their industry.

5. To assist the charitable institutions by distributing amongst them the proceeds of the Exhibition. The committee invite the young people of both sexes, under twenty-one years of age, in all the Australian colonics, to assist in making the Exhibition a success by contributing their handiwork.

NOTES OF A. LESSON ON THRIFT.*


Points.


Matter and Method.


Blackboard

Summary.


Points.


Matter and Method.


Blackboard

Summary.


(c) Clothes.


What Thrift is,

AND IS NOT.


HOW TO EXERCISE

Thrift in


(a) Money,


(b) Time.


Tell children how some people think that thrift means stinginess and not spending, but hoarding money. Point out, that if we were to put by money, and go without things we needed—such as food, or clothes, or medicine — we should not be thrifty.

Thrift is the habit of using all things for the best.

Point out, that we have many things which we may use well or ill—money, time, health, clothes, and little things of daily use.

1.    Tell children, that we should put by money only that we may use it to better advantage. If we wanted bread, or clothes, or medicine badly, and put our money away instead of buying these, should we be using our money to the best advantage ?

2.    Point out the folly of spending oue’s money in drink on a Saturday night, instead of keeping it for food during the week ; of feasting on Saturday and Sunday, and going to the pawn-shop on Monday morning.

3.    Elicit, or point out, necessity of “putting by for a rainy day ” against sickness, old age, loss of work, etc.

Point out, that people with small earnings can save as well as those who earn more ; that thrift consists in spending less than one earns, be it much or little. Advise them that if they have “ only a penny ” to save, to save it, instead of spending it. Tell them that most things are done ‘‘little by little,” One’s life is made up of little minutes ; big rivers are made by little brooks; and pennies saved soon become pounds.

Tell children, the story of the man who used to spend “only twopence ” a day on beer that he didn’t need ; how he put it by every day for a year, and found himself growing rich.

Let children work out the sum, and find out his yearly savings.

4.    Tell children how we may best save our money for sickness, age, etc.:—

(a)    By Penny Banks;

(b)    By Post-office Savings Banks;

(c)    By investing in Benefit or Building Societies;

(d)    By purchasing Annuities.

1.    Point out that we may waste time just as we waste money—

(a)    By spending it in what is not profitable to us — by idling when we should be working.

(b)    By paying only half attention to what we are at, and so doing neither one thing nor another well.

(c)    By half play (as well as half work) in poring over-, and muddling ourselves with books, when we ought to rest our minds, so as to be refreshed and fit for new work.

2.    Impress on children how important it is to use our spare moments well. Tell of men who have learnt foreign languages, by studying them only when they had “ five minutes ” to spare ; of men who have written books, by giving odd moments to them, when resting from harder work.


Thrift is the habit of using all things to the best advantage.


Thrift should be exercised

(a) In putting by money for a rainy day.”


(b) In using time well, and not wasting it.


' ‘ MefTit's How to Prepare Not» of Lesson«.” By T. J. Llvesey.


(d) Small things.


Summary.


1.    Point out, that some children are always having new'clothes, and are always in rags, because they use them roughly, and do not take care of them.

2.    Tell children how they can use their clothes to the best advantage—

(a)    By brushing them and keeping them clean ;

(b)    By taking off good coats in the house, and putting on old ones to sit or work in ;

(c)    By keeping clothes in repair.

“ A stitch in time saves nine.”

1.    Point out, that children should take care of their books, pens, pencils, etc.

2.    Warn children against wasting matches, salt, bits of bread, candle ends, etc.

Tell the story of the lady who scolded her servant for masting a match, and gave liberally to charities.

3.    Point out, that we should use no more than is needed of anything; beyond that, is extravagance. Impress upon children the fact, that if we are to become thrifty, we can only do so by being careful of little things; that “many a mickle makes a muckle/’and that if we—

Talie care of the little things, the big things mill take care of themselves.

Question on lesson, and summarize it.


(p) In taking care of one’s clothes.


(d) Tn taking care of small things.


GRAMMAR EXERCISE.

BY ROBERT CRAIG, M.A., BRIGHTON.

1.    Parse with full syntax the words in italics in the following :—

“ Oh,” cried the goddess, “ for some peddnt reign I Some gentle James to bless the land again ;

To stick the doctor’s chair into the throne,

Give law to words, or mar with w'ords alone;

For sure, if Dulness sees a grateful day,

’Tis in the shade of arbitrary sway.

Oh ! if my sons may learn one earthly thing,

Teach but that one, sufficient for a king ;

That which my priests, and mine alone, maintain,

Whioh, as it lives or dies, we fall or reign •.

May you my Cam and Isis,preach it long,

The right divine of kings to govern wrong.”

Prompt at the call, around the goddess roll Broad hats, and hoods, and caps, a sable shoal :

Thick and more thick the black brocade extends,

A hundred head of Aristotle’s friends.

Nor 7vert thou, Isis, wanting to the day (Though Christ-Church long kept prudishly away)

Each staunch polemic, stubborn as a rock,

Each fierce logician, still expelling Locke,

Came whip and spur, and dashed through thin and thick,

On German Crouzazand Dutch Burgersdyck.

Pope, The Dunciad.

2.    Analyse according to Morell’s second scheme the following :—

My mother ! When I learnt that thou wast dead,

Say, wast thou conscious of the tears I shed ?

Hovered thy spirit o’er thy sorrowing son,

Wretch even then, life’s journey just begun ?

Perhaps thou gav’st me, though unfelt, a kiss,

Perhaps a tear, if souls can melt in bliss.

Cowper, Lines on his Mother's Picture.

3.    Give fully the derivation of the following words, the meaning of the component parts, and the language from which each is derived :—Wood-stock, bibliophile, bracelet, commingle, collateral, diagnosis.

4.    How are Primary Derivative Verbs formed ? Give examples.

5.    Morell gives a list of words which may be used as “ Connectives the Adjective Sentence.'’ Name them, aud give examples.

6.    Write each of the following sentences in a correct form, and give the reason for each correction :—(a) When we seen these sort of things going on we run off as quickly as we could. (b) A lot of us, as knowed the lay of the country, started for Mount Feathertop. (c) I would not dress like you do without you paid me for it.


ANSWERS.


7. Scan the following verses, giving the name and number of feet in each line :—

Sweet are the uses of adversity,

Which, like the toad, ugly and venomous,

Hides yet a precious jewel in his head.

Know ye the land where the Cypress and Myrtle Are emblems of deeds that are done in their clime.

1. Parsing.

pedant    Adj. of quality, attributive to noun “ reign ”

James    Noun proper, 3rd per. sing., obj. case after the prep.

“ for ” understood. It would also be admissible to say that James is in apposition with reign.

war    Verb reg., intrans., inf, mood, pres, tense, attributive to

James.

for    Conj. cop. continuative joining the sentence implied in “ Oh,

for some gentle James to war with words alone/’ and the sentence “ ’Tis in the shade.” The first sentence would be similar to this :    “ How I wish for some gentle James !”

sure    (by poetical licence for surely) adv. of mood, modifying the

predicate “is in the shade.”

teach verb irreg. trans. imperative, present, 2nd per. plur., agreeing with its subj. “you ” understood,

but    adv. of quantity, modifying the sentence “Teach that one,”

that is, “ teach it without teaching other things, teach that one exclusively. ” Some may be disposed to see in hut the character of an adj. limiting one used as a noun. But the former view seems preferable.

sufficient adj. of quantity describing one which is used as a noun; that    demonstrative pronoun, 3rd. per. sing., objective case, in

apposition with “ that one.” It may also be called a distinguishing adj. pointing out “ thing ” understood, which would be in the obj. case in apposition with that one.” which Pronoun relative, referring to its antecedent “ that,” 3rd per. sing., obj. case to “ maintain.”

mine Pronoun personal (equivalent to my priests), 3rd per, plural, nom. case (or subject) to maintain, understood, the “ maintain ” expressed is required as the predicate of “ my priests.” alone May be taken in two ways. First it may be taken as an adv.

of quantity modifying the sentence “mine maintain,” it shows that the predicate “ maintain ” is restricted to “ mine,” that it cannot be affirmed of other priests. Suppose we had this statement, “ Not my priests alone, but others also, maintain it.” Are not alone and also performing similar functions, namely, adverbial functions? In Latin where the adjective has inflexions, the sentence would run: “Non solum mei sacerdotes, sed alii etiam, id defendunt” (solum not soli, is used, the adv. not the adj.) Again, let us in the sentence “ Mine alone maintain it,” substitute for “alone” some equivalent phrase or sentence, and we have “mine maintain it, no others maintaining it,” when the phrase “ no others maintaining it,” is an extension of the predicate; and this phrase correctly expresses the force of the word “ alone ” in the original sentence ; therefore this word is an adverb, not an adjective. Further, if we expand alone into a sentence, we get an adverbial sentence for it, “ Mine maintain it, while no others do so,” where the adverbial sentence “while no others do so ” appears to give correctly the force of the word “ alone ” in the original sentence. Secondly, if we regard alone not as restricting the application of the predicate to “ my priests,”but as describing the condition or state of my priests, describing them as alone, unassisted by others, in maintaining the doctrine referred to, then we should parse alone as an adjective. Compare the use of the adjective sola in this sentence :

“ Deorum nominibus appellant secretum illud quod sola re-verentia vident ” ; that is, “ they call by the names of gods that hidden thing which they perceive by reverence alone [or, which they do not perceive except by reverence.]” Tac. Germ. 9.

Having given the reasons for each view, I think I shall best serve the reader by leaving him to exercise his intelligence in deciding which is the better parsing, adverb or adjective, which Relative pronoun, referring to its antecedent “ that,” 3rd per, sing. It performs no office as either subject or object to any sentence ; it is therefore ungrammatical ; but it is inserted in the line partly to complete the metre, and; partly to show what it in the sentence “ as it lives ” stands for.

Isis    Proper noun, 2nd per. 6ing., nom. of address,

preach Verb reg,, trans., active, infinitive, present, being the principal verb to the auxiliary may, with which it here forms a soft imperative or an optative, “May you preach it,” being equivalent to, “ you are desired or requested by me to preach it.”

long    adv. of time (duration) modifying the verb “ preach.”

right Noun abstract, 3rd per. sing., obj. casein apposition with it, which is governed by “preach.”

to govern Verb reg. trans. (some obj. like “ their subjects “ or “ people ’ being implied), infinitive mood, pres, tense, attributive to the noun “right.”

wrong    Adv. of manner, modifying the verb “ to govern.”

prompt    Adj. of quality, pos. deg., attributive to “hats, hoods, and

caps.” To call it an adv. modifying “ roll ” might be admissible, but it seems more closely connected with “hats,” the subject, than with “ roll,” the predicate.

roll    Verb reg., trans., middle voice (following Mordi), or intrans.

active, indie, mood, pres, tense, 3rd per. plur (agreeing with its subject “ hats, hoods, and caps.” shoal Noun common (collective), sing,, 3rd por., nom. case in apposition with “hats,” &c. It is called singular, as expressing the one dense mass made up of the parts just specified : “ hats, hoods, and caps.”

thick adv. of 'quality modifying the verb “extends.” In this case “thick ” does not seem to be so closely connected with the subject “ brocade ” as with the predicate “ extends,” Compare the parsing of prompt above.

a    Numeral adj., meaning “ one,” limiting “hundred” used as

a noun, “a hundred head” being reasonably taken as an abbreviation of “ a hundred of head.” wert Verb irreg., intrans., neu., indie, mood, past tense, 2nd pers.

sing., agreeing with its subject “ thou.” This form, wert, is usually confined to the subjunctive, but the poets sometimes, as here, use it in the indicative.

wanting Participal adj. attributive to “ thou.” “Tobe wanting” can hardly be called the progressive form of “ to want,” because the sense and the construction of the one are different from those of the other. We could not say : “ Thou wantedst to the day,”

rock    Noun common, 3rd per. sing., nom. to “ is ” understood,

spur    Noun used adverbially as partof the phrase “ whip and spur,”

modifying the verb “ came.”

thick Noun abstract, 3rd per. sing., obj. case after prep. “ through.”

2. Analysis.

Sentence.

Kind of Sentence and relation.

Subject

with

Attributes

P edicate

| Object ' with Attributes

Extensions of Predicate.

(a) When I learnt

Adverb ot rime to (d) and (f)

(When)

I

learnt

: Sentence ¡(*)

1

Nil.

(h) that thou wast dead

Noun obj. to learnt in («)

(that)

thou

wast

dead

Nil.

Nil.

(c) Say, thou, my mother

Parenthetical principal sentence

thou, my mother

say

(the answers to the questions which answers are understood)

Nif.

(d) wast thou conscious of the tears

Principal to (a) and

(e)

thou

wust conscious

of the tears (indirect )

Nil.

(«) I shed

Adj. to tears in (d)

I

shed

(which

direct)

Nil.

(f) Hovered thy spirit o’er thy sorrowing son, wretch even then, life’s journey just begun?

Principal to (a) co-ordinate with (d) and the connective understood

thy spirit

hovered

Nil.

o’er thy sorrowing son (place); wretch even then (phraso in app. with son); life’s journey just begun, adv. phrase, nom. alls, modifying “wretch” used as an attribute.

(g) Perhaps thou gav’st me a kiss

Principal to (i)

thou

gavest

me

(indirect) a kiss (direct)

Perhaps (mood)

(h) Perhaps (thou gav’st me) a tear

Principal to (i)

thou

gavest

mo

(indireel) a tear (direct)

Perhaps (mood)

(i) though (it was) unfelt

Adverbial (concession) to (g) and (h)

it

was

unfclfc

Nil.

Nil.

(j) if souls can melt in bliss.

Adverb of condition to {ff) and (A)

(If) souls

can melt

Nil.

in bliss.

3.    See Morel 1’« Grammar and Chambers’ Etymological Dictionary.

4.    & 5. See Morell’s Grammar.

6. [a) “When we seen these sort of things going on, we run off as quickly as we could.” It should be : “ When we saw this sort of things (or things of this sort) going on, we ran off as quickly as wo could.” Seen the past part, is a vulgarism when put for the past tense saw. Sort is singular, and requires this, the singular, to limit it. Run, past part, is incorrectly used for the past tense ran.

(b)    A lot of us, as knowed the lay of the country, started for Mount Feathertop.” Jot and lay as here used are colloquial and not classical yet, though they are occasionally seen in print. As is a vulgarism for who, when it is not preceded by such. Knowed is a vulgarism for knew. In a correct form the sentence would be ; “A great number of us who ' knew the nature of the country (or the character or formation of the country), started for Mount Feathertop.”

(c)    “I would not dress like you do without you paid me for it.” Lilte and without are here used as conjunctions, a use which is not supported

I would not dress as you


Sw6et are Which, like Hides ytit

the u

ses óf

advér-

the tohd,

figiy

and vèn-

a prb-

cious jéw-

el in

sity omoùs his liéad


Myrtle

in their clime


Cypress and that are d6ne


Knów ye the | lànd where the Are 6m-


by good writers. The sentence should run do unless you paid me for it,”

This is Iambic pentameter, the prevailing foot being the iambus, and there being five feet in each line. The first foot in the first line is a trochee, and the third foot in the second line also is a trochee, ugly. The verse accent differs from that of the pronunciation in the last syllables of “adversity ” and “venomous.” blems of deeds

The first line contains four complete dactyls with the final foot one syllable short of being a dactyl; it is therefore Dactylic pentameter cata-lectic, catalectic meaning defective. The second line contains an iambus followed by four anapaests. As the prevailing character of the line is anapaestic and it has five feet, it is described as anapaestic pentameter.

ON THE GRAMMATICAL ERRORS OF MORELL.

By Alex. Gough, H.T., S. School 972, Cobaw.

PAGE 18.— “An auxiliary verb is one which aids in forming the voices, moods, or tenses of other principal verbs.” An auxiliary verb is not a principal verb, and when it becomes a principal it ceases to be an auxiliary. I cannot say, “An aasistant teacher is one who aids other principal teachers in large schools.” I may use the word other before teachers, because both are teachers, but not before principal teachers, because they are not hath principal teachers. The sentence should therefore read, “ An auxiliary verb is one which aids in forming the voices, moods, or tenses of principal verbs.”

Page 76,—1. “The indirect object may consist of another noun or pronoun in the objective case. This is called by the German grammarians the factitive object, since the verb to make is the type of that whole class of verbs which admit of this construction.” 2. “ The indirect object may consist of a noun or pronoun with a preposition.” On last page he defined indirect object. Now he evidently wishes to define factitive, dative, and genitive objects. These three must, of course, be included in the first. Definition, to be good, should express and explicate all that is in the thing defined, and nothing more. All, because, otherwise, it would be incomplete ; and nothing more, because, if it contained anything more, the defined thing would be confounded with things distinct from it. Therefore, to be certain that a definition is perfect, it should be proved, applying it to the thing defined, according to the following rule :—The definition should correspond to all the thing defined and to nothing more. Morell has not, in defining the factitive object, excluded the dative, nor in the dative excluded the factitive. In the sentences, “He gave permission to John,” “The fire reduced the house to ashes,” we have the same preposition, and, for indirect objects, a noun with a preposition, and a noun in the objective case. They are therefore both factitive objects, aud both dative objects 1 As it is possible for the factitive object to take the same preposition as the dative, he should have excluded the latter by saying, “ The factitive object represents the direct object repeated by a different noun, or by an adjective with the noun of the direct object understood, and is therefore in apposition to the direct object.” Neither has he defined correctly dative object, as is evident from the examples just given.

Page 109.—“ If the adjective be qualified by some adverbial expression, or accompanied by another adjective, it usually follows the noun.” The word usually shows the state alluded to as having had a past existence, and therefore the conjunction if (or more properly when) must be followed by the indicative. He should have said, “The adjective usually follows the noun, when qualified by some adverbial expression, or accompanied by another adjective.” Morell would have made less mistakes in tense, and be more perspicuous, if, in his Rules of Syntax, he had used when instead of if as is evident from the above rule and the two here following.

Page 106.—“ If there are two or more possessives together, the ‘s’ and the apostrophe are affixed only to the last.” Here the tense is quite correct, the conjunction if being equivalent to when. But on next page it is different. I will give Morell’s rule and then Crombie’s, and italicize the verbs of both.

Morell.—“ If no noun come between the relative and the verb, the relative is in the nomiuative case; but if a noun come between, that noun will be the subject of the verb, and the relative will be the object.”

Crombie.—“If no nominative intervene between the relative and the verb, the relative shall be the nominative to the verb. But if a nominative intervene between the relative and the verb, the relative shall be under the government of the preposition going before, or the noun or verb following.”

Page 109.—“ If one verb depends upon another, they must observe a proper succession of tenses.” Burston notices this sentence, and passes it as quite correct. He quotes it as au example of the conditional mood, and says the conditional mood is generally formed by putting if before the indicative form. Conceded. But what is the noun for which the pronoun they stands 'l It will not do to say, one verb and another. Morell can predicate of the first verb only, and must use a pronoun of the singular number. As he desired to predicate of both verbs, he should have said, “ If there be two verbs, one of which depends upon the other, they must observe a proper succession of tenses,”

Morell has given us some foreign information about case, page 29. It is a fair question for an inspector or a teacher to ask a boy, “What is case ?” and Morell should have given the answer. Again, he has not defined a preposition very well; but this seems no easy matter. In the adverb, it would have been better to substitute for the word attribute its equivalent, “ verb adjective or adverb,” the word attribute being less understood than the word to be defined.

Page 72.—“ The adverb of place or time is sometimes employed as a complement to the noun, as, 1 The church here is very fine.’ ” A boy would therefore be parsing according to Morell, if he said, Here, an adjective qualifying the noun, church. Is it because here is a word in the adjective sentence, Which is here, that it may be called an adjec'ive ? Would] Morell call every word in an adjective sentence an adjective ?

Page 106.—“ The verb to be takes two nominatives, the one before and the other after it.” This is saying more than enough, and therefore saying what is not true. Crombie, page 219, Davidson and Alcock, page 65, Sullivan, page 125, say, “ The verb to be takes the same case after it as before it.” Morell has said too much, and these grammarians too little. It is safer to say too little, but much better to say all that should be said and nothing more. The rule should have been expressed thus : “ The verb to be takes the same case after it as before it; when finite, it must be preceded and followed by the nominative case. ” Burston, following the rule as laid down by Morell, gives two sentences to prove that the infinitive to be must be followed by a nominative case : “ I know this man to be he,” I believe him to be a rogue.” He calls he nom. after to be, and in apposition with man ; rogue nom. after to be, and in apposition with him. The sentences should read, “ I know this is he,” “ I believe he is a rogue.” It matters not here how we would convey the same ideas in Latin. The above is good English. Besides, Burston is wrong in saying a nominative can be in apposition with an objective. The rule, as I have given it, is applicable in all cases. The infinitive having no subject, the case which precedes it is entirely independent of it. Some grammarians seem to forget this. They take their stand in Rome, the centre of ancient civilization, and address us in Latin. I respectfully advise them to go West (London). There is a greater empire, a higher civilization, and a more wide-spread language than the Roman.

“ Regions Csesar never knew,

Thy posterity shall sway ;

Where his eagles never flew—

None invincible as they.”

We must acknowledge the relationship between languages as between nations. But, is it not as absurd to expect the English language to be governed by the laws of the Latin Grammar, as to expect England to be governed by other than the British Constitution ? A grammarian can no more make a language, or even introduce laws from foreign languages, than a politician can make a constitution for a nation. Constitutions and languages are generated, not made, and everything necessary for their advancement to perfection is found within themselves.

Morell is not only ineorrect'in his rule for the verb to be, but contradicts himself. He says, page 106, “A noun or pronoun, standing in apposition to another noun or pronoun, is in the same case with it. The cognate nouns before and after the verb to be may be considered as in apposition with each other.” Apply these rules to the examples given, page 76. King and Edward, heir aud him are in the objective case in contradiction to his own rule for the verb to be. Again, in his Graduated Exercises, page 51, he gives, for correction, the sentence, “ I do not know who you profess to be.” I do not know whom you profess (yourself) to be. The trans. verb profess governs yourself (understood) ; and the infinitive “ to be ” takes whom (an objective) after it in apposition to yourself.

PAPERS FOR TEACHERS AND STUDENTS.

We take the following extracts from the March number of Papers for Teachers and Students.

ANSWERS TO QUESTIONS IN NATURAL SCIENCE.

By a B. Sc. (Honours) London University. May Examinations, 1881. Sound.—Elementary stage.

“ Sound is said to travel about four times as fast in water as in air. How has this been proved ? State your reasons for thinking whether sound travels faster or slower in oil than in water.”

The question refers to two points. First, (a) The velocity of sound in both air and water has been found by actual observation. Two stations

58,000 feet apart were chosen near Amsterdam, and a cannon was fired at one, while at the other a person observed the time which elapsed between the flash of light striking his eye and the report striking his ear. The data thus furnished gave 1000 feet per second, (b) Two boats were moored at a known distance asunder on the Lake of Geneva. From one was suspended a bell which had a lever attached to it, so fixed, that on its being worked one end struck the bell at the identical moment that the other inflamed some gunpowder. In the other boat was an observer, provided with an ear-trumpet, the receiving end of which was placed in the water, so that he could note the interval of time between the flash, that is between the production of the sound, and the perception of it by the ear after having travelled through the water. The velocity in water was thus found to be 4,708 feet per second, or about four times the velocity in air. Second, The velocity of sound in any medium depends on two elements, the elasticity and the density. It varies directly as the square root of the elasticity and inversely as the square root of the density.

“ I lov’d her from my boyhood —she to me Was as a fairy city of the heart,

Rising like water-columns from the sea,

Of joy the sojourn, and of wealth the mart.”

Byron : Childe Harold.

Sentence.

Kind.

Subject.

Predicate.

Object.

Extension

of

Predicate.

Connec

tives.

{a) I loved her from my boyhood

Principal

Sentence.

I

loved

her

from my boyhood

(b) She to me was, rising from the sea, of joy the sojourn and of wealth the mart

Principal Sentence, co-ord. with {a)

she, rising from the sea, of joy the sojourn and of wealth the mart

was

to me

(c) As a fairy city of the heart

Adv. Sent. {manner) to {b)

a fairy city of the heart

(is)

as

(d) Like water-columns.

Adv. Sent. {manner) to (6)

water-

columns

(rise)

like

(equal

as)


Both these elements must be known in order to obtain the velocity. Oil is generally lighter than water, and so far we should expect a higher velocity ; but as I have no special knowledge of the elasticity, I cannot positively decide the matter.

SoundAdvanced stage.

“Two tuning forks sounded together give four beats in a second. How would you find experimentally which of the two gives the highest note?”

Bring one of the strings of the monochord into unison with one of the forks. This may be done by altering the weights attached to the string. Note accurately the weight required. Go through the same operation for the other fork, again noting the weight required. The fork which has been in unision with the string when the heavier weight was attached is evidently that of higher pitch.

ElectricityElementary stage.

“ If you have a penny piece fastened to a stick of sealing wax, how could you give it a negative charge by the help of a positively^charged glass rod ?”

Bring the glass rod within two inches or so of the penny, and keep it in that position while you just touch the penny with your fiDger. The penny is now charged as required. Explanation : when the glass rod is near the penny it decomposes its neutral fluid, binding the negative and repelling the positive constituent. The moment the coin is touched the repelled kind flies through the body to earth, leaving it charged with negative.

Electricity—Advanced stage.

“ A positively electrified metal ball is hung by a silk thread above a goldleaf electroscope. Would the divergence of the leaves be altered ? (and if so, how and why) by putting (1) an unelectrified cake of resin, or (2) a metal plate held in the hand between the ball and the electroscope, so as not to touch either.”

Yes, the divergence would be altered in both cases. (1) One of the factors in the inductive effect of any electrified body is the di-electric. When the di-electric has a high specific inductive capacity, the effect is greater than when it has a lower. The specific inductive capacity of resin is If as much as that of air, and accordingly on the insertion of a resin cake as described in the question, an increased divergence must ensue. (2) When the metal plate is interposed, the metal ball acts inductively on it, attracting negative and repelling positive, and so spends itself as it were. Its influence then does not extend to the plate of the electroscope in its integrity, and therefore the leaves converge. The amount of convergence depends on the size of the metal plate and on its distance both from the electrified ball, and from the plate of the electroscope.

Inorganic Chemistry—Elementary stage.

“ How many volumes of carbon dioxide (carbonic acid gas) will be formed when a mixture of four volumes of carbon monoxide (carbonic oxide gas) and four volumes of oxygen is burnt ? State what volume of oxygen, if any, remains uncombined.” (This is the most highly marked question.)

The volume of carbonic acid gas produced is always equal to the volume of CO burnt. Hence four volumes of C02 are produced, and two volumes of oxygen remain.

The question is thus answered strictly, and probably no more is required for an elementary paper ; but in a more advanced examination, some account of the law from which this fact may be inferred would be expected.

Chemistry—Advanced stage.

The platinum electrodes of a powerful battery are placed in a solution of sodium sulphate, describe the changes which occur in the solution. How by means of electrolysis have the metals of the alkalies and the alkaline earths been obtained ?” (The most highly marked question.)

W'hen salts are reduced to the liquid state either by solution or fusion, and subjected to electrolysis as described in the question, the negative constituent (the acid) of the salts is deposited on the positive electrode, and the positive constituent (the metal) on the negative electrode. The change which takes place in the solution in question, is therefore a breaking up of the compound into free sodium, thrown down on the negative electrode, and H„S04 at the positive pole. The free sodium will again react on the H2S04 orHaO present, setting hydrogen free, with formation of NaHO, which combines with the disengaged sulphuric acid to form sulphate of sodium and II20 once more.

Davey was the first to decompose KHOand NaHO by placing them in a cup, which was made the electrode of a very powerful battery. The chlorides of the metals of the alkaline earths, when fused and subjected to electrolysis, are decomposed and the metals thus isolated.

Physiography—Elementary stage.

“ What chemical elements are present in water ? How can water be separated into its elements, and how can these elements be made to combine to form water ?” (One of the most highly marked questions.)

(1) Hydrogen and oxygen are the elements of water. (2) A beaker or other wide-mouthed vessel has two platinum wires passing into it through its bottom, and to their ends are soldered two pieces of platinum foils, Some water with a little sulphuric acid is placed in the vessel. Over each of the platinum foils is placed a large test tube filled with water. The poles of a battery are connected to the outer ends of the platinum wires above mentioned. Hydrogen bubbles up from the negative electrode and oxygen from the positive. The water is thus being decomposed.

(3) Hydrogen and oxygen may be made to combine to form water by burning a jet of one in the other.

Physiography—Advanced stage.

Describe the changes which take place in the volume of a pound of water, as its temperature is raised from 0° F. to 300° F.” (One of the two most highly marked questions.)

From 0° F. to 32° F. it does not change its volume. (Water may be retained as such below the freezing point by pressure, as shown by the brothers Thompson.) From 32° F. to 39° it contracts in the ratio of 100012 : 100000. From 39° F. upwards it expands, but not uniformly. It expands from 39° F. to 120°F. in the ratio of 1000 to 1010 ; from 120°

F. to 248°F. in the ratio of 1010 to 1060 ; and from 21S° F. to 300° F in the ratio 1060 to 1106.

ANALYSIS AND PARSING.

“.Thus, Venice, if no stronger claim were thine. Were all thy proud historic deeds forgot,

Thy choral memory of the Bard divine,

Thy love of Tasso, should have cut the knot Which ties thee to thy tyrants.”

Byron : Childe Ilarold-.

Sentence.

Kind.

Subject.

Predicate.

Object.

Extension

of

Predicate.

Connec

tives.

(a) Thus,Venice, thy choral memory of the Bard divine, thy love of Tasso, should have cut the knot

Principal

Sentence.

(Venice) thy choral memory of the Bard divine, thy love of Tasso

should have cut

the

knot

[thus

(b) If no stronger claim were thine

Adv. Sent. {condition) to (a)

no

stronger

claim

were

thine

(c) Were all thy proud historic deeds forgot

Adv. Sent. {condition) to (a)

co-ord. with

{b)

all thy proud historic deeds

were

forgot

(if)

(d) Which ties thee to thy tyrants

Adj.Sent. to (a)

which

ties

theo

to thy tyrants

(Notesto parsing.)

Thus i Conjunctive adverb, introducing sentence (a), and modifying verb “ should have cut.”

Venice Nom. of address (not entering into the construction of the sentence).

were Subjunctive mood.

thine Pronominal adjective, qualifying “claim.” thy Possessive case of “ thou.” Some grammarians call this a pronominal adjective, or adjective pronoun, forgot For “forgotten.” Perf. part., referring to “deeds.”

(Notes to parsing.)

8°^mart} | Nouns in apposition with she,”

[Note.—The parsing of the other words is clear from the Analysis.]

“ I can repeople with the past—and of

The present there is still for eye and thought,

And meditation chasten’d down, enough ;

And more, it may be, than I hoped or sought. ” .

Byron : Childe Harold.

NOTICE TO SUBSCRIBERS.

Sentence.

Kind,

Subject.

Predicate.

Object.

Extension

of

Predicat e.

Connec

tives.

(o) I can repeople with the past

Principal

Sentence.

I

can

repeople

with the past

(b) And of the present there is still for eye and thought, and meditation chasten’d down, enough

Principal Sentence, co-ord. with (a)

enough of the present for eye and (for) thought and (for) meditation chasten’d down

is

there

still

and

(c) And more

Prin. Sent, co-ord. with

(a) and (b)

more

(is

there)

and

(d) It may bo

Adv. Sont. supposition to (c)

it

may be

(e) Then I hoped

Adv. Sent. {degree) to (c)

I

hoped

than

(/) Or feared

Adv. Sent.

(idegree) to (c) co-ord. with (e)

(I)

foared

(than)

“Allan Baxter,” Tasmania.—Received, “John T. Wallis.”—June 30th, 1882.

“ R. N. Cole.’’—December 31st, 1882.


CON I ENTS

School Department—

Leader ............

152

The Growth of Intermediate

Notes of the Month ...

153

Education in Germany ...

146

Science and Art Gossip •••

154

Notes of a Lesson on Thrift

148

The Revised Education Code

154

Grammar Exercise ... ...

148

St. Andrew’s University ...

156

On the Grammatical Errors

Melbourne University ...

158

of Morell ... ... ...

150

Victorian Education Department—

Papers for Teachers and

Appointments ... •••

150

Students ... ......

150

Kinder Garten.........

159

Answers to Objections and Queries in Grammar ...

152

Velocity of Sound and Light

159

ANSWERS TO CORRESPONDENTS.


We would call the attention of those of our Subscribers whose Subscriptions are in arrear to the necessity for prompt payment of the same.

(Notes to Parsing.)

past |

present > Adjectives used as nouns, enough I

still Adv. of time, qualifying “ is.”

ANSWERS TO OBJECTIONS AND QUERIES IN GRAMMAR.

By Robert Craig, Brighton.

Some teachers have experienced difficulty in accepting beside as an adjective, so parsed in the February issue of this journal. Here is the passage in which it occurred :—

“ Valour, religion, friendship, prudence, died At once with him, and all that’s good beside.”

Now, “ all that’s good beside” means “ every good thing beside valour, religion, etc.,” where the phrase beside valour, religion, etc.” is adjectival to the notion expressed by “ every good thing,” and in the original sentence this adjectival phrase is represented simply by the word beside, which was therefore parsed as an adjective. Again, we can substitute the adjective else for beside, and not alter the meaning ; All else that’s good (died with him); everything else that’s good (died with him).

Others have found difficulty in seeing that e'er modifies, not “ so gently,” but “ smiled,” in the following lines : —

[He was] always tempered with an air so mild,

No April suns that e’er so gently smiled.

The sense is more obvious when the ellipses are filled out, thus :—He was always tempered with an air so mild that there have been no April suns that ever smiled so gently as he smiled.

Nature" in an article on the vegetable food of the New Zealander, in prehistoric times, says:—The Ancient New Zealander had plenty of good food, but only such as was to be obtained by labour. For them nature had no lavish gifts, no breadfruit, no cocoanut, no plantanes or bananas, fruits from trees growing almost spontaneously and yielding without toil their delights to mankind. Button the contrary, the Maories got their vegetable food by constant industry and hard labour, and this was doubtless in favour of the development of their race, helping the “survival of the fittest.” And not only were they great cultivators of the soil, but when first known they were in a state of civilization far beyond that in which our own forefathers were when Crnsar first led his victorious army among them; indeed Colenso doubts if any ancient people had ever (wanting the knowledge of metals) advanced so far ; and he in a very pleasant manner reminds us thaljas Xenophon remarked, “Agriculture is the nursing mother of the arts,” and that the agriculturist is bound to the soil ; it becomes sacred to him ; he is compelled to build houses ; unlike the nomad shepherd. Hence comes the town ard then the fortified places of strength, all of which the Maories had, and none of which their neighbours, the Australians and Tasmanians ever dreamt of. One of the oldest legends of the Maories treats of tbeir favorite hero Maui, as catching and binding the sun to prevent his travelling so fast, so that man might have longer daylight to work in. In their plantations all worked alike, the chief, his wife, and slave. It was a pleasing sight to see the evenness of their village, and the regularity of their planting and sowing.

NOTICE TO ADVERTISERS.

In sending advertisements for insertion in the Schoolmaster, advertisers will please remit stamps for amount at the following scale : —

16 words, One Insertion, - Is. Od. | 32 words, One Insertion -    2s. 6d.

24    „    ,,    -    2s. Od. 1 One Inch,_„_-    4s. Od.

NOTICES TO CORRESPONDENTS.

Advertisements and other business communications shoirld be addressed to the Publishers. No advertisements will be inserted without a written order, or prepayment. It is particularly requested that they may be sent early in the month.

Books, music, and school appliances for notice, and all letters containing anything connected with the literary portion of the paper should be addressed Po the Editor. Every communication accompanied by the name and address of the sender (as a guarantee of good faith, though not always for publication) will be acknowledged; but wo cannot attend to anonymous letters.

INSTRUCTIONS TO SUBSCRIBERS.

Lady subscribers, when remitting their subscriptions, will please state whether their papers are to be addressed Mrs. or Miss.

Subscribers will please send P. O. order or stamps, when remittance is under ¿£1.    __ _

Australasian Srljnolmastrr.

PUBLISHED EVERY MONTH.

MELBOURNE, APRIL, 1882.

India, as well as the colony of Victoria, has a Royal Commission now occupied making investigation into the working of State Elementary Education. The Indian Commission has been selected from three distinct sections of the community— namely, the highest officers of the Education Department, the Missionary bodies, and the leading natives. To which must be added one representative from the Finance Department, and the President, who represents the Supreme Government. The questions referred to the Commission amount to a public trial of State Education throughout British India, and its reconstruction on a sounder basis. Considering the similarity of function to be discharged, it is matter for regret that the Victorian Commission was not somewhat similarly constructed. More especially as the contrast between the progress made by the Indian Commission and that of this colony is so marked as to suggest the idea that the lack of energy shown by the latter arises from its faulty composition. In the Victorian Commission the Education Department is unrepresented. The result is that the tone assumed by the Chairman of the Commission, when addressing officers of the Department under examination, has been the reverse of what it should have been. From the way in which many of the questions have been put

from the chair, it would seem that the Commissioners take it for granted that the Education. Department was unwilling that its administration of the Act 1872, by which it was incorporated, should be fully inquired into. An assumption which is totally unwarranted. The corrupting influences of political patronage had become so destructive of discipline, and were so manifestly undermining the stability of the State school system, that the gentlemen entrusted with the administration of the Act were among the first to acknowledge the necessity for the appointment of a Commission. Again, instead of the dilatory sittings of the Victorian Commission, that of India meets daily. Three times a week as a Commission, and three times a week in Committee. The general questions are first threshed out with the utmost freedom of debate by a general committee, consisting of the official, missionary, and native members. Their conclusions then supply a firm basis for the more strictly-regulated debates of the Commission. Considering the lengthened time our Commissioners are likely to occupy in making their investigations, and the necessity for the Minister of Public Instruction becoming thoroughly versed himself in the actual position occupied by State Elementary Education in other parts of the British Empire, we would recommend the Hon. James Grant to commission one of the heads of his Department—say the Inspector-General—to visit India, Canada, and England, for the purpose of reporting to him the very latest phase of the Education Question. During the absence of the secretary in Europe, the chai'ge of the Department devolved on Mr. Bolam, and the extra labour thus cast upon him must have been very considerable. He has earned a holiday, and were he to undertake the mission suggested, he would be able to recruit his health, while,fat the same time, he would be rendering invaluable service to the cause of popular elementary education in this colony. The system of education which Mr. Grant administers is on its trial before a not too favourable jury, and he will be wise to prepare himself for an appeal to the judgment of the people of the colony against an unfavourable verdict.


The annual boat race between the Universities of Cambridge and Oxford was rowed over the usual course on Saturday, April 1st, and resulted in a victory for the Oxford by eight boat lengths.    _ _

Circulars have been issued by the Industrial School authorities, containing the following instructions to teachers from the Education department :—Teachers are requested to give all possible assistance to the visiting committees by furnishing the lady correspondent with early intimation of any systematic irregularity of attendance on the part of Industrial School children ; also, any impropriety of conduct, in or out of the school building, that may come under their notice, or of any habitual slovenliness in person or dress. They are informed that the children are required by the Industrial School department to attend twice daily on each school day, and that any deviation from this, unless with the special written permission of a lady correspondent, should at once be reported to her. It is further desired that immediate intimation may be given to the lady correspondent when any boarded-out child passes the examination for the standard of education.

Mr. Gilchrist, the head teacher of the Howard-street (I-Iotham) State School, has been appointed to take charge of the Yarra Park School during the absence on leave of the headjteacher, Mr. Walker, who has gone to England on a trip for the benefit of his health.

Some doubt has arisen as to the sanity of Dr. Lawson, who was convicted recently for the murder of Percy Malcolm John, at Wimbledon, and a respite has been granted him until full and exact inquiries into his condition of mind shall have been made.

Recent telegrams from England report that the State Agricultural College at Amherst, Massachusetts, in America, was burned on the 30th March, damage being done to the extent of £50,000.

DORA Greenwell, who has for some time past been a regular contributor to the American press, died on LMarch 30th. Some of her writings showed remarkable talent.

Mr. J. H. Templeton, master of the Malvern Road State School at Prahran, whom it was determined to remove to a smaller school on account of his having disobeyed orders in refusing to admit certain children to his school, has, instead, been fined £20. He will still remain at his old school, as it has been ascertained that he is doing good work, and the parents are all thoroughly well satisfied with him ; and a large number of them testified to his good management. However, it is considered that a penalty is necessary to prevent the occurrence of a similar breach of discipline in future.

The first portion of the new building for the Adelaide University was opened on Wednesday, the 5th of April, and his Excellency was present


to deliver the opening address, in which, among other subjects, he treated of the admission of ladies to the University. He stated his opinion that all ladies should have access to the higher branches of education as well as the men. The Vice-Chancellor returned thanks to his Excellency the Governor for his presence, and stated that the opening of the University of Adelaide had already raised the standard of education of the private schools considerably. Six students had the Degree of Bachelor of Arts couferred upon them, of whom Mr. E. D. Holden received the South Australia Scholarship of £200 per annum for four years; and Mr. G. Donaldson the Scholarship founded by Mr. J. H. Angas, tenable for three years at £200 per year. Three graduates of the Melbourne University were admitted r,d eundem to the M.A. Degree.

Denis Florence M'Carthy, an Irish poet of the patriotic school, died on April 8tli.

Marcus Ward intends shortly to publish a new novel, called “ Owlet Ash.”

The School Funds of New Zealand for the year of 1881 amounted in the aggregate to £4990 3s. 5d., and the total expenditure to £3951 11s. 2d., which leaves a profit of £1038 12s. 3d.

An application has been made to the Board of Education, New Zealand, for the erection of a new school and teachers’ residence at Opotiki. There are a hundred scholars in the school, and the present building is in a miserable and leaky state, so that the maps are being destroyed. It was referred to Mr. A1 wright to report.

Messrs. Edward King and J. B. Miller have been appointed members of the School Board of Advice for the School District of Dunmunkle, 332, Victoria.

The following summoning officers have been appointed to the various school districts mentioned : — North-East Riding of the Shire of Murhamite (792), and North Riding of the Shire of Warrnambool (260), Constable James Turnbull, vice Constable J. Lindsay, relieved ; North Division of the East Riding of the Shire of Warrnambool (326), Constable William Young, vice Constable J. Cronin, relieved.

Mrs. Bardin has been appointed a member of the Visiting Committee of the Industrial and Reformatory Schools, in the place of Mrs. J. G. Francis, who has resigned.

Of 731.528 persons in Victoria, the number of those able to read and write is 174,871; and those able to read only, 70,450 ; those unable to do either amounting to 180.550. There were more females able to read and write, in proportion to their numbers, than there were men.

Messrs, Wm. Bald and Jno. Dunning have been appointed members of the Board of Advice for Sandridge, and Mr. Robert Wieley for the Bellarine Riding of the Shire of Bellarine.

The last number of the Melbourne Review contains a symposium, giving the views of various prominent gentlemen on the subject of the teaching of religion in State schools. The Rev. Mr. Strong states his opinion in the following words :—*• I am in favor of introducing religion into the State schools during school hours, as an integral part of the State school system. I consider that the objections urged against this are not insuperable ; that the exclusion of religion is a grievous mistake; and that, by the exercise of a little forbearance and common sense, a scheme of religious instruction could be drawn up which all Protestant denominations at least might agree to accept.” Dr. Bromby agrees in general with these opinions, and only objects to the introduction of the words “ all Protestant denominations at least.”

During the month of March, eighteen new books of some importance were published in Australia, the principal being “Occident and Orient,” by the author of the “ Vagabond Papers;” and “ Stories Told around the Camp Fire,” by G. C. Evans.

A writer in Frazer's Magazine, in an article on the late William Ellis, says :—“ Only those who have studied the science of education, and, in doing so, have surveyed its history, are aware that Mr. Ellis devoted to its advancement many hours, and hard toil, and a large fortune. Had his mere pecuniary donations been publicly made, such munificence must have aroused admiration for his disinterested philanthropy, even among those who may have different thoughts of the theories of education. Over a quarter of a million of money was given away by Mr. Ellis for the furtherance of public education, so quietly, and with such total absence of selfish ostentation, that his name is unknown even to thousands of those whom he has benefited ; and he remains another instance of the frequently illustrated truth, that fame and greatness are not of necessity wedded together.

Chambers’ Journal ” has now reached its fiftieth year, and the Ia6t number contains a life of the proprietor and of the journal, which is most interesting.

There was great surprise at the statement which appeared in the Age that the Education Commission was projecting a trip to Canada. Of course it was soon seen that this was only a hoax.

On Wednesday, April 12th, the Minister of Education received tenders for erection of school-house at Brown’s Plains, and for repairs to school-house at Paterson’s Swamp. For the first, the tender of Mr. Oscar Smith was accepted, at £409 14s.; and for the other, that of Edward Stafford, at £75.

Mr. T. H. Guennett has issued circulars intimating the intention of initiating popular concerts in Melbourne, if sufficient support be accorded him. It is proposed to call them the Melbourne Popular Concerts, and they are to be after the plan of the Monday Popular Concerts of London. The required support is the guarantee of 150 subscribers.

The death is announced of Dante Rosotti, the well-known artist and author, at the age of fifty-four years.

“A Lover of Justice,” writing to the Argus, complains of the way in which the conduct of teachers, as it was many years ago, is again and again brought to public notice by the Education Commission.

SÜzuxim anír %xt


Baron Adolf Erik Nordenskjold is now preparing for another expedition to the shores of Siberia.

It is thought probable by many geologists that coal seams may be discovered directly under London, Professor Prestwitch thinks that a trial should be made in one of the southern suburbs, and he shows that, even if coal were not struck, it is probable that Lower Greensand would be met, which would likely afford so abundant a supply of water as to give a sufficient return for the money expended.

The death is announced of the Government Geologist and Archaeologist, Mr. Samuel Sharp. Theodore Schwarn, another well-known scien-list who distinguished himself in Physiology, after completing his seventy-second year, has also been removed.

The Royal Society of Sydney offers prizes for the best articles, the results of originaljresearch, upon the following subjects :—Series 1. To be sent in before September 30, 1882.    1. On the Aborigines of New South

Wales. 2. On the treatment of Auriferous Pyrites. 3. On the forage plants indigenous to New South Wales. 4. On the influence of the Australian climate and pastures upon the growth of wool. Series II. To be sent in before August 31, 1883. 5. On the chemistry of Australian gums and resins. 6. On the water supply in the interior of New South Wales.

7. On the development of the marsupials. 8. On the infusoria peculiar to Australia. The prizes are to be £25 each and the competition isopen to all.

It is reported that a Roumanian engineer, M. Theodwescu, has invented a submarine ship which can be guided for twelve to fourteen hours completely under water. Upon the surface the ship can be managed like any other vessel, its rate of speed, however, being less than that of ordinary steamers.

A paper was read at the Royal Society of London, at its meeting on June 12, “ On the New Electrical Storage Battery,” by Henry Suttar of Victoria.

The new observatory at Etna has been finished. One is to be built at Natal soon.

Dr. IIann has recently made several investigations with regard to the temperature of the Southern Hemisphere, and its mean is found to be almost the same as that of the Northern Hemisphere 15°. 4'.

During the year, ending June 31st. ,1881, 175 photographs of the sun were obtained at the Melbourne Observatory, showing an increase of spots and disturbances on the surface.

Mr. T. Jeffrey Parker, of Dunedin, New Zealand, has recently been making extensive experiments on the preservation of Cartilagenous Skeletons and other soft animal structures, and has found some very improved methods of preservation.

AT the last monthly meeting of the Committee of the Victoria Academy of Arts, Messrs. J. Gibb and T. McCubbin were elected associates. It was resolved, on the motion of Mr. Edwards, seconded by Mr. F. Goldstraw, that all pupils of schools be admitted to the Academy’s Exhibition at threepence per pupil, one teacher to be in charge of every 50 scholars. It was also resolved that a complimentary ticket be given to the secretary of the sketching club.

Mr. Folingsby has been appointed to the position of instructor and master of painting in the Victorian Art Gallery, in the place of M. Von Gerard who lately resigned.

The first ironclad battle ship of the Chinese fleet has been launched lately at the dock of the Vulcan Company, situate at the mouth of the Oder, The ship is called the Ting Yuen, and is a turret'corvette of the first rank. Another one is to be built by the same company.

For a good indelible ink to be used with a pen, the following is the best receipt that can be found : Dissolve Asphaltum in any essential oil, and cover it with old printer’s ink and a little lamp black. A little benzole will give the ink a greater fluidity, and' enable it to be used with freedom.

A WRITER in the Boston Journal of Chemistry, says :—I was very much interested, a few days ago, in hearing a friend give an account of a manuscript she had seen, which was written by Jonathan Edwards when nine years old. It was an account of the behaviour of certain small New England spiders, the manner they fly through the air, and the way to see them best, by getting into the edge of a shadow, and looking toward the sun. It is neatly and carefully written, and illustrated by little drawings, very nicely done. The philosophical tendencies of the young writer already appear, for his conclusion as to the “ final cause” of spiders and their flying is this : the little animals are scavengers, and since in New England the prevailing winds are west, they are carried to the sea in their flight with whatever filth they have consumed, and so the land is cleansed. Every one knows how, in sunny weather, the little creatures, standing on their heads, project from their spinnerets fine filaments of gossamer, which are caught by the breeze and float off into the air, though still attached to the spider. When she perceives that the thread is long enough, and the pull of the wind sufficient, she releases her hold and flies away on her gossamer like a witch on a broomstick ; by watching her chance, and letting go only when the breeze is favourable, she is carried to her desired haven.

On the 2nd of February last, a test was made of a “ fire proof paint,” introduced by the United Asbesto Company of London and Birmingham. A gauze curtain, stretched upon a wire frame, and having the name of the company painted upon it, was set on fire, and immediately consumed, with the exception of the material so painted, which exhibited only slight symptoms of contact with the flame. Perhaps the most important test, however, was that afforded by two wooden models of theatres, fitted with curtain, proscenium, and all the usual surroundings. One of these was painted over the whole exposed surface with the asbestos paint,

the other being left in the ordinary condition. Petroleum was plentifully dashed over both erections, which, being also filled and surrounded by shavings, were then set on fire. In ten minutes the theatre not treated with the preparation was reduced to a mass of charred timber, whilst the other, though exposed to heat quite as fierce, stood the test admirably. The painted woodwork was simply blistered ; an asbestos curtain was preserved intact; and the asbsetos ropes, representing those used in the lowering and elevation of the scenes, &c., were in no way injured.

It is now claimed that the coldest place on the earth is ¡not, as has hitherto been believed by meteorologists, Yakutsk, in Siberia, but Verkoy-ansk, in the same region, lying in 67£ degrees north latitude, on the river Yana. Its lowest mean winter temperature is 48-6 degrees below zero Contigrade, or—55'48 degrees Fah. This is the cold pole of the earth in Asia, the corresponding pole in America being to the north-west of the Parry Islands.

The farmers of Hunterdon and Somerset counties, New Jersey, use goats to protect their sheep from dogs. Two goats can drive away a dozen does, and two are about all each farmer puts in with his sheep. As soon as a dog enters the field at night the goatb attack him, and their butting propensities are too much for the canine, who soon finds himself rolled over and over. A few repetitions of this treatment causes the dog to quit the field, limping and yelling.

A new application for electricity, recently discovered, is for the hiving of bees. By introducing the ends of two connecting wires into a fully occupied honeycomb, and turning on the current, the bees are rendered inactive for about half an hour, while no bad results appear to follow their awakening.

Great advances have been made within the last year in the manufacture of guns of great power. With a 10-4 inch gun, a 4621b shot has been fired at a muzzle velocity of 2275 feet per second.

A SIMPLE improvement has been effected in the ordinary permanent way of the Great Western Railway Company, The chair is reversed and the wood key is shifted from the outside to the inside of the rail. Should the key work loose, the gauge of the line will be rigidly maintained under passing trains.

Dr. Reitter, of Austria, has collected a number of cases showing the danger of sleeping in a room with certain flowers and fruits. A number of vases containing hyacinths produced headache and sleeplessness, not only in a nervous and excitable woman, but in a thoroughly sound and healthy man. A young fellow surrounded the bed of a friend with boughs of lavender. He found in the morning that the young fellow had fallen asleep under this bower dead.

THE REVISED EDUCATION CODE.

The very heavy task undertaken almost a year ago by Lord Spencer and Mr. Mundella is just brought to an end. The Education Code, first drawn up by Lord Sherbrooke (then Mr. Robert Lowe) in I860, and revised by him in the autumn of 1861, had to run the gauntlet of severe criticism and debate in Parliament the following spring ; but the shape in which at length it appeared to the world without proved to be, upon the whole, so fairly well adapted to the circumstances of national education at that rather remote period of modern history, that it has remained, as to its chief regulations, practically unaltered to this very day. Minor changes, however, and changes corresponding to the letting out of a garment to make it fit a little less awkwardly upon the shoulders of a growing boy, have been matters of almost annual occurrence ; and, indeed, to a limited but rapidly increasing community, the publication, usually in March, of the Code for the year has been wont to produce quite as great a flutter of excitement as was ever felt in the more extended area of the nation at the appearance of a Budget in Mr, Gladstone’s palmiest days. But these successive changes, some of them confessedly tentative, many of them necessarily temporary, and not a few of them from time to time apparently retrograde in their character, have turned the Code into such a complicated piece of patchwork as to make absolutely hopeless the attempt for a plain man to trace out his course upon it. Officials themselves have sometimes been surprised when a regulation they had never seen, or else had wholly forgotten, has been pointed out to them ; and confused school managers have been known to make long pilgrimages to some prodigy in the study in order to be enlightened by him on a matter upon which depended, may be, the very continuance in life of their poor, feeble, half-expiring school. Successive Vice-Presidents of the Council have groaned under the burden, but they did not dare to grapple with the evil. So the Code grew bigger, and its rules became more complicated, and the multitude of the perplexed increased without bounds, till at last everybody confessed that something must be done to bring order into the chaos. That something was taken in hand by the present very energetic Vice-President, Mr. Mundella, and his chief, the Lord President of the Council ; and the result which has been arrived at, after a most unprecedented discussion of the whole question in the Press, at public meetings, by deputations to the Education Department, and in private conferences, is now before the public in the Code which, from the fact that it is not to come into operation anywhere till the 1st of April next year, might well have been called the Code of 1883. And what does this newest of new codes aim to bring about? Does it mean to destroy the voluntary system ? Is it going to turn the heads of ploughboys and make them look down upon their destined walk in life? Will it set about erecting forcing-houses for the premature development of young brains ? And is there any design in it to revolutionize the present national system of elementary education ? For the allaying of all such fears, and with the view of reassuring every disturbed and unquiet mind, let it at once be said that the new Education Code is simple, unambitious, and conservative to a most unexpected

degree. It will not, of course, please everybody ; for to do this it must needs be as full of contradictions as the world is of people of diverse and opposite opinions. Nor again, probably, will there be one single individual who will feel quite satisfied with every one of its regulations; for had it been otherwise its sworn foes would have been legion Possibly the best general description to give of it is that like a well-fitting boot it presses gently and equally on all sides ; special care, indeed, being taken here and there to give ease to a particularly painful corn. And now for the details.

I. Infant Schools.—If any thoughtful person were aSRed to set down upon paper the chief requisites for a successful infant school system, he would hardly fail to name in his list good teachers, bright methods, thoroughness of results, and simple and discriminating inspection. Now these are just the things aimed at in the New Code as it affects infant schools. 1. Hitherto too often the infant’s class of from ten to forty or fifty children of various ages below seven has been handed over to a young pupil-teacher or even to a paid monitor; the head teacher only now and then snatching a few stray minutes from the all-absorbing work of the higher classes to try and stir up a little spasmodic life in the restless, gaping, or sleeping gallery of uninterested infants. For the future this recognized and oft-lamented weakness of the system is to be thoroughly remedied. (1). Wherever the infants’ class numbers forty in average attendance a separate teacher, over eighteen years of age and specially approved by the inspector (although not necessarily one who has served an apprenticeship as a pupil-teacher, or undergone formal training at a normal school) must be engaged to carry on the instruction. This will practically put a class of from forty to sixty infants into the self-same position as a night-school of, maybe, thirty or more hobbledehoy adults. (2). When the number of infants exceeds sixty in average attendance, then their teacher must be certificated. But this again only places a large body of infants in the same position as that of any ordinary small school with the same average attendance. (3). The “ fixed” grant in each of the two cases just named is to be 7s. a head in average attendance; but if with over sixty infants their instruction is carried on in a separate department, the “ fixed” grant then will be 9s. a head. (4). Where the school managers for a class of from twenty to forty infants employ a teacher such as is required for a class of from forty to sixty; or where for a class of from forty to sixty they engage a regularly recognized “assistant teacher” (i.e., either a pupil-teacher who has successfully completed bis apprenticeship, or any candidate who has reached a place in the third or a higher class at the examination for admission into a training school), the “ fixed ” grant in each case will be 9s. instead of 7s. a head. 2. But besides providing good teachers, a successful infant school system demands the use of bright methods of teaching. The eternal A, B, C, and the interminable 1, 2, 3, &c., must be relieved everywhere with some of that variety of occupations and lessons to be met with already in every really well-conducted infant school. And here the Code is most explicit. (1) It first provides as a matter of course for reading, writing, and arithmetic, in the “ rudiments” of which “suitable” instruction must be given to all infants, (2) Then it prescribes simple lessons (not rigidly systematic as to their subjects) on common objects and on the more prominent phenomena of nature and common life— lessons intended to train the observation as well as to store the mind with useful facts. (3) “ Appropriate and varied occupations” are next to have their place in the daily routine—such as will tend to make school pleasant as well as useful. (4) Sewing is to be compulsory for every girl in the infant school; a special grant being awarded to it. Boys, too, are encouraged to learn this subject; the same grant of Is. a head in average attendance being offered for them as for girls. (5) Lastly, singing is expected to be taught; the grant allowed for it being 6d. if taught by ear and Is. if taught by note. But here it must be remarked that “singing by note” is a term in the Code which is left almost without definition. It is to be “satisfactorily” taught, and taught “according to some recognized system.” These are the words of the Code. But whether all are to be thus taught, whether a few songs sung by some or by all of the children with the “ notes” before them, or whether actual “ sight singing” will be required, are all of them points left apparently to the discretion of the inspector, who, it is charitably to be hoped, will himself have gone through a tolerably complete course of “ Hullah” or “Sol-fa,” or both. 3. But thoroughness rather than ambition is to be the ruling spirit of infant school work.

(1)    The report on reading, writing, and arithmetic must be “ satisfactory” or the “ Special Merit” grant to be presently mentioned will be altogether disallowed. (2) A child over seven who happens to be attending the infant school on the day of inspection will be examined in Standard I.; but no child who has been forced on to a higher standard will be examined above Standard I., “except in cases specially sanctioned by the Department.” (3) Thoroughness in providing for the requisites already named of (i.) reading, writing, and arithmetic, (ii.) object lessons, and (iii.) varied occupations, wiH°be encouraged by the “ Special Merit ” graut, which will be 2s. a head in average attendance in a school classed “fair, ’’4s. where it is classed “ good,” and 6s. where it is “ excellent.” 4. But the inspection should be simple, not too exacting, and yet discriminating. 1. It is especially satisfactory to notice that the projected individual examination of little infants under seven,, with all the horrors of standards and papers and percentages, &c. necessarily inflicted on older children, is entirely given up. The inspector is simply toso examine in reading, writing, and arithmetic, as to be able to pronounce of the results that they are satisfactory or the reverse.

(2)    The inspector is also relieved from the awkward dilemma of having either to award the self-same grant per head on “infants presented.” whether the work done was good or indifferent, or to adopt the harsh measure of recommending a reduction of grant of from 10 to 50 per cent, in the case of the unsatisfactory school. He will now be able to use his

own judgment and to put each separate school into its proper grade of merit and to recommend the proper grant for the same. 5. All the grants earned in an infant school will be assessed upon the average attendance. Under the old system those grants in an average school reached the sum of 12s. 8£d. ahead, and itwas practically impossiblefor theamount to come up to 16s. a head iu average attendance. By the new scale a really good school may earn 17s. a head ; and it must be a very indifferent infants’ class that fails to earn 10s. a head.

II.    Subjects of Instruction for Children over Seven.—The final pro

posals under this head show a very marked improvement upon those which were set forth in August last. For example—1. The “ Standards ” are made a little more elastic and somewhat simpler. The poetry required in the reading of Standard IV. is now struck out ;aud any “ other standard author ” is allowed to be substituted for Shakespeare, Milton, and English History in the readiug of Standard VI. and VII. “ No exercise books ” need be shown in Standard VI. In Standard

II. the pence table is to be learnt up to 12s. iustcad of £1, the “local measures    have    disappeared    from the reduction

rules of Standard IV., the highest denominator for the fractions in Standard V. is to be 10 and not 20, and in Standard VI. it is made plain that while both simple and compound proportion must be taught, only simple (and not compound also) interest is required. 2. In the “ Class subjects,” while the “ meaning” of the lines recited by Standard

I. will not be inquired into, the recitation of Standards VI. and VII. is not confined to Shakespeai’e and Milton, and Standard VII. will only need to take up 150 instead of 200 lines. The syllabus for elementary science is simplified. And, most important of all, the Standards I., II., and III. (with IV., if thought best by the school managers) may be considered as one group, and Standards V., VI., and VII. (with IV., if not otherwise arranged) may be dealt with as another group, or may be taught in two groups—an arrangement which will much facilitate matters, especially in small schools. It is also a distinct gain that sewing in a girls’ school should be allowed to count as a class subject, although in this case it will not be paid for also under the proposal for a special sewing grant. 3. The needlework schedule is made a little easier in almost every standard. 4. In the schedule of “specific subjects," which is now given in full, provision is made for two alternative schemes of mechanics, the separation of Algebra and Euclid into two distinct subjects, the combination of the two “ branches” of domestic economy into one, the recognition of attendance for forty hours of a girl over twelve years old at a cookery class as an equivalent for passing in a “specific” subject, and two scries of subjects in physics—namely, “sound, light, and heat” for one, and “magnetism and electricity” for the other. And while it is still arranged that these subjects shall not be begun till the 5th Standard, the only other condition of their being taken up is that at the previous inspection the percentage of passes was, at least, 70, instead of 75, as heretofore. 5. To meet the case of a child hitherto compulsorily promoted to a new standard before passing successfully in, for instance, the arithmetic of the lower standard, it is now arranged that, while ordinarily a child is supposed to advance a standard (or, in specific subjects, a stage) every year, exceptional cases of children being presented twice in the same standard (or stage) will be permitted if explained beforehand to the inspector, although, if many such cases were brought forward at the same time the special merit grant might be much reduced or even be converted into a negative quantity, (». Girls are still to go through the same syllabus of arithmetic as boys, their only privilege being that, their work will be judged with more leniency. 7. The full grant for singing cannot be earned unless it is taught “ from notes.” How far this requirement is likely to result in a real improvement upon the present mode of teaching the subject is open to considerable doubt, for. unless a system of “ sample” examination is introduced on a rather extensive scale, or a very large amount of time is given to the examination in singing, it will be simply impossible to apply any adequate test from which to estimate with accuracy the results produced.

III.    The Annual Examination.—1. The Special Merit grant is to be assessed on somewhat different principles from those first specified. The intelligence of the “ methods” of instruction used is not to form one of the elements,' for the simple reason that on the day of inspection this can in no way be tested. Instead of this a more general, term is used— “the intelligence employed in instruction”—which can, of course, be ascertained from the general quality of the children’s answers. Again, “the special circumstances” of each particular case must be regarded, which will lead, it is to be hoped, to a liberal allowance being made for special local difficulties. The way in which the children pass in reading, writing, and arithmetic will be a further element in the problem, it being obvious that while two schools may score precisely the same percentage of passes, each individual pass might be relatively far more meritorious in the one school than in the other. 2. The “ sample” examination is thrown over completely. This is a change which will be warmly welcomed everywhere among teachers and school managers,

3. Every child whose name has been on the school registers for the 22 school weeks immediately preceding the end of the school year, and whose name is still on the registers on the day of inspection, must be presented for examination, unless there is a “reasonable excuse ” for his absence. But two or three difficulties arise here. By the 12th supplementary rule it is stated that a continuous absence of a fortnight from school will, under ordinary circumstances, necessitate the remuval of the absentee’s name from.the registers. Suppose this were to happen within 22 weeks of the end of the school year and the child by-and-bye were readmitted, would that exempt him from examination ? Suppose an unscrupulous teacher (and now and then, unfortunately, such a person is to be met with) were to arrange with the parent of a dull child thus to remove him just before the examination, how is the fact to be exposed? And what, again, is a “reasonable cause” for absence? -This ought certainly to be defined, for, in the event of a conflict of opinion upon the point, the school will be fined “ three failures ” for each absentee whose excuse for being away is adjudged by the inspector not to be “reasonable.” 4. At any annual examination any child whether belonging to the school or not, may be examined in the standard appointed by the by-laws of the district for partial or for total exemption from attendance at school.

IV.    The “ Old ” Attendance Grant.—1. This has not been abolished as some people feared would be the case. On the contrary, it has been slightly raised, from 4s., that is, to 4s. Gd. This is a change in the right direction, and the only regret is that the increase made is not distinctly greater. It is no longer to be called the “ Attendance grant,” but the “ Fixed Grant.”—2. The “ old ” shilling singing grant was formerly part of the so-called “ attendance grant but it was taken from that grant a few years ago, and was only to be allowed in a school when “ singing forms part of the ordinary course of instruction.” Now this grant is to be only Gd, if singing is taught, as hitherto it has been taughtin five schools out of six, “ by ear.” 3. The “old” grant for “ dicipline and organization,” which was also once part of the attendance grant, is now replaced with the “special merit” grant of Is., 2s,, or 3s. per head in average attendance, according as the school is classed fair, good, or excellent. The sum of these changes may be briefly stated as follows :—By the Code of 1881 a school might have earned 4s. per head for attendance, Is. for singing, and Is. for discipline, &c.; but by the New Code it may earn 4s. Gd. for attendance (or fixed grant), Gd. or Is. for singing, and Is., 2s., or 3s. for the special merit grant. A fair school would thus get the very same under this head as formerly, while an excellent school might get 8s. 6d. a head in place of Gs. This is a decided gain, and it illustrates, in part, Mr. Mundella’s oft-quoted formula, “ Good schools will earn more and bad schools less than formerly.”

V.    School Expenditure.— One of the strongest objections to the “ proposals” which came out in August, 1881, was to the effect that under the altered regulations school expenditure would necessarily considerably increase. 1. This will still be the case (as will appear from what is stated under No. I. of this article) in infant classes or schools with over forty children. But the possible increase in the grant for infants will be some set-off for this increased expense. 2. But while stipendiary monitors, as such, are no longer recognized, two “candidates” for the office of pupil-teacher may be employed in place of one pupil-teacher. This will be a saving in some schools. 3. Again, while under the original “ proposals ” no more than three pupil-teachers were to be allowed in any single department of a school, the resulting deficiency having to be made up by employing the more costly agency of “ assistant” teachers, now three pupil-teachers are allowed for the first certificated teacher, and one each for every other certificated teacher on the staff. This will involve the disadvantage of having, by-and-bye, to turn a larger number of half-trained teachers adrift to seek employment in other walks of life ; but it will effect a considerable saving of school expenditure. 4. Two sets of reading-books (and not three, as in the “proposals ” of last August) must be provided for every standard ; this will result in a considerable saving. 5. Additional expenditure, however, will still be incurred because of the regulations (1) making an “assistant” to count for GO instead of, as formerly, for 80 children, and (2) no longer allowing humble school teachers to become certificated without examination. 6. One short clause inserted in the New Code will lead to considerable additional outlay in some schools. In order that “halftimers” should earn about the same under the new regulations as under the old, it is arranged that two attendances of a half-timer shall count as three of any other scholar. Dpon this calculation it may be shown that an average half-timer will earn Gd. more now than formerly. But this clause is added—that the same mode of reckoning is to be used in estimating “ the staff required in any school.” It follows that when, as in many large schools in manufacturing towns, there are, say, 80 half-timers at the morning school and 80 also at the afternoon school, and where, therefore, two pupil teachers would just have sufficed as the necessary staff for their instruction, for the future these 80 half-timers at each half-day will count for an attendance of 120 ordinary scholars, requiring, therefore, three pupil teachers—the additional cost being the salary of a pupil teacher, which will average £15 or £16, towards which, at most, the additional grant allowed will not be more than 160 sixpences, or £1. Was this intended?

VI.    Night Schools.—1, It is now made quite clear that any child under 12, who is under no obligation to attend the day school and is actually not a day scholar, may attend the night school. 2. It is also specially provided that in addition to reading, writing, and arithmetic, he may take up two “ classes” or “ specific” subjects. 3. The teacher, too, need not be a layman. 4. But no night scholars may be presented for examination in a standard lower than the third. 5. Where the night school has met, at least, 45 times, the “ fixed grant will be 4s. a head in average attendance ; and when it has met not less than 60 times, the “fixed” grant will be Gs. a head. G. There is no special syllabus or set of night-school “standards” set forth in the Code; but it is arranged that the grant for every pass in reading, writing, and arithmetic or in a “class,” or a “ specific” subject shall be 29. Night schools, therefore, are much favoured by the New Code.

VII.    School Attendance.—I. The familiar “ 260 attendances " will no longer figure in the Code ; and, except that in the case of a child over 13 who has not passed the standard for total exemption, the regulation in Lord Sandon’s Act requiring that number of attendances every year for five years as the qualification for a labour certificate is still operative, no reference would ever again have to be made to it in matters educational, 2, Whether the abolition of this long-standing requirement will

work beneficially in the matter of school attendance is a question whose answer depends upon a very plain issue—will school managers, parents, local authorities, and, in particular, magistrates, really lay themselves out to see that the Education Acts are properly obeyed ? If they would one and all do this, then, while the education of the masses would progress with enormous rapidity, the grants earned under every head would increase with the increase of average attendance, and little, if any, occasion would exist for getting up a grumble over the scale of Government grants.

VIII.    Pupil Teachers.—1. The admission examination is put up to Standard VI. or VII. This will do something to relieve these young people from being overweighted with studies during their apprenticeship. 2. Another regulation which is to take effect at once is that no pupil teacher may be employed more than 25 hours a week in “ keepiug aud teaching the school ”—a most valuable rule, and one which has long been demanded in the interests of mental development as well as of health.

IX.    Miscellaneous Regulations.—1. The rule requiring 20 per cent, of the children over seven to be presented above the 3rd Standard in order to earn the full grant on “ class subjects ” is wisely struck out. This will be a great boon to many small rural schools. 2. Practically only one rule connecting age and standard of examination will for the future remain in force, and that is the well-known one that a child over seven cannot be counted as an infant, but, if examined at all, must be examined in Standard I. 3. No more “ honour certificates ” will be issued, but those now in force will continue to carry with them the privilege of free education. 4. Acting teachers may for the future attend the certificate examination at 20 years of age instead of 21. This reduction of age will press a little unfairly as against the regularly “trained ” teacher. 5. A teacher who, after Christmas, 1SS3, obtains a certificate only upon the first year’s papers, will not be allowed to take pupil teachers until he has passed his examination in the second year’s papers; and failure in this latter examination will incapacitate him from being examined afresh till after the lapse of two full years. 6. Some little addition is made to the syllabus of studies for pupil teachers during their apprenticeship, but it is mainly with a view to make those studies more interesting, or to secure greater efficiency in professional skill.

X.    The Rate of Payment for reading, writing, and arithmetic.— Nothing can be simpler than the proposed plan of payment for examination in the elementary subjects. The number of actual passes, whether single, double, or treble, will be compared with the number of passes that could have been made by the scholars examined or absent without reasonable excuse, so as to ascertain the true percentage of passes. Then every unit in this percentage is to couut as a penny per head of the average attendance as the payment for reading, writing, and arithmetic. For example, if the“ percentage be 90, the rate of payment will be 90 pence—i. <?., 7s. 6d.—a head ; or if the percentage be 100, the payment will be at the rate of 100 pence, or Ss. 4d. a head, and so on. Whether in fixing the other items in the new scale of grants sufficient allowance has been made for the certain loss which must result under the head now beiug discussed is a question which probably will be warmly contested elsewhere. Whether the apprehended falling-off will ever probably be realized in fact depends upon certain statistics which are not fully in the possession of the public. Most people fear that the average percentage of passes for the future will come very far short of the 81 2 per cent, obtained for the past year, and few are sanguine enough to place the new figure higher than 72 or 75. This view is evidently not shared by the Department, It may, therefore, be fairly concluded, upon the whole, that where good work will be paid for better than formei’ly, and where simplicity and efficiency are made to go hand in hand, the scheme that is to bring about these satisfactory results has but little to fear from the adverse criticisms of the staunchest upholders of “ things as they were.”—The Times.


ST. ANDREW’S UNIVERSITY.

INSTALLATION OF SIR THEODORE MARTIN AS LORD RECTOR.

{Continued from page 141.)

DESIRABLENESS OF COMBINING THE CLASSICAL AND THE SCIENTIFIC.

I am not going to weary you with auy remarks upon the vexed question, whether a classical or so-called scientific education is the best. Like many other controversies of comparison, it seems to me to be rather a futile one, believing as I do, that it is quite possible to combine both, and that in all the higher education both ought to be combined, leaving the student to give the preponderance of his time and attention to that which he finds most congenial to his gifts and tastes, or most likely to prove of value for the work he has to do in life. No one will admit more frankly than myself, that the educational studies of our schools and universities, as formerly pursued, were both too narrow and too uui-form. They omitted instruction in many things which it was not only useful but necessary for every educated manto know ; and they did not sufficiently take into account the diversities in the social position, and in the quality and bias of mind of the studcnfs. All this is now in the failway to be corrected. With the facilities everywhere offered, it will be a man’s own fault if he finds himself, as many of the men who won distinction in the then only favoured studies used to fiud themselves, launched into active life in ignorance of the elements of physical science, of the phenomena of the material world, and of the laws and forces by which it is animated or controlled. The regret of such men at their own deficiencies was deepened by the thought, how easy it would have been to have acquired, by a little extra effort, all this knowledge side by side with their other pursuits, and how difficult it was to repair the defect, when the mind was either no longer so plastic or so receptive as in youth, or when the studies and duties of a business or profession left them little or no opportunity for the task. So, too, in the wide range of academic


studies, there is now scope for every variety of gift and inclinatien, and there is longer an excuse for the deadening of enthusiasm, often degenerating into habits of idleness, which was begotten by distaste for studies for which the student had no natural aptitude.

USES OP CLASSICAL STUDY.

But whatever a man’s special gifts may be, or whatever his future profession or pursuits in life, it seems to me that he cannot but be a gainer by the training which is to be had under a good system of classical study. Of course all studies are good by which the mind learns to think, to observe with precision, to seek out principles, to methodise facts, to draw reasonable conclusions from them, and to be able to find apt words for whatever it wants to express. But I know of no way in which all these ends are more likely to be arrived at than by a sound training in the classics. The man who has grappled successfully with the great Greek and Roman writers may be trusted to have developed a faculty which will stand him in excellent stead, whatever he may be called upon to do, or wherever he may be called upon to go. What he knows he will know thoroughly, and he will have acquired a habit of application and intellectual discernment, which will enable him to acquire and digest other knowledge with a rapidity, and to turn it to account with an address, that must give him an incalculable advantage over other men, who may be full of general information or practical knowledge, but who have not undergone the discipline of difficulty, of reasoning, and reflection involved in a mastery of the great classical writers. The young man who can put into terse well-chosen English all the meaning of any passage of Thucydides or Tacitus, or who can make a good précis of an oration of Demosthenes or Cicero, will go into active life well prepared to follow any intellectual pursuit. As a doctor, a lawyer, a clergyman—as an engineer, an artist, a merchant, or a manufacturer—he will find the benefit of the knowledge and aptitude which went to these achievements. They will make all the special studies of his vocation easier. At the same time, he will be better able to fulfil the duties of a good citizen, by bringing to the consideration of all social and political questions a judgment less likely to be captivated by plausible fallacies or fervid rhetoric, for he will know why States and Empires, which bear the closest analogy to our own, have risen and fallen. He will know, too, what the manly and sagacious thinkers of antiquity have thought upon such questions, and be able to call the experience of the past ages and states of society to his aid in judging of what is necessary or expedient for the present.

ITS VALUE IN REFERENCE TO MODERN POLITICAL QUESTIONS.

And who will say that such knowledge is not specially needed at the present time ? These are days in which, it seems to me, every man who can is bound to think of those things, and to be at pains to seek light from whatever quarter he can in forming his political opinions. And where will he find more instruction, whether to warn or to guide, than in the history of Greece and Rome, and in the recorded conclusions of the leading minds of those countries as to what makes the welfare and prosperity, and secures the stability of a state. We have chosen— whether wisely or not time will show—to set aside the principle which, among all civilized states of which we have authentic record, has been accepted as the only sound one. Cicero expressed it in ten words. “ Semper in republics tenendum est, ne plurimum valeant plurimi (There is one rule that must ever be observed in a State—the preponderance of powers must not be in the multitude”). And why ? Because wisdom and constancy have never yet in the world’s history been the characteristics of the “plurimi.” Is there anything in the state of our modern society to make us believe that this is less the case than it has ever been ? Look at any of our great cities, in which population multiplies with a startling rapidity, without a corresponding increase in the means of comfort, or even of bare subsistence. Is the proportion of the suffering, the discontented, the needy, the improvident, the unscrupulous, which will always be found in old communities, less than it has ever been ? Is the bitterness of those “ who have not ” against those “ who have ” likely to be less rancorous, where the extremes of wealth and poverty, of luxurious idleness and ill-paid toil, of profuse extravagance and “ looped and windowed raggedness,’’ are brought into such sharp contrast ? Is thejdisposition to think that “ whatever is is wrong ” likely to be less widely spread, when the numbers who have nothing to lose by change are so great? Still it is in the hands of the “ plurimi ” that we have deliberately chosen to place the preponderance of power; and, being there, to recall or to retract it is impossible. We must therefore make the best of the altered state of things, trusting to the average good sense, and to the patriotism of the mass of the body politic, not to use that power without deliberation, or a true regard to the teachings of history and experience. But the experience we are making is a momentous one ; and it is incumbent upon the educated youth of the country to show by their example that they are alive to the fact. If British liberty shall ever be in danger, the danger will come, not from above, but from below. The old party distinctions bave lost well nigh all their meaning. Be in no haste, I would say to you, if I might, to make up your views on great political questions. None are more intricate. The interests of the nation are so vast and so complex ; our relations to our own colonies and dependencies, as well as to the other great States of Europe, demand such cautious handling, that there are in truth no subjects on which it so much behoves men to ponder well before coming to a conclusion. Think of the magnitude of the interests involved ; think of the disastrous consequences of any great mistake in legislation or in policy ! A colony lost, the stability of property shaken, the belief disturbed that Britain can hold, by her own strong arm and her wise administration, all that generations of her sons have won for her. Let any of these things happen, and who can say how great, how swift may be her decline ? These are the great issues on which political questions lean. How needful, then, that they should be approached with minds unwarped by the bias of party ties, or of party passions. Even without such bias it will always be hard enough to keep the judgment clear Why, then, should young men, who may fairly hope hereafter, each in his sphere, to assist—some it may be even to lead—in the formation of public opinion, fetter their judgment or their independence by adopting the catchwords of the hour, or by subjecting themselves to the prejudices from which no political party can be wholly exempt?

CLASSICS AND PHILOSOPHY AS A GUIDE TO THE SCIENTIFIC SPIRIT.

Just as I consider what may be learned in a sound course of classical study an admirable preparation for approaching the political questions which agitate modern society, so, too, I venture to think that to none will such studies, and those studies of mental philosophy with which they are generally combined, be of more advantage than to those whose lives are to be devoted to the natural sciences. They will have learned that there is a large number of ultimate facts and phenomena in man’s nature, as real and significant as any of the material phenomena of the universe. They will know how thoroughly most of the problems about man, the world he lives in, bis place and duties in it, and his future, have been canvassed and discussed by the wisest heads of which the world has left any record. They will, therefore, approach their studies with a modest and reverential spirit, and be less likely to launch into profitless speculations on what can never be known, and to promulgate those rash deductions from very limited data, which characterise so mauy works of modern scientific philosophy. Were such studies as I have indicated more general, many weak and mischievous books would never see the light, and many a tortured heart and brain would be saved from bewilderment and despair.

THE CLASSICS AS MODELS OF STYLE.

Neither, gentlemen, I am sure, will you fail to join with me in rating highly the advantage of being trained in youth upon books, written in languages which, as vehicles of expression, have neither been surpassed —books which Time’s severe but kindly hand has winnowed for us from the mass of ephemeral and commonplace work, which was, no doubt, produced in abnndance both at Athens and in Rome. Depend upon it, ancient civilisation, like our own, was prolific in men like the Etruscan Cassius, of whom Horace speaks in the Tenth Satire of his First Book—

Capsis quern fama est esse librisque

Ambustum propriis.

whose poems were so voluminous that, as Mr. Conniugton puts it in his admirable translation :—

When ho died, his kinsfolk simply laid

His works in order, and his pyre was mado.

The ancients had a most laudable horror of big books. They felt how true in regard to books, as well as to other things, is the proverb ; “ The half is better than the whole.” Above all, they knew that the man who studies to condense, acquires in the process, the sense of proportion, the art of separating what is essential from what is accidental; makes, in short, that reserve of power to be felt in his work, which leaves upon the reader’s mind a delightful impression of symmetry and finish. Moreover, the tone of thought in the best Greek and Roman writers is essentially noble and manly. Trained upon such standards, the mind is less likely to be attached by what is false or feeble, unwholesome, sickly, or sentimental, of which there is enough and to spare in modern literature ; just as, if our early years have been surrounded by specimens of the best art, ancient or modern, we insensibly imbibe such a knowledge of pure form, of elevated expression, of what is essentially true to nature, that the eye turns aside with indifference from bad drawing, vapid sentiment, or meretricious colour.

VALUE AND BLESSEDNESS OF PERSISTENT WORK.

The main thing after all is, that in youth we “ learn to learn,” and having done this, that we then find out for ourselves what interests us most, and what we are, therefore, likely to do best. With that knowledge let us then determine to work out what gifts we have with all our might. “ Quid quid vult, vade rult,” says Cicero of his friend Brutus. It is this doing what he has to do with a will, with the determination that what he wills that he shall accomplish, which makes the useful, the influential, the successful man. He is sure, however his lot may be cast, to find scope for his energy. An intelligent persistency, which is a very different thing indeed from a resolute obstinacy, is the quality of all others a young man should cultivate in himself. Be modest, but determined ; measure your own powers carefully and even sternly ; but resolve that whatever gift is in you shall, with God’s help, be fully and strenuously worked out. Aim high, but take care that your aim is within your compass, and that, come what may, it is pursued by honourable means. Above all, cultivate the habit of work. “I consider the capacity of labour,” writes Sir Walter Scott to his friend Adolphus “as part of the happiness I have enjoyed. Part of the happiness? Most men who have gone through a life crowded with demands upon their capacity for labour, will rather say that it has been their chief enjoyment, nay, their chief blessing, that they have been called upon to exercise that capacity, and that it has answered to the call. Many such men have I myself known ; and I wish I could tell you how much to them has been that delight in recurring to the books and studies of their youth, which makes them look back with tender reverence to the school or Alma Mater iu which their love for literature, philosophy,^or science was first developed, Others I have known, men “ gifted with predominating powers,” which have found vent in pursuits that have crowned them with wealth and all the good things which wealth places within our reach who hare bitterly mourned, either that in their youth they had no chance of acquiring a knowledge of books or the arts, or that they had not duly availed themselves of such opportunities as they had. How poor and maimed do such men feel their life to be, when they find the strength or the occasion for active pursuits fail, and they cannot

the ecstacy of winning for his bride the divine Helena, whom he is able to invoke from Hades in all the splendour of her immortal charms—

“ Fairer than the evening air,

Clad in the beauty of a thousand stars.”

Even in the transports of enjoyment and triumphant power a sense of insufficiency and incompleteness is ever present with him. It is only when he has grown old, when his thoughts are not for himself, but for others—when he takes to reclaiming land from the sea, and building harbours, and succeeds by these and other works in making hundreds of his fellow-creatures happy—that the cravings of his heart are for the first time satisfied. Then the wish mounts to his lips, that the state of things in which he finds himself, and the mood of soul which it has wrought within him may last. The moment has come when he can say to it—

“Stay, stay, oh stay ! Thou art so fair.”

and be content to die. Let none of you think that, because your career in life may afford no scope for doing good to your fellow-men, that the principle here indicated does not apply to you. Which of you can say to what great work he may not some day be called, or what his power for good over others may be? But, whatever your sphere of influence, be it large or be it small, there is sure to be ample scope in it for unselfishness and for active good—for proving yourself to be gentle, generous, sympathetic, forbearing, courteous. Determine that such you shall be. Keep this resolve steadily in view, and it will “make the path before you always bright,” and keep alive within you that sacred fire of enthusiasm which, if fostered and directed to worthy ends in youth, will not burn itself out, but will prove to be the purifier and sustainer of your riper age. “ Beautiful is young enthusiasm,” said Thomas Carlyle : “ keep it to the end, and be more and more correct in fixing on the subject of it. It is a terrible thing to be wrong in that—the source of all our miseries and confusions whatever.” Many of you, I dare say, know well some lines with which Thackeray ended one of his pleasant Christmas books—lines as full of wisdom as they are beautiful in tone and in their simple force. They express much of the feeling with which I close the few and feeble words in which I have addressed you—

“ Come wealth and want; come good or ill;

Let young and old accept their part,

And bow before the Awful Will,

And bear it with an honest heart.

Who misses or who wins the prize—

Go, lose or conquer as you can,

But if you fall, or if you rise,

Be each, pray God, a gentleman 1”

The address was frequently cheered in the course of its delivery. In response to a call from the students, Sir Theodore intimated a holiday for to-day. Sir Theodore and Lady Martin held a reception last night in St. Mary’s College, at which about 400 persons attended.”—Scotsman, Nov. 21.


HI .ellmmxc (Llnibcrsitjj.


beguile the weariness of the heavy hours, by availing themselves of the delights which they see are found by other men in the very books which stare at themselves in mute rebuke from their library shelves 1 It is no paradox to say that there is nothing like work—pursued, of course, with a due regard to the claims of the body to exercise and care—for maintaining the elasticity of the mind, and preparing it for what we should all aim at, the carrying on the spirit of youth, and freshness of enjoyment, into riper years, and even into old age. Idleness and frivolity are the cankers of the soul, and bring upon it premature disgust, decrepitude. and palsy. There is a sentence of Cicero’s on this point, which experience has often recalled to me as full of truth.—“ At envm adoloescentem, in quo senile aliquid, seu senem, in quo est aliquid juven-tutis 2>robo ; quod qui sequitur, corpore s°nex esse qioterit, animo nun-qvam erit” “ What I delight to see is a youth with something of an old man in him, even as I do to see an old man who has in him something of a youth ; where these qualities are, a man may become old iu body, but never in mind.” In this great world of moral and material wonder, where there is so much of beauty, of grandeur, of mystery, of struggle, of noble effort, of pitifnl failure, of magnificent enterprise, of fascinating discovery—so much to love, so much to admire and revere, so much to help forward, so much to fight against and to subdue—in this world which we believe to be but the training-ground of our souls for nobler and higher and less encumbered work hereafter, in this quasichildhood of our real lives—why should we not try to keep our souls as open to new impressions in our riper years as in the days of our youth ? It is not years that make age. Frivolous pursuits, base passions unsubdued, narrow selfishness, vacuity of mind, life with sordid aims or without an aim at all—these are the things that bring age upon the soul. Healthful tastes, an open eye for what is beautiful and good in nature and in man, a happy remembrance of youthful pleasures, a mind never without some active interest or pursuit—these are the things that carry on the feelings of youth even into the years when the body may have lost most of its comeliness and its force.

THE PROBLEM OP CULTURE.

There is so much to be known, so much that it befits all educated men to know, that the work of learning cannot be begun too soon, or prosecuted too earnestly. At the best, do what we will, we shall do little more than learn, at every step we advance, of how much we are ignorant, and see dim vistas before us of the path to be explored, and catch glimpses of fresh fields where knowledge is to be reaped. Is there not something significant, deeply touching, in Goethe’s dying words— “ Light, more light I” Light to keep our feet from stumbling, light to cheer us on our onward path through toil and trial, light to lighten the darkness by which “our haughty life is crowned.” It is for this that the daily cry goes up from all good men everywhere, and from none more earnestly than from those to whom most of it is vouchsafed. To bring light and sweetness into our lives is the doctrine pi’essed unweariedly upon us by one of the most impressive writers of our time. It is not for me to define all that Mr. Matthew Arnold includes within those two words ; but I think I will not wrong him, if I say that he means them at least to include that culture of the whole man which, while it sharpens, enriches and fortifies the intellect, feeds the imagination with noble images and aspirations, develops and cultivates the taste for the arts and courtesies which embellish life, and the sympathies which make men unselfish, forbearing and helpful one to another. Wisdom of the head, he would tell us, is all very well ; but there is a something greater and better than that, without which no man can be indeed truly wise—wisdom of the heart. It is only when the two combine that the world gets its really great men and women. And whom are they whom we best love, and admire the most among those that cross our path in daily life ? Is it not those in whom we recognise that blending of intelligence with refinement, and consideration for others which gives a harmony and beauty and calm strength to the character, and commands instinctively our confidence and respect? What Mr. Arnold says, Goethe also meant; for as I have read his teaching, he includes sweetness in the light—the glow of the heart as well as the illumination of the brain—which he craved as needful for man’s happiness. The first part of his greater work, the “ Faust,” is only a splendid illustration of the doctrine, that the cultivation of the mere intellect, and the quest of happiness in and for the individual merely, must culminate in disappointment and despair. The second part, which may be fairly held to contain his last views on the great problem of human life, illustrates the truth, which wise men of all ages have preached, that happiness is only to be reached through active benificence, through the application of knowledge and power to the welfare of mankind. While Faust poured iu his study over musty volumes of medicine, jurisprudence, and theology, and perplexed his reason with problems which, by their very nature, admit of no solution, the accumulation of pedantic scholarship, and the bewilderment of brain, in which his studies resulted, brought only bitterness of heart, a feeling that every higher aspiration of his nature was left unsatisfied, a deadness of all belief in whatsoever makes life worth living, a total recklessness as to the great hereafter, what it might be, or whether there was any hereafter at all. In this mood, we know, Mcphistopholes found it easy to get him into his toils ; and Faust, confident in his sad creed that all mortal struggle, all mortal happiness, was vain, made his unholy compact, on the footing -that the fiend may claim him as his own, should a moment ever arrive in which he should Bay—•

“ Verweilo doch t du bisb so schon.”

“ Stay, stay, oh stay ! Thou art so fair.”

That moment he believes will never come, but come it does. And how? Not when he holds poor Gretchen in his arms in the rapture of triumphant passion ; not in the contemplation of the beauty and grandeur of the universe,? not in the possession of wealth and power ; not even in

The annual commencement took place in the ^University Library, on Saturday, the 15th inst.

In the absence of the Chancellor (Sir William Stawell), the ViceChancellor (Dr. Brownless) presided. There was a large attendance of graduates and undergraduates, and the proceedings were characterised by the absence of the disorderly scenes which have been so conspicuous on previous occasions. A great many visitors were present, by whom the ceremonies were witnessed with interest. The following members of the Council took part iu the proceedings :—Revs. Alex. Morrison and Dr. Bromby, Dr. Hearn, Dr. Morrison, Dr. Mackay, Dr. Fetherston, and Messrs. Andrew and Leeper. On behalf of the Professorial Board, Professor M'Coy presented the successful candidates.

The following are the names of the gentlemen who were admitted to degrees :—

Bachelors of Arts. — Isidore Myers, C. L. Andrews, Robt, H, Bromby (Trinity College), T. J. Byrnes, Samuel M'Meckin (Ormond College), Thos. H. Newing, Thos. Palmer, Wm. Chas. Pritchard (Trinity College), H. J. Robertson (Ormond College), Reginald Stephen (Trinity College), George Wilson, Fredk. W. T. Woodward, Fredk. Chas. Appleton, Thos. Wilson Serjeant, Ernest Thurston Williams, and Wm. G. Irvine (ad eundem).

MASTERS OF Arts. — Roderick Aitcheson, Frederick Eddy, Frank Gavan Duffy, Bernard Loughrey, Thos. P. MTnerney, and G. W, Brown (ad eundem).

Bachelors of Laws.—Samuel B. Backhouse, W. H. Bryant (Trinity College), Robt. E. Hayes, John Wilson, Henry S. Cole, F. W. Dickson (Trinity College), Ernest 0. Joske, F. Gavan Duffy, and William Officer (admitted during his absence).

Doctors of Laws.—John Quick and Thos. P. M'Inerney.

Bachelors of Surgery.—Thomas Loughrey, Timothy B. Ryan, and

G. A. Symc.

Doctors of Medicine.—William Armstrong, John D. Thomas, and Wm. B. Walsh (ad eundem).

The following exhibitions were awarded : Matriculation.—Classical Exhibition — Charles H. Stretton. Mathematical — John Monash. English and History—T. Davies.    French and German—Miss J. M. S.

Huntsman. First Year Arts.—Classical Exhibition- Divided between Arthur Alston and James W. M‘Cay (Ormond College), equal. Mathematical—John H, Mitchell. First Year Medicine.—Wm. Atkinson Wood. Second Year Arts.—Classical—John C. Baird. Natural Science—George Horne. Second Year Engineering.—William H. Brockenshire. Third Year Laws.—Francis W. Edmondson (Trinity College).

Ridomm (Bimaiiicm JRjnirfmcnt

APPOINTMENTS.

Wm. Heritage, H.T., Yenngroon, 1942; Emma Thomas, H.T., Tarran-ginnie West, 2456 ; Margaret Scott, H.T., Toongabbie South, 2281 ; Henry B. Weldon, H.T., Malakoff, 1398 ; J. M. Harris, H.T., St. James, 1764; Herbert Williamson, H.T., South Hanson, 1385 ; William Thorpe, H.T., Crosbie, 1215; Henry W. French, H.T., Pine Grove, 1565; Charlotte Yallins, H.T., Green Hills, 1170; Elizabeth Scott, H.T., Langdon’s Hill, 961; John J. Kiernan, H.T., Lamplough, 1239 ; Henry G- A. Brown, H.T., Garvoc, 996; Mitchell Doig, H.T., Tallygaroopna, 1816; Raymond N. Frost, H.T., Barmah, 1725; Josiah Ingamells, H.T., Hastings, 1098; William Errey, H.T., Bambora and Boonah (half-time), 1285; Wm. H. Collier, H.T., Jung Jung, 2372 ; H. J. Horneman, H.T., Rye, 1667 ; Sarah A., Collier, H.T., Menzie’s Creek, 2457; Eliza Burston, 2nd Asst-, Carlton, 1252; Robert Smyth, H.T., Wangandary, 1717 ; John H. Murphy, H.T., Campbell’s Creek, 2144 ; Edmund J.|Wilson, H.T., Jumbunna, — ; William Webster, H.T., Goomalibee and Upotipotpon (half-time), 2445; John N. Barrett, H.T., Portland, 4S9; Henry Robinson, H.T., Bright, 776 ; Margarat Kiernan, H.T., Wareek, 953 ; Angelina Canald, H.T., Bloomfield, 2187 ; Thomas Rainford, H.T., Mount Pleasant Dam, 1891.

THE KINDER GARTEN.

We have pleasure in acceding to the request of a correspondent to print the following communication :—

“ What is the meaning of Kinder Garten?” “ Can you tell me what the Kinder Garten system of teaching is ?” Such being the questions I am repeatedly asked by English people, I think a few remarks may do good in making so excellent a system a little more widely known, for I do not imagine that it is only amongst my acquaintance that information is required. For years the system has been in use in Germany, and I am glad to find that it is at last becoming known in England, though it gains ground far too slowly to please those who know its real worth. It is in fact a system very suitable for little children, as it combines amusement with instruction, and that amount of life and energy which so often causes children to get into mischief because not properly directed is by this means turned to good account ; not that I mean to go so far as to say that young children educated by this system are never in mischief, but I do believe they are less likely to be so, as they have so much to interest them,And are really gaining much knowledge without the feeling that they are doing lessons, which so many little ones look upon (not unnaturally, all things considered) with something approaching alarm. One fault in the greater number of ordinary schools is, that the children are taught and treated in masses, and not individually, which is one of the things which Herr Frobel considers very necessary. Of course, this applies principally to the infants of the lower classes, as those of the middle and upper classes gain what instruction they do receive in the nursery, where it is too often the case that they are left to those who are really not fitted to instruct even infants, though they may be most kind and devoted to the children ; but if the nursery is superintended by a Kinder Garten governess or nurse, or the children are sent to a Kinder Garten, then they learn to observe, to compare, and to reason, so that they gradually obtain the habit of exercising their faculties upon all which comes within their range. Much is done by object teaching, for which purpose the first six gifts are used, viz, : 1st gift is a box containing six coloured balls ; 2nd gift, ba'1-cube roller ; 3rd gift, cube divided once in every way ; 4th gift, cube divided into eight planes cut lengthways ; oth gift is an extension of the third, the cube is divided into twenty-seven equal cubes, three of these are further divided into halves and three into quarters ; 6th gift stands in the same relation to the fourth as the fifth stands to the third, and by its aid all the exercises given under the fourth may be carried out to a far greater extent. The above brief description of the six gifts is copied from ‘ The Guide to the English Kinder Garten,’ of much use to those wishing to instruct in the Kinder Garten system, which was purchased at Miss Frost’s Kinder Garteu establishment, 57 Bernersstreet, who also supplies many other equally good books on the subject for instruction in the Kinder Garten system. Besides the gifts there are stick laying, stick plaiting, paper folding, cutting, and pricking ; the alphabet box, containing pieces of cardboard in various colours, cut into triangles, quadrangles, and squares, from which letters can be formed and colour taught. Children can commence drawing almost as soon as they can hold a pencil, and when some little progress has been made in that, they may be allowed to take up modelling. Gymnastic movements are gone through to music, the children generally at the same time singing, music being much used in the Kinder Garten schools, which, as every mother and governess knows, is a great attraction to children. At most of these schools, I believe, visitors are allowed on certain mornings, and it really is most interesting to see the wee ones at their several occupations, and notice the interest they manifest. It must be understood that the amusements are absolutely play in work ; their little minds are kept interested in all that is going on. I have frequently heard of dull children becoming bright and cheerful. As an instance, a little boy who was almost thought to be an idiot was sent to a Kinder Garten school, he positively was there for three months hardly uttering a sound, but after six months had expired, he was one of the most apt pupils. A gentleman called upon me the other day, and was speaking upon education. I asked him what he thought of the system. He said he had two little girls, the eldest was at an ordinary school, and the youngest at a Kinder Garten, and in a very short time the latter was a long way before the former.”

VELOCITY OF SOUND AND LIGHT.

Since sound and light are due to vibrations, it is clear that they must take time to travel. If you look up into the sky at night, and observe a bright star, you are not looking at the light which is at. that moment coming from the star, but at light which left the star years ago, and has been travelling through space all the time. The light which is at; this moment leaving the nearest fixed stars will not reach our earth for about three years. The sun is so much nearer to us that its light reaches us in about seven minutes. The sun is ninety-one millions of miles from the earth, and light travels at the rate of nearly 190,000 miles in a second, Now, you may read and remember this, but neither you nor any living person can form the slightest idea of such a rate of motion. It was a Danish astronomer, called Roemer, who first found out the velocity of light. There is a planet called Jupiter, which is sometimes comparatively near to the earth, and sometimes far from it ; this planet has certain moons or satellites moving round it; and Roemer observed that when Jupiter was farthest from the earth the moons appeared to be sixteen minutes behind time as observed from the earth. He concluded, no doubt rightly, that the difference was due to the fact that the light from the moons took a longer time to reach the earth from the greater distance; and as the paths of Jupiter and his moons aro known, it was easy to calculate the distance denoted by this sixteen minutes of time.

You may, then, consider that light is practically instantaneous for any distances that can be measured upon the earth’s surface ; for instance, if we wish to learn what is the velocity of sound, we can easily do so by producing a flash, accompanied by sound, and observing the interval between the perception of the light and the sound at a given distance. You have often uoticed, if you have been standing a few yards away from a workman using a hammer, that you do not hear the blow of the hammer until an appreciable time has elapsed since you saw the hammer fall. If you were to stand on the seashore, and a vessel out at sea were to fire a gun, there would be a lapse of several seconds between the flash or the smoke and the report. A more familiar example is afforded by thunder and lightning, the length of time elapsing between the two enabling us to tell how far away the discharge took place.

Experiments prove that the velocity of sound in air is affected by the temperature ; in dry air at o°C. it is about 1090 feet per second, and the rapidity increases about 2 feet per second for every degree Centigrade. But sound travels more quickly through liquids than gases, and more quickly through solids than through liquids. It travels through water about four times as fast as through air, and very rapidly along the grain of certain kinds of wood ; for instance, along the grain of fir it travels

15,000 feet per second.

What is it which gives different substances different capacity for transmitting ? It has been found that a fixed law is applicable to the transmission of sound through solids, liquids, and gases. It is that the velocity depeuds on the elasticity of the medium in relation to its density. The elasticity of any body is measured by its resistance to compression ; that is, if a body resists being compressed within a smaller space by pressure brought to bear upon it, its elasticity is great. Now, though water is more dense than air, yet it is so much more elastic that the ratio of its elasticity to its density is greater than in air, and hence sound travels faster in it. And in most solids the elasticity is very great in proportion to the density.

Hence you will see why sound travels faster along than across the fibre of wood : the molecular structure of wood makes its elasticity greatest along the fibre.—Elementary Science.

ASSISTANT in Melbourne, salary, £114, dosires exchange with male Head Teacher in better position. Omega,” Post Office.

HEAD TEACHER, 30 x o0, results, 75’863, five miles from Geelong, near bay, four-roomed house, will exchange Second Assistant, large school, Melbourne or suburbs. Address, “ Omicron,” Schoolmaster office.__

HEAD TEACHER, allotment 30 x 50, good residence, vacancy Work-mistress, post office, and other conveniences at hand, would exchange with another. Lower allotment accepted, within 50 miles of Melbourne. Address, “ Spectrum,” Schoolmaster office.    ___

HEAD TEACHER, ten miles from Ballarat, on main road. Allotment,

. 60 x 75, safe, results, 87. Workmistress vacant. Post office, mail twice a day, would exchange with a similar school. Address, “ K. C.B.,” caro of Mr. Yale, bookseller, 1 Sturt-street, Ballarat. ________

HEAD TEACHER, country, desires exchange, results 81, allotment 20 to 30, near large town, healthy climate, residence 3 rooms. Address, 11 Mitto,” care of Mr. ,!. Hearne, Newbridge.

HEAD TEACHER, 30 X 50, Inglewood district, desires exchange with Head Teacher, 20 x 30. Vacancy for W. M., Ballarat district preferred, or convenience to railway station in direct communication with Ballarat. Address, “ Romeo,” Salisbury West.

HEAD TEACHER, 30 x 50, 30 miles from Sandhurst, would exchange with similar allotment. Satisfactory reasons given. Address, “U.R.E.” Toolleen.

HEAD TEACHER, 15 miles from Melbourne, 50 x 75 present basis, wishes exchange, assistant Melbourne or suburbs. Address, “T. E.,” G.P.O., Melbourne.

riAEACHER, Gippsland, allotment under 20, post office attached, six miles J- from railway station, would exchange for higher allotment, near Melbourne, Geelong, or Ballarat. Apply, “R. G.,” Schoolmaster Office.

rpATE’S PARCELS POST EXPRESS

FIXED price.

NO EXTRAS. NO TROUBLE.


Delivery to door at any

21b

4lb

Gib.

!01b.

201b.

address in

d.

s.

d.

s.

d-

s.

d.

s

d.

Great Britain

4

6

6

0

7

610

0

14

0

Continent of Europe,

America, & Canada Sydney, Hobart, Launceston

7

6

9

6

12

0

16

0

21

0

2

6

3

0

3

6

4

0

6

0

New Zealand Ports

(except West Coast) Adelaide, Brisbane.

4

0

4

6

5

0

5

6

7

6


No further charge whatever. Very small increase for heavier weights. Delivery at country addresses in Australia, inland carriage only added.

Receiving office—

FREDERICK TATE, 13 Market-st., Melbourne T A T K’S PARCBXS PO ST EXPRESS. DELIVERY at DOOR any address in the world.


EITHER to or from Britain from 4s. 6d. T<) or from other Australian ports from 2s. 6d. NO further CHARGE whatever. No trouble. Any SIZE, weight, or shape.

E\ER\ possible ASSISTANCE afforded. jNQUl Rl 88 plainly answered.


Henceforth Parcels handed to W. R. SUTTON’S Branches, every town in Great Britain, delivered in Melbourne at nearly similar rates.

Head Office :—

FREDERICK TATE, CUSTOMS AGENT, &c., ___18 Market street, Melbourne._

^LEX. M‘KIN LEY & CO.,

GENERAL

PRINTERS AND PUBLISHERS,

61 Queen-street, Melbourne.


In Bookwork and General Publishing our large experience is a guarantee of all work being executed in the best stylo, while having a first-class stock of the best and latest material ensures expedition and good workmanship.

I he following pnpers are issued from this office :—


“ Punc’n,


WEEKLY.

“ Bulletin,” “Once a Week.”


fortnightly.

“ Australian Law Times," “Jewish Herald,” ^    MONTHLY.

Schoolmaster.”    “ Monthly Messenger.”

“ Faithful Words.”


A SECOND CLASS PUPIL TEACHER desires re-appointment. Country preferred. Recommendations good. Address, ‘B.B.’ Post-office, Murtoa.


T WARREN BALL’S “Hints to Candidates JL* for Teachers’ and Matriculation Examinations,” Is.; posted, Is. Id. Muller, Melbourne.


CANDIDATES for EXAMINATIONS prepared by correspondence or otherwise. I. Warren Ball, South Yarra.


IMPERIAL REVIEW,

Quarterly, 2s. Four Numbers, Subscription, 7s. fid., including postage anywhere.

Publishers :

ALEX. M'KINLEY & CO.,

61 Queen Street, Melbourne,


Tp X AMINATIONS.—UNIVERSITY

^    and DEPARTMENTAL.

TUITION in CLASS by Correspondence or otherwise. Terms, &c.,

JAMES L. ROBERTSON, B.A. (Melbourne). 71 Clarendon Street, Emerald Hill.


Q C. EXAMINATION.

TUITION BY CORRESPONDENCE.


MR. THOMAS BOARDMAN, First-class Honor-man of the Denominational School Board, Prepares Teachers for the Certificate Examination by Correspondence. Terms modorate.

Address—

45 PRINCES STREET, CARLTON


TAMES C L E Z Y, M. A.,

^    5 GORE STREET,

Prepares for Matriculation. Pass or Honours,and other University Examinations. Since February, 1874, One Hundred and Fifteen Pupils have passed various Examinations, ranging from Civil Service to the final Examination for B.A. Degree.

In October Term, 188i, Four passed Matriculation, including Civil Service. One was first of the first-class in Greek and Latin Honours. One (a lady) obtained a second class in Greek and Latin. Two passed for first year LL.B.

N.B.— Students after Matriculating, and before proceeding to the University, should read the First Year’s work of their course; otherwise the Lectures are of little benefit to them, and their chance of passing at the end of their first year is very small.


g^ MULLEN’S

New Classified Catalogue of School, College, and Technical

EDUCATIONAL WORKS

May be had gratis on application, or posted on receipt of address.


SAMUEL MULLEN,

Wholesale & Retail Bookseller & Stationer, 29 & 31 COLLINS ST. E., MELBOURNE.


Price One Shilling,

By Post—In Victoria, Is. 6d. ; Out of Victoria, 2s


D


MR. H.

M'KTNLEY

gOLICITOR,

CONVEYANCER,

And

PROCTOR,

86

COLLINS

STREET WEST.


BUY AND READ

THE NEW WEEKLY PAPER. Price Threepence.

“ONCE A WEEK

ll> PACKS. PRICE THREEPENCE.

ALL BOOKSELLERS.


Alex. MKinley and Co., Publishers,

- -    61 Queen Stroot, Melbourne.

g TATE SCHOOL, No. 17 7,

TRADES’ HALL, LYGON-ST., CARLTON.


MR. SERGEANT,

(Late District Inspector of Schools.) Conducts Classes for Certificates of Competency on Saturday mornings in the abovenamed school. YV ork will commence on Saturday, 22nd instant. At the examination in December last thirteen passed, though only six months under Mr. Sergeant’s tuition.


CANDIDATES ARE ALSO PREPARED BY CORRESPONDENCE.


EPARTMENTAL EXAMINATION OF TEACHERS.

TEACHERS’ GUIDE

TO

AUSTRALASIAN EXAMINATIONS.

Containing the

PROGRAMMES & EXAMINATION PAPERS

of

Victoria    South Australia

Now South Wales    Auckland, N.Z.

Queensland    Wellington, N.Z.

And Tasmania.

108 PAGES WITH STIFF COVER.


The above book contains the Programmes and Examination Papers of December, 1877, of all the colonies, and is reduced to the low price of

ONE SHILLING.

By Post—In Victoria, Is. 6d.; Outof Victoria, 2s


ALEX. M'KINLEY & CO., PRINTERS AND PUBLISHERS, 61 QUEEN STREET,

MELBOURNE,


T


0 HEAD MASTERS, SECRETARIES OF BOARDS OF ADVICE, AND OTHERS.

ALEX. M'KINLEY & CO.,

Having made considerable additions to their stock of Bookwork and Jobbing Type, are prepared to execute orders in every description of

GENERAL PRINTING.


All orders entrusted to them will be printed in a satisfactory manner.


Alex. M'Kinley & Co., Printers, 61 Qdeen-st.


M


NOW READY.

ILTON PARSED.

Price 2s.


By J. J. BURSTON,

(Author of “ State School Arithmetic”).

Also Ready, the Fourth Edition of the

STATE SCHOOL ARITHMETIC.

JOHN J. BURSTON,

North Sandhurst State School.


Printed and Published by Alex. M'Kinley & Co., 61 Queenstreet, Melbourne, under the auspices o£ the Victorian Teachers Union.


AUSTRALASIAN

®®©<S)©0GO

AND LITERARY REVIEW,


Yol. III., No. 35.


MAY, 1882.


(    Subscription

( Yearly, 6s. Gd.; Half-yearly, 3s, 6d.


BLACKIE


SON’S SCHOOL SERIES.


Arithmetic.

Thesk Arithmetics are graded to meet the requirements of the Code, though they are not limited in scope to the rules named there.

In large Schools it is essential not only to provide practice in each successive rule and stage of a rule, but to supply intelligent exercise for the brighter half of the class, showing the application of the rules. That they provide this throughout the Course is one of the special features of these Books.

A careful study of the causes of failure in Arithmetic has suggested other special features, such as

The abundance of word exercises, as a preparation for the dictation of sums by the Inspector.

The diversity of style in sotting tho sums, as a means of preparing scholars for any kind of test, and sustaining their interest much better than long arrays of sums set to pattern.

The difficulties of each rule are introduced gradually, and a model example given at the head of each exercise.

Problems arc given in abundance.

Instruction is given in the working of each rule for the higher standards, for too often the teacher has little time to spare for them ; but in the lower classes, no attempt is made to do that which only a teacher can accomplish.

Easy Lessons in Fractions are given at the beginning of Book V., that the pupil may the more readily and thoroughly do his Practice and Proportion.

The Tables required by each Standard are given in advance, that the pupils may not be able to refer to them instead of learning them.

Answers have been thoroughly tested, and are believed to be entirely correct.

Standards I. II. III.,............paper cover, 2d., cloth 3d. each.

Standard IV. (English Code)    „    3d.    „    4d.

Standard IV. (Scotch Code)    „    4d.    „    6d.

Standards V. VI................... ,,    4d.    „    5d.    each.

Keys to Standards I. to VI,, paper 3d. each, or bound together in cloth, ls^

OPINIONS    OF    THE    PRESS.

The set of Arithmetic books is well graduated, so that the scholar will pass easily and naturally from one book to the next. The examples are numerous, and stated so as to exercise tho reflecting faculties of the pupils without perplexing them.”—Daily Pevieiv.


Geography.

ELEMENTARY GEOGRAPHY.

By W. G. BAKER, Associate of King’s College, Lecturer at Cheltenham Training College.

PartI. For Standard II. Elementary notions of Geography. Paper 2d.; cloth, 3d.

Part II. For Standard III. England and Wales. Two coloured Maps. Paper, 4d.; cloth 5d.

Part III. For Standard IV. Scotland, Ireland, and the Colonies. Paper, 6d.; cloth, 8d.

“ Mr. Bakei has shown his mastery of the difficulties inherent to the teaching of small children in a science which is only just beginning to assume its proper position in schools. The pictures and diagrams are all to the point, and not

too complex.....If the succeeding numbers exhibit the same amount

of care and judgment in their compilation, Mr. Baker’s completed work must take high rank as an elementary treatise on geography.”—Public Opinion.


Third Edition—Now Ready.

A Manual of Method.

Fob Pupil-teachers and Assistant Teachers. Intended for the Government Inspected Schools of Great Britain and Ireland, and for the use of Students in Training Colleges, By Abr. Park, F.R.G.S., F.E.I.S., &c., Headmaster Albion Educational Institution, Ashton-under-Lyne. Interleaved with ruled paper. Third edition, foolscap 8vo, cloth, 2s.

OPINIONS.

“ No young teacher could puruse, and, above all, master, this really valuable little manual without receiving much mental stimulus and much wise direction in all that pertains to the difficult matter of school management. In fact, we do not know where else to find in so brief a space so much sound practical wisdom and suggestiveness on this subject.”—The Literary World.


Just Published. Price, 2s. Gd.

A History of the British Empire.

With numerous Pictorial Illustrations, Geneal ical Tables, Maps, and Plans. By Edgar Sanderson, M.A., 1 ,te Scholar of Claro College, Cambridge. 444 pp., cloth, red edges, 2s. Gd,

The writer of this brief record of a great nation’s fortunes has aimed at producing a narrative that should be equally interesting and instructive. He has believed that it is possible, even in a summary perforce so short as this, to bo clear throughout, to be strong and vivid where force and liveliness aro needed for a due effect, to rise, upon occasion, into eloquence, and to give to this compression of our country’s annals something of the heat and glow which must exist in all who, with sufficient knowledge, love her name and fame.

The convenience of not only the young student, but also of tho general reader, has been consulted in a close adherence to the order of time in tho narration of events, except so far as regards occurrences in Scotland bofore tho union of the crowns, and also as concerns the history of British India. Tho treatment of this most important subject in the present work is, so far as tho writer is aware, a new feature in a book of this kind. A short, separate account is given of the transactions, exploits, and events which have made India the greatest of dependencies attached to any empire in modern times. Tho reader is thus enabled to peruse, with ease and interest, in a continuous talc, that which it has hitherto been needful to pick out painfully and piecemeal from tho history of divers periods and reigns.

A fuller account than'has been usual, in books of this kind, has been given of tho great Civil War of the seventeenth century, and of the Peninsular War.

The history of England is, above all, a history of a nation’s growth in constitutional freedom. To this grand subject special attention has boon paid in all its salient points.

It is hoped that the arrangement in paragraphs, with marginal notes of the most important matters ; the maps, plans, and illustrations ; the genealogical tables inserted in their proper places ; the list of chief events and documents with dates; and a full and accurato index, will prove of substantial service to the reader.

Religious and political bias has been scrupulously avoided throughout, and it is believed that hardly any proposition has been laid down, scarcely one view advanced that would not find a general adherence amongst fair-minded and educated people, of every shade of political and religious opinion.


School Classics.


Selections from Standard Authors, with Biographical Sketches and Explanatory Notes.

32 pages; price in paper cover, 2d. each ; in cloth, 3d.


Lady of the Lake, Canto I... .Scott.

The Armada, &c..........Macaulay.

The Prophecy of Dante, C. 1.

II...............................Byron.

L’Allegro and II Pense-

roso ...........................Milton,

Essays (selected) ......Lord Bacon.

Prisoner of Chillón ......Byron.

The Fire Worshippers, Parts I.

II...............................Moore.

Ancient’Mariner ......Coleridge.

Deserted Village.....Goldsmith.

Marmion, Canto VI. ., ..... Scott.


The Lay of the Last Minstrel.


C. I...............................Scott

Cotter’s Saturday Night..Burns.

The Village..................Crabbe.

The Pleasures of Hope, Part I...........................Campbell.

Essay on Bunyan ......Macaulay.

The Queen’s Wake............Hogg.

The Merchant of Venice, Acts

I. III. IV............Shakspeare.

The Traveller.........Goldsmith.

Evangeline 64 pp. (price


4d.) ............H, W. Longfellow,


THE PROLOGUE TO THE CANTERBURY TALES OF

Geoffrey Chaucer. The text collated with the seven oldest MSS., and a Life of the Author, Introductory Notices, Grammar, Critical and Explanatory Notes, and Index to Obsolete and Difficult Words. By E. F. Willoughby, M.D. Price Is. 6d.

PARADISE LOST. By John Milton. Book I., with Life of Milton and Prefotary and Explanatory Notes. By E. F. Willoughby, M.D. Limp cloth, lOd.


Further Particulars and Catalogues from all Booksellers, or from their Melbourne House, Carrington

Place, Collins-street West.


Scboal gcpnrimcnt.

MR. PUZZLE, F. M. I.

A Philosophical Study. By James Runciman.

For a very long time Mr. Puzzle was only a myth to me. I could not formulate a distinct belief in his existence, although the rumor of him was in the air, and came faintly to my ears again and again. When amid my professional brothers I engaged in discourse concerning standards and inspectors and other institutions, it often happened that some earnest man said in a grave tone, “ Do you know Puzzle?” When I acknowledged ignorance 1 was congratulated in a way which surprised me. Gentlemen would relate to me wild legends having Puzzle for hero. Haggard masters would say, “Oh, you don’t know anything about school inspection until you’ve seen Puzzle up to his ordinary games.” Sometimes scraps of conversation like this would reach me : I’m tired. Puzzle has been with us all the week. He fell asleep in the middle of a grammar lesson, and made an awful row when he woke up and found the assistant had changed. He set all the mistresses in our girls’ department crying.” So it came about that Mr. Puzzle was often in my thoughts. At last I heard a most outrageous anecdote which had the unhappy merit of being true. Five young assistant masters and a headmaster were talking together during the midday recess when a gleeful messenger entered and said : “Have you heard about Puzzle ?” A heartless assistant propounded a counterquery, “ Is he dead ?”    “ No,” said theAmcssenger ; “ but he’s going to

leave our district.” Joy overpowered all sense of decorum. Those seven men joined hands and extemporised a species of war dance which was remarkable for originality and vigour. This tale determined my course. I resolved to track Mr. Puzzle in his native wilds, and to study him. I would submit to no more uncertainties. I did not succeed in seeing the object of my curiosity for many days, but until the moment of fruition came I occupied much time in gathering knowledge regarding his habits and customs. Sometimes I became involved in doubts ; but my doubts were always dismissed very speedily. On one occasion I was sitting among some two score teachers when the inevitable name began to be buzzed around. A woe-begone gentleman was relating how Puzzle got hold of my Standard II,, and he said, ‘ If all the water were land, and all the land were water, what would England be ?’ and he floored them because they didn’t say ‘ A lake,’ ” Keen in the pursuit of information, I turned to a lady who was near me and said, “ This puzzle is a very silly man, is he not ?” AVhereupon the lady frowned on me and observed, “No! On the contrary, he’s a very wise man, and very indulgent.” My whole conception of Mr. Puzzle’s character became chaotic at once, for this statement wms evidently made in perfect good faith. Half-an-hour after I mentioned the lady’s reply to a keen, good-liumoured gentleman who happened to know Puzzle’s district very well, and this gentleman smiled a broad land significant smile. When he had managed to straighten his face he said, I dare say she’s quite right from her point of view. You know Puzzle’s a crotchety old ass, and he takes likes and dislikes just the same as a spoiled baby. But he really is sensible in some things, and when he visits one of his pets he can be as nice as anybody, and they get along well with him.” My lady friend’s generous defence of her Inspector became comprehensible. Soon after this I saw my man, and got a good look at him, which taught me more than hundreds of woe-begone legends. It is a curious head. The brow is very good in its feebly intellectual way. The owner of that brow might have been top boy in his youth, and might have also been everything that a fond mother could desire. But the marks of the large nature are wanting. The eye is very fair, although chance gleams give a sardonic expression to certain involuntary glances. The mouth is feeble, nervous, cynical, cruel with the cruelty of a weak nature. There are odd lines sloping away from the sensitive nostrils to the jaw, and those lines have been bitten into the face by sheer ill-temper. A long course of unchecked self-indulgence is answerable for them. Not self-indulgence of a low sort by any means. But Mr. Puzzle lias never denied himself the luxury of small tyranny ; he has never held back a cruel word or a useless sneer. Thus it is that his face is netted with unkindly lines, and thus it is that his every feature gives notice of an inward likeness that passes words, If this man had been rebuffed and beaten by the winds of the world, he might have been a kindly, feminine sort of creature—a benevolent vicar, perhaps. But he has bad constant opportunities for exercising mastery, he has not been startled by burly, frank resistance, and so there lie is with his clever, feeble, sardonic face—a small tyrant, and a very poor specimen at that. For one grotesque instant, I thought “Ishould like to place Chinese Gordon alongside of that person for a minute.” Alas 1 my hero, forgive the irreverence ! It was but a grotesque thought, and yet, seriously speaking, one might do worse than think of a king like Chinese Gordon in connection with a pottering fidget like Puzzle. Look at Gordon’s royal head set on the roya' throat. The mystic eye is calm and holy, as though it had never looked on sin or on aught ignoble. The splendid repose of the whole face tells of kind thoughts and great thoughts. The man has never known littleness ; he is incapable of a sneer. I really do not think ho could manage to say to a frightened girl, “Now, Mary, you shall not have your certificate this year.” So the story of his heroic life, and his heroic gentleness is written on his face. That is why I wanted to see him alongside of Puzzle. We English are a queer set. When we get hold of a person like Chinese Gordon, we give him drains to inspect during the odd weeks when he is not organising empires ; when we get hold of a person like Puzzle, we give the minds of a few thousands of children into his keeping.

Since the time when I studied^ Puzzle’s face, and watched the twitching of his tell-tale hands, I have thought much about him. I find myself asking how it is that individuals like to him are permitted to thrive and exercise influence in our midst. The question is a wide one. We English used to be a fierce and jealous people. We would loyally bear a certain amount of pressure from men in authority, but no one dared tempt our forbearance too far. We are not less fierce or jealous of authority in these days. The democratic spirit is spreading, and the hand of authority is impatiently brooked. Yet whole sections of our community are dominated by beings like Puzzle and his prototypes, and dare not resent the domination effectually. Is it because Puzzle has superior claims ? Let us see. When Puzzle was a boy he was well taught. He went to the university, and when he was about twenty-three years of age he passed a good examination in certain languages and certain exact sciences. As a reward for his boyish feat, the nation puts Puzzle into an honourable and lucrative situation, and pays him well for life. All this comes on the strength of this boyish examination. But Puzzle is not humbly grateful to the nation. On the contrary, he is aggressive, and accounts himself a very superior being because he is well paid out of the Imperial taxes. Now it happens that Puzzle’s business is to assess a certain grant which the State pays for the encouragement of education. Under proper arrangements he ought to go very humbly and respectfully to the schools, do his duty without any prating, and depart, Mark how he enlarges his functions, and how lie is suffered to go on enlarging them. His very entrance to a school is made with an air of authority which is entirely gratuitous and impertinent. He addresses the teachers as though he were their natural superior, and this impudent assumption is not resented. Then he begins his work. He is paid to assess a certain grant for reading, and his duty is simply to find out whether the master or mistress has taught the subject of readme: properly during the year. He looks at the books and finds that they are not sold by a certain vendor. Then says Puzzle, “You must not use these books. You must teach reading not as you like, but as I like, and therefore your employers shall buy the necessary books from the tradesmen whom I patronise.” Now, instead of instantly reporting this audacious piece of jobbery to the Department, the teacher smiles, and orders the books commanded by Puzzle. Here is a gross exercise of illegal pressure, and yet a whole borough inhabited by men of our fierce nation submits tamely to the pressure. Why 1 Because they do not look minutely into their own affairs. They dabble in generalities.

Well, Puzzle is encouraged by this submission. Then his petty mind is stimulated to further excess. He finds the teachers anxious to please him ; he finds some of them who say Sir” with every tenth word, and his notion of his own superiority is indefinitely increased. He takes the teachers at their own valuation, and plays off his humors upon them. He satirises this, scolds about that, threatens about something else, instead of going quietly about his work. The teacher, who ought to stop the peevish creature’s mouth with a few curt, rasping words, is too much in the peevish creature’s power, and    therefore endures him.

Then Puzzle grows more autocratic.    Without any check, he

is allowed to impose upon the children in the schools an examination far harder than almost any other Inspector in the country. The teachers of his district are actually represented as being men and women of inferior ability, simply because Puzzle gives tests which are often capricious, often extravagant, and nearly always unfair. No one makes a resolute stand, no one threatens exposure, and Puzzle goes on. Then, ever and again, we hear that some poor wretch who cannot satisfy Puzzle’s requirements by reason of certain local disadvantages is cast adrift and sent to hunt for a new situation.

Now, I contend that Puzzle has no business to be satiric, be has no business to interfere in the slightest degree with any school arrangement, and he has no business to set examinations of exaggerated difficulty.

From all these things it comes about that a large number of estimable, hard-working, right-minded people are kept more or less in a state of apprehension from year’s end to year’s end, while Puzzle triumphs and prides himself on being the kind of man who keeps subordinates up to the mark. When be speaks at public meetings he always talks of “ My teachers,” and explains that though he has to blow them up occasionally, yet it is all for their good. The inconceivable effrontery of the allusion to “My teachers” never seems to strike the hearers. A Government clerk who is paid to assess grants ventures to talk of some four hundred ladies and gentlemen as his teachers and no one reproves him. If the gas company’s clerk, who takes the readings from our meters, were to mention a certain street as being inhabited by “his” tenants, we should laugh; but we never laugh at Puzzle. With the same indifference we treat Puzzle’s allusions to “blowing up,” although this same blowing up has very serious social consequences. 1 spoke just now about people living in a state of apprehension. I do not for an instant imply that Mr. Puzzle’s teachers lead lives of constant wretchedness, but I assert that they are subjected to incessant petty worry, and to many harassing and needless doubts. This tells with especial hardness on the schoolmistresses. The mention of mistresses brings me to the most painful subject which I have to consider.

There are few things in life that seem to me sadder than the obscure troubles endured by an ordinary schoolmistress who teaches in a district presided over by a person like Puzzle. The average female teacher is nervously eager to perform the uttermost part of her duty ; she is minutely conscientious in her attention to small things ; she has that patience which men sometimes lack. Above all, she is apt to have an exaggerated reverence for those in authority, and to manifest her reverence in various ways, which I find singularly pathetic. To see one of the poor souls on the morning of an examination hay, and to watch her flushed face and her undecided hands is not exhilarating.

I know that some mistresses are able to take things lightly. They have tact, and resource, and steady nerve ; they do their work thoroughly, and they do not fear any official in the world. There is no trembling of hands, nor shaky voice where they are concerned, and an Inspector like Puzzle has to keep himself within bounds when he is in their schools. But the average mistress, unhappily, has neither tact nor resource nor self-possession. She has seen little of the world, she is anything but reposeful in her demeanour, and a paternal government has educated her very ineffectively indeed. She can teach well, as a rule, because of her long mechanical training, and she tries her utmost all the year round. But in personal adroitness and courage she is deficient by reason of her education. Her very conscientiousness is against her, for she is continually doubting her own powers. The approach of an inspection-day is torture to her. Not once, but a hundred times I have heard girls and women say, “ Oh, dear! I do wish the examination was over. It would be such a weight off my mind.” Merry girls who are soon to be married, and taken out of the worry, do not fret themselves much. But there are many good women who do not marry speedily, and who teach on for years. Fancy what these people endure at the hands of Puzzle and of Puzzle’s peers ! I know one poor lady who happened unwittingly to offend the individual who inspects her school. The individual said, “ I will make you repent it as long as you live.” No one was there to apply a retributive boot to this bully. He has kept his word. His victim goes about in a very foolish way. She is thin and worn, she does not care for food, and she does not sleep well at night. She has worn herself away with fear of her tyrant.

Now women of the type that I have named suffer most severely from Puzzle. They cannot reply to his half-hidden gibes; they are overawed by him; the faults in their work are not kindly explained, but are made the subject of cowardly ironic remarks. So it happens that sometimes Mr. Puzzle makes two or three ladies cry in the course of a single day ; and so it also happens that blameless, toiling women look forward to Puzzle’s appearance with dread. It may be said that all inspections are dreaded by nervous women, and that not the inspector 'out the system is blameable. This is not true. Did ever any schoolmistress dread inspection by Mr. Matthew Arnold ? No one will suspect me of reverence for Mr. Arnold’s opinions. It has been my duty again and again to expose his inaccuracies. But it is certain that although as an educational talker he is a failure, yet as an inspector he is perfect. He has not room in his heart for a jeer ; he bestows delicate courtesy where-ever he goes; and the teachers under his charge look forward to his coming with pleasure, and see his parting with regret. He has always been the same. He is too high a soul to find pleasure in satirising a frightened woman ; and yet I fancy that the schools inspected by him are quite as efficient as the schools inspected by Puzzle. Here, then, is another query for Englishmen. Why should an official visit of Mr. Arnold be anticipated as a delightful experience, while unofficial visit of Mr. Puzzle is dreaded ? It cannot be because the one man is a courtly gentleman and the other is not, because in properly regulated societies a snob is always efficiently held down and rendered inoffensive. No. The difference arises from the fact that no check is applied.    In Mr. Arnold’s case a check on

tyranny does not happen to be needed. But school inspections are arranged as though all men were like Mr Arnold, whereas Mr. Puzzle is not in the very least like Mr. Arnold. The whole evil arises from the fact that Englishmen are forgetting that they govern themselves. We elect servants to do our bidding, and we suffer them to become our masters. The official.class is becoming the plague of the community because the community will not look sufficiently into its own affairs. There is too much centralisation and too little organisation. There is a lack of healthy jealousy, a lack of keen supervision. The descendants of the people who cut Strafford’s head off are actually beginning to take it for granted that it is useless to fight against an official at all. This belief must be altered or ruin will eusue. I never yet heard of a State in which the official class obtained supreme power without also finding that that State came ultimately to disaster. We pride ourselves on our system of local self-government. But what is the use of a dummy system? What is the use of local authorities if the schools in every district are really governed by an officer from a central bureau 1 Mr. Puzzle does as he chooses because the managers of the schools fear his official power and do not insist on controlling him. If half-a-dozen resolute mauagefs were to assert their undoubted right, and superintend a few of Puzzle’s examinations, his tyranny would end. Of course he would be insolent at first, because long immunity has pampered him, but a quiet man could soon stop his insolence. A series of interruptions, like this would soon bring him to his senses : “ Mr. Puzzle, the managers have consulted the head teacher, and they have decided not to use the books ordered by you, as there are others better fitted for our work.” “Mr. Puzzle, be kind enough to report on the organisation of this school, but do not suggest alterations.” “ Mr. Puzzle, your remark to this lady is unwarranted. Your business is to assess grants and not to scold our teachers.”

As to the teachers themselves, I can only say they will never be rid of Puzzle’s crotcheteering until every adult teacher and every student in training is known to belong to a strong Union. Personally they may get on better with Mr. Puzzle by thoroughly snubbing him, but single individuals cannot fight effectively. The men in Puzzle’s district might determine to combat their Inspector on a test case, but they would be crippled by want of funds if the affair became serious. In the same way the local effort which I have suggested would be rendered less effective if the managers who agreed to protect the teachers from Puzzle were not sure of being backed by a great Union. Therefore, my last advice will

be, Let every teacher join the Union, and the reign of Mr. Puzzle will be ended. Schoolmistresses, above all, should remember that a strong Union is the best protection for weak women. For two years a few of us incessantly preached this doctrine. We did not get much encouragement for our trouble. I hope the teachers will grow wiser, And now I will conclude the remarks into which I have been led by my study of the charming personality of Mr. Puzzle,— The Schoolmaster.


ON THE GRAMMATICAL ERRORS OF MORELL.

James J. Courtney, Garibaldi.

An interesting paper, from the pen of Air. Gough, on the above subject, appeared in your last month’s issue. Will you kindly allow me to make a few remarks thereon ? Mr, Gough objects to the parsing of the word here in the sentence, “The church here is very fine,” as an attribute to church. Now, in my opinion, Morel 1 is quite correct in the syntax he gives of this word, although I do not think his reason is sufficiently clear, viz. : “In such instances here, there, &c., are evidently used as equivalents for the phrases, in this place, in that place.” If Morell had said such words as here, there, &c., are used to represent the adjective sentences, which is here, which is there, the matter would have been very plain. Mr. Gough asks (a) “Is it because here is a word in an adjective sentence that it may be called an adjective ?” No, certainly not, but because here is an integral part of an attribute of the third degree, the whole of the remainder of which is suppressed as unnecessary, this portion of the attribute is taken as the representative of the whole, and made to qualify the noun, that the adjective sentence of which it is a part would qualify.” (h), “Would Morell call every word in an adjective sentence an adjective?” No. Morell would say there is no adjective in this adjective sentence, and yet it is an attribute to church; and as here is the key word of the whole attribute, it is only fair to make it represent that of which it is a part. But it may be urged that this is analysis, and not parsing. I think not. We arc dealing with a single word, and we are bound to parse it as it is used in the sentence. It is the essence of the adjective sentence, and can bo expanded into no other kind of sentence ; therefore we call it an attribute. Do the vulgar expressions, “This here church,” “That there house,” throw any light on this question ? Do they not show that the people who use them are endeavouring in their own ignorant way to show that they feel the words referred to belong to the nouns ? Teach these people grammar, and they would immediately place here, there, after instead of before the nouns ; but they would not alter their opinion very much as to their connexion with church, house. Morell’s remarks (page 106), “ The verb To be takes two nominatives, the one before and the other after it,” has caused some difficulty, because it is only true as regards the finite form of this verb, a fact which studeuts to whom Morell is the Alpha and Omega of grammar will not readily admit. But I think Crombie’s rule (as quoted by Mr. Gough), “The verb To be takes the same case after that it does before,” cannot be improved upon, as it applies equally to both forms (finite and iufinite) of the verb. Assuming this rule to be correct, the sentence (referred to by Mr. Gough), “ I know this man to be he,” is ungrammatical. It should read, “I know this man to be him,” because man is in the objective case. It is obvious wc could not say, I know he to be the man. The question is, does the noun preceding the infinite express the same thing or person under a different name as that following it, and are both the nouns affected by principal verb in the same way ? Does “I believe him to be a rogue” mean the same as “I believe a rogue to be him ?” If so (and it appears to me to be so), the difficulty is ended. Both nouns are objectives ; they are in apposition, and the infinite is simply the grammatical sign of equality between them. Mr. Gough says “The sentence should read: ‘I know this is he,’or ‘I believe he is a rogue,’ ” but this is evading, not solving, the difficulty, which on account of Morell’s words has become a real one to many. Now, kindly allow me to express an opinion on a word or two in Morell not already referred to, (Page 1Remote from towns lie run his godly race.” Morell gives the underlined words as an example of a participle phrase used as a complement to the noun. This w'ord remote is one of the best discussed words in Morell, some holding that it is an attribute to he, and backing up their opinion by referring to their text book ; others as stoutly contending that it is an extension of ran. I never knew either side to convince the other. My own opinion is that the word remote is an attribute to a noun which occurs in an extension of the verb ran, and therefore that it forms part of an extension, and has nothing to do with he. .Supplying an omission that, in my opinion, exists, we have “In a place remote from towns he ran his godly race ” thus—


Subj. and Att.

Predicate.

Obj. and Att.

Ex.

He

ran

his godly race

in a place remote

from towns.


Of course, taking this view, if wc omit the phrase, in a place, remote from towns would still appear as the extension of which it is a portion. Perhaps we shall hear more about this.

Page 21. Morell says, “The Preposition is a word which shows the relation of a noun or pronoun to some other word in the sentence. ” I have often wondered why, in the face of these words, he gives in his “Table of Parsing,” (page 11), the syntax of the preposition as governing -, instead of showing the relation of - to-. By his

method of parsing this word it seems to me we fail to show that wo grasp its real use in the sentence,


Heads,


Matter and


Method.


Cleanliness of Person.


Recapitulate. Cleanliness in Dress,


Cleanliness of Dwellings.


1. The skin is porous.


2. The pores are the waste pipes of the body.


3. The skin throws off in 24 hours—

(a)    1 pint of water.

(b)    J oz, of carbon.


4. The skin of the body should be washed—

(a) Regularly.

(fr) Frequently.


Examine and summarise.

1.    Elicit, that the waste matters with which the perspiration is charged must get into the clothes next the shin. Show that this dirt fills up the qmres of the clothing, and so checks free action of perspiration,

2.    Point out, tb at there is wisdom in having shirts, etc., white ; they sham dirt and warn us to change them.


6. Decaying vegetable and animal matter is poisonous. It should be removed from dustbins, ash-pits, etc.


7. Dust in the air is had for the lungs. Rooms should be kept free of it by (a) Washing, scrubbing, etc.

(o) Removing old paper from walls.


Heads.

Matter and Method.

Blackboard

Summary.

Recapitulate,

Elicit, that if the dust from floor, walls, etc., is floating about in the air we breathe, it must enter the lungs and injure them. Allude to stonemasons, and the dry grinders of Sheffield, who are victims to lung disease from inhaling dust.

How are we to prevent dust getting into the air of our rooms?

Dwell upon the necessity (1) of sweeping and scrubbing floors, walls, etc., regularly and frequently, (2) of taking away old paper from walls when they are newly papered.

Warn children against covering up and hiding away in corners dirty clothes, etc., that ought to be removed. Impress upon them the desirability, not of looking clean, but of being clean ; not of hiding, but of getting rid of dirt.

Examine on whole lesson and summarise.

By T. J. Livesy.


NOTES OF A LESSON IN SANITARY SCIENCE.4

Cleanliness with Reference to Health.

Apparatus.1. Diagram of skin. 2. Blotting paper. 3. Blackboard. 4. Slates.

Blackboard

Summary.

1.    Saw a boy, who had been playing hard at football, sit down, take out a handkerchief, and wipe his face, which was quite wet. How came it to be wet ? Where did the perspiration come from ?

2.    Draw attention to the pores of skin through which perspiration flows. Compare to blotting paper. Exhibit a diagram of a portion, of the skin as seen through a microscope.

3.    Elicit, if a boy perspires much and neglects to wash his face, it becomes dirty. Where docs the dirt come from ?

Tell how the pores are intended to carry off the waste matters of the body ; how the perspiration is impregnated with solid matter. State the amount of water (one pint), and of carbon (half-ounce), exuded in twenty-four hours. [Show a pint of water and piece of charcoal weighing half-ounce.]

4.    Elicit what would occur if this solid dirt were allowed to remain on skin, blocking up the pores. Tell how the waste matters would be driven bach into the body to poison the blood.

Compare with blocked-up drains, which send back poison into our houses.

Deduce the necessity for regular and frequent washing, not of face and bands simply, but of the whole body.

5. Perspiration clogs pores of underclothing, which should be—

(a)    Frequently changed.

(b)    Frequently

washed.

3. Deduce the necessity for—

(a)    Frequent change of underclothing.

(b)    Frequent washing of underclothing.

1.    Elicit that all accumulations of decaying vegetable or animal matter, or human excrement, give out a stench. Point out, that this ought to be a warning to us that they arc unhealthy and poisonous, Tell how these exhalations give rise to cholera, typhoid fever, etc. Allude to the system of sewage adopted in London, and the remarkable absence of typhoid fever, etc., in consequence. Dwell upon the necessity of removing excrement, decaying matter, etc., as soon as possible from the atmosphere w<e are to breathe; not allowing it to putrefy in dust-bins, ashpits, etc., but burying it, if possible, in the earth.

2,    Ask children if they have seen the sunbeams shining through the window, and the motes floating about in it. Tell them the searepar-ticles of dust suspended in the air.

Elicit, that, they must be there always (in a dusty room) though we only see them when they are lit up by the sunbeams.

HINTS ON THE PREPARATION OF LESSONS.

AAlue of Preparation.—There can be little doubt as to the advantage of preparing a speech, a sermon, or even a set lesson.

_ Ah off-hand, ready made-speech, sermon, or oral lesson generally tells its own tale, and abruptly reveals its immature origin. Under extraordinary circumstances, and with exceptional men, it may turn out a success ; but under ordinary conditions, and with the average speaker, preacher, or teacher, it is more likely to miscarry and end in failure and confusion.

_ Bishop Blomfield relates how he once, being called upon to officiate in a country church, ventured on an impromptu sermon. His text was, “ The fool hath said in his heart, there is no God.” After service, being anxious to hear how far his teaching had struck home, he accosted one of his rustic audience. “ What did you think of the sermon ?” said the bishop.

“ Well,” said the rustic hesitatingly, “ I liked the sermon well enough. But I don’t quite agree with you, Mr. Blomfield ; for I think there be a God !”

It was Bishop Blomfield’s first, and it was his last, effort at impromptu preaching.

Notes of Lessons.—1. The careful and sound preparation of a lesson is facilitated by the drawing up of notes.

By Notes of a Lesson is generally meant such a sketch of a lesson as will suggest to the teacher—(a) The nature and extent of its subject matter, (b) The plan of arranging and developing it. (c) The method by which the teacher is to impart it.

2. Notes of lessons are often classed, as—(a) Outline notes ; or, (&) Full notes.

Outline Notes are the bones, or skeleton of the lesson. They show the framework or plan on which the lesson is built up, and indicate facts with just sufficient fulness to recall them to the teacher’s mind.

Full Notes are more explicit and exhaustive. They not only develop the subject matter in greater detail, but they exhibit the method of unfolding that subject matter, point out such illustrations as will throw light upon it, and indicate the use which is to be made of the blackboard.

The Use of Notes.—1. No teacher cau teach what he does not know, and thoroughly know. Half-knowledge to him is no knowledge at all, or worse than no knowledge. He must not only know his subject thoroughly, but the knowledge must be fresh in his memory, present to his mind, and available for immediate use.

2. The teacher is a guide leading the children through an unknown land, and he ought to know and remember every foot of the way : the highways and byeways, the smooth and rugged roads, the dangerous pitfalls and morasses, the quiet resting places where one may pause for a moment and look back on the receding landscape.

If the teaclier is not,sure of his knowledge, the moment he approaches the confines of certainty—the border land between the known and unknown—he falters, hesitates, doubts, turns back, goes on again—stops, loses confidence, gets confused—and the children no longer heed him.

But a careful drawing up of Notes of Lessons tends to secure a thorough knowledge on the part of the teacher ; it enables him to see, and prepares him to meet difficulties ; it compels him to collect, concentrate, and arrange his facts, and gives him that self-possession and confidence which are necessary to sustain him in his task.

What to Prepare.—1. All lessons are better given when carefully prepared. It is no doubt true that time and practice give teachers a certain familiarity with the ordinary subjects of instruction, and a complete mastery over the more mechanical operations in class teaching ; but even with the most experienced preparation adds clearness, compactness, and thoroughness to their lessons.

2. To the young teacher, careful preparation in all but the most mechanical lessons is absolutely indispensable. He might as well offer his pupils an uncooked dinner and require them to digest it, as place before them an unprepared lesson, and expect them to assimilate the unpalatable stuff.

3.    Oral lessons ; explanatory reading lessons ; grammar and arithmetic lessons, where principles or rules are to be expounded ; and history and geography lessons, should always be prepared with notes.

Experienced teachers, even when familiar with a subject, find it profitable to jot down the heads or outline of their lessons, or to refer to notes used in previous lessons, so as to familiarise them with the ground over which they intend to travel.

4.    If the teacher is not only systematic in drawing up his notes, but careful in preserving them for future reference, he will have resources to fall back upon in his need that will save him an infinity of time and trouble.

If he leaves a margin to his notes for remarks, he may jot down for his future guidance little historic comments on the result of his experiments: as “This lesson, or this portion of the lesson, was ‘too long,’ or ‘ too short;’ ‘ too difficult,’ or ‘ too easy ;’ ‘ too general,’ or ‘ too specific ;’ was ‘ dull,’ or 1 interesting ;’ ‘ a failure,’ or 1 a success.’ ”

How to Prepare the Subject.—1. The nature of the preparation will depend upon, and be in proportion to—(a) The character of the subject, whether easy or difficult. (7;) The ages and capacities of the children for whom the lesson is inteuded. (c) The time to be devoted to the lesson. (d) The amount of knowledge already possessed by the teacher.

2.    The teacher will first fill his mind, either by referring to books or by recalling what is stored away in his memory, with the subject matter of the lesson. He should be thoroughly familiar with it in all its bearings, so as to be able to anticipate every conceivable difficulty, and be prepared for every possible question. He will then sift it, and select such portions of it as can be fitly presented to the particular class under instruction. The parts selected should be such as can be taught—not talked—in the time at his disposal.

Young teachers are apt to attempt too much, and burden the children with a multitude of facts. They seem to think the chief object of a lesson is to display their own knowledge of the subject under treatment. A good teacher will show his skill and judgmeut as much by what he does not teach as by what he actually teaches.

3.    He will next prepare his experiments, or seltct his illustrations— which may be objects, pictures, maps, or models ; or mere verbal illustrations, such as apt quotations, or examples and instances, or comparisons and historical allusions.

How to Arrange the Subject.—1, Nothing so much distinguishes a prepared lesson from one which has not been prepared, as order and arrangement. A prepared lesson ought to bear these marks :—

(a)    One part of the subject should be dealt with exhaustively before the next is taken up. There should be no turning back, no wedging into the middle of the lesson what ought to have been dovetailed in at the beginning.

(b)    The parts should follow each other in their natural or logical order, so that there may be from first to last, not a blurred and confused phantasmagoria of facts, but a clear, distinct, connected picture—a continuous and moving diorama.

(c)    There should be restiug-places at regular intervals, where the children may look back on what lies behind them, and where the teacher may summarize the information they have gained, so as to connect it with what is to follow, and weld it into a consistent whole,

2. Having determined on the nature and amount of the subject matter, the teacher should—

(a) Think over his lesson, and jot down the points of it in any order in which they may occur to him.

(?;) Having done this, he should re-arrange his points under heads, as i., ii., iii., etc., adopting what he considers the best order for securing clearness, continuity, and effect ; and bearing in mind that, as far as possible, he should proceed by easy steps :—

(i.) From the known to the unknown, (ii.) From the simple to the difficult, (iii.) From the concrete to the abstract, (iv.) From the particular to the general.

ON TEXT BOOKS FOR STATE SCHOOLS.

By Alex, Gough, H.T., S. School 972, Cobaw.

Nelson’s Royal Readers are a great improvement on the Irish National School Readers. Teachers and pupils of State schools and of private schools prefer the Royal Readers. But are they so well provided with good Text Books on grammar, geography, and arithmetic? I think not. The officers of the Education Department should write, or cause to be written, Text Books on the above subjects, with the words, “Published by direction of the Education Department of Victoria” impressed on the covers. Speculators have tried and failed to supply this want. No private individual can accomplish it to the satisfaction of the department, the teachers, and the pupils. A Board of Inspectors can do this to perfection. They know exactly what is wanted.

Grammar.—On the subject of grammar they should take Morell’s as a basis. It is certainly the best we have got. Abetter table is required, more specimen parsing, more Latin and Greek roots—for these are of immense importance to the non-classical scholar. I think it would be better to have two tables for parsing ; one for simple parsing and one for parsing fully. Davidson and Alcock would give hints for parsing fully ; but the table for simple parsing should be on the same principle as Abbott’s. His table could be greatly improved on, especially the adjective, verb, and preposition. His principle of “ therefore it is a,” is excellent, causing the children to exercise their intelligence in discovering the function of the word in the sentence, before deciding the question, what part of speech the word is. In the arrangement of the new grammar, the programme of instruction should be kept in view. The language should be more simple and grammatical than that of Morell. I pointed out some of Morell’s errors in the April issue of this paper.

Geography.—We learn the position of places from a good atlas, not overcrowded with names. There are few of this description. The plan I take is to mark with red ink on the Imperial Atlas bf Australia and the Universal Atlas, the places which ought to be known, either because of their position, population, commerce, productions, or history. Then, when the position of any one of these places is proposed, “ the eye remembers ” its situation. Smith’s geography is little more than a list of places which he thinks should be known. The State school geography should be by authority, and that authority should decide the question, What places must be known? The geography should contain all the information necessary for the correct answering of every question in geography, proposed at Teachers’ examinations and at Result examinations. For instance, if a third class must know Cape Willoughby is E. and Cape Borda W. of Kangaroo Island ; or if a teacher must know Batoun is on the Black Sea ; it would be but fair to both pupil and teacher to have these places mentioned in the Text Book. Captains and historians even want their charts and note-books as reminders. Historical and other information should be given in small print after describing the ¡positions of all the places of importance in any particular country. For instance, under the heading, Tasmania, I should commence thus :

Coast.—Eddystone Point, Saint Helen’s Point, Freycinet Peninsula, Fowesteir Peninsula, Tasman Peninsula, Oyster Bay, Cape Pillar, Port Arthur, Cape Raoul, Storm Bay, Norfolk Bay, Tasman Head, Cloudy Bay, Cape Bruni, D.’Entrecasteaux Chi., S.E., S., S.W. Capes, Port Davey, Rocky Cape, Port Hibbs, Cape Screll, Macquarie Harbour, Sandy Cove, Cape Grim, Circular Head, Emu Bay, Port Frederick, Port Sorell, Port Dalrymple, Anderson Bay, Riugaroma Bay, Cape Portland, Franklin Inlet, Banks Straits.

.Islands:—Schontcn, Maria, Bruni, Robbins, Hunter, Three Hammock, Swan, Clark, Bawen, Furneaux Group, Flinders.

Rivers.— Tamar into Bass’s Straits, Tributaries, Esk and South Esk, Macquarie is tributary of South Esk. Derwent into Storm Bay, tributaries Ouse, Clyde, Jordan. Eicon into D’Entrecasteaux Channel. King and Gordon into Macquarie Harbour, Arthur and Pieman W. into Southern Ocean.

Lakes.—The Great Lake, Lakes Echo, Sorell, St. Clair, Crescent, Arthur.

Mountains.—Great Western, Eastern Tiers, Arthur Ra, Surrey Hills, Mount Humboldt, Mount Wellington, Ben Lomond.

Towns.—George Town, Launceston on Tamar, Ross, Campbelltown, Longford on Macquarie, Perth on South Esk, Hobart Town, Bridgewater, New Norfolk on Derwent, Brighton on Jordan, Hamilton on Clyde, Bathurst on Port Davey, Eddystone, St. Helen’s, Falmouth on East coast, Swansea on Oyster Bay, Adelaide on Jiuon.

Here I have given only the positions of places, and I would do this in all cases before giving any historical or other information about the country in general, or any particular part of it. The geography should contain more information about Victoria and the other colonies than we find in Smith’s or Buckley’s. Smith notices a few Victorian rivers, but describes none. I will describe the Murray and the rivers flowing into it, as an example.

Murray.—Forest Hill, Australian Alps, flows N.W. into South Australia, then S.W, into the Southern Ocean, N. of Encounter Bay. Towns. —Albury (N.S.W.), Wodonga, Wahgunyah, Moama (N.S.W.), Echuca, Swan Hill, Wentworth (N.S.W.), (Wellington (S.A.). Mitta Mitta, Australian Alps, drains the country between Benambra and Bogong Ranges, flows N.W., enteis the Murray 8 miles east of Wodonga. Ovens, Australian Alps, drains the country between Bogong and Buffalo Ranges, flows N.W., enters the Murray 20 miles north of Wan-garatta. Towns.—Buckland Diggings, Bright, Myrtleford, Wangaratta, Tributary, The King. Broken River, near Mansfield, flows north, then, west, enters the Uoulburn two miles south of Shepparton. Town, Benalla. Broken Creek, effluent of Broken River, euters the Murray at Lake Moira (N.S.W.). Goulburn, Snowy Mountains, flows north, then west, then north-west, enters the Murray east of Echuca. Towns.— Matlock, Wood’s Point, Jamieson, Alexandra, Yea (tributary), Tallarook, Seymour, Mitchelstown, Nagambie, Murchison, Shepparton. Campaspe, Blackwood Ranges, flows north, enters the Murray at Echuca. Towns.— Kyneton, Rochester, Echuca. Tributary, Cobban. Towns.— Malmsbury, Taradale. Tributary, Bendigo Creek. Town, Sandhurst. Loddon, near Daylesford, flows north, enters the Murray at Castle Don-nington (Swan Hill).    Towns.—Guildford, Swan Hill. Tributaries,

Deep Creek, Bet-Bet.

In like manner, the Avoca, Tyrrell Creek, Avon, Richardson, Wimmera, Yarriambiach, Genoa. Snowy,Tainbo, Mitchell, Dargo,Latrobe,Macalister, Thomson, Yarra, Plenty, Saltwater, Werribee, Barwon, Leigh, Moora-bool, Hopkins, Emu Creek, Fiery Creek, Eumerella, Moyne, Glenelg, Wannon, should be described. We not only want good text books on geography, but good atlases to correspond, good wall maps of the Australian colonies, unlettered wall maps and globes.

Arithmetic.—Barnard Smith’s School Arithmetic, and the S. S. Arithmetic by the late J. J. Burston, should be in the hands of every teacher preparing for certificate examination. There is as yet no arithmetic arranged according to the programme of instruction, and adapted to the wants of the pupils. Burston is much superior to Smith in his definitions and rules. The new arithmetic should contain the definitions, rules (with a few exceptions), hints upon cancelling by inspection, proof by casting out the nines, tables, the greatest and least common multiple, directions to be observed in working fractions, exactly as given by Burston. B. Smith is less clear and precise, especially when explaining the principle of proportion and making the rules, than the most obscure of German philosophers, when attempting to elucidate Descarte’s famous fundamental principle, I think, therefore, I am. The price of the arithmetic should not be more than Is. or Is. 3d., and it should be arranged according to the programme, and contain one or more questions solved under each rule, with numerous examples for practice. If questions in “ Tare and Tret,” and such questions as those in Ex. 92 and Ex. 95, Colenso’s progressive examples, are fair questions for the fourth class programme, the rules for same should be given in their proper places.

City teachers may think an arithmetic is not necessary. The Teacher, Burston, and the B. B. are sufficient. That may be, for them, but not for country schools, where, with a good text-book, each pupil may be profitably employed. There can be no copying, as each is doing a different question, and therefore the teacher can give his attention to the other classes. Certainly, at stated times, the teacher, with the help of the class, works a question, or explains a rule on the B. B.

If a question cannot be solved without algebra in disguise—and there are many such in Burston’s and Smith's—it should not be given in the new arithmetic. Teachers should know Algebra, but it is not on the programme of instruction. The pupils should be made to understand that any term of proportion may be made the required term, and that all sums in proportion may be put in the form of equal fractions. Cancelling should be insisted on. Burston gives only one rule for division of decimals, Smith two. Here, Smith should be copied instead of Burston. When, for instance, I want to divide 3T4159 by 23,1 do not add five cyphers to the divisor. Common sense tells me I should never make my divisor greater than I find it. I simply ask myself how often will 23 go in 3. This must be a whole number, then in 31 ; this must be a decimal. Thus I find the quotient = 0'13659.

Algebra would help us to make rules for simple and compound interest, which would be more easily remembered than those given in the Arithmetics. Thus, given the interest (simple) and two of the quantities, principal, rate, time, to find the other. Rule—Multiply the interest by 100, and divide by the product of the other two. Given the amount, rate and time, to find the principal. Rule—Multiply the amount by 100, and divide by 100 +the product of the rate and time. To find the amount (compound interest). Rule—Raise the amount of one pound for the length of one payment, to the power denoted by the number of payments; and multiply the result by the principal. We may work by vulgar or decimal fractions. In compound, as in simple, the interest is equal to the difference between the amount and principal.

Book-Keeping.—Chambers’s and the I.N. S, Book-Keepings are good ones, but neither is complete without the other, A good Book-Keeping could be made by adopting the principle and questions of the latter, and addiDg a Bill-Book and Appendix as in the former. In the I. N. S. Book-Keeping, the pupil is compelled at every article to fall back on first principles ; but in Chambers’s, he has simply to copy what is placed before him. A boy learns more in a week, from the I.N. S. BookKeeping, than in a month from Chambers’s. The compendium of Merchants’ Accounts, from page 108 to the end, is unnecessary. It would be well to substitute, instead of this, an appendix containing all the extra information which is found in Chambers’s.

PAPERS FOR TEACHERS AND STUDENTS.

We take the following from the March number of Payers for Teachers and Students.

A SYNOPSIS OF LATIN PARSING.

By a B. So. (Honours) London University, May Examinations, 1881, I. Noun or Substantive.

(1)    Its Kind.

(rt) Proper ; as Roma, Cicero.

(b)    Common ; as liber, mensa, homo.

(c)    Abstract; as virtus, magnitude,

(d)    Collective ; or of Multitude ; as populus, exercitus.

(e)    Compound ; as senatus-consultum, jus-jurandum.

(2)    Its Declension.

1st, nom. in a ; as mensa, aquila.

2nd, nom. in vs, er, or von(neut.); as dominus, magister, regnum. 3rd, gen. inis/ as leo, leonis ; virgo, virginis : nomen, nominis. 4th, nom. in vs or u, gen, in vs ; as gradvs, gradus ; genu, genus. 5th, nom. in es, gen. in ei; as dies, diei.

[Note.—State if irregular.]

(3)    Its Gender.

(a)    Masculiue ; as hortvs, puer.

(b)    Feminine ; as amicitia, mors.

{c) Neuter ; as tempus, donum.

(4)    Its Number.

(a)    Singular ; as insula, oculus, bos.

(b)    Plural; as insulae, oculi, boves.

(5)    Its Case.

(a)    Nominative ; as dominus, leo.

(b)    Vocative; asdomine.

(c)    Accusative ; as dominvm, leonem.

\d) Genitive ; as virginis, leonum.

(e)    Dative ; as aetati, leonibus.

(f)    Ablative ; as virgine, diebus.

[Note.—It in the nominative case state what Verb it is subject to. If in the accusative case state what verb or preposition it is object to or governed by. If in the ablative case state what preposition (if any) it 1depending on. State the rule of syntax (if any) which causes thq noun to be in any particular case.]

II.    Adjective.

1. Its Kind.    _

(a)    Qualificative ; as bonus, tenor.

{b) Numeral; divided into—

I. Cardinal; as unus, duo.

II. Ordinal; as primus, seoundvs.

III. Distributive ; as singuli, bini.

{c) Of three terminations ; as bonus, bona, bonum.

[d)    Of two terminations ; as tristis, tristis, triste.

(e)    Of one termination (in the singular) ; as felix, felix, felix

(but felioes, felloes, felicia in the plural).

(2)    Its Gender.

(«) Masculine ; as magnus dux.

(b)    Feminine ; as bona mater.

(c)    Neuter ; as dulce carmen.

(3)    Its Number.

(a)    Singular ; as fortis vir.

(b)    Plural; as fortes viri,

(4)    Its Case.

(a)    Nominative ; as bonus vir.

(¿0 Vocative ; as bone vir,

(e) Accusative; as filium carum.

(d)    Genitive ; as fluvii alti.

(e)    Dative ; as felici homini,

_ GO Ablative ; as corpore sano.

(5)    Its Degree of Comparison.

{a) Positive; as vir eruditus.

^ (The positive need not be noted.)

(b)    Comparative ; as nox longior.

(tf) Superlative ; as mons asperrima.

[Note.—State if irregular.]

(6)    V hat noun it determines or qualifies, and agrees with (in num

ber, gender, and case).

III.    Pronoun.

(1)    Its Kind.

(a)    Personal; as ego, tv, nos.

(b)    Reflexive ; as se.

(c)    Possessive ; as meus, tuns.

{d) Demonstrative ; as is, ea, id; hie, liaec, hoc.

(O Definitive ; as idem, eadem, idem.

(/) Relative ; as qui, quae, quod.

(g)    Interrogative; as qnis ? quid?

(h)    Indefinite ; as quis (any one).

(i)    Compound ; as quisnam, quisquis, aliquid.

(2)    Its Person.

(a)    First Person; as ego, nos, mihi.

(b)    Second Person ; as tv, vos, vobis.

(c)    Third Person ; as ille, is.

(3)    Its Gender.

{a) Masculine ; as ille, qui.

(b)    Feminiue ; as ilia, quae.

(c)    Neuter ; as illud, quod.

(4)    Its Number.

{a) Singular ; as ego, tu.

(b)    Plural ; as nos, vos.

(5)    Its Case.

(a) Nominative ; as ego.

{b) Vocative ; as O mi pater.

(c)    Accusative ; as me, te, Jmnc.

(d)    Genitive ; as cjus, cujus, mei.

(e)    Dative ; as mihi, vobis.

(7) Ablative; as quo, qua, quo; illo, ilia, illo.

(6)    The noun to which it refers, and with which it agrees (in person gender, and number). Relative pronouns agree with their antecedents. Possessive, demonstrative, and definitive pronouns have an adjectival force, qualifying a noun and agreeing with it in gender, number, and case.

IV.    Verb.

(1)    Its Kind.

(a)    Auxiliary ; as amatus sum.

(b)    Regular ; as amare, monere.

(c)    Irregular ; as dare, crescere.

(d)    Transitive ; as trahere, clauclere.

(e)    Intransitive ; as fluerc, gemere.

(7) Deponent; as loquor, orior, sequor. ig) Impersonal; as constat, pluit, accidit.

{h) Defective ; as aio, inquam, memini.

(i) Inceptive or Inchoative ; as mitescere, vesperascere.

{k) Frequentative ; as rogitare, cantare.

(2)    Its Conjugation.

[a)    First, in -are; as amare, laudare.

(b)    »Second, in -ere ; as monere, gaudere.

Cc) Third, in -8re ; as regere, scribere. if) Fourth, in -ire; as audire, dormirc.

(3)    Its Voice.

(a)    Active ; as amo, reget, audivit.

(b)    Passive ; as amatur, regetur, auditus est,

(4)    Its Mood.

(a)    Indicative ; as Rex regit,

(b)    Subjunctive (sometimes called Conjunctive); as Legit ut

disca-t.

(c)    Imperative; as surge, studete.

(5)    Its Tense.

(a) Present; as Pueri ludunt.

longius, naves interim longas aedificari in flumine Ligeri, quod influit in Oceanum, remiges ex Provincia institui, nautas gubernatoresque com-parari jubet. His rebus celeriter administratis, ipse, cum primum per anni tempus potuit, ad exercitum contendit. ”

(Translation.—Cajsar, being informed of these things by Crassus, because he was too far distant, meantime commands ships of war to be built on the river Loire, which flows into the ocean, rowers to be raised from the province, sailors and steersmen to be procured. These things being quickly performed, he, as soon as he could through the time of year, hastens to the army.)


Quibus

de

rebus

Caesar

ab

Crasso

certior

factus

quod

ipse

aberat


longius

naves

aedificari

Ligeri

quod

jubet


his

rebus


primum

per

potuit


(b)    Imperfect; as 1'ulgebat luna.

(c)    (Simple) Future; as TJret ignis.

(d)    Perfect; as Gecinit avis.

(e)    Pluperfect ; as Cicero scripserat.

if) Future Perfect; as Pomum ceciderit.

(6)    Its Person (agreeing with nominative, or subject).

(a)    First Person ; as (ego) amo, (nos) regimus.

(b)    Second Person ; as (tu) manes, (vos) videtis.

(c)    Third Person ; as (die) audit, (illi) pugnant,

(7)    Its Number (agreeing with nominative, or subject).

(a)    Singular; as Homo est; Dies venit.

(b)    Plural; as Homines sunt; Dies veniunt.

(8)    Nominative or subject with which it agrees.

The above are the forms of the Verb Finite. The following are the forms of the Verb Infinite, consisting of Verbal Nouns and Verbal Adjectives:—

(1)    Verbal Nouns.

(a)    Infinitive (also called Infinitive Mood).

Present; as amare.

Perfect; as amavisse.

Future ; as amaturus esse.

(b)    Supine ; as cubitum ire.

(c)    Gerund ; as amor legendi.

Note.—In parsing a Verbal Noun, it is necessary to give the verb, adjective, noun, or other word on which it depends.]

(2)    Verbal Adjectives.

(a)    Participles—

Present, or Imperfect; as monens.

Future ; as amaturus, rccturus.

Past, Passive, or Perfect ; as monitus.

(b)    Gerundive ; as Amici novi veteribus non sunt anteponendi.

[Note 1.—In parsing a Verbal Adjective it is necessary to give the same particulars as in other adjectives, and to state the noun or pronoun which it qualifies and agrees with.]

[Note 2.—In parsing any Verbal form, whether of the Verb Finite or Infinite, give the principal parts of the verb, namely, the Present Indicative (1st per. sing.), the Perfect Indicative (1st. per. sing.), the Present Infinitive, and the Supine. Thus : amo, amavi, amare, amatum.]

V.    Adverb.

(1)    Its Kind.

[a)    Manner ; as sap lent er, bene.

(b)    Time ; as heri, hodie, eras, semper.

(e) Place ; as ibi, hie.

(d)    Degree ; as valde, admodum.

(e)    Affirmation ; as certe, sane.

if) Negation ; as non, nc, haudguaquam.

(g)    Numeral; as semel, bis, ter.

(h)    Interrogation ; as cur, ubi, quomodo.

(2)    Its Degree of Comparison ; only if—

Comparative ; as longius, scepius.

Superlative ; as faomime, saepissime.

VI.    Preposition.

State the case (or cases) which it governs, whether—

(1)    Preposition governing Accusative ; as ad, contra, per.

(2)    Preposition governing Ablative ; as ab, de, cum.

(3)    Preposition governing Accusative or Ablative ; as in, sub. Name the words between which it denotes a relation. Thus, in the

-sentence, “ Venit ad Romam,” the preposition ad denotes the relation between venit and Rvmam. -

VII.    Conjunction.

(1)    Its Kind.

(a)    Co-ordinative, joining co-ordinate sentences together ; as,

et, ac, amt, sed. (These do not affect mood.)

(b)    Subordinative, joining a subordinate sentence to the sen

tence on which it depends ; as ut, ne, quum. (These affect mood.)

(2)    The mood (if any) which it governs. This relates principally to some conjunctions which govern the subjunctive mood.

(3)    Name the sentences, phrases, or words which it connects.

VIII.    Interjection.

(1)    The emotion of the mind or other operation which it expresses.

(a)    Joy; as 0! euge!

(b)    Grief ; as heui cheul vae!

(c)    Surprise ; as 0! hem! babae!

(d)    Disgust; as phui ! phy !

(e)    Derision ; as aha! hahahae !

(/) Invocation ; as Of lieus ! eia !

(g)    Calling attention ; en! ecce!

(h)    Encouragement; eu ! euge !

(i)    Deprecation ; pro ! or proh !

(2)    The case (if any) which it governs. (Some interjections govern

a case.)

(a)    Vocative, or Nominative; as heus! puer! heus tu quid

agis ? pro sancte Jupiter !

(b)    Accusative ; as pro deum fidem! hem Davum tibi.

(c)    Dative ; as hei mild misero !

The following sentences are taken from Caesar’s Gallic War, Book IIP, p. 9. We parse some of the words in illustration of the above scheme. “ Quibus de rebus Caesar ab Crasso certior factus, quod ipse aberat

Relative pronoun, 3rd pers., fern., plur., ablative, agreeing with and in apposition with “rebus.”

Preposition governing ablative, denoting the relation between “ certior ” and “ rebus.”

Com. noun, 5th declension (res, rei), fem., plur., abl., after the prep. “ de.”

Prop, noun, 3rd declension (Caesar, Caesaris), masc., sing., nom., subject to “jubet.”

Preposition governing ablative, denoting the relation between “factus” and “Crasso.”

Prop, noun, 2nd declension (Crassus, Crassi), masc., sing., abl. after prep, “ab.”

Qualificative adj., masc., sing , nom., cornpar. degree, agreeing with “Caesar.”

Perf. part, of verb facio (feci, facere, factum), masc., sing., nom., agreeing with “Caesar.”

Subordinative conjunction, connecting “Caesar jubet naves aedificari,” etc., with “ipse aberat longius.”

Personal pronoun, 3rd pers., masc., sing., nom., subject to “ aberat,” agreeing with “ Caesar.”

Verb, compouncbof “ esse. ” Principal parts : absum, abfui, abesse, no supine. Indie., imperf., 3rd pers., sing., agreeing with subject, “ ipse.”

Adv. of place, cornpar. degree, qualifying “aberat.”

Com. noun., 3rd declension (navis, navis), fem., plur,, accus., object to “jubet.”

Verb infinite, passive, infinitive used with the accus. “ naves ” as indirect object.

Prop, noun, 3rd declension (Liger, Ligeris), masc,, sing., ablative, in apposition with “flumine.”

Relative pronoun, neufc., sing., nom., subject to “influit.” Irreg. trans. verb, 3rd conjug. (jubeo, jussi, jubere, jussum), active, indie., pres., 3rd pers., sing., agreeing with subject, “ Caesar.”

Demonstrative pronoun, fem,, plur,, ablative, agreeing with “ rebus.”

Com. noun, 5th declension (res, rei), fem., plur., ablative, absolute.

Subordinative temporal conjunction, connecting “ ad exercitum contendit ” with “ primum potuit.”

Adv. of time, superlative degree, qualifying “potuit.” Preposition governing accus., denoting relation between “ tempus ” and “ potuit.”

Verb finite, compound of “ esse.” Principal parts ; possum, potui, posse, no supine. Indie., perf., 3rd pers., sing., agreeing with subject “ ipse” understood,

At a recent meeting of the San Francisco Academy of Sciences, a specimen of the desert land tortoise, captured at Cajon Pass, was shown, which, on dissection, was found to contain about a pint of clear water. This kind of tortoise is found in regions where there is no water, and traveller suffering from thirst could in an emergency supply himself with water by killing one. A Mr. Redding stated that he had captured 92 of these tortoises and taken them on ship board to San Francisco. They were on board for two months, during which time they neither ate nor drank, although both food and water were offered to them, and when killed considerable quantities of water were found in each of them.

At a recent meeting of the Engineers’ Club of Philadelphia, Mr. John E. Codman exhibited drawings of and described Nicholson’s fire-escape, which consists of a fire-proof brick tower, octagonal externally and cylindrical internally, with central shaft about 18 inches diameter, around which is formed a winding passage, of a U-shaped section 2ft. 3in. in width, with smooth or glazed surface, and inclined at angle of 35°, with retarding curves of less gradient. Fireproof doors would connect with each floor and roof, and a vestibule with the surface of the ground below. It is intended that those escaping shall assume a sitting posture on entering the spiral and slide to the bottom, and it is claimed to be safer than other escapes for those unaccustomed to ladders, or weakened by fright or excitement.

The tunnel under the St. Lawrence is to be as follows Entire length, about 21,700 feet; open cuttiDgs on the Hochelaga side, 2,500 feet, and on Longueuil side. 4,220 feet; actual length of tunnel proper, 14,980 feet. It is to be 26ft. wide inside and 23ft. high. It will be lined with brick masonry throughout, except the fronts, which will have facades of stone. The arch will vary from 20in. to 30in, in thickness, according to the character of the ground to be supported.

At a recent meeting of the Newcastle-on-Tyne Photographic Association a paper was read by Mr. John P. Gibson, chemist and druggist, of Hexham, in which the author described his successful photography of a flash of lightning on the night of the last July 5, by means of Swan’s plates of ten times the rapidity of wet collodion, and backed with red paper to prevent halation.

NOTICE TO ADVERTISERS.


In sending advertisements for insertion in the Schoolmaster, advertisers will please remit stamps for amount at the following scale : —

16 words, One Insertion,

- Is. Od.

1 32 words, One Insertion

- 2s. 6d.

24 „ „

- 2s. Od.

1 One Inch, „

- 4s. Od.


NOTICES TO CORRESPONDENTS.

Advertisements and other business communications should be addressed to the Publishers. No advertisements will bo inserted without a written order, or prepayment. It is particularly requested that they may be sent early in the month.

Books, music, and school appliances for notice, and all letters containing anything connected with the literary portion of the paper should be addressed To the Editor. Every communication accompanied by the name and address of the sender (as a guarantee of good faith, though not always for publication) will be acknowledged ; but we cannot attend to anonymous letters.


INSTRUCTIONS TO SUBSCRIBERS.

Lady subscribers, when remitting their subscriptions, will please state whether their papers are to be addressed Mrs. or Miss.

Subscribers will please send P. 0. order or stamps, when remittance is under ¿81.


¿tiistralashiii Stljualmasitr,

PUBLISHED EVERY MONTH.


CONTENTS



Papers for Teachers and

Students ... ...

... 163

Leader .........

... 168

Notes of the Month ...

... 168

Science and Art Gossip

... 169

Education Commission

... 170

Victorian Education Department—

Appointments ...

... 175

Friendly Advice to

Pupil

Teachers ... ...

... 175


ANSWERS TO CORRESPONDENTS.

“ S. Trend ”—June 30, 1882.

Received.—“W. W. Birrell,” “ Miss S. Presland,” “T. W. Blanche,” to December 31, 1882.

“ N.O.”—In our next issue.


NOTICE TO SUBSCRIBERS.

We would call the attention of those of our Subscribers whose Subscriptions are in arrear to the necessity for prompt payment of the same.


the council be nominated by the subscribers, the Government, the Trades’ Hall, and the trustees of the Public Library. My desire being to help the artisans of the colony in their efforts to improve themselves, I should like that the college be especially for working men, but open to all who may wish to attend its art, science and miscellaneous classes and lectures. Of course, fees will be charged, but the council will, I am sure, recognise the importance of fixing them as low as possible, so that the advantages offered by the college be kept easily within the reach of all. The college will not be self-supporting —at least for a time ; but as the institution will, I trust, prove a most valuable link in the educational chain, I hesitate not in allowing myself to believe that the Government- -any Government which may be in power—will give a small annual grant to assist in carrying on the work of the institution. At the invitation of the president of the Public Library, I have had the pleasure of conferring with the trustees of that institution as to the desirability of establishing the college in connection with the Public Library and National Gallery, and here I would express my thanks for the earnest and kind way the trustees entered into the question. I am indebted to them for the suggestion that I should have an interview with the representatives of the Trades’ Hall; as also for their kindness in offering to assist in any and every way to carry out the scheme. Again, it is due to them to state that I am impressed with the belief that they are willing to act in conjunction with other associations, or, if need be, to take upon themselves the responsibility of meeting my conditions. If it should be found necessary to establish a fund for the purpose of covering the maintenance and repairs of the building, I shall be prepared, as soon as the college is erected, to give £1000 towards such fund, provided a sum of £2000 is raised from other sources.” We are glad to find that the Hon. the Premier has announced the intention of the Government to second Mr. Ormond’s noble effort.


’aies af fjxe Iflanflj.


MELBOURNE, MAY, MSL

It is a pleasing indication of the altered circumstances of colonial life that magnificent gifts in aid of education are becoming of frequent occurrence. The Wilson Hall of the University of Melbourne, and the Ormond College are evidences that provision for the teaching of the higher branches of learning will keep pace with the extension of our elementary system of instruction. From the following letter it will be seen that the founder of Ormond College has determined upon originating a new college, one that shall constitute a complement to the State school, into which the artisan class may enter and qualify themselves for industrial and manufacturing pursuits. The enlightened view Mr. Ormond takes of the responsibilities of wealth in this young community is worthy of all commendation, and should find many disciples. In regard to his proposal to assist in the establishment of a working man’s college, he says:—'‘The conditions of my offer are simply two:— — (1.) Towards the erection of a building to cost £10,000, and to be known as the Melbourne Working Men’s Art, Science and Technological School, I will give £5000, provided the community contribute a like sum and the Government afford a suitable site. (2.) That the council or governing body should consist of twenty members, and that I should have the privilege of appointing one-fifth of the number ; and that the remainder of

A considerable portion of our space this issue is devoted to the proceedings of the Education Commission, a full report of which, we think, cannot fail to be of the utmost importance to State School Teachers in Victoria.

As about £6,500 has already been subscribed for the extension of Trinity College, Melbourne, it is proposed to set about that work as soon as possible. Extension will be more after the fashion of the Old English Colleges than any other in Victoria, and will adjoin the present "portion of Trinity College. The building was designed by the same architect who designed the Sydney University and several other handsome buildings in the colonies, and is to be capable of accommodating about 25 new students A long felt want will be thus supplied.

The first progress report of the Education Commission, which recommended that the architectural branch of the department should be amalgamated, has been considered and approved of by the Cabinet.

With a view7 of having the opinions of the boards of advice throughout the colony laid before the Education Commission, the council of the Victorian boards of advice have prepared a series of printed questions, which will be sent to the various boards in the colony with a request that the replies be filled in and returned at the earliest opportunity. The answering of these questions will not in any way interfere with any evidence which individual boards of advice may desire to tender direct to the commission.

By permission of the Minister of Education the hon. secretary of the Victorian Society for the Protection of Animals has issued a circular addressed to the correspondent of every board of advice in Victoria, inviting the boards to provide in the State schools under their charge for the inculcation of the principles of kindness to animals, and suggesting various methods by which this may be carried out.

We have received from Mr. S. Mullen, bookseller, of Collins-street, east, a copy of a new work entitled “ An Introduction to French Authors,” by Antonie Charlin. The work is admirably adapted for teachers, and students who wish to acquire facility in reading the French language.

Mr. M. L. Hutchinson, bookseller, of Collins-street west, has lately received copies of “The International Atlas and Geography, Modern, Historical, Classical and Physical.” Containing 120 maps. Descriptive letter-press by James Bryce, LL.D., William F. Codier, LL.D., and Leonard Schmitz, LL.D. This is certainly one of the most complete works of the kind yet published by Messrs, William Collins and Sons. It contains a mass of information invaluable to teachers on each branch of geographical knowledge. The maps are all new, and are not overburdened with matter. The paper and letter-press are of the very best description. It has been published at a most reasonable price, and as Mr. Hutchinson makes a liberal allowance to teachers it should find a large sale in this colony.

ticiicc aitb %xt (gossip.


A meeting of the Council of the University of Melbourne was held on Monday, May the 1st, the chief business being the election of a Chancellor and Vice-Chancellor of the University. The discussion wrns lengthy and strong, and the voting resulted in the re-election of Sir. W. Stawell as Chancellor, and Dr. Brownless as Vice-Chancellor. The council, however, seemed so divided that at the next meeting Sir W. Stawell thought fit to resign his position. The chief point of discussion at the first meeting was whether a new Chancellor should be elected each year, but this is only the form in which a long standing difference has broken out. A certain number of the members of the council representing the outsiders have always been at variance with the others, and being dissatisfied with the rulings of the Chancellor, they held a meeting, at which they determined that a new Chancellor should be elected, and thus 'arose the discussion.

At the meeting of the Council of the Melbourne University, held on May 1st., Mr. Gregory was appointed lecturer on the law of property in the place of Mr. Billiug, Q.C., who lately resigned.

A manual of Elementary Drill for the use of pupils and teachers has been prepared by Mr. Edmund D. Hunt. It comprises instruction in all the drill that is usually taught to a recruit, and is divided into four different parts, the first being “Drill with intervals,” the second “Drill in single rank,” the third “Drill in two ranks,” and the fourth “ Company drill with exercises in battalion movements,” The instructions are given in the plainest possible manner, and so as to be thoroughly adapted for the use either of master or pupil. The book ought to supply a want. Another book which has been recently published, and ought to be of great use both for amusement and for instruction in schools and at home, is “The Shakspeare Reading Book,” prepared by Mr. H. C. Bowen. It consists of fifteen of the best of Shakspeare’s play’s abridged, with a few illustrations.

The National Schoolmaster is very sarcastic upon Mr. Mundella’s speech at Liverpool on the 21st of January last. The following is a specimen of their attack upon him :—“ It* certainly is a most remarkable outpouring of hysterical grief. It is a spectacle that happens sometimes in private families. Everybody is amiable and saying pleasant things, and suddenly someone bursts forth into hysterics and says she is deeply injured, that everybody is saying unkind things about her, and that she doesn’t want to live auy longer. We always knew that Mr. Mundella was of a feminine temperament, but we never thought he would give way like this. Everybody has been saying smooth things. The Schoolmaster has been—to use Mr. Gray’s phrase—redolent with compliments. The School Guardian has never ventured to express its doubts about the code that is coming, without revelling in expressions of admiration and appreciation of Mr. Mundella’s Educational enthusiasm. Archbishops and Cardinals have spoken in high terms of praise, and yet Mr. Mundella is not satisfied. We cannot help saying a few words to Mr, Mundella about the exhibition of more than womanly weakness.”

The London School Board held a meeting on January 26th, at which it was resolved as follows :—The School Board calls the attention of the Education Department to the conditions of the admission to the training colleges, and to the conditions which impede the admission of Board School pupil-teachers to the full educational advantages of training, and urges the Education Department to take such steps as may seem to them necessary in order to secure the Board School pupil-teachers equal advantages for training with those who come from denominational schools.

The London School Board Budget for 1882, which was presented to the board at its meeting of January 26th, gives the following estimates for the probable cost of maintenance for the year :—-For the maintenance and Board Schools £100,223. For additions to buildings, repairs, &c., £25,000. For enforcement of compulsion, maintenance of boards, &c. £31,154. For the industrial schools, £37,728. For office expenses, £21,172. For interest and repayment of loans, £206,311. For legal expenses, £2,500, and for stamp duties. £750. After sundry receipts and the surplus left over from 1881, the amount required will be £676,579. The approximate estimate for 1883 is also given, and is given at £679,595.

At the distribution of the Lancastrian prizes won by the scholars of the elementary schools, Mr. Mundella, in pointing out the advantages of the present education system, made the following remarks :—It has been said by some, wbo admitithese advantages, that the cost is so great as to overbalance them, but, admitting that it is large, 1 think it can be easily shown that the advantages are of greater value. The total expenditure upon education for the past ten years has been £72,000,000, just one-half the sum spent upon tobacco during the same period. I think England can well afford to spend in education half what it spends in smoking. Why, in six months the country spent in drink just as much as it spent upon education in ten years !

Several schoolmasters in England have been lately agitating to have themselves made civil servants. Of this the School Guardian remarks : —Mr. Mundella made clear in his speech at Liverpool that they were not civil servants, and he congratulated them thereon. In his opinion teachers had far better prospects in the open market, where brains, and work, and character will generally command their price, than they would have if they occupied the position of government servants, We are not sure that the teachers’ case is quite so glowing as Mr. Mundella paints it. But we have always thought that the claim to belong to the civil service could not be sustained, and it is perhaps as well that on the eve of the new code’s appearance an unmistakable authority should pronounce upon the subject.

The Minister of Education has given his sanction to the starting of a penny subscription in the State sohoolsfor the relief of the Jews in Russia, who have been so cruelly persecuted. It is hoped that a large sum will be raised in this manner.

The Mayor of Adelaide has arranged for Tennyson’s song to be sung at the military ball to be given on Queen’s Birthday.

The Auckland Weekly News says :—By the first of January, 18S3, two professors are to be in Auckland—a professor of natural philosophy, and a professor of classics and English literature. The deliberations of the university senate have led to results which will materially affect the future of Auckland, aud no time have been lost in following out the suggestions of that body. Distinguished commissioners have been appoiuted to select proper candidates for the posts, and in the course of a few months we shall probably hear the names of those to whom a great part of the education of our young men will be entrusted. The opinion was held, and is still held, we believe, by some, that it is impossible to induce good men to leave England. But this we consider, and have always considered, a fallacy. It is contradicted by experience, as of late superior and successful men have landed on our shores. Twenty years ago several unfortunate selections were made from England with this result, that it was instantly said “ it is no use sending home, we must fall back upon the colony,” the fact being that sufficient care was not exercised in the choice of candidates.

The following appointments of summoning officers under the Education Act have been gazetted:—For Walhalla, Thomas Ellis, Senior-Constable. ForEchuca, Senior-Constable Henry Williamson, and for Tarna-gulla, Constable John Considine.

Mr. S, R. Persse has been appointed returning officer for the school district of the Shire of Bungaree.

The Public Schools Annual Boat Race was rowed on Friday and Saturday last. On Friday the Wesley College and Melbourne Grammar Schools were defeated, and on Saturday the Geelong Grammar School and the Scotch College contested the final heat. The Geelong Grammar School crew pulled ahead, at first, and obtained a lead of three lengths. The Scotch College crew then pulled up, but their opponents maintained the lead and won the race by half a length.

At the next monthly meeting of the Horticultural Society of Victoria Mr. A. C. Neate will deliver a lecture on Horticulture, considered as an auxiliary of moral and social education. This lecture was postponed from last meeting on account of the business being too long.

The South Australian Electric Company are making great preparations for supplying the electric light to shops and dwelling-houses in the city.

In the February issue of the Popular Science Monthly, a paper is published from Mr. Charles S. Bryant, of St. Paul. The object of this paper is to show that there is no real disparity between the ages of the patriarchs of old and modern men. He endeavours to show that the characters for the Hebrew numbers have always been read, and states that by the correct reading Adam was 139 years as compared with 930 in the Bible. He gives a list of the ages—Seth, 121; Enos, 114; Cainan, 119; Mahalabel, 122; Jared, 117; Enoch, 114; Methooselah, 124; Lamech, 117; Noah, 189 ; making the average age 120|, and this, he says, chimes in with the passage in Genesis iv. 3, “And the Lord said, my spirit shall not always strive with man, for that he also is flesh, yet his days shall be an hundred and twenty years.”

The latest piece of naval architecture is an entire novelty. Mr. R. Fryer has constructed a vessel which rests on three 10-feet diameter globular floats arranged like the wheels of the tricycle. The weight of the vessel submerges these about one quarter, and the globes are set revolving like paddles. The inventor claims the following advantages of it:—1. That, at least as far as the model is concerned, a given amount of power will drive it further and faster on the water than on a level track, which indicates the possibility of attaining as great speed on the water by wheels as on railroads. 2. That obstructions placed in the path of the wheel in the water are more easily ridden over or deflect the wheel with less jar or damage than they would on land. 3. That the vessel can be guided by the wheels or globes as readily as by a rudder. 4. That in rough water there is a minimum strain on the body amidships, 5. That the globe-wheels run readily out of the water and up a steep plane as a beach ; or that they can be run on a track by fitting the keel of the globe.

Ti-ie introduction of electric lighting at the Grand Opera House Paris has been found to improve the acoustic properties of the place.

To remove grease stains from paper, scrape some pipe-clay over it, cover with a piece of thin paper, and pass a heated iron on it for a few seconds.

Paper can now be pressed into such a hard substance as to be incapable of being scratched by anything but a diamond. It will be, perhaps, soon used for furniture.

Edison is trying to convert coal directly into electricity.

The concluding lecture of the series under the auspices of the Glasgow Science Lectures Association, was one on the Biological Relations of Australia and New Zealand, contributed by Mr, Alfred Wallace, F.R.S. Mr. Wallace, after pointing out that New Zealand is really a continental and not an oceanic island, proceeded to describe the differences and resemblances between the fauna and the flora of the two countries. The absence of mammals from New Zealand showed that there could have been no land connection with Australia since the marsupial and other mammals first entered Australia, which must have been at a remote period. All the facts concerning the animals of the two countries indicated a very ancient union, if a union at all; but it looked more like a union of both with some common land rather than a direct union of each

other, and this was probably in the secondary period. The flora of the two countries threw more light on the subject even than the animals. Australia has a wonderfully rich flora, and New Zealand a very poor one. He pointed out that the form of the sea bottom gives indications as to the direction in which connection existed between lands now separated. Islands whose natural productions are almost the same as those of the adjoining continentare connected to the continent by shallow water notexceedinglOO fathomsin depth. It was on this basis that heexplaiDed the differences in the fauna and flora of the two countries: New Zealand, he concluded, gathering up his argument, has a distinct amount of resemblance to Australia in its species of animals, and especially in its groups of animals and plants ; but, at the same time, it has a very wide and deep-seated diversity. Though further south than Australia, its affinities are not with temperate Australia, but with tropical Australia and Polynesia. These features were explained on the hypothesis that it never was connected with the mainland of Australia at all, but was really connected with the eastern island at a time when Australia was divided into two islands, and had derived its animals and plants from a different source. The form of the sea bottom shows that there is a bank connecting New Zealand and tropical Australia, while there is a very deep sea between New Zealand and temperate Australia. All the important features of New Zealand fauna and flora, as well as those of Australia, were explained by this hypothesis, and in this way it was seen how important to the study of natural history was a knowledge of the past history of the earth. On the motion of Sir James Watson, a cordial vote of thanks was given to Mr. Wallace for his lecture.

The second annual report of the City and Guilds of London Institute for the advancement of technical education has been published, and so much progress is evidenced by it that very soon there will be a widespread system of technical schools all over the country, and those in London are increasing rapidly.

W. II. Flower, in his book on Fashion in Deformity,” after discussing the various fashions in barbarous countries which encourage individuals to deform their bodies, gives a timely warning to the ladies of fashion in England at the present day.

Nature publishes a letter from South Australia which gives some idea of the weather near Cambridge in summer. The following is an extract:—“To-day (Jan. 19th) was, I think, as hot indoors as Tuesday. Metals in the room were unpleasantly hot to the back of the hand. The leaves are falling off the trees from the intense heat and dryness of the air. The ants which, up to 9 a.m., were busy forming a column varying in width from eighteen inches to three feet, and almost colouring the • ground, retired to their nests. Not one was to be seen. Neither a bird, fly or other insect was visible, unless disturbed by the rustling of the leaves.

1 In the English new code provision is made for the teaching of science. Under elementary science the object of instruction is stated to be the cultivation “ of habits of exact observation, statement and reasoning.” For the first standard lessons in “ common objects such as familiar animals, plants and substances employed in ordinary life,” are to be given. For standard IV. there is required “a more advanced knowledge of special groups of common objects, such as (a) animals or plants, with particular reference to agriculture ; (5) substances employed in arts and manufactures, {o) the ample kinds of physical and mechanical appliances. e.g., the thermometer, barometer, lever, pulley, wheel and axle, spirit level.” For standard V. we have—‘ ‘ (a) animal and plain life, (b) the chemical and physical principles involved in one of the chief industries of England, (c) the physical and mechanical principles involved in the construction of the commoner instruments, and of the simpler forms of industrial machinery.” For standard VI. and VII. the preceding subjects are set down in fuller detail.

Nature says :—(,A few months ago the Rev. W. S. Green, of Carriga-line, County Cork, started on a mountaineering expedition to New Zealand. Mr. Green was accompanied by two Swiss guides, and a telegram just received announces that the party has succeeded in making the ascent of Mount Cook.”

A Bill for compelling railway companies to use continuous breaks has passed the second reading in the House of Lords.

At the Paris Academy of Sciences M. Blavies, mining engineer, called attention to the disappearance of sardine from the coast of Brittany, where it used to bring the fishermen 15,000,000 fr. a year. Ho attributed this to a change in the direction of the Gulf stream, which also accounted for the mild winter and early spring. On the suggestion of M. Faye, the question was referred to a committee composed of M.M. Faye, Jaussen, Daubrde, and Admiral Jarien.

An exhibition of all objects relating to bee culture was held at Vienna from April 8 to April 15.

In a letter to Nature Mr. H. II. Johnston states that the following rebuke was given to Miss F. P. Cobbe, the antivivisectionist, by a distinguished man of science, when she called upon him to gain him over to her cause :—Madam, charity begins at home. When you have given up wearing ostrich feathers which are plucked from the living bird, causing the most exquisite pain, and birds of paradise, which, in order to enhance their beauty and lustre, are shinned alive ; when you have abjured the use of ivory, because you know that the tusks are cut out of the dying elephant’s jaw, then, and then only, come and upbraid me with the cruelty of my operations. The difference between us is, Madam, that I inflict pain in the pursuit of knowledge, and for the ultimate benefit of my fellow creatures ; you cause cruelty to be inflicted merely for your personal adornment.” To this, in the next issue, Miss Cobbe replies that she never wears any of these, and that she never called upon “ the distinguished man of science,” and was never rebuked.

Sir Charles Wyville Thomson died from the effects of paralysis on the 10th March in his 53rd year.

The Emperor of Russia has granted 20,000 rubles to the St. Petersburg Geographical Society, as a subsidy towards the erection of a second Russia polar station in Nova Zembla.

The question of the utilisation of the Falls of Niagara as a source of power has become so serious that the method to be adopted is being discussed. In one project it is proposed to have three turbines, each about 3i feet in diameter, utilising a fall of 80 feet by means of a pipe 8 feet in diameter. Each of the three turbines would be of 1000 horse-power, and since the supply of water would be that of the great lakes and Niagara, itjwould be practically inexhaustible. Calculations have also been made of the power which may be drawn from the falls of the principal rivers of the State, From the Passaic, at Pattison, N.J., 1000 horse-power; the Merrimac, at Lowell, 10,000 ; the Mohawk, 14,000 ; the Connecticut, 17,000 ; the Androscoggin, 11,000 ; the Mississippi and St. Anthony’s Falls, 15,000 ; and some others. The total force of these falls represents 75,000 horse-power, and as each might be made to do at least double work it may be set down as in all 225,000 horse-power. As an equal amount of energy might be drawn from the smaller rivers in mountainous regions, the total available hydraulic force of the United States may be set dosvn as not less than 500,000 horse-power. That Niagara and other waterfalls in America will be utilised for machinery there can be little doubt; it is only a question of time ; though in actual experience a considerable deduction may have to be made from the above calculation.

EDUCATION COMMISSION.

[IN view of the great importance to teachers generally of having full reports of the proceedings of the Education Commission, a large portion of our space will be devoted thereto each issue? until the Commissioners bring their labours to a close.]

REPORT.

TO HIS EXCELLENCY THE GOVERNOR.

We, Your Excellency’s Commissioners, ■whose hands and seals are hereunto set, having been appointed by Your Excellency to enquire into the whole administration, organization, and general condition of the existing system of Public Instruction in this colony, with the object of ascertaining its deficiencies, improving its working, and while retaining its efficiency, providing the most economic mode'of further extending its operation, do most humbly submit to Your Excellency the following, our First Report :—

Before we had proceeded far with the enquiry we were commanded by Your Excellency to make, our attention was directed to the fact that the Education Department possesses an Architect’s Branch, whose duty it is to construct and repair school buildings under the superintendence and control of the Department This privilege, which no other public department of the State possesses, naturally led us to investigate the advantages or otherwise of such a system. Accordingly we somewhat interrupted the course of our inquiry in order to give Your Excellency our opinion on this important subject. Having made the fullest investigation, we have come to the conclusion to recommend Your Excellency to transfer the duties now performed by the Architect’s Branch of the Education Department to the Public Works Department, to which we think such duties more naturally appertain. The chief argument advanced in favor of the retention of the present system was that the Education Department by this means is enabled to have repairs and other necessary works effected more expeditiously than could otherwise be done. We, your Commissioners, do not think that this contention is borne out by the evidence adduced from the officers of the Department whom we have examined, whatever may have been the case when the present Act came into operation. On the other hand, we are of opinion that, if our recommendation is carried out, greater economy, without any loss of efficiency, will thereby be promoted, besides the advantages which a concentration of the work of inspection and supervision of all State buildings will undoubtedly confer.    -

If the transfer now recommended is effected, we think that the expenditure upon the construction, repairs, and supervision of school buildings should be shown separately and distinctly in the accounts of the Public Works Department. An approximate amount will effect the object in view, namely, ascertaining from year to year the total cost to the State of the Education system of the colony.

We shall proceed with due diligence to consider the other matters embraced in Your Excellency’s Commission ; and we humbly submit to Your Excellency’s consideration this, our First Report.

J. WARRINGTON ROGERS,

Chairman,

A. Grant McIntyre, Secretary, 1st May, 1882.

Precis op the Minutes of Evidence taken before the Royal Commission on Education.

The Commission is constituted as follows:—J.Warrington Rogers, Esq., Q.C., in the chair; Duncan Love, Esq., Edmund Keogh, Esq., J.P., William II. Archer, Esq., J. P., C. J. Ham, Esq., John M. Templeton, Esq., J.P., George Meares, Esq,, J.I’., Francis Ormond, Esq., II. N. Loughnan, Esq., J.P., Hon. J. Macgrcgor, J.P., W. IL Cutts, Esq., M.D., J. P., Professor McCoy, F.G.S.

The Commission met for the taking of evidence on Wednesday, February 8th, 1882, when the Secretary of the Education Department, Gilbert Wilson Brown, was sworn and examined.

la reply to questions put by the Commission, Mr. Brown said he had been Secretary to the Education Department since March, 1878. Had been away on leave since the commencement of February, 1881. Mr. Bolam, the Inspector-General, had acted for him while he was on leave.

His own salary is £800 per annum, the duties included the charge of the w'hole Department.

Mr. Brown then gave the following outline of the several branches into which the Department is divided :—

Taking the sub-branches, there is first of all the Registration Branch, in which all letters are registered, the substance of them epitomised, and by which branch they are distributed to the several other branches for which they are intended. Next the Despatch Branch, for the despatch of all letters, circulars, and so on; to keep an account of all franked envelopes ; to issue all forms to teachers, and all circulars issued by the Department. Next the Teachers’ Branch, which keeps records of the appointmentand promotion of teachers ; what we call the “records” of the teachers—that is, the abstracts from the inspectors’ reports on teachers, and all examination papers of teachersand pupil-teachers ; it also attends to the revision of the .allotments to teachers, which has to be done half-yearly ; and all the clerical work in connection with training and with exhibitions. Then we have the Books and Stores Branch, through which all requisitions for free stock to schools go, where the accounts are checked, and also through which all stationery for the department is procured. Next there is the Board of Advice Branch, for dealing with the compulsory clause; keeping the records of attendance of scholars during the several quarters and the returns of attendance at private schools; arranging fur prosecutions of parents for not sending their children to school ; carrying on all correspondence with the truant officers ; in it the business of the election of boards of advice and all changes in the boards are registered, and the correspondence is carried on by it. Next the General Correspondence Branch, in which all the outward correspondence is carried on ; and in which registers of deeds are kept, conveyances of land, and registers of all applications for new schools. Then there is the Paper-room, in which all the papers of the department are kept. The officers have to keep a daily record of all cases not in their proper places ; to sort the papers as they are finally dealt with, and put them away : to keep an index of all building cases, and applications for new schools. Those are the different branches under the supervision of the chief clerk. We have also two examiners, who sift the correspondence before it goes to the inspector-general or myself. The work of the one who deals with the inspector-general's correspondence relates to the drawing out of programmes of inspection for the inspectors ; reading and making an abstract of inspectors’ reports; generally dealing with questions as to the inspection and discipline of schools and the interpretation of rules and regulations ; drawing up time-tables for the singing and drawing masters ; advising as to carrying on schools as full-time or half-time schools. Those are the duties of what we call the inspector-general’s examiner, relating rather to the internal working of the schools. We have a second examiner, called the secretary’s examiner. His duties have reference to the establishment of new schools ; applications for removal of schools, and for additions to school buildings ; questions connected with leasing buildings for school purposes ; applications for school furniture ; and the fixing of allotments for schools. Those are the general headings, I have them more in detail in writing: In the Accountant’s Branch, the duties of the officers are to examine and pass the accounts, and to keep the statistics of attendance in all the schools. In the Architect’s Branch, the duties are to prepare plans for school buildings; and inspect them periodically, or as may be necessary from time to time. Those are generally the branches of the department.

What is the total number of officers in the department, exclusive altogether of the teaching staff?—On what we call the permanent office staff, including the secretary, chief clerk, accountant, and clerks, we have 3G ; and, in addition, two messengers, a caretaker, and a housekeeper. The temporary clerical staff includes 31, with, in addition, a messenger and three female office-cleaners. The inspectoral staff comprises the inspector-general, the assistant inspector-general, two inspectors, who act as examiners (they are the two examiners I have been speaking of—they are in rank inspectors, and are paid as inspectors) ; and 18 inspectors engaged in the work of inspection. The architect’s staff comprises 13 actually employed in l he central office, three inspectors of buildings, nine clerks of works, and one messenger. There are also 25 truant officers.

Does that include all the officers ?—Yes.

What is the salary of the chief clerk ?—£610.

What is the salary of the principal inspectors?—They range from £375 up to £610. The assistant inspector-general gets £650, and the inspectorgeneral £700.

What is the total, cost of the Department, exclusive of the teaching branch altogether?—I have a return here for 1880—that is, up to last June. The permanent office staff is £9,650 3s. lOd. ; temporary clerical assistance, £6,226 3s.; inspectors, £10,023 os.; inspectors’ travelling expenses, £3,659 12s. 8d. I have not here the cost of the architect’s staff. That is paid out of the loan ; it is not a charge upon the annual vote.

The cost of the architect’s branches is altogether outside the annual vote ?—Yes.

Do you know ho w much it is, 1880-81 ?—Yes, I have the salaries here. Those I have here only amount to £6,440, but to that there will be a considerable sum to add for travelling expenses, which I know runs it up to nearly £9,000 a year.

During 1881 what was the cost of buildings and repairs under their supervision ?—£99,761 2s. lOd,

Under whose direct control is the architect’s branch?—Under the control of Mr. Bastow, the architect of the Department.

Have all repairs to be referred to them, or are such small repairs as say £20 done without reference to the architect’s branch ?—All the repairs are referred to him. He advises what course shall be taken, whether they shall be done through him, or that it is so trifling a matter that it may be done through the teacher; but every question of repairs is referred to him.

Did not the auditors, on one occasion, suggest that there would be a great saving to the public by utilizing the Public Works Department for your work ?—Yes.

That report was in December 18S0, I think ?—I think January, 1881, was the particular month to which they took exception—it was on the returns for that month that the suggestion was made.

What is the general rule as to appointments and promotions in your department ?—Simply we have applications for employment, which are submitted to the Minister, and he selects ; and promotions are made generally by seniority as vacancies occur ; if the next man is suitable he goes up.

That is as to officers, not as to the teaching department ?—Yes, I understand that.

Have there been any exceptions as to that promotion by seniority ?— Yes, I believe there have been one or two cases in which an officer has had an increase of salary, which has had the effect of putting him over the head of somebody else.

Can you-give the instances ?—I think I know them ; but I should be sorry to make a mis-statement, I will look them up.

Can jmu make any suggestion to the Commission, as to the more economical and more efficient discharge of the duties of the department, outside the teaching altogether ?—No, I do not think that the department can be more economically worked.

Have you read Mr. Pearson’s report?—Yes.

Do you remember this remark in it ?—“ The department has never received any proper organization, and, as its field of exertion widens, will soon break down altogether, if not remodelled” ?—Yes, I remember that.

Is that your view as to the state of the department ?—No. I do not know the special facts on which that opinion was formed.

What is the total cost of the department, including the teaching power, for 1881 ? The amount expended during the year 1880-1 (the twelve months ending June, 1881) was £632,044 8s. lOd. That includes everything—the whole expenditure of the department.

Does that include the buildings for the year?—Yes.

Is there any other reason than that of expedition, which you have given, for having a separate architect’s branch in your department apart from the Public Works Department?—No. Perhaps I may be allowed to say I question whether it would be done more cheaply elsewhere. One of the Commissioners remarked that 10 per cent, was double the ordinary charge. 1 am informed that the ordinary architect’s commission is 5 per cent., and beyond that he charges 1 per cent, for the plans, and beyond that again there would be expenses of clerks of works. The charge that I quoted includes everything.

Was not the establishment of a separate architect’s department in connection with the Education Department originally done with a view to control the plan of the buildings, from a professional point of view, buildings to be suitable for school purposes ?—No. I know, as a matter of fact, though I cannot show it in black and white, that in the first instance Mr. Stephen, who was the first Minister of Education, made enquiries a^to the cost of erecting a certain building—what it could be erected for by the Public Works Department, and he got the information, and the estimate was so high, as compared with what we, under the old Educational Board, had been in the habit of paying for school buildings, that he decided at once to have a branch of his own. That was the origin of the architect’s staff in our department.

Why should there be separate persons to look after repairs where there is a post-office and court house, and so on, and all under the care of Government, all repaired at the cost of Government ; is there any reason for that. What economy is there in that—might not the same person look after the repairs of all ?—Undoubtedly.

Professor Pearson suggested that the board of advice should have authority to expend, say £20—would not that save travelling inspectors going where the cost of travelling is more than the expense of the work? --1 think there is one point I ought to call attention to. We have a staff that is larger than is necessary for the expenditure of this £99,000. The same staff could supervise the expenditure of a very much larger sum of money, and the department was formed in order to spend at a much larger rate than that.

Then the staff had not been reduced concurrently with the expenditure?—No. There was £237,000 of work in 1877-8, supervised by the staff. The staff was formed, and it could do the work if it could get the money, and it has not been reduced in view of its still being necessary to put up the required buildings.

What was the expenditure in 1880 ?—I have the amount expended on buildings and repairs each year since the department was formed. In 1872-3, it was £6,483 6s.; 1873-4, £148.614; 1874-5, £181,439 ; 1875-6, £114,000; 1876-7, £162,874 ; 1877-8, £237,203 ; 1878-9, £89,252. The next year is the first one in which I [.can give the repairs and buildings separately. In 1879-8, repairs £10,000—wc spent just the ]vote, buildings £66,085 ; 1880-1, repairs £14,929, buildings £84,980.

What do you estimate for 1881-2?—We have £80,000 for buildings and £20,000 for maintenance repairs. The architect’s branch costs about £9,000.

Is it left to the teachers to report on the state of the buildings, or is it the duty of the architect’s branch to report on them from time to time?

—The inspectors are instructed to visit buildings as they can in their travels; their work is continually calling them to distant places, and the inspector or one of his clerks of works is sent off, and he sees the buildings he can cn route without unnecessary delaying.

Is it on the report of the teacher ?—The teacher, or the school inspector, or the board of advice.

Suppose a roof is leaking at Warrnambool, how do you get information on that and proceed to have it repaired ?—The teacher would report it, and the architect would direct his building inspector in the Western district to visit if he could, or instruct some tradesman there in whom he has confidence to repair the leak.

There are a series of inspectors throughout the country, then ?—Yes, our building inspectors are stationed at different parts—Sandhurst, Ballarat, Warrnambool, Geelong ; aud clerks of works are stationed at the several districts, one in the Ovens district, for instance.

How many of those inspectors are there throughout the country ?— Twelve in all : three building inspectors and nine clerks of works. We call them clerks of works, but they inspect buildings.

Suppose the roof is blown off ?—The inspector would visit in that case. •

Where there would be injury, if the repair was not effected at once, to preserve the building?—An inspector would be sent at once, generally.

Then the board of advice could not expend a pound on it? No, we would do that through the teacher.

But he could not do it without authority?—No, but every now and then, where it is an urgent thing, the teacher does expend money, and it is recouped to him afterwards.

Are the architects in your department permanently employed ?—Yes.

You said, if the Public Works Department carried out the same work that you do, they would require the same staff ?—I think so. I know our men are very heavily worked ; all the architect’s staff.

In the different departments of the Education Department there are many not permanently employed?—All those I have mentioned are permanently employed, but they are called temporary clerks; but, as a matter of fact they have been employed for years.

How do you pay the local building inspectors ?—The three we call inspectors of buildings get £400 a year each, permanent salary.

Where are they stationed ?—One in Melbourne, one at Sandhurst, and one at Ballarat.

How are the others paid ?—Five are paid £4 10s. a week, and four are paid £5 a week.

Then are the architect’s department all permanently employed?— Yes.

With reference to the records—I know you have very full records in your department—will you tell the Commission how they are classified. For instance, you have a record of the teachers—their character, promotion, and so on ; a record of all correspondence with each school; a record as to the children, and so on ?—The first record is the “inward ” correspondence. Each school has a number, and the whole of the letters received with regard to any school are registered under that particular number. So each school has an “ outward ” letter-book, in which is entered every letter written to that particular school. Each school has its own book, and each school is known by its particular number. As regards the teachers also, each teacher and pupil-teacher and workmistress has a number and a corresponding folio, in which are entered extracts from the reports of the inspectors, as they are received, as to the ability of the teacher ; also full entries of their classification and their promotion from one school to another, aud any censure that may be passed upon them. The records as regards the children are the reports of the inspectors upon the schools as they are examined. The inspectors examine the children individually. All those reports are filed in numerical order, so that if we want to trace a child at any school, the report can be seen of how he acquitted himself at any examination.

Might there not be a statute of limitations, as it were, in regard to destroying these records ?—We have never experienced any inconvenience in keeping them.

You have a great accumulation in ten years?—Yes.

In another ten you will hardly be able to keep them ?—As a matter of fact, I have found it necessary to ask for papers fifteen, eighteen, and twenty years back —under the old Boards—-questions about teachers; and I have found it a great convenience to refer to them. Questions arise about teachers in which it. is most desirable to do so. I have most persistently opposed the destroying of the papers. The question has been raised in the department, and I have opposed it, because I have found the advantage of having them.

I wish to ask now about the school children and the schools generally. What age does the department treat as the school age ?—We record their attendance at three years old. We have no superior limit at all.

You know, of course, that the Statute is distinct as to the ages of children, that they must be taught between six and fifteen ?—Yes.

Have you noticed the discrepancy between the schedule of the Amending Act and the clauses of the Act?—You pointed it out to me.

In the schedule of the Amending Act the age with regard to which the returns are to be made is between three and fifteen, but in the enacting portions of the original and Amending Act the school age for returns is between six and fifteen ?—Yes.

So that there is an inconsistency between the Statute and the schedule ? —Yes,I did regard that as intentional; that the object of making the age three in the schedule was that we might not only know what children are at the present time of school age, but also what children are coming on, in order to look them up when they become of sufficient age.

But if you look at the section referring to the schedule, you will see that the section refers to the schedule for the purposes of prosecution, the school age in the clause itself being between six and fifteen ? —Y'es.

Were your circulars prepared with reference to the schedule or the enacting part of the Act requiring returns to be made by the parents? —The circulars were made in accordance with the schedule.

As a matter of fact, is the school census taken in reference to three and fifteen or six and fifteen ?—The school census is taken with reference to the ages three and fifteen.

Have you a complete reliable census, prepared upon any fixed basis, in the department?—No, not at all reliable. We have compiled a census on two occasions, but on neither has it been satisfactory ; that is to say, we have known for a fact that we did uot get a full return of all the children. We had more children on our school returns than the census returned.

How much did that census cost ?—I think about £3,000 it generally costs us.

Upon each occasion ?—lres.

And it has been done twice ?—Yes.

In Mr. Pearson’s report (page 7), he says—(i At present the whole system is confused and disorderly”—that is, as to the taking of the census. And at page 34 he says—“ There exist no returns at present which would give the information for the colony.” Upon what principle have you proceeded to make the school census?—In the first instance, we apply to the shire councils and boroughs and road boards, asking them to put at our service their officers to compile this census, and to advise what remuneration they would do it for. They have generally assisted us as far as they could, and where we have not been able to get the assistance of shire officers, we have advertised for tenders for taking the census. We have not accepted the lowest tender, but each tenderer has had to submit good testimonials, and we have been guided by his testimonials, without regard to the fact of his beiug the lowest or not. And then before paying, wherever I had reason to suppose that I had not got a fair return of the district, we have required the collector to make a statutory declaration as to his having called at every house in the district, left a census paper, and called again and had it returned.

Is this correct, as a matter of fact, which is stated in Mr. Pearson’s report, that the departmental report every year shows more children of school age on the rolls than are alive in the country ?—Possibly some of our figures may. We give two sets of figures. We give the gross total on the roll for the whole year, and we then eliminate from those a number in allowance for children who have attended at more than one school. If you take the highest number, which we admit does contain more than are in our schools, it is possible it may be more than those living in the colony, but we base nothing on that.

That makes the census unreliable ?—The census is unreliable ; it does not show the number of children that we have actually attending.

If the law were altered, could not the census be taken without that additional expense. For instance, in the shires, boroughs, towns, and cities, certain persons are sent round in reference to the assessment of houses. They first leave a notice, and then about a fortnight afterwards call for it—that is with reference to the rates. Now, if upon the paper left in the first instance there was a notice under the Statute that a return was to be made upon the stamped paper and that the same person would call for it when demanding the rates, no additional labour would be cast upon the officers of the municipalities, and the whole thing could be done almost gratuitously—there would be no additional labor, would there ?—No.

I am assuming that the law threw this duty upon the municipalities ? —That would apply to nearly the whole of the colony. There are a few small portions outside the municipalities.

Can the number upon the roll be relied upon as being correct ?—I consider that the reduced number which we give in our returns can always be relied on. We give here—[referring to report]—the gross total on the roll of our schools during 1S80, 229,000; but then we make considerable reduction on that for children who have attended more than one school —we reduce that to 187,000.

On what principal do you go in making the reduction to the smaller figures?—We get from each teacher in the month of December every year a statement of the number of schools, if more than one, that each child has attended during the year. Of every child present the teacher makes the enquiry as to how many schools he has attended. Sometimes it is two, some three, and some four. We get that from every school, and make the reduction.

Would it be possible for a child to appear on the roll of two State schools at the same time ?—Yes, he may do so.

Do I understand that if a child has attended any school during any portion of the year, however small, he is considered to be on the roll of that school for the year ?—Yes.

Do you think you have a complete record of all the instances in which the same child has attended different schools in the same year ?—1 think so. We have the actual figures of the children attending each quarter from the teachers, and they would seem to confirm the other figures. In those cases we have the names and attendances of each child.

What is the number which is ascertained and not estimated ?—For each quarter I have the number ascertained.

Take the last quarter of 1880 ?—170,740. The December quarter is always a bad quarter.

The Michaelmas quarter ?—174,054.

Midwinter quarter?—174,080.

March quarter?—171,150.

Can you give the ages?—I can give the ages for all the children who attended during the year 1880. This is a return—[exhibiting apaper~\

night schools also:—

Summary.—Ages and Sexes, 1880—Night Schools.

Ages.

Boys.

Girls.

T otal.

Percentage.

12 to 13 ... .

726

228

954

9-74

13 to 14 ......

1,421

477

1.898

19-37

14 to 15 ......

1.676

577

2,253

22-90

15 to 16 ......

1,574

383

1,957

19-97

16 and above ...

2,345

392

2,737

27-93

7,742

2,057

9,799

100-00


SO


—made by the teachers of the schools. I have them at each age, and some under three years who ought not to be there.

We will take them as you have got them ?—

Summary.—Ages and Sexes, 1880—Day Schools.

Ages.

Boys.

Girls.

Total.

Percentage.

Under 3 ......

34

43

77

•03

3 to 4 ......

1,903

1,904

3.807

1-76

4 to 5 ......

5.402

5,131

10,533

4-87

5 to 6 ......

8,017

7,326

15,373

7 TO

6 to 7 ......

9,598

8,902

18.500

8'55

7 to 8 ......

10.203

9,871

20,074

9-28

8 to 9 ......

10,629

10,141

20,770

9-60

9 to 10 ......

11,183

10,618

21.801

10-08

#10 to 11 .....

11,152

10.S09

21.961

10*35

11 to 12 ......

10,702

10,389

21,091

9'75

12 to 13 ......

10,381

10,147

20.528

9-49

13 to 14 ......

8,784

8.803

17,587

8T3

14 to 15 ......

6,329

6.704

13,033

6-02

15 to 16 ......

3,620

3.947

7,567

3-50

16 and above ...

1,540

2,119

3,659

1-69

109,507

106,854

216,361

lOO'OO

That is the percentage on the total attendance on the return of 216,361 children. That does not include night schools. I have a return for


Can you state what places have night schools ?—Yes, I have a list of them here.—[.The witness read the same, ss follows] :—

T3


O


3 S A? -3

•st: c a

ci ci cj ci OOcoO


TJ

O t» 03

— — o r-.OM


CO I'!>. t>> 6) 03 r-i O ^ CO


PQ


O ¡>2


O o -O


! ¿3 oJ

bo ri .


o

~ a

03 ' CO CQ

£ r-T

O rc3


o


O K i-1 O W P3 ix >-4 O


<M r~- (N O f- O r—I to    r-i

>—l O >0 CO CS t- O CO 1-^ 1C

HONcoV'iioeoao


Can you tell me how many children there were under six years of age—the actual number?—25,776 distinct children.

That leaves about 4,000 under four years of age ?—Yes.    _

How many children are there above the age of fifteen that is above the statutory age ?—11,226.

How many above twelve years of age, that is between twelve and fifteen ?—51,148,

If we add that number to the number above fifteen, that will give the number above twelve altogether? —Yes.

How many does that make above twelve altogether ?—62,374,

Can you give the average age at which the pupils obtain the required staudard ?—We have not this calculated ; but I have had a hundred schools taken out indiscriminately, town and country, and made a calculation for this hundred. The average age of 691 boys in those schools who passed the standard was twelve years and nine months, and 621 girls passed at the average age of twelve years and eleven months. ^

I suppose in town schools they attain the standard earlier than in the country schools ?—Yes, decidedly. I have also the ages at which the different numbers attained this standard. Nine and under ten, 4 boys and four girls ; ten to eleven. 42 boys and 30 girls ; eleven to twelve, 139 boys, 100 girls; twelve to thirteen, 196 boys and 168 girls ; thirteen to fourteen, 185 boys and 197 girls ; fourteen to fifteen, 99 boys and 95 girls ; fifteen to sixteen, 21 boys and 23 girls ; sixteen to seventeen, 5 boys and 3 girls ; seventeen to eighteen, 1 girl.

Of the total number of those attending, what was the proportion of the boys to the girls ?—In day schools, boys to girls are in the proportion of 100 to 97'43 ; in night schools, the boys 100 to 25 T2.

Have you any record of thcreligious denominations of either the parents or the children ?—None whatever.

Can you obtain that at all—have you any means under your control for doing so?—There is nothing in our existing records that would give it. The teachers know nothing about it, and we know nothing.

About the ages you have records in your department of all the children taught at each school age, have you not ?—Yes.

Have you at the present moment, or have you ever had, a similar account of the school population out of doors at those ages, or the population rather that ought to be at school at each school age ?—No.

So that you do not know, and never have known, how far your department has overtaken the population ?—No ; we do not know that. Our census ought to show that, and we have never known that.

And do not know at this minute ?—No, we do not.

What are the number of pupils learning extra subjects ?—I only have the returns up to the end of 1880.

The number learning each subject will do?—In 182 schools instruction is given in extra subjects.

In those only ?—Yes. In those there were 1,128 children learning Latin; 858 French; 111 German; 155 History; 990 Euclid; 1,298 Algebra; 123 Mensuration; 1,772 Bookkeeping; 150 Chemistry; 9 Mathematical and Physical Geography ; 1 Trigonometry ; 1 surveying ; 60 Botany ; and 3 Shorthand.

; a

_ PS H PP ¡>>§,£3 <U[X) a) m 2 n -tS ,; 2    ^ r

4_T    t-. « 44 H <u-rfN 03 O

pqpp!LC3Ccpqpqpqw>Hi2;<fopiP£lpqo coHcoOMmojfoo'UooNNNiMroH««

CO H IO 10 to -< raOOOOOOClOHtSOq h (M q 10 01 f-l n q -f -f q >0 10 co D o - h

What subjects are taught ?—The ordinary school subjects.

Are those night schools open every night?—No, three times a week.

Have you had opportunities of forming an opinion as to the moral effect of those night schools, first where the sexes are kept apart, and also the effect of the mixed schools, from any information that comes to you ?

_No ; we have had two or three representations from Boards of Advice

that it was undesirable to have mixed night schools, and in towns, where we have been able, we have separated them.

Have any of your pupil-teachers been educated at your night-schools ? —No, they would all be smart scholars in the day schools.

You do not recognise night schools practically—do you examine?—Part of the payment depends on their examination.

Suppose a child was being educated at a night school, it would exempt him from attendance at the State school ?—If he passed the examination. If a child went to be examined for the standard at the night school, ho might; but generally they are beyond the school age.

At what age does it exempt them from attending day schools ?—We do not admit pupils into night schools under twelve.

Does the attendance at the night schools exempt from attendance at the day school ?—No, we do not recognise it that way at all.    _

Have you had any expression of opinion in the reports of inspectors with reference to night schools ?—Yes.

In which way, favourable or unfavourable ?—I think they were about equally divided when I had them last abstracted, A great many object, but a great many thought they were doing good work.

Do people of influence, in the neighbourhood visit night sehooiS— gentlemen with the commission of the peace, or ladies?—No, I have not heard that they do.

Could you supply a return of children entering the State school at the age of six as to what percentage failed to pass at the age of thirteen ?— I am afraid we could not get at that. I am afraid we could not trace them sufficiently for that.

Those examinations for the statutory standard are really made at the request of the pupils ?—No, we instruct inspectors every time they inspect the school to offer facilities to teachers to get standard certificates for any scholars who are desirous of getting them. Of course where they are making their ordinary examination of the school, the standard certificate work forms part of the other. It works into or crops out of the other,    .    _    ,

They give facilities, but they do not actually require a child to bo examined 1—That is it exactly.

You stated that the Department had received reports, as I understood, from outsiders as to the undesirability of night schools ; would you kindly state the grounds on which that undesirability was suggested^?—Excuse me, it was the undesirability of having mixed night schools. The letters I have in mind are some letters from Boards of Advice suggesting that we should have separate night schools for boys and girls.

On what ground ?—On the ground of morality.

Did they give any instances of immorality ?—Only general statements ; not any specific ones.

Sufficient to convince you?—Yes, we quite concurred in their view.

Is there any record in those night schools of the visitors ?—Provision is made on our roll-sheets for members of the boards of advice to make any remarks they like, but it has never come to my knowledge that any entry has been made in the case of night schools.

What are the hours ?—They require to be open two hours on three days a week, and the usual hours are seven to nine, but there are no compulsory hours, it rests with the teacher.

Might it be later than nine ?—It might, but I do not know of any case.

It might be up to any hour?—Practically it is nine—seven to nine, I think, are the universal hours.

Have any complaints been made to the department with regard to girls’ night schools?—I am not aware of any, I cannot call any to my recollection.

Have any of the inspectors given any special cases of immorality at the night schools ?—I am not aware. I will make a note and make inquiries, but at the present moment I am not aware that they have.

Could you tell the number and general character of ragged schools ?— The ragged schools are not at all connected with our department. The only information I can give is the return we receive at the end of the year. Under the last clause of the Amending Act, every school has to make a return to us of the children attending during the year.

What return have you for ragged schools ?—I have only returns for three ragged-schools. One in William-street, Melbourne, teacher, Miss Legg ; 18 boys and 20 girls attended during the year. Little Lonsdale-street, Mrs. McCormick ; 53 boys and 34 girls, llokeby-street, Colling-wood, Miss Carter, teacher ; 48 boys and 38 girls. I know nothing of any ragged schools beyond those, and I can get no statistics anywhere about them. Are the children attending those schools excluded from the State schools ?—No.

If the compulsory sections of the Act were thoroughly enforced, could there be any excuse for the existence of either night or ragged schools ?— No ; children should be able to pass the standard, and so render it unnecessary to attend night schools.

If the Act was thoroughly carried out there could be no reason for the existence of night or ragged schools ?—We have some beyond fifteen, who attend night schools, who would be exempt from compulsion to attend day schools.

You do not know how many children of school age there are in the colony ?—No.

Have you read Mr. Pearson’s remark at page 9 of his report, speaking of “ gutter children,” “ larrikins,” and so on ;—“Hitherto the practice of the department has been to abstain from sweeping these children into our schools, lest they should impair their tone ; or to provide for them in some school of a special character.” Is there a practice in the department to recognise the exclusion of children on account of the want of clothing, boots, and so on ?—We only insist on cleanliness, nothing more. The following are the only grounds upon which authority has been given to exclude children from schools—[reading from circular to teachersissued in February 1873].—“You will therefore understand that immoral conduct or language on the part of any scholar, if other means of correction fail, will justify you in excluding him from your school ; that any child affected with infectious desease should be excluded till all danger of infection is removed ; that personal cleanliness should be made a condition of admission into your school, and that appliances for washing should be provided, and their use enforced.”

If a schoolmaster should refuse admission to a child on account of being shoeless or ragged, would not that be a ground of complaint against him in the department, as a breach of duty on his part?—If the child were shoeless, we would not regard that as aground for exclusion ; but if his clothing were ragged, we would insist upon its being mended ; we would not object to the clothes being old, but he must be tidy. If a complaint were made against a teacher that he had excluded a child on those grounds, we would not uphold him—that is, simply because a child could not afford boots or shoes.

Was there ever a complaint or a case of exclusion under such circumstances ?—I think there was one case. I will make a note and try and find it.

In your opinion, is there a sufficient number of truant inspectors to carry out the compulsory clauses efficiently ?—No, we ought to have many more to do it efficiently.

How many ?—I think we might double them.

Have truant inspectors ever complained to the department that the teachers refuse children and the parents have complained of such refusal ?—No, I am not aware of any complaints of that kind.

I may mention that I have had cases up before the bench, where the parents have sent the children, and the teachers have told the parents the school was full ?—That may be correct.

And then they have brought them up and fined them for not going?— We get a very full report from every truant inspector in every case before prosecuting, and the truant inspector would not prosecute in such a case as that, he knowing the neighbourhood and the school and all the circumstances.

Have you any record of the occupation, the trade, the property, or the amount of rental or rating of the parents who send children to the State schools?—We have nothing about the rating or rental, we have their address. We have not their occupation.

1 am asking you that principally fox this—have you any means of ascertaining to what extent persons in easy circumstances, who could educate their children at their own expense, send their children to State schools?—No, we can only judge by observation of the children themselves.

From your observation to what'extent would you say that the State schools are availed of by persons who could not afford to pay ?—I would say to a very large extent.

I think it is important to give the relative proportion ?—I am afraid we have no means of obtaining that.

Still your impression is that the schools are very largely availed of by persons in good circumstances?—There is no question about that.

You spoke of small and inefficient schools having been done away with —have not there also been efficient schools done away! with?—lres, some. I said inefficient, because a great many of the teachers of those private schools came to us to be employed as teachers, and failed to pass— a considerable portion of them could not pass our examinations.

Will you tell the Commission how the truant officers are supervised ?— They send in weekly diaries of their work, and if it does not show a full amount of work they are taken to task for it.

They are supervised by the department generally?—Yes, from the office.

Does the teaching by the teacher of a number of extrasubjects militate against their teaching the ordinary statutory subject-; ?—No. Those have to be taught outside the four hours that the Act requires to be devoted to schedule subjects. The Act provides that the children shall receive four hours’ instruction.

If those men are teaching German and bookkeeping, is not their attention drawn off the general school ?—They must give the four hours the Act requires. Those extra subjects arc taught altogether outside the school hours.

Do you think it at all impairs their efficiency ?—Not at all. It is generally done by the assistants. The head-master has enough to do with his ordinary work, and any benefit of the extra teaching is generally understood to go to the assistant teachers.

Does the teaching of the extra subjects have a tendency to influence the teacher to collect the better class of scholars to the exclusion of the poorer ?—I do not think the teachers show any great interest in teaching the extra subjects at all. I do not think it pays sufficiently well. As a rule they do not care about it sufficiently.

Are there any fixed rates for pay ?—Yes. The fee for any subject must not exceed one shilling a week. There is no minimum, but the maximum must not exceed one shilling a week.

Can you tell us how many have taken out the certificates of efficiency spoken of in the Statute?—That is what I propose to get for you.

Is there not great indifference that the large numbers of those that attain the standard neglect to take out the certificate ?—Yes.

Can you at all account for the indifference there is about those certificates. Of course, there is a great desire to get a certificate for the civil service or matriculation ; but there seems to be an indifference about the certificate of this statutory standard ?—I think the certificate is generally only sought by children who want to get exemption from further attendance. If they are going on with attendance they do not trouble about it.

They can continue after they have got it ?—Certainly.

After they attain the standard provided by the Legislature, they can still receive a gratuitous education ?—Certainly. That question has once or twice been raised. Masters have been under the impression that they have not to teach such children any more, but it has been overruled at once.

Do many attend ?—All our fifth and sixth class children should be able to pass the standard. A very large proportion who have passed sufficiently for the standard go on,

Then a large number are being educated gratuitously who have long since attained the standard of efficiency ?—Yes.

Beyond fifteen ?—They may be beyond fifteen.

Have you any means of knowing the number of children who have ceased to attend without attaining the standard ?—Who leave school finally without attaining it ?

Yes?—No, we have not.

Then you have no means of knowing how far those children attending the schools have obtained the benefit oE the Act ?—A great many stay up to fifteen, and will not have attained the standard. They will then be exempt from attending in consequence of their age.

Suppose they leave before the age of fifteen ?—Then we look them up through our truant officers.

If they have not obtained the certificate then, and if they do not attend ?—Then we prosecute the parents.

They may have attained the standard and you have not examined them, so they may have left without your knowing whether they have attained it ?—Yes.

In your opinion how many attend the State schools that have obtained a certificate ?—I cannot give that. I can only find out the number of certificates of standard that we have issued.

The children are still going on ?—A great many. We may assume that all the children in the oth and Gth class have attained the standard.

Have you no report of the number of children who are entitled to certificates if they choose to apply for them ?—No, we have not.

Do you look upon this standard of education as the ultimate aim of the Education Department?—We fight against that idea altogether, The parents too often regard that as all that has to be done, and the Legislature considers it sufficient, but we do not.

What does the department consider sufficient ?—We provide a programme of six classes, and the standard work is all included in the fourth class.

The Legislature has one standard and the department another ?—I

think the standard fixed by the Legislature is only to exempt the children from further attendance—that that is the minimum of education ; it is not intended that they should not go further. Our programme provides for six classes, whereas the standard work is really comprised in the work of the fourth class. We have two classes above that that we encourage them to go on with.

How many children who have attained the standard are still attending the State schools?—Unfortunately I am unable to answer that.

If a child be sent to school at six years of age and he attend regularly, a child of fair average ability, at what age ought that child to attain the statutory standard do you think, from your own observation ?—I do not know that we have any data to go on except what we have here that the standard work is really the work taught in our fourth class.

Then put it in another way : a child going to school at six years of a°‘c, and entering the first class, should, at what age, attain to the fourth cfass i—We assume the average age of the fourth class to be eleven and a half years.

Then you think the standard should be attained at eleven and a half years ?—No, hardly ; because he has still to work on in that class till he gets to the next class at about thirteen. I think between twelve and thirteen is about the average.

If sent at an earlier age than six, would you say the standard should be attained at an earlier age, or at the same ?—Perhaps a little earlier ; but not much.    %

The only difference would be,' the State would pay more for the child? —Yes.

By the statutory standard they ought to be well grounded in reading, writing, and arithmetic by twelve years of age?—Yes.

You would say, supposing that every child in the community of average ability commenced at six, he ought to be well grounded, as the subjects are now taught at the schools, in the three “It’s” by twelve years of age ?—Yes, from twelve to thirteen.

That is, six or seven years’ teaching ?—Yes, I think so.

As a matter of fact, do many children commence at six.?—No, I think a very small proportion enter as late as six.

Supposing a teacher has a child of average ability where he goes in at five, six, or seven, will he be well grounded in the course of seven years after he enters?—I think a child entering at six ought to be able to pass our standard between twelve and thirteen.

At thirteen at the outside ?—Yes.

If efficiency could be attained at twelve instead of fifteen, I suppose the cost to the State would be reduced by the years between twelve and fifteen?—If the State says that children shall not be kept at school after they have attained the standard ; but, at present, we carry them on still, and encourage them to stay on.

The State only requires, at present, that they shall attain a certain statutory standard ?—Yes.

And inasmuch as it compels that, it pays for that standard ; so that if a child takes from six to fifteen instead of from six to twelve to attain that the Slate is at more expense ?—You see the statutory standard only requires the statutory proficiency in reading, writing, and arithmetic ; but the schedule requires grammar, geography, sewing, and so on.

But a child having attained to the reading, writing, and arithmetic standard, is entitled to be discharged from attendance ?—Yes.

Diet orhm (Eimtafton gcjiarfmcnt.

APPOINTMENTS.

Thomas Gray, 2nd Asst., Mount Pleasant, 1436; Jane Trathan, H.T., Moore’s Flat, 1575; W. II. Bourke, H.T., Kalkallo, 195; Catherine Mitchell,

H.T., Pine Lodge East, 24G6 ; Daniel Kennedy, H.T., Wurdi Boluc, 1556 ; Henry Derrick, H.T., Tallangatta Creek and Tallangatta Creek North (half-time), 2337 ; Phccbe Ellis, H.T., Lake Elizabeth, 2461 ; William Mclver, UpperlBoggy Creek, 2459 ; A, W. Haslam, H.T., Doogalook, 2467; J. M. Tweedie, II.T., Kyabram East, 2291; Edward Richards, H.T., Muntham and Merino Downs, 2112 ; William Stewart, H.T., Warrong and Clonleigli, 1937; Edward C. Perrin, II.T.. Teesdale, 2065 ; Frederick C. Green, H.T., Mount Eccles, 1246; John B. Winnicott, H.T., Moyhu South, 2464; Lawrence S. Stevens, H.T., Won Wron, 1957; Samuel Bolitho, H.T., Wakiti Creek and Echuca Vineyard (half-time), 2298 ; Elizabeth Cade, H.T., Lower Moira,—; Hannah F. Plues, 2nd Asst., Hoathcote, 300; Charles Lightfoot, H.T., Victoria Valley and Mirranatwa (half-time) 1209’; Thos. Bedford, H.T., Torrumbarry North, 1738.

FRIENDLY ADVICE TO PUPIL TEACHERS.

By Mrs. W. T. Greenup.

Kindness—How much kindness does to make life pleasant, and by no one is it more fully appreciated than by a little child. Children are so sensitive and tender that they naturally expect to be met with tenderness on our part, and are swift to detect a repulsive and overbearing disposition in adults. Kindness should be a ruling element in school life. If you are kind in your manner and just in your dealings with them, the children you have charge of will soon learn to love you, and will put forth all their childish efforts to please you. Thus you will add much pleasure to your own work as well as theirs. By kindness you will be able to conquer many a stubborn disposition, and, at the same time, to encourage a timid one, as well as show your appreciation of a willing one,

“ A word will fill the little heart With pleasure and with pride ;

It is a harsh, a cruel thing,

That such can be denied. .    .    .

“ No ; only taught by love to love. Seems childhood’s natural task ; Affection, gentleness, and hope Are all its brief years ask.”

Let the children see that you sympathise with them in all their little personal affairs—even from your admiration of a new hat, book, or toy which “ grandmother has brought them,” to your expression of regret on beholding a plastered head or finger, and your patient attention to their accounts of recent pain and suffering during absence. How often a little child will come and take hold of your hand or dress. Don’t shake him off, but give his little hand a squeeze, and say a kind word to him. How often, too, will a child bring you a flower. Don’t put it aside, be it ever so common or faded, but wear it somewhere about your dress where it may been seen all morning. This simple appreciation of his kindness will make him value yours all the more. Be careful always to encourage kindness to each other amongst the children, not passing unnoticed any unkind action on the part of one towards another.    •

Cheerfulness.—This should enter very largely into the atmosphere of school life. It is sad to think many of the homes the children in our schools are anything but cheerful. I have often seen faces look sad when it has been time to go home. They have lingered about the school as if they were happier there than at home. What a boon it must be to such as these to have one cheerful spot in the world. There are others who have such cheerful happy homes that they are almost unwilling to leave them with the idea of being under “stern rule” in school for several hours. But if school is made cheerful and happy, these too will be glad to come. Cheerfulness in school depends on its teachers. If the teachers and pupil-teachers are dull, cross, and sternly exacting in their manner, cheerfulness is utterly impossible. Try to cultivate a lively disposition, and infuse it into all your words and actions. Enterinto thefeelings of the children which are so naturally lively; and though you are a ‘teacher now,’ don’t forget that you were once a little child. Don’t think it beneath your dignity to be provoked to a smile by their funny sayings or doings, and don’t be short of something to say that shall restore a lively atmosphere when they have become rather tired. Don’t think it a sin for them to laugh, and you with them, just for a moment, so long as you have sufficient command over them that they know it must be “ thus far and no farther.” Let all work be made cheerful, and you will find the children will do it cheerfully.

Conscientiousness.—Discharge all your duties conscientiously. Too much will not be imposed upon, nor too much expected of you ; but whatever you may have to do, you will be expected to do faithfully, a id to the best of your ability. There are some pupil-teachers who are very fond of taking their classes into a class-room; otheis whom the headteacher is pleased to send with their classes into a class-room, knowing that, while there, they are to be trusted, and that they will work as hard and as truly do their duty as if they were under immediate supervision. What a pleasure it is to have pupil-teachers answering to the latter description, and how happy and comfortable they must themselves feel to know that they are so implicitly trusted ; There is no confused look either in their face or manner when the head teacher goes in unexpectedly ; but there they are, fully occupied, their children all at work and in good order, looking as happy as doves and as busy as bees, Bucli pupil-teachers as these are sure to get on in their profession ; for where is the master or mistress who would not do all in their power to promote their welfare ? Of the former description nothing need be said, except that they are the greatest trouble and hindrance that a head teacher can have in school. Make up your minds to belong to the conscientious class, and you are sure to win the esteem of those in authority over you, and to succeed in doing well in your future career.

Self-Respect.—By self-respect is not implied self-satisfaction or pride—very far from it. It implies having such a respect for yourself as will enable you to be a pattern to those you have to teach—a regard for yourself in your position that shall help you to conduct yourself with lady-like or gentlemanly deportment; to be always the essence of cleanliness and neatness in your habits and dress ; and above doing anything that might lower you in the estimation of your scholars or their parents, such as being bribed into favouritism by presents of trifling value. If you have sufficient regard or respect for yourself, you will be able to rise above anything mean. Try, therefore, to cultivate a self-respect that shall make you genuine, well-behaved, clean, neatly dressed, and above doing anything of which you would afterwards feel ashamed.

ririEACHER, Gippsland, allotment under 20, post office attached, six miles JL from railway station, would exchange for higher allotment, near Melbourne, Geelong, or Ballarat. Apply, “It. G.,” Schoolmaster Office.

HEAD TEACHER, 30 x 50 (safe), Colac District, would exchange . similar school in Geelong District. Address “Exchange,” Geelong P,Q.________________

rrEAD TEACHER wishes exchange, allotment 20 to 30, good, results 75-

_J_L. Situated on bay, steamer daily to and from Melbourne; in Geelong

District. Address “ J. 6., 264 Nicholson-street, Fitzroy.

DRAWING—MISS TRICKEIT, holding Certificate under the Education Depar.ment, and Teacher of Drawing in the Training Institution, gives instruction in Preparation for the Departmental Examination. Ice for instruction by correspondence Thirty Shillings per term of twelve weeks, 1 Lansdowne-street, East Melbourne.

Any SIZE, weight, or shape.


rpATE’S PARCELS POST EXPRESS

FIXED PRICE.

NO EXTRAS. NO

TROUBLE.

Delivery to door at any

21b

4ib

Clb.

101b.

201b.

address in

S. (1.

s.

d.

s.

d-

S.

d;

S

d.

Great Britain ...

4 6

6

0

7

6

10

0 14

0

Continent of Europe, America, & Canada

7 6

9

6

12

0

16

0

21

0

Sydney, Hobart,

Launceston

2 6

3

0

3

6

4

0

6

0

New Zealand Ports (except West Coast)

Adelaide, Brisbane.

4 0

4

6

5

0

5

6

7

6

No further charge whatever. Very small increase for heavier weights. Delivery at country addresses in Australia, inland carriago only added.

Receiving office—

FREDERICK TATE, 13 Market-st., Melbourne TATE’S f Alt CE L S P 0 8 T EXP RE S S.

DELIVERY at DOOR any address in the world.

EITHER to or from Britain from 4s.    6d.

TO or from other Australian ports from 2s. 6d. NO further CHARGE whatever. No trouble.

EVKKY possible ASSISTANCE afforded. IN QUI RIES plainly answered.

Henceforth Parcels handed to VV. R. SUTTON’S Branches, every town in Great Britain, delivered in Melbourne at nearly similar rates.

Head Office :—

FREDERICK TATE, CUSTOMS AGENT, &c., _ 13 Market-street, Melbourne.

^LEX. M‘ KIN LEY & CO.,

GENERAL

PRINTERS AND PUBLISHERS, G1 Queen-street, Melbourne.

In Bookwork and General Publishing our large experience is a guarantee of all work being executed in the best stylo, while having a first-class stock of the best and latest material ensures expedition and good workmanship.

The following papers are issued from this office :— WEEKLY.

“Punch,”    “Bulletin,”

“ Once a Week.”

FORTNIGHTLY.

“ Australian Law Times,” “Jewish Herald,” MONTHLY.

Schoolmaster.’’    “ Monthly Messenger.”

“ Faithful Words,”

ITTEAD TEACHER, 15 miles from Melbourne.

J__L allotment 30 x 50, safe, vacancy for work-

mistress, desires exchange Assistant, Melbourne or suburbs. Address “ T.E.,” care of Mrs. Darling, 264 Sim pso n’s - road, Richmon d.

T WARREN BALL’S “Hints to Candidates L. fur Teachers’ and Alatriculation Examinations,” Is.; posted, Is. Id. Alullen, Melbourne.

CANDIDATES for EXAMINATIONS prepared by correspondence or otherwise. I. Warren Ball, South Yarra.

JMPEKIAL REVIEW,

Quarterly, 2s, Four Numbers, Subscription, 7s, Gd,, including postage anywhere.

Publishers :

ALEX. M'KINLEY & CO., 61 Queen Street, Melbourne.

TpX AMINATIONS.—UNIVERSITY

and DEPARTMENTAL.

TUITION in CLASS by Correspondence or otherwise. Teems, &c.,

JAMES L. ROBERTSON, B.A. (Melbourne). 71 Clarendon Street, Emerald Hill.

BUY AND READ

THE N EW WEEKLY PAPER. Price Threepence.

“ o N C E A WEE K

16 PAGES. PRICE THREEPENCE. ALL BOOKSELLERS.


Alex. M'Kinley and Co., Publishers, 61 Queen Street, Melbourne.


g TATE SCHOOL, No. 17 7,

TRADES’ HALL, LYGON-ST., CARLTON.


MR. SERGEANT,

(Late District Inspector of Schools-) Conducts Classes for Certificates of Competency on Saturday mornings in the abovenamed school. W ork will commence on Saturday, 22nd instant. At the examination in December last thirteen passed, though only six months under Air. Sergeant’s tuition.


CANDIDATES ARE ALSO PREPARED BY CORRESPONDENCE.


s.


MULLE


’ S


New Classified Catalogue of School, College, and Technical


EDUCATIONAL WORKS

May be had gratis on application, or posted on receipt of address.


SAMUEL MULLEN,

Wholesale & Retail Bookseller & Stationer, 29 & 31 COLLINS ST. E., MELBOURNE.


Price One Shilling,

By Post—In Victoria, Is. Gd. ; Out of Victoria, 2s


D


EPARTMENTAL EXAMINATION OF TEACHERS.

TEACHERS’ GUIDE

TO

AUSTRALASIAN EXAMINATIONS.

Containing the

PROGRAMMES & EXAMINATION PAPERS

of

Victoria    South Australia

New South Wales    Auckland, N.Z.

Queensland    Wellington, N.Z.

And Tasmania.

108 PAGES WITH STIFF COVER.


The above hook contains the Programmes and Examination Papers of December, 1877, of all the colonies, and is reduced to the low price of

ONE SHILLING.

By Post—In Victoria, Is, 6d. ; Out of Victoria, 2s


ALEX. M'KINLEY & CO., PRINTERS AND PUBLISHERS, 61 QUEEN STREET,

MELBOURNE.


Q C. EXAMINATION.

TUITION BY CORRESPONDENCE.


MR. THOMAS BOARDMAN, First-class Honor-man of the Denominational School Board, Prepares Teachers for the Certificate Examination by Correspondence. Terms moderate.

Address—

45 PRINCES STREET, CARLTON TAMES CLEZY, M. A.,

^    5 GORE STREET,

Prepares for Matriculation,Pass or Honours,and other University Examinations. Since February, 1874, One Hundred and Fifteen Pupils have passed various Examinations, ranging from Civil Service to the final Examination for B.A. Degree.

. Iu October Term, 1881, Four passed Matriculation, including Civil Service. One was first of the first-class in Greek and Latin Honours. One (a lady) obtained a second class in Greek and Latin. Two passed for first year LL.B.

N.B.—Students after Matriculating, and beforo proceeding to the University, should read (he First Year’s work of their course; otherwise the Lectures are of little benefit to them, and their chance of passing at the end of their first year is very small.


MR. H.

M'KTNLEY

S

OLICITOR,

CONVEYANCER,

And

PROCTOR,

86

COLLINS

STREET WEST.


TO HEAD MASTERS, SECRETARIES OF BOARDS OF ADVICE, AND OTHERS.

ALEX. M'KINLEY & CO.,

Having made considerable additions to their stock of Bookwork and Jobbing Type, are prepared to execute orders in every description of

GENE R A L P RINTIN G.


All orders entrusted to them will be printed in a satisfactory manner.


Alex. M'Kinley & Co., Printers, 61 Quben-st.


M


NOW READY.

I L T 0 N PARSED.

Price 2s.


By J. J. BURSTON,

(Author of “ State School Arithmetic”).

Also Ready, the Fourth Edition of the

STATE SCHOOL ARITHMETIC.

Ty

JOHN J. BURSTON,

North Sandhurst State School.


Printed and Published by Alex. M'Kinley & Co., G1 Queenstreet, Melbourne, under the auspices of the Victoriau Teachers Union.


AND LITERARY REVIEW.


Yol. Ill, No. 36.


JUNE, 1882.


SUBSCRIPTION

Yearly, Cs. 6d.; Half-yearly, 3s, 6d. Payable in Advance.


BLACKIE & SON’S SCHOOL SERIES.


Thesb Arithmetics are graded to meet the they are not limited in scope to the rules In large Schools it is essential not onl; rule and stage of a rule, but to supply ii halt of the class, showing the applicatioi throughout the Course is one of the spec:

A careful study of the causes of failur


Arithmetic.

lirements of the Code, though -here.    _

de practice in each successive nt exercise for the brighter ules. That they provide this ires of these Books.

Arithmetic has suggested other


special features, such as    .    ,

The abundance of word EXERCISES., as a preparation for the dictation of sums by the Inspector.    .

The diversity of style in setting the sums, as a means of preparing scholars for any kind of test, and sustaining their interost much better than long arrays of sums set to pattern.

The difficulties of each rule are introduced gradually, and a model example given at the head of each exercise.

Problems are given in abundance.

Instruction is given in the working of each rule for the higher standards, for too ofVn the teacher has little time to spare for them; but in the lower classes, no attempt is made to do that which only a teacher can accomplish.

Easy Lessons in Fractions are given at the beginning of Book V., that the pupil may the more readily and thoroughly do his Practice and Proportion.

The Tables required by each Standard are given in advance, that the pupils may not be able to refer to them instead of learning them.

Answers have been thoroughly tested, and are believed to be entirely correct.

Standards I, II. Ill,............paper cover, 2d, cloth 3d. each.

Standard IV. (English Code)    „    3d.    „    'Id.

Standard IV. (Scotch Code)    „    4d.    „    6d.

Standards V. VI................... ,,    4d.    ,,    5d.    each.

Keys to Standards I. to VI., paper 3d. each, or bound together in cloth, Is,

OPINIONS OF THE PRESS.

“ The set of Arithmetic books is well graduated, so that the scholar will pass easily and naturally from one hook to the next. The examples are numerous, and stated so as to exercise the reflecting faculties of the pupils without perplexing them.”—Daily Deview.


Geography.

ELEMENTARY GEOGRAPHY.

By W. G. BAKER, Associate of King’s College, Lecturer at Cheltenham Training College.

PartI. For Standard II. Elementary notions of Geography. Paper 2d.; cloth, 3d.

Part II. For Standard III. England and Wales. Two coloured Maps.

Paper, 4d.; cloth 5d.    _

Part III. For Standard IV. Scotland, Ireland, and the Colonies. Paper, 6d.; cloth, 8d.

“ Mr. Baker has shown his mastery of the difiiculties inherent to the teaching of small children in a science which is only just beginning to assume its proper position in schools. The pictures and diagrams are all to the point, and not

too complex.....If the succeeding numbers exhibit the same amount

of care and judgment in their compilation, Mr. Baker’s completed work must take high rank as an elementary treatise on geography.”—Public Opinion.


Third Edition—Now Ready.

A Manual of Method.

For Pupil-teachers and Assistant Teachers. Intended for the Government Inspected Schools of Great Britain and Ireland, and for the use of Students in Training Colleges. By Abr. Park, F.R.G.S, F.E.I.S., &c. Headmaster Albion Educational Institution, Ashton-under-Lyne. Interleaved with ruled paper. Third edition, foolscap 8vo, cloth, 2s.

^    OPINIONS.

“ No young teacher could puruse, and, above all, master, this really valuable little manual without receiving much mental stimulus and much wise direction in all that pertains to the diiGcult matter of school management. In fact, we do not know where else to find in so brief a space so much sound practical wisdom and suggestiveness on this subject.”—The Literary World.


Just Published. Price, 2s. Gd.

A History of the British Empire.

With numerous Pictorial Illustrations, Genealogical Tables, Maps, and Plans. By Edgar Sanderson, M.A., Late Scholar of Clare College, Cambridge. 444 pp., cloth, red edges, 2s. 6d.

The writer of this brief record of a great nation’s fortunes has aimed at producing a narrative that should be equally interesting and instructive. Ho had believed that it is possible, even in a summary perforce so short as this, to bs clear throughout, to bo stroug and vivid where force and liveliness are needee for a due effect, to rise, upon occasion, into eloquence, and to give to this com. pression of our country’s annals something of the heat and glow which must exist in all who, with sufficient knowledge, love her name and fame.

The convenience of not only the young student, but also of the generalreader, has been consulted in a close adherence to the order of time in the nar ration of events, except so far as regards occurrences in Scotland before the union of the crowns, and also as concerns the history of British India. Tire treatment of this most important subject in the present work is, so far as the writer is aware, a new feature in a book of this kind. A short, separate account is given of the transactions, exploits, and events which have made India the greatest of dependencies attached to any empire in modern times. The reader is thus enabled to peruse, with ease and interest, in a continuous tale, that which it has hitherto been needful to pick out painfully and piecemeal from tho history of divers periods and reigns.

A fuller account than has been usual, in books of this kind, has been given of the great Civil War of the seventeenth century, and of tho Peninsular War.

The history of England is, above all, a history of a nation’s growth in constitutional freedom. To this grand subject special attention has been paid in all its salient points.

It is hoped that the arrangement in paragraphs, with marginal notes of tho most important matters; the maps, plans, and illustrations ; the genealogical tables inserted in their proper places ; the list of chief events and documents with dates; and a full and accurate index, will prove of substantial service to the reader.

Religious and political bias has been scrupulously avoided throughout, and it is believed that hardly any proposition has been laid down, scarcely one view advanced that would not find a goneral adherence amongst fair-minded and educated people, of every shade of political and religious opinion.


School Classics.

Selections from Standard Authors, with Biographical Sketches and Explanatory Notes.

32 pages', price in paper cover, 2d. each ; in cloth, 3d.


Lady of the Lake, Canto I... .Scott.

The Armada, &c..........Macaulay.

The Prophecy of Dante, C. I.

II...............................Byron.

L’Allegro and II Pen.se-

roso...........................Milton.

Essays (selected) ......Lord Bacon.

Prisoner of Chillon ......Byron.

The Fire Worshippers, Parts I.

II...............................Moore.

Ancient Mariner ......Coleridge.

Deserted Village.....Goldsmith.

Marmion, Canto VI. .< ..... Scott.


The Lay of the Last Minstrel


C. I...............................Scott

Cotter’s Saturday Night..Burns

The Village..................Crabbe

The Pleasures of Hope, Part

I...........................Campbell.

Essay on Bunyan ......Macaulay.

The Queen’s Wake............Hogg.

The Merchant of Venice, Acts

I. III. IV............Shakspeare.

The Traveller.........Goldsmith.

Evangeline 64 pp. (price


4d.) ............H W. Longfellow.


THE PROLOGUE TO THE CANTERBURY TALES OF

Geoffrey Chaucer. The text collated with the seven oldest MSS., and a Life of the Author, Introductory Notices, Grammar, Critical and Explanatory Notes, and Index to Obsolete and Difficult Words. By E. F. Willoughby, M.D. Price Is. 6d.

PARADISE LOST. By John Milton. Book T., with Life of Milton and Prefotary and Explanatory Notes. By E. E. Willoughby, M.D. Limp cloth, lOd.


Further Particulars and Catalogues from all Booksellers, or from their Melbourne House, Carrington

Place, off Bank Place, Collins-street West.


Stjjml Qcpar.tin cut.


ON THE GRAMMATICAL ERRORS OF MORELL-

By A. A. de Mobnay, Sandhukst,

TO THE EDITOR OF THE “AUSTRALASIAN SCHOOLMASTER.”

Sir,—You will greatly oblige me by publishing in your next issue the remarks I desire to make on the “ errors” discussed in your April and May numbers by Messrs. Gough and Courtney. The discussion was opened by the former, who found fault with the word “ other” in the definition of an auxiliary verb, which plainly implies that an auxiliary verb is also a principal one. This is one out of many examples of the careless use of words to be found in the book, and it is so obvious as to be of little consequence. But there is a serious error of fact in this definition ; for an auxiliary verb does not “ aid in forming the tenses of other verbs.” There is not one auxiliary of tense. I have shown in my grammar that there arc but two tenses, present and_p«si, neither of which is formed by the aid of an auxiliary verb.

Mr. Gough next takes exception to the definition of the factitive object, given improperly under the head of the Indirect Object (p. 76). He says that, in defining the factitive object, Morell has not excluded the dative. But there I think he is mistaken, at least, its exclusion seems to bo intended by the words ‘‘noun or pronoun in the objective case,” by which Morell meant no doubt, to exclude all prepositions, which he evidently docs not regard as a part of the case, but as governing it. Morell had not a clear notion of the factitive object, which I have shown in my grammar (p. 92) to be complex. It is also direct, as Morell’s explanation of the word “ factitive” shows. Now, the second member of the factitive object is predicative in signification and form, and may, therefore, be an adjective, which is one of Morell’s varieties ; but it never is a “secondary completion,” by which term I understand him to mean a second or indirect object. An exhaustive discussion of Morell’s errors in treating of the Indirect Object would take up too much space, as the whole section is full of confusion. But Mr. Gough’s proposed definition of the factitive object is no improvement on Morell’s. There is the same fundamental error in both, viz., the separation of its two members into two distinct objects, the one direct and the other indirect, while the latter is really in a predicative relation to the former, and not an object at all by itself. For is it not absurd to call an adjective an object ? But Morell uses vague terms, such as completion for object, in order to disguise the absurdity of his propositions. In Mr. Gough’s definition a new error is introduced through his confounding the predicative with the attributive relation ; for a noun in apposition belongs to the category of attributes.

Here follow a few more cases of bad grammar, which, though they certainly should not be found in a treatise on grammar, yet, as they seldom obscure the rules, are of comparatively little importance ; but Mr. Gough seems generally to note these minor faults, and to overlook the material errors involved, such as that in Rule 6, relating to verbs (p. 109) : “If one verb depends upon another, they must observe a proper succession of tenses.” This rule is indefinite enough ; but the explanation that follows renders it incorrect—“ i.e., a verb in a purely past tense cannot be connected with a verb in a present tense, and contrarily, e.g., I thought he could succeed.” Compare I know he did succeed.

From Morcll’s statement that “the adverb of place or time is sometimes employed as a complement to the noun,” Mr. Gough infers that in the sentence, “ The church here is very fine,” a boy would be parsing according to Morell, if he said, Here, an adjective qualifying the noun church, and asks some irrelevant questions about the reason why it may be called an adjective. But Morell does not say it should be parsed as an adjective; he calls it an adverb, and as an adverb it must be parsed. Mr. Courtney justly observes that “ Morell is quite correct in the syntax of this word, though he does not think his reason is sufficiently clear.” In fact, it is no reason at all, it is just begging the question ; for the phrase “ in this place" has as great need of a reason as the adverb “here." The reason assigned by Mr. Courtney is the true one; but it might, with advantage, be expressed more definitely, thus : because here is the predicate of an adjective sentence attribute to church, of which the subject is a relative pronoun. Mr. Courtney will readily perceive that no other member of such an adjective sentence can represent the whole of it.

With respect to the last passage criticised by Mr. Gough, “ The verb to be takes two nominatives, the one before, and the other after it,” apart from the omission of the words “when in a personal (fiuite) form,” there is undoubtedly some difficulty, especially when the predicate is expressed by a pronoun. Is it generally improper for a personal pronoun to be used as a predicate, as it has no predicative signification ; and its so-called nominative case ought to have no other function than that of the subject of a sentence, in which capacity it is used conjunctively with the verb. In German (and it should be borne in mind that old English (AngloSaxon) was a Teutonic dialect) the neuter pronoun es (it) in the singular is the only personal pronoun that cau be used predicatively ; so that a German says, ’ Ich bin es (I am it), where we are taught to say, it is I, lhe i rench say, “ C est moi ” (it is me). It would also be better if we followed the vulgar usage, and said, “ It is me.” But this we is not the objective case of I; it is the form which the pronoun of the first person singular takes in all dependent cases. Now the subject of a sentence is the only independent member, therefore in every other grammatical icLition the form commonly, but improperly called the objective case, should be used : as, It was me; I speak of him ; Give it (to) me ; Look at usl You are not taller than me. The predicative case of a noun does not differ in any respect from the subjective, attributive (apposition), objective (accusative), or vocative case. All these cases of nouns having no preposition, might be called nominative, this term not indicating any particular relation ; but it is not applicable to pronouns, because they do not name (nominate) anything. When, therefore, I say that a noun in apposition is in the nominative case, I only mean that it is the noun alone (without a preposition).

After writing the above I borrowed The Queen's English, by Dean Alford, and found there that the author’s opinion of the sentence “ Itis me,” coincided with my own. He quotes, in support of that opinion, from Dr. Latham’s History of the English Language, a discussion on this form of speech, so severely censured by the common grammarians, by which he is led to the same conclusion, and, as it appears on the same grounds as mine. I only extract a short passage. Perhaps we may say, is there any real custom at all in favour of /,"except so far as the grammarians have made one ? It is clear that the French analogy is against it. It is also clear that the personal pronoun as a predicate may be in a different analogy from the personal pronoun as subject.”

The sentence “ I believe him to be a rogue,” is an example of logical factitive (in imitation of the Latin construction of the accusative with the infinitive, foreign to the Teutonic class of languages). The factitive object is him to be a rogue, which can be resolved"[see my grammar p. 93] into the sentence : “ He is a rogue,” of which the subject he represents him, and the predicate is a rogue represents to be a rogue. In both these forms the noun rogue is evidently in the predicative relation to the pronoun (he or him); and I have therefore named that which Morell calls the direct object, the subjective member, and that which he calls the indirect or factitive object, the predicative member of the factitive object. Taking now the other example quoted by Mr. Gough, “I know this man to be he,” in which the second or predicative member of the factitive object is represented by a personal pronoun, the resolution of the factitive object Hves “ This man is he.” Now it is repugnant to the genius of the English language to finish a sentence, unless in the interrogative or inverted form, with the subjective form of a personal pronoun ; therefore, and for the reasons given above, the fact would be better expressed thus: “I know this man to be him,” and still better : “I know this to be the man,” because a personal pronoun is not to stand for a predicate.

Remote from tonus be ran bis godly race.” I do not wonder that Mr. Courtney dissents from Morell’s syntax of the italicised phrase • I only wonder that he did not say more about it. First, it is given as'au example of a complement to a noun, and there is no noun in the sentence to which it could be a complement; for he is a pronoun, and a personal pronoun cannot be qualified, because it designates a particular person or thing with all its qualities ; and lastly, the adjective remote may qualify nouns denoting places and some abstract ideas, but not persons as individuals ; as, a remote district, a remote possibility. Mr. Courtney’s analysis of the sentence is correct. I have given it as an example of an adjective used adverbially. [Grammar, p. 98.]

Mr. Courtney’s remark on the parsing of prepositions is also just. Prepositions are governed, rather than governing words; or more correctly, they are joined to substantives to form some of their cases.

By Alex. Gough, H.T., School 972, Cobaw.

In my last paper on the above subject, I said, “ Is it because here is a word in the adjective sentence, which i? here, that it may be called an adjective ? Would Morell call every word in an adjective sentence an adjective?” The answers to these questions are so self-evident, that I wonder at any one seriously replying to them. By such ironical questions I more emphatically showed the absurdity of calling here an adjective. I have a great respect for Dr. Morell, and am quite certain we would agree in giving an emphatic “ No ” to each of the above questions. The Doctor is aware, and so am 1, that here is a contraction for the adjective sentence, Which is here. University students of logic might be expected to know this short out, and be allowed to call here an attribute to the noun church. There is danger, however, of our pupils not comprehending this fine point, and therefore I believe with Mr. Morgan and with Dr. Morell himself, who sanctioned Mr. Morgan’s correction, that “It would be more simple, not to say correct, to call here an adverb, and part of an adjective sentence attached to the noun church. Thus 'there would be no clashing with the fundamental principle, early taught to children, that an adverb cannot be joined to a noun, to express a quality of a thing.” (See‘‘ Companion to Morell,” page 25.)

I have shown that Morell is incorrect in his rule for the verb To be. Here, too, the doctor would acknowledge his error, and has actually acknowledged it through Mr. Morgan, Head-master, Cathedral Grammar School, Bristol, who is author of “ Companion to Morell,” and “Key to Morell’s Graduated Exercises.” He has omitted, accidentally of course, the words, when finite, in the rule referred to. Neither Mr. Morgan nor I would attempt to teach an “ LL.D.” English. The corrections are for those to whom Morell is a Text-book. Mr. Courtney says, “ Crombie’s rule cannot be improved upon.” I shall be thankful to him if he proves the clause which i have added incorrect. The subject to a (finite) verb is in the nominative case ; and when To be is that verb, the appositional word after it must, by the first part of the rule, be in the nominative. Here the whole is better than a part. The additional clause becomes necessary, as our young teachers, having such au admiration for Morell, are liable to accept his errors as grammatical beauties peculiar to himself. Morell is a very respectable guide ; but sometimes as accurate and intelligible as the Bout’un chap (gentleman from Bolton) who informs the traveller that Oud’am (Oldham) lies wide of Staley bridge. A young gentleman inquired for the residence of Dr. Syntax. I consider Morell has directed him correctly to turn to the right, but did not say when; Crombie has been more definite and pointed out the street; and I have given him the number of the house. I should have hired a cab for him, and given the address to the cabman if I thought it necessary.

Page 73.—“ Remote from towns he ran his godly race.” The sentences, “He, being remote from towns, ran his godly race,” and “ He ran his godly race in a place remote from towns,” are quite grammatical. Dr. Morell has analysed the first correctly, and Mr Courtney the second as correctly. But there is more beauty, peuetration, sympathy with the author, poetry and religious feeling in Morell’s interpretation, than in Mr. Courtney’s. The poets are remarkably unanimous in their admiration of Nature ; and are never more sublime than when alone with Nature and Nature’s God, pouring out their souls in adoration. They are too much inclined to believe, that, to run a godly race, we must first get remote from towns. In analysing passages from poets, we should be very careful about supplying the proper ellipsis. It is not sufficient to make sense and grammatical sentences. If we are not poets, we must, at least for the time being, ascend Parnassus and travel in spirit with the particular poet under consideration. I can appreciate Goldsmith’s sentiment, and therefore analyse him correctly. But as I am not a poet myself I would prefer £1000 and the heart of London, to £40 and the village of Auburn. I have worshipped God in temples built by the hands of man, and in Nature’s own. Whether on the snow-covered plains of Canada, the picturesque canons of Sierra Nevada, or surrounded by thousands of my fellow beings in Broadway, New York, I was in spirit with Goldsmith and Lamartine, in worshipping Eternal Truth and Beauty.

I’ll seek by day some glade unknown,

All light and silence like Thy throne,

And the pale stars shall be, at night,

The only eyes that watch my rite. ”

That the student may see the immense difference made by supplying the ellipsis being, I will, with Goldsmith’s permission, consider the prosaic sentence, Itemote from towns he studied agriculture. He, being remote from towns, acted as such a privileged person ought to act— he studied agriculture. There are certain things can be done in towns, but much better in the country. The poet does not mean to tell us as an accident, or simple matter of fact, that it was in a country place he ran his godly race. He means much more. He, being free from distractions peculiar to life in towns, being on the road to perfection, which leads from Nature to Nature’s God, being attracted continually and irresistibly (he) ran his godly race. The clause, Remote from towns, conveys all this and more.

PROGRAMME OF INSTRUCTION IN GEOGRAPHY,

By Alexander Allan, Keysborough.

The remarks of Mr, Gough, re “Text Books for State Schools,” are well worthy of the thoughtful consideration of every earnest teacher. Whilst agreeing with him in much he advances, yet I feel compelled to differ from him in regard to his suggestions concerning the teaching of geography.

As geography is at present taught in our schools, there is too much of the trivialities of the science and too little of what is really essential and interesting. Many of the children attending the State schools (more especially those in the country) do very little more than complete the thirty days in the quarter, therefore to attempt so much cramming of geography as he recommends would, I think, be great folly.

I am of opinion that the programme should be so amended that a scholar would get a knowledge of all the principal facts of geography before leaving the fourth class, because a large proportion of those obtaining the “ certificate” leave school immediately on getting it.

Only the broad principles should be taught. What is the use of a boy or girl, just from school, able to tell the position of some unimportant river, headland, or bay, and yet be in ignorance of the mail routes or chief commercial cities of North or South America ? In works of fiction, journals and newspapers, the names of towns may often be seen, but not so frequently capes or inlets.

I shall now give a rough sketch of what, in my opinion, would constitute a judicious course of instruction for the II. III. and IV. classes.

II.    Class.—Continents and oceans, with their boundaries. Also a few simple facts relative to the people who inhabit them, the colour of their skin, cast of features, modes of dress, and occupations. Whether the climate is hot, cold, or temperate. The character of the chief animals, and other things of an interesting nature, adapted to the comprehension of the class. There would be little difficulty, I think, in giving easy lessons on the above, more especially if they are well illustrated with coloured pictures.

III.    Class.—Principal seas, inlets, capes, islands, rivers, mountains, lakes, etc., marked on the maps of the World and Australasia, and the rudiments of Physical and Political Geography.

IV.    CLASS.—Countries of the World, boundaries, chief physical features, manners and customs, populations, governments, etc.

As a knowledge of the mail routes is not given in the Third or Fourth Books, it would be desirable to add this to the Fourth Class Programme.

So that prominence may be given to the descriptive part of the geography, only a very few names of places should be selected, and those really important.

Scholars remaining at school after obtaining “ certificate” could, of course, learn advanced geography, but the schemel propose, if thoroughly matured, would, I think, enable every child to get a fair knowledge of it prior to leaving school.

As regards the teaching of geography, I would suggest that only the .names of places with their positions, lengths of rivers, heights of moun

tains, populations, etc., should be committed to memory at home. Thede-scriptive part of the subject would be better imparted by the teacher, in a series of carefully prepared class lessons. He has the power, if he only uses it, of making them attractive, aud what is of an interesting character is easily remembered.

The results, at present, depend on a knowledge of places and their situation, and thus little attention is given to the truly useful, and to the scholars, the most interesting part of the science. Make the results to a certain extent hinge on the teaching of the descriptive part of the subject, and before long there would be a great change in the teaching of geography. No doubt there would be extra trouble entailed on the Inspectors in examining for results, but the allotting of two p’s, as in arithmetic, would greatly simplify matters

The remodelling of the Programme and the issuing by the Education Department of a aheap geography, judiciously compiled, having all in it, and no more than a State school scholar ought to know, would, l think, lead to a more satisfactory treatment of this subject in our schools.

SCHOLARSHIPS.

At a recent meeting of the Otago Education Board the following correspondence was read :—

Manse, Palmerston, May 2nd, 1882.

Dear Sir,—I beg leave respectfully to bring under your consideration, as chairman of the Education Board, the following reasons for making some alterations on regulations for provincial scholarships, especially the senior ones :—

1.    Subjects, viz., mathematics, Latin, French, and principal portion of English, are required for senior scholarships which arc not allowed to be taught in district schools during school hours, but they form a part of the regular course in all the High Schools of the Province.

2.    It is impossible for teachers and pupils in district schools to give the necessary time and attention to the extra subjects outside school hours. Teachers generally have pupil teachers to attend to either before or after school hours. It cannot be expected that pupils can give the sustained attention to extra and more difficult subjects after attending to their ordinary lessons. Besides, they have not the necessary time to prepare at home the extra branches in addition to their ordinary lesso ns. Further, the teacher is not able, after a hard day’s work, to enter with the vigour and enthusiasm necessary on the extra branches. Hence, if he attempts to prepare pupils for examination, either they or the pupil teachers suffer. Few teachers of district schools can in such circumstances compete with High Schools, nor will they care to undertake the task.

3.    District schools have to compete, in case of senior scholarships, with the best of those who were their own pupils, after getting two years of training in subjects of examination. The great majority of those who gain senior scholarships, had previously gained junior ones.

4.    The High Schools in Dunedin have no standards to pass, consequently the classes and subjects can be so arranged and taught as to prepare for the scholarships ; whereas in districts schools the passing of the required standards must be kept steadily in view.

5.    As a matter of fact, showing that the reasons above given are substantial, the High Schools have gained nearly ali the senior scholarships since juniors began to compete. In 1880, eight out of nine ; in 1881, seven out of nine.

6.    The above needs to be modified, as- the Normal School, which gained one last year, has an advantage over district schools, inasmuch as a special class is provided for those who pass the 6th standard, where all the extra branches can be, and are, I believe, taught during school hours. Hence the Normal School is on the same footing, or nearly so, as High Schools.

7.    Out of the 1,200 marks attainable, half are given for subjects that must be taught outside school hours in district schools, viz., mathematics, 150; French, 150; Latin, 150; English, 150. It must not be overlooked that in English subjects are prescribed (as Richard II. this year) which are not allowed as part of ordinary course.

The above reasons seem to me to show conclusively that the common schools cannot compete on equal terms with the High and Normal Schools. This accounts for the fact which the Education Board publishes, that the number of schools sending competitors to senior scholarship examinations is gradually decreasing. I know of two teachers whose pupils were successful before keeping back pupils they were preparing for the examination of 1881, because they considered it a hopeless task to compete successfully under present arrangements.

Would it not be advisable to modify the regulations so that the High and Normal Schools should compete in oneclas3 and district schools in another ? Would it not be advisable also to change the ages of the junior to fourteen and senior sixteen ? From the education reports I see the average age of 5th standard is over thirteen, and in a larger number of cases the average of 6th is over fifteen, or to make the ages thirteen and fifteen, as at present, at the time of examination. I. know some who were precluded from trying because they were so unfortunate as to be born towards the end of December.

Another point to be remembered is that these scholarships were intended to foster and stimulate the study of the higher branches taught in schools, and that not in some centre, but throughout the length and breadth of Otago, so that pupils might be prepared to enter the University. If senior scholarships are to be confined chiefly to High Schools, the sources whence our students come must be circumscribed, seeing our present Education Act is not favourable to the higher branches being taught in schools. If such a stimulus as these scholarships furnish is withdrawn, the higher education will suffer,

Points.


Introduc

tion.


St. Louis. Compressed.

Concave.

Indistinguish

able.


Blackboard,


Com - pressed = pressed together, or close.


Concave = hollowed out.


It is the glory of Scotland that the way to the University is open to all her sons however remotely situated. I. for one, am anxious to see a like spirit evoked in Otago ; and these scholarships, if judiciously distributed, will help, I am persuaded, to beget a love for knowledge which the power5 fulinfluence of the “ race for riches ” cannot destroy.

I have long thought on the above subject, and spoken to teachers and others interested on education about it. In a conversation I had with S. Macdonald, of the High School, a year ago, he then expressed himself as if he considered the district school too heavily handicapped. When last in town I intended to call on you to ventilate the subject, but you were away in Auckland. Pardon the fulness and length of this letter.— Yours truly,    James Clark.

Dunedin, May 15th, 1882,

Dear Sir—I have to thank you for your well-considered letter on the working of our scholarship regulations. You have given too much attention to the subject not to be aware that it is surrounded with difficulties, arising from the ill-adjusted relations at present subsisting between the primary and secondary schools. The considerations which you have adduced received full consideration from the Board at the time when the scholarship regulations were framed, but I will offer a few comments on them seriatim,

1.    I am glad you do not suggest that the extra subjects (Latin, mathematics, &c.) should be excised from the examination for senior scholarships. This course has been urged, and it would without doubt get rid of the inequalities of which you complain ; but it would be at the cost of rendering the scholarship valueless. If a boy at the age of fifteen has not made some progress in the elements of a secondary education, it is then too late for him to begin. In order, however, to mitigate the burden on district schools, the number and standard of extra subjects demanded for the examination have been fixed much lower than otherwise would be desirable.

2.    and 6. The difficulties enumerated in these paragraphs are not remediable, I think by any change in the regulations. In order to remove them we require to get the Education Act so modified as, under certain circumstances, to permit teachers in public schools to instruct their advanced pupils in extra standard subjects within the usual school hours.

3.    and 5. This also seems to be in a great measure unavoidable. It ■would be very unfortunate if junior scholars (in most instances) did not succeed in gaining senior scholarships, as in that case the money already expended upon them would to a great extent be wasted. It is fairly open to consideration, however, whether it would not be proper to offer a larger number of senior than junior scholarships, in order to give those who are not junior scholars a better chance of success.

4.    The relation of the Dunedin High Schools, and also of the district high schools, to these scholarships is a subject of considerable delicacy, regarding which those interested may be expected to take widely different views. The report of the Royal Commission, 1880, page 35, deals fully with this question, and contains recommendations which appear to me to be expedient and equitable.

7. I think you have somewhat over-stated the disadvantage the district schools are under as regards the marks allotted to the different subjects. These were apportioned with extreme care, so as to be as favourable as possible to the district schools. It was not intended by the Board that English should be treated as a non-standard subject. I know no reason why the portion of English literature prescribed should not be read in the ordinary course ; but if this cannot be done I should be decidedly in favour of excising the prescribed portion of literature, so as to make the examination a general one. Of the other non-standard subjects, you must recollect that not more than two can be taken by any candidate. Regarding English as a standard subject, the case really stands thus :— Out of an attainable maximum of 950 marks a district school candidate can obtain 750 marks on standard subjects alone ; with one extra subject he can get 900 marks, and with two extra subjects the maximum,

Looking at the matter in this respect, I am unable to see any good reason why public school teachers should become so faint-hearted as to give up the contest. I know of one district school in a neighbouring province which has not been afraid to enter the lists with the most famous schools of the colony in the examination for the scholarships of the New Zealand University, and it has attained, too, a most astonishing degree of success. You will admit that this is an undertaking fully ten times ns arduous as preparing for our senior scholarship competition.

I do not think you have adduced any sufficient reason for interfering with the limit of age fixed in the regulations. No one is excluded from the competition by the fact of his having been born in December or any other month ; but no doubt some candidates are placed under the disadvantage of having to compete with others who are nearly a year older than themselves. This is a difficulty, however, which is inherent in the nature of things, and it cannot be removed by any alteration in the regulations.

I most cordially agree with you in your closing remarks. To my mind, the educational reform most urgently wanted is the introduction of    the elements of secondary instruction into the public

schools. The late Education Commission considered this question also, and made valuable—although perhaps too timid—recommendations on the subject (Report, p. 5). If the Education Act were so modified as to allow the elements of secondary education to be taught in the public schools for one hour a day, the hardships in the working of our scholarship schemes would at once vanish. But this would only be a small part of the advantage gained. The main consideration is that our primary and secondary school systems would thus be brought into workable relations with one another, and a broad foundation laid for national culture. I shall be glad if the hardships inseparable from working our scholarship scheme, under present circumstances, have the effect of directing public attention to this all-important question.

If you desire it, I shall be happy to lay your letter before the Education Board, as it contains matter which is well deserving of the attention of that body.—1 am, &c.,    John Shand.

Rev. James Clark, Palmerston.

Mr. Fulton : I propose that these letters be handed to the Press for publication, because it is impossible for us to discuss them fully now.

Mu. Feaer : I think that is a very good suggestion. I think that Professor Shand has very fally answered the letter sent by the Rev. Mr. Clark.

It was agreed that the letters should be discussed at the next meeting of the Board.

NOTES OF A READING LESSON ON A BALLOON VOYAGE IN AMERICA.5

For Upper Class (Standard IV.)

As we passed up, the great city of St. Louis beneath seemed to be compressed and drawn together on a concave, the valleys and woods melting into each other until their outlines became almost indistinguishable; the great, snorting steambots below looking like toy-houses floating in a gutter, and emitting faint puffs of smoke. .    . At about eight o’clock

we could see the people below were having their sunset, although we were in full blaze of light. The prairies looked like vast fields of polar ice, slightly tinged with green, but quite destitute of light. .    .    .

Between us and them hung susqoended, evidently, a dark and almost opaque belt, which seemed like a veil drawn over the country. The alternate patches of cultivated grounds, water sheets, and little hills and gorges, gave to all a diversified appearance ; the hills had lost their relative cone-like appearance, and seemed to be vast sugar loaves, fretted with raisins and lemon-parings—the effect of alternate forest and grass. Very gradually the darkness stole up from below. It was as though invisible hands were lifting up the veil as it approached and enveloped us. In a few moments the sun left us, disappearing in a hazy, luminous bank of red. It did not become dark. .    , Throughout the night we

were able at all times to distinguish the prairies from the wooded country below, even when at a height of two miles. We were floating in a sort of transparent vapour, which without possessing any perceptible body, yet seemed to be made up of luminous particles, .    .

The effect of this light was very peculiar : it gave the balloon a sparkling appearance, as though it were charged with fire. So powerful was this, that every line of the netting, every fold of the silk, every cord and wrinkle, were as plainly visible as if illuminated by torches ; and I could at any moment tell the time by consxdting my watch. This appearance became more striking as we increased our height.

Scheme op Reading Lesson (Standard IV.)

(jy.B.—The scheme is the same for Standards V. and VI.)

1.    Teacher reads words at the head of the lesson, children repeating them after him simultaneously.

2.    Children read same words, forwards, backwards, horizontally, simultaneously, without teacher.

3.    Teacher reads first paragraph slowly, and elicits or explains meaning of underlined words, Sec. (See notes below.)

4.    Teacher reads first paragraph in phrases with expression, the children reading after him simultaneously, imitating his emphasis, pauses, &c.

5.    Teacher calls on individual children to read the same paragraph.

(The other paragraphs to be treated similarly,)

6.    At the end of lesson, children close books, and teacher examines on text and subject matter.

Explanation.

Ask if any of the children have seen a balloon. What is it for? Who would like to go up in one ? How strange everything must seem ! We are going to read of what a man saw from a balloon. Open books, page 73.

Tell child ren that this is a city on the shores of the Mississippi, in United States.

Divide the word on B.B., thus, corn-pressed. Elicit, or tell, that prefix com = together; then let children give meaning of “ compressed.”

Show a watch glass. Any difference in the sides?    Elicit

that one surface is hollowed out, the other raised up :—the hollow side called concave, the other convex.

(a)    Ask the meaning of “distinguish,” in a sentence ; as, “He cannot distinguish green from blue.”

(b)    Then, “distinguishable as, “ The colours were not distinguishable.”

Points.


Explanation.


Blackboard.


ftlmbcrsifn


of lEcIirountc.


Emitting.

Prairie.


Destitute of light.

Suspended


Opaque.


Alternate.


Gorge.

Diversified.


Relative conelike appearance.


Fretted.

Alternate

In-visible.

Enveloped.

Luminous,

Transparent,

Perceptible.

Particles.

Peculiar.

Consulting.

Questions.


(c) Elicit meaning of prefix “in,” as in in-sensible, insufficient, etc., and lastly in in-dis-tinguis hable.

Let children put into their own words the whoie sentence, “ Their outlines became almost indistinguishable.”

Elicit meaning from context.

Tell how a great part of America is occupied by rolling plains, or prairies, covered with long grass.

Let children put this in their own words—without light—dark.

The map is suspended on the wall—the meat on the butcher’s hook. Meaning of suspended ? Point out, that we speak also of clouds being suspended (or held up) in the air.

I can see through the window ; if you close the shutter I can’t see through. The shutter is opaque. The window is not. Meaning of opaque ?

Illustrate what is meant by giving sums to alternate boys ; or a boy taking alternate sums. Here the cultivated patches are separated by wood and water; they alternate—first a patch, then wood and water, then patch again, &c.

Elicit, or tell meaning.


Indistinguishable == not able to be made out, or recognised.


Emit = to send out. Prairies ■= great grassy plains.


Suspended = held up.


Connect with “ divers” and “ diverse,”—varied.

Have children ever seen a sugar-loaf in the grocer’s shopwindow ? It is cone-like, or shaped like a cone. Draw section on blackboard.

Point out the force of the expression “relative.” The mountains, seen from the level, looked, in relation to (that is in connection with) the surrounding objects, like a cone. Looked down upon, they seemed different.

Compare with “fret work,” raised work. The mountains looked like sugar loaves, ornamented with lemon-peel, etc.

Elicit meaning of “alternate forest and grass.”

Divide the word, and elicit meaning.

Compare with envelopes. What is an envelope for? When may we be said to be enveloped ?

Compare with “ luminary” and “illuminated,” and elicit meaning.

Refer again to the window and shutter. Why did we call the shutter opaque ? A thing which we can see through is said to be ? —[Transparent,]

Meaning of “ transparent vapour”?

Compare with “perceive.” Elicit meaning of “perceive” : “ It was so dark I could perceive nothing.” Tell, that what can be seen, or perceived, is said to be. perceptible. Meaning of imperceptible!

Compare with parts—drops.

Elicit meaning: “He was a peculiar looking boy.” Strange, not common, unusual, singular.

When ill, we coyisult a doctor : that is, ask his advice, seek information from him. Meaning of “ consulting my watch” ?

Where is St. Louis? Meaning of compressed? Tell me something that is concave. When is a thing indistinguishable, etc., etc.


Gorge = deep narrow passage between hills.

Diversified = varied.

Cone-like =• sugar-loaf shaped.


Fretted =* adorned with fret-work or raised ornaments.


Invisible = not seen,


Luminous = light.

Transparent «= can be seen through,


Perceptible^ can be seen.


Particles = small parts.

Peculiar = singular,


Consult = ask for advice or information.


The following are the Papers set by Messrs. I. Warren Ball and H. J. L* Batten at their Test Matriculation Examination held at the Athenæum :—

FRENCH.

1.    Write the (A) masculine singular of : bonnes, maîtresses, jalouses, belles, celles-ci. (?>) feminine plural of : gentil, mon chat, frère, vieux, lequel, (c) third person singular—present indicative, imperative, imperfect, subjunctive, and also the present and the past participle, simple and interrogative, of : aller, offrir, s’asseoir, devoir, craindre, savoir.

2.    Translate into English—[a) En voyage le meilleur instrument, le plus efficace passeport, est de parler couramment la langue du pays où l’on se trouve ; on peut alors agir directement sur les esprits ; il y a peu de gens qui apprécient toute la puissance de ce moyen : tout est là. (b) Un brave chevalier français s’attaqua au roi d’Angleterre et lui porta des coups terribles. Henri ne dut la vie qu’à la solidité de son armure. Dans la mêlée, un anglais saisit la bride du cheval de Louis-le-Gros en criant: “ Le roi est pris 1”

3.    Translate into English and French respectively—1. Avez vous de l’argent ? I have none. 2, Votre père est-il chez lui ? He has gone out to-day. 3. Y aura-t-il un concert ? There will be one this evening. 4. Etes-vous prêts, Messieurs? Yes, we are. 5. Où est le marché au poisson ? Near the Town Hall. 6. Quel temps fait-il ? It is still as cold.

4.    Distinguish—en, dans ; de and à in compound nouns ; avant, devant ; prêt à, prés de.

5.    Translate into French—Dr. Franklin was born at Boston, in America. From his earliest youth he was placed with one of his brothers, who was a printer ; he made rapid progress in this useful art and from that time had for the trade of a printer a likiug that lasted all his life.

LATIN.

1.    Translate into English—

(a)    Mugitus veluti cum prima in preelia taurus Terrificos ciet atque irasci in cornua temptat,

Arboris obnixus trunco, ventosque lacessit Ictibus, aut sparsa ad pugnam proludit arena.

(b)    Talibus occurrit dictis : hac, Turne, sequamur Trojugenas, qua prima viam victoria pandit ;

Sunt alii, qui tecta manu defendere possint.

2.    Translate—(æ) Hæc loca viciuitatibus erant nota, magnamque res diligentiam requirebat non in summa exercitus tuenda—nullum enim poterat universis ab perterritis ac dispersis periculum accidere—sed in singulis militibus conservandis ; quee tamen ex parte res ad salutem exercitus pertinebat.

(b) Atque unus ex captivis, “ Quid vos,” inquit, “ hanc miserarn ac tenuem sectamini prædam, quibus licet jam essefortunatissimis.”

3.    Translate—After remaining in this locality for a few days, he sent messengers to King James to sue for peace. Meantime fresh forces were sent to his assistance, and upon their arrival he immediately determined to cross over to the mainland and renew the contest, After disembarking his troops, he received information that the enemy was approaching his camp by forced marches.

4.    Translate—Quapropter hoc maneat in causa, judices : non enim dubito quin probaturus sim vobis defensionem meam, si id memineritis quod oblivisci non potestis, insidiatorem jure interfici potest.

5.    (a) Decline—aliquis, plebs. (b) Explain and illustrate the formation of—frequentative, desiderative and inceptive verbs.

6.    Give the etymology and syntax of words italicised in the foregoing selections.

EUCLID.

Book I.

1.    If two angles of a triangle are equal the sides opposite them also are equal.

2.    To describe a triangle having its sides equal to three given straight lines, any two of which are greater than the third.

3.    Equal triangles on the same base and on the same side of it are between the same parallels.

Book II.

4.    The square on the sum of two straight lines equals the sum of the rects. contained by their sum and each of them.

5.    The sum of the squares on two unequal straight lines is equal to the square on their difference and twice the rect. contained by them.

6.    If one angle of a triangle is obtuse, the square on the side opposite it is greater than the squares on the sides containing it by twice the rect. contained by either of them and the projection on it of the other.

Book III.

7.    If any two points be taken in the circumference of a circle, the straight line that joins them shall fall within the circle.

8.    If a straight line touches a circle, the straight line drawn from the centre to the point of contact shall be perpendicular to the line touching the circle.

9.    To bisect a given circumference,

PHYSICS.

1.    What is meant by (I) the linear (2) the cubical co-efficient of expansion ?

2.    Convert 15 degrees C. into the Fahrenheit scale.

3.    Distinguish between the convection and conduction of heat.

4.    What is the scientific definition of boiling point ? Prove the truth of the definition by referring to experiment.


G. Simplify 7. Solve


■ +


x + 3 y - 2« = 21.


a Solve-§-—f- + 8-S*+ ^


5.    Describe Breguet’s Thermometer, and also the gridiron pendulum, carefully explaining the action of each.

6.    How many pounds of ice at 0 degrees C. can be melted by one pound of steam at 100 degrees C. ?

ALGEBRA,

1. Find the sum of

2 b o,l 3    1 4c

~ a+ ~4~ ~1T ~6 a ~ ~8~^~°> and 2a + ~g-2> -~g~

2.    Two boys have each x + y—z marbles, and agree to share winnings and losses. They each play three games with other boys; one of the two loses in three games xy + z marbles; the other wins 2xy + 2z. How many marbles has each after settling with the other.

3.    Simplify (a + b + c) (<z2 + 52 + c2 + be + ca + ab) - (J + c)3 - (c + a)—{a + b)3 + [a + b + c) (a2 + 52 + c2).

4.    Divide [a - by - c2 by a - b + c.

5.    Prove that 4(x + y) 0-y) + 6(s-y)®-2{x + y)* =8x(x-2y),

yz

x - z zx—yx y - z x + 4 x + 3 x 6 x + 5 x + 3 x + 2~x + 5~ x + 4 8. Solve lOx -2y + 4z = 10 ; 3z + 5y + 3z = 20;

2

10. If in squaring a number I inadvertently increase it by one previously, and obtain as result a number too large by 25, what is the number ?

GEOGRAPHY.

I.

1.    Draw to the full size of your paper so much of Africa as lies between the tropics.

2.    Give a short descriptive account of the settlement and progress of any of the Australian colonies except Victoria, describing the towns, imports and exports.

3.    What useful products are exported from Italy, Mauritius, and Scandinavia respectively.

4.    W hat are the most important facts in the political geography of France ?

1.    Through what counties and past what towns does a train pass in going from Melbourne to Wodonga ?

2.    Explain clearly that the sun never sets on the British Empire.

3.    Briefly describe the surface of Ireland.

4.    Describe five lakes having no outlet, one on each of the continents

.    III.

1.    Briefly describe an eclipse.

2.    What are the chief causes influencing climate?

3. Mention some of (a) the causes, (b) the effects of currents.

4.    What countries are not subject to volcanic action ?

ENGLISH.

.1. Analyse fully—He will often argue that, if this part of our trade were well cultivated, we should gain from one nation ; and if another, from another.

2.    Parse the words in italics in :—That great man who has a mind to help rue, has as many to break through to come at me as I have to come at him.

3.    Quote ten lines beginning thus Go, wiser thou !”

4.    W rite a précis of the essay on Labour and Exercise.

5.    Derive iully :—Presume, isthmus, sceptic, suspend, peccant, lunatic, empty, pyramid, spectator, clergyman.

G. Explain and refer to the context :—Of a mild or choleric disposition. During my nonage. His taste for books is a little too just for the age he lives in. He will excuse generals for not disposing according to men’s desert, or inquiring into it. A handsome elocution. The modish world found too great a constraint in them.

7. Write a short essay upon Winter.

The following are the papers setiorthe Second Matriculation Examination, February Term, 1882 :—

GERMAN.—Honours.

1. Translate into English—

Ich will nun einmal Euch nicht weiter sehn !

Nicht hören ! Will von Euch an eine That

Nicht fort und fort erinnert seyn, bei der

Ich nichts gedacht; die, wenn ich drüber denke,

Zum Rätbsel von mir selbst mir wird. Zwar mccht’

Ich sie nicht gern bereuen. Aber seht;

Ereignet so ein Fall sich wieder : Ihr

Seyd Schuld, wenn ich so rasch nicht handle ; wenn

Ich mich vorher erkund’,—und brennen lasse,

Was brennt.

Au dem grossen Flusse, der eben von einem starken Regen geschwollen und übergetreten war, lag in seiner kleinen Hütte, müde von der Anstrengung des Tages, der alte Fährmann und schlief. Mitten in der Nacht weckten ihn einige laute Stimmen ; er hörte, dass Reisende übergesetzt sein wollten.

Als er vor die Thiire hinaus trat, sah er zwei grosse Irrlichter über dem angebundenen Kahne schweben, die ihm versicherten, dass sie grosse Eile hätten und schon an jenem Ufer zu sein wünschten. Der Alte säumte nicht, stiess ab und fuhr mit seiner gewöhnlichen Geschicklichkeit quer über den Strom, indess die Fremden in einer unbekannten, sehr behenden Sprache gegen einander zischten und mitunter in ein lautes Gelächter ausbrachen, indem sie bald auf den Rändern und Bänken, bald auf dem Boden des Kahns hin und wieder hüpften.

2.    When are viel, wenig, and all used as indeclinable ?    ’

3.    “ The Reflective Verb is sometimes used for the English passive.” Illustrate this.

4.    By what instances is the partitive relation expressed by von instead of the Genitive ?

5.    What prepositions govern the accusative ? Illustrate the different uses of um.

6.    What prepositions may either be separable or inseparable from the verb ? Illustrate the respective differences in meaning.

7.    What are the principal suffixes by which adjectives are formed ?

8.    Illustrate the use of doch, ja, gerade, and wenn.

9.    Give the derivation of Schlacht, Dankbarkeit, Schlosser, Erziehungsanstalt, Regierungsantritt.

10.    Translate into German—He is poor in money, but rich in knowledge. The consul will speak about the affair in good time. The picture will be found among the other things. I dare not confess to him what evil I forbode. The old man was allowed to escape all the more readily because every one was anxious to discover what had happened. As the highest point of the range was not far from the upper end of the valley, and as we could reckon on fine weather, we determined to go.

11.    Numerous translations of this work and countless editions of them, in every language, bespeak its adaptation to mankind ; no critic has withheld his admiration, no reader has ventured to confess a want of relish for that in which the young and old in every climate have, age after age, taken delight. They have doubtless believed that they understood the author’s meaning.

GERMAN.—Pass.

1. Translate into English—    •

(a)    Bei seiner Rückkehr von Wien verirrte er sich im Schwarzwalde, und kam, als es fast ganz finster war, nach einem kleinen Dorfe, worin er nur ein sehr erbärmliches Wirthshaus fand. Es schien ihm ganz unmöglich, dass er darin die Nacht zubringen könnte. Zugleich sah er wohl ein, dass er bei der Unbekanntschaft mit der GegendTicht weiter reisen könnte, da die Nacht sehr finster und Niemand im Dorfe zu bewegen war, ihn als Wegweiserzu begleiten.

(b)    Du bist wie eine Blume So hold un schön und rein ;

Ich schau’ dich an, und Wehmuth Schleicht mir ins Herz hinein.

2.    Parse the words kam, schien, könnte, worin.

3.    Decline iu full Jeder fleissiger Knabe, alter guter Freund. Is such a phrase as “kalt Wasser” allowable ?

4.    Decline the pronouns wer and welcher. Is there any distinction between der and welcher as relative pronouns ?

<5, What is the force of the terminations erlei and fach when added to cardinal numbers ?

6. Give the Impf. Indie, of können, gehen, achten, herabsteigen.

i. What is a separable verb? Give examples of its use.

8.    What classes of verbs are conjugated with haben, and which with sein ? Give examples.

9.    What is the rule for the agreement of the adjective when it qualifies the noun, and when it is predicate ?

„ 10. Translate into German—What sort of a book is he reading ? Is the Yarra as broad as the Murray? I have only been a few days in Melbourne. The writing of this letter is important. Do come with me. He is about to go away.

11. translate into German—A spendthrift lord was once rebuked by a friend, who pointed out to him the result of extravagance, <! You are right,” said the lord, “• I am afraid that I shall die a beggar.” “ That is not the worst, said his friend, “ I am afraid that you will live as a beggar.”

GE OM ETRY. —[For Pass.]

1.    The side BC of a triangle ABC is produced to D. Prove that the angle ACD is greater than the angle B.

2.    I he opposite sides and angles of a parallelogram are equal, and a diagonal bisects it.

0.    To describe a parallelogram equal to a given rectilineal figure and having an angle equal to a given angle.

4.    C is any point in the straight line AB ; prove that the square on AB is equal to the squares on AC, CB, and twice the rectangle contained by AC, CB.

5.    The angle B of a triangle ABC is acute; AD is drawn at right angles to BC meeting it in D. Prove that the square on MG is less than the squares on AB, BC by twice the rectangle contained by CB, BD.

G, Describe a square which shall be equal to a given triangle.

t. If from a point within a circle more than two equal straight lines can be drawn to the circle, that point is the centre of the circle.

8.    From a given point without a circle to draw a straight line to touch the circle.

9.    Angles in the same segment of a circle are equal.

ELEMENTARY PHYSICS.

Dynamics.

1.    Explain carefully the meaning of the terms Molecule, Cohesion, Capillarity, Endosmose.

2.    State the second law of motion, and describe an experiment illustrating it.

3.    Describe Atwood's Machine, and explain how it is used in order to

prove that while the viass remains the same the velocity generated in unit of time varies as the force.    .

4.    Describe Bramah’s Hydraulic Press. What ratio must exist between the diameters of the pistons in order that a power of unity may overcome a resistance of 1000?

5.    Describe the Air Pump. Assuming the capacity of the glass vessel or receiver to be four times that of the cylinder, what quantity of air will have been extracted after four strokes of the piston?

Heat.    .

1.    Describe Leslie’s Differential Thermometer. For what purposes is this instrument employed ?

2.    How would you proceed in order to construct a compensated balance for a chronometer ?

3.    What is Regelatiou ? How do you account for this phenomenon ?

4.    What is Conduction ? Describe an experiment for determining the difference of conducting power of copper and iron.

5.    The hottest part of a perfect heat engine being at 130 deg. Centigrade and the coldest at 30 deg. Centigrade, calculate what proportion of the heat passing through the engine will be converted into mechanical work,

Sound and Light.

1.    State clearly your reasons for believing that sound and light consist of vibrations propagated through an elastic medium.

2.    Explain carefully the meaning of the terms qjhasc, front of a wave, amplitude, fluorescence, interference.

3.    What is meant by reflexion ? State the laws by which this phenomenon is governed in the case of light and sound.

4.    What is meant by total internal reflexion ? Describe an experiment proving it existence.

5.    Give all the information you can relative to the Solar Spectrum.

Electricity and Magnetism.

1.    Explain carefully what is meaut by electric potential ?

2.    A magnetized steel rod is suspended so as to be perfectly free to place itself in any position whatever. What direction will it assume in London, in Melbourne, at the equator, and at a magnetic pole ?

3.    Describe carefully some simple form of Voltaic Battery.

4.    Describe the construction and explain the mode of action of Graham Bell’s Telephone.

5.    What is the electric light, and how is it produced ?

ELEMENTARY CHEMISTRY.

1.    What is the atomicity of the following elements?—H, O, N, Cl. Br,

S, P.    .

2.    Explain why you consider two atoms of hydrogen as equivalent to one atom of oxygen.

3.    What are empirical, rational, and constitutional formulae ? Give examples.

4.    Give the names of the substances denoted by the following formula; : HC1, Si02, H? SO,, S02, NH3.

o. What evidence can you adduce that nitrogen occurs as a monad, as a triad, and as a pentad element?

0.    Calculate the percentage composition of nitric acid.

7.    What are the chief sources whence ammonia and its compounds are derived ?

8.    What are the most important properties of phosphorus ? How is it made ?

9.    You have given you a piece of charcoal weighing 5 grms. and 6 litres of oxygen. On burning the charcoal in the oxygen how much carbonic acid by weight and volume will, you obtain, and how much carbon, if any, will be left ?

ELEMENTARY PHYSIOLOGY.

1.    Describe the mechanism of inspiration and expiration.

2.    Where is the lachrymal gland ? What does it secrete, and what becomes of its secretion ?

3.    What are the essential conditions of the production of the voice ?

4.    Describe the mode of digestion of lean meat.

5.    Describe the coagulation of the blood, and mention the principal circumstances which influence the time of its occurrence.

6.    By what means is the body kept at a uniform temperature ?

ELEMENTARY BOTANY.

1.    Describe the structure and function of the leaves of plants.

2.    Describe the mode of origin of the roots in dicotyledonous and iu monocotyledonous plants respectively.

3.    Describe the structure of an embryo in any of the higher orders of plants.

4.    How is the propagation of seaweeds effected ?

r 5. Describe the parts of a flower in any :of the higher orders of plants.

6.    Describe all the kinds of vascular tissue found in the first year’s growth of an exogenous stem.

7.    Describe the part of a potato used for foods in botanical terms, indicating the nature of the external parts, and the chief contents of the internal cells.

8.    Give the chief systematic characters of an Acacia or common wattle.

9.    How are roots distinguished from stems ?

10.    Describe in botanical terms the seed of any of the higher orders of plants.

Ditforian (Jpbucatimt department.

APPOINTMENTS.

John Nicholson, H.T., Nareen, 2303; Marion Hayes, H.T., Wohla, 320; Frank W. Powell, H.T., Edi and Carboor East (half-time), 1422; Donald A. Schulze, H.T., Tallengowor, 1S71 ; Mary E. Cahill, H.T., Pan-noobamawm North, 1853 ; Duncan Cameron, H.T., Waterloo Plains, 2119, John P. Whitfield, H.T., Witchipool East, 2117; David Hayes, H.T., Youarang, 1923; Mary Madden, H.T., Havelock and Chintin, 1269; Elizabeth A. Nicol, H.T., Mitiamo, 1521; Charles E. Tranter, H.T., Little River, 1961; Ruth Williamson, H.T., Daylesford-road, 1000; Ernest Williams, H.T., Nanneela North, 2469; John Delahcnty, H.T., Mounta-jup, 1618; Agnes Broadbent, H.T., Brooklyn, 2465 ; Harriot Egan, H.T., Koyuga East, 2370; James B. Willis, H.T., Mount Gellibrand, 1175; F. H. Cockrem, H.T., Poowong East, 2251 ; John B. Jones, 11.T., Enoch’s Point, 1180; Phoebe Astloy, ll.T., Mincha West, 1931; Christina Sutherland, H.T., Yarrawonga South, 2194; Margaret Cousin, H.T., Wharparilla West, 1537 ; Wm. Beiiby, 2nd Asst., Carisbrook, 1030 ; Henry Brent, H.T., Woollert, 1861; Annie Rogers, 1st Asst., Essendon, 483; Sarah D. Strahan, II.T., Puckapunpal, 1855 ; John T. Daly, H. T.,Yea, 699; Thomas

S. Robinson, H.T., Frankston, 1464; Elizabeth Bailey, H.T., Moore’s Flat, 1575 ; Ada Heath, ll.T., Terrick Terrick East, 2006 ; Jane Christian, H.T., Croxton East, 1550 ; Frank Oldfield, H.T., Moree and Connowirricoo (halftime), 1543; William F. Gates, 2nd Asst., Footscray, 1912; Mary M. Mathison, 1st Asst., Stanley, 550; Harry Hardy, H.T., Shipton, 582; Alice Devlin, II.T., Labertouclie, 2471 ; Charles Rankin, H.T., Ardno and Strathdownie West (half-time), 2397 ; Alice M. Hill, ll .T., Darragan, 2430 ; William H. W- Rail, H.T., Junction Dungaree, 1960 ; Jemima Birrell, H.T. Parwan, 916; Marion L. Hayes, H.T., Broadford North, 1897.

The following are the papers set by the Department for Examination in Drill at the examination held 19th June, 1882 :—

Morning Paper,

(Time allowed three hours.)

Squad Drill with Intervals.

1.    Give in detail the “ position of the soldier.”

2.    Describe position of soldier when marching.

3.    What is the length of pace in slow, quick, and in double time ; also in stepping short and stepping out.

4.    Describe the formation of a squad with intervals in two ranks.

5.    Describe the method of dressing a squad with intervals.

6.    A squad stepping out is required to resume former pace of 30 in. Give words of command to do so.

In Single Ranh.

7.    Describe the method of forming a squad in single rank.

8.    Give words of command and dress a squad in single rank.

9.    Describe in detail “ Dressing when halted.”

10.    Describe the (a) “Inner,” (&) “ Outer,” (n) “Directing,” and (d) “ Reverse flanks.”

11.    Describe the method of dressing a squad together.

In Two Ranks.

12.    Describe the method of forming a squad in two ranks.

13.    Describe the formation of “Fours”—(a) “Right,” (h) “Left,” and (c) “ About.”

14.    A squad turned to the rear having received the word “ Front ” did not turn about properly, what order would you give to turn it to the rear again?

15.    Give the words of command necessary to dress a squad, and describe how the rear rank will act.

16.    Give words of command and explain the method of forming “ Fours Deep ” on the march.

17.    What is the rule with regard to the rear rank when marching in file ?

18.    A squad marching in files to the right is ordered to form to the right, give words of command and all detail.

19.    A squad advancing, describe in detail the method of (a) Forming fours, (h) Closing on a flank, and (c) again Re-forming two-deep.

Afternoon Paper.

(lime allowed two hours and a half.)

Company Drill.

20.    Describe the method of sizing a company.

21.    Tell off a company of thirty-four-and-a-half-file.

22.    How many paces are allowed for the company you have just told off? Show how you find it.

23.    Place the officers and non-commissioncd officers of a company, and describe what their duties are when so placed.

24.    Describe in detail a company in line taking open order.

25.    Describe how a company in line at the halt will wheel from “ line ” into “ column.”

26.    Describe how a company in column at the halt will form into line to the left.

27.    Describe how a company in column will change direction on the move.

28.    A company moving to a flank in fours is ordered to form to the right; detail the movement.

29.    A company moving to the front is ordered to form fours and close on the right; describe the movement and give full detail.

30.    A company at the halt is ordered to counter-march ; describe the movement.

31.    Describe how a company will reduce its front by breaking off files, also to bring the files broken off to the front.

32.    Describe bow a company will reduce its front still advancing and re-form company.

33.    A company on the march is ordered to counter-march ; describe the movement, giving all words of command and how guides and markers will act.

34.    A company in line advancing ; describe the manner of diminishing the front to column of sections.

35.    Place the captain, guides, and markers,'wken in column as above, right in front.

36.    Describe how a company in column on the march will wheel into line, and how the guides and markers will act,

37 Instruct a company (a) how to change ranks, and (b) how to resume its original front.

The following is the paper set by the Department for Certificate in Drawing, at the Examination held loth June, 1882 :—

PRACTICAL GEOMETRY.

1.    Construct a square that shall be equal to a given rectangle.

2.    Draw an inscribed and a circumscribed equilateral triangle to a given circle.

3.    Draw the joints of the arch-stones of an elliptic gothic arch.

The following are the papers set by the Department for License to Teach Drawing, at the Examination held 14th June, 1882 :—

I.—ANGULAR PERSPECTIVE, WITH INSTRUMENTS.

Given the plane of the picture, and one edge of a cube against the plane, draw the cube in angular perspective, showing Horizontal line, point of sight, or centre of vision, vanishing points, measurement points, station point, ground plan, geometric scale.

II.—PRACTICAL GEOMETRY.

1.    Trisect a right angle, and show an angle of 75 degrees.

2.    Describe a circle whose circumference shall pass through three given points not in a straight line.

3.    Draw a line from a given point outside a given line, making with the given line an angle equal to a given angle.


CONTENTS

School Department—

Victorian Education Department—

On the Grammatical Errors

Appointments ... ...

183

of Morell ... ... ...

178

Examination Papers ...

183

Programme of Instruction

Science and Art Gossip ...

184

in Geography ......

179

New Book ... ... ...

184

Scholarships ... ... ...

179

Leader ... .........

184

Notes of a Reading Lesson

The New Education Code ...

185

on a Balloon Voyage in

Primary Education in France

185

America .........

180

Notes of the Month ......

186

University of Melbourne—

Education Commission ...

187

Test Examination Papers...

181

Punishment ... ... ...

191

Matric. Exam. Papers ...

182

Order ... ... ... ...

191


ANSWERS TO CORRESPONDENTS.

“Caddy.”—Wc certainly could not undertake to comply with your request. J. Tipping,” “ Zeno.”—Received.

“ T. IIall.”—30th June, 1882.

“ Gbo. H. Watson (1794).’ ’—Received. Will expire May 31, 1883. Received.—P. J. Brennan, Geo. Boulton, E. H. Burgess, J. K. Brown, Miss M. Bill, Miss J. Christian, Miss S. J. Cole, Miss M. D. Cowan, C. H. Cecil, F. E. Craig, R. Craig (1586), M. Cody, John B. Doyle, Miss E. R. Day, Miss Edmiston, G. W. Eastwood, Miss Ford, James Gray, J. Gardiner, E. Gilsenan, Roland Gray, Miss Gillies, T. II. Gill, Miss Guest, M. Hogan, Miss A. Hodges, Jas. Irvine, Miss Kirwood, Wm. Lonev, R. Lewi», Chas. Madigan, R. McCormack, T. M'Naught, C. M‘Aliece, D. M'Lachlan, John M'Swiney, No. 2056, J. Nally, Miss M. A. O’Meara, Miss C. M. Phillips, S. M. Planck, C. S. Potts, Thos, Russell, Miss C. M. Rees, Matthew Robinson, J. C. Rowe, II. Stielow, C. F. Schoive, Miss M. Smethurst, Thos. Tindall, Miss A. M. White, “Winkle,” J. K, Ward, L. C. Young._


Science anb %xi <S0ssi|i.


NOTICE.

Subscribers are reminded that the Subscriptions for the ensuing year are now due, and an early remittance is requested.


Professor Thomson, Professor of Experimental Physics in University College, Bristol, has recently published a volume entitled Elementary Lessons in Electricity and Magnetism, which places before students ail the most recent doctrines on these subjects in a most accurate and a very popular form.

The controversy on vivisection is still being carried on in the English scientific journals. A student of medicine in reply to the accusations against Professor Schifit, writes to Nature that that gentleman never operates on feeling animals. He says : It is true that there do exist experiments in which the animal must retain consciousness, in order that the effects may be watched; but just because the animal would suffer pain, these experiments are neier carried, out by Professor Schiff.

A PAPER was read before the Otago Institute (N.Z.) on September 2]

on the skeleton of the Notormis Mautelli, a very rare flightless rail_the

Tahahe of the Maories—of which, until recently, there had been discovered only a few fossil bones and two skins. Lately a third specimen was killed on the eastern shores of Lake Te Anau, and was examined by Mr. T. Jeffery Parker, and was then sent to England for sale.


NOTICE TO ADVERTISERS.

In sending advertisements for insertion in the Schoolmaster, advertisers will please remit stamps for amount at the following scale : —

16 words, One Insertion, - Is. Od. | 32 words, One Insertion -    2s. 6d.

24___^____,,    -    2s. Od. | One Inch, „    .    4s. Od.


INSTRUCTIONS TO SUBSCRIBERS.

Lady subscribers, when remitting their subscriptions, will please state whether their papers are to be addressed Mrs. or Miss.

Subscribers will please send P. 0. order or postage stamps, when remittance is under <£1. Duty stamps cannot be accepted as payment.

MARRIAGE.

Andrews—Pineo.—On Wednesday, 21st June, at the residence of the bride’s parent, by the Rev. W. Souter, Frederick Andrews, S. School, Woodstock, eldest son of Mr. Wm. Andrews, Newham, to Susan, eldest daughter of G. Pineo, S. Morang School.


NEW BOOK.

“An Introduction to French Authors,” by Antoine Charlin, late French master at the Presbyterian Ladies’ College, Melbourne contributor to Littre’s dictionary of the French language. Melbourne : S. Mullen, Collins-street east, Mr. Charlin’s book will be found a very useful aid by all who wish to acquire facility in reading and translating French. “In order to facilitate research, dictionaries are arranged alphabetically; in order to facilitate a thorough and scientific under• standing in the present work, each example has been placed where it could most advantageously be to illustrate and throw light on some point that could most gain by it. Sometimes the choice of an illustration, its brevity, its translation, might be criticised, when the defect was, however, intentional. A common phrase, a complete sense, an exact and elegant translation, were not in all cases the best adapted for the purpose in view ; the conclusions derived from the examples when passages bearing analogy to them were to be met with in future reading had also to be foreseen. Most generally sentences or words have been grouped so as to point out etymological affinities. The meanings are then presented in their logical sequences, so as to give the student^ with the help of his teacher, an insight into the process of thought in the formation of idioms, in the shifting and interchange of ideas"in words and their derivatives. Sometimes words or phrases have been placed in proximity to warn more impressively against a possible confusion arising from a fortuitous likeness. Sometimes, when the idiomatic transhv tion, combined with previously acquired knowledge, is not sufficiently explanatory, a sentence will be immediately preceded by the words that must be studied by themselves, before they, are studied throuo-h an otherwise misleading, though idiomatic translation. Sometimes also words or sentences will be found unconnected, which could not profitably be placed under any particular heading.”

^usiralasian Siljhûlmasltr,

PUBLISHED EVERY MONTH.

MELBOURNE, JUNE, 1882.

^ We have before expressed regret at the mistake made by the Government in the selection of gentlemen to serve on the Education Royal Commission, and events have transpired to justify our remarks. At the time, it appeared to us that as the Commissioners were not confined to an examination of the alleged grievance of the Roman Catholics, but were to ascertain how far the Act of 1872 had been administered in conformity with the intentions of Parliament, one of the permanent heads of the Education Department, and one of the leading State school teachers should have been placed on the Commission. Had this course been adopted some of the mistakes into which the Commission have fallen, and much of the unpleasantness that has been created, would have been avoided. Anyone who has read the published report of the questions put to the gentlemen already examined must see, that not only are many of the questions altogether irrelevant to the subject of inquiry, but that they have been put in such a form as to make the answers given not unfrequently convey a different impression to the reader’s mind to what the gentlemen answering intended by their answers. At a recent sitting of the Commission, for instance, tire Secretary of the Department is reported to have said that the Minister of Public Instruction—the Hon. Major Smith—made the large number of appointments complained of as being in excess of the number of teachers allowed by the Education Act, with his eyes open to the fact that such appointments were in contravention of the Act.” The fact is, however, that the secretary simply gave the answer  yes,” to the categorical questions put to him by the chairman. The implied reflection upon the late political head of the department was couched in the question put, not in the answer given. Now, it is evident that this is unfair to the gentlemen examined, more especially in the case of those who are officials, and who may be materially injured by this mode of procedure. The changes in the relative positions of political parties in these colonies are rapid, and we would therefore suggest to the Commission a more guarded and generous treatment of officers of theEducationDepartment and teachers who maybe called before them. If they wish to obtain the true views of the State school teachers and the officers of the Department upon the practical working of our school system, then they should see to it that no one giving evidence is endangered by having his answers interpreted other than in the sense in which he himself gave his replies.

THE NEW EDUCATION CODE.

It would be impossible, even if it were desired, to pass over the new Code without comment, especially when its provisions are likely to affect materially—as will immediately be seen— the interests of Secondary Education. We do not, however, propose to deal with the Code in detail. It may be, as the Times cheerfully assures us, that it may be compared to a wellfitting boot, which presses gently and equally on all sides ; it may be, on the other hand, that the new arrangements will be found to involve a very considerable increase of anxiety and trouble, with more uncertainty as to income than has hitherto been the case. For our own part, we are prepared rather to wait and look on, than to prophesy as regards these points. What we desire especially to point out here, is the importance of two points in the Code. The first of these provides that graduates of any University in the United Kingdom, and women over eighteen years of age who have passed University Examinations recognised by the Department, and persons who have passed the examination for admission to a Training College, may be employed as assistant teachers. This, it will be seen, is a most important provision in itself. Coupled with one which we shall immediately call attention to, it is, we think, one requiring the most serious consideration. First, however, let us enumerate the Examinations recognised by the Department. They are—the Oxford Local Examination for Senior Students, the Oxford Local Examination for Women over Eighteen, the Cambridge Local Examination for Senior Students, the Cambridge Higher Local Examination, the University of London Matriculation Examination, the Durham Examination of Senior Candidates not members of the University, the Dublin Senior Examination for Women, the Edinburgh Local Examination for Senior Certificates, the Glasgow Local Examination for Senior Certificates, the Aberdeen Local Examination for Honour Certificates, the St. Andrew’s Local Examination for Senior Certificates. Let us ask ourselves, who and what are the holders of these Certificates compared with the ordinary certificated teachers'? Some, no doubt, may know a good deal less, and be far worse teachers ; others, again, with less experience, may yet have a wider area of knowledge ; all of them will be ambitious to increase the aim of their schools, and to magnify their office by expanding the range of subjects taught. In the admission of graduates, then, does the Department intend to provide posts for the more unfortunate or the less efficient of that great body, or does it propose to enter into competition with the Secondary Schools 1

This is a question which intimately concerns, not only our Secondary Teachers, but the public at large. We can only judge of the intentions of the Department from the Code itself. The table of “specific subjects” is henceforth to include the

following,—Algebra, Euclid and Mensuration, Mechanics, Chemistry, Physics, Animal Physiology, Botany, Principles of Agriculture, Latin, French, and Domestic Economy. It is presumable, therefore, in order that these subjects may he taught in Board schools, that graduates are to be admitted. Let us remember that these are all, without exception, subjects hitherto considered as belonging exclusively to the higher education. It has been held that the children of the poor should receive sound instruction in the elements of knowledge. They were to read, write, cypher ; singing, drill, and sewing have been added without opposition. Everything that helps to make them useful, and to know how to make the best of life, should be, and has been, cheerfully accorded to them. We were anxious also to see our poorest children instructed in such things as patriotism, love of order, liberty, civil duties, rights and obligations of Englishmen, and such knowledge of trade and manufactures as comes under the head of Technical Education. But such things as Botany, Latin, French, Algebra, Mensuration, and so forth, which belong to professional and scientific training, we did not, and we do not, desire to see taught in Board Schools supported by public funds; and we are prepared to expect that an outcry will be raised in the House, and outside of it, against an attempt which, if it be successful, will greatly increase the educational rate, and deal a death-blow at all self-sustaining education for the middle classes. It was not for this that Board Schools have been created. We leave the question at this point, but we shall return to it again. Meantime our readers will do well to keep their attention upon the subject.—Educational Times.


PRIMARY EDUCATION IN FRANCE.

The Primary Education Act recently published in France presents several points of interest, and has already attracted the attention of English educationists. It should be borne in mind that across the channel primary education is only part of a vast machinery which, under the name of the University of France, takes in the education of the whole country. This vast system is presided over by the minister of education, who is assisted by a council. Under their authority are twelve Inspehtors-general, and a number of ordinary Inspectors, who visit all parts of France and examine both public and private schools, whether primary or secondary. In each department there are Lycees, or secondary schools. Each department has also a normal school for the training of teachers, and every commune or parish is bound by the law to support at least one primary school. The subjects and methods ot instruction are controlled by the central authority to an extent we Englishmen can scarcely understand, and to which we should hardly submit. A French minister of education is said to have declared that by looking at his watch he could tell what lesson was being taught in any public school in France. The strict military precision which characterises French education comes out very strongly in the regulations for enforcing compulsory attendance. A School Commission, or School Board, is to be established in every commune, of which the Mayor is always to be president, and the primary Inspector an ex-officio member. This commission is required to draw up annually a list of all the children in the commune aged from six to thirteen, and to inform the persons having charge of these children of the date when the school opens. The father or guardian, as the case may be, must inform the Mayor at least a fortnight before the school commences whether he intends giving the child instruction at home, or in a public or private school. In the latter case he must specify the school. Should he neglect to do so his child will be enrolled in any one of the schools under the Board. A week befoi’e the resumption of classes the heads of public and private schools receive a list of the children who are to attend, and a duplicate of these lists is sent to the Inspector.

In this way it is scarcely possible for a child to escape being placed under instruction, and, if under instruction, he is compelled to learn. Both public and private schools in France are inspected, and if the child is taught at home he must annually undergo examination in such subjects as he would have been taught according to his age in the public school; and if the


child’s examination is deemed unsatisfactory, the parents are summoned to send him to a public or private school within a week of the notification, and to inform the Mayor what school has been selected. The means adopted for securing regularity of attendance are equally stringent. The head teachers keep for each class a register of non-attendance. This, by-the-way, is a hint we might adopt. It would be much easier to mark those who are absent than those who are present. At the end of every month these registers are sent to the Mayor and the inspector, with a list of the absentees and the reasons assigned. When a child shall have absented himself from the school four times in the month, during at least half-a-day each time, without a justification admitted by the School Board, the father, or other responsible person, is summoned before the Board and admonished. If the fault be repeated within the next twelve months, the names and occupations of the responsible persons ” are to be publicly exhibited on the door of the town hall. In case of a fresh relapse the defaulter will be handed over to the justice of the peace, and he will be liable to fine and imprisonment. This country, we are afraid, is scarcely ripe for such drastic measures, and we must be content with seeing our compulsory bye-laws less efficiently carried out. English people also dislike uniformity in educational matters. They are not in love with the half-military system established in France. They think both teachers and pupils ought to have more individual liberty and greater freedom in mental development. But even in England we are, to a considerable extent, “ cabined, cribbed, and confined ” by the regulations of Code. Still, we struggle with our fetters as much as we can, and rejoice in the little freedom we have.

There is one other respect in which the French system of primary education differs from the system in England; there is no religious instruction in the public schools. It is enacted “ that the primary public schools shall be closed one day a week besides Sunday, so as to allow parents, if they desire it, to give their children religious instruction elsewhere than in the school buildings.” Two things are to be noticed in this clause —first, “ if they desire it,” which implies that few parents would take advantage of the offer; secondly, the religious instruction is not to be given in the school-buildings, which virtually indicates ffiat it shall not be collective, but individual. The truth is that, in France, rightly or wrongly, the Government are afraid of the influence of the priests, and strive by every means to eliminate it from their educational system. Happily, in this country, we have more confidence in our religious teachers. It is true that dogmatic teaching is excluded from our board schools, and, considering the differences of opinion which exist among our Board members, it could scarcely be otherwise. But the almost universal opinion of the country is in favour of religious education. This, we think, is a subject for congratulation on two grounds. In the first place, we believe, there can be no sound system of morality which is not based on religious motives. Our second reason is a different one, and we think is not sufficiently recognised. We think it would be a great intellectual loss to the children if the Bible were excluded from our schools. The Bible, it should be remembered, is a complete literature, and the literature of a people who have exercised an immense influence upon all civilised nations. We are constantly being told of the advantages of a classical education, and the cultured refinement which results from a study of Latin and Greek authors. Of late years also great importance has attached to the study of English literature, and our own poets have become classics in our schools. But we should not forget that in the Bible we have poetry. We have there a series of writings of various kinds, extending over two thousand years, and no one can study these writings carefully without imbibing something of their beauty. We are not now referring to the religious aspect of the question. Looking at the Bible as a mere collection of books, we say that, from an educational point of view, its value can scarcely be over-estimated. Again, something might be said about the rhythm and purity of the language with which it has been translated. This need not be dwelt upon. We have, we think, said sufficient to explain why, in our opinion, even upon purely secular grounds, the exclusion of the Bible from our schools would be a subject for the deepest regret.—Schoolmaster.

|lo.tcs .of fíje iflonib.


Mr. Mullen has in the press and will shortly be published, Mr. Morris's edition of “ Dryden’s Annus Mirabilis,” and “ Hazlitt’s Selected Essays,” with notes ; and Mr. J. Clezy’s edition of “ Virgil’s iEneid,” Book I. and “ Caesar's Gallic War,” Book I, with notes. These texts are prescribed for the Matriculation Examination at the Melbourne University.

Mr. J. F. Blanche has sent us a copy of his “ Prince’s Visit ” and other poems, second edition, with considerable additions. It is creditable to the teaching profession in Victoria that it includes within its ranks one who can write poetry as Mr. Blanche can; We believe it was the late Rev. A. M. Henderson who recommended a verse of Mr. B.’s composition (so as to include the Royal family in the National Anthem) as suitable to be sung at the demonstration in honor of Prince Alfred, by the children of the Common schools. It is a pity that the author himself, by altering the verse without the sanction of the late Board of Education, caused that body to withdraw its previous order that the verse, as adopted, should be so sung. Mr. Blanche has a good command of choice language, and both his rhythm and rhyme are excellent. He seems to delight in the use of the short and pithy expressions of AngloSaxon origin. The first of his “Cantata” poems, and that on the closing of the exhibition, are really fine productions.

Major Mauley has been promoted to be Lieutenant-Colonel of the Adelaide rifles, vice Fitzroy, resigned. Colonel Fitzroy, who is now in England, was very popular with the force, and by the death of the Duke of Grafton, becomes next heir to the dukedom, with the title of Earl of Euston. It may be interesting to teachers to know that LieutenantColonel Madley is the principal of the Training College, Adelaide.

“ A treatise on Mathematics, as applied to the Constructive Arts, and Applicable to the Requirements of Practice,” is the title of a very excellent work, by Francis Campiu, C.E., forwarded by Mr. S. Mullen, of Collins-street East, Melbourne. This treatise has been specially prepared with a view to meet the wants :—1st. of youths who arc debarred the advantages of a thorough mathematical education; and 2nd, of practical mechanics, who commence the study later in life. The present is a revised and enlarged edition of Mr. Campin’s work, in which the author deals with the higher mathematics, on the same principle as in the first edition, namely, excluding Greek symbols, and explaining in text such processes as appear complicated in their algebraical demonstration. This work should find a large sale. The London publishers are Messrs. Crosby, Lockwood & Co.

We have received from Mr. S. Mullen, of Collins-street East, Melbourne, a copy of a new edition of the “ Short and Simple History of England,” by the Rev. B. G. Johns, and published by Crosby, Lockwood & Co., London. This little work gives a brief and succinct account of the History of Great Britain and the Colonies from the earliest times down to the end of the year 1881. The object of the writer is “ to present in the account of each reign some one picture of the times, or some one remarkable event, which, in the pupil’s mind, may remain as a definite mark of a particular age, or as clearly showing the character of the sovereign, without encumbering his memory with long, wearisome descriptions.” We should be glad to see Mr. John’s little book placed in the hands of every child who can read.

The Melbourne agent—Mr. S. Mullen—for the publishers of DeFiva’s “ Grammaire des'Grammaires, ” has forwarded a copy of the “ Forty-fifth edition, revised and cularged, 1882.” The Grammar of French Grammars has been so long and favourably known as a text book for matriculation and other students that we need say little to recommend it to teachers. The emendations and additions made in this new edition will be found useful to all who have chosen French as a subject of study for examinations.


EDUCATION COMMISSION.

Wednesday, 15th February, 1882.

Present—J. Warrington Rogers, Esq., Q.C., in the chair ; Herbert J, Henty, Esq., J.P., W. H. Archer, Esq., J.P.. F. Ormond, Esq., E. Keogh, Esq., J.P., J. M. Templeton, Esq.,J.P., G. Mearcs, Esq., J.P., D. Love, Esq., Hon. J. Macgregor, J.P., H. N. Loughnan, Esq,, J.P,, F. McCoy, Esq., F.G.S., W. H. Cutts, Esq., M.D., J.P.

G. Wilson Brown further examined.

By the Commission.—J think there was some information you were to give the Commission ; first, the proportion of non-paying pupils under the old education system?—Yes; I have extracted that from the last report of the Board of Education. I find that out of a total of 131,145, 14,884 were paid for as destitute by the Board, that is 11-349 per cent. ; that was the report for the year 1871, the last that was published.

Another matter you were to inform the Commission upon was as to the promotion of officers not in accordance with the rule of seniority— you thought there were some instances of that?—I have a list here of all the officers who have been promoted out of the order of seniority, but I should explain that the so-called promotions are given by recommending for a slight increment of salary when the annual estimates are prepared. I have a list of all officers who have got such an increment as has placed them over the head of somebody who was previously senior to them.

And have you the dates of those ?—Yes.

Would those be at the instance of the Minister of the day, or of the head of the department ?—I can state in each case whether it was by the Minister himself, or on the recommendation of the permanent head. [The witness read the following return\ :—


Promotions—Office Staff.

Date.

Promoted over—

Name.

Salary.

Name.

Salary.

1st July, 1875 ...

J. Bagge ... ...

£215

0

0

F. J. McCann ... ...

£200

0

0

H. Shelton ... ... ... ...

200

0

0

F. Adams ... ...

200

0

0

25th Jan., 1878 ...

II. Burke

200

0

0

J- Bagge (*)* ... ...

425

0

0

L. H. Hart......

360

0

0

A, Leslie ... ...

350

0

0

F. Brown ...

320

0

0

J. M. De Lacy ... ... ...

320

0

0

25th Jan., 187S1 ...

L. H. Hart (2)* ...

...

360

0

0

A. Leslie ... ... ...

350

0

0

33 33 "

A. G. Dumas (3)* ...

250

0

0

F. Adams ... ... ...

240

0

0

>> n •••

L. W. Stach......

155

0

0

J. II. Water held ... ... ...

155

0

0

1st July, 1878 ...

D. Courtney .. ... ...

125

0

0

C. W, H. James ...

• • •

...

230

0

0

H. O. B. Lane

22 5

0

0

F. 0. Handhold ...

225

0

0

B. Parker ...

225

0

0

R. Patterson ... ...

225

0

0

C. II. Streeton ... ...

225

0

0

1st July, 1878 ...

F. 0. Handheld (4)* ...

225

0

0 )

33 >5 •••

n j> •• ♦

B. Parker (s)* ... ... R. Paterson (°)* ...

225

225

0

0

o !

0 (

A. C. Witton... ... ... ...

200

0

0

53 3? •••

C. H. Streeton (7)* ...

225

0

o )

1st July, 1880 ...

A. Leslie .. ...

390

0

0

L. H. Hart ... ... ...

385

0

0

1st’July, 18813 !!'.

J. Symon ... ...

136

13

4

J. Martin ... ... ., ...

130

0

0

T. V. Foote ... ...

280

0

0 )

3

33 3 » •**

3

3 3 3 3 •••

T. McM. Callan ... T. E. Tatham ... ...

280

280

0

0

0 (

0 (

J. P. Hornidge ... ... ...

275

0

0

3 3 5 3 •••

C. W. H. James ...

2S0

0

o )

3 3 3 3

F. 0. Handheld ...

...

275

0

0

H. 0. B. Lane

270

0

0

3 3 3 3 •••

J. Symon ... ...

160

0

0

F. B. Smith ... ... ...

150

0

0

3 3 3 3 •••

John Baker • ... ...

145

0

0

J. Martin ... ... ... ...

140

0

0

1    See promotion, 1st July, 1880.

2    Mr. Adams is now in receipt of same salary as Mr. Dumas.

3    These three officers are senior to Mr. Hornidge by service.

In the cases marked * the promotion was granted by the Minister of his own motion, without the recommendation of the heads of the department. Nos. (1) and (2) occurred at the time of the general dismissals in January, 1878. The papers containing authority for Nos. (8), (4), Wj (6). (7), do not show the reasons for which the increase of salary was granted. Nos. (8) and (9) were special cases, the increase of salary being granted in recognition of the gentlemen in question being employed in work of a specially confidential character.

In all the remaining cases the increase» of salary were granted on the recommendation of the heads of the department.

1st July, 1875 1st July, 1875

3 3    3 3

1st July, 1876


1st July, 1877 11th June, 1878 1st July, 1879

1st July, 1879


1st July, 1879


1st July, 1879 1st July, 1880 1st July, 1881


lst July, 1874 ^oth March, 187Í


Temporary Staff.

R. Paterson ... ... ,. ...

J j

£0

11

0

C. H. Streeton ... ... ...

0

11

°)

C.W.H. James ... ... ...

1

0

11

0

R.T.C. Yates ...... ...

0

11

0

F. M. Hughan ... ... ...

180

0

0

F. C. Bannester ... ... ...

175

0

°)

W. Potter ............

175

0

0 I

D. C. Newham ... ... ...

170

0

0

J. F. Archibald ... ... ...

170

0

o]

W. M, M, Campbell (s)* ... ...

200

0

0

A. Jackson (°)* ... ... ...

200

0

0

D. W. Ramsay .........

185

0

0

F. Reddin ... ... ... ...

185

0

0

J. H. Chads............

180

0

01

J. M. Dwyer ... ... ... ...

180

0

o(

F. R. Pohlman ... ... ...

180

0

0

C. T. Crespigny ... ... ...

180

0

0)

W. E. Whitlow ... ... ...

170

0

0

G. Stanway .. ... ... ..

165

0

0

J. C. Knight ... ... ... ...

165

0

0

W. E. Whitlow .........

180

0

01

R. Gibbs ... ... ... ..

175

0

of

W. A. D. Burke ... ... ...

175

0

0}

J. Madden ... ... ... ...

175

0

0

J. P. Tennent ... ... ...

175

0

o y

H, F. Richardson ... .. ...

175

0

0

J. C. Knight... ... ... ...

175

0

oj


J. Baldwin ... T. Bolam ... C. Tynan ... A. C. Curlewis S. J. Swindley H. Shelton ... S, Summons ...


Inspectoral Staff.

£400

0

0

650

0

0

411

13

4

411

13

4

411

13

4

375

0

0

375

0

0


M. M. Campbell ... ... ...

£0

10

0

C. Harper ... ... ... ...

S

O

0

10

0

M. M. Campbell ... ... ...

y

0

10

0

C. Harper ... ... ... ...

t-i

0

10

0

F. M. Hughan ... ... ...

0

10

0

M. M. Campbell ...... ,..

160

0

0

M. M. Campbell ... ... ...

160

0

0

T. E. Venables ... ... ...

160

0

0

Private Secretary to Minister ... ...

Private Secretary to Minister ... ...

M. M. Campbell ... ... ...

170

0

0

T. E. Venables ... ...

170

0

0

J. H. Chads ... ... ...

180

0

0

M. M. Campbell ...... ...

170

0

0

T. E. Venables ... ... ...

170

0

0

M. M. Campbell ... ... ...

170

0

0

T. E. Venables ... ... ...

170

0

0

R. Gibbs ............

0

10

0

G. Stanway ... ... ... ...

165

0

0

W. A. D. Burke .........

0

10

0

T. Burton ... ... ... ...

0

10

0

R. Gibbs .........

0

10

0

R. W. N. Lind .........

0

10

0

T. E. VenableB ... ... ...

170

0

0

T. E. Venables ... ... . .

170

0

0

T, Burton ... ... ... ...

170

0

0

R. Craig ... ... ... .

£300

0

0

J. Main ... ... ... ...

610

0

0

J. Holland............

411

13

4

R. Philp ... ... ... ...

375

0

0

R. Philp ............

375

0

0


What is meant when it says “on the recommendation of the secretary”?—That simply means that certain gentlemen were recommended for an increase of salary and certain others were left where they were, not being thought worthy of an increase.

When you say “promoted over” others, would that imply merely an increase of salary or would it give a right to further promotion by virtue of that standing?—No, not necessarily. Each man’s case is considered separately when the estimates are being prepared, and therefore although one man may get an increase of salary, which will place him over the head of another, still the following or a subsequent year’the man whom he has passed may get a larger increase, which may bring him back to his old place.

So that it merely means giving him a larger salary ?—Yes ; and for the time being, while he has t that larger salary, he is regarded as senior.

Does that apply to the permanent staff ?—Yes.

Are not the permanent staff classified?—No.

So there is no difference between permanent and temporary in that respect ?—No, they are both dealt with in the same way.

Would you inform the Commission on what particular grounds the increase of salary was given to persons you named as “ promoted at the instance of the Minister”?—I am not aware, beyond the Minister giving an order that they should receive those increases.

Was there nothing stated in the minute of the Minister giving the grounds on which he made those virtual promotions ?—No. It is simply an order for an increase of salary.

And no grounds stated for that ?—No.

A case of sic volo, siejubeo ?—Quite so.

The officers were not consulted at all in the promotions you allude to as having been made by the Minister ?—It certainly did not emanate from the officers of the department, and I do not think they were consulted in it.

What other information were you to supply ?—As to night schools. During 1881, the schools in operation were 41 ; on the rolls there were 6,801, and in average attendance 2,166. I was also to furnish a return of the number of children who passed the standard of education each year since 1873. I have a return here ; and I have to modify my evidence of last week with regard to the issue of certificates. I then said that the certificates were only issued to those children who applied for them. That was correct up to about three years ago ; but I find that, some time in 1878 or 1879, the inspectors met together and discussed the matter amongst themselves, and found that there was some inconvenience from applications being made for certificates after they had examined, of which they had no previous knowledge, and which therefore they were not prepared to give. And so they determined amongst themselves in all cases to examine for and issue certificates to all children who could pass them.

Who had passed the statutory standard?—Yes. So I find it is the practice of the inspectors, by arrangement amongst themselves, to examine all the children in the fifth and sixth classes "for certificates.

What date was that?—I have nothing in black and white to show, but I understand it was some time in 1878 or 1879—1 think in the beginning of 1879.

Can you give the number of certificates granted in each year ?—Yes. In 1873 there were 2,334; in 1874, 3,879; in 1875, 4,508; in 1876, 3,410 ; in 1877, 5,039 ; in 1878, 2,919. The inspectors were dismissed for a time, and there were much fewer inspections. In 1879, when the new arrangements came in, 8,312; in 1880, 8,604; in 1881, 9,575, Total, 48,580.

Can you give the number, if any, of those who, having obtained those certificates, remained in the schools?—I am afraid I cannot give information on that point.

Could not that be got, because the State had accomplished its contract, so to speak, with those children ; they were educated as the State required, and 1 want to know how many of those children remained afterwards, receiving gratuitous education ?—We could get it by calling for a return from the teachers—the school records show the date at which a child gets a certificate, and the date at which he leaves the school.

Will you take a note of that and get it ?—Yes.

You remember in your previous examination you stated that you had no record of the number of children who have been educated up to the statutory standard ?—I am correcting that now. As I have stated, I am modifying my previous evidence in that respect.

Do your figures cover the entire period since the Education Act came in ?—Yes.

Have you materials in the office for furnishing the full statement?_

Yes ; the return is made up from the reports of the inspectors since the commencement of 1873. They have to make a return for each school of the number who have passed for the certificate.

I suppose the certificates lie in the office for people to call for them ? —No, the inspectors, immediately after they examine, fill up the certificates, and cither distribute them to the children themselves, or hand them to the teachers to give to the children.

On this point you wish to amend your evidence of last day (questions 262 and 263) ?—Yes ; I say that since 1878 we have issued certificates to all children ; prior to that we did not.

Previous to 1S78 you cannot say how many passed the standard ?—No.

I was also to give information with regard to the school census. I Stated that we did not lay any store by our school census, because, as a matter of fact, we know we have more children in the schools than our census returned. We know that we actually had, between the school ages of six and fifteen, in the day schools in 1880, 151,720 children. Our school census, taken two years ago, showed only 130,087 children. In two districts we got no census ; the returns were not satisfactory, and we refused to accept them, and got nothing, and had to make an estimate of the children in those districts. But making allowance for those, it would only add about a thousand children, so there would be a discrepancy still of 20,000 ; and, therefore, the school census, I say, was not reliable, and could not be quoted as a reliable thing. I have the full returns of the census here, if the commission wish for them.

Was there any other matter ?—I wish to modify my evidence of last week with regard to the architect’s staff. I said then that when we had a heavy expenditure in 1S77-8, the staff was pretty much the same as it is now. I was not quite correct in that statement. There have been larger changes in the staff than I thought there had been. It is not that we have dismissed officers from the staff, but a few have dropped out, and their places have not been filled up. The staff of 1877-8, compared with 1882, shows four more draftsmen and two clerks over and above the present staff, and also ten clerks of works. The next large expenditure was in the year 1874-5, aud then our office staff was no larger than at present, but we had four clerks of works more than we have now.

A question was also asked you about the police ?—Yes ; I said that the enquiries by the police were not discontinued on account of the Chief Commissioner taking exception to them. That was correct, but I think I know the circumstances at which the question was directed. The Chief Commissioner did object to the police being engaged to obtain evidence in support of prosecutions of parents, and for that reason truant officers were appointed to look up the cases and get the evidence. There was one more point. I was asked about the circumstances of the parents ; our school registers contain a column giving the occupation of the parent or guardian, but I thought we had abolished the column. We have cut out some columns to make way for others, but on looking I see that particular item is still there, so that a general return could be got from all the schools of the occupation of the parents of the children.

Would that throw heavy work upon the department?—It would be a heavy work upon the teachers. We should require some sort of classification.

There was one other point, to enquire of the inspectors whether they have reported any cases of immorality in the night schools ?—Yes, I have a note of that. The answer is, none. I made inquiry of the inspectors.

So that the department is not really aware that a case of immorality ever occurred ?—No.

That same answer would apply to the day schools ?—Oh no.

Have you had any cases of immorality charged against the teachers or scholars in the day schools?—I understand there have been a few of late. I dare say there may have been possibly a dozen since the department was formed.

Would you give a return of all the cases reported to the department of immorality or indecency on the part of teachers or scholars in the schools throughout the colony, giving all particulars, the name of the school, names of the persons, the sex of the children, the steps taken by the department, and with what result in each case ? —I will do so.

Are the inspectors expected to report on matters of that kind—have they any instructions to report, where they hear of cases of immorality ? —JNo special instructions, but they would no doubt report immediately. Special instruction is not necessary in such a thing. When we hear of it, we refer it to the inspector at once to inquire into.

And hearing of it, are they not necessarily obliged to do it ?—Yes; but it would not be fair to infer that the schools are in such a state that we would need to specially instruct our inspectors to report on that.

It would be naturally their duty, you would say 1—Undoubtedly.

Mr. Pearson, in his report (page 8), says that efficiency ought to be attained at the age of twelve years ; do you agree with that ?—I think Mr. Pearson says that, with an improved system of attendance, that with an increased number of attendances, the children could attain the standard by that age. He thinks the attendances should be about 45, instead of 30.

Yes, I see he says that; and assuming that, that the efficiency ought to be attained by twelve, do you agree with that?—Yes; I say from twelve to thirteen.

What is the number of days’ attendance in a quarter which, from your experience, you would require in a State school ; the present number is 30, Mr, Pearson proposes 50?—I would rather work gradually. We find it pretty hard to get the 30 ; but I do not think it would be any harm to ask for 40.

Do you mean compulsory attendance ?—Yes.

Mr. Pearson proposes to increase it according to the ages—in fact, have a sliding scale; would you propose such a system as that; that is, with the view, apparently in reference to the ages of the children, of not interfering with the labour that might be beneficial to the parents ; it is a system of some elaboration ; have you considered it?—No ; I am not prepared to say.

Would you have one standard for all, or a sliding scale according to age?—I cannot say I have given that particular question sufficient consideration.

What is your own opinion as to the effect of children being sent to schools at’ an earlier age thau six years ?—I should certainly prefer having them before six, though not necessarily quite so young as they come at present.

Can you give a return of what has become of the exhibitioners ?—Yes ;

I can give a return of all of them as to whether they went through their full course, and, if not, for what reason. In some cases the exhibitions are forfeited for failure to pass the necessary examinations. I will get that return.

Will you get also what the exhibitioners have done at the University— the honours they have carried off, and so on.—whether they have taken a degree, and with honours or not?—Yes, I will supply that ; do you want it from the very first ?

Yes.—I will get that.

Have you any record of the occupations which are entered upon by State school pupils after they leave the schools?—No ; we know nothing about them after they leave. By the way, I think Mr. Elkington, in his report one year, quoted some information he had obtained with regard to the future career of boys at one particular school. That is the only instance, I think ; that is in one of our annual reports. We have no official records.

It is not attempted to be got by the department ?—No.

About the exhibitions : do the examinations show that a much larger number of pupils are worthy than there are exhibitions to allot, where, for instance, the result of an examination may be to bracket a considerable number of pupils, for only one of whom an exhibition may be available—are there a greater number of people educated to the degree which would render them worthy of exhibitions than there are exhibitions to award ?—We have never adopted a particular standard up to which a competitor should come in order to qualify him ; it has been purely competitive. The eight best lads have been selected, and no particular standard adopted as one by passing which pupils should get exhibitions.

In the course of the honour or competitive examinations the information must have been gained by the examiners that a large number of names were worthy of exhibitions, or, it may be, that the number the State provides is a reasonable number for the requirements of the school system ?—No ; the examiners have confined themselves to this : they have to select the eight best candidates.

Is there the information in the department that there are a much larger number of deserving persons than there are exhibitions to award — with a view to recommending an increased number of exhibitions?—I think the later work is better than before ; some of those unsuccessful in the later years, would; have been successful in the earlier years. In other words, they have got more marks, and if we had adopted the marks gained by those in the earlier years, several more would have gained exhibitions in the later years.

You have no standard ?— No.

Would a greater number of exhibitions induce a greater number of competitors?—I think it would, because we have the fact that when this last year there were eleven, the number of candidates has been larger.

And virtually the standard has been higher p—-Yes.

The work has been of a higher character ?—-Yes.

As a matter of fact, you have no standard ?—No, because, as it is, eight lads might do very badly ; but if they were the best, they would get the exhibitions,

Irrespective of all qualifications ?—Yes ; we could not refuse to award the exhibitions.

What the scholars did afterwards would show whether they were worthy of exhibitions ?—Yes.

Have you any record which would show how pupils at the State schools are maintained—whether by the parents or by their own exertions?—No; we have no record of that. Of course, it is constantly pleaded as an excuse, by many who do not attend the thirty days in the quarter, that they are at work.

Is there any arrangement for the pupils attending schools of mines or technological schools ?—No ; we have no arrangement for that.

What books are prescribed for reading at the State schools?—The Royal Readers, or books of equivalent difficulty are allowed to be used. As a matter of fact, I do not think any others are used but the Royal Readers and certain books on English History.

Who regulates the selection of the books ?—If an inspector found other books than the Royal Readers in the schools, he would state what equivalent value he would give as compared with the Royal Readers.

Who prescribed the Royal Readers ?—The Minister, upon the recommendation of the permanent head of the department.

When was the Royal Reader adopted as the book ?—I fancy about

1876.

What was the book before that ?—The Irish National series.

Was any reason given for the change?—I forget the particular grounds upon which the Royal Reader was taken up, but there was a general feeling that we might improve on the Irish National books.

Perhaps you could find that out?—Yes.

By whom, and on what principle, are the books selected ?—We refer it to the senior inspectors, when any book is submitted for use in our State schools, for their opinion.

Who submits the book?—Anybody who wants to sell one. We constantly have books submitted to be placed on our list. You are looking rather at the reading books. I am thinking now of treatises on arithrfietic, and so on.

You will understand we want to know upon what principle the change was made, and upon what principle books are selected, and by whom ? —Yes.

Is there some principle on which you act in selecting books ?—We do not adopt a book unless it is recommended by the senior inspectors.

It would seem as if you may be influenced by a bookseller or publisher, who should have no power to submit them ?—It is frequently the people who have the books to sell who take the initiative.

The agent of Collins and Co. several times recommended books ?— Yes, they frequently submit books. They are the contractors for the supply of our books, and if they have what they consider a good book they submit it, and request that it be placed on our list.

And that is not accepted unless the inspectors recommend it ?—No.

Will you give the terms on which the booksellers give you books ; for instance, you obtained the National school books at a certain rate from Thom, of Dublin ?—Yes.

When the change was made, was it not only in the interest of the scholars, with regard to the literary contents of those books, but also with regard to price?—The new books were much more expensive than the old ones, so we were guided really by the contents. There was no saving in price ; it was the other way.

As to the statutory standard of education—the compulsory portion— reading, writing, and arithmetic, to the satisfaction of the inspectors of schools—I want to know whether there is any general standard applied by all the inspectors, so as to secure uniformity?—Yes, they have instructions.

What is your rule as to that ?—It was originally recommended by the then Inspector-General, and his recommendation was adopted, as to what should be the interpretation put on the standard, and that was issued to all the inspectors for their guidance.

It is important for the Commission to know that ?—That forms at present a part of our confidential instructions to the inspectors.

You can let us have that, under cover?—Yes. The regulations give the general interpretation.

Those are issued under the Governor-in-Council ?—Yes. The interpretation of the standard of education is given in our general regulations, which I have here ; but the further detailed instructions are contained in confidential instructions to the inspectors.

You refer there to the regulations passed by the Governor-in-Council ?— Yes, the rules and regulations of the department; the regulation as to the standard is number 8, on page 9.

Would there be any objection to your furnishing an idex of all your private and confidential instructions ?—The only confidential instructions are with reference to examinations in the schools, on which a great part of the payment of the teachers depends, so that it is necessary that those examinations should be on one system. To secure that, we issue those instructions to all inspectors.

The point is : there are ascertain number of printed rules and other sets of private instructions ?—They have only reference to this one subject.

Will you give an index of those, showing what they are, for the private use of the Commission ?—Yes.

Is there any teaching at the State schools of the elementary knowledge of the arts and sciences, in their application to agriculture, horticulture, manufactures, handicrafts, or trades?—Nothing more than is contained in the ordinary reading books and in a little book we issued on agriculture, a pamphlet by a Mr. Luplau, of which we sent a copy to all the schools ; but I have not heard that it is in use in any of the schools.

Suppose in a school no assistant teacher were competent, is it permissible on the part of the head teacher to create a private school for matriculation and exhibition purposes by employing external teachers and paying them by the fees received from the State school children ? —I can answer that in this way : that when no teacher is competent to teach extra subjects, it is allowable for the head teacher to employ an outsider to teach.

So he may employ strangers and keep up a private school for exhibitions ?—I do not understand your calling it a private school, because this instruction is given outside the four hours he must give to free instruction.

Those persons employed from outside share the benefit with him ?— Yes.

They would be outside the control of the department ?—Yes.

Have you had any complaints from private schools under that head ? —I do not really know. If you can give me a hint as to where one might have come from, I could ascertain, but I do not remember one.

One shilling is the maximum charge for an extra subject?—Yes. I seem now to have a notion that there was a complaint once, but I cannot remember where it came from ; I will have it looked up by next meeting.

When students at the State schools pay for extra or matriculation subjects—latin, mathematics, and the like—what is the provision for seeing that they get the worth of their money—that the teachers really teach them those subjects for which they pay ?—They are examined by the inspectors at every inspection ; the inspectors are instructed at every inspection to report on the extra subjects.

What is the provision for seeing that those who undertake the subjects really devote the time for which they receive! those fees ?—The head teacher would see to that.

Who looks after the head teacher. I know a case where the head teacher received a large proportion of the fees for latin, mathematics, and the like, and neglected to teach them—how is such a case as that looked to by the department?—The only check would be the parents themselves, and the inspector at his periodical visits.

Then there is no check on the head teacher, who may receive a large quantity of fees for those extra subjects, to see that he is actually doing the work?—I consider the inspector is a check.

He will find whether the students know the subject or not, but that does not meet the point as to whether there is any series of returns or supervision to ensure that the bead teacher or other receiving the fees really devotes the regulation time to it?—No; those are the only two checks. The parents would complain, or the inspector report it. Beyond that there is no check.

Is there any security that all who take up extra subjects will be inspected by the inspector—could not the head teacher keep some back if he thought fit?—Yes, he might. The inspector would call fpr a class on any particular subject; but I do not see that the inspector could tell exactly how many ought to be presented to him.

Take it this way : Suppose a large number in a given school enter for mathematics and latin, and the head teacher, receiving fees from all, were to devote his time to a fraction of the number who alone appearand do well before the inspector, while the others get little or no instruction— is there any provision for seeing that those who pay for instruction get it—any check apart from the examination ?—No, there is nothing further than the examination.

Have you in the records of your department a list of the schools which have exhibition or matriculation classes, the number of scholars taught from year to year in those classes, and by whom ?—No, we have nothing more than a list of the schools in which extra subjects are taught, and the masters by whom they are taught.

Even though those masters do not belong to the Department of Education ?—Yes.

If you choose to call it a distinct private school. I mean a very numerous class out of the State school learning extra subjects, and those subjects taught by persons who have no connection whatever with the Education Department, and that thereby the income of that master would be very materially increased, and yet the department itself would not have that effective control over the head master and those external masters that you have in regard to the master and his assistants within the school proper ?—If I understand your question, you mean the master might have a large class formed out of his State school, being pupils of the school, who would be taught extra subjects by outside masters altogether, with whom he might have an arrangement that he should receive a certain proportion of the fees for teaching those extra subjects —that is quite possible.

Do you know whether it is the case ?—No, I do not.

You are not aware how far they supplement their income by that process?—We know the whole amount received for extra subjects, and we know who gets it ; so we should know.

You could give a return of that?—Yes.

Even in cases where the master employs others outside to impart the knowledge ?—Yes.

Apart from the pecuniary point, as far as I understand, it is this way : Certain extra subjects are taught the State School pupils whilst they are, as it were, under the acjis of the State by persons over whom the State has no control ?—That is correct.

In some instances the higher education is given by'persons over’whom the State has no control ?—Yes, except the examination.

That is supervision merely as to the result?—Yes.

Will you give a return of the children taught by persons whom we will call strangers to the department ?—Yes,

As a matter of fact, has the master or mistress of a State school any recognised control over the pupils out of school?—Yes, certainly. We hold the teachers responsible for the supervision of the children between the morning and afternoon school meetings.

I meant the question to go further than that, A child is attached to the school as a pupil, and the esprit decorps is concerned in the conduct of scholars, in and out of school—has the master or mistress any control out of school ?—Not beyond the school premises. From the time he comes to the ground in the morning till he leaves.

Are those persons who teach the extra subjects necessarily certificated teachers ?—No, they are not. .

Will you explain the present system of discipline in the State schools? —The matter of punishment rests with the teachers. It is restricted by a circular, which permits only the head teacher, and such assistant teacher as he authorises, to inflict corporal punishment. We do not interfere in the matter of discipline beyond that.

Is there any moral teaching in any of the schools?—Nothing special, beyond what is contained in the reading lessons, and the lessons that may be got from them ; no special moral lessons beyond that.

There is no moral standard taught?—No, there is nothing special.

Are they, for instance, at liberty to tell a lie without reproof ?—We leave that to the teacher ; he would certainly punish them.

On what ground would it be said that is wrong—what is the moral standard ?—I think according to the standard of the morality of the teachers generally.

Then the morality standard of the school depends upon the moral standard of the particular teacher ?—Yres ; I think that is correct.

A moral standard forms the basis of morality, and it may be based on various systems, say from Aristotle down to Adam Smith, and it differs in different countries ; that is why I want to know whether you have any standard? —No.

No standard based on any religious teaching ?—No.

Do you consider that there is a good moral tone running through the reading books in the schools ?—Unquestionably there is.

Do the children get any marks for good and bad conduct in the State schools, as they do in private schools ?—That depends on the teachers ; some of the teachers keep records of that, and give marks. We supply no special information, or documents or cards bearing on it,

Is there any system of removal from one school to another for the children ?—No ; no restriction is placed on removal from one school to another.

So, in fact, if a child were reproved for telling a lie, he might take offence at it and go to another school ?—He could.

Or, in fact, for any other deflection from morality he could go away. You have nothing in the nature of a bene decessit on a child going away ? —No.

Would a teacher be at liberty to say, “it is sinful to tell a lie ” ?_ves:

that is not debarred in our lesson books. I refer to our reading books being called “Godless.”

Will you let the Commission have a few copies of each of the books used ? —Yes.

If the number of scholars fall off a quarter or a half in any school, is the teaching staff reduced correspondingly and immediately ?—Yes, as soon as we can do it. The regulation provides that it shall be reduced, and we do it as soon as we can find an opportunity of disposing of the teacher.

Would your school census be simplified if each municipality, shire, city, borough and town were divided into school district b so that a’school district or group should be co-terminous with a given municipality—that is, with no overlapping?—That is the case at present ; it is only that the larger shires and boroughs have been cut up ; for instance, the city of Melbourne is divided into wards.

Could not you break up your districts to suit those requirements ; and, if that were done, could not you use the municipal rate-collecting machinery for collecting your census?—The objection is that a district would be larger than one board of advice could attend to. The city of Melbourne is more than one board of advice can attend to. It has, because of its size, been divided into wards ; and so with the shires. ’

Would you recommend any alteration in the duties and powers of boards of advice ?- -No, I have no suggestion to make as to that.

Do the teachers receive any payment for the use of their school buildings ?—We allow half-a-crown for each meeting of the board in the school, for which the teacher is expected to provide fires where necessary and pens, ink, and paper.

Do you consider that the boards of advice are of any great utility to the Education Department ?—Yes, I think they may be, and they are in some cases of great utility. They assist us very much in some things— in the selection of sites for schools and the supervision they exercise over the schools.

You mean where the boards of advice act spiritedly and look after their duties?—Y"es.

As a rule do they do that ?—Some do, and a great many do not.

Do the majority carry out the object for which they were appointed with zeal and energy ?—A very fair proportion do.

Do you mean the majority by that ?—I think the majority. I was going to say that the central department has so little immediate supervision over the teachers ; the inspector visits, say twice a year, and beyond that there is no immediate supervision. Now I think the boards of advice are very useful in that way, by exercising constant supervision.

No doubt if they do it, but do you know by your knowledge that they do exercise that supervision that is so desirable ?— Yes ; I think I may safely say they do.

From your experience, would you tell the Commission what size school you consider the most easily and efficiently worked ?—That is to say, what is the maximum number that it is advisable to teach in any one building ?

lies ?—I think a thousand may very well be taught in one building and I do not think that is too many.

What system do you adopt to ascertain if any buildings are requisite ; do you wait till you are moved by outside authority or do you do it yourselves—I refer to new school buildings ?—We may be moved either by the board of advice or the school inspector, or again by the teacher. I am speaking of an existing school wanting a new building.

No, I referred rather to a new school building, for new accommodation in a district?—We are always moved either by individuals or by the board of advice. In one or two cases, I think inspectors have given us the first intimation of the rvantof a school ; but, as a rule, the application always comes from the people themselves, or the board of advice for the district.

If you had a system of districts by which scholars were obliged to attend the nearest school, you would know exactly when you had a sound reason for erecting a new building ?—We could get that through the board of advice.

Now somebody interested in getting the building may move you, but if you knew the school resources of the district, and the number each’school will contain, you would know then the number you must provide accommodation for ?—Yes.

Suppose children were obliged to attend the nearest school ?—The district would generally cover a large area; the best position for a new building would require a further enquiry.

What system is adopted in the choice of materials for the school building; who moves in that ?—The first recommendation we <mt on the subject is from the inspector. We always get a report from the inspector, who has to furnish answers on several points, on the best material to use ; if bricks are plentiful in the locality, or stone and wood and so on.

Do you consult the municipalities as to new buildings ?—No.

lrou receive suggestions from the boards of advice ?—Certainly.

As to the nature of the building ?—Yes.

You would erect a wooden building where the population was not fixed ?—Exactly ; we are guided in that w-ay.

If the population leaves a particular district, and you had a school

building erected, what becomes of the building ; what is done with it_

that has occurred?—Yes. Sometimes we moVe the buildin". If it is not movable, not worth moving, we see if it is required by^ any other Government department, and if not, we sell it.

About £11,000 a year is spent for repairs ?—Yes.

As the buildings get old, the amount of repairs necessarily will increase ; have you considered at all whether there will be a savin» of expense in every way, assuming it was adopted, if the school buildtn» when built, were handed over to the municipality, and the municipalities undertake the repairs, would not that be a great saving ?—No doubt it would be a saving. We have never had the question raised "at all.

Suppose the State erect the building and hand it over to the municipality interested, and they keep up the repairs—that would be a

saving ?—It would ; but we would not get much assistance in that way, We have tried to get assistance in planting or in fencing, and we cannot get local assistance in those things.

Have you found people in the neighborhood disposed to give you sites for schools ?—Selectors will generally give up a piece for a trifling consideration. Sometimes they give it altogether free of cost.

Where the land is their own ?—Occasionally.

Have you any system of insurance ?—No, we do not insure at all.

Has the department sustained many losses by fire, as a matter of fact ? —No, very few. Our losses would not come up to the amount for insurance.

Do you require a teacher to reside within a certain distance ?—No ; when we do not provide a residence, he may live where he likes.

Would it not be an advantage to make him live amongst the children ? — I am afraid the teacher would not think so.

Would not it be a benefit to the State ?—I do not know whether it would be a fair requirement.

Apart from that point, would it be beneficial for the children ?—I can hardly say that. At present he has to supervise them on the school premises, and I do not know whether he could do more by living in the immediate neighborhood of the school.

Is there any arrangement for the separation of the scholars of eight or ten, or any given age, from those over that age ?—No.    The infant class

may contain elder children ; it depends upon their attainments.

So those little children of three and four might have scholars of fifteen amongst them ?—No, hardly that. Children of eight or nine might be amongst them. We have not children so old as fifteen of such absolute ignorance that they would be put in the same classes as children of three or four.

They might be in the same room ?—Yes ; iu the small schools all the classes are in the same.

In that case the children of three and four are tumbled about with the others?—No, they are not tumbled about ; they are taught in the same room.

Is there any accommodation for children being able to eat theirdinner in the school buildings ?—Yes, they have always been allowed to have the use ci the schoolroom between hours. Sometimes we have complaints that the teacher locks up and turns them out, and we immediately request him to admit them.

Would you not do more than desire him to do it ?—Yes, instruct him, in fact.

Have any of the school buildings any accommodation provided for chemical or mechanical appliances, smithies and carpenters’ benches?— One school in Geelong has, I know, a good deal of chemical apparatus, but it is the property of the head master. It depends upon the teacher we have not supplied auything.

Do you keep any record of the use of school buildings for other purposes than .State school purposes ?—Yes.

Have you any detailed record of applications for the use of school buildings for the purposes of religious instruction out of school hours ?— Yes ; 1 think we generally publish it annually. There is a paragraph iu our last report that bears on that :—“The increased facilities afforded to all denominations for holding public worship on Sundays in State school buildings have been largely availed of during the year. In 1879, only 39 distinct congregations assembled in State schoolrooms for religious worship, whereas the use of 365 schools was granted for the purpose iu 1880. In 30 schools religious services have been conducted on week-day evenings, and 167 schoolrooms have been used for Sunday schools.”

Can you give a return of all the instances for the last year where the State school buildings have been used for other than school purposes, and at whose instance, as far as you can ?—Ye3, in all cases it is done by the board of advice, with the consent of the Minister; that is, they are always the two consenting parties.

Beyond that, you would not know who set it in motion ?—The return will speak for itself on that point.

PUNISHMENT.

As a rule, young pupil-teachers are apt to punish indiscriminately. They will often inflict punishment when a child has not intentionally donwrong. A child becomes hardened by being frequently punished for mere trifles, and when more serious occasion for punishment occurs, it has not the effect upon him that it might have if he were not so used to it. It is quite possible to make a bad boy worse by the indiscriminate use of punishment. Corporal punishment is a thing which I have always disallowed. It is, I think, entirely out of place iu an infaut school, looks very ugly and masculine in a girls’, and the less it is had recourse to in a boys’ the better. Young pupil-teachers are apt to consider a small stick or cane a very necessary appendage. They often seem to depend upon it for enforcing order and obedience. This is a great mistake. You will ask: ‘ How are naughty children to be managed without a cane?’ 1 answer : ' Many nays ; and try all ways before that.’ Many children are the unlucky subjects of much corporal punishment at home, and in such cases, which may be easily ascertained, it will be wise to adopt a very different course at school. Persuasion will go a long way—friendliness farther still. By this I mean giving a child to feel that you are interested in him—taking nolice of all he says or does that may give you the slightest chance of praising him. A little praise goes a long way with a child who seldom hears anything of himself, but that he is a bad boy.’ He begins to appreciate these tiny bits of good iu himself, and becomes anxious for more. Much depends upon your patient dealing with such children. Don’t give them up all at once as incorrigible. I can call to mind several very unmanageable boys whom the most patient teachers in the school have prouounced “ incurable,” and of whom I have myself almost despaired. But as you will wonder if such were ever cured, and how, allowr me to say they were cured—not by putting them in some dark place, or to stand all day in some unnatural position, but by 'putting them in office ! If you have such an ungovernable boy in your class, make him your “ assistant.” Give him charge of books, slates, or anything else to help you. Keep him near you in the class, and require his assistance often, if only to pick up your chalk or clean the blackboard. Let him feel that he has your confidence, and that you depend upon him for all these little things. By and by he will begin to feel himself so important and so near to you, in his importance, that he will not condescend to be as troublesome to you as he was. Then, when Friday evening comes, thank him for havingjhelped you so nicely all the week; and when Monday morning comes, you will soon see that he has not forgotten how much you seemed to value his help.

Again, if a child is generally tiresome, and not inclined to please you, don’t let him sec that you think so ; but notice any good quality he may have rather than the bad ones. Look for the good that is in him. Remember that

“ There is in every human heart Some not completely barren part,

Where seeds of love and truth might grow,

And flowers of generous virtue blow ;

To plant, to watch, to water there—

This be your duty—th is your care.

ORDER.

Order must be school’s first law ; for without it there is no good to be accomplished. A disorderly school is neither creditable nor comfortable. It is impossible either to learn or to impart knowledge where confusion reigns. The attempt to do so is a very common fault amongst young pupil-teachers. Many attempt to maintain order by making a great noise themselves—constantly calling out to John Smith or Sarah Brown, or some other unlucky child to ‘ be quiet.’ Others are tempted to use their hands as a means of securing order. As a rule, this is strictly forbidden by head teachers, and rightly so too. It proves a source of annoyance to them in the complaints they receive from parents. If you will only learn to use them rightly, you will find eyes and ears far more powerful agents in maintaining order than either tongue or hands. Much time is wasted through disorder—therefore order must be literally infused into everything connected with school life. Be orderly yourselves. Train your class to orderly habits. Caps, books, slates, &c. must all be kept in order. Don’t pass unnoticed a slovenly boy or girl with a carelessly written home-lesson, or a torn and dirty book. Bring them conspicuously before the class, so as to make them feel ashamed. Insist upon orderly movements in changing lessons. Your class may be in good order during one lesson, but if allowed to make a disorderly change you will find great difficulty iu regaining order for the next. One of the things most conducive to order is hard work. Children must be doing something, even if it be mischief. But if they are kept hard at work there will not be time for anything else. Keep them constantly employed, so that when it is time to go home they may march out with an air of relief at having finished their work, and of satisfaction at having done so much. Attention to these little matters will help you on a long way towards that ‘ order,’ without which you can never become a successful teacher.

f^EMALE Assistant, large school, Sandhurst, desires immediate exchange, Assistant or Head Teacher. Seaside. Salary .i'll t. Address, ‘‘Immediate, care Mrs. Thompson, Hargrave-street, Sandhurst.”

HEAD TEACHER, of 20 X 30 school, on the coast, in a healthy locality rising district, wishes to exchange. Salary (with Post-oflice) £174. Teachers’ quarters attached. Address, “Dominic, P.O., Bimgurra.”

HEAD TEACHER, 50 to 75, results 94, six miles from Ballarat, on Alain Road, will exchange Teacher, higher allotment. Address, “ A. Z., care R. T. Ahíle, bookseller, Ballarat,”    ___________________

HEAD TEACHER, 50 X 75, railway township, Post and Telegraph Office, desires exchange. Address, “Theta, care of Air. Souter, bookseller, Sandhurst.”_______

HEAD TEACHER, 30 X 60 school, Echuca District, would exchange with . another of similar allotment. Post-oflice (daily mail); workmistress vacant. Good quarters. Geelong district preferred. Address, “ Magister, P.O., Echuca.”    ____

READ TEACHER, near town (20 x 30) would exchange with any Assistant, Melbourne and Suburbs. Address, “ Urgent, Mr. Bruce, stationer Ehzabeth-street.”_________

HEAD TEACHER, 20 X 30 (good), results 60. Post Office, NorthWestern District, would exchange with Assistant, or lower allotment near town. Address, “H. Y., Narrewillock P.O.”

( j VEACHER, Gippsland, allotment under 20, post oilice attached, six miles JL from railway station, would exchange for higher allotment, near Melbourne, Geelong, or Ballarat. Apply, “ R. G.,” Schoolmaster Office.

ANTED for Lad (14) situation as Pupil Teacher in State School. Passed all subjects of Gth class, June, 1882. Reference, Head-teacher, George Street State School, Eitzroy.” ________

I WARREN BALL’S “Hints to Candidates for Teachers’ and Matricula-« tion Examinations,” Is.; posted, la. Id. Mullen, Melbourne, _

CCANDIDATES for EXAMINATIONS prepared by correspondence or J otherwise. I. Warren Ball, South Yarra.

rp ATE’S PARCELS POST EXPRESS

FIXED PRICE.

NO EXTRAS. NO TROUBLE.


Delivery to door at ar y addr« s in

21b s. d.

4ib. 61b. s. d. s. d-

101b. s. d.

201b.

s d.

Great Britain

4 6

6 0 7 G

10 014 0

Continent of Europe, America, & Canada

7 G

9 612 0

16 0

21 0

S y d n e y, H chart, Launceston

2 G

o

co

o

co

4 0

G 0

N ew Zealand Ports (except West Coast) Adelaide, Brisbane.

4 0

4 g| 5 0

5 6

7 G

No further charge whatever. Very small increase for heavier weights. Delivery at country addresses in Australia, inland carriage only added.


Receiving office—

FREDERICK TATE, 13 Market-st., Melbourne TATE’« PARC E L S P OST EXPRESS'. DEL1YLRT at DOOR any address in the world. EITHER to or from Britain from 4s. 6d. TO or from other Australian ports from 2s. 6d. M) further OIIAK.GK whatever. No trouble. Any SIZE, weight, or shape.

EVERY possible ASSISTANCE afforded“ ' INQUIRIES plainly answered.

Henceforth Parcels handed to W. R. SUTTON’S Branches, every town in Great Britain, delivered in Melbourne at nearly similar rates.

Head Office :—

FREDERICK TATE, CUSTOMS AGENT, Sec., 13 Market-street, Melbourne.


TpXAMINATIONS.—UNIVERSITY

^    and DEPARTMENTAL.

TUITION in CLASS by Correspondence or otherwise. Terms, &c.,

JAMES L. ROBERTSON, B.A. (Melbourne). 71 Clarendon Street, Emerald Hill.


USE

Schurer’s Champion Ink Powder.

50% CHEAPER THAN ANY OTHER INK POWDER.

And Unsurpassed in Qual.ty.


To be obtained from all Booksellers, Stationers, and Storekeepers throughout the Colonies, Sole Wholesale Agents for Australasia;

SCHÜTZE, STEFFENS & CO., 18 Collws-st East, Melbourne.

Apply for Samples and Directions.


NOW READ Y.

]y£ILTON PARSED.

Price 2s.

By J. J. BURSTON,

(Author of “State School Arithmetic”).

Also Ready, the Fourth Edition of the

STATE SCHOOL ARITHMETIC.

By

JOHN J. BURSTON,

North Sandhurst State School.


s.


MULLEN’S


New Classified Catalogue of School, College, and Technical

EDUCATIONAL WORKS

May be had gratis on application, or posted on receipt of address.


SAMUEL MULLEN,

Wholesale & Retail Bookseller & Stationer, 29 & 31 COLLINS ST. E., MELBOURNE.


TAMES C L E Z Y, M. A.

5 GORE STREET,

Prepares for Matriculation, Pass or Honours,and otho University Examinations. Since February, 1874 One EUmdred and Fifteen Pupils have passed various Examinations, ranging from Civil Service to the fina Examination for B.A. Degree.

In October Term, 1881, Four passed Matriculation, including Civil Service. Ons was first of the first-class in Greek and Latin Honours. One (a lady) obtained a speond class in Greek and Latin. Two passed for first year LL.B.

N.B.—Students after Matriculating, and before proceeding to the University, should read the First A ear’s work of their course ; otherwise the Lectures are of little benefit to them, and their chance of passmg at the end of their first year is very small.


IMPERIAL REVIEW,

Quarterly, 2s. Four Numbers, Subscription, 7s. (id., including postage anywhere.


Publishers :

ALEX. M ‘KIN LEY & CO., Cl Queen Street, Melbourne.


(J c. EXAMINATION.

TUITION BY CORRESPONDENCE.


MR. THOMAS BOARDMAN, First-class Honor-man of the Denominational School Board, Prepares Teachers for the Certificate Examination by Correspondence. Terms moderate.

Address—•

4 5 PRINCES STREET, CARLTON


N PREPARATION.


I.    Mr. Morris’s Edition of Dryden’s Annus Mirabilis and Hazlitt’s Selected Essays, with notes.

II.    Mr. Clezy’s Edition of Virgil’s JEneid, Book I., and Caisar’s Gallic War, Book I., with notes.

* The above texts are prescribed for the Matriculation Examinations at the Melbourne University.

SAMUEL MULLEN, Publisher, Melbourne.


Price One Shilling.

By Post—In Victoria, Is. Gd. ; Out of Victoria, 2s


J^EPARTMENTAL EXAMINATION OF TEACHERS.

TEACHERS’ GUIDE

TO

AUSTRALASIAN EXAMINATIONS. 108 PAGES WITH STIFF COVER.


MR. II.

M'KTNLEY

S

OLICITOR,

CONVEYANCER,

And

PROCTOR,

GO

COLLINS

STREET WEST.


OTATE SCHOOL, No. 17 7,

TRADES’ HALL, LYGON-ST., CARLTON.


MR. SERGEANT,

(Late District Inspector of Schools.) Conducts Classes for Certificates of Competency on Saturday mornings in the abovenamed school. W ork will commence on Saturday, 22nd instant. • At the examination in December last thirteen passed, though only six months under Mr. Sergeant’s tuition’.

CANDIDATES ARE ALSO PREPARED BY CORRESPONDENCE.


The above book contains the Programmes and Examination Papers of December, 1877, of all the colonies, and is reduced to the low price of

ONE SHILLING.

By Post—In Victoria, Is. 6d.; Out of Victoria, 2s

ALEX. M‘KINLEY & CO., PRINTERS AND PUBLISHERS, 61 QUEEN STREET,

MELBOURNE.


Laws relating to newspapers.

Judging from a recent case, a great many newspaper subscribers are ignorant of the law bearing upon the subject. For their benefit we make the following Statutory Extracts, which wo commend them to

“READ, MARK, and LEARN:”—

“ Any person who takes a paper regularly from a post-ofti e — whether directed to his name or another’s, or, whether he has subscribed or not—is responsible for the payment.” “ If a person orders his paper to be discontinued, he must pay all ARREARS, or the publisher may continue to send it until payment is made, and collect the whole amount whether the paper is taken from the offico or not-.” “ The Courts have decided that in refusing to take newspapers and periodicals from the post-office, or removing and leaving them uncalled for, is prima facie evidence of intentional fraud.”

The Trade Protection Society sued sixteen of their subscribers for arrears of subscription to their paper. The defendants admitted that they were subscribers, hut refu ed to pay the amount sued for, alleging by their attorney as a defence that the paper was sent to them after they had given notice to the proprietors to discontinue sending same to them. The plaintiffs, by their attorney, contended that the defendants were liable, as at the time the notice -was sent the defendants were in arrears in their subscriptions, and that his client could, in face of that notice, continue sending the paper until all arrears were paid. In support of this view he referred the Court to several authorities on this matter. The magistrate after hearing the attorneys on both sides, sustained the view of tne plaintiff’s attorney, and gave the verdict for the amount claimed in each case, together with witnesses’ expenses.

Persons having papcis addressed to them which they have not ordered, should decline te receive them, and thus avoid the legal responsibility.

Printed and Published by Alex. M‘Kinley & Co., G1 Queenstreet, Melbourne, under the auspices of the Victorian Teachers Union.


----——-

f SUBSCRIPTION

Yol. IV., No. 37.

JULY, 1882.

| Yearly, Gs. Gd.; Half-yearly, 3s, Gd. ( Payable in Advance.

BLACK!E à SON’S SCHOOL SERIES.

Lady of the Lake, Canto I... .Scott.

The Armada, &c..........Macaulay.

The Prorhecy of Dante, C. I.

II...............................Byron.

L’Allegro and Id Pknse-

roso...........................Milton,

Essays (selected) ......Lord Bacon.

Prisoner of Chillon ......Byron.

The Fire Worshippers, Parts I.

II...............................Moore.

Ancient Mariner ......Coleridge.

Deserted Village.....Goldsmith.

Marmion, Canto VI. ..... Scott.


Arithmetic.

Thesb Arithmetics arc graded to meet the requirements of the Code, though they are not limited in scope to the rules named there.    ,

In large Schools it is essential not only to provide practice in each successive rule and stage of a rule, but to supply intelligent exercise for the brighter halt of the class, showing the application of the rules. That they provide this throughout the Course is one of the special features of these Books.

A careful study of the causes of failure in Arithmetic has suggested other special features, such as    .    .

The abundance of word EXERc ‘s, as a preparation for the dictation of sums by the Inspector.    .

The diversity of style in sett    the sums, as a means of preparing

scholars for any kind of test, and su    ing their interest much better than

long arrays of sums set to pattern.

The difficulties of eacli rule are introduced gradually, and a model example given at the head of each exercise.

Problems are given in abundance.

Instruction is given in the working of each rule for the higher stand ards, for too ofti-n the teacher has little time to spare for them; but in the lower classes, no attempt is made to do that which only a teacher can accomplish.

Easy Lessons in Fractions are given at the beginning of Book V., that the pupil may the more readily and thoroughly do his Practice and Proportion.

The Tables required by each Standard are given in advance, that the pupils may not be able to refer to them instead of learning them.

Answers have been thoroughly tested, and are believed to be entirely correct.

Standards I, II. HI-,............paper cover, 2d., cloth 3d. each.

Standard IV. (English Code)    „    3d.    „    4d.

Standard IV. (Scotch Code)    „    4d.    „    6d.

Standards V. VI................... „    4d.    „    5d. each.

Keys to Standards I. to VI., paper 3d. each, or bound together in cloth, Is

OPINIONS OF THE PEESS.

“ The set of Arithmetic books is well graduated, so that the scholar will pass easily and naturally from one book to the next. The examples are numerous, and stated so as to exercise the reflecting faculties of the pupils without perplexing them.”—Daily Deview.

Geography.

ELEMENTARY GEOGRAPHY.

By W. G. BAKER, Associate of King’s College, Lecturer at Cheltenham Training College.

PartI. For Standard II. Elementary notions of Geography. Paper 2d.; cloth, 3d.

Part II. For Standard III. England and Wales. Two coloured Maps.

Paper, 4d.; cloth 5d.    >

Part III. For Standard IV. Scotland, Ireland, and the Colonies. Paper, 6d.; cloth, 8d.

“ Air. Bakei has shown his mastery of the difficulties inherent to the teaching of small children in a science which is only just beginning to assume its proper position in schools. The pictures and diagrams are all to the point, and not

too complex.....If the succeeding numbers exhibit the same amount

of care and judgment in their compilation, Mr. Baker’s completed work must take high rank as an elementary treatise on geography.”—Public Opinion.

Third Edition—Now Ready.

A Manual of Method.

For Pupil-teachers and Assistant Teachers. Intended for the Government Inspected Schools of Great Britain and Ireland, and for the use of Students in Training Colleges, By Abr. Park,

F.R.G.S., F.E.I.S., See., Headmaster Albion Educational Institution, Ashton-under-Lyne. Interleaved with ruled paper. Third edition, foolscap 8vo, cloth, 2s.

OPINIONS.

No young teacher could puruse, and, above all, master, this really valuable little manual without receiving much mental stimulus and much wise direction in all that pertains to the difficult matter of school management. In fact, we do not know where else to find in so brief a space so much sound practical wisdom and suggestiveness on this subject.”—The Literary World.

Just Published. Price, 2s. Gd.

A History of the British Empire.

With numerous Pictorial Illustrations, Genealogical Tables, Maps, and Plans. By Edgar Sanderson, M.A., Late Scholar of Clare College, Cambridge. 444 pp., cloth, red edges, 2s. 6d.

The writer of this brief record of a great nation’s fortunes has aimed at pro» during a narrative that should be equally interesting and instructive. He had believed that it is possible, even in a summary perforce so short as this, to bs clear throughout, to be strong and vivid where force and livoliness are ncedee for a due effect, to rise, upon occasion, into eloquence, and to give to this com. pression of our country’s annals something of the heat and glow which must exist in all who, with sufficient knowledge, love her name and fame.

The convenience of not only the young student, but also of the generalreader, has been consulted in a close adherence to the order of timo in the nar ration of events, except so far as regards occurrences in Scotland before the union of the crowns, and also as concerns the history of British India. The treatment of this most important subject in the present work is, so far as the writer is aware, a new feature in a book of this kind. A short, separate account is given of the transactions, exploits, and events which have made India the greatest of dependencies attached to any empire in modern times. The reader is thus enabled to peruse, with ease and interest, in a continuous talc, that which it has hitherto been needful to pick out painfully and piecemeal from the history of divers periods and reigns.

A fuller account than has been usual, in books of this kind, has been given of the great Civil War of the seventeenth century, and of the Peninsular War.

The history of England is, above all, a history of a nation’s growth in constitutional freedom. To this grand subject special attention has boon paid in all its salient points.

It is hoped that the arrangement in paragraphs, with marginal notes of the most important matters ; the maps, plans, and illustrations ; the genealogical tables inserted in their proper places ; the list of chief events and documents with dates; and a full and accurate index, will prove of substantial service to the reader.

Religious and political bias has been scrupulously avoided throughout, and it is believed that hardly any proposition has been laid down, scarcely one view advanced that would not find a general adherence amongst fair-minded and educated people, of every shade of political and religious opinion.

School Classics.

Selections from Standard Authors, with Biographical Sketches and Explanatory Notes.

32 pages-, price in paper cover, 2 d. each ; in cloth, 3 d.

The Lay of the Last Minstrel

C. I..............................Scott

Cotter’s Saturday Night..Burns

The Village..................Crabbe

The Pleasures of Hope, Part

I.......... Campbell.

Essay on Bunyan ......Macaulay.

The Queen’s Wake............Hogg.

The Merchant of Venice, Acts

1 III. IV............Shakspeare.

The Traveller.........Goldsmith.

Evangeline 64 pp. (price

4d.) ............H. W. Longfellow.

THE PROLOGUE TO THE CANTERBURY TALES OF

Geoffrey Chaucer. The text collated with the seven oldest MSS., and a Life of the Author, Introductory Notices, Grammar, Critical and Explanatory Notes, and Index to Obsolete and Difficult Words. By E. F. Willoughby, M.D. Price Is. 6d.

PARADISE LOST. By John Milton. Book I., with Life of Milton and Prefotary and Explanatory Notes. By E. F. Willoughby,

M.D. Limp cloth, 10d.

Further Particulars and Catalogues from all Booksellers, or from their Melbourne House, Carrington

Place, off Bank Place, Collins «street West.


School gcjmrtnrmt.    j

ON THE GRAMMATICAL ERRORS OF MORELL.

By James J. Courtney, Garibaldi.

Page 73.—“Remote from towns he ran his godly race.”

In a paper on the above subject which you kindly inserted in your May number I expressed an opinion to the effect that the word remote is an attribute to a noun (place) which forms part of an extension. In your June number my analysis was criticised by Mr. De Mornay, who agrees with it, and by Mr. Gough, who also agrees with it so far as to admit that the construction is grammatical and the analysis correct, but who asserts the same of Morell’s treatment. But Morell’s interpretation being the more beautiful, -poetical, fyc,, pleases Mr. Gough better, and he adopts it for this reason.

I cannot agree with this free and easy way of dealing with the difficulty, and with your permission I shall give my reasons for thinking that Morell is wrong, not only in his construction, but that he is also wrong in not making his syntax agree with that construction.

Before referring to Morell’s treatment of this sentence, which I consider involves not merely a technical but a fundamental error, I wish to show that in his “Table of the Component Parts of a Sentence,” page 70, the term participle should appear along with adverbs in the division, “Mod. Term. 1st degree,” and that the term participial phrase should appear in addition to prepositional phrase as a Mod. term, of the 2nd degree, while in bistable of parsing (page 110) the syntax of the participle should be Att. to — or Mod —.

As these omissions are liable to mislead the student, it is necessary to show that they should not exist, and I shall do so in Morell’s words, page 77 The predicate is extended 2 (d). by a participle or a partioipal phrase used adverbially.” Again (page 78), “ There can be no doubt that the noun, adjective, and participle in each of these instances takes the place of an adverbial adjunct, qualifying the force of the verb.” Page 64, " The participal form of the verb is sometimes used adverbially°

I think I. have quoted enough to show that the omissions I have referred to should not exist, and the sentence, “Retnote,” &c., may now be dealt with on the clear understanding that a participle or participal phrase may be used to modify the predicate.

Mr. Gough, according to Morell, makes the sentence read : “ Being remote from towns he ran his godly race.” Now, while I hold this construction to be opposed to the spirit of the poem, I wish first to show that, even if it be accepted, the syntax given by Morell. and adopted by Mr. Gough, is incorrect. What do the words mean ? For, after all, the answer to this question must decide the issue. Let Mr. Gough reply. After a spiritual ascent into the realms of poetic fancy, he sums up, “ Tie being free from distractions peculiar to life in towns, being on the road to perfection, which leads from Nature to Nature’s God, being attracted continually and irresistibly (he) ran his godly race."The clause “ Remote from towns conveys this and much nioref Mr. Gough, in these words, adopts the precise style of reasoning that must result from the construction he has given. But to what does this conclusion lead? Why, clearly to the fact that the clergyman’s being remote from town had something to do with his running a godly race. That he, being remote from town, or because he was remote from town, {he) ran, &c. If this be the case, and, I contend, Mr. Gough’s own reasoning shows it to be so, what have we ? Not certainly an extension showing place, but clearly one showing cause. How, then, can the phrase be called an attribute ? Possibly it may be said Mr. Gough’s words do not carry the extended meaning I have attached to them. Let me repeat a few of them “ Being attracted continually and irresistibly he ran,” &c. I think it will be admitted that no stronger language could be used to show that this man was compelled to act in a certain way, i.e., run a godly race, bv a power of attraction which acted continually, and which he was unable to resist, this power consisting of his being remote from towns.

I dare say many will be surprised to find the phrase referred to taken adverbially instead of as an attribute, but I think if we use our judgment without, reference to the examples to be found in our grammar, we shall find little difficulty in deciding to what part of the sentence the phrase belongs, and in reality the function of the author of a grammar is not to ascertain for us the force of any word in a sentence, but to give us rules for dealing with the word after we have determined its meaning. Perception is the eye that detects the force of a word; grammar is the finger that points out, by its lules, how to treat words when we clearly apprehend their force in the sentence. Take an example or two to illustrate the case refcircd to. A doctor says to the nurse, “ Wine being good for this disease, you may give some.” Here, according to Morell, the participal phrase, which is the nom. absolute, is an extension ; but if the voice of the verb be changed, and the doctor says wine being good foi this disease, may be given, it appears that by some grammatical sleight of hand, we are now to consider being good as an attribute to wine. Let us suppose the doctor to be about to leave, wc may imagine we hear the following conversation :—

Nurse,—“ Ah ! doctor, you have told me why wine may be given, but not what kind.

Doctor -“ Dear me ! yes, I forgot that. Give the best old port. GeneralIj 1 dislike to prescribe wine, and I allow it in this case only because as 1 have said it is good for the disease.”

Take the sentence, “The poor man could get no food.” This shows two things, (a) 1 he man was poor, (b) He could-get no food, but it does not show way, for 1 may say The poor man being lost in the desert could get no food. Here, Being, fc., gives the reason, so if Isay, The man being poor could get no food,” wc have the reason also in the participal

phrase. The student can construct any number of similar sentences. In Abbott’s “ How to parse,” (page 172), in the sentence “ Being very poor he could hardly maintain his family,” the phrase Being, ftc., is treated as an adverb.

The same reasoning will apply to the perfect participles which, in my opinion, are frequently treated as attributes, when they are used as extensions. For an example see “ University Syllabus of Parsing” (page 30), “Having often received an invitation, &c., I last week accompanied him, &c.,” where “Having received” is parsed as an attribute to I. Does not the sentence include a cause and an effect ? Cause—Having received. &e.; Effect—I last week accompanied him, kc., i.e., as I had often received, See., I accompanied, See. De Mornay in his grammar, a copy of which the author kindly sent me a few weeks since, gives examples of participles used as adverbs (page 98). But it may be asked, how are we to decide as to the syntaxof these difficult participles ? Let those who see a difficulty expand the word or phrase into an adjective sentence, and also into an adverbial sentence. Use each separately in the sentence, and then adopt whichever appears to correctly express the author’s meaning.

As to the spirit of the poem, I do not think the fact that the clergyman lived in the country had anything to do with his running a godly race. He would have been a godly man anywhere, and did not select a country life to keep him out of harm’s way. Neither was he an ascetic, for we find him always surrounded by his people, whose companionship he greatly enjoyed.

Taken as an extension of place, it appears to me to show that bis piety was so sincere, and his affection for the people among whom he had first been cast so warm, that be was enabled to remain willingly amongst his little flock, notwithstanding the advantages which a city life held out to him. Indeed, we are told as much in the next line :

“ Nor e’er had changed or wished to change his place.”

If we treat the words from a synthetical point of view, it will be seen that the portion of the extension which I have supplied is absolutely necessary to complete the verb run, while there is no necessity whatever for an attribute to “ he.” Whether, then, we consider the necessities of the construction, or the poetic spirit of the poem, I hold that Morell’s syntax is incorrect.

By a Country Teacher.

“ Semper ego auditor tantum ?”

WITHOUT in any way desiring to take a hand in the triangular duel in grammar, now being fought by Messrs. Gough, Courtney, and De Mornay, I am tempted to make a few remarks on the “ wranglers ” and their moot-points in the interest of the readers of the Australasian Schoolmaster.

Lord Macaulay finely remarks in one of his essays that it is pitiable to hear barristers noted for their forensic eloquence stultify themselves iu the House of Commons. Judging by their speeches in the Law Courts and in the Commons, one could scarcely believe them to be the same men. The cause of this, he goes on to say, is, in law they reason from data without questioning or explaining them ; in the House of Commons they have to find theinown data and reason therefrom.

Now if we apply the brilliant essayist’s remarks to grammar we are apt to understand what we are speaking about. And if we know what we are speaking about, we become less arrogant and less authoritative in a sense of the terra.

Both the prose and poetry of Goldsmith are simple and polished, for he was, a master of such compositions. He knew the art of writing too well to be turgid. “ His descriptions and sentiments have the true zest of nature. He is refined without false delicacy, and correct without insipidity.” We should read him with care and interpret him with modesty. To say that we “can appreciate Goldsmith’s sentiments .and analyse him correctly,” is a rashness bordering on impertinence begat of ignorance. When Macaulay asserts that Dr. Johnson was not a fit judge or critic of the higher flights of poetry because he had not a poetic genius, how many men living are fit to explain or comment upon the sentiments of our classic English poets? Goldsmith says of his model parson :—

“ A man he was to all the country dear,

And passing rich with forty pounds a year ;

Remote from towns he ran his godly race,

Nor ever changed, nor wished to change his place,”

What place? His place remote from towns—because of his modesty, piety, and disregard for worldly pomp. The context evidently shows that a place remote from towns is au extension of the verb ran, notwithstanding that Mr. Gough was inspirit with Goldsmith in worshipping Eternal Truths and Beauty (sublime sentiment) on the snow-covered plains of Canada, the canons of the Sierra Navada, and in Broadway, New York ; and notwithstanding that Mr. Gough is not a poet, and would prefer £1000 a year in London, to £40 and the village of Auburn, whatever the value of that township may be.

But to return to my argument. What I assert is, that the discussion of rules of grammar per se is worse than useless. As the English language was being developed and enriched, certain modes of expression became fixed by the usage of the best writers, on which the grammarians made rules. Therefore, in order to prove whether a grammatical rule is good or bad, it is necessary to show by quotations from classical English writers that custom has ratified that rule. To say that this is wrong because I assert that it is, or that that is right because I pronounce it so, is giving a lady’s reason in a pedantic manner. That Mr. Gough should desire to air his travels, that Mr. De Mornay should desire to advertise his grammar, or that Mr, Courtuey should modestly desire to exhibit his close reading of Morell, are matters beside the subject.

To speak to the other moot-points introduced by these gentlemen is not my present purpose. My object in writing is to assert that such controversies from a grammatical point of view are not worth the paper they have been written upon. Lessons on the noun and adjective suitable to a second class ; or a lesson on each part of speech suitable to a third or fourth class would be of gi'eat value to young teachers. And papers on the idioms of the English language, if well and carefully written, would be instructive and interesting to all concerned in the profession of teaching. Such subjects I would welcome with delight.

[This controversy must now close.—Ed, A.&]

NOTES OF A LESSON ON LEAD.

By a Pupil Teacher in a Country School.

Recapitulation.—Hold up a piece of lead before the class, and ask what it is.

(1)    Locality.—Found in England and Wales. In the rough state called lead ore. In the pure state called pig and sheet lead.

(2)    Description.—Lead is (a) bluish-gray colour ; (b) fusible ; (c) flexible ; (d) malleable ; (e) ductile.

(3)    Uses.—Used for making (a) bullets and shot; (b) gas and water pipes ; (e) roofing and house gutters ; (d) different kinds of vessels, baths, and inkwells.

Time.—Thirty minutes. _____________

Method.

Lead is found in England and Wales. (2.)

It is Taken out of deep holes in the ground called mines. (3)

In its rough state it is called “ lead ore.” (4)

In order to separate the pure lead from the dross the lead ore is put into a large fire (o), which causes the pure lead to melt and flow into troughs, leaving the dross behind.

The lead in the troughs is then called “ Pig Lead.” (6)

If the pure lead be poured upon a table and rolled very thin it is called “ Sheet Lead” (7)

Recapitulate.

Lead is:—

(a)    Bluish-gray colour— between blue and gray.

(b)    Fusible — can be brought to a liquid state.

(c)    Flexible—can be bent.

(d)    Malleable— can be beaten out.

(e)    Ductile—can be drawn out.

Recapitulate.

Lead is used for making :

(a)    Bullets and shots.

(b)    Gas and water pipes.

(cj Roofing and house

gutters.

(.d) Different kinds of vessels—baths and inkwells.

Recapitulate.

A Final Recapitulation.

School Boards have all manner of men as members. It is by no means a novelty, says The Schoolmaster, to find the would-be rulers at these educational councils very far from being the shining lights which they ought to be in the way of setting a good example to those around them. In various forms of moral delinquency the members of School Boards, both in England and Scotland, have at times been remarkable in a manner quite the reverse of satisfactory. The latest example of this undesirable state of affairs is reported from Yorkshire, where a School Board member w as captured by a gamekeeper while in the act of poaching ! He was setting snares for the unwary game of the neighbourhood and other contrivances were found upon him for the same purpose. It will surely be within the power of the Education Department to order this educational prowler in the woods to be relieved of all responsibility for the future management of the schools which are at present, in some degree, under his control. Were a teacher to enrol himself among the poaching fraternity, there would be little hesitation on the part of the authorities to cancel his certificate and thus dismiss him from the service. It is certainly quite as incumbent that the educational world should be free from the business propensities of men who are like the Yorkshire game-snarer.

NOTES OF A LESSON ON PUKE AIR.*


Heads,


Matter and Method.


Blackboard

Summary.


Introduc

tion.


Air, a Gaseous Body.


Let children take a book, etc., and pass it before their faces rapidly like a fan. They feel something. What is it ? The air. Tell them it is on every side of us —we cannot get away from it— surrounds the earth to a height of forty-five miles, etc.

1.    Turn on the gas. Ask children if there is anything escaping into the room. What is it ? Can they see it? Air is like this, an invisible gas, or rather, a mixture of two invisible gases, oxygen and nitrogen—21 parts of oxygen to 79 of nitrogen, with a small percentage of carbonic acid gas and some watery vapour.

2.    Presence of invisible oxygen gas might be illustrated by plunging a red-hot piece of charcoal into a jar of oxygen, and letting it burn. The presence of nitrogen might be similarly shown by its extinguishing a flame plunged into it. Breathing on glass, etc., will illustrate the presence of the watery vapour.

1.    Just as fish cannot live out of water, we cannot live without air. Let children make the experiment by closing their mouths and pressing their nostrils together between the thumb and finger.

2.    Elicit why we cannot remain long under water when diving.

3.    Tell children how a frog or other animal dies when placed under exhaustive receiver of an airpump.

1.    Allude to unhealthiness of people who work in “stuffy” workshops, and live in narrow alleys and overcrowded rooms.

2.    Air is pure when it contains the ingredients, oxygen, etc. (let children enumerate them) in their proper proportions. If too little or too much oxygen, or nitrogen, or carbon, or watery vapour, the air is impure and unhealthy.

1.    By mixture with mechanical impurities. Ask children if they have noticed the motes in a sunbeam that streams into a room through a narrow opening. Tell them that these particles are dust, minute portions of decayed substances, etc. Tell children how, in flax-mills of Yorkshire, operatives suffer from diseased lungs, and die young from inhaling the dust of flax fibre. How the knife-grinders of Sheffield are subject to lung diseases, and die at the average age of 29 from inhaling steel and stone dust. Draw attention to comparative short life of stone-masons from same causes.

Lastly, show that much of this disease might be avoided by wearing suitable respirators.

2.    By mixture with poisonous gases.

* (a) Elicit from children, that if we turned on the gas, and allowed it to escape into the room, the atmosphere would become impure and poisonous.

Tell them that burning gas gives off carbonic acid gas largely, and makes the air impure and unhealthy. Tell how fatal accidents sometimes occur in breweries from workmen going into vats filled with carbonic acid ; how dogs and other-animals become insensible from


Air =

(«) Oxygen, 21. (b) Nitrogen, 79. (e) Carbonic acid gas.

(á) Watery vapour.


Matter.


Heads.


(1)    Elicit the meaning of this word.

(2)    Show the places on the map where lead is found.

(3)    Refer to other metals which are taken out of mines.

(4)    Exhibit a piece of lead ore.

(5)    Elicit the consequence if the piece of lead ore were put into a large fire.

(G) Show a small lump of lead which has been cast in a mould to illustrate “ Pig Lead.”

(7) Roll a piece of dough thin.

(2)    Elicit the meaning of this word.

(b)    Show by melting a thin, narrow piece of lead in the flame of a candle.

(e) Show by bending a thin, narrow piece of lead.

(rZ) Show by beating out a piece of lead on a smoothing-iron.

(e) Illustrate by stretching a piece of toffee.

(3)    Ascertain the meaning of this word.

(u) Show a few bullets and shot.

(Z>) Show a piece of gas pipe, and also a piece of water pipe, and refer to their uses in large cities,

(c)    Refer to the uses of house gutters.


Air, necessary to OUR EXISTENCE.


Pure Air essential to Health.


How Air is

MADE IMPURE.


Air is necessary to our existence.


Pure-air is essential to health.


A ir is made impure 1. By mixture with mechanical impurities.


(1) Locality.


(2) Description.


(3) Uses.


2. By mixture with

poisonous gases.

(a) With coal gas.


* “Moffatt’s How to Prepare Notes of Lessons.” By T. J. Liveey.


Heads.


Matter and Method.


Blackboard

Summary.


Recapitula

tion.


same cause when thrown into the Grotto del Cane, near Naples.

(i) Same would occur if we burned carbon or charcoal and allowed the fumes to remain in the room.

Tell children how the French often commit suicide by suffocating themselves with carbon vapour.

(c)    The air is poisoned by decomposition of animal and vegetable matter. If air contains part of sulphuretted hydrogen (given off by decomposing animal remains) it will kill a dog.

Tell how workmen are sometimes killed by entering sewers suddenly, before allowing time for the sulphuretted hydrogen to become dissipated.

(d)    The air is made impure by human exhalations. Ask children if they have noticed the vapour that issues from the mouth and nostrils on a cold day. This exhalation, which cold makes visible, is always going on. Mixed with watery vapour is carbonic acid gas in a very large proportion. Impure gases also given out by perspiration, etc.

Tell of the suffocation of seventy cabin passangers on board the Londonderry, in 1848, from overcrowding in a storm ; and of the 120 English prisoners suffocated in the *• Black Hole,” of Calcutta, by the infamous Surajah Dowlah, in 1756.

Examine on the whole lesson.

Note.—This lesson should be followed by one on Ventilation.


EDUCATION COMMISSION.

Wednesday, 22nd, February, 1882.

Present—J. Warrington Rogers, Esq., in the chair ; F. McCoy, Esq., F.R.S., George Meares, Esq., J.P., Duncan Love, Esq., John M. Templeton, Esq., J.P., C. J. Ham, Esq., H. N. Loughnan, Esq., J.P., Hon. Francis Ormond, M.L.C., Hon. J. Macgregor, Edmund Keogh, Esq., J.P., W. H. Cutts, Esq., M.D., J.P.

G. Wilson Brown further examined.

You were to produce certain returns?—Yes.

The first was a return of the number of children who have obtained their certificates and continued to attend school?—I have not a return yet, but I have issued a circular. The information will have to be got from the schools. They will take a little time. I have the copy of the circular here.

The next was a return of cases of immorality or indecency charged against the teachers or scholars in the schools, giving all particulars, name of the school, names of the persons, and sex of the children, and steps taken by the department in the schools, and with what result in each case ?—That return is still in course of preparation. It has involved a great amount of labour, for it has been necessary to examine the register of something like 300,000 letters. The return is now in hand, and will be sent over in the course of the afternoon. I was instructed to give the names of nil concerned. I shall be very glad if the Commission will accept the return without the names being given, as in many cases the charges were not sustained, and it would be a very cruel thing if the names were published. The return will be over this afternoon in two forms, one giving the names, and the other without the names ; but even the latter will give the school, and the position of the person in the school, so that they may be traced even in that way.

The next was information re the substitution of the Royal Reader books for the Irish National series?—I have no written statement on that point. I can give the information verbally.

Will you do so ?—The first objection that appears to have been taken to the National Irish series was by the Anglo-Israel Association, who objected to the religious teaching in the books, and gave instances of the passages they objected to. One they quoted, I remember, was “ It was God who sent Christ to save me,” and similar passages ; that was in 1874. In 1875 they renewed their remonstrances. During the year 1875 the Council of Education in New South Wales made a suggestion that we should have prepared a set of reading books for use in the several Australian colonies. Upon that it was determined to send up the InspectorGeneral to Sydney, to confer with other officers from other colonies, with a view to drawing up a set of books suitable for Australia. It was arranged that this should be done, but before the meeting was held the Council of Education in New South Wales seemed to have been specially pressed,


(b) With vapour of burning charcoal.


(c) With sulphuretted hydrogen.


(¿) With human exhalations.


“Black Hole,” Calcutta, Surajah Dowlah, 1756.


and they closed with Collins and Sons for their book, even though the arrangement had been come to with us that we should have a meeting about it. That meeting was therefore stopped. A discussion took place in the House of Assembly in July 1876. Mr. Higinbotham specially called attention to the sectarian and otherwise unsuitable nature of the books in use in the schools, and Mr. Ramsay, the then Minister of Education, in reply, stated that the objection which had previously occupied the attention of his predecessors had been brought under his notice immediately he took office, and that inquiries were being made as to the best course to be adopted. He stated he was aware that there were numerous objections to the books, not merely on account of their sectarian character, but also for other reasons, one of which was that they contained a mass of information relating to Ireland, and very little relative to other countries, particularly Australia. At that time it had been arranged that we should have a conference with the other colonies. The conference, however, as I have said, fell through. The next action taken in the matter was Mr. Ramsay’s requesting the head masters of the Church of England Grammar school, the Scotch College, Wesley College, St. Patrick’s College, Presbyterian Ladies’ College, the Hawthorn Grammar-School, and the Kew High School, to meet and advise as to the best books for use in our schools. The meeting was held in July, 1876, and, as the result of that meeting, Nelson’s series was adopted, and the Minister’s order for the adoption was given in August, 1876, and arrangements were then made with Nelson.

Did those gentlemen attend ?—I have not got a return of those who attended. I know of two who did not atteud, but I cannot say that more did not attend, so it would be no use to give the names of the two.

That is the way the Nelson series were substituted for the Irish National series ?—Yes.

The books which you have supplied the Commission with, the Nelson series, are those the books now actually in use and read ?—Yes.

There is no portion of those eliminated in the teaching ?—No, those are the books now actually in use.

Do they go through these books, or does the teacher select some passages and omit others ?—No, there is no passage in those that the teacher is justified in omitting.

Were those books not edited after they were adopted by the Education Department?—Yes, those are the edited edition. Certain emendations were made in Nelson’s series as they stood then.

But those copies you have given are in the'state they are used in the schools now ?—Yes, those are the books actually in use. That is the approved edition.

The next thing was as to the modes of arriving at a uniformity in the standard applied by the inspectors and your special instructions given to the inspectors?—I have the two confidential instructions which are given to the inspector, one for examination for standard of education and the other for the examination for results.—[The same were handed under cover to the Chairman.]

The next point was as to the employment of teachers outside the department to teach extra subjects, &c. ?—Yes, I have a return of outside persons employed in that way. There are 9 teachers engaged in 15 schools. They teach 442 pupils, and the amount they received during 1881 was £372-4s. 7d. That is the whole number.

In connection with the same subject, the question was asked whether any complaint had ever been made about extra subjects interfering with private schools ?—I had an impression at the time that one such complaint had been made, but I have remembered the circumstances. We have had no official complaint, but a friend of my own, who has a grammar school, did make a private remonstrance to me on the subject, and that was what I was thinking of when I was speaking on the subject.

As a fact there has been no official complaint ?— No ; not any.

There was a question asked as to the machinery for ensuring the necessary instruction by the teachers paid by the pupils for extra subjects ?—No, I have nothing further on that subject. I may say there is one other check besides what I mentioned before—that the time table at the school shows the whole of the w7ork that is done in it, and the inspector sees whether the time-table is carried out, that is, so far as he can tell by the observation of one day.

Have you also a return of the largest number of schools in any one district?—Yes, it is 31 in school district No. 239, south riding of the shire of St. Arnaud, The approximate area of the district is 1,000 square miles, and no application has ever been made for its subdivision.

The nextpointwas the largest sum expended on any one building, and the lowest, and the price paid for sites?—The largest sum paid on any building was on the central school, Sandhurst, £10,575 Is. lid. The smallest sum on any building was at Murrindindi, £37 5s. The largest sum expended in the purchase of a site was at Emerald Hill £4,210. I should state, however, that we gave up a site at Emerald Hill to the Government, which they sold for £4,275, and it was in consequence of giving that up that we had to get a new one, so that actually, on the gross cost, we saved £35. As regards the smallest sum expended on any one site, we have given as little as a shilling ; but, under the advice of the Crown Solicitor, we always give a pound now.

That is a nominal consideration as a legal safeguard ?—Yes, that is done under the advice of the Crown Solicitor. The largest sum spent on any building and site together was on this same school at Emerald Hill, £11,533 14s. 7d.

Against which you received the £4,000 odd ?—Yes.

The next was a return of all the purposes other than school purposes for which the State school buildings are used ?—It was also asked on whose application. That cannot be got beyond what I stated at the time. The application is virtually made by the Board of Advice. They write

to us that the building is wanted for such and such a purpose, and the Minister gives his consent. I tried to get a complete return in the form in which the Commission asked for it, but I find I have only a very imperfect return of the number of schools used for other than State school purposes, during 1881, as it does not include those schools the use of which extending into 1881, was granted prior to 1881. It does not therefore show all the schools being used during 1881, as the arrangement for letting some of them was made in 1880, but I have a return here which I think the Commission want—a complete return of 1880. It is a thoroughly complete return, except in the one particular I have mentioned, that I have not the names of the applicants. It gives the Board of Advice in which the school is situated, the name of the school, and then the purposes for which it was granted under these headings :— Public worship on Sundays, public worship on week days, Sunday classes, religious instruction on week days, charitable purposes, temperance meetings, electoral purposes, educational purposes, and a column miscellaneous. That is the most complete return I can get. This other return [exhibiting a paper] might be taken as supplementary. This shows the new cases which came in during 1881. Possibly some of those may overlap the others, because some may have ceased to be used for a portion of 18S0, and then afresh application made. [The witness handed in two returns marked A. and B.] The last thing asked for was a map showing the school districts. That is here [pointing to a map hanging on the mall. ]

We were coming now to the subject of teachers ; but before going into that, I would draw your attention to a paragraph in the Argus of 22nd February, 1882, this morning:—“The residents of Frankston recently complained to the Minister of Education that the head teacher of the State school there had been guilty of conduct which rendered it undesirable that he should retain his position. Mr. Grant promised that an inquiry should be instituted, and Mr. Main, Assistant Inspector-General, investigated the charges. He subsequently reported that the teacher had administered corporal chastisement to female pupils, that he had addressed indiscreet expressions to the children generally, and that he had failed to efficiently instruct the scholars. Mr. Main advised that the teacher should be removed, and it has been decided accordingly to place him in charge of another school of smaller average attendance, where his emolument will be less than he received at Frankston.” Is that correct as a matter of fact, that the Inspector, after investigation, had advised that the teacher should be removed, having reported that the teacher had administered corporal chastisement to female pupils, and had addressed indiscreet expressions to the children generally, and failed to efficiently instruct the scholars, and upon that he had been placed in charge of another school ?—That is correct. He was reduced, and is to be placed in charge of another school.

Will he not be less open to surveillance in a smaller school than a larger one ? Is it not a dangerous thing to entrust those children in a small school to a teacher who has misbehaved to children in a larger school ; “Addressed indiscreet expressions to the children generally, and that he had failed to efficiently instruct, the scholars.” “ Indiscreet expressions,” would seem to imply something of an immoral tendency ?—No, it was not that.

Do you think it wise or politic to transfer to another school a man who has been found guilty of speaking indiscreetly to his female pupils ? —I forget the particular expressions used, but there was nothing of an immoral tendency in them.

If he could not “ efficiently instruct the scholars” in a large school, why would he be likely to do so in a smaller school ?—I think it was not so much that he could not teach, but that he would not—that he could have done better than he did, but that he was lazy.

What is the amount of salaries and allowances for teachers and their grades for 1881 ?—I can give it for the year ending the 30th June, 1881, the end of the financial year. They are made up always to the end of June. The amount paid for teachers’ salaries was £317,691 19s. 10d., and forresults, that is on the result of the examinations, £99,743 14s 8d. There are also sundry special payments for drill and gymnastics, £2,402 4s., and bonuses for passing pupil teachers, £2,659 7s. lOd. They also receive what we call a “ maintenance allowance,” that is for keeping the schools clean and providing water for the children, towels, and pens, ink, paper, and such purposes. The amount of that is £29,891 8s. lOd.

Does that include the stores and material ?—Pens, penholders, ink, slate pencils, chalk, sewing material, and all stationery, except copybooks.

How are those things obtained by the teachers—from the Government contractors ?—No.

Why not ?—They are obtained in such small quantities that it is considered cheaper for the teacher to get them from the nearest bookseller, lean give a complete list of the offices the teacher has to perform for that :—

(a) To keep the floors of all the school rooms, class rooms, porches, out offices, &c., properly swept and thoroughly scrubbed out with sufficient frequency to keep the rooms clean and healthy.

(h) To provide water for the use of the children for drinking and washing, also soap and towels.

(c)    To keep the chimneys and windows clean, and all glass in windows, and locks, latches, &c., to doors, in proper repair.

(d)    To keep properly emptied and attended to the pans in earth closets, and the cesspits where there are no earth closets.

(e)    To provide pens, penholders, ink, slate pencils, chalk, sewing

materials, and stationery (except copy-books).

(/) To provide fuel for warming the school rooms.

All articles must be provided, and the duties specified must be performed, to the satisfaction of any officer of the Education Department who may be authorised by the Minister of Public Instruction to visit and examiue the schools.

I have also here a schedule of rates paid. [The witness handed in the same.] They vary according to the attendance at a school. From £10 for a school of 20 scholars up to £120 for a school of 1000.

That is per year ?—Yes.

That exhausts the enquiry as to payments?—Yes.

What is the total number of teachers of all grades for the same period ? —All grades, including head teachers, assistants, work-mistresses, and pupil-teachers :—1,879 males, 2,408 females ; total, 4,2S7. That is on the 31st January, 18S2, right up to date.

This is as to the schools ?—Yes. 1,318 males and 395 females arc head-teachers ; 219 males, 658 females are assistants ; 570 females arc work-mistresses; and we have, in addition, 15 males employed as relieving teachers to take the place of others who are on sick leave, and so on.

I notice that amongst the head teachers, some are certificated teachers and others are licensed teachers only, whilst, at the same time, there are amongst the assistant teachers, certificated teachers ?—Yes.

It occurred to me that if the certificated teacher is the more highly qualified grade, your appointing the merely licensed teacher as a headteacher amounts to placing a man of inferior qualifications over the head of a superior man ?—But licensed teachers are not allowed to hold larger schools than those with an attendance from 50 to 75, and many of the assistantships are very much more valuable than the head-mastership of such a school would be. The assistantships carry more money with them, and are in very much more desirable localities.

Does not that produce this result, that, in a locality which is sparsely populated aud where a school would be smaller, the instruction would be inferior to that given in a school where the population is larger. Besides, while you give the more highly qualified person the head-mastership at the larger place, you put a person of inferior acquirements over the smaller school—as a fact, is that not so?—Yes, so far that in a small school you will have a teacher of inferior classification.

So that your teaching, in a school in a sparsely peopled and scattered place, must be inferior to that in the larger schools ?—No, not necessarily ; because the licensed man may know how to teach as well as if he had a certificate.

But he will be inferior to a certificated man ?—Yes.

And yet you put him in the head position at a small place, and keep the certificated teacher as assistant in the larger place?—As regards the children, the office of assistant in a large school is a very much more responsible position than that of head-teacher in a small one. The assistant has very large classes to attend to aud other people to supervise, and, at times, the first male assistant has to take the place of the headmaster.

That would occur very seldom—and the head-teacher is the responsible person ?—Yes.

Then the head in a small place is a person of inferior acquirements to an assistant teacher who is only second in command in a large school ? —Yes.

So that it must result in this, that in a sparsely peopled district, the education is inferior to that obtainable where the population is large ? —Yes, so far that they will not have so good a man, but they do not require so good a man, I say, aud the more difficult position is that of assistant.

Is it not the case that sometimes as good results are got out of small schools under a licensed man working for a certificate as under a certificated man ?—That is the case.

Is not classification a test of qualification ?—Service has something to do with it. I think a man with a license, who has held a school for some years, will probably be a better man than a raw certificated man.

Will you now give the maximum and the minimum salaries and allowances of the teachers of each class ?—The salaries will depend on the size of the school.

If you give the maximum and minimum of each class of teachers it will do?—I have got here the six highest salaries paid to male and female teachers, and the six lowest.

That will do for the present ?—The six highest paid to male teachers ranged from £477 18s. to £515 18s.; and to the six highest female teachers from £312 11s. to £320 13?. The six lowest paid to male teachers are £93 10s. up to £100 12s.; and to female teachers the lowest £73 up to £78 17s. Pupil teachers—the males for their four years get respectively £20, £30, £40 and £50 per annum, and the female pupil-teachers £16, £24, £32, and £40 respectively. Workmistresses are paid £30 a year.

Have you certificated teachers who are waiting for appointments which you are not able to get them ?—Yes.

Then you have a higher class of people out of employment than those who are in employment, of the inferior grade?—Yes.

And it is the fault of those who do not attain the higher grade that keeps them in the lower ?—I ought to mention here that the rights of those licensed teachers are conserved under the act; we cannot get rid of them,

You mean they are protected by the Act ?—Yes ; the 21st clause of the Act says—“Notwithstanding anything to the contrary contained in this Act, all teachers who have been classified under the Act CXLIX shall, on being employed under this act, be entitled without examination to a classification equal to that enjoyed by them under such Act and this classification is all that we require of them to hold appointments in the schools.

Have they been promoted to which were an advance on their previous position, and which would be equally a position attainable by certificated teachers ?—Yes ; licensed teachers do receive promotion within the limit ot a 75 school.

They have no right to that by the statute ?—They do not get it except on good reports from the inspectors, and good service,

You have certificated men who could take those positions ?—Other things being equal, the certificated man gets the appointment. We have licensed men who get very good reports, and on the other hand, certificated men of whom the reports are not so good.

We should like to have that ?—I will get it by nest meeting.

What is the whole number at present qualified who are out of employment ?—We have pupil-teachers eligible for promotion to assistantships or charge' of schools. There are 232 holding pupil teacherships of from £•10 to £50 per annum.

They are waiting promotion?—Yes. There are also 92 trained teachers waiting appointment.

Are they altogether out of employment ?—Yes; then there are other applicants, that is outsiders who have come up and passed our examination—170, making a total of 494.

Are there different salaries in the different classes ?—No, the salary depends on the size of the school and not on the teacher’s qualifications.

Then a man of inferior qualifications may get a higher salary than a man of superior qualification ?—Yes, some of our licensed teachers are paid more than some of our certificated teachers.

The literary qualification is what gives the steps from class to class ?— Yes, combined with the practical test. There is the practical test in each programme of examination—he has to teach a class and conduct a school and so on.

Is it always the case that a man excels in both—may not a man carry ofTmarks in his literary examination and fail in his teaching?—Yes, it frequently happens that a man attains the literary position, but is some time before he passes the teaching. He is then what is called “ pending” certificate.

Then there are many licensed teachers who are better instructors of the young than the certificated men?—Yes.

Then you would say that a certificate is not an absolute guarantee of proficiency in teaching ?—No, I do not say that.

What does the certificate profess to show?—The man has to pass a superior literary examination, and also to show ability to give a good lesson and conduct a school well. But it is possible that a man may give a good show lesson before the inspector, and in his ordinary work not really keep up to the same standard.

By passing the literary without the practical test a man would not get the certificate ?—No. The practical test is a part of the requirements for the certificate. The examination consists of the two parts, the literary and the practical.

The certificate shows a superiority in general literary attainments and the teaching of a class the practical ability?—Yes, he cannot get the certificate without both.

If he has passed both, I suppose he is superior in attainments, both practical and theoretical, to one who has not passed either—that is the licensed teacher ?—Yes. It does not follow that he will work so well in his school though.    >

In testing the ability to teach, you fix on a certain minimum of ability, and do not test them beyond that—is that so ?—Yes.

You have no means of distinguishing different degrees of ability in teaching ?—Exactly. There is a certain pass examination that a man has to come up to.

The licensed teacher’s test is that he can instruct up to the standard required by the Act ?—Yes, it is a guarantee that he can do so.

Kegarding teachers who held their positions under the Common Schools Act, I apprehend that they hold something like a vested interest, and cannot be reduced or deprived of their position except in case of misconduct ?—That is the case.

Do any of those teachers who have passed no examination hold schools of the first class?—All have passed an examination, or have got either a license or a certificate. Some of the licenses were issued upon length of service, coupled with teaching power.

What is the position now ?—Nobody can get any license or certificate except upon actual examination.

Is there any limitation to the number of appointments to be made in any one year ?—We simply fill up vacancies as they arise. The appointments will depend on the vacancies.

New schools and so on—is there any limitation to the number ?—No.

Have the Audit Commissioners complained at any time of appointments of teachers being greatly in excess of the number authorised by the regulations ?—Yes.

That was in December 7th, 1880 ?—It was about that date,

I think Mr. Grant, in his report on the 20th of September, 1881, speaks of an increase of 44 head teachers—that was after the Audit Commissioners’ report?—Yes, 44 head teachers.

Will you tell the Commission what action the department took on that report of the Audit Commissioners ?—We made out a list of the teachers who were in excess, and drafted them off to vacancies as they occurred.

What did you do with them in the meantime till vacancies arose?— Kept them where they were.

Then if the Commissioners were right you must have kept a greater number than the regulations authorised ?—Yes, there will always be a number in excess of the numbers authorised by the regulations. Perhaps I ought to explain a little on that point to show how it will be so. The number of teachers varies with the attendance at a school, and the attendance may go down and so leave us with a teacher in excess of the proper staff. The attendances do fluctuate very much owing to local fluctuations of population, and there are constant excesses caused in that way. You will see that generally at the fifties we put on an additional teacher, 150, 250, 350, and so on, and when the attendance at the school falls below the last 50 it leaves the staff with one in excess of what it Bhould be. And the same thing occurs in the case of pupil teachers. They are put on at the hundreds and the attendance may go down.

The Minister mentioned in December, 1881 that there were 443 persons qualified for teachers waiting for appointments ?—That will agree pretty well with what I have stated. It will be a similar return to that I have just given of those 494—made up in the same way.

Having so many in excess of those required—the supply so much exceeding the demand—are you still training persons for this occupation at the expense of the country ?—Yes, still training them.

And, of course, as you increase the number of trainees you will increase the number of those waiting for appointments?—We have not admitted so many this last term. We have not admitted so many this last year. We used to admit at the rate of 40 every half-year—we only admitted 25 in January, and the majority of those were pupil-teachers, so that we have brought very few new people into the field. The pupil-teachers were already eligible for promotion, but they will proceed to a higher degree of qualification with the training given them. So we have been bringing on very few new candidates for employment.

I believe you find, sometimes, in allotting au appointment to a person qualified, that, if it is not in a position that person likes, he declines to take it ?—That is frequently the case.

Do you strike such person off your list altogether ?—No, not altogether; but we note that they have had an offer and refused it, and do not give them another for a long time.

Will you tell us your present system of training and examination to qualify for the teaching staff, from the first step to the highest ?—A teacher may get his classification in first honours without any training at all. In that case he would come up for our examination, pass for a certificate, serve for two years in a school without a bad report, pass a further examinatiou at the University, and upon that he may be classified at once in first honours.

Then, in poiut of fact, you have two classes of persons who come up for your certificates ; one set trained in the department, and another set of extras trained elsewhere?—Yes.

What are the steps required for those trained in the department; how many training establishments have you ?—We have one central training institution and eighteen associated training schools. A candidate for training must be either a first-class pupil-teacher, have passed the matriculation examination, or have passed our own examination prescribed for entrance to training. Upon admission he is sent to one of the eighteen associated schools, practises teaching in the school all day, and receives at least seven hours’ instruction from the headmaster out of school hours, at the end of twelve months he is summoned to examination, and upon passing it enters the central institution. He remains there another twelve months, and is then again examined for a trained teacher’s certificate. At the end of the two years’ course he is ready for appointment to a school- -that is, after twelve months in the associated school and twelve in the central institution.

If I understand you rightly, there is no competition for the initiatory step of entering as a trainee?—No ; we have no competitive examination.

Then it appears that your present system creates a class of trained persons beyond the requirements of the State. You have 480 persons now qualified and waiting for appointments, and reckoning the possible vacancies at the English rate, at about 160, it will take three years before you can find employment for them all, and you are still creating qualified teachers at the rate of about a quarter of a hundred every year?—We are not now bringing in many new people. They are generally pupil-teachers.

But there are some fresh ones ?—A few.

I understood there were 25 ?—No ; some of those are pupil-teachers going on further in their course.

They are exempt from the first year’s course, are they not?—Yes, after they have served twelve months as pupil teachers.

How many pupil-teachers entered this year, can you say approximately ?—I would rather give the actual number next time.

At present you have a larger number of females qualified than males —are you taking any steps to equalise—I understood you were more in want of male teachers ?—1 am not able to say for certain whether of those 494 the majority are males or females ; yes, generally speaking, I know the majority are females.

If you adopted a system of competition, would not that tend to keep down the numbers and lessen the expense, and at the same time ensure a better class of persons to begin with ?—The best way to limit it would he to limit the number of studentships we give.

How are the students selected at present?—On the recommendation of the Inspector-General and myself, and our practice is to select first of all the first-class pupil-teachers ; we always give the preference to them, and then those that have passed the matriculation, choosing of those the apparently mo3t eligible.

Would it be a better plan or not to have it made competitive in the first instance ?—I would rather confine it to the first-class pupil-teachers ; we know they are pretty good, as they have generally had at least four years’ training.

How do they acquire the position of pupil-teacher—is it by competition ?—Virtually you may say it is by competition, because when there is a vacancy we ask the head teacher to recommend the best pupil-teachers. He tells us they have passed such and such examinations, and that is a thing we can check in the office.

Would it not be fairer to have a public competitive examination for all schools ?—I think the most desirable class we have are the first-class pupil-teachers. If we had more of those than we wanted it might be desirable to have a competitive examination, but not so long as we can, as at present, admit all the first-class pupil-teachers.

Are they not really turning out the best teachers ?—By far the best.

Even if not trained ?—Yes, they make the best teachers, even without the training.

You have a central training establishment and eighteen supplementary establishments ?—Yes.

Would it be an advantage if you had one training college in connection with the university, the entrance to which should be competitive, aud it should be made like studentships, enough being paid to maintain the student during the course ?—I should be very glad to see the training transferred to the university.

You think it would be an advantage ?—Yes.

And that the admission should be made competitive, so that the position of being supported in the college should be acquired only by competition ?—As I have said, I should prefer that preference should always be given to pupil-teachers, and if they ever became too numerous I should advocate competitive examination amongst them.

There could be only a limited number of studentships for the university ?—Yes.

Suppose the number were 20 and there were 40 pupil-teachers eligible, in that case you would approve of a competitive examination amongst them ?—Yes, certainly. I see nothing else for it.

Seeing that there is an excess of candidates over the appointments open, would it be a good mode of lessening the list if you were to strike out unsuccessful candidates at examinations for the successive steps. You are aware that that is done in some places. For instance, at the West Point Institution in the United States of America, where all the studentships are supported by the States, each State having a certain number of nominations, if a student fails in getting a step each six mouths he has to leave altogether. Now, supposing you have licensed teachers who do not take their next step, say in twelve months, ought they not to go altogether?—The licensed teachers are not required to be examined—they have sufficient qualifications to hold their present appointments and we cannot call upon them for anything further.

Are you creating any new licensed teachers ?—No, we have ceased to hold examinations for licensed teachers;

What are the steps now ?—We are not holding examinations for license, but we have still the four other classes.

Yes ?—I do not agree with that—we could not get the men at all.

As to future appointments—I mean it this way—if you had a training college in connexion with the university with studentships, maintained by the State, open to competitive examination, and if the students had to pass a succession of examinations in regular gradation till they got to the highest grade of the department, then if a man failed to pass his steps he should go out ?—I do not think it would do to propose so high a standard as that. We could not get men to go in for it.

How many first-class honour men have you at present on your list employed in the schools ?—I cannot say exactly.